SOLUTIONS MANUAL for Financial Accounting in an Economic Context 9e Jamie Pratt

Page 1


Part: 1 CHAPTER 1 FINANCIAL ACCOUNTING IN AN ECONOMIC CONTEXT ISSUES FOR DISCUSSION ID1–1 Security analysts and stockholders: These users would use financial statements to try to estimate the future earnings and cash flow potential of the company, which would be used to project a value for the company’s stock. Bank loan officers: These users would use the financial statements to determine the ability of a company to repay loans to the bank. A company’s customers and suppliers: These users would use financial statements to determine whether to extend credit to the company (suppliers) or whether to rely upon the company to be a supplier (customers). Both suppliers and customers would also use the financial statements to monitor the company’s profit margins. Public utilities: This group would use the financial statements to determine the company’s growth rate and how that might impact upon the company’s utility needs. Also, they would evaluate the company’s ability to pay its bills. Labor unions: These groups would use the financial statements to monitor the profitability of the company to help determine the amount of pay raises and benefits that it will negotiate for from the company. A company’s managers: The company’s managers will use the financial statements to assess the overall financial health of the company. This could impact the managers in a number of ways: raises, promotion opportunities, performance of other departments, etc.

ID1–2 The board of directors serves various functions for a company. One is to represent and protect the interests of the stockholders who are not on the board. Another is to provide oversight and input to management. The managers are involved in running the business on a day to day basis whereas the board is more focused on the bigger, long term picture. A weak board may not ask probing questions of management but instead may take everything at face value and believe anything that management says to them. A healthy management team would want a strong board that delivers valuable input. A management team that wants a weak board of directors may be trying to hide something (management fraud). Auditors are concerned with management fraud because, if there is a problem, in many cases the auditors will be sued by the stockholders on the basis that the auditors should have detected the fraud. 1


ID1–3 The function of the audit committee is to provide a channel whereby the auditors report their findings and concerns, if any, to the board of directors. Typically there are outside members of the board that are on the audit committee so that if the auditors have concerns about management’s financial statements or activities, then the auditors have a way to speak directly to the board of directors. The auditors are in a sensitive position because the financial statements and activities that they are auditing are prepared by the same people who hire and pay the auditors. Therefore, they may be reluctant to jeopardize their relationship with the company by being too negative.

ID1–4 Banks make loans to customers and depend on those customers to repay the loans (called the “principal”) plus interest for the banks to earn a profit. If customers are not able to pay the interest, the banks cannot make a profit; further, if the customers are not able to repay the principal the banks will show a loss that reduces the equity on the balance sheet. Banks look at a number of factors, both “macro” and “micro” in nature. Banks will look at the overall strength of the economy and the likelihood for future growth; these are the macro issues a bank considers. Banks will also examine the specifics of a company’s individual performance within the economy; these micro issues often are seen in the financial statements of companies. Issues such as the amount of debt, the level of profits, the amount of cash on hand and the amount of cash generated by the business, and the quality and size of the assets can all be seen from the financial reporting system. Banks require borrowers to submit financial statements to show these performance measures. During the 2008-2009 recession and related credit crunch, banks were concerned about the macro issues shown in general economic data, as well as the micro issues shown in companies’ individual financial reports. The reluctance of banks to lend has been cited as one of the reasons for the length of the economic downturn.

ID1–5 Sales for Home Depot increased during the time period because of the slow rebound of the economy after the worst of the “Great Recession”. As conditions improved, home owners and to a lesser extent home builders purchased more materials from Home Depot to improve and construct houses. Profits increased because the bump in sales was not offset with an equal increase in expenses, probably due to some economies of scale experienced by the large hardware retailer. Assets increased as the company boosted inventory in stores to meet the higher sales, as well as the company increasing its total number of stores. Equity remained flat because the company rewarded its owners by paying out dividends equal to its profits. Finally, the cash balance slightly increased due to the fact that the improved operations (from the stronger economy) generated more cash than what was needed to invest in new stores and pay out to the providers of capital.

ID1–6 Creditors would impose these types of restrictions on United Continental so that the creditors would be protected for their loans. These types of restrictions are fairly common and act as a trip wire to warn the creditors that business may not be going well. The cash restriction would force United Continental to have enough cash to pay the interest on the debt and the minimum cash flow coverage assures that the airline’s operations are functioning well enough to generate sufficient cash flow to meet obligations.. These restrictions act as trip wires in that as soon as a restriction is violated the creditors can call the debt and force the company to pay back the loans. What is more typical is for the loans to be 2


restructured. This usually means higher interest rates and fees to do the restructuring. These all put the creditors in a better position to protect their loans.

ID1–7 Companies would usually engage in this type of behavior to try to improve their stock price. By showing higher revenues or lower expenses investors are more likely to reward the company with a higher stock price. Companies that have negative cash flow are under a lot of pressure to maintain a high stock price since selling stock is the only way to fund the business. This type of incentive can lead to questionable behavior. The ethical implications are significant because if investors lose faith in the financial statements that are reported this would severely impact the stock market. A strong driver to a robust economy is access to capital (stock markets). If this source is reduced because investors don’t believe the numbers that are reported, a very bad impact on the overall economy would result.

ID1–8

This is the normal statement that an auditor would make about a company whose books it had audited and found no significant problems. This would be part of what is called a “non-qualified opinion”. If there was a particular item that the auditors did not agree with they would issue a “qualified opinion” – they would agree with everything except the qualified item that would be identified. “In our opinion”, shows that the statement represents the auditor’s opinion and not a fact; “fairly, in all material respects” means that the auditors can not say that every single number is exactly accurate to the penny but that the numbers are generally accurate. This reflects the concept of materiality; the auditors believe that all material items have been presented accurately. Finally, “in conformity with accounting principles generally accepted in the United States of America” means that the financial statements have been compiled in a way that meets all of the accounting principles that are called GAAP in the U.S but not necessarily in conformance with international standards. ID1–9 Corporate governance describes the relationship among the stakeholders of a company, mainly : the shareholders, the Board of Directors, management and the company’s auditors. Corporate governance mechanisms encourage management and the Board of Directors to act in the best interest of the shareholders and to provide the shareholders with accurate and timely financial information. The Sarbanes-Oxley Act was passed to upgrade the financial transparency of corporate operations, requiring increased financial disclosures and management responsibilities for the intergrity of the financial statements. Improved information provided to shareholders and other providers of capital 3


will strenghten the confidence in the financial system, ultimately benefitting both providers and users of capital.

ID1–10 Management is charged with the responsibility to benefit the shareholders’ investment in the company. Choosing investments that will boost the short-term results of the company in lieu of long-term gains does not meet this requirement. While satisfying the needs of Wall Street analysts for short term results, a management decision to forego larger long term returns violates the relationship between the owners of the company and the management of the company. Many observers feel that short term profit pressures from analysts have caused management to ignore its responsibility to work for the long term benefit of the shareholder. ID1–11 Financial analysts are charged with the task of following companies in specified industries and evaluating the past financial performance of those companies, as well as providing guidance for expectations for future financial performance. Until financial reporting is consistent across global lines, analysts must be in a position to understand, interpret, analyze and forecast financial performance using different financial systems. An analyst following the pharmaceutical industry needs to understand how companies compare against each other, how Novartis stacks up against Johnson & Johnson. Now, not only does an analyst have to understand two sets of financial reporting systems (IFRS and GAAP), but that analyst then has to perform some type of conversion, so the companies can be compared under the same (“apples to apples”) basis. Fluency in GAAP is not sufficient; an analyst must also speak the language of IFRS and be able to translate back and forth between the two systems. ID1–12 Accounting guidelines that are established based on a number of general principles have the advantage of being simplified and easier to understand and document. On the other hand, guidelines that are principle-based are more ripe to be exploited by companies desiring to present their financial statements in good light. Guidelines that are based on a detailed set of rules are, by definition, lengthy and complicated, trying to anticipate every possible business situation. However, some argue that the more detailed rules-based approach allows the users of financial statements to review companies’ financial performance from a consistent perspective.

ID1–13 Managerial accounting is the accounting system that generates information that is used exclusively by the managers of the company. Financial accounting refers to the financial statements that are prepared and distributed outside of the company. So in many cases management accounting information is the operational information used by the managers of the company. This can be very proprietary to the company and so is not made public. Management accounting numbers are not subject to audit and therefore are prepared in whatever form is helpful to the manager. 4


Financial accounting information is audited and therefore has to follow GAAP. Its primary purpose is to be used by people outside of the company.

ID1-14 a. Google is primarily a service business, providing internet search functions and selling targeted online advertising. However, due to a recent acquisition of part of Motorola’s cell phone business, the company also is a manufacturing company. b. The firm of Ernst & Young audits the financial statements of Google. The audit report states what years and financial statements were audited and therefore being commented upon by the auditor. The second paragraph explains what an audit is intended to do and how the company has gone about doing this audit. The company’s internal control procedures are discussed. Finally, the report states the auditors’ opinion regarding the financial statements that have been audited. The auditors do not evaluate the financial strength of the company; the auditor states that the financial statements “present fairly” the position of Google; it is up to the user of the financial statements to analyze the company’s performance. c.

Net income in 2010 was $8,505,000,000, for 2011 net income was $9,737,000,000 and for 2012 net income was $10,737,000,000.

d. The amounts shown below are in millions:

Total liabilities Total assets TL/TA (%)

2012 $22,083 $93,798 23.54%

2011 $14,429 $72,574 19.88%

.

Total liabilities include both Current and Long Term liabilities. From 20111 to 2012 Google increased the percentage of its assets that were financed by liabilities. This fact, of course, means that the company decreased the percentage of its assets that were financed by equity. e. Cash from operating activities was $11,081,000,000 in 2010, in 2011 it was $14,565,000,000 and in 2012 it was $16,619,000,000. f.

Google increased its profitability (in both raw dollars) but as a percentage of revenue the company was slightly less profitable, 2012 versus 2010. The addition of the Motorola business might be an explanation behind the margin shift. As noted above, the company used slightly more leverage than previously, but debt levels are still quite low in comparison to other companies. Finally, the cash generated by the company’s core operations grew significantly from 2010 to 2012. The company is quite strong and well-positioned for the future.

5


CHAPTER 2 THE FINANCIAL STATEMENTS BRIEF EXERCISES BE2–1 2012 2012 Beginning Retained Earnings

+

2012 Revenues

$40.3

+

$65.5 X

2012 Expenses

$59.3

=

$3.3

2012 Dividends

=

2012 Ending Retained Earnings

X

=

$43.2

2012 Dividends as a percentage of 2012 net income: 2012 Dividends 2012 Net income ($65.5-$59.3)

=

$ 3.3 $ 6.2

= 53.2%

BE2–2 (1)

Current Liabilities financed $45 billion of the assets. Current Liabilities divided by Total assets = $45/$89 = 50.6%

(2)

Long-term debt financed $38 billion of the assets. Long-term debt divided by total assets = $38/$89 = 42.7%

(3)

Stockholders’ equity financed $6 billion of the assets. Stockholders’ equity divided by total assets = $6/$89 = 6.7%

BE2–3 (a)

Working capital = current assets – current liabilities. Boeing’s current assets total $57 billion, less $45 billion of current liabilities, gives the company working capital of $12 billion. Another measure of solvency would be the current ratio. The current ratio is current assets divided by current liabilities or $57 billion divided by $45 billion = 1.27. Both measures indicate that Boeing appears to have reasonable solvency. Current assets are sufficient to cover current liabilities.

(b)

No, Boeing has $19.1 billion of liquid current assets (cash, short term investments, and accounts receivable) but it has $45 billion of current liabilities.

(c)

Boeing would be more solvent if accounts receivable were $37.8 billion and inventory was $5.6 billion. Accounts receivable are closer to cash than inventory. This means that accounts receivable are expected to be converted to cash in a shorter period of time than inventory. 1


BE2-4 2012

2011

2010

Net cash flow from operating activities ......................... Net cash flow from investing activities .......................... Net cash flow from financing activities ..........................

$ 36,100 $ 34,743 $ 35,222 (19,680) (21,250) (21,449) (17,673) (11,649) (15,849)

Net change in cash .........................................................

$ ( 1,253)

Cash at beginning of period ........................................... Cash at end of period………………………………. ...................

3,045 $ 1,792

$ $

1,844

$ (2,076)

1,201 3,277 3,045 $ 1,201

AT & T’s cash management activities over the three-year period of 2010- 2012 appear to be extremely strong. The company is generating significant amounts of cash flow from operating activities, with all three years in excess of $34.5 billion. AT & T is then able to reinvest substantial amounts in its asset base. At the same time AT & T is also able to fund its financing activities from its operating cash flow. The large amount of funds being used in investing activities indicates that AT & T is growing its capital-intensive business.

BE2–5 IFRS Format Non-current assets Current assets Less: Current liabilities Total

154,073 115,397 (94,384) 175,086

Non-current liabilities Equity Total

49,118 125,968 175,086

GAAP Format Non-current assets Current assets Total

154,073 115,397 269,470

Current liabilities Non-current liabilities Equity Total

94,384 49,118 125,968 269,470

Many non-US companies begin with non-current assets, add current assets, and then subtract current liabilities to reflect the resources available to generate revenues and profits. The IFRS balance sheet then lists non-current liabilities and shareholders’ equity, which represent the financing sources of company resources; this amount is often labeled “capital employed.” GAAP balance sheets, on the other hand, list all assets owned (current and long-term) and then categorizes the financing sources (current and long-term liabilities, as well as shareholder equity) for those assets.


EXERCISES E2–1

1 2 3 4 5 6 7 8

Operating, Investing, or Financing Financing Operating Operating Investing Financing Financing Investing Operating

Balance Sheet

Income Statement

Statement of Cash Flows

Yes Yes Yes Yes Yes Yes Yes Yes

No Yes Yes No No No No No

Yes Cannot tell Yes Cannot tell Yes Yes Yes Yes

Operating, Investing, or Financing Financing Operating Operating Operating Investing Investing Financing Operating

Balance Sheet

Income Statement

Statement of Cash Flows

Yes Yes Yes Yes Yes Yes Yes Yes

No No Yes Yes No Cannot tell No No

Yes No Yes No Yes Yes Yes Yes

Statement of Stockholders Equity Yes Yes Yes No No Yes No No

E2–2

1 2 3 4 5 6 7 8

Statement of Shareholders Equity No No Yes Yes No Cannot tell No No

E2–3 a. b. c. d. e. f.

E2–4

Balance sheet Income statement Balance sheet Income statement Balance sheet Income statement

g. h. i. j. k. l.

Balance sheet Balance sheet Balance sheet Balance sheet Income statement Income statement

m. n. o. p. q. r.

Balance sheet Balance sheet Balance sheet Income statement Balance sheet Balance sheet


1. Statement of Stockholders’ Equity (Retained Earnings); Statement of Cash Flow, Income Statement 2. Income Statement 3. Balance Sheet 4. Statement of Cash Flow, Balance Sheet 5. Statement of Stockholders’ Equity; Statement of Cash Flow 6. Income Statement, Balance Sheet 7. Income Statement 8. Balance Sheet, Income Statement

E2–5 2010 2010 Beginning Retained Earnings $1.5

+ +

2010 Revenues $14.3 X

– –

2010 Expenses $13.0

=

$1.3

– –

2010 Dividends X

= =

2010 Ending Retained Earnings $1.5

2011* 2011 Beginning Retained Earnings

+

2011 Revenues

2011 Expenses

2011 Dividends

=

2011 Ending Retained Earnings

$1.5

+

$14.5

X

$1.3

=

$1.9

X

=

$12.8

= =

2012 Ending Retained Earnings $1.9

2012 2012 Beginning Retained Earnings X

+ +

2012 Revenues $19.6 X

– – =

2012 Expenses 17.8 $1.9

– –

2012 Dividends $1.8

*you must calculate the 2012 equation before you can calculate the 2011 equation 2012 Sales growth ($) $5.1 Sales growth (%) 35.2% Profits ($) $1.8 Profits (% of sales) 9.2% Dividends (% of net income) 100%

2011 2010 $0.2 N/A 1.4% N/A $1.7 $1.3 11.7% 9.1% 76.5% 100%

The company saw significant sales growth, but profits grew relatively less. Dividends are a consistently high percentage of profits, which is common in the utility industry.


E2–6 2010 2010 Ending Retained Earnings or 2011Beginning Retained Earnings

($523) X

= =

($499) $1,407

=

2010 Beginning Retained Earnings + Revenues for 2010 – Expenses for 2010 – Dividends for 2010

+

$1,383

X

$0

+

$1,522

$1,608

X

+

X

$1,550

$5

Expenses for 2010 are $1,407. 2011 ($758) X

=

($523)

=

$149

Dividends declared for 2011 are $149.

2012 ($596) X

= =

($758) $1,717

Revenue for 2012 is $1,717.

Sales growth (%) .......................................................... Profits ............................................................................ Profits as a percentage of sales...................................... Dividends........................................................................ Dividends as a percentage of net income ......................

$

2010

2011

2012

N/A ($24) (1.7%)

10.0% ($86) (5.7%)

12.8% $ 167 9.7%

0 N/A

$

149 N/A

$

5 3.0%

The advertising agency had modest sales growth from 2010 to 2012. However, from 2011 to 2012, the Company was able to go from losses to a profit. Even though the Company had a loss in 2011 the Company paid a healthy dividend. Then in 2012, when the Company showed a profit, it virtually eliminated the dividend. There is reason to be optimistic going forward. In 2012 the Company was able to show a nice growth in its sales while at the same time showing a reduction in its expenses.


E2–7 Solvency primarily indicates a company’s ability to meet its debt payments as they come due. Current liabilities are obligations that will be settled within one year or the company’s operating cycle, whichever is longer, through the use of current assets or the creation of new current liabilities. Current assets are those assets that will be consumed or converted to cash within one year or the company’s operating cycle, whichever is longer. Consequently, comparing current assets to current liabilities provides an indication of a company’s ability to meet its short-term debts. In this case, current assets were 3.33 ($500/$150) and 3.27 ($432/$132) times greater than current liabilities as of December 31, 2012 and December 31, 2011, respectively. Although comparing current assets to current liabilities provides a measure of a company’s solvency, this measure is not perfect. A true test of a company’s short-term solvency would be to compare the cash value of its current assets to the cash value of its current liabilities. For current liabilities, the book value is usually a good approximation of the cash value, since a company cannot, from a legal viewpoint, unilaterally change its debts. The situation is different for current assets though. The book value may or may not bear any relation to the cash value. Consequently, comparing the book value of current assets to current liabilities may not give an accurate measure of a company’s solvency.

E2–8 Method 1 Working capital as of 12/31/2012 ($500 – $150) .................................................................

$

Method 2

350

$

350

Impact of method on current assets ..............................

0

0

Impact of method on current liabilities ......................... New working capital as of January 2013 ........................

(200) $ 150

0 $

350

It seems that only the second method would be acceptable to the company in terms of maintaining compliance with the minimum working capital covenant.

E2–9 2012

2011

2010

Beginning cash balance .................................................. Net cash flow from operating activities ......................... Net cash flow from investing activities .......................... Net cash flow from financing activities .......................... Ending cash balance .......................................................

$

7,662 11,491 (3,815) X $ 9,799

$

Y* X (2,934) (4,064) $ 7,662

$

X equals

(5,539) $

10,079

$ (11,931)

................................................................. $

5,718 10,173

X 621 $ 4,581

*Beginning cash balance for 2011 = Ending cash balance for 2010. Cisco Systems’ cash management activities over the three-year period of 2010, 2011, and 2012 appear to be strong. The Company is generating a significant amount of net cash flow from operations each year and then is investing in its business. Financing activities (including dividends and/or share repurchases) reduced cash in 2012 and 2011, but were positive in 2010.


E2–10 2012 Beginning cash balance .................................................. Net cash flow from operating activities ......................... Net cash flow from investing activities .......................... Net cash flow from financing activities .......................... Ending cash balance .......................................................

$

X equals

(833)

................................................................. $

$

829** $ 2,064 X (947) 1,113

2011

2010

1,261 $ 1,356 (1,022) X $ 829

X

$

1,114

(766)$

1,561 (1,265) (149) $ 1,261*

*2011 Beginning balance = 2010 Ending balance **2012 Beginning balance = 2011 Ending balance. Southwest Airlines’ cash management activities appear to be very good for the years 2010 and 2011, but even better in 2012 (due to the economic recovery). The company generated a net cash inflow from its operating activities for the years shown. A look at its investing activities reveals that the company is expanding its asset base, as necessary in such a capital intensive industry. During 2011 and 2012, the company increased its cash outflows due to financing activities. Overall, Southwest Airlines is a strong company emerging from a difficult time in its cyclical business.

E2–11 Lana & Sons Statement of Cash Flows For the Year Ended Cash flows from operating activities: Cash collection from services provided.................................................. Cash payment for expenses ................................................................... Net cash increase (decrease) from operating activities .................. Cash flows from investing activities: Purchase of machinery ........................................................................... Net cash increase (decrease) from investing activities ................... Cash flows from financing activities: Proceeds from stockholders’ contributions ........................................... Payment of dividends ............................................................................. Net cash increase (decrease) from financing activities ................... Increase (decrease) in cash balance.............................................................. Beginning cash balance ................................................................................. Ending cash balance ......................................................................................

$4,000 (3,000) $1,000 $(3,000) (3,000) $7,000 (1,500) 5,500 3,500 13,000 $ 16,500 $

Based on just one year’s statement of cash flows it is difficult to comment adequately on Lana & Son’s cash management activities. However, one can observe that most of the cash during the year was generated as a result of issuing equity. The company seems to be investing in its asset base. That will certainly help it grow in the future. Cash flows from operations is positive, which certainly is a good sign.


E2–12 Emory Inc. Statement of Cash Flows For the Year Ended Cash flows from operating activities: Cash collection from services provided.................................................. Cash payment for expenses ................................................................... Net cash increase (decrease) from operating activities .................. Cash flows from investing activities: Purchase of equipment .......................................................................... Net cash increase (decrease) from investing activities ................... Cash flows from financing activities: Proceeds from the bank loan ................................................................. Payment of dividends Net cash increase (decrease) from financing activities ................... Increase (decrease) in cash balance.............................................................. Beginning cash balance ................................................................................. Ending cash balance ......................................................................................

$40,000 (23,000) $17,000 $(23,000) (23,000) $30,000 (24,000) 6,000 $ 0 25,000 $ 25,000

Based on just one year’s statement of cash flows, it is difficult to comment adequately on Emory’s cash management activities. However, it seems that the company is generating a substantial portion of its cash flows from operating activities. The company is taking some loans to finance its asset base which would be helpful in the future. Return on total assets and return on equity would probably increase.

E2–13 George’s Business Income Statement For the Year Ended Lease revenue................................................................................................... Expenses ........................................................................................................... Net income .......................................................................................................

George’s Business Statement of Stockholders’ Equity For the Year Ended

Beginning Balance Stock Issue Net Income Cash Dividends Ending Balance

Contributed Capital $ 0 6,000 _____ $6,000

Retained Earnings $ 0 500 (800) $ (300)

$3,000 2,500 $ 500


E2–13

Concluded George’s Business Balance Sheet As of Assets Cash ................................................................................................................ Land ................................................................................................................ Total assets ....................................................................................................... Liabilities & Stockholders’ Equity Note payable .................................................................................................... Contributed capital ........................................................................................... Retained earnings ............................................................................................. Total liabilities & stockholders’ equity .............................................................

$

2,700 8,000 $ 10,700 $

5,000 6,000 (300) $ 10,700

George’s Business Statement of Cash Flows For the Year Ended Cash flows from operating activities: Cash collections from customers .................................................. Cash payments for expenses ......................................................... Net cash flow from operating activities ................................... Cash flows from investing activities: Purchase of land ............................................................................ Net cash flow from investing activities .................................... Cash flows from financing activities: Proceeds from equity investor ...................................................... Proceeds from borrowing ............................................................. Cash payments for dividends ........................................................ Net cash flow from financing activities .................................... Increase in cash .................................................................................. Beginning cash balance ...................................................................... Ending cash balance ...........................................................................

$

3,000 (2,500) $

500

$ (8,000) (8,000) $

6,000 5,000 (800) 10,200 2,700 0 $ 2,700 $

Upon examining George’s financial statements the bank would certainly be concerned because George paid out more in dividends than the net income he realized during the year. George’s statement of retained earnings shows a negative balance, which means that the payment to equity investors which was disguised as return on capital was in fact a return of capital. Generally, dividend payments cannot exceed the Retained Earnings balance.


E2–14 Mary’s Business Income Statement For the Year Ended Lease revenue................................................................................................... Expenses ........................................................................................................... Net income .......................................................................................................

$ 12,000 14,000 $ (2,000)

Mary’s Business Statement of Stockholders’ Equity For the Year Ended

Beginning Balance Stock Issue Net Income (Loss) Cash Dividends Ending Balance

Contributed Capital $ 0 30,000 ______ $30,000

Retained Earnings $ 0 (2,000) (1,000) $ (3,000)

Mary’s Business Balance Sheet As of Assets Cash ................................................................................................................ Land ................................................................................................................ Total assets ....................................................................................................... Liabilities & Stockholders’ Equity Note payable .................................................................................................... Contributed capital ........................................................................................... Retained earnings ............................................................................................. Total liabilities & stockholders’ equity .............................................................

$

2,000 40,000 $ 42,000 $ 15,000 30,000 (3,000) $ 42,000


E2–14

Concluded Mary’s Business Statement of Cash Flows For the Year Ended Cash flows from operating activities: Cash collections from customers .................................................. Cash payments for expenses ......................................................... Net cash flow from operating activities ................................... Cash flows from investing activities: Purchase of land ............................................................................ Net cash flow from investing activities .................................... Cash flows from financing activities: Proceeds from equity investor ...................................................... Proceeds from borrowing ............................................................. Cash payments for dividends ........................................................ Net cash flow from financing activities .................................... Increase in cash .................................................................................. Beginning cash balance ...................................................................... Ending cash balance ...........................................................................

$ 12,000 (14,000) $ (2,000) $ (40,000) (40,000) $ 30,000 15,000 (1,000) 44,000 $ 2,000 0 $ 2,000

Mary should not have paid a cash dividend of $1,000 because of her dwindling cash position and negative earnings during the year. The dividend was a return of capital rather than a return on capital.


PROBLEMS P2–1 1. 2. 3. 4. 5. 6. 7. 8.

e e a a g c f c

9. 10. 11. 12. 13. 14. 15. 16.

a a c d c b e a

17. 18. 19. 20. 21. 22. 23.

c a d b e e e

X Company Balance Sheet (Date) Assets Current assets: Cash ................................................................................... Short-term investments .................................................... Accounts receivable .......................................................... Less: Allowance for uncollectible accounts ....................... Inventory ........................................................................... Prepaid rent ....................................................................... Total current assets ...................................................... Long-term investments: Land held for investment .................................................. Investment fund for plant expansion ................................ Total long-term investments ........................................ Property, plant, & equipment: Property............................................................................. Building .............................................................................. Less: Accumulated depreciation (building) ....................... Net book value of building ................................................ Machinery and equipment ................................................ Less: Accumulated depreciation (machinery & equipment) ................................................................... Net book value of machinery and equipment................... Total property, plant, & equipment ............................. Intangible assets: Patents............................................................................... Trademarks........................................................................ Total intangible assets .................................................. Total assets .............................................................................

$XX XX $XX XX

XX XX XX $XX $XX XX XX $XX

$XX XX XX $XX XX XX XX $XX XX XX $XX


P2–1

Concluded Liabilities and Stockholders' Equity Current liabilities: Accounts payable .............................................................. Wages payable .................................................................. Dividend payable ............................................................... Short-term notes payable ................................................. Current portion due of long-term debt ............................ Payments received in advance .......................................... Total current liabilities ................................................. Long-term liabilities: Bonds payable ................................................................... Total long-term liabilities ............................................. Stockholders' equity: Capital stock ...................................................................... Retained earnings.............................................................. Total stockholders' equity ............................................ Total liabilities and stockholders' equity ................................

P2–2 1. 2. 3. 4. 5.

e b e a e

6. 7. 8. 9. 10.

e e f c c

11. 12. 13. 14. 15.

e f f d c

$XX XX XX XX XX XX $XX $XX XX $XX XX XX $XX


P2–2 Concluded X Company Income Statement For the Period Ended Revenues: Sales................................................................................... Fees earned ....................................................................... Interest income ................................................................. Dividend income................................................................ Gain on sale of short-term investments............................ Total revenues .............................................................. Expenses: Cost of goods sold ............................................................. Operating expenses: Office salary expense ................................................... Insurance expense........................................................ Salesmen commission expense .................................... Depreciation expense................................................... Office supplies expense................................................ Advertising expense ..................................................... Total operating expenses ........................................ Other expenses: Interest expense ........................................................... Loss on sale of equipment............................................ Loss on sale of building ................................................ Total other expenses ............................................... Total expenses ................................................................... Net income .............................................................................

$XX XX XX XX XX $XX $XX $XX XX XX XX XX XX XX $XX XX XX XX XX $XX


P2–3 Nimmo Brothers Corporation Statement of Cash Flows Balance Sheet for the year ending 12/31/2014 as of 12/31/2014 Cash-Operating 275 Cash Cash-Investing (200) Other Current Assets Cash-Financing 330 Long-term Assets ∆ in Cash 405 Total Assets Cash-12/31/13 420 Cash-12/31/14 825

825 1,550 1,600 3,975

Income Statement Current Liabilities 995 for the year ending 12/31/2014 Long-term Liabilities 1,200 Revenue 4,200 Contributed Capital 1,200 Expenses 4,050 Retained Earnings 580 Net Income 150 Total 3,975

Statement of Stockholders’ Equity for the year ending 12/31/2014 Contributed Retained Capital Earnings 12/31/13 1,000 500 Net Income 150 Dividends (70) Stock Issuance 200 ___ 12/31/14 1,200 580


P2–4 Johnson Company Balance Sheet December 31, 2014 Assets Current assets: Cash ................................................................................................ Short-term investments .................................................................. Accounts receivable ........................................................................ Less: Allowance for uncollectible accounts .................................... Net accounts receivable ........................................................... Inventory ......................................................................................... Total current assets .................................................................. Property, plant, & equipment: Buildings .......................................................................................... Less: Accumulated depreciation ..................................................... Total property, plant, & equipment ......................................... Total assets ........................................................................................... Liabilities & Stockholders' Equity Current liabilities: Accounts payable ............................................................................ Taxes payable .................................................................................. Total current liabilities.............................................................. Long-term notes payable ....................................................................... Stockholders' equity: Contributed capital.......................................................................... Retained earnings............................................................................ Total stockholders' equity ........................................................ Total liabilities & stockholders' equity ...................................................

$ 8,000 40,000 $125,000 2,400 122,600 161,000a $331,600 $ 35,000 8,000 27,000 $358,600

$110,000 29,400 $139,400 79,100 $100,000b 40,100c 140,100 $358,600

__________________ a Inventory is reported at the lower of its cost or its market value. b $100,000 = $12,500 shares  $8 per share. c $40,100 = $65,000 cumulative earnings – $24,900 cumulative declared dividends. Based on only one year’s balance sheet it is a very difficult question to answer. This fact proves the point that (1) all the financial statements must be interpreted as a whole, and (2) that the information should be analyzed over a number of years to draw any meaningful conclusions.


However, based on what we have, I would not invest in this company. The current ratio is 2.379 but debt/equity ratio is 1.560, which is a cause for concern in the long term. Further, the company seems to be paying approximately 38% of its retained earnings beginning balance in dividends, which is good for the investors who are looking for short-term return on their capital.

P2–4 Concluded Johnson Company Balance Sheet December 31, 2014 Property, plant, & equipment: Buildings .......................................................................................... Less: Accumulated depreciation ..................................................... Total property, plant, & equipment ......................................... Current assets: Cash ................................................................................................ $ 8,000 Short-term investments .................................................................. Accounts receivable ........................................................................ $125,000 Less: Allowance for uncollectible accounts .................................... 2,400 Net accounts receivable ........................................................... Inventory ......................................................................................... Total current assets .................................................................. Less: Current liabilities: Accounts payable ............................................................................ Taxes payable .................................................................................. Total current liabilities.............................................................. Total…………………………………………………………………… $219,200

$ 35,000 8,000 $ 27,000

40,000

122,600 161,000a 331,600

$110,000 29,400 139,400

Capital Employed: Long-term notes payable ....................................................................... Stockholders' equity: Contributed capital.......................................................................... Retained earnings............................................................................ Total stockholders' equity ........................................................ Total ...........................................................................................

79,100

$100,000 40,100 140,100 $219,200

Many non-US companies begin with non-current assets, add current assets, and then subtract current liabilities to reflect the resources available to generate revenues and profits. The IFRS balance sheet then


lists non-current liabilities and shareholders’ equity, which represent the financing sources of company resources; this amount is often labeled “capital employed.” GAAP balance sheets, on the other hand, list all assets owned (current and long-term) and then categorizes the financing sources (current and long-term liabilities, as well as shareholder equity) for those assets.


P2–5 2011 Contributed Capital: Total assets = Total liabilities + Total stockholders' equity ($300 + $200 + $500 + $100 + $700) = ($200 + $500) + (Contributed cap. + $400) Contributed capital = $700 Net Income: Net income = Sales – Expenses = $1,000 – $400 = $600 Dividends: Ending retained earnings = Beginning retained earnings + Net income – Dividends $400 = $0 + $600 – Dividends Dividends = $200 2012 Inventory: Total assets = Total liabilities + Total stockholders' equity ($300 + $300 + Inventory + $200 + $600) = ($300 + $600) + ($400 + $800) Inventory = $700 Expenses: Net income = Sales – Expenses $400 = $1,100 – Expenses Expenses = $700 Dividends: Ending retained earnings = Beginning retained earnings + Net income – Dividends $800 = $400 + $400 – Dividends Dividends = $0 2013 Accounts Receivable: Total assets ($200 + Accts. rec. + $400 + $400 + $700) Accounts receivable

= Total liab. + Total stockholders' equity = ($500 + $800) + ($600 + $300) = $500

Expenses: Net income = Sales – Expenses ($100) = $700 – Expenses Expenses = $800 Dividends: Ending retained earnings = Beginning retained earnings + Net income – Dividends $300 = $800 + ($100) – Dividends Dividends = $400


P2–5

Concluded

2014 Accounts Payable: Total assets = Total liabilities + Total stockholders' equity ($500 + $700 + $400 + $400 + $800) = (Accts. pay. + $700) + ($600 + $600) Accounts payable = $900 Net income: Ending retained earnings = Beginning retained earnings + Net income – Dividends $600 = $300 + Net income – $200 Net income = $500 Sales: Net income = Sales – Expenses $500 = Sales – $600 Sales = $1,100 In order to assess the financial performance of this company, we need to calculate the measures of solvency and earning power. Respective measures are computed as follows: Measures of Solvency

2011

2012

2013

2014

Current Ratio: Working Capital: Debt/Equity Ratio:

5 $800 .64

4.33 $1,000 .75

2.20 $600 1.44

1.78 $700 1.33

The only measure of earning power that we can compute for this company is Return on Equity. The other measures, such as EPS and P/E Ratio, cannot be computed since the relevant information is not available. Measures of Earning Power

2011

2012

2013

2014

Return on Equity:

.55

.33

—*

.42

*No return on stockholder’s equity during 2013 since the company suffered a loss of $100. Overall, looking at the measures of solvency and earning power, one can safely conclude that the financial performance and position of the company has deteriorated since its inception in 2011. The current ratio has continued to decline and working capital has also gone down. While the company has taken more debt, it has been unable to leverage against the interest of the stockholders, since the return on equity has declined considerably. In one year, 2013, the company even suffered a loss. The company paid dividends even during the year of loss, indicating a poorly devised dividend policy.


P2–6 Kroger Balance Sheet December 31, 2012, 2011 2012 Assets Cash ............................................................................................. Accounts receivable ....................................................................... Inventory ........................................................................................ Property, plant, and equipment (net) ............................................ Other assets ................................................................................... Total assets .................................................................................$ Liabilities and Stockholders’ Equity Accounts payable ........................................................................... Other short-term debts .................................................................. Long term debt............................................................................... Stockholders’ Equity....................................................................... Total liabilities and stockholders’ equity........................................

2011

$

1,193 1,051 5,146 14,875 2,387 24,652 $

$

$

$

4,524 6,533 9,381 4,214 $ 24,652

974 949 5,114 14,464 1,975 23,476 4,329 4,776 10,405 3,966 $ 23,476

Kroger Income Statement For the Years Ended December 31, 2012, 2011 Sales ............................................................................................. Expenses......................................................................................... Net income .....................................................................................

2012 $ 96,751 95,254 $ 1,497

2011 $ 90,374 89,772 $ 602

Solvency refers to a company’s ability to pay its obligations as they come due. The current ratio provides a measure of solvency by comparing those obligations that are coming due in the near future against those assets that the company expects to convert into cash or consume in the near future. Based on its current ratio, Kroger does not have sufficient current assets to cover its existing current liabilities in either year. In 2012 the current ratio was 0.67 ($7,390/$11,057), while it was 0.77 ($7,037/$9,105) in 2011. However, grocery companies, due to the perishable nature of their inventory, often have low current ratios. Earning power refers to a company’s ability to generate net assets through operations. Income has more than doubled, as measured in terms of dollars and as a percentage of sales. Margins are thin in the company’s industry, but Kroger has shown consistent earning capabilities in the time period.

P2–7 a. Assets are, for the most part, recorded at original cost. Over a period of time, the value of an item will change. For instance, the value of Eat and Run's property, plant, and equipment will most likely change as the items become older. Consequently, over time the cost of an item may have no relation to the item's market value. Since the cash received from selling an asset is based on the asset's market value, the asset's book value is not an accurate indicator of a company's value. b. The value of the firm would equal the sum of the fair market value of the assets less the sum of liabilities. The value of Eat and Run would, therefore, be as follows:


Market Value Cash ................................................................................... Short-term investments .................................................... Accounts receivable........................................................... Inventory ........................................................................... Prepaid insurance .............................................................. Property, plant, & equipment ........................................... Other assets ....................................................................... Total market value of assets.............................................. Less: Total liabilities .......................................................... Value of Eat and Run ......................................................... c.

$

25,000 19,000 25,000 33,000 0 100,000 0 $ 202,000 196,000 $ 6,000

If Eat and Run were to go bankrupt, the stockholders would receive anything left after all the assets were sold and the creditors were paid. In this case the fair market value of the assets exceeds the total liabilities, so the stockholders would receive the residual, which would be $6,000. As a practical matter, Eat and Run might have to hire lawyers and accountants for the bankruptcy proceedings. If this were the case, the lawyers and accountants would have to be paid before the stockholders received anything. So in this particular case, there may be nothing left for the stockholders once the creditors, lawyers, and accountants are paid.

P2–8 First, let us compute some relevant ratios that would help to evaluate the financial statements submitted by Romney Heights in support of its loan application to Acme Bank. Ratios 2014

2013

Liquidity Current Ratio (Current Assets ÷ Current Liabilities) Working Capital (Current Assets – Current Liabilities)

2.00

2.00

$7,000

$6,000

1.06

0.96

0.45

0.33

Long-Term Debt Paying Ability Debt/Equity Ratio (Total Liabilities ÷ Stockholders’ Equity) Operating Cash Flow to Total Debt


(Operating Cash Flow ÷ Total Debt) Ratios 2014

2013

Profitability Net Profit Margin (Net Income ÷ Sales)

0.34

0.19

Total Asset Turnover (Sales ÷ Total Assets)

0.55

0.58

Return on Assets (Net Income ÷ Total Assets)

0.19

0.11

Return on Assets (Net Profit Margin  Total Asset Turnover)

0.187

.110

Return on Equity (Net Income ÷ Stockholders’ Equity)

0.39

0.21

A thorough review of the various ratios reveals that Romney Heights is worth the risk. The bank should consider its loan application, at least for a short-term loan. The short-term solvency position is reasonably good. Working capital is positive and the current assets are twice the current liabilities. Regarding long-term debt paying ability the company seems to be heavily leveraged. The debt to equity ratio is more than 1 and has increased from 2013 to 2014. However, the concern is somewhat mitigated by a substantial increase in the proportion of operating cash flows to the total debt held by the company. The overall profitability of the company is on the rise, but the asset utilization is poor and flat. Since the return on equity has almost doubled, the company seems to be able to effectively leverage the increment in its debt to the advantage of its stockholders. Regarding the statement of cash flows, the company seems to be doing fine. Net cash flow from operating activities is positive. The company is investing in its asset base, probably intending to expand in the future by supplementing its cash flow from operating activities with financing either from bank loans or from equity.


P2–9 First, let us compute some relevant ratios that would help us evaluate the financial statements of Ted Tooney. Ratios 2014

2013

Liquidity Current Ratio (Current Assets ÷ Current Liabilities)

1.29

2.00

Working Capital (Current Assets – Current Liabilities)

$2,000

$4,000

Debt/Equity Ratio (Total Liabilities ÷ Stockholders’ Equity)

1.45

0.92

Operating Cash Flow to Total Debt (Operating Cash Flow ÷ Total Debt)

0.75

1.36

Net Profit Margin (Net Income ÷ Sales)

0.15

0.19

Total Asset Turnover (Sales ÷ Total Assets)

3.41

3.87

Return on Assets (Net Income ÷ Total Assets)

0.52

0.74

Return on Assets (Net Profit Margin  Total Asset Turnover)

0.51

0.74

Return on Equity (Net Income ÷ Stockholders’ Equity)

1.27

1.42

Long-Term Debt Paying Ability

Profitability

Looking at the declining trends of all financial indicators, it would be safe to decline Ted’s request for an equity investment in his company. The short-term liquidity of the company is going down. The working capital as well as the current ratio has declined. The company is becoming highly leveraged and the amount of operating cash flow as a percentage of total debt has considerably declined. This all indicates a worsening position. The profitability and return on assets are declining. The return on equity has also declined as the company is not able to leverage its debt to the advantage of its stockholders. Even though the overall liquidity position is not that serious, the trend is towards the decline. In summary, a loan position may be taken with the company, but certainly not an equity position.


P2–10 a. As of 12/31/14 the current asset balance of Ellington Industries is 1.33 times the current liability balance. Since the debt covenant requires this balance to be 2 times the current liability balance, Ellington Industries must have current assets of at least $18,000. Since, it already has $12,000 invested in current assets, it will need to invest an additional $6,000 out of the long-term borrowing of $40,000 to comply with the debt covenant. That would leave $34,000 ($40,000 – $6,000) for additional investment in the land. b. Ellington Industries Balance Sheet January 1, 2015 Assets Current assets ......................................................................... Land investment ..................................................................... Total assets ............................................................................. Liabilities & Stockholders’ Equity Accounts payable ................................................................... Long-term liabilities ................................................................ Stockholders’ equity ............................................................... Total liabilities and stockholders’ equity ................................

$

18,000 89,000 $ 107,000 $

9,000 70,000 28,000 $ 107,000

Ratios Current assets/Current liabilities = $18,000/$9,000 = 2 Total liabilities/Total assets = $79,000/$107,000 ........ = .74 c. Ellington Industries Balance Sheet December 31, 2015 Assets Current assets ......................................................................... Land investment ..................................................................... Total assets ............................................................................. Liabilities & Stockholders’ Equity Accounts payable ................................................................... Long-term liabilities ................................................................ Stockholders’ equity ............................................................... Total liabilities and stockholders’ equity ................................

$

36,000 89,000 $ 125,000

$

7,000 70,000 48,000 $ 125,000

Since the dividend has to be paid in cash, it will come out of the current assets. According to the restrictions imposed by the debt covenant, the current assets must be twice the current liabilities, i.e., at least $14,000. This would result in an excess of $22,000 ($36,000 – $14,000) in the current assets.


Therefore, the company can pay a maximum of $22,000 in dividends without violating the debt covenant. If the company declares and pays $22,000 in dividends, then total liabilities/total assets would be equal to .75 ($77,000/$103,000).

ISSUES FOR DISCUSSION ID2–1 a. Net income represents the change in net assets (i.e., assets less liabilities) generated during the year from operating activities. Alternatively, cash flows from operating activities is the amount of cash the company generated during the year from operating activities. Since cash is simply one of many assets a company has, it is obvious that net income and cash flows from operating activities are not the same. Thus, it is quite possible for a company to have an increase in net assets from operating activities (i.e., net income) and at the same time have negative cash flows from operating activities. The ability of a company to pay dividends is a function of how much cash the company has available. A company could generate negative cash flows from operating activities but have large cash reserves from generating cash from operating activities in prior years. Alternatively, a company may have obtained enough cash to pay a dividend by borrowing the money or by selling assets. Remember, companies can generate cash from investing activities and financing activities in addition to cash from operating activities. b. A company could not continue generating negative cash flows from operating activities and expect to continue in business. A company cannot borrow money or issue stock indefinitely. At some point the creditors will demand to be repaid and the owners will demand some return on their investment. Sooner or later the company will have to generate cash from its operations to repay the creditors. Paying out dividends while generating negative cash flows from operating activities will only increase the company's cash problems.

ID2–2 Analysts and investors following Netflix would react positively to new subscribers, as well as leading market share of subscribers in general. A critically acclaimed show, similarly, would have a positive effect on interested subscribers and would therefore be welcome news to investors. Subscribers, of course, are the source of cash flow for the company, so additional subscribers will mean additional cash flow for the company. Now whether that incoming cash results in profitable operations depends on the associated expenses of gaining and serving those subscribers. But, as shown on the quarterly financial statements, for the first quarter of 2013, at least, the growth of subscribers resulted in revenues exceeding expenses (and well ahead of the previous year’s performance). Analysts and investors would look to the income statement and the statement of cash flows for the resulting effects of the subscriber growth.

ID2–3 a. The excerpt indicates that the Cummins Engine Company's creditors have imposed restrictions on Cummins as part of the borrowing agreement. The covenants restrict Cummins' abilities to pay dividends and borrow money and the relative amount of its current assets and current liabilities. If Cummins fails to comply with the covenants, its creditors could require Cummins to repay the loans immediately.


b. A bank or other creditor would impose such restrictions to protect itself from a loan default. That is, creditors impose restrictions on borrowers, such as the amount of cash that can be paid out for dividends, that increase the probability that the borrower will have sufficient resources to be able to make the interest and principal payments required under the borrowing agreement. c.

Debt covenants are often explicitly based on financial accounting numbers. For example, the current ratio is based on the amount of current assets and current liabilities reported on Cummins' balance sheet. Similarly, compliance with the dividend restriction can be assessed by examining the amount of dividends declared reported in the statement of retained earnings.

ID2–4 Sears Holdings – Sears is a company struggling to survive in the competitive marketplace of retail sales. The company lost cash in the basic operation of its stores, as well as in its needs to repay financing resources. The only source of cash, and this is a troubling one for a company, has been its sale of longterm assets. Most healthy companies (like the other two shown in this problem) invest cash in their long term assets in order to be viable companies in the future. Sears has had to sell off assets in order to keep its cash balances from getting too meager. Amazon.com – Amazon appears to be a classic growth company. Operations generate a significant amount of cash, yet the company’s thirst for growth causes a great outflow of cash for investments in long term assets (such as distribution warehouses in strategically located places). The cash generated from operations is not sufficient to fund the growth, so the company turns to the financing function (debt and/or equity issuances) to generate additional cash for its expanding business. Boeing – Boeing is further along the growth curve than Amazon, as shown by its cash flow pattern. Strong operating cash flow is used to fund cash outflows for long-term investments, but there is still sufficient cash to fund outflows to return cash to shareholders and debt providers. Whereas Amazon was raising financing, Boeing is in a position to return cash to those who provided financing.

ID2–5 From the data given about the Goodrich Corporation it can be surmised that Goodrich has done a good job of generating positive operating cash flow, especially in the most recent year shown. Given the fact that the company is a defense contractor and that defense spending has been heavy in the years shown, the increasing operational cash flow is an understandable trend. Strategically, the large investment in long term assets shown by the company’s cash from investing activities implies that the company is investing in long term assets such as equipment that will be used in the operations of the business. The performance of cash from financing activities shows that Goodrich had raised funds from debt and/or equity in 2009 and 2010 but was able to return cash to shareholders and/or pay down debt, in 2011. Overall, cash balances have remained relatively constant, as the strong cash inflows from operations have been used primarily for the outflows of investing.


ID2–6

A U.S. GAAP balance sheet shows the most liquid accounts first and then lists accounts in the order that they are convertible to cash. Those accounts being closest to cash are listed first. Secondly, liabilities are not shown in parentheses. Finally, some of the equity accounts carry slightly different titles. GlaxoSmithKline Consolidated Balance Sheet As of 12/31/2012 2012

2011

ASSETS: Cash Short term investments Accounts receivable Inventory Other assets Current assets

4,184 130 5,242 3,969 167 13,692

5,714 254 5,576 3,873 750 16,167

Non-Current Assets Investment in Affiliates Property, Plant & Equipment Other Investments Total

579 8,776 3,908 13,263

560 8,748 4,049 13,357

14,520

11,556

Goodwill & Other

Total Assets

41,475

41,080

LIABILITIES Loans Accounts payable Other current liabilities Current liabilities

3,631 8,054 2,130 13,815

2,698 7,359 4,953 15,010

Long term liabilities Loans Long term payables Long term liabilities

14,671 6,242 20,913

12,203 5,040 17,243

Shareholders’ Equity Common stock Additional paid in capital Minority Interest Retained earnings

1,349 2,022 937 2,439

1,387 1,673 795 4,972


Total shareholder’s equity

6,747

Total Liabilities and Shareholders’ Equity ID2–7

41,475

8,827

41,080

Earnings according to GAAP are accrual numbers, meaning that they don’t represent cash. For example, net income is derived by subtracting expenses from revenue, but revenue can be recognized even if the company has yet to receive the cash (accounts receivable are booked). If the accounts receivable, which represent a promise from a customer to pay cash, never convert into cash, the accrual net income figure is an overstatement of the company’s earnings power. Investors, therefore, look at net income in conjunction with operating cash flow to determine if the various components of accrual net income are supported by cash flows.

ID2–8 Both GE and Comcast are interested in focusing efforts on core business activities: for GE, running a television network did not fit in with its manufacturing and financial businesses, while Comcast saw a television network as a logical vertical extension of its core business of providing cable television services to consumers. The NBC transaction was completed simultaneously, with NBC’s ownership switching from GE/Vivendi to Comcast/GE. From GE’s perspective, it saw a net cash inflow (cash from investing activities decreased to purchase Vivendi’s 20% share and then increased when the 51% stake was sold to Comcast), while its balance sheet ultimately showed a decrease in NBC-related assets (from a consolidation of all NBC assets to a line item investment in NBC).

ID2–9 An analyst following both Nike (GAAP) and Adidas (IFRS) would not be pleased with the SEC decision. An analyst would like to review the financial results of the companies in a side-by-side, “apples-to-apples” comparison. With the previous requirement, the analyst could take the reconciliation prepared by Adidas and compare its net income and stockholders’ equity to those of Nike. Once the requirement was dropped, the analyst (with the same need for industry peer comparison) would effectively have to do the reconciliation by herself. The analyst would therefore need to be an expert in both GAAP and IFRS in order to compare the results of the two footwear and athletic apparel firms.

ID2–10 a. Sales Cost of sales S G & A expenses Taxes

2012 $ 50,175 $ 37,905 20,634 41.1% 9,988 19.9% 2,598 5.2%

2011

2010 $

.

29,321

13,188 34.8% 7,313 19.3% 2,589 1.6%

10,417 35.5% 4,761 16.2% 2,291 7.8%


Net income $ 10,737 21.4% $ 9,737 25.7% $ 8,505 29.0% Google’s net income has increased in raw dollars but has decreased as a percentage of sales as cost of sales, especially after the Motorola purchase, have eaten up a greater share of revenues.

b. Current assets Noncurrent assets Total assets

2012 $ 60,454 64.5% 33,344 35.5% $ 93,798

2011 . $ 52,758 72.7% 19,816 27.3% $72,574

From 2011 to 2012 there has been a slight change in the allocation between current and non-current assets. The company has decreased its share of shorter term assets. c. Current liabilities Long-term liabilities Total assets

2012 $14,337 7,746 $93,798

2011 . 15.3% $8,913 12.3% 8.3% 5,516 7.6% $72,574

From 2011 to 2012 Google increased the percentage of assets financed with current and long-term liabilities, meaning it decreased the percentage of assets financed with equity. d. Google is rapidly growing and finances that growth with strong cash from operations as well as cash garnered from financing sources. The company uses that cash to invest heavily in long term assets. e. The company does not pay dividends, as it is currently using that cash to fund its growth. No dividends appear in the Financing section of the statement of cash flows, nor are any dividends shown in the Retained Earning column on the statement of stockholders’ equity.


CHAPTER 3 THE MEASUREMENT FUNDAMENTALS OF FINANCIAL ACCOUNTING BRIEF EXERCISE BE3–1 1. Fiscal period 2. Economic entity 3. Conservatism 4. Consistency 5. Revenue recognition

6. Materiality 7. Matching 8. Objectivity 9. Objectivity 10. Stable dollar

EXERCISES E3–1 At the beginning of the period, $10 billion would allow the corporation to buy a "basket of goods." Due to the increase in the general price level, the same basket of goods would cost more than $10 billion at the end of the period. Therefore, the corporation would have less purchasing power at the end of the period than at the beginning of the period. The decrease in purchasing power would be computed as follows: 1. Compute the cost of the basket of goods at the end of the period: = $10,000,000,000 (1 + inflation rate) = $10,000,000,000  1.02 = $10,200,000,000 2. Compute change in cost of the basket of goods for the period: = $10,200,000,000 – $10,000,000,000 = $200,000,000 This decrease in purchasing power would not be reflected in the corporation's financial statements. Accountants adhere to the stable dollar assumption, which means that changes in the general price level are ignored when determining the value of assets and liabilities. This assumption allows users of financial statements to compare financial statements from different points in time. Boeing would want to keep its cash balance as low as possible because money sitting as cash makes very little to no investment income. Most companies would try to maximize its interest income by investing most of its cash. At the same time the company would not reduce its cash balance to zero because it has to have cash on hand to pay bills as they come due everyday.

E3–2 a. Each land acquisition would be recorded at its original cost of $15,000, for a total of $30,000. b. No, the company could not purchase the same basket of goods for $15,000 in 2014 as in 1996. To purchase the same basket of goods in 2014, the company would need $24,000 [$15,000  (1 + 60%)]. Therefore, cash held by the company from 1996 to 2014 would be subject to an economic loss of $9,000 ($24,000 – $15,000).

1


c.

There are two alternatives for reporting the value of the land if the stable dollar assumption is ignored. The first alternative is to report both pieces of land at 1996 dollars. The second alternative is to report both pieces of land at 2014 dollars. The two alternatives are shown below.

E3–2 Concluded 1996 land in 1996 dollars 2014 land in 1996 dollars Total land in 1996 dollars a b

$ $

15,000 9,375a 24,375

2014 land in 2014 dollars 1996 land in 2014 dollars Total land in 2014 dollars

$ $

15,000 24,000b 39,000

$9,375 = $15,000 cost of land in 2014 ÷ 1.6 $24,000= $15,000 cost of land in 1996  1.6

E3–3 Original Cost Cash Short-Term Investments Inventories Prepaid Expenses Long-Term Investments Notes Receivable Machinery Equipment Land Intangible Assets Short-Term Payables Long-Term Payables 1 2 3 4

2 X 3

Fair Market Value

Present Value

X 1 2 3

Replacement Cost

2 3 X

4 4 X 4 X X

Short-term investments are recorded at fair market value. Inventory is reported at the lower of original cost or market value, where market value can be based on fair market value or replacement cost. Long-term investments are carried on the books at original cost, fair market value, or amortized value, depending upon the type of investment. Long-lived assets, such as machinery, equipment, and intangible assets, are reported on the balance sheet at net book value, which equals original cost less the portion of original cost amortized to date.

E3–4 a. If Cisco were to determine that a portion of its inventory were obsolete, the company would lower the value of the inventory (an asset on the balance sheet) and would book an expense on the current income statement (which would ultimately lower stockholder equity on the balance sheet). b. Ultimately, the management of Cisco is responsible for forecasting future demand for its products (currently held in inventory) and the valuation to be used in the financial statements. Ultimately, therefore, it is management who controls the value of the asset and any associated writedown expense on the income statement.

2


c.

Driving the valuation of inventory is the aim to not overstate the value of items yet to be sold. We do not want financial statements to list the value of inventory at its cost if market forces have changed to the point that the company could only sell the inventory for a price below its cost. In this sense, conservatism rules the day, requiring companies to list their inventory at the lower of the cost of the inventory or the inventory’s value in the market. In equivalent terms, an owner of a gas station should not list his gasoline inventory at his cost of $2.50 per gallon if market forces have pushed down the selling price (the value in the market) of gasoline to $2.25 per gallon. We need, however, the company’s management to be able to back up the market values used with objective data (such as the published current selling prices of gasoline) in order to prevent manipulation of the financial statements. If a company uses a subjective measure of market value to book a loss in one reporting period (due to the writedown expense) and then generates a larger profit the following period (when the marked-down inventory is sold in the market at typical market prices), the financial statements have not provided the reader a clear understanding of the business performance.

E3–5 a. The most common point at which a company would recognize revenue is at the time of delivery. So in this case McKey and Company would recognize revenue in February. b. The four criteria for recognizing revenue are (1) the company has completed a significant portion of the production and sales effort, (2) the amount of revenue can be objectively measured, (3) the company has incurred the majority of costs, and remaining costs can be reasonably estimated, and (4) cash collection is reasonably assured. Presumably McKey and Company is reasonably assured that Cascades Enterprises will eventually be able to pay the $40,000, or McKey would not have entered into the agreement with Cascades Enterprises. Since the production and sales effort was not really complete until McKey shipped the brackets on February 9, February 9 appears to be the appropriate date to recognize the revenue. c.

Under the appropriate conditions, revenue can be recognized at several points in time. Revenue could be recognized (1) during production, (2) at the completion of production, (3) at the point of delivery, or (4) when the cash is collected. Case 1 normally arises in long-term construction projects such as office buildings, bridges, and so forth. Case 2 arises where goods are manufactured to the exact specifications of a customer, and the goods cannot be sold to another party. Case 3 is the most common point of revenue recognition. Case 4 arises when cash collection is not reasonably assured.

d. McKey's managers could be interested in the timing of revenue recognition due to incentives provided by contracts. For example, the managers may be paid a bonus based upon accounting income. A manager who is trying to maximize his or her bonus might prefer recognizing revenue in a particular period rather than in a different period. Another contract that might influence the actions of managers would be a debt covenant. If a debt covenant stipulates a maximum debt-to-equity ratio, and the company is nearing the ratio, the managers could improve the ratio by increasing stockholders' equity. One way to increase stockholders' equity is to increase net income. Consequently, speeding up the recognition of revenue (called front loading) might prevent the company from violating a debt covenant.

E3–6 a. (1) Revenue recognized at the end of the project. Lahmont Bridge Builders Income Statement For the Period Ended

Revenues from long-term contracts 3

Period 1

Period 2

$0

$600,000


Construction expenses Net income

E3–6

0 $0

400,000 $200,000

Period 1

Period 2

$ 450,000a 300,000 $ 150,000

$ 150,000b 100,000 $ 50,000

Concluded

(2) Revenue recognized during production. Lahmont Bridge Builders Income Statement For the Period Ended

Revenues from long-term contracts Construction expenses Net income

a $450,000 = ($300,000 ÷ $400,000)  $600,000 b $150,000 = $600,000 – $450,000, or ($100,000 ÷ $400,000)  $600,000

(3) Revenue recognized when payments are received. Lahmont Bridge Builders Income Statement For the Period Ended Period 1 $ 400,000 266,667 $ 133,333

Revenues from long-term contracts Construction expenses Net income Note:

In all three cases, costs are recognized as expenses in accordance with the matching principle. That is, the costs are not expensed until the costs have helped generate a benefit in the form of revenue.

b. Assumption (1) (2) (3)

Period 2 $200,000 133,333 $ 66,667

Period 1 Income $

Period 2 Income

0 150,000 100,000

$200,000 50,000 100,000

4

Total Income $200,000 200,000 200,000


E3–7 a. Original cost Depreciation expense Accumulated depreciation Net book value

2014

2015

2016

2017

2018

$25,000 5,000 5,000 20,000

$25,000 5,000 10,000 15,000

$25,000 5,000 15,000 10,000

$25,000 5000 20,000 5,000

$25,000 5,000 25,000 0

b. Since the truck has an estimated useful life of five years, it is assumed that RDP and Brothers will receive a benefit from using the truck in each of the five years. Consequently, RDP and Brothers expect to receive benefits from the truck in the future. According to the matching principle, costs should be matched against the benefits the costs help generate. Since the benefits from the truck will not be realized until future periods, the cost of the truck should be capitalized. In addition, an asset, by definition, is something that a company controls that is expected to provide benefits in the future. In order to receive the future benefits from using the truck, RDP and Brothers must continue to exist. In other words, RDP and Brothers is assumed to be a going concern. If accountants did not use the going concern assumption, it would be inappropriate to capitalize costs because the company may not exist when the expected benefits resulting from the cost are to be realized. c.

It is assumed that the truck will help generate a benefit (i.e., revenue) in each year of its useful life. Under the matching principle, the cost of an item should be allocated to the period(s) in which the cost helps generate a benefit for the company. In this particular case, the truck is expected to provide a benefit for five years. If the entire cost was expensed in 2014, then an improper matching of costs with the related benefits would arise in 2014–2018. However, by capitalizing the cost of the truck in 2014 and then allocating a portion of the cost to each of the next five years, RDP and Brothers is able to match the cost of the truck with each period in which the truck is expected to provide a benefit to the company.

E3–8 a. Costs that are expected to provide future benefits to a company are, by definition, assets. Hence, all such costs should be capitalized. As these costs help generate benefits, such as revenue, the costs are recognized as expenses and matched against the corresponding benefits. b. Capitalizing expenditures and subsequently amortizing these costs are not costless activities. A company incurs costs, such as bookkeepers' salaries, supplies, and so forth, when engaging in such activities. In certain instances, these bookkeeping costs may exceed the benefits derived from properly capitalizing and amortizing expenditures. This situation is most likely to arise when the amount of an expenditure is very small in relation to some criteria such as total expenditures, net income, or total assets. Such an expenditure is so small that users of financial statements would not care whether the expenditure was capitalized or immediately expensed. The financial statement users' decision process would not be influenced by the accounting treatment given such expenditures. In these cases, a company would apply the concept of materiality to decide whether an expenditure should be capitalized or expensed.

5


E3–9 a. (1) During 2013 the company changed depreciation methods. This change resulted in an increase of the book value of the assets versus if no change in accounting method had occurred. In other words, the depreciation expense went down by the same amount, i.e., $5,000. A decrease in the depreciation expenses would increase the net income by the same amount, i.e., $5,000. (2) During 2015 the company changed its method of inventory valuation, which also increased the book value of the inventory. Since the cost of inventory is allocated either to the cost of goods sold account or to the ending inventory account, this change implies that the Cost of Goods Sold decreased by $9,000. This would also increase the net income by $9,000. Overall it seems the company is having a bad year and is attempting to use liberal accounting policies to paint a “rosy” picture of the operations. b. Net income as reported Effect of depreciation change Effect of inventory change Adjusted net income

2012

2013

2014

2015

$ 21,000 0 0 $ 21,000

$ 24,000 (5,000) 0 $ 19,000

$ 23,000 (5,000) 0 $ 18,000

$ 29,000 (5,000) (9,000) $ 15,000

The adjusted net income figures indicate that if the company had not changed accounting methods, it would have reported declining profits. In fact, the company would have reported net income of only $15,000 in 2015. The reported net income figures have been enhanced with accounting techniques rather than by sound economic health. Consequently, the company's performance would be viewed less positively.

6


E3–9 c.

Concluded

Companies should adhere to the principle of consistency. This principle states that a company should use the same accounting principles and methods from year to year. Such a practice promotes the comparability of the company's financial statements over time and also promotes user confidence in the financial statements. If a company was free to switch accounting principles and methods at will, financial statement users would place very little faith in the statements. Under certain conditions, companies may switch accounting principles. The primary condition that must be met before a company may switch methods is the approval of the company's auditors. The company must convince its auditors that the environment it faces has changed sufficiently so that the new accounting principle, rather than the old principle, more appropriately reflects the company's financial position and performance.

E3–10 a. Under U.S. GAAP, conservatism and objectivity are important concepts in the valuation of assets such as inventory. GAAP statements are going to list inventory at its historic cost (an objective number), unless it can be documented objectively that market value has dropped below cost, in which case the inventory will be carried at the lower (more conservative) market value figure. IFRS statements, on the other hand, are going to first look to market value to determine carrying amounts. If management determines (without necessarily providing objective verification) that market value is different than the current carrying cost, the balance sheet value will be changed— and it may be changed higher or lower, depending on the move in market prices. (Inventory changes in GAAP will only be write-downs, never write-ups.) b. It is possible that year-end adjustments for inventory at Adidas will be positive (that is, the carrying amount of inventory will increase), due to management’s belief that its current ending inventory is more valuable than previously thought. If, for example, Adidas has a shoe line that is very popular in current youth fashion and the company is able to sell the shoes for a higher price (due to the demand from its customers), the company could write up the value of the ending inventory to its higher market value. Nike, on the other hand, follows U.S. GAAP, which would preclude such a move; Nike could have a hugely popular line of shoes (which may indeed prove to sell at higher market prices than previously thought), but the company would have to leave its ending inventory at its (objective) historic cost. Any increase in value in Nike’s inventory would have to be recorded only when the company actually sells the inventory at that higher price; GAAP does not allow the higher market value to be figured until the asset is sold in the market.

7


PROBLEMS P3–1 a. The company would report a gain of $10,000. b. No. During 2014 the purchasing power of money decreased by 10%. On December 31, 2014, it would require $1,100 [$1,100  (1 + 10%)] to purchase the same basket of goods that $1,000 would have purchased on January 1, 2014. The difference in purchasing power gives rise to an economic loss of $100. Therefore, $20,000 would not allow someone to purchase twice as many goods and services on December 31, 2014 as on January 1, 2014 To be able to purchase the same amount of goods, an individual would need $11,000 [$10,000  (1 + 10%)], which implies that to be able to purchase twice as many goods and services, an individual would need $22,000. c.

The $10,000 gain can be broken down into two components: a gain due to the increase in the value of the property and a gain due to general inflation. Since the inflation rate during 2014 is 10%, the value of the land would be expected to increase during 2014 by 10%, or $1,000. The remaining $9,000 of the gain is due to an increase in the value of the property, which represents an economic gain. Accountants ignore the effects of inflation due to the stable dollar assumption. This assumption allows financial statement users to compare financial statements from different points in time. Further, the stable dollar assumption gives rise to more objective financial statements. In order to adjust for the effects of inflation, the inflation rate must be known. Should the adjustment be based on wholesale, retail, global, national, state, industry, or company-specific inflation rates? Company-specific rates are probably the most relevant rate, yet they are probably the most subjective. The other rates may not be relevant for some companies. If managers were allowed to select the appropriate rate for their companies, they could manipulate the financial statements. On the other hand, if the FASB or SEC mandated the use of a particular inflation rate, the rate would not be relevant for many companies. Consequently, the choice of an inflation rate would be arbitrary and could lead to distortions in the financial statements. The use of original costs is arbitrary and also leads to distortions in the financial statements. However, the use of original costs has at least two advantages over the inflation-adjusted amounts. First, the use of original costs eliminates a potential source of manipulation of financial statements by managers. Second, users may disagree on the appropriate inflation rate for a company, and original-cost financial statements allow users to individually adjust the financial statements for their perceptions of inflation.

8


P3–2 a. The Banking Corporation will recognize interest revenue of $240. The amount of cash given to Bush Enterprises was $4,760 and in exchange Banking Corporation received a note receivable for $5,000. The difference is the amount of interest revenue that Banking Corporation will recognize in its books on December 31. b. Banking Corporation is better off at the end of the year than if the company had not invested the $4,760 on January 1. Overall, however, the company is worse off financially on December 31 than on January 1. To purchase the same basket of goods on December 31 as it could purchase for $4,760 on January 1, Banking Corporation would need $5,236 [$4,760  (1 + 10%)]. In other words, the company would need an additional $476. By loaning the money to Bush Enterprises during the year, Banking Corporation acquired $240. Hence, during the year the company became economically worse off by $236 ($476 – $240). Just to maintain its purchasing power, Banking Corporation would have to loan money at the inflation rate. To improve its purchasing power, Banking Corporation would have to loan money at a rate that exceeds the inflation rate. c.

As indicated in Part (b), Banking Corporation actually lost $236 of purchasing power during the year. On the other hand, Bush Enterprises gained purchasing power during the year. Bush could have invested the $4,760 it borrowed in a basket of goods on January 1. On December 31, Bush could sell the basket of goods for $5,236, repay Banking Corporation $5,000, and still have $236 left over. Consequently, Bush Enterprises ended up with the better deal. Whenever the interest rate on a loan is less than the inflation rate, the borrower has an advantage. Since accountants adhere to the stable dollar assumption, inflation is not reflected in financial statements. Consequently, the financial statements of Banking Corporation would indicate the company is better off by the amount of interest revenue, while the financial statements of Bush Enterprises would indicate the company is worse off by the amount of interest expense.

P3–3 a. Cash Inflows From Sale Asset A: Option 1 Option 2 Option 3

$1,500 1,500 0

Asset B: Option 1 Option 2 Option 3

500 500 0

Asset C: Option 1 Option 2 Option 3

3,000 3,000 0

Cash Outflow for Replacement $

0 (1,000) 0

Future Cash Flows 0 5,000 2,500

$1,500 5,500 2,500

0 (2,000) 0

0 3,500 2,500

500 2,000 2,500

0 (3,500) 0

0 5,000 2,500

3,000 4,500 2,500

9

$

Total Cash Flows


P3–3

Concluded

Kathy made the correct decision with respect to Assets B and C, but not to Asset A. As demonstrated above, Option 3 (i.e., retaining the asset) yields the highest net cash flows for Asset B. For Asset C, Option 2 (i.e., selling and replacing the asset) yields the highest net cash flows. However, the best option for Asset A is Option 2. If Kathy had selected this option, she would expect to generate a total of $5,500 in net cash inflows, an increase of $3,000 over the net cash inflows that are expected under the option she selected. b. The original cost information should not be used in evaluating Kathy’s decisions. Original costs represent sunk costs, and sunk costs should not be considered in future decisions. In evaluating the performance of a manager, we are interested in the cash flows generated by the manager. If the cash flow information is not available, then proxies for the cash flows must be used in the evaluation process. One such proxy is original cost data, which may be helpful in computing net income. However, if the cash flow information is available, then this information should be used in evaluating the performance. Since the cash flow information is available in this case, the original cost data can and should be ignored. c.

Under generally accepted accounting principles, assets should be carried on the balance sheet at original cost. Assuming that Kathy proceeds with her decision and keeps Assets A and B and replaces Asset C, the company should report the following amounts for each asset. Asset A Asset B Asset C

$4,000 1,500 3,500

The company is applying the principle of objectivity, which states that financial accounting information must be verifiable and reliable and that the value of transactions be objective. In many cases, original cost is the most objective of the potential valuation bases.

P3–4 a. Real sales did not actually increase by 22% from 2010 to 2012. To compute the real percentage change in sales, inflation must be considered. Converting 2012 sales to 2010 dollars reveals that 2012 real sales were actually $9.13 = [$9.5 ÷ (1 + 4%)]. Consequently, sales increased from 2010 to 2012 by $1.33 million, which is only a 17% increase in sales. b. (1) 2012 sales in 2010 dollars = $9.5 ÷ (1 + 10%) = $8.64 (2) Real change in sales = 2012 sales in 2010 dollars – 2010 sales in 2010 dollars = $8.64 – $7.8 = $0.84 (3) Real percentage change in sales

= Real change in sales ÷ 2010 sales in 2010 dollar = $0.84 ÷ $7.8 = 10.77%

10


P3–4 c.

Concluded The stable dollar assumption assumes that inflation does not exist. So under this assumption, sales actually increased by 22%. However, once one realizes that the stable dollar assumption is simply an assumption that promotes the comparability of financial statements from different points in time and that it does not accurately reflect reality, one must consider price changes when comparing financial data from different points in time.

P3–5 a.

The first step in a comparison across currencies is to convert the different statements into one currency, using the latest available exchange rate. Converting the pounds of GlaxoSmithKline into U.S. dollars is shown below: Sales 26.4 pounds x $1/.62 pounds = $42.6 Assets 41.5 pounds x $1/.62 pounds = $66.9 Equity 6.7 pounds x $1/.62 pounds = $10.8 Converting the Euros of Sanofi into U.S. dollars is shown below: Sales 34.9 Euros x $1/.76 Euros = $ 45.9 Assets 100.4 Euros x $1/.76 Euros = $132.1 Equity 57.5 Euros x $1/.76 Euros = $ 75.7

From all three perspectives, Pfizer is the largest of the three companies. b. If exchange rates move drastically, the comparison might yield a different answer. For example, if the exchange rate between the dollar and the Euro changed from .76 to .52 (due to macroeconomic events, such as a global debt crisis) then the Assets of Sanofi would be larger (compared U.S. dollar to U.S. dollar) than those of Pfizer.

P3–6 a. In this case, the purchase price should equal the stream of future cash flows discounted to reflect the time value of money. The purchase price would be calculated as follows (millions). Purchase price

= Present value of future cash flows = $198  Present value of an ordinary annuity factor for r = 12% and n = 10 = $198  5.65022 (from Table 5 in Appendix) = $1,118.7

b. Book value represents the residual ownership interest in the company based upon the financial statement values. This residual interest is, by definition, total stockholders' equity. Therefore, the book value of Manpower, Inc. is $2,501 ($1,399 of common stock + $1,102 of retained earnings). Using the accounting equation, the book value can also be calculated as total assets less total liabilities. c.

The purchase value of a company can be different from the book value of the company because the fair market value of individual assets and liabilities may be different from the book value of individual assets 11


and liabilities. The book values of many assets are largely based upon original costs, which ordinarily do not reflect fair market values.

P3–7 a. The book value of the building equals the value of the building according to Barry Smith's company's financial records. Long-lived assets are initially recorded at their cost, and then over time the assets are reported at net book value, which is original cost less the portion of the asset's cost amortized to date. In this case, Barry Smith paid $90,000 for the apartment building, and as of January 1, 2014, none of the cost had been amortized. Thus, the book value of the building on January 1, 2014 is $90,000. The economic value of the building is equal to the present value of the cash inflows the building will generate in the future less the present value of cash outflows the building will require in the future. In this particular case, there are two different types of cash inflows: the annual rental amounts of $65,000, which would be an annuity, and the expected proceeds of $40,000 from selling the building. The net present values of the cash flows are calculated below. Annual net cash flows Present value = ($65,000 cash inflow – $45,000 cash outflow)  Present value of an ordinary annuity factor for i = 10% and n = 10 = $20,000  6.14457 (from Table 5) = $122,891.40 Proceeds from sale of building Present value = $40,000  Present value factor for i = 10% and n = 10 = $40,000  .38554 (from Table 4) = $15,421.60 Total present value

= $122,891.40 + $15,421.60 = $138,313

Since the present value of future cash flows exceeds the purchase price of $90,000, it appears that Barry made a wise investment. b. Barry Smith Income Statement For the Year Ended December 31, 2014 Rental revenue ................................................................................................... $ Management expenses ......................................................................................... Depreciation expense............................................................................................ Net income ..........................................................................................................

12

65,000 (45,000) (5,000) $ 15,000


P3–7

Concluded Barry Smith Balance Sheet As of December 31, 2014

Assets

c.

Liabilities & Stockholders' Equity

Cash Building Accumulated depreciation

$ 20,000 90,000 (5,000)

Total assets

$ 105,000

Liabilities Contributed capital Retained earnings Total liabilities and stockholders' equity

$

0 90,000 15,000

$ 105,000

Present value of future cash flows on December 31, 2014: Present value of annual net rentals

Present value of sale proceeds

Total present value

= $20,000  Present value of an ordinary annuity factor for i = 10% and n = 9 = $20,000  5.75902 (from Table 5) = $115,180.40 = $40,000  Present value factor for i = 10% and n = 9 = $40,000  .42410 (from Table 4) = $16,964

= $115,180.40 + $16,964.00 = $132,144.40

Economic income = Net cash received during 2014 + (12/31/14 present value – 1/1/14 present value) = ($65,000 cash inflow – $45,000 cash outflow) + ($132,144.40 – $138,313.00 (from part [a]) = $20,000.00 – $6,168.60 = $13,831.40 Accounting income differs from economic income because economic income incorporates the time value of money. Hence, economic income reflects that the purchasing power of $1.00 received on December 31, 2014 is not equivalent to the purchasing power of $1.00 received on December 31, 2015. Accounting income, through the stable dollar assumption, ignores the time value of money. Further, economic income considers future events (i.e., discounted future cash inflows and outflows), whereas accounting income considers only past events. d. The book value of the building on December 31, 2014 equals the cost of the building less the associated accumulated depreciation. Therefore, the book value is $85,000 ($90,000 – $5,000). The present value of the building equals the present value of future cash flow, which as of December 31, 2014, is $132,144.40 (from Part [c]).

13


P3–8 a. Book value on 12/31/14

= Total book value of assets – Total value of liabilities = $124,000 – ($8,000 + $20,000) = $96,000

b. The economic value of Myers and Myers equals its future cash flows discounted to reflect the time value of money. Myers and Myers have two streams of future cash flows. The first type is annual cash flows, which is an annuity, and the second type is the cash flow from the sale of the business. The present values of these two cash flows are calculated below. Annual cash flows Present value = = =

$20,000  Present value of ordinary annuity factor for i = 10%, n = 10 $20,000  6.14457 (from Table 5) $122,891.40

Proceeds from sale of business Present value = $80,000  present value factor for i = 10%, n = 10 = $80,000  .38554 (from Table 4) = $30,843.20 Total present value of future cash flows c.

Liquidation value

= = =

= =

$122,891.40 + $30,843.20 $153,734.60

Total fair market value of assets – Total value of liabilities $124,000 – ($8,000 + $20,000) $96,000

d. Book value is based upon the original cost of individual assets. This value provides little indication of a company's current value due to price changes. The problem is magnified as the company's assets age. Liquidation value is based upon the fair market values of individual assets and liabilities. This value provides an accurate measure for a company planning to cease operations. However, such a measure provides little indication for a company that is a going concern. Under the going concern assumption, accountants are concerned with providing accounting numbers for companies that will continue operating indefinitely. Present value is based upon future cash flows; as such, it incorporates all non-quantifiable assets, such as employee loyalty and customer loyalty. This value more accurately reflects the economic value of a company, since it captures items not included on a balance sheet under GAAP. Unfortunately, it is difficult, if not impossible, to accurately predict future cash flows. Hence, in most instances, present value amounts do not satisfy the principle of objectivity. A difference between a company's book value and its economic value (i.e., present value of future cash flows) can arise for two reasons. First, this difference can be due to a difference between a company's book value and the fair market value of its individual assets and liabilities. The assets are usually carried on the books at their original cost. However, over time the actual value of the assets would be expected to diverge from their original cost. Second, the difference between a company's book value and its economic value can be due to the excess of the company's economic value over the fair market value of its net assets (i.e., total assets less total liabilities). The net assets are worth more grouped together than individually. Companies generate customer loyalty and name recognition that has value, yet is not reflected in the value of any particular asset. This value is, however, reflected in its economic value. The excess of the company's economic value over the fair market value of its net assets represents goodwill. In this case, Myers and Myers goodwill would be approximately $153,734.60 – $96,000, or $57,734.60.

14


P3–9 a. Ending retained earnings = Beginning retained earnings + Net Income – Dividends $40,000 = $16,000 + Net Income – $0 Net Income = $24,000 b. 2015 FMV

= = =

FMV of total assets – Total liabilities $148,000 – ($6,000 + $20,000) $122,000

2014 FMV

= = =

FMV of total assets – Total liabilities $124,000 – ($8,000 + $20,000) $96,000

2015 Net income

c.

= = =

2015 FMV – 2014 FMV $122,000 – $96,000 $26,000

Present value of future cash flows as of December 31, 2015: Annual cash flows Present value = $20,000  Present value of ordinary annuity factor for i = 10%, n = 9 = $20,000  5.75902 (from Table 5) = $115,180.40 Proceeds from sale of business Present value = $80,000  Present value factor for i = 10%, n = 9 = $80,000  .42410 (from Table 4) = $33,928.00 Total present value of future cash flows

= =

$115,180.40 + $33,928.00 $149,108.40

Economic income = Net cash received during 2015 + (12/31/15 present value – 12/31/14 present value) = $20,000 + ($149,108.40 – $153,734.60) = $20,000 – $4,626.20 = $15,373.80

15


P3–9

Concluded

d. All three income measures provide a performance measure of Myers and Myers. Of the three measures, economic income is the only one that incorporates the time value of money. In theory, holding everything else constant, economic income is probably more accurate than the other measures. Unfortunately, in the real world it is extremely rare that the future cash flows can be predicted with any reasonable degree of accuracy. So in the end, economic income is simply a guess based upon estimates of the timing and amount of future cash flows. Lack of objectivity can also plague income computed using fair market values. If a strong market exists for each of the company's assets, such as with marketable securities, then the company could probably obtain reasonably accurate estimates of what it could receive for selling the assets. In this case, net income would be more relevant than income computed using original costs, since more up-to-date values are being used. Unfortunately, it is not possible to find a market for all assets. For example, a manufacturing company may use highly specialized equipment in its production process. If no other company would use this equipment, does the equipment have a fair market value? Do we assign it a value of zero, assign it a scrap value, or assign it an arbitrary fair market value? The end result is that the value assigned to some assets will be arbitrary and not objective. Computing net income under GAAP circumvents the problem of arbitrary values and lack of objectivity. The values assigned to most assets are based upon their original costs. Assets are usually acquired in arm's-length transactions. Since each party would have opposing interests, the purchase price should accurately reflect the value of the asset on the purchase date. Further, anybody examining the value of the asset could verify the original cost. Although using original cost as a basis for valuing assets provides objective values, original cost amounts can be extremely outdated.

P3–10 a. ABC Inventory Method

Depreciation Method

Income

XYZ Working Capital

Income

Working Capital

B Y $28,000 $26,000 $24,000 $30,000 B X 20,000 26,000 16,000 30,000 A Y 18,000 16,000 14,000 20,000 A X 10,000 16,000 6,000 20,000 Note: Changes in the companies' inventory balances affect net income through Cost of Goods Sold. b. ABC and XYZ both have the highest net income and working capital under the combination of Method B and Y depreciation. Managers could have many reasons for selecting one accounting method over another method. Management is a party to many contracts that may rely on accounting numbers. For example, a manager may have an incentive compensation contract based upon accounting income. A company may have a debt covenant with a creditor that stipulates a minimum level of working capital (or some other relevant measure). Or a manager may have incentives to minimize the company's tax liability. To the extent that the accounting methods used for tax reporting must also be used for financial reporting, a manager may select those methods that provide a tax benefit. In selecting a particular accounting method, a manager will consider the factors that provide an incentive for selecting one method over another, and in the end the manager would be expected to select the accounting method that gives him or her the greatest benefit. In some cases, the accounting method selected by the manager may actually cause net income to decrease. The most likely reason for a manager to select an

P3–10 Concluded 16


accounting method that would cause net income to decrease would be to minimize taxable income, thereby minimizing cash outflows for taxes. c. As an investor, one must realize that different companies may face different environments. To the extent that two companies face different environments, we would expect them to select the accounting methods appropriate to their particular environments. Further, an investor must realize that managers have their own interests and will work to satisfy their interests. In some cases the interests of the managers will be congruent with the investors' interests, and in some cases they will not. Generally accepted accounting principles allow companies to use different accounting methods because it is impossible to select a method that would be appropriate across different companies and environments. Consequently, companies are allowed to select those methods that they deem appropriate for their situation. To an investor, the underlying economic reality (i.e., expected future cash flows) of the company is of interest. Consequently, if companies use different accounting methods, the effects of the different methods on the amounts reported in the financial statements must be considered in comparing different companies.

P3–11 a. Revenues Assumption 1 [$2,400,000  (2/12)] [$2,400,000  (6/12)] [$2,400,000  (3/12)] [$2,400,000  (1/12)] Assumption 2 [$2,400,000  (380/1,140)] [$2,400,000  (380/1,140)] [$2,400,000  (285/1,140)] [$2,400,000  (95/1,140)] Assumption 3 [$2,400,000  (600/2,400)] [$2,400,000  (900/2,400)] [$2,400,000  (300/2,400)] [$2,400,000  (600/2,400)]

Year 1

Year 2

Year 3

Year 4

$400,000 $1,200,000 $600,000 $200,000

800,000 800,000 600,000 200,000

600,000 900,000 300,000 600,000

17


P3–11 Concluded b. Costs Assumption 1 [$1,140,000  (2/12)] [$1,140,000  (6/12)] [$1,140,000  (3/12)] [$1,140,000  (1/12)] Assumption 2 [$1,140,000  (380/1,140)] [$1,140,000  (380/1,140)] [$1,140,000  (285/1,140)] [$1,140,000  (95/1,140)] Assumption 3 [$1,140,000  (600/2,400)] [$1,140,000  (900/2,400)] [$1,140,000  (300/2,400)] [$1,140,000  (600/2,400)]

Year 1

Year 2

Year 3

$190,000 $570,000 $285,000 $ 95,000

380,000 380,000 285,000 95,000

285,000 427,500 142,500 285,000

Net Income Assumption 1 $ 400,000 1,200,000 600,000 200,000

Year 1 – – – –

$190,000 570,000 285,000 95,000

$210,000

Assumption 2 $ 800,000 800,000 600,000 200,000

– – – –

$ 380,000 380,000 285,000 95,000

420,000

Assumption 3 $600,000 900,000 300,000 600,000

– – – –

$ 285,000 427,500 142,500 285,000

315,000

Year 2

Year 3

Year 4

$630,000 $315,000 $105,000

420,000 315,000 105,000

472,500 157,500 315,000

c. Assumption 1 Assumption 2 Assumption 3

Year 4

Total RevenueTotal Cost $2,400,000 2,400,000 2,400,000

Total Net Income $ 1,140,000 1,140,000 1,140,000

18

$1,260,000 1,260,000 1,260,000


P3–12 a. Hydra Aire would recognize the following revenue in each of the 3 years based on the number of toasters produced times the selling price per toaster. Year 1: Year 2: Year 3:

200 200 100

  

$100 $100 $100

= = =

$20,000 $20,000 $10,000

b. Hydra Aire would recognize the following revenue in each of the 3 years based on the number of toasters delivered times the selling price per toaster. Year 1: Year 2: Year 3:

150 200 150

  

c. Assumption 1 Revenues (from part [a]) Expenses Net income

$100 $100 $100

= = =

$15,000 $20,000 $15,000

Year 1

Year 2

Year 3

Total

$ 20,000 8,000 * $ 12,000

$ 20,000 8,000* $ 12,000

$10,000 4,000* $ 6,000

$ 50,000 20,000 $ 30,000

* Expenses = Number of units produced  $40 per unit.

Assumption 2 Revenues (from Part [b]) Expenses Net income

Year 1

Year 2

Year 3

Total

$ 15,000 6,000 * $ 9,000

$ 20,000 8,000* $ 12,000

$ 15,000 6,000* $ 9,000

$ 50,000 20,000 $ 30,000

* Expenses = Number of units delivered  $40 per unit. d. If Hydra Aire’s management is compensated based on the net income of the company, they would prefer to recognize revenues at the point of production. Why? Because it results in higher net income in year 1 and therefore in a higher bonus for the management.

P3–13 a. Cost of Error 1: If Joe McGuire requires disclosure of the lawsuit, and Nelson Repairs, Inc., does not lose the lawsuit, McGuire could incur some costs. If the president of Nelson Repairs, Inc., is serious about not wanting the lawsuit disclosed and McGuire requires that it be disclosed, Nelson could fire McGuire. In this case, McGuire would lose the audit fees of his biggest client. If these audit fees make up a substantial portion of McGuire's total revenues, it is even possible that the loss of Nelson Repairs, Inc., as a client could cause McGuire to cease operations. Cost of Error 2: If McGuire does not require disclosure of the lawsuit, and Nelson Repairs, Inc., loses the lawsuit, McGuire could incur some costs. If any of the stockholders or creditors relied on the financial statements and incurred a loss, these stockholders could sue McGuire for their losses. Since the lawsuit could force Nelson Repairs, Inc., out of business, the potential losses to stockholders and creditors could be quite substantial. A Type 2 error could also damage McGuire's reputation. Financial statement users might view McGuire as a "low quality" auditor and might be unwilling to accept financial statements audited by McGuire. Loss of reputation might cause some potential clients to no longer be interested in hiring McGuire. Even if some 19


P3–13 Concluded new clients did hire McGuire, they might demand a lower audit fee to compensate for a "lower quality" service. Furthermore, some of McGuire's existing clients may no longer wish to engage him as their auditor. Consequently, it appears that the cost of a Type 2 error exceeds the cost of a Type 1 error. McGuire faces a tough decision, though. Although the cost of a Type 2 error appears higher, the probability that Nelson Repairs, Inc., will lose the lawsuit is not very likely. Which factor should McGuire focus on: the cost of the errors or the probability of an adverse outcome? Luckily, McGuire can refer to Statement of Financial Accounting Standards No. 5, "Accounting for Contingencies" for guidance on whether to disclose this lawsuit. =

Cost of an error  Probability of an error

Expected cost of a Type 1 error

= =

$10,000  80% $8,000

Expected cost of a Type 2 error

= =

$50,000  20% $10,000

b. Expected cost of an error

Based upon the expected cost of each type of error, it appears that Joe McGuire should disclose the lawsuit. c.

Conservatism means that "when in doubt, understate rather than overstate." This statement means that when a company faces some uncertainty concerning how to value or record an event, the company should understate, rather than overstate, the financial health of the company. In this case, McGuire has some doubt as to whether Nelson Repairs, Inc., will win or lose the pending lawsuit. McGuire's framework focuses on the relative costs of errors. He wants to determine the cost of a Type 1 error compared to the cost of a Type 2 error. Since the cost of overstatement (i.e., a Type 2 error) exceeds the cost of understatement (i.e., a Type 1 error), McGuire should risk making a Type 1 error. Hence, due to McGuire's doubt concerning the outcome of the lawsuit, he should understate the financial health of Nelson Repairs, Inc., by requiring management to disclose the lawsuit.

20


ISSUES FOR DISCUSSION ID3–1 a. Revenue recognition refers to the recording of revenues when they are earned. Matching refers to recognizing costs as expenses when the costs help generate a benefit (such as revenue). The criteria for recognizing revenue are: (1) The company must have completed a significant portion of the production and sales effort. (2) The amount of the revenues can be objectively measured. (3) The major portion of the costs have been incurred, and the remaining costs can be reasonably estimated. (4) The eventual collection of cash is reasonably assured. The FASB requirement that airlines defer a portion of the current revenues is consistent with these criteria in that at the time of selling a ticket the airlines have not completed a significant portion of the production and sales effort associated with the eventual free trip. In essence the airlines are charging more for the tickets now to cover the cost of "free" trips later. Accordingly, deferred revenue should be established for each ticket sold and then recognized as revenue when passengers use their "free" tickets. Related costs should be capitalized and matched against the revenue when it is recognized. b. As a result of implementing this new accounting policy, Continental Airlines would recognize less revenue. This would cause income to be lower than if Continental continued to use its previous policy for frequent flyer tickets. The new approach to recognizing revenue does a better job of matching revenues with expenses. Part of the overall revenue is recognized at the time when the passenger uses the frequent flyer ticket and the expense for the ticket is incurred by Continental Airlines.

ID3–2 a. Priceline’s method of booking revenue has the potential to mislead investors. It is not the same method that traditional companies in this industry use. It does not make sense from the standpoint that Priceline is reporting revenues for products and services that it does not provide. Priceline is not an airline or a hotel but yet is reporting the revenues that relate to those activities. Priceline provides a service of matching buyers and sellers (like stock brokers) and should only report revenues that relate to the service that it actually provides. b. If investors are going to value the stock of a company based on a multiple of revenue then management has an incentive to report the highest amount of revenue as possible. So by reporting these “gross bookings” as revenue Priceline is able to increase its stock price. This is particularly significant for a company that is losing a lot of cash in its operations. The most common way for a company that is losing cash from its operations is to raise money by selling stock. Typically companies that are losing money do not have the option of issuing bonds and so the only way the company can fund itself is to sell stock. A higher stock price allows the company to give up fewer shares for the needed amount of cash. c.

Allowing internet companies to record revenues differently than traditional companies has a couple of impacts, both of which are negative. One of the goals of GAAP accounting is to have financial statements comparable from one company to another. If different accounting methods are used then this is not possible. If investors are going to value the stock of a company based on a multiple of revenue then 21


management has an incentive to manipulate this number. It is much easier to manipulate revenues than net income.

ID3–3 a. Because Blockbuster is the franchiser, it can dictate policies that the franchises must follow. For example, Blockbuster could dictate when new franchises must purchase merchandise from Blockbuster and how much they must purchase. Blockbuster cannot, however, dictate when the franchises will actually generate revenue; that depends on the franchises' customers. By recognizing revenue when it ships merchandise to franchises, Blockbuster can manage earnings because it has some control over when and how much merchandise is shipped to the franchises. In the case of U.S. Robotics, it is possible to “manage” earnings by shipping some orders early or conversely delaying the shipping of orders to the next period. There is always some risk of capturing the true amount of sales since dealers have the ability to return product to U.S. Robotics. b. The criteria for recognizing revenue under the revenue recognition principle are (1) the company must have completed a significant portion of the production and sales effort, (2) the amount of the revenues can be objectively measured, (3) the major portion of the costs have been incurred, and the remaining costs can be reasonably estimated, and (4) the eventual collection of cash is reasonably assured. If a company meets these criteria when it ships merchandise, then recognizing revenue when merchandise is shipped does not violate GAAP. In fact, most manufacturers and distributors recognize revenue when they ship merchandise to their customers.

ID3–4 a. If Campbell Soup had not made the accounting change, it would have reported $626 for net income in the third year. The restated amounts are more consistent—and hence more comparable—because they are all based on the same accounting principles and methods, whereas the original amounts are based on different accounting principles and methods. b. Changes in accounting methods would be referenced in the audit opinion and the footnotes to the financial statements. In addition, the dollar amount associated with a change in accounting method would be reported separately in the income statement. c.

Based on the information provided in the initial section of the question, it seems Campbell Soup is following the “big bath theory,” which means clean-up the books by taking all the possible losses that you can. The intention is to be able to report higher net income numbers from next year onwards.

ID3–5 a.

Investors are interested in earnings that can be repeated, earnings that can be counted on in future fiscal periods. Therefore, investors would be interested in the 81 cents EPS, because future quarters will not contain any more charges related to the Pfizer acquisition.

b.

Since the analysts were expecting earnings of 79 cents per share and the (repeatable) earnings were 81 cents per share, the analysts should have been pleased. The analysts would take into consideration the one-time expenses related to Pfizer and would adjust the 74 cents EPS to 81 cents.

c.

Companies such as J & J are constantly buying and selling subsidiaries, so comparing results from quarter to quarter, or from year to year, is problematic because the company changes so much over time. Excluding one-time expenses (or gains, for that matter) is one technique analysts use to improve comparability over time.

22


ID3–6 To be able to compare financial results across companies, financial statement users would like those companies to use uniform accounting methods. If the companies do not use uniform accounting methods—as is the case with General Electric and IBM—the financial statement users would have to adjust the amounts reported in the financial statements as if both companies used the same accounting methods. For example, financial statement users would have to adjust IBM's financial statements so that they would reflect accounting numbers as if IBM had used the same depreciation method as General Electric or, alternatively, adjust General Electric's financial statements to reflect accounting numbers as if General Electric had used the same depreciation method as IBM.

ID3–7 a. Capitalizing an item simply means putting that dollar amount on the balance sheet. When WorldCom put $3.9 billion on the balance sheet, that same amount was not put on the income statement; in other words, expenses were understated by $3.9 billion since the expenditures were capitalized. b. The matching principle is the first violation. WorldCom inaccurately capitalized the expenditures in the attempt to spread the cost over future years when it should have “matched” the current expenditures with the (current) revenues derived from those expenditures. Another principle that was violated was the consistency principle, as this accounting treatment was a change from past practices by the company.

ID3–8 When Citi erroneously reported results in 2008, only to correct them in 2009, the company did not accurately reflect its financial results in the 2008 fiscal period. A reader of those financial statements—such as the U.S. Government after its bailout investment in the bank—would have seen an inaccurate picture of Citi. Correcting those mistakes in 2009 again put the reader of the financial statements in the position of not seeing the results in the correct period. Financial statements are designed to tie together, in the same time period, results and the efforts needed to get those results. The statements should “match” revenues and expenditures that are related to the activities of the fiscal period; by mismatching the 2008 costs and benefits, the bank did not accurately tell the reader of its financial statements the net result of its efforts and accomplishments in 2008—some of those results were not reported until 2009. Ideally, the financial statements should consistently attach costs in a period to the benefits that resulted in that same period. An analyst reading the financial statements of CitiBank, or a taxpayer analyzing the investment made by her government, would need to judge the results of 2008’s cost and benefits, not the results of 2008 tied to the costs of 2009.

ID3–9 a. Writedown: 246 million Euros – Recovery of prior Writedowns: 23 milliion Euros = Net Writedown: 223 million Euros b. If Unilever followed U.S. GAAP, the company would not have “written up” the inventory that had previously been written down. The writedown would have been 246 million Euros. c. IFRS is driven by a focus on market values, which allows for assets such as inventory to be both written up and written down to market value. U.S. GAAP, on the other hand, is driven by 23


conservatism and objectivity, which limits inventory adjustments to writedowns, if market values have been objectively determined to be below historic cost.

ID3–10 a. Smoothing earnings means that companies are making accounting assumptions to eliminate the fluctuations in the net income over a period of time. Many suspect that companies smooth earnings to meet targets set by Wall Street analysts; often, meeting earnings targets assures a healthy stock price. Financial services companies can manipulate their earnings by adjusting the annual “bad debt expense” (see Chapter 6 for more discussion) with higher or lower estimates of the number of loans that will be uncollectible. The charge for bad debts is one of the largest for financial services companies and is subject to management discretion for estimates. A financial services company could manipulate its net income by adjusting its annual bad debt expense to achieve its earnings goals for the year. b. Some analysts argue that smoothing earnings is an accurate way for a financial services company to show its net income over a long period of time, due to the extreme fluctuations in loan losses (due to macroeconomic conditions, for example). The financial statements could be seen as more “conservative” if the company takes a large charge for bad debts today, in anticipation of future losses. However, the counterargument is that future earnings, therefore, would be inaccurately shown as higher when the financial services company does not have to take the charge, as it has already been booked.

ID3-11 The economic entity assumption states that individual entities can be shown as distinct from their owners and all other entities and that financial results can be measured for an entity separate from all others. By requiring consolidation, FASB is attempting to force companies to accurately portray their financial condition by including all resources and obligations that belong together. In Enron’s case, shareholders and creditors could not accurately gauge the financial condition of the company because certain assets, and more importantly, certain liabilities that were the responsibility of Enron were not directly shown on the Enron balance sheet. If Enron had consolidated the entities that carried the debt, shareholders and creditors (as well as government regulators) could have better understood the financial condition and the related risks of the company. That increased level of understanding would have allowed the financial markets to better judge and price the risks associated with the company.

ID3-12 a.

KeyCorp valued its marketable securities and other equity investments at fair market value. In the case of equity and real estate investments that were not able to be valued at FMV, Key made estimates of value based on present value.

b.

Investments in privately held firms, by definition not subject to valuation on public markets, can be difficult to objectively value in terms of fair market value. Mainly subjective considerations are entered into the valuation analysis. Privately held firms do not actively turn over their ownership and therefore do not have external, objective valuations. Analysts looking into KeyCorp’s assets would need to review the subjective assumptions made when the valuations were placed on the balance sheet.

c.

If the estimation of fair value used present value as its driving determinant, then the assumptions regarding time and interest rates would need to be reviewed. By definition, present value requires the future cash flows to be determined. If those future cash flows are projected based on management assumptions, a responsible analyst would review the assumptions that underlie the valuation process.

24


ID3–13 a. The FASB is suggesting that companies use a valuation that is like net realizable value. This is the fair value of the asset minus the cost of disposal or the fair value of the liabilities plus the cost of repurchase. This is slightly different than fair market value because the FASB is saying that the cost of disposal or repurchase be reflected in the value reported on the balance sheet. b. Reporting these securities at fair market value has the impact of adding more volatility to reported income. Previously these assets and liabilities would not change in value and so there would have been no impact on the income statement. Now the companies have these additional items that will have to be included on the income statement. c.

Forcing companies to report equity and debt securities at fair value would improve the value of the balance sheet. It would give a truer representation as to the value of assets and liabilities. This policy would also increase the volatility of the reported net income of the company. Profits and losses unrelated to the operations of the company could be somewhat confusing to the users of the financial statements. Overall, the benefit of having the balance sheet reflect more current values probably outweighs any potential perceived negative impacts on the income statement. d. Under U.S. GAAP, the principle of objectivity ensures that fair market values are not used unless they can be objectively determined; also, the concept of conservatism dictates that fair market value is used only if it is below historical cost. IFRS, conversely, allows adjustments to the balance sheet values of assets for changes in market value, and these adjustments can be upward or downward.

ID3–14 Economic entity assumption: This assumption states that the financial statements report financial information about an identifiable and measurable entity that is separate and distinct from its owners and all other entities. The financial statements of Google are for Google, Inc. and subsidiaries. Thus, the identifiable and measurable entity is Google and its subsidiaries. Fiscal period assumption: This assumption states that the operating life of an economic entity can be divided into arbitrary time periods. Google has broken its operating life into fiscal years, where its fiscal year is defined as the 12 months ending on December 31. The fiscal years are reflected on the company's consolidated statements of earnings, cash flows, and stockholders' equity and on its consolidated balance sheets. Going concern assumption: This assumption states that a company's life extends beyond the current period. Google reports assets and liabilities on its consolidated balance sheets. Because assets are defined in terms of expected future benefits and liabilities are defined in terms of probable obligations that will be settled in the future, it is necessary to assume that Google will exist beyond the current period to derive the benefits from the assets or to pay its obligations. Stable dollar assumption: This assumption states that the U.S. dollar is used to measure economic events and that the purchasing power of a dollar is constant across time. The absence of any adjustments for inflation in Google’s financial statements is an example of the application of this assumption. Principle of objectivity: This principle states that financial accounting information must be verifiable and reliable. An excellent example of Google applying this principle is the company’s recording of property, plant and equipment at historic cost.

25


Matching principle: This principle states that the efforts of a given period should be matched against the benefits that result from them. Examples of Google applying the matching principle include recording expenditures, such as prepaid expenses, as assets because the expenditures have not yet helped generate a benefit; recognizing depreciation and amortization as the company consumes a portion of the fixed assets; and recognizing operating expenses for the products that were sold. Revenue recognition principle: This principle provides guidelines for when it is acceptable for a company to recognize revenue. As disclosed in the first footnote, Google recognizes revenue upon shipment or delivery of products and services to the customer. Principle of consistency: This principle states that companies should choose a set of accounting methods and procedures and use them from one period to the next. An example of Google applying the principle of consistency is using similar average useful lives of its fixed assets when calculating annual depreciation. Materiality: Materiality states that only those transactions dealing with dollar amounts large enough to make a difference to financial statement users need to be accounted for in a manner consistent with the principles of financial accounting. It is difficult to identify immaterial events in a company's financial statements, because by their very nature, immaterial events would not be disclosed in a way that would make them very noticeable. However, one example of Google applying the materiality exception in reporting items is lumping together various other assets. Conservatism: Conservatism states that, when in doubt about how to record or report an event, a company should understate assets, overstate liabilities, delay recognizing revenues or gains, and accelerate recognizing expenses or losses. According to the first footnote, Google values its inventory at the lower of cost or market, an application of the conservatism principle of U.S. GAAP. Fair Valuation: The first footnote indicates that Google values its financial assets and liabilities (including assets that include cash and cash equivalents, marketable securities and foreign currency and interest rate derivative contracts) at fair value.

26


122

Chapter 4

CHAPTER 4 THE MECHANICS OF FINANCIAL ACCOUNTING BRIEF EXERCISES

BE4–1 Transaction

Assets

=

Liabilities

+ Stockholders’ Equity

Paid $11,027 to purchase property, plant and equip.

+ 11,027 - 11,027

Issued common stock for $2,111

+2,111

=

+2,111

Recorded depreciation of $6,357

-6,357

=

-6,357

Net effect

-4,246 =

-4,246

b. The transaction to purchase property, plant and equipment does not appear to affect the accounting equation. This is because both sides of the transaction affect the asset side of the balance sheet. Intel pays cash for p,p,&e; this reduces cash and increases fixed assets. All of the other transactions affect both sides of the balance sheet.

BE4–2 Transaction Repaid $57 of long-term debt

Assets - 57

= Liabilities + Stockholders’ Equity

=

- 57

Paid cash dividends of $1,449 - 1,449 =

- 1,449

Repurchased common stock -629 for $379

=

-629

Net effect

-2,135 =

- 57

-2,078

b. Both transactions reduce assets and equity and can be viewed as alternate ways to return cash to shareholders, by either paying cash in the form of dividends or paying cash in return for shares.


BE4–3 Transaction

Assets

= Liabilities + Stockholders’ Equity

Recognized revenues of $6,325, in exchange for accounts receivable.

+6,325

=

+6,325

Paid $1,264 for sales and marketing.

-1,264

=

-1,264

=

+167

Issued common stock for $167 Purchased marketable securities for $2,503

+167

-2,503 +2,503 =

Net effect

+5,228

_____ =

+5,228

b. The first and second transactions would be reflected on the income statement. Yahoo would show $6,325 of revenue on the income statement. Yahoo would also show sales and marketing expense of $1,264. The last three transactions would be reflected on the statement of cash flows. The first transaction does not involve cash and therefore would not directly show on the cash flow statement. Paying cash for marketing expenses would be shown in the operating section; the third transaction would be in the financing section; and the fourth transaction would be in the operating section.

EXERCISES E4–1 Assets = (1) + 30,000 (2) – 20,000 + 20,000 (3) + 9,000 (4) + 8,000 (5) – 5,500 (6) – 500 Total 41,000 Note:

Liabilities

+

Stockholders' Equity + 30,000

+9,000 + – – 9,000

8,000 5,500 500 32,000

Transactions (4), (5), and (6) are initially recorded in temporary accounts and are closed into the Retained Earnings account, which is part of stockholders' equity.


E4–2

Cash

Assets Accounts + Receivable +

Land

= Liabilities + Stockholders' Equity Notes Contributed Retained = Payable + Capital + Earnings

(1) (2) (3) (4) (5) (6) Total

+ 30,000 – 20,000 + 9,000

+30,000

Note:

Transactions (4), (5), and (6) are initially recorded in temporary accounts and are closed into the Retained Earnings account, which is part of stockholders' equity.

+20,000 + 9,000 +8,000

– –

5,500 500 13,000

+ – –

8,000

20,000

9,000

30,000

8,000 5,500 500 2,000

E4–3 X Company Income Statement For the Year Ended Revenues ............................................................................................................. Operating expenses ............................................................................................. Net income ..........................................................................................................

$ 8,000 5,500 $ 2,500

X Company Statement of Stockholders’ Equity For the Year Ended

Beginning balance Net income Dividends Owner contribution Ending balance

Contributed Capital $ 0

30,000 $ 30,000

Retained Earnings $ 0 2,500 (500) _______ $ 2,000

X Company Balance Sheet As of Assets

Liabilities and Stockholders' Equity

Cash ........................................ Accounts receivable.................. Land ........................................

$ 13,000 8,000 20,000

Total assets ...............................

$41,000

Notes payable ............................. $ 9,000 Contributed capital ..................... 30,000 Retained earnings ....................... 2,000 Total liabilities and stockholders' equity .............. $ 41,000


E4–3

Concluded X Company Statement of Cash Flows For the Year Ended Cash flows from operating activities: Cash payments for expenses .............................................

$ (5,500)

Cash flows from investing activities: Purchase of land ................................................................

(20,000)

Cash flows from financing activities: Cash contributions from owners ....................................... Proceeds from bank loan .................................................. Payments of cash dividend................................................ Net cash flow from financing activities ........................ Net increase in cash................................................................

$ 30,000 9,000 (500)

Beginning cash balance .......................................................... Ending cash balance ...............................................................

38,500 $ 13,000 0 $ 13,000

E4–4 Assets +10,000 + 8,000 – 3,000 +12,000 – 2,000 (5) – 400 (6) + 7,000 – 6,000 Total 25,600 (1) (2) (3) (4)

=

Liabilities

+ 3,000 + 10,000

+

Stockholders' Equity + 10,000 + 8,000 – 6,000

– + 13,000

400 1,000 12,600

Cathedral Enterprises Income Statement For the Year Ended Fees earned ............................................................................................................. Expenses .................................................................................................................. Gain on sale of land ................................................................................................. Net income ..............................................................................................................

$ 8,000 (6,000) 1,000 $ 3,000


E4–4

Concluded Cathedral Enterprises Statement of Stockholders’ Equity For the Year Ended

Beginning balance Net income Dividends Stockholder contribution Ending balance

Contributed Capital $ 0

10,000 $ 10,000

Retained Earnings $ 0 3,000 (400) ______ $ 2,600

Cathedral Enterprises Balance Sheet As of Assets

Liabilities and Stockholders' Equity

Cash ........................................ Receivables ............................... Land ........................................

$ 17,600 2,000 6,000

Total assets ...............................

$25,600

Misc. payable .............................. $ 3,000 Long-term note ........................... 10,000 Contributed capital ..................... 10,000 Retained earnings ....................... 2,600 Total liabilities and stockholders' equity .............. $25,600

Cathedral Enterprises Statement of Cash Flows For the Year Ended Cash flows from operating activities: Cash collected from customers ......................................... Cash paid for expenses ...................................................... Net cash increase from operating activities ................. Cash flows from investing activities: Proceeds from sale of land ................................................ Cash paid for land .............................................................. Net cash increase from investing activities .................. Cash flows from financing activities: Contributions from stockholders ...................................... Dividends paid to stockholders ......................................... Net cash increase from financing activities.................. Increase in cash ......................................................................

$

6,000 (3,000) $

$

3,000

7,000 (2,000) 5,000

$ 10,000 (400) 9,600 $ 17,600


Beginning cash balance .......................................................... Ending cash balance ...............................................................

0 $ 17,600

Note: Even though $12,000 worth of land was purchased only $2,000 is shown on this statement because the balance ($10,000) was paid for with a promise to pay cash in the future (loan). So only $2,000 of cash was used this year.

E4–5 (1) This financial event does not have accounting significance. Entries are made to record financial events that affect the company's current financial condition. In this case, the new contract will affect the company's future financial condition by affecting the dollar value of future events as the new contract is implemented. Simply signing the contract does not affect the company's current financial position. (2) This financial event does have accounting significance. The receipt of cash in exchange for issuing debt affects the company's current financial position by increasing both the amount of cash the company has and the obligations the company has to other entities. Thus, an entry is necessary, and the entry would be: Cash (+A) ........................................................................................... Bonds Payable (+L) ..................................................................... Issued bonds.

200,000 200,000

(3) This event does not have accounting significance. The retirement of an official does not influence the company's current financial position. (4) This financial event does have accounting significance. Receiving cash from a customer would change the company's current financial position. The entry would be: Cash (+A) ........................................................................................... Accounts Receivable (–A) ........................................................... Collected cash from customers.

10,000 10,000

(5) This financial event does have accounting significance. Payment of a liability will change a company's current financial position by decreasing both the amount of cash the company has and the company's obligations to other entities. The entry would be: Accrued Interest Payable (–L) ........................................................... Cash (–A)..................................................................................... Paid interest previously incurred.

1,000 1,000

(6) This financial event does not have accounting significance. Long-lived assets are reported at original cost less accumulated depreciation. Increases in market value above the reported amounts are not reported because market values on long-lived assets are not objective (i.e., are not reliable). (7) This financial event does have accounting significance. The purchase of an insurance policy represents a change in the company's financial position because the company has less cash and because the company has acquired the benefit of insurance coverage. However, the value of the policy has no influence on the company. The appropriate entry would be: Prepaid Insurance (+A) ...................................................................... Cash (–A).....................................................................................

1,500 1,500


Purchased insurance coverage. (8) This financial event does not have accounting significance. Simply placing an order does not affect a company's financial position. That is, the company has not experienced a change in the amount of cash it has, the amount it owes other entities, and so forth. The company's position does not change until it legally owns the goods.

E4–6 Account

Financial Statement

Flight Equipment Balance Sheet Passenger Revenue Income Statement Retained Earnings Balance Sheet Notes Payable Balance Sheet Interest Expense Income Statement Accounts Receivable Balance Sheet Prepaid Expenses Balance Sheet Accounts Payable Balance Sheet Common Stock Balance Sheet Fuel Expense Income Statement Other Revenues Income Statement Short-Term Investments Balance Sheet Depreciation Expense Income Statement Landing Fees Income Statement

Accounting Equation Assets Owners’ Equity Owners’ Equity Liabilities Owners’ Equity Assets Assets Liabilities Owners’ Equity Owners’ Equity Owners’ Equity Assets Owners’ Equity Owners’ Equity

E4–7 Bristol-Myers Squibb Income Statement For the Year Ended December 31, 2012 Net Sales ................................................................................. Expenses: Cost of goods sold ............................................................. Selling and adm. Expense .................................................. Advertising and product expense...................................... Research and dev. expense ............................................... Impairment Expense ......................................................... Other expenses ................................................................. Total expenses.............................................................. Net income .............................................................................

$ 17,621 $

4,610 4,220 797 3,904 1,830 (241) $

15,120 2,501

Accounts payable ........................ $ Accrued payables ........................

2,202 2,573

Bristol-Myers Squibb Balance Sheet As of December 31, 2012 Assets Cash and equivalents ................ Marketable securities ...............

Liabilities and Stockholders' Equity $

1,656 1,173


Accounts receivable.................. Other current assets ................. Current Assets ..........................

3,083 Short-term borrowings ............... 3,609 Other current liabilities............... 9,521 Current Liabilities ...........................

826 2,678 8,279

Property, plant and equipment Other noncurrent assets...........

5,333Long-term liabilities ................................. 21,043 Stockholders’ equity ...................

13,980 13,638

Total assets ............................... E4–7 Concluded

$ 35,897

Total liabilities and stockholders’ equity

$ 35,897

Bristol-Myers Squibb Statement of Cash Flows For the Year Ended December 31, 2012 Cash flows from operating activities: Net income ......................................................... Adjustments: Total adjustments.......................................... Net cash increase (decrease) due to operating activities .......................

$

2,501 4,440 $ 6,941

Cash flows from investing activities: Net cash increase (decrease) due to investing activities ....................................

(6,727)

Cash flows from financing activities: Net cash increase (decrease) due to financing activities.................................... Change in cash balance ...........................................

(4,333) $ (4,119)

Beginning cash balance ........................................... Ending cash balance ................................................

5,775 $ 1,656

The company appears to be in very good financial condition. The company is very profitable with a 14.2% net income margin ($2,501/$17,621). The company has an extremely strong balance sheet with very good liquidity; working capital is $1,242 ($9,521 - $8,279).

E4–8 a.

Ending cash

= = =

Beginning cash + Cash inflows – Cash outflows $9,000 + $133,500 – $99,500 $43,000

Note: Since Cash is an asset, cash inflows are recorded on the debit, or left-hand side of the T account, and cash outflows are recorded on the credit, or right-hand side of the T account.


E4-8 Concluded b. Miller Manufacturing Statement of Cash Flows For the Year Ended December 31, 2015 Cash flows from operating activities: Cash collections from customers ...................................... Payment of salaries ........................................................... Payment of miscellaneous expenses................................. Payment of rent................................................................. Payment of interest ........................................................... Net cash increase from operating activities ................. Cash flows from investing activities: Proceeds from sale of land ................................................ Purchase of long-term investments .................................. Purchase of equipment ..................................................... Net cash decrease from investing activities ................. Cash flows from financing activities: Proceeds from issuance of common stock ........................ Proceeds from borrowing ................................................. Payment of bank loan........................................................ Payment of dividends ........................................................ Net cash increase from financing activities.................. Increase (decrease) in cash balance .......................................

$ 95,000 (26,500) (13,000) (7,000) (3,000) $ 45,500 $

7,500 (10,000) (24,000) (26,500)

$ 15,000 16,000 (12,000) (4,000) 15,000 $ 34,000

Beginning cash balance .......................................................... Ending cash balance ...............................................................

9,000 $ 43,000

E4–9 a. (1) Cash (+A)..................................................................................... Common Stock (+SE) ............................................................. Issued common stock.

15,000

(2) Cash (+A)..................................................................................... Fees Earned (R, +SE) .............................................................. Sold services for cash.

4,000

(3) Wage Expense (E, –SE) ............................................................... Cash (–A) ...............................................................................

1,600

15,000

4,000

1,600


Incurred and paid wages.

(4) Investment in Land (+A) ............................................................. Cash (–A) ............................................................................... Purchased land as an investment.

9,000

(5) Dividends (–SE) ........................................................................... Cash (–A) ............................................................................... Declared and paid dividend.

2,000

E4–9

9,000

2,000

Continued

(6) Cash (+A)..................................................................................... Land (–A) ............................................................................... Gain on Sale of Land (Ga, +SE) .............................................. Sold land.

3,500

(7) Interest Expense (E, –SE) ............................................................ Note Payable (+L) ....................................................................... Cash (–A) ............................................................................... Made principal and interest payment.

600 900

(8) Miscellaneous Expenses (E, –SE) ................................................ Cash (–A) ............................................................................... Incurred and paid miscellaneous expenses.

1,800

3,000 500

1,500

1,800

b. B.B. (1) (2) (6)

E.B.

Cash 5,000 15,000 (3) 4,000 (4) 3,500 (5) (7) (8)

1,600 9,000 2,000 1,500 1,800

11,600

c. Small and Associates Statement of Cash Flows For the Month Ended January 31, 2015 Cash flows from operating activities: Collections from customers............................................... Payment of wages ............................................................. Payment of interest ........................................................... Payment of miscellaneous expenses.................................

$ 4,000 (1,600) (600) (1,800)


Net cash from operating activities ............................... Cash flows from investing activities: Proceeds from sale of land ................................................ Purchase of land ................................................................ Net cash decrease from investing activities ................. Cash flows from financing activities: Proceeds from issuance of stock ....................................... Repayment of note............................................................ Dividend payment ............................................................. Net cash increase from financing activities.................. Increase in cash balance ......................................................... Beginning cash balance .......................................................... Ending cash balance ...............................................................

$

0

$ 3,500 (9,000) (5,500) $15,000 (900) (2,000) $

12,100 6,600

5,000 $ 11,600


E4–10 a. Cash (1) (2) (3) (4) (5) (6) (7) (8) Total

+ 12,000 + 5,000 – 10,000 – 5,000 + 10,000 – 4,000 + 2,800 – 2,200 8,600

Assets Accounts + Receivable +

Land

= Liabilities + Stockholders' Equity Notes Contributed Retained = Payable + Capital + Earnings +12,000 + 5,000

+ 10,000 +4,000 – 4,000

3,000 7,000

5,000

12,000

– 5,000 + 14,000 – 4,000 – 200 – 2,200 2,600

Ed's Lawn Service Income Statement For the Year Ended December 31, 2015 Revenue .................................................................................. Rent expense .......................................................................... Miscellaneous expense........................................................... Loss on sale of land................................................................. Net income .............................................................................

$14,000 (5,000) (4,000) (200) $ 4,800

Ed's Lawn Service Statement of Stockholders’ Equity For the Year Ended December 31, 2015 Contributed Capital Beginning balance, January 1, 2015 $ 0 Net income Dividends Stockholder contribution 12,000 Ending balance, December 31, 2015 $ 12,000

Retained Earnings $ 0 4,800 (2,200) ______ $ 2,600

Ed's Lawn Service Balance Sheet As of December 31, 2015 Assets Cash ........................................ Accounts receivable.................. Land ........................................ Total assets ...............................

Liabilities and Stockholders' Equity $

8,600 4,000 7,000 $19,600

Notes payable ............................. $ 5,000 Contributed capital ..................... 12,000 Retained earnings ....................... 2,600 Total liabilities and stockholders' equity .............. $ 19,600


E4–10

Continued Ed's Lawn Service Statement of Cash Flows For the Year Ended December 31, 2015 Cash flows from operating activities: Cash collected from customers ......................................... Rent payments on lawn equipment .................................. Payment of miscellaneous expenses................................. Net cash increase from operating activities ................. Cash flows from investing activities: Proceeds from sale of land ................................................ Cash paid for land .............................................................. Net cash decrease from investing activities ................. Cash flows from financing activities: Stockholder contributions ................................................. Proceeds from bank loan .................................................. Dividend payments............................................................ Net cash increase from financing activities.................. Increase in cash ......................................................................

$ 10,000 (5,000) (4,000) $ $

1,000

2,800 (10,000) (7,200)

$ 12,000 5,000 (2,200)

Beginning cash balance .......................................................... Ending cash balance ...............................................................

$

14,800 8,600

$

0 8,600

b. (1) Cash (+A)..................................................................................... Contributed Capital (+SE) ...................................................... Collected cash from stockholders.

12,000

(2) Cash (+A)..................................................................................... Notes Payable (+L) ................................................................. Borrowed cash from bank.

5,000

(3) Land (+A)..................................................................................... Cash (–A) ............................................................................... Purchased land.

10,000

(4) Rent Expense (E, –SE) ................................................................. Cash (–A) ............................................................................... Incurred and paid rent expense.

5,000

(5) Cash (+A)..................................................................................... Accounts Receivable (+A) ........................................................... Fees Earned (R, +SE) .............................................................. Rendered services.

10,000 4,000

(6) Miscellaneous Expenses (E, –SE) ................................................ Cash (–A) ............................................................................... Incurred and paid miscellaneous expenses.

4,000

12,000

5,000

10,000

5,000

14,000

4,000


E4–10

Continued

(7) Cash (+A)..................................................................................... Loss on Sale of Land (Lo, –SE) ..................................................... Land (–A) ............................................................................... Sold land.

2,800 200

(8) Dividends (–SE) ........................................................................... Cash (–A) ............................................................................... Declared and paid cash dividend.

2,200

3,000

2,200

Cash B. B. 0 (1) 12,000 (3) 10,000 (2) 5,000 (4) 5,000 (5) 10,000 (6) 4,000 (7) 2,800 (8) 2,200 _____________________________________ E. B. 8,600

Accounts Receivable B. B. 0 (5) 4,000 _________________________________ E. B. 4,000

Land B. B. 0 (3) 10,000 (7) 3,000 _____________________________________ E. B. 7,000

Notes Payable B. B. 0 (2) 5,000 _________________________________ E. B. 5,000

Contributed Capital B. B. 0 (1) 12,000 _____________________________________ E. B. 12,000

Retained Earnings* B. B.

_________________________________ E. B. 2,600

Dividends B. B. 0 (8) 2,200 _____________________________________ E. B. 2,200

Fees Earned B. B. 0 (5) 14,000 _________________________________ E. B. 14,000

Rent Expense B. B. 0 (4) 5,000 _____________________________________ E. B. 5,000

Miscellaneous Expenses B. B. 0 (6) 4,000 _________________________________ E. B. 4,000

Loss on Sale of Land B.B. 0 (7) 200 _____________________________________ E.B. 200 *The Ending Balance in the Retained Earnings account is derived by the following formula:

0


Beginning Balance + Revenues – Expenses – Dividends. For a check, refer to the statement of retained earnings.


E4–10

Concluded Ed's Lawn Service Income Statement For the Year Ended December 31, 2015 Revenue ............................................................................................................ Rent expense .................................................................................................... Miscellaneous expense..................................................................................... Loss on sale of land........................................................................................... Net income .......................................................................................................

$ 14,000 (5,000) (4,000) (200) $ 4,800

Ed's Lawn Service Statement of Stockholders’ Equity For the Year Ended December 31, 2015 Contributed Retained Capital Earnings Beginning balance, January 1, 2015 $ 0 $ 0 Net income 4,800 Dividends (2,200) Stockholder contribution 12,000 _____ Ending balance, December 31, 2015 $ 12,000 $ 2,600 Ed's Lawn Service Balance Sheet As of December 31, 2015 Assets Cash ........................................ Accounts receivable.................. Land ........................................

Liabilities and Stockholders' Equity $ 8,600 4,000 7,000

Notes payable ............................. Contributed capital ..................... Retained earnings ....................... Total liabilities and Total assets ............................... $19,600 stockholders' equity .............. Ed's Lawn Service Statement of Cash Flows For the Year Ended December 31, 2015 Cash flows from operating activities: Cash collected from customers ......................................... Rent payments on lawn equipment .................................. Payment of miscellaneous expenses................................. Net cash increase from operating activities ................. Cash flows from investing activities: Proceeds from sale of land ................................................ Cash paid for land .............................................................. Net cash decrease from investing activities ................. Cash flows from financing activities: Stockholder contributions ................................................. Proceeds from bank loan .................................................. Dividend payments............................................................ Net cash increase from financing activities.................. Increase in cash ...................................................................... Beginning cash balance ..........................................................

$ 5,000 12,000 2,600 $19,600

$ 10,000 (5,000) (4,000) $ $

1,000

2,800 (10,000) (7,200)

$ 12,000 5,000 (2,200) $

14,800 8,600 0


Ending cash balance ...............................................................

$

8,600


E4–11 a. Ending cash balance

=

$8,000 + $109,500 – $90,000

=

$27,500

b. Holcomb Manufacturing Statement of Cash Flows For the Year Ended December 31, 2015 Cash flows from operating activities: Cash collections from customers ...................................... Payments for inventory ..................................................... Payment of wages ............................................................. Payment of administrative expenses ................................ Payment of interest ........................................................... Net cash increase due to operating activities ..............

$ 74,000 (34,000) (16,000) (12,000) (3,000)

Cash flows from investing activities: Proceeds from long-term investments ............................. Purchase of equipment ..................................................... Net cash increase due to investing activities ...............

$ 12,500 (11,000)

Cash flows from financing activities: Proceeds from issuance of common stock ........................ Proceeds from borrowing ................................................. Repayment of bank loan ................................................... Payment of dividends ........................................................ Net cash increase due to financing activities ............... Increase in cash balance .........................................................

E4–12 a. (1) (2) (3) (4) (5)

The entry is to record rent incurred but not yet paid. The entry is to record the expiration of a previously purchased insurance policy. The entry is to record the expiration of a portion of a fixed asset cost. The entry is to record interest revenue earned but not yet collected. The entry is to record the earning of a deferred revenue.

b. (1) (2) (3) (4) (5)

Accrual adjusting entry Cost expiration adjusting entry Cost expiration adjusting entry Accrual adjusting entry Accrual adjusting entry

E4–13 Accrual adjusting entry Operating cash flow Financing cash flow Cost expiration adjusting entry Cost expiration adjusting entry Operating cash flow

(7) (8) (9) (10) (11) (12)

9,000

$

1,500

$ 14,000 9,000 (10,000) (4,000)

Beginning cash balance .......................................................... Ending cash balance ...............................................................

(1) (2) (3) (4) (5) (6)

$

Investing cash flow Cost expiration adjusting entry Operating cash flow Cost expiration adjusting entry Operating cash flow Cost expiration adjusting entry

9,000 $ 19,500 8,000 $ 27,500


E4–14 a. 12/31/15

Wage Expense (E, –SE) ................................................ Wages Payable (+L) ................................................ Incurred, but did not pay, wages. _________________ * $42,000 = $70,000  (3 days in December ÷ 5 days total)

b. 1/2/16

Wage Expense (E, –SE) ................................................ Wages Payable (–L) ..................................................... Cash (–A) ................................................................ Paid wages.

42,000* 42,000

28,000 42,000 70,000

c. Wage expense Cash outflow associated with wages

2015 $42,000 0

2016 $28,000 70,000

Total $70,000 70,000

d. The purpose of the adjusting journal entry on December 31, 2015 is to recognize an economic event that has not yet been captured by an exchange transaction. The economic event is that the Hurst Corporation consumed the benefits of its employees' labor, and in doing so, has become obligated to its employees. Hurst Corporation will not fulfill its obligation to its employees until the subsequent period when it actually pays the employees their wages. Consequently, an accrual adjusting entry is required on December 31 to record this economic event in the correct accounting period.

E4–15 a. 2012 Book value, 1/1/12 ............................................................................ 2012 Depreciation expense............................................................... Less: Accumulated depreciation, 12/31/12 ...................................... Book value, 12/31/12 ........................................................................

$856,000 $214,000 214,000 $ 642,000


E4–15

Concluded

2013 Book value, 1/1/12 ........................................................................ 2012 Depreciation expense........................................................... 2013 Depreciation expense........................................................... Less: Accumulated depreciation, 12/31/13 .................................. Book value, 12/31/13 ....................................................................

$214,000 $214,000

2014 Book value, 1/1/12 ........................................................................ 2012 Depreciation expense........................................................... 2013 Depreciation expense........................................................... 2014 Depreciation expense........................................................... Less: Accumulated depreciation, 12/31/14 .................................. Book value, 12/31/14 ....................................................................

$214,000 $214,000 $214,000

2015 Book value, 1/1/12 ........................................................................ 2012 Depreciation expense........................................................... 2013 Depreciation expense........................................................... 2014 Depreciation expense........................................................... 2015 Depreciation expense........................................................... Less: Accumulated depreciation, 12/31/15 .................................. Book value, 12/31/15 .................................................................... b.

c.

2012 Depreciation expense $ 214,000 Cash outflow associated with the purchase of the $ 856,000 equipment

$856,000

428,000 $ 428,000

$856,000

642,000 $ 214,000

$ 856,000 $214,000 $214,000 $214,000 $214,000 856,000 $ 0

2013 2014 2015 Total $214,000 $214,000 $214,000 $856,000 0

0

0

$856,000

The purpose of the adjusting journal entry at the end of each period is to recognize the economic event of the portion of the fixed asset cost that expired during that year. Specifically, the purpose of the adjusting journal entries is to allocate the cost of the equipment to the periods that benefited from the equipment. Since the equipment has a useful life of four years, it is assumed that it will help generate revenues for four years. The cost of a fixed asset should be matched with the periods in which the fixed asset helps generate revenues.

E4–16 a. With cash-basis accounting, cash inflows and outflows are the critical events. A company will recognize revenue when it has cash inflows, and the company will recognize expenses when it has cash outflows. So in this case, Washington Forest Products would recognize the following expenses under cash-basis accounting. Insurance expense Supplies expense Rent expense

$29,000 27,000 8,000

With accrual-basis accounting, inflows and outflows of assets and liabilities are the critical events. That is, a company will recognize revenue when it has an inflow of assets or an outflow of liabilities


associated with operating activities. Similarly, the company will recognize expenses when it has an outflow of assets or an inflow of liabilities associated with operating activities.


E4–16

Concluded

Consider the revenues being generated when the company is entitled to cash. The company could collect the cash at the exact same time it becomes entitled to the cash (which is an asset account), the company could expect to collect the cash after it has become entitled to the cash (which would give rise to a receivable, an asset account) or the company could become entitled to cash after it had already collected the cash (which would result in the company reducing its unearned revenue, a liability account). Just as with revenues, a company can consume benefits at three different points in time relative to the cash outflow. The company would consume the benefit at the same time it disburses cash (which is an asset account), the company could intend to disburse the cash after consuming the benefit (which would give rise to a payable, a liability account), or the company could consume a benefit for which it has already disbursed the cash (which would result in the company consuming a prepaid expense, an asset account). Because accrual-basis accounting is not based on the inflow and outflow of one asset (i.e., cash), accrual-basis accounting provides a much broader measure of revenues and expenses than provided by cash-basis accounting. Thus, in the case of Washington Forest Products, the difference between its expenses under cash-basis accounting and under accrual-basis accounting is due to expenses being defined differently under the two approaches. b. Insurance: Ending balance

= = =

Beginning balance + Insurance purchased – Insurance expense $ 0 + $29,000 – $20,000 $9,000

Since the company acquired more insurance than it used during 2014, the company expects to receive future benefits from the remaining insurance. Consequently, the company has an asset, and the appropriate account title is Prepaid Insurance. Supplies: Ending balance

= = =

Beginning balance + Supplies purchased – Supplies expense $0 + $27,000 – $11,000 $16,000

Since the company acquired more supplies than it used during 2014, the company expects to receive future benefits from the remaining supplies. Consequently, the company has an asset, and the appropriate account title is Supplies Inventory. Rent: Ending balance

= = =

Beginning balance + Cash disbursed for rent – Rent expense $0 + $8,000 – $14,000 $(6,000)

Since the company incurred more expense than it disbursed in cash for rent, the company expects to fulfill the remaining obligations in the future. Consequently, the company has a liability, and the appropriate account title would be Rent Payable.


E4–17 a. Lauren Retailing Income Statement For the Month Ended July 31 Sales ....................................................................................................................... Cost of goods sold ................................................................................................... Accrued expenses .................................................................................................... Net income ..............................................................................................................

$ 8,000 (3,700) (2,500) $ 1,800

Lauren Retailing Statement of Cash Flows from Operating Activities For the Month Ended July 31 Cash flows from operating activities: Cash collections from customers ....................................................................... Payment for inventory ....................................................................................... Net cash increase due to operating activities ...............................................

$ 7,000 (2,800) $ 4,200

b. Cash flows from operating activities are based on the inflow and outflow of cash. Net income is based on the accrual method of accounting. Under accrual accounting, revenue is recognized when it is earned, and expenses are recognized when incurred. This means that cash flows do not trigger the recognition of revenues and expenses. Rather, the inflow or outflow of operating assets, which includes more than just cash, triggers revenues and expenses. Consequently, cash flows from operating activities and net income will be equal only by coincidence. The difference between the cash flows from operating activities and net income can be explained as follows: Net income ............................................................................. Adjustments: Collections on accounts receivable ................................... Excess of cost of goods sold over cash disbursed for inventory ................................................ Expenses incurred but cash not disbursed ........................ Sales generated but cash not collected............................. Total adjustments......................................................... Net cash increase due to operating activities ........................

$ 1,800 $

2,000 900 2,500 (3,000) 2,400 $ 4,200


E4–18 a. (1) Cash (+A)..................................................................................... Common Stock (+SE) ............................................................. Issued common stock.

24,000

(2) Cash (+A)..................................................................................... Accounts Receivable (–A) ...................................................... Collected cash from customers on account.

3,900

(3) Wage Expense (E, –SE) ............................................................... Cash (–A) ............................................................................... Incurred and paid wages.

1,530

(4) Investment in Land (+A) ............................................................. Cash (–A) ............................................................................... Purchased land as an investment.

12,000

(5) Dividends (–SE) ........................................................................... Cash (–A) ............................................................................... Declared and paid dividend.

6,000

(6) Cash (+A)..................................................................................... Equipment (–A) ..................................................................... Gain on Sale of Equipment (Ga, +SE)..................................... Sold equipment.

7,000

(7) Interest Expense (E, –SE) ............................................................ Note Payable (–L) ....................................................................... Cash (–A) ............................................................................... Made interest and principal payment.

1,100 900

(8) Miscellaneous Expense (E, –SE).................................................. Cash (–A) ............................................................................... Incurred and paid miscellaneous expenses.

5,000

24,000

3,900

1,530

12,000

6,000

5,000 2,000

2,000

b. Cash Beginning balance Issued common stock Collected from customer Sold equipment

4,000 24,000 3,900 7,000

Paid wages 1,530 Purchased land 12,000 Paid dividends 6,000 Paid interest 1,100 Repaid loan 900 Paid misc. expenses 5,000 ______________________________________________________________________ Ending balance 12,370

5,000


E4–18

Concluded

c. Rahal and Watson Statement of Cash Flows For the Month Ended January 31, 2015 Cash flows from operating activities: Cash collections from customers ...................................... Payments for wages .......................................................... Payments for interest ........................................................ Payments for miscellaneous expenses.............................. Net cash increase (decrease) due to operating activities ................................................... Cash flows from investing activities: Proceeds from sale of equipment ..................................... Purchase of land ................................................................ Net cash increase (decrease) due to investing activities .................................................... Cash flows from financing activities: Proceeds from issuance of common stock ........................ Dividend payment ............................................................. Repayment of note payable .............................................. Net cash increase (decrease) due to financing activities .................................................... Increase in cash balance .........................................................

$

3,900 (1,530) (1,100) (5,000) $ (3,730)

$

7,000 (12,000) (5,000)

$ 24,000 (6,000) (900)

$

Beginning cash balance .......................................................... Ending cash balance ...............................................................

17,100 8,370

4,000 $ 12,370

E4–19 a. Peters Company Income Statement For the Years Ended December 31, 20X1 and 20X2

Sales ....................................................................... Cost of goods sold ................................................... Rent expense ........................................................... Wage expense ......................................................... Net income ..............................................................

Year 1

Year 2

$ 24,000 (5,000) (6,000) (10,000) $ 3,000

$30,000 (5,000) (6,000) (12,000) $ 7,000


E4–19

Concluded Peters Company Statement of Cash Flows For the Year Ended December 31, 20X1 Cash flows from operating activities: Cash collections from customers ...................................... Payment for wages ............................................................ Payment for rent ............................................................... Net cash increase due to operating activities ..............

$ 20,000 (6,000) (12,000) $

Net cash flows due to investing activities .............................. Cash flows from financing activities: Proceeds from stockholders.............................................. Net cash increase (decrease) due to financing activities .................................................... Increase in cash balance .........................................................

2,000 0

$ 24,000 24,000 $ 26,000

Beginning cash balance .......................................................... Ending cash balance ...............................................................

0 $ 26,000

Peters Company Statement of Cash Flows For the Year Ended December 31, 20X2 Cash flows from operating activities: Cash collections from customers ...................................... Payment for wages ............................................................ Payment for inventory ...................................................... Net cash increase due to operating activities ..............

$ 34,000 (16,000) (10,000) $

Net cash flows due to investing activities .............................. Cash flows from financing activities: Payment to stockholders................................................... Net cash increase (decrease) due to financing activities .................................................................... Decrease in cash balance .......................................................

0 $ (34,000)* (34,000) $ (26,000)

Beginning cash balance .......................................................... Ending cash balance ............................................................... * $34,000 =

$

$26,000 Cash at beginning of Year 2 + $8,000 of Year 2 cash from activities + $0 Year 2 cash from investing activities

b. Performance Measure Net income Net cash from operating activities

Year 1 $3,000 2,000

8,000

Year 2 $7,000 8,000

Total $10,000 10,000

26,000 0 operating


E4–20 a. Prior to the 5 transactions mentioned in the Exercise, Goodyear’s current ratio and debt/equity ratios are as follows: 2012 Current Ratio

= Current Assets ÷ Current Liabilities = $8,498 ÷ $5,322 = 1.60 2012 Debt/Equity Ratio = Total Liabilities ÷ Total Stockholders’ Equity = $15,814 ÷ $1,159 = 13.64 Now, let us see how each transaction would independently affect the above two ratios. Effect of Transaction on Accounts

Effect on Current Ratio

Effect on Debt/Equity Ratio

1. Increases Inventory and Accounts Payable

will go down

will go up

2. Increases Cash and Stockholders’ Equity

will go up

will go down

3. Short-term liability decreases and Long-term liability increases

will go up

will stay the same

4. Fixed assets will increase and Long-term liability will increase

will stay the same

will go up

5. Decreases Cash and Short-term debt

will go up

will go down

b. Prior to any of the 5 independent transactions mentioned in the problem, Goodyear’s current ratio is 1.60. As mentioned in (a) above, only transaction (1) will lead to a decrease in the current ratio. Entering into Transaction (1) will lower the current ratio to 1.50 ($9,498/$6,322), causing a violation of a covenant. c.

Goodyear could pay $249 for a long-term investment and still be in compliance with the covenant. The current ratio with the investment would be ($8,498 – $249)/$5,322 = 1.55.

E4–21 a. Wages Payable as of 12/31/15

=

Wages Payable as of 12/31/14

+

Wage Expense on 2015 Inc. St.

$17,000

=

X

+

$39,000

$35,000

X

=

$13,000

Prepaid Rent = as of 12/31/14

+

Cash Paid for Rent during 2015

Rent Expense on on 2015 Inc. St.

$15,000

=

$12,000

+

X

$21,000

X

=

$24,000

=

Accounts Rec. as of 12/31/14

+

Sales Revenue on 2015 Inc. St.

Cash Collected during 2015

=

$14,000

+

$45,000

$38,000

b. Prepaid Rent as of 12/31/15

c. Accounts Rec. as of 12/31/15 X

Cash Paid for Wages during 2015


X

=

$21,000

E4–22 a.

Hamilton Direct method Cash collections from customers Cash paid for inventory Cash paid for other expenses Net cash flow from operating activities Indirect method Net income Adjustments: Depreciation expense Net cash flow from operating activities

$

$

900,000 (400,000) (200,000) 300,000

Watson $

$

900,000 (400,000) (200,000) 300,000

$ 250,000

$ 200,000

50,000 $ 300,000

100,000 $ 300,000

b. Cash flows from operating activities measures all the cash inflows and cash outflows associated with a company's operating assets and liabilities. Alternatively, net income measures the inflows and outflows of operating assets and liabilities, not just the cash associated with operating assets and liabilities. Thus, cash flows from operating activities can differ from net income due to items affecting net income that do not affect cash and due to cash flows that do not affect net income. In this particular case, the difference is due to depreciation expense. Depreciation is the systematic allocation of the cost of fixed assets. Since depreciation is simply the allocation of the asset's cost, depreciation does not affect cash flows. The indirect method more clearly shows that net income is different from cash flows because this method explicitly reconciles the difference between the two.

c.

Disagree. Many people think that depreciation expense represents a fund established to finance future acquisitions of fixed assets. If this were true, it would follow that companies using accelerated depreciation methods would have more cash available than companies that use straight-line depreciation. However, as demonstrated in part (a), depreciation expense has absolutely no effect on the cash flows from operating activities. Both companies have the same cash flows, even though each company uses a different method to compute depreciation. One must remember that depreciation is simply the allocation of the net cost of fixed assets. Cash flows associated with fixed assets arise when fixed assets are acquired or sold, not when the cost of the fixed asset is allocated to expenses.


E4–23 a. 1.

2.

3.

4.

5.

6.

7.

Cash (+A) ........................................................................................ Contributed Capital (+SE) ....................................................... Owner contributed capital.

20,000

Cash (+A) ........................................................................................ Payable to Bank (+L) ............................................................... Borrowed money from the bank.

60,000

Property, Plant, & Equipment (+A) ................................................ Cash (–A) ................................................................................. Purchased long-lived assets.

25,000

Inventory (+A) Cash (–A) ................................................................................. Accounts Payable (+L) ............................................................. Purchased inventory.

40,000

Cash (+A) ........................................................................................ Accounts Receivable (+A) .............................................................. Sales (R, +SE) ........................................................................... Made sale.

20,000 60,000

Cost of Goods Sold (E, –SE) ............................................................ Inventory (–A) ......................................................................... Recognized cost of inventory sold.

25,000

Operating Expenses (E, –SE) .......................................................... Payable to Bank (–L) ...................................................................... Dividend (–SE)................................................................................ Cash (–A) ................................................................................. Made cash disbursements.

18,000 5,000 2,000

Operating Expenses (E, –SE) .......................................................... Operating Expenses Payable (+L) ............................................

15,000

Depreciation Expense (E, -SE)……………………………….. ............ 1,000 Accumulated Depreciation (-A)…………………………… ................ Incurred, but did not pay, expenses; recorded annual depreciation.

20,000

60,000

25,000

25,000 15,000

80,000

25,000

25,000

15,000

1,000


E4–23

Continued

b. Tony’s Business Income Statement For the Year Ended December 31, 2015 Sales ........................................................................................................... Cost of goods sold ..................................................................................... Operating expenses ................................................................................... Net income ................................................................................................

$ 80,000 (25,000) (34,000) $ 21,000

Tony's Business Statement of Shareholders’ Equity For the Year Ended December 31, 2015

Beginning balance:1/1/2015 Net Income Dividends Stock Issuance Ending balance: 12/31/2015

Retained Earnings $ 0 21,000 (2,000) $19,000

Contributed Capital $ 0

20,000 $20,000

Tony's Business Balance Sheet December 31, 2015 Assets Cash ....................................................... Accounts receivable .............................. Inventory ............................................... Property, plant & equipment, net.........

Total assets............................................ c. Beginning balance Owner's contribution Proceeds from bank loan Proceeds from sale

Ending balance

$

25,000 60,000 15,000 24,000

$ 124,000

Liabilities & Stockholders' Equity Accounts payable...................... $ 15,000 Operating expenses payable .. 15,000 Payable to bank ........................ 55,000 Contributed capital ................... 20,000 Retained earnings ..................... 19,000 Total liabilities and stockholders' equity ................ $ 124,000

Cash 0 20,000 Purchase of fixed assets 60,000 Purchase of inventory 20,000 Operating expenses Principal payment Dividend payment 25,000

25,000 25,000 18,000 5,000 2,000


E4–23

Continued Tony's Business Statement of Cash Flows For the Year Ended December 31, 2015

Cash from operating activities: Cash collections from sales ..................................................... Cash paid for inventory ........................................................... Cash paid for expenses ............................................................ Net cash increase (decrease) due to operating activities ..........................................................................

$ 20,000 (25,000) (18,000) $ (23,000)

Cash from investing activities: Purchase of fixed assets .......................................................... Cash from financing activities: Proceeds from owner's contribution ...................................... Proceeds from bank loan ........................................................ Principal repayment on debt ................................................... Payment of dividend ............................................................... Net cash increase (decrease) due to financing activities .......................................................................... Net increase (decrease) in cash.................................................... Beginning cash balance, January 1, 2015 ..................................... Ending cash balance, December 31, 2015....................................

(25,000)

$ 20,000 60,000 (5,000) (2,000) 73,000 $ 25,000 0 $ 25,000


E4–23

Concluded

d. Tony's Business Statement of Cash Flows For the Year Ended December 31, 2015 Cash from operating activities: Net income ................................................................ Depreciation Expense …………………………… .................. Adjustments: Increase in accounts receivable ........................... Increase in inventory............................................ Increase in accounts payable ............................... Increase in operating expense payable................ Total adjustments ........................................... Net cash increase (decrease) due to operating activities ...........................

$ 21,000 1,000 $ (60,000) (15,000) 15,000 15,000 (45,000) $ (23,000)

Cash from investing activities: Purchase of fixed assets ............................................ Cash from financing activities: Proceeds from owner's contribution ........................ Proceeds from bank loan .......................................... Principal repayment on debt ..................................... Payment of dividend ................................................. Net cash increase (decrease) due to financing activities .............................................. Net increase (decrease) in cash...................................... Beginning cash balance, January 1, 2015 ....................... Ending cash balance, December 31, 2015......................

(25,000)

$ 20,000 60,000 (5,000) (2,000) 73,000 $ 25,000 0 $ 25,000


PROBLEMS P4–1 a. (1) Equipment (+A)........................................................................... Cash (–A) ............................................................................... Purchased equipment.

150,000

(2) Wage Expense (E, –SE) ............................................................... Cash (–A) ............................................................................... Incurred and paid wages.

30,000

(3) Cash (+A)..................................................................................... Accounts Receivable (–A) ...................................................... Collected cash from customers.

15,000

(4) Cash (+A)..................................................................................... Accounts Receivable (+A) ........................................................... Fees Earned (R, +SE) .............................................................. Rendered services.

16,000 8,000

(5) Interest Expense (E, –SE) ............................................................ Note Payable (–L) ....................................................................... Cash (–A) ............................................................................... Made interest and principal payment.

10,000 40,000

(6) Advertising Expense (E, –SE)....................................................... Cash (–A) ............................................................................... Purchased advertising.

5,000

(7) Building (+A) ............................................................................... Cash (–A) ............................................................................... Long-Term Note Payable (+L) ................................................ Purchased building.

250,000

(8) Cash (+A)..................................................................................... Investments (–A) ................................................................... Gain on Sale of Investments (Ga, +SE) .................................. Sold investments.

35,000

150,000

30,000

15,000

24,000

50,000

5,000

130,000 120,000

20,000 15,000


P4–2 a. Sold services worth $28,000; received $7,000 cash and an account receivable for the balance of $21,000. Assets Increased by $28,000 and Owners’ Equity increased by $28,000 via increase in Retained Earnings. b. Purchased $6,000 worth of inventory on credit. Assets would increase and Liabilities would increase by $6,000 each. c.

Paid $2,000 cash to suppliers on previously purchased inventory. Assets and liabilities would both decrease by $2,000 each.

P4–2

Concluded

d. Purchased equipment worth $50,000 by paying $20,000 cash and signing a note payable for the balance of $30,000. Assets and Liabilities would go up by $30,000 each. e. Incurred and paid rent of $1,200 cash. Assets and owners’ equity would go down by $1,200 each. f.

Collected $5,000 cash from the customers on account of previously made credit sales. No effect on the accounting equation as assets would increase and decrease by the same amount.

g. Issued common stock for $25,000 cash. Assets and Owners’ Equity both would go up by $25,000.

P4–3 a. (1) Cash (+A)..................................................................................... Common Stock (+SE) ............................................................. Issued common stock for cash.

95,000

(2) Rent Expense (E, –SE) ................................................................. Cash (–A) ............................................................................... Incurred and paid rent ($2,600 per month for 12 months).

31,200

(3) Cash (+A)..................................................................................... Accounts Receivable (+A) ........................................................... Fees Earned (R, +SE) .............................................................. Rendered services.

65,000 125,000

(4) Land (+A)..................................................................................... Cash (–A) ............................................................................... Purchased land.

32,000

(5) Cash (+A)..................................................................................... Long-Term Note Payable (+L) ................................................ Borrowed cash.

75,000

(6) Salary Expense (E, –SE) ............................................................... Cash (–A) ............................................................................... Incurred and paid salaries.

80,000

95,000

31,200

190,000

32,000

75,000

80,000


(7) Other Expenses (E, –SE) .............................................................. Cash (–A) ............................................................................... Incurred and paid expenses.

40,000

(8) Cash (+A)..................................................................................... Accounts Receivable (–A) ...................................................... Collected cash from customers on account.

56,000

(9) Dividends (–SE) ........................................................................... Cash (–A) ............................................................................... Declared and paid dividends.

26,000

P4–3

40,000

56,000

26,000

Continued

b. Cash B.B. (1) (3) (5) (8)

0 95,000 65,000 75,000 56,000

(2) 31,200 (4) 32,000 (6) 80,000 (7) 40,000 (9) 26,000 _____________________________________ E.B. 81,800

Accounts Receivable B.B. 0 (3) 125,000 (8) 56,000 _________________________________ E.B. 69,000

Land B.B. 0 (4) 32,000 _____________________________________ E.B. 32,000

Notes Payable B.B. 0 (5) 75,000 _________________________________ E.B. 75,000

Common Stock B.B. 0 (1) 95,000 _____________________________________ E.B. 95,000

Dividends B.B. 0 (9) 26,000 _________________________________ E.B. 26,000

Fees Earned B.B. 0 (3) 190,000 _____________________________________ E.B. 190,000

Rent Expense B.B. 0 (2) 31,200 _________________________________ E.B. 31,200

Salary Expense B.B. 0 (6) 80,000 _____________________________________

Other Expense B.B. 0 (7) 40,000 _________________________________


E.B.

80,000

E.B.

40,000


P4–3

Continued

c. Hope, Inc. Income Statement For the Year Ended December 31, 2015 Fees earned ............................................................................ Operating expenses: Salary expense ................................................................... Rent expense ..................................................................... Other expenses ................................................................. Total operating expenses ............................................. Net income .............................................................................

$ 190,000 $ 80,000 31,200 40,000 151,200 $ 38,800

Hope, Inc. Statement of Stockholders’ Equity For the Year Ended December 31, 2015

Beginning Balance, January 1, 2015 Net income Dividends Stock Issuance Ending Balance, December 31, 2015

Common Stock $ 0

95,000 $ 95,000

Retained Earnings $ 0 38,800 (26,000) ______ $ 12,800

Hope, Inc. Balance Sheet As of December 31, 2015 Assets

Liabilities and Stockholders' Equity

Cash ........................................ $ 81,800 Accounts receivable.................. 69,000 Land ........................................ 32,000

Long-term notes payable ............ $ 75,000 Common stock ............................ 95,000 Retained earnings ....................... 12,800 Total liabilities and stockholders’ equity $ 182,800

Total assets ............................... $ 182,800


P4–3

Concluded Hope, Inc. Statement of Cash Flows For the Year Ended December 31, 2015 Cash flows from operating activities: Cash collections from customers ...................................... Cash paid for rent .............................................................. Cash paid for salaries......................................................... Cash paid for other expenses ............................................ Net cash decrease due to operating activities .............

$ 121,000 (31,200) (80,000) (40,000)

Cash flows from investing activities: Purchase of land ................................................................ Net cash decrease due to investing activities ..............

$ (32,000)

Cash flows from financing activities: Proceeds from stock issuance ........................................... Proceeds from debt issuance ............................................ Dividend payment ............................................................. Net cash increase due to financing activities ............... Increase in cash balance .........................................................

$ (30,200)

(32,000) $ 95,000 75,000 (26,000) 144,000 $ 81,800

Beginning cash balance .......................................................... Ending cash balance ...............................................................

0 $ 81,800

P4–4 a. (1) Cash (+A)..................................................................................... Fees Earned (R, +SE) .............................................................. Rendered services.

7,000

(2) Cash (+A)..................................................................................... Accounts Receivable (–A) ...................................................... Collected cash from customers on account.

3,000

(3) Liabilities (–L) .............................................................................. Cash (–A) ............................................................................... Made payment on outstanding liabilities.

3,000

(4) Long–Term Assets (+A) ............................................................... Note Payable (+L) .................................................................. Purchased long–lived assets.

6,000

(5) Miscellaneous Expenses (E, –SE) ................................................ Cash (–A) ............................................................................... Incurred and paid miscellaneous expenses.

4,000

(6) Dividends (–SE) ........................................................................... Cash (–A) ............................................................................... Declared and paid dividends.

800

7,000

3,000

3,000

6,000

4,000

800


P4–4

Continued

b.

a b c

Transaction

Current Ratio

Return on Equity

Debt/Equity Ratio

1. 2. 3. 4. 5. 6.

Increase No Effect Increasea No Effectb Decrease Decrease

Increase No Effect No Effect No Effect Decrease Increasec

Decrease No Effect Decrease Increase Increase Increase

Assuming that liabilities on balance sheet are current. Assuming the note payable is long-term liability in nature. Assuming that return on equity is computed after closing Dividend account to the Retained Earnings account.

c. Morrison Home Services Income Statement For the Month Ended January 31, 2015 Revenues ............................................................................................................. Miscellaneous expenses ...................................................................................... Net income ..........................................................................................................

$ 7,000 4,000 $ 3,000

Morrison Home Services Statement of Stockholders’ Equity For the Month Ended January 31, 2015

Beginning balance Net income Dividends declared Ending balance

Common Stock $ 10,000 ______ $ 10,000

Retained Earnings $ 8,000 3,000 (800) $ 10,200

Morrison Home Services Balance Sheet January 31, 2015 Assets

Liabilities and Stockholders' Equity

Cash ........................................ Receivables ............................... Long-term assets ......................

$ 12,200 1,000 16,000

Total assets ...............................

$ 29,200

*Represents nontrade notes payable.

Liabilities ..................................... $ 3,000* Notes payable ............................. 6,000 Common stock ............................ 10,000 Retained earnings ....................... 10,200 Total liabilities and stockholders’ equity .............. $ 29,200


P4–4

Concluded Morrison Home Services Statement of Cash Flows For the Month Ended January 31, 2015 Cash flows from operating activities: Cash collections from customers ...................................... Payment of expenses ........................................................ Net cash increase due to operating activities ..............

$ 10,000 (4,000) $

Cash flows from investing activities .......................................

6,000 0

Cash flows from financing activities: Repayment of liabilities ..................................................... Dividend payment ............................................................. Net cash decrease due to financing activities .............. Net increase in cash................................................................

$ (3,000)* (800) $

Beginning cash balance .......................................................... Ending cash balance ...............................................................

(3,800) 2,200

10,000 $ 12,200

*Represents nontrade notes payable. d. Morrison Home Services Statement of Cash Flows For the Month Ended January 31, 2015 Cash from operating activities: Net income ................................................................ Adjustments: Decrease in accounts receivable ..........................

$ $

3,000

3,000

Total adjustments ........................................... Net cash due to operating activities ...........................

3,000

(1) Wage Expense (E, -SE) ................................................................ Cash (-A) ................................................................................ Paid employee wages.

5

(2) Cash (+A)..................................................................................... Accounts Receivable (–A) ...................................................... Collected cash from customers on account.

10

(3) Equipment (+A)........................................................................... Cash (–A) ............................................................................... Purchased Equipment.

5

(4) Supplies (+A) ............................................................................... Cash (-A) ................................................................................

2

$ 6,000

P4-5 5

10

5

2


Purchased Supplies. (5) Accounts Payable (–L)................................................................. Cash (–A) ............................................................................... Paid Cash to suppliers.

3

(6) Interest Payable (–L) ................................................................... Cash (–A) ............................................................................... Paid interest accrued in previous period.

3

(7) Cash (+A)..................................................................................... Accounts Receivable (+A) ........................................................... Sales (R, +SE) ......................................................................... Rendered services.

9 9

(8) Long-term Note Payable (-L) ...................................................... Cash (–A) ............................................................................... Reduced long-term debt.

15

(9) Cash (+A)..................................................................................... Common Stock (+SE) ............................................................. Issued stock.

5

(10)Unearned Revenue (-L) ............................................................... Sales (R, +SE) ......................................................................... Rendered services.

3

(11)Interest Expenses (E, –SE) ........................................................... Interest Payable (+L).............................................................. Accrued interest expense.

1

(12)Depreciation Expense (E, –SE) .................................................... Accumulated Depreciation (–A) ............................................ Depreciated equipment.

4

(13)Supplies Expense (E, -SE) ............................................................ Supplies (-A) .......................................................................... Physical count of supplies.

5

(14)Dividends (-SE) ............................................................................ Cash (–A) ............................................................................... Declared and paid dividends.

3

(15)Retained Earnings (–SE) .............................................................. Dividends (+SE) ...................................................................... Closed dividends to Retained Earnings.

3

(16)Sales (-SE) .................................................................................... Wage Expense (+SE) .............................................................. Interest Expense (+SE) ........................................................... Depreciation Expense (+SE)................................................... Supplies Expense (+SE) ..........................................................

21

3

3

18

15

5

3

1

4

5

3

3

5 1 4 5


Income Summary (+SE) ......................................................... Closed sales and expenses to Income Summary.

6

(17)Income Summary (–SE) ............................................................... Retained Earnings (+SE)......................................................... Closed Income Summary to Retained Earnings.

6 6

Tybee Corporation Income Statement For the Month Ended January 31, 2015 (in millions) Sales ................................................................................................................. $ Supplies Expense ................................................................................................. Depreciation Expense .......................................................................................... Interest Expense .................................................................................................. Wage Expense ..................................................................................................... Net income ..........................................................................................................

21 5 4 1 5 6

$

Tybee Corporation Statement of Stockholders’ Equity For the Month Ended January 31, 2015 (in millions)

Beginning balance Stock issuance Net income Dividends Ending balance

Common Stock $ 20 5

$

Retained Earnings $ 12

__ 25

$

6 (3) 15

Tybee Corporation Balance Sheet January 31, 2015 (in millions) Assets Cash ........................................ $ Accounts Receivable ................. Supplies .................................... Prepaid Insurance ..................... Current Assets .......................... Equipment ............................ 55 Less: Acc. Dep. ...................... 16 Land ........................................

Liabilities and Stockholders' Equity 12 14 3 12 41 39 10

Accounts Payable ........................ $ 1 Interest Payable .......................... 1 Unearned Revenue ..................... 9 Other short-term payables ......... 4 Current Liabilities 15 Long-term Note Payable ............. 35 Common stock ............................ Retained earnings .......................

25 15


Total assets ...............................

$

90

Total liabilities and stockholders’ equity .............. $ 90

Tybee Corporation Statement of Cash Flows For the Month Ended January 31, 2015 (in millions) Cash flows from operating activities: Cash collections from customers ...................................... Cash paid to suppliers ....................................................... Cash paid for interest ........................................................ Cash paid for expenses ...................................................... Net cash increase due to operating activities ..............

$

19 (5) (3) (5) $

Cash flows from investing activities: Cash paid for equipment ................................................... Net cash decrease due to investing activities .............. Cash flows from financing activities: Repayment of liabilities ..................................................... Stock issuance ................................................................... Dividend payment ............................................................. Net cash decrease due to financing activities .............. Net change in cash .................................................................

6

(5) (5) $

(15) 5 (3)

Beginning cash balance .......................................................... Ending cash balance ...............................................................

$

(13) (12)

$

24 12

Tybee Corporation Statement of Cash Flows For the Month Ended January 31, 2015 (in millions) Cash from operating activities: Net income ................................................................ Depreciation Expense …………………………… .................. Adjustments: Decrease in accounts receivable .......................... Decrease in supplies............................................. Decrease in accounts payable .............................. Decrease in interest payable................................ Decrease in unearned revenue ............................. Total adjustments ........................................... Net cash increase (decrease) due to operating activities ...........................

$ 4 $

1 3 (3) (2) (3) (4) $ 6

Cash flows from investing activities: Cash paid for equipment ................................................... Net cash decrease due to investing activities .............. Cash flows from financing activities: Repayment of liabilities .....................................................

6

(5) (5) $

(15)


Stock issuance ................................................................... Dividend payment ............................................................. Net cash decrease due to financing activities .............. Net change in cash .................................................................

5 (3)

Beginning cash balance .......................................................... Ending cash balance ...............................................................

$

(13) (12)

$

24 12

P4–6 Accounts Direction (1) (2) (3) (4) (5) (6) (7) (8)

Net Income

Cash

+

Contributed Capital

+

Inventory Accounts Payable Accounts Receivable Fees Earned Cash Accounts Receivable Equipment Cash

+ + + + + – + –

Wages Payable Cash Cash Fees Earned

Cash Long-Term Note Payable (9) Cash Interest Expense (10) Cash Land Gain on Sales of Land

Net Operating Cash Flow

NE

NE

NE

NE

+

NE

NE NE

+ NE

– –

NE

+ +

+

+

NE

NE

+

NE

– – – + + – +


P4–7 (1) (2) (3) (4) (5) (6) (7)

Classification

Assets

Liabilities

A C A C C C C

NE – + NE – – NE

+ NE NE – NE NE –

Stockholders' Equity – – + + – – +

Revenues

Expenses

NE NE + + NE NE +

+ + NE NE + + NE

P4–8 a. Supplies Expense (E, –SE) .................................................................. Supplies Inventory (–A) .............................................................. Adjusted for supplies used.

55,000

b. Rent Expense (E, –SE) ........................................................................ Rent Payable (+L) ........................................................................ Incurred, but did not pay, rent.

2,400

c.

Unearned Service Revenues (–L) ....................................................... Fees Earned (R, +SE) ................................................................... Rendered services for cash collected in advance.

12,000

d. Depreciation Expense (E, –SE) ........................................................... Accumulated Depreciation (–A) ................................................. Depreciated fixed assets for 2011.

50,000

e. Interest Expense (E, –SE) ................................................................... Interest Payable (+L) ................................................................... Incurred, but did not pay, interest.

600

f.

55,000

2,400

12,000

50,000

600

Advertising Expense (E, –SE) ............................................................. Advertising Payable (+L) ............................................................. Incurred, but did not pay, advertising.

28,000

g. Insurance Expense (E, –SE) ................................................................ Prepaid Insurance (–A) ............................................................... Adjusted for expiration of prepaid insurance.

175

28,000

175

P4–9 (1) Rent Expense (E, –SE) ........................................................................ Prepaid Rent (–A) ....................................................................... Adjusted for expiration of prepaid rent.

2,700

(2) Insurance Expense (E, –SE) ................................................................ Prepaid Insurance (–A) ............................................................... Adjusted for expiration of prepaid insurance.

700

(3) Depreciation Expense (E, –SE) ........................................................... Accumulated Depreciation (–A) ................................................. Depreciated fixed assets.

2,400

2,700

700

2,400


P4–9

Concluded

(4) Salary Expense (E, –SE) ...................................................................... Salaries Payable (+L) ................................................................... Incurred, but did not pay, salaries.

1,200

(5) Unearned Revenues (–L) ................................................................... Fees Earned (R, +SE) ................................................................... Rendered services for cash collected in advance.

200

1,200

200

P4–10 Beginning payable Interest expense Interest paid Ending payable

Case 1 $ 400 2,800 (3,000) $ X

Case 2 $ 800 2,800 X $ 300

Case 3 $ 400 2,800 (2,300) X

Case 4 $ X 2,800 (2,600) $ 200

Case 5 $ 200 2,800 X $ 400

Case 6 $ X 2,800 (2,500) $ 0

X equals

$

$ (3,300)

$

$

$ (2,600)

$

200

900

0

(300)

P4–11 a. (1) Cash (+A)..................................................................................... Unearned Insurance Premium (+L) ....................................... Collected insurance premium in advance.

240,000

(2) Unearned Insurance Premium (–L) ............................................ Premium Revenue (R, +SE) .................................................... Earned insurance premium.

70,000*

240,000

70,000

* $70,000 = ($240,000 premium ÷ 24 month total coverage) x 7 months expired (3) The purpose of the adjusting journal entry is to recognize an economic event not captured on a timely basis by a transaction. That is, the adjusting entry is to record the fact that Prustate fulfilled part of its obligation to Jacobs by providing insurance coverage to Jacobs, and in doing so, Prustate earned part of the $240,000 it collected from Jacobs on May 31, 2014. The journal entry results in Prustate allocating the revenue to the period in which it earned the revenue. (4) Insurance revenue Cash receipts

$

2014 70,000 240,000

2015 $120,000 0

2016 $50,000 0

b. (1) Prepaid Insurance (+A) ............................................................... Cash (–A) ............................................................................... Paid insurance in advance.

240,000

(2) Insurance Expense (E, –SE) ......................................................... Prepaid Insurance (–A) .......................................................... Adjusted for expiration of prepaid insurance.

70,000

Total $240,000 240,000

240,000

70,000


P4–11 (3)

Concluded The purpose of the adjusting journal entry is to recognize an economic event not captured on a timely basis by a transaction. That is, the adjusting entry is to record the fact that Jacobs Printing Corporation consumed a portion of one of its assets (i.e., prepaid insurance), and in doing so, Jacobs Printing incurred an expense. The journal entry results in Jacobs Printing allocating the cost of the insurance policy to those periods for which Jacobs Printing received coverage from the policy.

(4) Insurance expense Cash payment

2014 $ 70,000 240,000

2015 $120,000 0

2016 $50,000 0

Total $240,000 240,000

P4–12 Net Income NE NE NE + NE

(1) (2) (3) (4) (5)

Current Ratio + – – + – +b NE –

Working Capital + – – + –

Debt/Equity Ratio – NE – –a +

(6) NE NE – (7) NE NE NE (8) – – +c __________________ a Transaction (4) increases revenues, thereby increasing the net income amount closed into Retained

b

c

Earnings. Since increasing Retained Earnings increases stockholders' equity, Transaction (4) decreases the debt/equity ratio. Note that the inventory was sold above cost. Transaction (6) reduces cash and wages payable by the same amount. Reducing the numerator and the denominator by the same amount has the impact of increasing the ratio ($10,500/$8,722 becomes $10,300/$8,522). The $200 has a larger relative effect on the smaller denominator. Transaction (8) increases expenses, thereby decreasing the net income amount closed into Retained Earnings. Since reducing Retained Earnings reduces stockholders' equity, Transaction (8) increases the debt/equity ratio.

P4–13 a. Current ratio

= = =

Current assets ÷ Current liabilities ($10,000 + $40,000) ÷ $20,000 2.50

Debt/equity

= = =

Total liabilities ÷ Total stockholders’ equity ($20,000 + $20,000) ÷ ($30,000 + $50,000) .50

Book value

= = =

Net assets ÷ Number of shares outstanding $80,000 ÷ 12,000 6.67


P4–13

Concluded

b. Financial Alternative 1. Stock issuance 2. Long-term note

Current Ratio 2.50 2.50

Debt/Equity Ratio .36 .88

Book Value 7.33 6.67

3. Open account

1.00

.88

6.67

c.

Stock Issuance: Issuing stock is attractive because it improves the company’s debt/equity ratio. By decreasing this ratio, the company might become more attractive to creditors in the future, thereby allowing the company to borrow money at more favorable rates. Further, issuing stock, as opposed to issuing debt, will not hurt future cash flows. Stock does not require mandatory interest payments, and it never matures. On the negative side, issuing stock has the potential to dilute the ownership rights of existing stockholders. Long-Term Note: Issuing debt could place severe cash restrictions on the company. Each year the note is outstanding the company will incur $3,600 in interest charges. Also, the company will have to pay back the $30,000 when the note matures. If the company is unable to meet any of these obligations, it could be forced into bankruptcy. On the positive side, interest on the loan is deductible for tax purposes, so issuing the note would decrease the company’s cash outflows from income taxes. Further, with a long-term note, as opposed to a short-term note, the company has more time in which to generate cash to pay off the obligation. Open Account: Purchasing the equipment on account would allow the company to avoid interest charges. However, the company would be placed in a severe financial bind by purchasing the equipment on account. The $30,000 must be repaid within 30 days, and the company currently has only $10,000 in cash. The company would have to raise an additional $20,000 either through operations or by selling some of its assets. Either solution may not be desirable. Diverting money from operations to pay off the obligation means that the company would not be able to plow as much of its earnings back into the business. If the company reduces the amount reinvested in the business, it could suffer long-term negative consequences. Using money raised by selling assets may cause the company to hurt its base of operations, thereby hurting future operations. Of the three options, issuing stock appears to be the most attractive.

d. The balance in Retained Earnings represents the excess of cumulative net income over cumulative net losses and declared dividends. Retained Earnings does not represent cash (or any other particular asset). Retained Earnings is simply a measure of one source of capital. In other words, Retained Earnings is an accounting concept and does not indicate a company’s financial wherewithal to purchase items or declare dividends. Items are purchased and dividends are paid with assets (usually cash). Since the company has only $10,000 in cash, it would have to use other assets to purchase the equipment and pay a dividend. In doing so, the company would hurt its future base of operations.


P4–14 Return on Equity

Current Ratio

Debt/ Equity

1. Purchase inventory on account.

NE

-

+

2. Sell assets for cash at a gain. 3. Provide services to customers, receiving cash in return. 4. Make a principal payment on an outstanding long-term liability.

+

+

+

+

NE

5. Issue common stock for cash.

+

Return on Sales

Current Ratio

Debt/ Equity

NE

NE

NE

NE

+

– NE

– –

+ +

NE

+

Return on Assets

Current Ratio

NE

+

+

+

+

+

P4–15

1. Purchase equipment for cash. 2. Purchase machinery in exchange for a long-term note payable. 3. Pay salaries, which have not been accrued, to employees. 4. Declare a dividend. 5. Issue common stock to satisfy a current obligation.

P4–16

1. Purchase equipment in exchange for a note payable. 2. Pay cash for marketing its services. 3. Sell equipment for an amount less than its book value. 4. Pay wages that were accrued in a previous period 5. Provide a service for which cash was collected in a previous period


P4–17 a. All T-accounts for P4−17 appear in this section. Transactions are keyed to numbers in parentheses, adjusting journal entries are keyed to lowercase letters, and closing entries are keyed to upper case letters. Cash B. B. (1a) (5) (7) (9)

170,000 350,000 850,000 37,000 120,000

E. B.

9,000

Accounts Receivable (3) (6) (8) (11) (12)

400,000 870,000 148,000 50,000 50,000

B. B. (1a)

188,000 1,350,000

E. B.

616,000

Interest Receivable B. B.

1,620

E. B.

1,620

B. B (10)

200,000 820,000

E. B.

320,000

E. B.

B. B. 44,400

B. B. (4)

72,000

E. B.

40,000 110,000

25,000

E. B.

25,000

40,000

B. B. (12)

160,000 50,000

E. B.

180,000

E. B.

29,600

B. B.

480,000

B. B. (g)

98,000 48,000

B. B.

950,000

E. B.

480,000

E. B.

146,000

E. B.

950,000

Equipment

Accum. Depr.—Equipment

Accum. Depr.—Machinery B. B.

B. B.

47,500

E. B.

277,500

Wages Payable

(g) E. B.

30,000

Accounts Payable

75,000 (6)

(g)

(7)

Machinery

Patent 230,000

110,000

Long-Term Investments

0

(c)

700,000

Supplies Inventory

0 72,000

Prepaid Advertising

74,000 (a)

(3)

(1b)

(b)

Prepaid Insurance B. B.

850,000 72,000

Notes Receivable

0

(f)

(5) (10)

Merchandise Inventory B. B. (2)

870,000

B. B. (2)

220,000 820,000

E. B.

170,000

12,500

62,500

Op. Exp. Payable

Interest Payable

B. B.

73,000

B. B.

0

B. B.

0

(e)

43,000

(d)

6,000

(h)

3,361

E. B.

43,000

E. B.

6,000

E. B.

3,361

73,000

Short-Term Notes Payable

Mortgage Payable

Bonds Payable

B. B. (4)

0 110,000

B. B.

300,000

B. B.

500,000

E. B.

110,000

E. B.

300,000

E. B.

500,000


P4–17

Continued Common Stock B. B. (9)

Retained Earnings 500,000 120,000 (C)

E. B.

620,000

B. B. (1a)

0 1,700,000

50,000

Sales

Dividends

B. B.

416,000

(B)

243,859

E. B.

609,859

E. B.

0

E .B.

B. B. (3) (e)

0

(A) E. B.

E. B.

B. B. (8)

0

E. B.

B. B. (g)

E. B.

(c) (d) (A)

25,000 36,000 87,000

(a)

(A) 0

0 47,500

48,000

0

(A) E. B.

Interest Expense B. B. (h)

44,400

Depr. Expense—Machinery B. B. (g)

(A)

0 44,400

E. B.

0 48,000

E. B.

0 12,500

0

Insurance Expense B. B.

0

110,000

0

(A)

0

370,000

Depr. Expense—Equipment

0 110,000

Amortization Expense B. B. (g)

0 148,000

E. B.

(A)

0

0

42,000

Supplies Expense B. B. (b)

E. B.

7,000

Miscellaneous Op. Expense

0 36,000 6,000 (A)

0 7,000

0 327,000 43,000

700,000

Operating Expense B. B. (d) (d)

1,620 (A)

B. B. (7)

Wage Expense

0 700,000 (A)

0

0

1,620

Cost of Goods Sold

E. B.

E. B.

50,000

Gain on Sale of Investment

B. B. (f)

(A)

0 50,000 (C)

Interest Revenue

(A) 1,700,000

B. B. (1b)

B. B. (11)

47,500

0

Income Summary

0 3,361

12,500

(A) E. B.

3,361

(B)

243,859

(A)

243,859

0

b. Entries are posted to the T-accounts in Part (a). (1a) Cash (+A)..................................................................................... Accounts Receivable (+A) ...................................................... Sales (R, +SE) .................................................................... Made sales.

350,000 1,350,000

(1b) Cost of Goods Sold (E, –SE)......................................................... Merchandise Inventory (–A)............................................. Recorded cost of inventory sold.

700,000

1,700,000

700,000


P4–17 (2)

Continued Merchandise Inventory (+A).................................................. Accounts Payable (+L) ...................................................... Purchased inventory on account.

820,000

Wage Expense (E, –SE) .......................................................... Wages Payable (–L) ............................................................... Cash (–A) .......................................................................... Paid wages.

327,000 73,000

Supplies Inventory (+A) ......................................................... Short–Term Notes Payable (+L)........................................ Purchased supplies by note.

110,000

Cash (+A) ............................................................................... Accounts Receivable (–A) ................................................. Collected cash from customers.

850,000

Accounts Payable (–L) ........................................................... Cash (–A) .......................................................................... Made payment to suppliers.

870,000

Cash (+A) ............................................................................... Long–Term Investments (–A) ........................................... Gain on Sale of Investment (Ga, +SE) ............................... Sold investment.

37,000

Miscellaneous Operating Expense (E, –SE) ........................... Cash (–A) .......................................................................... Incurred and paid expenses.

148,000

Cash (+A) ............................................................................... Common Stock (+SE) ........................................................ Issued common stock.

120,000

(10) Notes Receivable (+A) ................................................................ Accounts Receivable (–A) ................................................. Accepted a note in payment of an open account.

72,000

(11) Dividends (–SE) ........................................................................... Cash (–A) .......................................................................... Declared and paid a cash dividend.

50,000

(12) Long–Term Investments (+A) ..................................................... Cash (–A)......................................................................... Purchased investments.

50,000

(3)

(4)

(5)

(6)

(7)

(8)

(9)

820,000

400,000

110,000

850,000

870,000

30,000 7,000

148,000

120,000

72,000

50,000

50,000


122

P4–17

Chapter 4

Continued

c. Account Cash Accounts Rec. Interest Rec. Merchandise Inv. Supplies Inventory Prepaid Insurance Prepaid Advert. Notes Receivable L-T Investments Equipment Accum. Depr.—Equip. Machinery Accum. Depr.—Mach. Patent Accounts Payable Wages Payable Interest Payable Oper. Exp. Payable S-T Notes Payable Mortgage Payable Bonds Payable Common Stock Retained Earnings Dividends Sales Interest Revenue Gain on Investment Cost of Goods Sold Wage Expense Operating Exp. Miscellaneous Op.Exp. Insurance Exp. Supplies Exp. Interest Exp. Amortization Exp. Depr. Exp.—Equip. Depr. Exp.—Mach. Income Summary Total

Unadjusted Trial Balance Dr.

Adjusting Entries Dr.

Cr.

Cr.

9,000 616,000 (f)

1,620

320,000 150,000 74,000

(b) 110,000 (a) 44,400 (c)

25,000

72,000 180,000 480,000 98,000

(g)

Adjusted Trial Balance Dr. Cr.

Cr.

230,000

(g) (g)

47,500 12,500

146,000

146,000

(e) (h) (d)

43,000 3,361 6,000

277,500

277,500 62,500

170,000 43,000 3,361 6,000 110,000 300,000 500,000 620,000 416,000

110,000 300,000 500,000 620,000 416,000 50,000

950,000

62,500

170,000

(C)

50,000

50,000 1,700,000 (f)

1,700,000 1,620 7,000

1,620

7,000 700,000 327,000

(e) (d)

43,000 42,000

(a) (b) (h) (g) (g) (g)

44,400 110,000 3,361 12,500 48,000 47,500

148,000

(c&d) 61,000

377,381

377,381

700,000 370,000 42,000 87,000 44,400 110,000 3,361 12,500 48,000 47,500 4,300,481

4,300,481

(B) (C)

243,859 50,000

(A) (A) (A) (A) (A) (A) (A) (A) (A) (A) (A)

700,000 370,000 42,000 87,000 44,400 110,000 3,361 12,500 48,000 47,500 243,859 2,002,479

170,000 43,000 3,361 6,000 110,000 300,000 500,000 620,000 609,859

(A) 1,700,000 (A) 1,620 (A) 7,000

(B) 4,151,000

Cr.

9,000 616,000 1,620 320,000 40,000 29,600 25,000 72,000 180,000 480,000

950,000

75,000

4,151,000

Final Trial Balance Dr.

9,000 616,000 1,620 320,000 40,000 29,600 25,000 72,000 180,000 480,000

48,000

950,000

Closing Entries Dr.

243,859 2,002,479

2,785,720

2,785,720


Chapter 5

P4–17

54

Continued

d. Entries are posted to the T–accounts in Part (a). (a) Insurance Expense (E, –SE) ......................................................... Prepaid Insurance (–A) .......................................................... Adjusted for expiration of prepaid insurance.

44,400 44,400

(b) Supplies Expense (E, –SE) ........................................................... Supplies Inventory (–A) ......................................................... Adjusted for supplies used.

110,000

(c) Prepaid Advertising (+A) ............................................................. Miscellaneous Operating Expense (E, –SE) ........................... Recognized advertising to be used in subsequent periods.

25,000

(d) Operating Expense (E, –SE)......................................................... Operating Expense (E, –SE)......................................................... Miscellaneous Operating Expense (E, –SE) ........................... Op. Exp. Payable (+L) ............................................................. Incurred, but did not pay, and reclassify.

6,000* 36,000

110,000

25,000

36,000* 6,000

* $6,000 = ($3,500 per month  12 months) – $36,000 misc. exp. payment (e) Wage Expense (E, –SE) ............................................................... Wages Payable (+L) ............................................................... Incurred, but did not pay, wages.

43,000

(f) Interest Receivable (+A) ............................................................. Interest Revenue (R, +SE) ...................................................... Earned, but did not collect, interest.

1,620*

43,000

1,620

* $1,620 = $72,000  9%  3/12 (g) Depreciation Expense—Equipment (E, –SE)............................... Depreciation Expense—Machinery (E, –SE) ............................... Amortization Expense (E, –SE).................................................... Accumulated Depreciation—Equipment (–A) ....................... Accumulated Depreciation—Machinery (–A) ....................... Patent (–A) ............................................................................ Amortized fixed and intangible assets.

48,000 47,500 12,500

(h) Interest Expense (E, –SE) ............................................................ Interest Payable (+L).............................................................. Incurred, but did not pay, interest.

3,361*

* $3,361 = $110,000  10%  110/360

48,000 47,500 12,500

3,361


P4–17

Continued

e. Closing entries are posted to the T–accounts in Part (a).

a.

(A) Sales ........................................................................................... Interest Revenue ....................................................................... Gain on Sale of Investment ........................................................ Income Summary .................................................................. Cost of Goods Sold ............................................................... Wage Expense ...................................................................... Operating Expense ................................................................ Misc. Operating Expense ...................................................... Insurance Expense ................................................................ Supplies Expense .................................................................. Interest Expense ................................................................... Amortization Expense ........................................................... Depreciation Expense—Equipment ..................................... Depreciation Expense—Machinery ...................................... Closed revenues and expenses into Income Summary.

1,700,000 1,620 7,000

(B) Income Summary........................................................................ Retained Earnings.................................................................. Closed Income Summary into Retained Earnings.

243,859

(C) Retained Earnings ...................................................................... Dividends .............................................................................. Closed Dividends into Retained Earnings.

50,000

243,859 700,000 370,000 42,000 87,000 44,400 110,000 3,361 12,500 48,000 47,500

243,859

50,000

J.D.F. Company Income Statement For the Year Ended December 31, 2015 Revenues: Sales............................................................... Interest revenue ............................................ Gain on sale of investment ............................ Total revenues .......................................... Operating expenses: Cost of goods sold ......................................... $700,000 Wage expense ............................................... 370,000 Operating expense ........................................ 42,000 Miscellaneous operating expense ................. 87,000 Insurance expense......................................... 44,400 Supplies expense ........................................... 110,000 Amortization expense ................................... 12,500 Depreciation expense—Equipment .............. 48,000 Depreciation expense—Machinery ............... 47,500 Total operating expenses ......................... Interest expense ................................................. Total expenses .................................................... Net income .........................................................

$

1,700,000 1,620 7,000 $1,708,620

$

1,461,400 3,361 $

1,464,761 243,859


P4–17

Continued J.D.F. Company Statement of Retained Earnings For the Year Ended December 31, 2015 Retained earnings balance, January 1, 2015 ....................................................... Plus: Net income.................................................................................................. Less: Dividends declared ..................................................................................... Retained earnings balance, December 31, 2015 .................................................

$ 416,000 243,859 (50,000) $ 609,859

J.D.F. Company Balance Sheet December 31, 2015 Assets Current assets: Cash ............................................................... Accounts receivable ...................................... Interest receivable......................................... Inventory ....................................................... Supplies inventory ......................................... Prepaid insurance .......................................... Prepaid advertising expenses ........................ Notes receivable ............................................ Total current assets .................................. Long-term investments ...................................... Fixed assets: Equipment ..................................................... Less: Accumulated depreciation ................... Net book value of equipment ....................... Machinery...................................................... Less: Accumulated depreciation ................... Net book value of machinery ........................ Total fixed assets ...................................... Patent ................................................................. Total assets ......................................................... Liabilities & Stockholders' Equity Current liabilities: Accounts payable .......................................... Wages payable .............................................. Interest payable............................................. Operating expense payable ........................... Short-term notes payable ............................. Total current liabilities ............................. Long-term liabilities: Mortgage payable ......................................... Bond payable ................................................. Total long-term liabilities .........................

$

9,000 616,000 1,620 320,000 40,000 29,600 25,000 72,000 $1,113,220 180,000

$ 480,000 146,000 $ 334,000 $ 950,000 277,500 672,500 1,006,500 62,500 $ 2,362,220

$ 170,000 43,000 3,361 6,000 110,000 $

332,361

$ 300,000 500,000 800,000


Common stock .................................................... Retained earnings ............................................... Total liabilities & stockholders' equity ...............

$

620,000 609,859 2,362,220


P4–17

Continued J.D.F. Company Statement of Cash Flows (Direct Method) For the Year Ended December 31, 2015 Cash flows from operating activities: Collections from customers on cash sales ........................ Collection from customers on open accounts .................. Payments for wages .......................................................... Payments to suppliers on open account ........................... Payment for misc. oper. exp ............................................. Net cash increase (decrease) due to operating activities ................................................... Cash flows from investing activities: Proceeds from sale of investment..................................... Purchase of investment ..................................................... Net cash increase (decrease) due to investing activities .................................................... Cash flows from financing activities: Proceeds from issuance of common stock ........................ Payment of dividends ........................................................ Net cash increase (decrease) due to financing activities .................................................... Decrease in cash balance ....................................................... Beginning cash balance .......................................................... Ending cash balance ...............................................................

$ 350,000 850,000 (400,000) (870,000) (148,000) $ (218,000) $

37,000 (50,000) (13,000)

$ 120,000 (50,000) 70,000 $(161,000) 170,000 $ 9,000


P4–17 b.

Concluded J.D.F. Company Statement of Cash Flows (Indirect Method) For the Year Ended December 31, 2015 Cash flows from operating activities: Net income ........................................................... Adjustments: Depreciation expense—Equipment ................ Depreciation expense—Machinery ................. Amortization expense ..................................... Gain on sale of investments ............................ Increase in accounts receivable ...................... Increase in interest receivable ........................ Increase in merchandise inventory ................. Increase in prepaid advertising ....................... Increase in notes receivable ............................ Decrease in prepaid insurance ........................ Decrease in accounts payable ......................... Decrease in wages payable ............................. Increase in operating exp. payable ................. Increase in short-term notes payable ............. Increase in interest payable ............................ Total adjustments ...................................... Net cash increase (decrease) due to operating activities.....................................

$ 243,859 $ 48,000 47,500 12,500 (7,000) (428,000) (1,620) (120,000) (25,000) (72,000) 44,400 (50,000) (30,000) 6,000 110,000 3,361

Cash flows from investing activities: Proceeds from sale of investment........................ Purchase of investment ........................................ Net cash increase (decrease) due to investing activities ...................................... Cash flows from financing activities: Proceeds from issuance of common stock ........... Payment of dividends ........................................... Net cash increase (decrease) due to financing activities...................................... Decrease in cash balance .......................................... Beginning cash balance .......................................................... Ending cash balance ...............................................................

(461,859) $ (218,000) $ 37,000 (50,000) (13,000) $ 120,000 (50,000) 70,000 $ (161,000) $

170,000 9,000


P4–18 For 2015 the net cash flow from operations is $62,400 which is composed of: (1) cash collections from services rendered, and (2) cash payments due to operating activities. Let us compute the cash collections from services rendered for 2015: Cash Collections from Services Rendered for 2015

$54,000

=

=

Revenue from Services per Income Statement in 2015

$54,700 + $600

+

Decrease in Accounts Receivable

Increase in Accounts Receivable

+

Increase in Unearned Revenue

Decrease in Unearned Revenue

$1,300

Now, we can compute the cash payments due to operating activities: Net Cash Flow from Operations during 2015

=

Cash Collections from Services Rendered during 2015

+

Cash Payment Due to Operating Activities during 2015

$62,400

=

$54,000

+

X

X

=

$8,400

P4–19 In order to answer this question, we need to know how much total revenue is generated by Mayberry from each of its 2 sources: (1) advertising display sales; and (2) consulting services. We are given the total revenues as reported on the Income Statement, and selected activity of cash receipts and changes in Accounts Receivable account for the advertising display sales. First, based on the following relationships, we will compute the revenue generated by advertising display sales for 2015 and 2014. Cash Collections from Advertising Display Sales during the Year

=

Advertising Display Sales Revenue as Reported on the Income St.

+ –

Decrease in Acc. Rec. Increase in Acc. Rec.

For 2014 $41,500

=

X + $2,800

X

=

$38,700

$43,500

=

X + $2,700

X

=

$40,800

For 2015

Based on the above calculations we know that total revenue generated from advertising display sales during 2014 and 2015, respectively, was $38,700 and $40,800.


P4–19

Concluded

Second, based on the above information and the total revenue as reported in the 2014 and 2015 income statements, we can compute the amount of revenue generated by Mayberry Enterprises from its consulting services. For 2014 $76,000

– $38,700

=

$37,300

– $40,800

=

$48,700

For 2015 $89,500

Overall, the revenue from each segment can be broken down as follows.

2014 2015 Total

Display Sales $ 38,700 40,800 $ 79,500

Consulting Service $37,300 48,700 $ 86,000

Total $ 76,000 89,500 $ 165,500

Based on the above, it seems that revenue stream is growing for both product lines. However, the growth is higher in the consulting service product line.

P4–20 Cash Account ..................................................................................... Accumulated Depreciation ................................................................ Equipment .................................................................................. Gain on Sale of Equipment ......................................................... a b

23,400 6,600a 24,400b 5,600

$24,300 + $8,700 – $26,400 = $6,600 $84,800 + $37,000 – $97,400 = $24,400

It is true that Badger’s net income has more than tripled from 2014 to 2015. However, this increase is entirely due to selling the equipment with a book value of $17,800 ($24,400 – $6,600) for $23,400. This has resulted in a gain of $5,600. Since net income is $5,200, it appears that Badger has lost money from its regular, recurring operations. I am not sure if we should extend any loans to Badger, let alone grant the preferential interest rates.


ISSUES FOR DISCUSSION ID4–1 a. The amount at which MCI would report the SBS system on its balance sheet is the amount MCI gave up to receive it. Since MCI "paid" $376 million in stock and took on a note for $104 million (with the proceeds going to IBM), MCI would report the SBS system at $428 million (i.e., $480 million less $52 million of miscellaneous assets received). b. The exchange increased assets by $480 million, liabilities by $104 million, and stockholders' equity by $376 million. The accounting equation remained in balance, as it should for all transactions. c.

MCI's balance sheet would show new assets composed of $52 million in miscellaneous assets and $428 million ($480 million – $52 million) for SBS. Its liabilities now include a note payable for $104 million. MCI's stockholders' equity would increase by $376 million for the common stock MCI issued to IBM.

d.

Account Miscellaneous Assets (asset) Investment in SBS (asset) Notes Payable (liability) Common Stock (equity)

Effect Increase Increase Increase Increase

Dollar Value $ 52 million 428 million 104 million 376 million

e. Miscellaneous Assets (+A) ................................................................. Investment in SBS (+A) ...................................................................... Notes Payable (+L) ...................................................................... Common Stock (+SE) .................................................................. Acquired SBS and other assets from IBM.

52,000,000 428,000,000 104,000,000 376,000,000

ID4–2 a.

From Campbell Soup’s perspective, assets increased (inventory, fixed assets, intangible assets—less the decrease in cash) and liabilities increased (short-term notes payable).

b.

From Unilever’s perspective, since Campbell’s paid a premium for the acquisition, assets were increased (cash received increased more than the drop in fixed assets and inventory) and the premium received would have increased equity (through a gain).

c.

See a above

d.

Fixed Assets and Inventory (+A)

100

Trademarks (+A)

490

Goodwill (+A)

330

Cash and ST Notes Payable(-A, +L) ID4–3

920


a. Issuing stock increases stockholders' equity and assets (for the cash raised) by the value of the stock issued. If the proceeds were then used to reduce debt, liabilities would decrease by the amount of liabilities paid off and assets would decrease for the cash used to retire the debt. If the proceeds from the stock issuance were used to acquire plant and equipment, assets would increase by the value of the new plant and equipment but decrease for the cash paid out for the new plant and equipment. The effect would be similar for current assets. Thus, the net effect of issuing stock to reduce debt would be to increase stockholders' equity and decrease liabilities. There may be a positive or negative residual effect on assets if the amount of cash raised through the stock issuance is different from the amount of cash used to pay off the debt. The net effect of issuing stock to acquire plant and equipment or to increase current assets would be to increase stockholders' equity and increase assets. b. A ratio that is often used to assess the riskiness of a company is the debt/equity ratio. This ratio is calculated as a company's total liabilities divided by the company's total stockholders' equity. The higher the ratio is, the less the stockholders have at stake in the company relative to the debtholders. This implies that the higher the ratio, the more risky the company is. By issuing additional stock, the company increases its stockholders' equity and thereby decreases its debt/equity ratio. In this way, issuing stock would affect the riskiness of the company and, hence, its credit rating.

ID4–4 a. There are a number of items that could explain why cash flows from operations would be less than net income. These items fall into two general classes: (1) items that affect net income but not cash flows from operating activities and (2) items that affect cash flows from operating activities but not net income. Specific examples of the former include gains, sales on account, accrual of other receivables, and so forth. Specific examples of the latter include payments to suppliers for inventory purchased previously on account, payments of other previously accrued payables, and so forth. b. There are four places in the financial statements that would probably be particularly informative about cash flow problems. First, a big increase in the balance of Accounts Receivable, particularly if the balance in Sales has not increased, could indicate that the company is having trouble collecting on credit sales. This would imply that the company has less cash flowing in. Second, a big increase in the company's payables could signal cash flow problems. That is, if a company is having cash flow problems, it would try to delay paying its payables as long as possible so that it can use its cash for more immediate problems. Third, a big decrease in inventory could result from cash flow problems because the company cannot afford to tie up as much cash in inventory. Finally, the statement of cash flows can provide lots of information about a company's cash flows and solvency. For example, a company that repeatedly fails to generate cash from operating activities could experience some solvency problems because a company cannot borrow or issue stock indefinitely. In addition, the investing activities section of the statement of cash flows would reveal whether the company is selling off lots of assets. This could indicate that the company is liquidating its productive base in an effort to raise enough capital to stay in business.

ID4–5


a. When WorldCom capitalized an expenditure as an asset, it did not show that dollar amount as an expense and therefore underreported expenses and overstated earnings. For example, if the company spent $10 million on its telecommunications lines and should have recorded a $10 million expense, it actually put the $10 million on the balance sheet as an asset, with the expense to be recorded in future periods as the asset depreciated. Future profits, of course, would be lower under this treatment, as those time periods would be charged with a depreciation expense. WorldCom was putting off the inevitable; the expenditures would eventually be called expenses but due to earnings pressure the company opted to capitalize the expenditure and show the expense at a later date. The company clearly violated the matching principle with this approach. b. If management feels pressure from public investors and Wall Street analysts, it may be tempted to employ techniques to defer expenses so that current earnings are higher than permitted under GAAP. Many observers have commented that management of publiclytraded companies often act with short-term profit targets in mind, ignoring long-term health and strategy. c. Given the size of publicly-traded companies, auditors cannot look at each and every transaction. Employing the materiality exception, auditors will focus their time on larger transactions. However, many small transactions (not material taken singly) become material when viewed in aggregate. d. Internal controls are designed to act as a “check-and-balance” mechanism so that the decisions of one manager are subject to review by an independent party. Had the audit committees of the Boards of Directors of WorldCom, Tyco and HealthSouth been truly committed to protecting shareholders, they would have established internal procedures where auditors—reporting directly to the Board, not to senior management—would have been reviewing the accounting procedures. An internal auditor of WorldCom, for example, would have spotted the inaccurate treatment of current expenditures and would have notified the Board’s audit committee, who could have immediately addressed the situation. ID4–6 Mr. Bailey is drawing a distinction between cash flows from operating activities and net income, and stressing how cash flows are more important for the company's solvency. Net income is a measure of the net assets the company generated during the year from operating activities, whereas cash flows address only one asset (i.e., cash). Thus, the generation of net income (i.e., "my bottom line is black") does not mean that the company generated cash. One thing that Mr. Bailey specifically refers to is credit sales (which increases net income) for which the company is not collecting cash on a timely basis. Because cash is the medium of exchange in the U.S. economy, cash is necessary for companies to be able to pay their employees, to obtain merchandise to sell, to pay obligations, to pay dividends, and so forth. Thus, without cash a company cannot stay in business. For these reasons, Mr. Bailey advises to "watch cash flow."

ID4–7


A.

Net income would not be affected. BMW’s change moves items from one part of the income statement (operating) to another (non-operating), but the items are still deducted as expenses when the final net income is calculated. The change will be to a line item within the income statement called “operating income”. B. BMW, as a global manufacturing company, wants its financial statements to conform to international standards so that users of financial statements can accurately compare BMW’s financial performance to that of other global companies. Additionally, BMW will show a higher operating income if it moves expenses into the non-operating section of the income statement. Many financial analysts focus on operating income as a measure of a company’s ability to generate a profit by the daily running of its core business. A higher operating income might improve BMW’s reputation in the financial community. C. An investment bank such as Goldman Sachs has significant business relationships with companies all across the globe and in all different industries. Goldman Sachs would ideally like to compare companies, using the same set of standards and measures. If a European company uses one set of accounting rules while a U.S. company uses another, it will be difficult for Goldman to determine the relative strengths and weaknesses of the companies. D. BMW would need to follow U.S. accounting standards if it would choose to be listed on the NYSE. Countries are attempting to synchronize their accounting standards, but certain differences still exist. If BMW wanted access to U.S. capital markets, it would have to conform to U.S. standards. In the age of globalization, international standards become that much more important. A global concern like BMW might want U.S. and European capital, but the different accounting standards, and the different requirements of the national capital markets, makes that more challenging.

ID4–8 The military has many users of its financial statements; these users need to be able to rely upon the integrity of the financial information in order to make good decisions. While these decisions are of a different nature than the typical commercial enterprise, they have significant financial impact on the enterprise. There are a couple of systematic errors that could develop from having to use poor financial information to make decisions. The first is that the decision makers decide to not use financial information at all due to the lack of faith in the numbers that are being generated. The second problem would be if the producers of the financial information (the military) resorts to overriding the financial systems in place and thereby “creating” numbers as they think they should be. In both cases the quality of the decisions made will be poor. This could lead to money being spent on the wrong projects and having the military unable to perform its function properly.

ID4–9 Borrowing money to pay dividends affects the right side of the accounting equation. Liabilities increase (Notes Payable, for example) while Stockholders’ Equity (through Retained Earnings) decreases. The combined effect is to leverage up a company’s balance sheet, causing it to finance assets with relatively more debt (and relatively less equity). The risks, of course, revolve around the increased debt on the company’s books and the consequent requirement to repay that debt. A blip in the company’s operations, or a downturn in general economic activity, and the company may be unable to repay its now-increased obligations. The benefit to companies such as Intel comes from cheap financing. When rates are low and borrowing money is relatively inexpensive and easy, companies can please their shareholders with a return on capital without incurring expensive debt. The low rates from 2006 to 2010 made such a move attractive.

ID4–10


The consolidation process of Umbro’s statements with those of Nike, Inc. had two major obstacles: 1) the conversion of pounds to U.S. dollars so that the statements can be presented in a common denomination; and 2) the conversion of Umbro’s IFRS-based statements (with its principles-based approach) to GAAP statements (with its rules-based approach). The currency conversion is relatively straight-forward and is something many multinational companies handle in their global operations. The second conversion, however, is trickier as management of Nike has to take statements in essentially a foreign language and translate them into U.S. GAAP—and do so in such a manner that share owners, analysts, regulators and other readers of Nike’s statements can understand the company’s business performance and see its results presented in a consistent manner.

ID4-11 Information obtained on a real-time basis is information that is available as soon as the underlying transaction has taken place. In the past financial systems might update once a week or once a month. In that environment managers were only able to see what sales were after the system was updated at the end of the month. Today, however, managers are able to access financial systems to see what sales are so far today. Today, virtually any transaction that hits the information system on a real-time basis can be tracked on a real-time basis. The most common items that are tracked this way are sales, inventory, accounts receivable and accounts payable. For service firms it might also include hours worked on each account. The items that are least likely to be available on a real-time basis are items that have very little day to day activity or that require manual input of some kind. This might include items like depreciation, bad debts expense and prepaid expenses. Real-time accounting improves the quality of the information available to the manager to make decisions. It is not always possible to wait until the end of the month to make certain decisions and, therefore, with real-time accounting better information is at the managers’ fingertips everyday to make those decisions.

ID4–12 a. According to the first footnote Google’s prepaid expenses also include amounts for revenue that will be shared with content providers. If Google pays the provider its share of advertising prior to the display of the content, the amount is held in prepaid until delivery of the content. Prepaid expenses, including these other amounts, totaled $1,710 and $2,132 million as of 12/31/11 and 12/31/12, respectively. b. At times, Google will book an expense before the cash is paid, “accruing” the liability to the balance sheet (to record the obligation to pay cash in the future) and recording the expense in the current period (and thereby reducing net income and, through Retained Earnings, equity). The Matching Principle is a driving force behind the accrual process. c. The Statement of Cash Flows shows that $3,273 million was spent on Capital Expenditures (Investing section) and that $0 was spent on Dividends (Financing section). When Google spent cash on long-term assets, total assets did not change, but cash was reduced and fixed assets were increased. The Statement of Cash Flows indicates that $287 million of cash was spent for the stock awards, reducing assets and equity (Financing section). d. The Balance Sheet shows $2,012 million of accounts payable as of 12/31/2012. If Google purchases inventory without paying cash at the time of the purchase, a trade accounts payable is


created. The large increase in payables from 2011 to 2012 is related to the Motorola acquisition; the cell phone manufacturing operation requires more inventory (purchased on account). e. Management represents that its internal controls are designed to provide reasonable assurance that assets are protected and that the accounting transactions are properly recorded. Further, management represents that it continually reviews and updates its internal controls. f. The statement of cash flows represents the in- and out-flows of cash during a period of time. The indirect presentation of the statement of cash flows begins with net income (a number based on accrual accounting that does not have anything to do with cash) and, through a series of steps, converts this number to cash from operating activities. The first step to convert net income to cash from operations is to add back depreciation expense (and any other non-cash charges). Depreciation reduced net income but it did not reduce the company’s cash balance; in order to get the net income number closer to a cash number, we add back any expenses that reduced profits but did not reduce cash. The second group of steps required to convert net income into a cash figure is to look at the changes to the operating (current) accounts, including accounts receivable. In Google’s case, accounts receivable increased over the period of fiscal 2012, meaning that the company booked sales but did not collect all the cash from those sales—the amount not collected from the sales is still sitting in the receivables account. In order to convert the profit number (which, after all, started from sales on the income statement) into a cash number, any sales that have not yet resulted in cash (meaning those sales still represented in the receivables balance) should be subtracted away.


CHAPTER 5 USING FINANCIAL STATEMENT INFORMATION BRIEF EXERCISE BE5–1 (a) ROE = Net Income/Average Stockholders’ Equity ROA = (Net Income +[Interest Expense (1-Tax Rate)])/ Average Total Assets Common Equity Leverage = Net Income/(Net Income + [Interest Expense(1-Tax Rate)]) Capital Structure Leverage = Average Total Assets/ Average Stockholders’ Equity Return on Sales = Net Income + [Interest Expense (1- Tax Rate)]/Net Sales Asset Turnover = Sales/Average Total Assets

Coke 27.7%

Pepsi 28.5%

11.2%

9.3%

96.7%

90.4%

2.55

3.41

19.4% .58

10.4% .89

Pepsi earns slightly more relative to its equity base and slightly less relative to its assets. Coke, however, has a higher return on sales (profits relative to sales). Pepsi shows higher use of leverage (Capital Structure Leverage ratio) and is much more efficient generating sales from its asset base (Asset Turnover ratio). (b) ROA x Common Equity Leverage x Capital Structure Leverage = ROE Coke: .112 x .967 x 2.55 = .277 (rounding) Pepsi: .093 x .904 x 3.41 = .285 (rounding) (c) Return on Sales x Asset Turnover = ROA Coke: .194 x .58 = .112 Pepsi: .104 x .89 = .093 (d) Pepsi’s advantage in producing a return for shareholders’ equity investment is driven by more aggressive use of leverage. Coke’s advantage in ROA is driven by its profitability for each dollar of sales. Given the higher returns on equity and the companies’ respective costs of capital, both companies are creating value for their shareholders.

1


BE5–2 (a) With $27.46 billion in 2012 revenues, J & J’s medical devices business is the largest. From 2010 to 2012, the pharmaceutical division showed the largest percentage growth at 13.4%. (b)

2010 2011 2012 52.2% 55.6% 55.6%

(c) Based on 2012 results, the consumer division generated the highest percentage of sales outside the U.S. at 65.1% (with medical devices next at 54.9%).

EXERCISES E5–1 Profitability Ratios: Return on Equity = Net Income ÷ Average Stockholders’ Equity 2012: $8,041 ÷ 49,280 2011: $6,490 ÷ 45,772

= .163 = .142

Return on Sales = (Net Income + [Interest Expense (1 – Tax Rate)]) ÷ Net Sales 2012: ($8,041 + [596 x (1 - .208)]) 2011: ($6,490 + [628 x (1 - .208)])

÷ $46,061 = .185 ÷ $43,218 = .162

Solvency Ratios: Current Ratio = Current Assets ÷ Current Liabilities 2012: $ 61,933 ÷ 2011: $ 57,231 ÷

$17,731 = $17,506 =

3.49 3.27

Leverage Ratios: Capital Structure Leverage Ratio = Average Total Assets ÷ Average Total Stockholders’ Equity 2012: $89,427 ÷ $49,280 = 1.81 2011: $84,112.5 ÷ $45,772 = 1.84 Overall, by examining the above computed ratios, it appears that Cisco would be a good investment. Profitability improved, while solvency and leverage were consistent.

2


E5–2 Profitability Ratios: Return on Equity = Net Income ÷ Average Stockholders’ Equity 2012: 2011:

$11,005 ÷ $12,942 ÷

48,557 = 47,670.5 =

.227 .272

Return on Sales = (Net Income + [Interest Expense (1 – Tax Rate)]) ÷ Net Sales 2012: 2011:

($11,005 + [94 x (1 - .26)]) ÷ $53,341 = .208 ($12,942 + [192 x (1 - .26)]) ÷ $53,999 = .242

Solvency Ratios: Current Ratio = Current Assets ÷ Current Liabilities 2012: 2011:

$31,358 $25,872

÷ ÷

$12,898 $12,028

= =

2.43 2.15

Leverage Ratios: Capital Structure Leverage Ratio = Average Total Assets ÷ Average Total Stockholders’ Equity

2012: 2011:

$77,735 $67,152.5

÷ $48,557 ÷ $47,670.5 = 1.41

=

1.60

Profitability numbers are down, solvency is up slightly, as is leverage. However, the company is still quite strong and would represent a good investment opportunity.

E5–3 Based on the information provided by Ginny’s Fashions, we can compute the following ratios: 1. Return on Equity = Net Income ÷ Average Stockholders’ Equity* 2014: $17,000 ÷ $31,000 = .548 2015: $18,000 ÷ $35,500 = .507 *2013 numbers needed for averages, so the above ratios use the ending number from the balance sheet 2. Return on Sales = (Net Income + [Interest Expense (1 – Tax Rate)]) ÷ Net Sales 2014: 2015:

$17,000 + [2,000 (1 - .3)] ÷ $70,000 = .263 $18,000 + [2,000 (1 - .3)] ÷ $74,000 = .262

3


E5–3

Concluded

3. Current Ratio = Current Assets ÷ Current Liabilities 2014: 2015:

$14,000 $21,000

÷ ÷

$7,000 $9,000

= =

2.0 2.33

4. Debt/Equity Ratio = Total Liabilities ÷ Total Stockholders’ Equity 2014: 2015:

$33,000 $33,000

÷ ÷

$31,000 $40,000

= =

1.065 .825

Generally, a lot more information is available to a bank loan officer to decide upon a long-term loan. However, given the limited information, I would support only a short-term loan rather than a long-term loan. The return on equity has declined while the return on sales has remained stable, indicating that the company is not making any gains in its profitability. The current ratio is encouraging, indicating the company’s short-term solvency is not in question. In terms of cash flows, the company’s cash flow from operating activities is positive but declining considerably. It seems it is financing its asset base partly from long-term loans and partly from its own operations. Overall, as a bank loan officer, my bank’s interest will be safely protected if I approve only a short-term loan and not a long-term loan.

E5–4 a. Profitability Ratios: Return on Equity

= = =

Net Income ÷ Average Stockholders' Equity $16,500 ÷ [($29,000 + $36,500) ÷ 2] .504

Return on Assets

= = =

(Net Income + [Interest Expense (1–Tax Rate)]) ÷ Average Total Assets ($16,500 + [$5,000 x (1- .34)]) ÷ [($81,000 + $99,000) ÷ 2] .22

Earnings per Share

= = =

Net Income ÷ Average Number of Common Shares Outstanding $16,500 ÷ [(2,000 shares + 2,000 shares) ÷ 2] $8.25

Return on Sales

= = =

(Net Income + [Interest Expense (1 – Tax Rate)])÷ Net Sales ($16,500 + [$5,000 x (1 - .34)]) ÷ $72,000 .275

Interest Coverage

= (Net Income Before Taxes and Interest Expense) ÷ Interest Expense = $30,000 ÷ $5,000 = 6.00

4


E5–4

Continued

Solvency Ratios: Current Ratio

= = =

Current Assets ÷ Current Liabilities ($9,000 + $12,000 + $18,000) ÷ $16,500 2.36

Quick Ratio

= = =

(Cash + Marketable Securities + Accounts Receivable) ÷ Current Liabilities ($9,000 + $12,000) ÷ $16,500 1.27

Activity Ratios: Receivables Turnover =

Inventory Turnover

Net Credit Sales ÷ Average Accounts Receivable = $72,000 ÷ [($9,000 + $12,000) ÷ 2] = 6.86 = = =

Cost of Goods Sold ÷ Average Inventory $30,000 ÷ [($15,000 + $18,000) ÷ 2] 1.82

= = =

Return on Equity – Return on Assets .504 – .22 .284

Capitalization Ratios: Financial Leverage

Debt/Equity

= = =

Total Liabilities ÷ Total Stockholders' Equity ($16,500 + $46,000) ÷ ($20,000 + $5,000 + $11,500) 1.71

Market Ratios: Price/Earnings Ratio

Dividend Yield

= = = = = =

Return on Investment = Market Price0

Market Price per Share ÷ Earnings per Share $36 ÷ $8.25 4.36

Dividends per Share ÷ Market Price per Share ($9,000 ÷ 2,000 shares) ÷ $36 .125 (Market Price1 – Market Price0 + Dividends per Share) ÷ = =

($36 – $30 + $4.50) ÷ $30 .35

5


E5–4

Concluded

b. Balance Sheet Cash Accounts receivable Inventory Long-lived assets (net) Total assets

2015

2014 9% 12% 18% 61% 100%

9% 11% 19% 61% 100%

Accounts payable Long-term liabilities Common stock Additional paid-in capital Retained earnings Total liabilities & stockholders' equity

17% 46% 20% 5% 12% 100%

15% 49% 25% 6% 5% 100%

Income Statement Sales Cost of goods sold Gross profit Operating expenses Income from operations Interest expense Income from continuing operations (before taxes) Income taxes Net income

100% 42% 58% 17% 41% 7% 34% 12% 22%

c.

The company is making a handsome return of 27.5% on sales. Its return on equity is more than 50%. Since the return on equity measures a company’s ability to use equity investor’s capital to generate net assets through operations, a return of more than 50% indicates that Ken’s Sportswear has exceptional earning power. The current ratio of Ken’s Sportswear has gone down from 2.58 for the year 2014 to 2.36 for the year 2015, but it is still very good. It is indicative of the fact that the company has more than twice the current assets to meet its short-term obligations. Just as the current ratio provides information about the short-term solvency position of the company, the debt/equity ratio provides information about the long-term solvency of the company. Ken’s Sportswear has a debt/equity of 1.79 in 2014 and 1.71 in 2015. This means that the company has more debt than equity. A ratio of 1 would indicate that 50% of the company is financed by the stockholders and the remaining 50% is financed by the creditors. Therefore, a ratio of more than 1 indicates that the company has more debt than equity, which often can be a cause for concern. In summary, the company does not have any solvency problems in the short run but could face solvency problems in the future if its return on equity, financial leverage, and other activity ratios decline.

6


E5–5 a. Current Ratio = Current Assets ÷ Current Liabilities 2010: $3,926 ÷ $2,095 = 1.87 2011: $4,309 ÷ $2,128 = 2.02 2012: $4,132 ÷ $2,344 = 1.76 b. Gross Profit as a % of Sales = Gross Profit ÷ Net Sales 2010: $5,889 ÷ $14,664 = .40 2011: $5,274 ÷ $14,549 = .36 2012: $6,171 ÷ $15,651 = .39 c.

Inventory Turnover = Cost of Goods Sold ÷ Average Inventory 2011: $9,275 ÷ [($1,615 + $1,620) ÷ 2] = 5.73 2012: $9,480 ÷ [($1,758 + $1,615) ÷ 2] = 5.62 Average Days Supply of Inventory = 365 ÷ Inventory Turnover 2011: 2012:

365 ÷ 5.73 = 63.7 days 365 ÷ 5.62 = 64.9 days

Accounts Payable Turnover = Cost of Goods Sold ÷ Average A/P 2011: $9,275 ÷ [($1,066 + $1,049) ÷ 2] = 8.77 2012: $9,480 ÷ [($1,144 + $1,066) ÷ 2] = 8.58 Average Days Accounts Payable = 365 ÷ A/P Turnover 2011: 2012:

365 ÷ 8.77 = 41.6 days 365 ÷ 8.58 = 42.5 days

d. Over the period shown, solvency has deteriorated, as best demonstrated by the drop in the current ratio and the longer time required to sell inventory and pay trade payables.

7


E5–6 a. 2013 2013 Ending Cash Balance

= = =

2013 Beginning Cash Balance + Change in Cash $0 + $78 $78

Change in Cash = Cash from Operating Activities + Cash from Investing Activities + Cash from Financing Activities $78 = Cash from Operating Activities – $400 + $800 Cash from Operating Activities = $(322) 2014 2014 Ending Cash Balance = $76 = 2014 Beginning Cash Balance =

2014 Beginning Cash Balance + Change in Cash Beginning Cash Balance – $2 $78

or 2014 Beginning Cash Balance

= =

2013 Ending Cash Balance $78

Change in Cash = Cash from Operating Activities + Cash from Investing Activities + Cash from Financing Activities $(2) = $(252) + Cash from Investing Activities + $400 Cash from Investing Activities = $(150) 2015 2015 Ending Cash Balance $156 Change in Cash

= 2015 Beginning Cash Balance + Change in Cash = $76 + Change in Cash = $80

Change in Cash = Cash from Operating Activities + Cash from Investing Activities + Cash from Financing Activities $80 = Cash from Operating Activities + $150 – $200 Cash from Operating Activities = $130 Beecham was using debt and/or equity during 2013 and 2014 to finance the acquisition of productive assets (i.e., investing activities) and to cover cash outflows from operating activities. During 2015, the company started generating cash from operating activities and used this cash—along with selling productive assets—to reduce its debt and/or equity and to build a cash reserve.

8


E5–6

Concluded

b. Other than at the beginning of 2013, the company always had a positive cash balance. From that standpoint the company was solvent throughout the three-year period. A more detailed analysis of Beecham's solvency, however, requires an analysis of the company's operating performance, financial flexibility, and liquidity. During 2013 and 2014, Beecham did not generate cash flows from operating activities. The company remained solvent by issuing additional debt or equity. Since the company was able to acquire additional debt or equity financing in 2013 and 2014 and was able to sell off assets during 2015, it appears that Beecham does have some financial flexibility. However, without having the associated balance sheets, it is not possible to adequately assess Beecham's financial flexibility and liquidity. Based upon the limited information provided, it appears that Beecham faced some potential solvency problems in 2013 and 2014, but was able to overcome these problems by issuing additional debt or equity.

E5–7 a. (1) Current Ratio = Current Assets ÷ Current Liabilities 2014: 2015:

$385,000 ÷ $170,000 = 2.26 $400,000 ÷ $460,000 = 0.87

(2) Quick Ratio = (Cash + Marketable Securities + Accounts Receivable) ÷ Current Liabilities 2014: 2015

($30,000 + $10,000 + $95,000) ÷ $170,000 = 0.794 ($15,000 + $225,000+ $90,000) ÷ $460,000 = 0.717

b. Receivables Turnover = Net Credit Sales ÷ Average Accounts Receivable 2014: 2015:

$780,000 ÷ [($100,000 + $95,000) ÷ 2] = 8.00 $800,000 ÷ [($95,000 + $90,000) ÷ 2] = 8.65

Number of Days Outstanding = 365 ÷ Receivables Turnover 2014: 2015: c.

365 ÷ 8.00 = 45.625 365 ÷ 8.65 = 42.197

Solvency refers to a company's ability to meet its debts as they come due. Current liabilities represent the debts that are expected to come due first. Therefore, to be solvent, a company must have sufficient cash or near-cash assets to meet these current liabilities. Total current assets is one measure of near-cash assets. As indicated by the change in the company's current ratio, the company has insufficient current assets available to settle its current liabilities. The company's quick ratio worsened during 2015. Given that the company has insufficient current assets and insufficient cash, marketable securities, and accounts receivable to meet its debts, it can probably be concluded that the company's overall solvency position is not strong.

9


E5–8 a. Return on Equity = Net Income ÷ Average Stockholders' Equity 2012: 2013: 2014: 2015:

$510,000 ÷ [($100,000 + $100,000) ÷ 2] = 5.10 $490,000 ÷ [($100,000 + $290,000) ÷ 2] = 2.51 $515,000 ÷ [($290,000 + $315,000) ÷ 2] = 1.70 $505,000 ÷ [($315,000 + $510,000) ÷ 2] = 1.22

It appears that the additional capital provided by the owners has not been used to generate net income. The company's net income has been relatively constant from 2012 to 2015. If the company had been effective at using the additional capital, the company's net income should have increased, and return on equity should have been relatively constant or increasing over time. However, if the company has used the additional capital for long-term projects, such as a new product, these projects may not generate any net income for several years. Once these projects begin generating income, the company's return on equity may increase to more appropriate levels. Therefore, the effectiveness of the company at using the owners' capital cannot be adequately evaluated without additional information. b. It appears that the company has overinvested in inventory. The inventory turnover and the days' supply of inventory for each year are:

Inventory turnover Days' supply

2012

2013

2014

2015

12.00 30.42

5.93 61.55

4.85 75.26

4.09 89.24

These ratios indicate that the company went from having one month's supply of inventory on hand to having almost three months of inventory on hand. It appears that the company has more inventory on hand than is warranted, given demand for the inventory. The company could reduce inventory on hand and invest the proceeds in income-producing assets such as marketable securities. Such a move would make the company more profitable and provide owners a greater return on their investments. This change in investment policy would increase the company's return on equity.

E5–9 a. Current Ratio = Current Assets ÷ Current Liabilities 2012: 2013: 2014: 2015:

$20,000 ÷ $8,000 = 2.500 $24,000 ÷ $13,000 = 1.846 $31,000 ÷ $25,000 = 1.240 $35,000 ÷ $30,000 = 1.167

Debt/Equity Ratio = Total Liabilities ÷ Total Stockholders' Equity 2012: 2013: 2014: 2015:

($8,000 + $15,000) ÷ ($20,000 + $10,000) = 0.767 ($13,000 + $35,000) ÷ ($20,000 + $20,000) = 1.200 ($25,000 + $40,000) ÷ ($20,000 + $32,000) = 1.250 ($30,000 + $40,000) ÷ ($20,000 + $38,000) = 1.207

10


E5–9

Continued

Return on Assets = (Net Income + [Interest Expense (1 – Tax Rate)]) ÷ Average Total Assets 2012: 2013: 2014: 2015:

($13,000 + [$2,000 (1 - .3)]) ÷ [($53,000] = 0.272 ($14,000 + [$4,000 (1 - .3)]) ÷ [($53,000 + $88,000) ÷ 2] = 0.238 ($21,000 + [$5,000 (1 - .3)]) ÷ [($88,000 + $117,000) ÷ 2] = 0.239 ($24,000 + [$5,000 (1 - .3)]) ÷ [($117,000 + $128,000) ÷ 2] = 0.224

b.

c.

2015

2014

2013

2012

Current assets Noncurrent assets Total assets

27.34% 72.66 100.00%

26.50% 73.50 100.00%

27.27% 72.73 100.00%

37.74% 62.26 100.00%

Current liabilities Long-term liabilities Capital stock Retained earnings Total liabilities and stockholders' equity

23.44% 31.25 15.62 29.69

21.37% 34.19 17.09 27.35

14.77% 39.77 22.73 22.73

15.09% 28.30 37.74 18.87

100.00%

100.00%

100.00%

100.00%

Solvency measures a company's ability to meet its debts as they come due. The current ratio provides one measure of a company's solvency. Based upon this ratio, Lotechnic has sufficient current assets to meet its current obligations. However, the trend in its current ratio indicates that the company's excess of current assets over current liabilities is decreasing. Therefore, the company has relatively fewer current assets available to meet its current obligations. This trend indicates that Lotechnic Enterprises' solvency position may be worsening. The debt/equity ratio provides an indication of a company's capitalization, which, in turn, indicates how risky a company is. Lotechnic is relying increasingly on debt relative to stockholders' equity to finance operations. At some point in time, the company will have to repay this debt. The company will either have to repay this debt by (1) generating cash from operations, (2) selling assets, (3) borrowing additional cash, or (4) acquiring cash by issuing stock. From the statement of cash flows, the cash generated from operations has been decreasing and is now negative; therefore, it appears that the company cannot rely on operations to generate cash. The statement of cash flows also indicates that the company has been using cash for investment purposes every year. This implies that the company may have some assets that it could sell. But if these assets are used in operations, the company's operations may be adversely affected by selling them. Since total assets equal the sum of total liabilities and stockholders' equity, the proportion of total liabilities to the sum of total liabilities and stockholders' equity reported on the common-size balance sheet equals the proportion of total liabilities to total assets. This measure indicates the proportion of total assets (based upon book value) that would have to be sold to satisfy all the company's obligations. To meet its obligations, Lotechnic Enterprises would have to sell approximately 55% of its total assets, which would virtually decimate its asset base. Based upon the trend in the current ratio, the debt/equity ratio, cash flows from operations, and the proportion of total liabilities to total assets, it appears that Lotechnic Enterprises may face severe solvency problems as its long-term debt matures.

11


E5–9

Concluded

Earning power is defined as a company's ability to increase its wealth through operations and to generate cash from operations. Earning power and solvency are closely related. A company must have adequate resources to generate wealth. If a company experiences solvency problems, it will most likely have to divert its resources to paying its obligations. Therefore, due to its solvency problems, Lotechnic Enterprises may not have strong earning power. Although Lotechnic's net income has increased every year, the company's effectiveness at managing capital, as indicated by ROA, has decreased every year. This trend indicates that the company may have limited earning power. This conclusion is also supported by the trend in the company's cash flows from operations. It must be remembered, however, that this analysis is based on very limited information. To adequately analyze a company, additional information would be needed. Complete financial statements, financial information for similar companies, and general economic information should all be considered when analyzing a company's earning power and solvency position.

E5–10 Transaction

Quick Ratio

Current Ratio

Debt/Equity Ratio

(1) – – + (2) NE NE + (3) – – – (4) – – +a b (5) + + – (6) + + – __________________ a Wage Expense would be closed into Retained Earnings at the end of the accounting period as part of the closing process. Thus, recording wage expense would decrease stockholders' equity, and thereby increase the debt/equity ratio. b

This transaction would increase both Sales and Cost of Goods Sold. Both of these accounts would be closed into Retained Earnings as part of the closing process. Since the sales price exceeds the cost of the inventory, the net effect of this transaction would be to increase Retained Earnings. Thus, total stockholders' equity would increase, and thereby decrease the debt/equity ratio.

E5–11 a. Debt/Equity Ratio

= Total Liabilities ÷ Total Stockholders' Equity = ($130,000 + $150,000) ÷ $200,000 = 1.40

b. The maximum debt that Montvale can have outstanding is 1.5 times its total stockholders' equity. This means that the total debt Montvale can have outstanding is $300,000 (1.5  $200,000). Since Montvale already has $280,000 of outstanding debt, it can incur an additional $20,000 in debt without violating its debt covenant.

12


E5–11 c.

Concluded

The minimum level of stockholders' equity that Montvale can have is total debt divided by 1.5. This means that the total stockholders' equity Montvale can have is $186,667 ($280,000 ÷ 1.5). Since Montvale currently has $200,000 of stockholders' equity, and since dividends decrease stockholders' equity, the maximum dividend that Montvale can declare is $13,333.

d. If Montvale had declared, but not paid, a $20,000 dividend prior to obtaining the loan, then the $20,000 is already included in the current liabilities reported on the balance sheet. Paying the dividend would decrease both current assets and current liabilities by $20,000. Thus, the debt/equity ratio after paying the $20,000 would be 1.3 ([($130,000 – $20,000) + $150,000] ÷ $200,000). Since this ratio is less than the maximum debt/equity ratio allowed under the debt covenant, Montvale could pay the $20,000 dividend without violating its debt covenant.

E5–12 a.

2010: 2011: 2012:

$2,408 ÷ $2,610 ÷ $2,897 ÷

b. Price Earnings Ratio

2011: 2012:

$4,946 = 48.7% $5,503 = 47.4% $5,465 = 53.0% = Market Price per Share ÷ Earnings per Share = Market Price per Share ÷ (Net Income ÷ Average Number of Common Shares Outstanding)

$100.33 ÷ {$5,503 ÷ [(1,066 + 1,032) ÷ 2]} = 19.11 $ 88.21 ÷ {$5,465 ÷ [(1,032 + 1,010) ÷ 2]} = 16.49

Dividend Yield = Dividends per Share ÷ Market Price per Share 2010: 2011: 2012:

($2,408 ÷ 1,066 shares) ÷ $ 76.76 = .0294 ($2,610 ÷ 1,032 shares) ÷ $100.33 = .0252 ($2,897 ÷ 1,010 shares) ÷ $ 88.21 = .0325

Stock Price Return= Market Price0 2011: 2012:

(Market Price1 – Market Price0 + Dividends per Share) ÷

($100.33 – $76.76 + $2.53) ÷ $76.76 = 34.0% ($88.21 – $100.33 + $2.87) ÷ $100.33 = - 9.22%

13


E5–12 c.

Concluded

An investment in McDonald’s stock from 2010 to 2012 would have provided a very good return for investors in 2011, but a loss in 2012. The dividend yield was solid over this time period but the capital appreciation from the stock price was strong in the first year but negative in the second.

E5–13 a. (1)

Earnings per Share = Outstanding

Net Income ÷ Average Number of Common Shares = =

$6,168 ÷ [(3,041 + 3,026.6) ÷ 2] $2.03

(2)

Price/Earnings

= = =

Market Price per Share ÷ Earnings per Share $40.94 per Share ÷ $2.03 per Share 20.17

(3)

Dividend Yield

= = =

Dividends per Share ÷ Market Price per Share ($5,173 ÷ 3,033.8 Shares) ÷ $40.94 per Share .042

(4)

Stock Price Return = (Market Price1 – Market Price0 + Dividends per Share) ÷ Market Price0 = ($40.94 – $38.30 + $1.71) ÷ $38.30 = 11.4%

b. Return on equity equals net income divided by average stockholders' equity. Thus, only those items that affect net income or stockholders' equity would affect a company's return on equity. Declaring and paying dividends would decrease stockholders' equity but would not affect net income. Thus, the return on equity ratio would increase for item (1). A change in the market price of a company's stock would not affect return on equity. Repurchasing common stock would decrease stockholders' equity, but it would not affect net income. Thus, the return on equity ratio would increase for item (3).

E5–14

a. Based on the 2012 numbers the Medical devices unit generated the highest operating profits as a percentage of sales at 26.2%, slightly ahead of the Pharmaceutical unit at 24.0% b. All three areas experience a drop in the total dollars of operating profits, but the percentage drop was the smallest in the Medical devices unit.

14


E5–15 The formulas that are used by the ROEmodel are as follows: ROE

=

ROA

*

Common Equity Leverage

=

Profit Margin

*

Capital Structure Leverage

and ROA

*

Asset Turnover

The first item that stands out is the steady increase in ROE from 2013 through 2015. This is being driven completely by the increase in Capital Structure Leverage. Both ROA and Common Equity Leverage have been decreasing over the three years. ROA has dropped because of the drop in Asset Turnover. From 2013 through 2015 both ROA and common equity leverage have decreased slightly. But the significant increase in capital structure leverage has more than offset the declines in the other components so that ROE has risen in both 2014 and 2015. These ratios show that the improvement in ROE is due primarily to increased debt leverage used by the company. ROA has decreased because of the decrease in asset turnover. ROA can be computed by multiplying profit margin x asset turnover. In this example, while the profit margin has increased from 2013 through 2015 asset turnover has deteriorated significantly. This reduction in ROA could be due to slowing sales or a large increase in the asset base of the company. With the increase in the capital structure leverage the most likely reason is an increase in the assets of the company.

E5–16 The formulas that are used by the ROE model are as follows: ROE

=

ROA

*

Common Equity Leverage

=

Profit Margin

*

Capital Structure Leverage

and ROA

*

Asset Turnover

From 2013 through 2015 both the common equity leverage and the capital structure leverage have remained fairly stable. ROA has dropped significantly which has caused the decline in ROE. In this example asset turnover has remained stable from 2013 through 2015. During this same time the company’s profit margin has declined precipitously. Therefore the drop in LBS Products’ profit margin has caused its ROA to fall which in turn caused its ROE to also fall.

E5–17 a. ROE = ROA * Common Equity Leverage therefore, ROE = .095 * 0.685 * 2.50 = 16.3% b. ROE = ROA * Common Equity Leverage ROE will equal .095 * 0.685 * 2.75 = 17.9%

*

*

Capital

Structure

Capital Structure Leverage, therefore 2016

c. ROA = Profit Margin * Asset Turnover and ROE = ROA * Common Equity Leverage * Capital Structure Leverage; therefore, 0.625 * .16 = .1; ROE = .1 * .685 * 2.50 = 17.1% 15

Leverage;


E5–18

Sales Net income Total assets Total shareholders’ equity

2012 $67,224 10,853 121,347 64,826

Historic Relationship NA 16.14% of Sales 1.81 times Sales 53.4% of Assets

If Sales are projected to grow at 8% and if the historic relationships maintain, the projected 2013 financial statements can be summarized below:

Sales Net income

2013 $72,602 11,718

Total assets Total shareholders’ equity

131,410 70,173

16


PROBLEMS P5–1 (1) Current Ratio

(2) Quick Ratio Liabilities

=

Current Assets ÷ Current Liabilities = $557 ÷ $341 = 1.63 =

(Cash + Marketable Securities + Accounts Receivable) ÷ Current

= =

($109 + $221) ÷ $341 0.97

(3) Receivables Turnover

= Net Credit Sales ÷ Average Accounts Receivables = $1,100 ÷ ((221 + 235) ÷ 2) = 4.82

(4) Interest Coverage =

(Net Income Before Taxes + Interest Expense) ÷ Interest Expense = ($-341 + $3) ÷ $3 = (112.7)

(5) Return on Assets =

(Net Income + [Interest Expense (1 – Tax Rate)]) ÷ Avg. Total Assets = NA (need Total Assets)

(6) Inventory Turnover

= = =

Cost of Goods Sold ÷ Avg. Inventory $897 ÷ $187.5 4.78

(7) Return on Equity

= = =

Net Income ÷ Avg. Stockholders' Equity $-341 ÷ $562 (.61)

Receivables and Inventory turnover ratios are at respectable rates, but the loss on the income statements hurts the other ratios. The current ratio remains solid.

P5–2 a. Return on Equity = Net Income ÷ Average Stockholders' Equity 2014: $14,000 ÷ [($34,000 + $38,000) ÷ 2] = .389 2015: $25,000 ÷ [($38,000 + $51,000) ÷ 2] = .562 Return on Assets = (Net Income + [Interest Expense(1 – Tax Rate)]) ÷ Average Total Assets 2014: ($14,000 + [$1,000 (1 - .30)]) ÷ [($49,000 + $55,000) ÷ 2] = .283 2015: ($25,000 + [$5,000 (1 - .342)]) ÷ [($55,000 + $113,000) ÷ 2] = .337 Common Equity Leverage = Net Income ÷ [Net Income + (Interest Expense (1 – Tax Rate)] 2014: $14,000 ÷ [$14,000 + ($1,000 (1 - .30)] = .952 2015: $25,000 ÷ [$25,000 + ($5,000 (1 - .342)] = .883

17


P5–2

Concluded

Capital Structure Leverage = Average Total Assets ÷ Average Shareholders’ Equity 2014: [($49,000 + $55,000) ÷ 2] ÷ [($34,000 + $38,000) ÷ 2] = 1.444 2015: [($55,000 + $113,000) ÷ 2] ÷ [($38,000 + $51,000) ÷ 2] = 1.888 Profit Margin = (Net Income + [Interest Expense(1 – Tax Rate)]) ÷ Net Sales 2014: ($14,000 + [$1,000 (1 - .30)]) ÷ $45,000 = .327 2015: ($25,000 + [$5,000 (1 - .342)]) ÷ $70,000 = .404 Asset Turnover = Sales ÷ Average Total Assets 2014: $45,000 ÷ [($49,000 + $55,000) ÷ 2] = .865 2015: $70,000 ÷ [($55,000 + $113,000) ÷ 2] = .833 b. Return on Equity

= =

Return on Assets =

($25,000 + $4,000 interest saved) ÷ {[($38,000 + $40,000) + ($51,000 + $40,000 + $4,000 interest saved)] ÷ 2} .335

[($25,000 + $4,000 interest saved) + $1,000] ÷ {[$55,000 + ($113,000 + $4,000 cash saved on interest)] ÷ 2} = .349

Common Equity Leverage = ($25,000 + $4,000 interest saved) ÷ [($25,000 + $4,000 interest saved) + ($1,000 (1 – .34)] = .978 Capital Structure Leverage = [($55,000 + ($113,000 + $4,000 interest saved)) ÷ 2] ÷ [$38,000 + ($51,000 + $4,000 interest saved) ÷ 2] = 1.849 Profit Margin = [($25,000 + $4,000 interest saved) + ($1,000 (1 – .34)] ÷ $70,000 = .424 Asset Turnover = $70,000 ÷ [($55,000 + ($113,000 + $4,000 interest saved)) ÷ 2] = .814 c.

The company appears stronger by issuing equity rather than debt if one examines return on assets, common equity leverage and profit margin. However, based on return on equity, capital structure leverage and asset turnover, the company appears stronger by issuing debt rather than equity. The company can effectively manage additional debt capital to the benefit of its stockholders, but it does not effectively manage additional equity capital to the benefit of its stockholders. In fact, the company manages equity capital to the detriment of its stockholders. Therefore, based upon this limited ratio analysis, it appears that the company made the correct decision to issue debt rather than equity. Remember, though, that the company may want or need to consider other factors in deciding whether to issue debt or equity. It may want to consider the effect of the alternative financing arrangements on existing debt covenants, incentive compensation schemes, taxes, and so forth.

18


P5–3 a.

Dollar Change Assets Current assets: Cash Short-term marketable securities Accounts receivable Inventory Other current assets Total current assets Property, plant, & equipment Other assets Total assets Liabilities and Stockholders' Equity Current liabilities: Short-term borrowings Accounts payable Wages payable Dividend payable Income taxes payable Other current liabilities Total current liabilities Long-term debt Contributed capital Retained earnings Total liabilities and stockholders' equity

Percentage Change

$

(1,904)

(32.2%)

$

691 (262) 344 (78) (1,209) 0 1,948 738

70.9% (7.3%) 15.0% (5.4%) (8.5%) 0.0% 16.7% 2.2%

(1,510) 63 135 (1) (118) 890 (541) 43 47 1,189

(99.2%) 5.6% 16.8% (0.2%) (45.0%) 19.0% (6.1%) 0.4% 3.5% 8.0%

738

2.2%

$

$

$

19


P5–3

Concluded

b.

2012

2011

Change

Assets Current assets: Cash Short-term marketable securities Accounts receivable Inventory Other current assets Total current assets Property, plant, and equipment Other assets Total assets

11.68% 4.84% 9.70% 7.69% 3.99% 37.90% 22.56% 39.54% 100.00%

17.60% 2.90% 10.69% 6.83% 4.31% 42.33% 23.05% 34.62% 100.00%

(33.64%) 66.90% (9.26%) 12.59% (7.43%) (10.44%) (2.13%) 14.21%

Liabilities and Stockholders' Equity Current liabilities: Short-term borrowings Accounts payable Wages payable Dividend payable Income taxes payable Other current liabilities Total current liabilities Long-term debt Contributed capital Retained earnings Total liabilities and stockholders' equity

00.03% 3.45% 2.73% 1.57% 0.42% 16.18% 24.39% 32.66% (3.82%) 46.77% 100.00%

4.52% 3.34% 2.39% 1.61% 0.78% 13.90% 26.54% 33.25% (4.05%) 44.26% 100.00%

(99.34%) 3.29% 14.23% (2.48%) (46.15%) 16.49% (8.07%) (1.77%) (5.68%) 5.67%

(Other Current Assets and Other Current Liabilities are rounded to result in a value of 100% in column addition.) c.

Common-size financial statements provide relative comparisons of account balances rather than absolute comparisons of account balances. Absolute comparisons only provide information about whether an account balance has increased or decreased. Alternatively, relative comparisons provide information about whether an account balance has increased or decreased relative to a benchmark measure. This relative comparison allows financial statement users to determine more easily if a company is altering the composition of its assets, liabilities, or stockholders' equity. Relative comparisons of account balances may also provide users with insights into why account balances are changing.

P5–4 (1)

Return on equity measures a company's effectiveness at managing equity investments. Return on equity is calculated as net income divided by average stockholders' equity. 2014: 2015:

$515,000 ÷ [($450,000 + $755,000) ÷ 2] = .855 $510,000 ÷ [($755,000 + $795,000) ÷ 2] = .658

The company generated returns on its owners' investments in excess of 65%, which appears to be rather substantial. However, without being able to compare Gidley Electronics' performance to 20


industry averages, it is difficult to conclude whether the company is really effective in managing the owners' capital.

21


P5–4 (2)

Continued Return on equity measures a company's effectiveness at managing owners' investments, while return on assets measures a company's effectiveness at managing all investments, both debt and equity. The excess of return on equity over return on assets indicates a company's effectiveness at using debt to generate returns for the owners. This measure is called financial leverage. Since financial leverage is calculated using return on assets, the first step is to calculate return on assets. Return on assets is calculated as the sum of net income and [interest expense x (1- tax rate)] divided by average total assets. Gidley's return on assets for 2014 and 2015 is: 2014: 2015:

($515,000 + [$165,000 (1 - .40)]) ÷ [($1,450,000 + $1,470,000) ÷ 2] = .421 ($510,000 + [$150,000 (1 - .40)]) ÷ [($1,470,000 + $1,465,000) ÷ 2] = .409

Gidley's financial leverage for 2014 and 2015 is, therefore: 2014: 2015:

.855 – .421 = .434 .658 – .409 = .249

The company is using debt to the benefit of its equity owners. The positive leverage indicates that proceeds from debt are generating sufficient profits to provide a return for the equity owners. In other words, the return from using debt exceeds its cost. (3)

The current ratio measures whether a company has sufficient current assets to meet its current liabilities. The current ratio equals current assets divided by current liabilities. Gidley's current ratio for 2014 and 2015 is: 2014: 2015:

$1,010,000 ÷ $275,000 = 3.673 $980,000 ÷ $290,000 = 3.379

Gidley Electronics' current assets are over three times greater than its current liabilities. The company therefore appears to have no solvency problems. However, the company may be unable to convert some of its current assets to cash quickly enough to meet some of its current liabilities. Another measure of solvency that compares near-cash assets to current liabilities is the quick ratio. The quick ratio equals the sum of cash, marketable securities, and accounts receivable divided by current liabilities. Gidley's quick ratio for 2014 and 2015 is: 2014: 2015:

($115,000 + $220,000 + $400,000) ÷ $275,000 = 2.673 ($110,000 + $175,000 + $350,000) ÷ $290,000 = 2.190

Gidley Electronics appears to have sufficient cash and near-cash assets available to meet its current obligations. Therefore, the company should have no significant short-term solvency problems.

22


P5–4 (4)

Concluded The price/earnings ratio measures the sensitivity of stock prices to changes in earnings. This ratio is calculated by dividing the market price per share by earnings per share. Since this ratio uses earnings per share in the calculations, the first step is to calculate earnings per share. Earnings per share is calculated by dividing net income by the average number of common shares outstanding during the year. Gidley's earnings per share for 2014 and 2015 are: 2014: 2015:

$515,000 ÷ [(17,000 + 17,000) ÷ 2] = $30.29 $510,000 ÷ [(17,000 + 22,000) ÷ 2] = $26.15

Gidley's price/earnings ratio for 2014 and 2015 is: 2014: 2015:

$69.00 ÷ $30.29 = 2.278 $54.00 ÷ $26.15 = 2.065

It appears that the price of Gidley Electronics' stock is rather sensitive to changes in earnings. A change in earnings per share should cause the market price to change by approximately twice the change. To obtain a better idea of how sensitive the company's stock is to changes in earnings, the company's price/earnings ratio should be compared to industry averages. (5)

The average number of days accounts receivable are outstanding is calculated as 365 days divided by accounts receivable turnover. The accounts receivable turnover is, in turn, calculated by dividing net credit sales by average accounts receivable. Gidley's accounts receivable turnover for 2014 and 2015 is: 2014: 2015:

$3,010,000 ÷ [($400,000 + $400,000) ÷ 2] = 7.525 $2,450,000 ÷ [($400,000 + $350,000) ÷ 2] = 6.533

The number of days outstanding for receivables during 2014 and 2015 is: 2014: 2015:

365 ÷ 7.525 = 48.505 days 365 ÷ 6.533 = 55.870 days

The average number of days accounts receivable are outstanding increased slightly. Therefore, customers are not paying their open receivables as quickly as before. If this problem persists, Gidley may have to consider more stringent credit and/or collection policies.

P5–5 a. Return on equity provides a measure of a company's effectiveness at managing the owners' capital. The formula for calculating return on equity is net income divided by average stockholders' equity. The 2015 return on equity for Hathaway Toy Company and Yakima Manufacturing would be: Hathaway: Yakima:

$875,000 ÷ [($1,585,000 + $2,460,000) ÷ 2] = .433 $755,000 ÷ [($70,000 + $825,000) ÷ 2] = 1.687

Note: Beginning Stockholders' Equity = Ending Stockholders' Equity – Net Income Based on return on equity, Yakima Manufacturing has been almost four times more efficient than Hathaway Toy Company at managing owners' capital.

P5–5

Concluded 23


b. Return on assets provides a measure of a company's effectiveness at managing all investors' capital. The formula for calculating return on assets is the sum of net income and tax-adjusted interest expense divided by average total assets. The 2015 return on assets for Hathaway Toy Company and Yakima Manufacturing would be: Hathaway: Yakima:

($875,000 + $0) ÷ [($1,825,000 + $2,700,000) ÷ 2] = .387 ($755,000 + $195,000[1 – 0.00]) ÷ [($1,945,000 + $2,700,000) ÷ 2] = .409

Note: Beginning Total Assets

= =

Ending Total Assets – Net Income Total Liabilities and Stockholders' Equity – Net Income

Based on return on assets, Yakima Manufacturing and Hathaway Toy Company are essentially equal in their abilities to manage all investors' capital.

c.

Earnings per Share = Net Income ÷ Average Number of Common Shares Outstanding Hathaway: Yakima: Note:

$875,000 ÷ [(80,000 + 80,000) ÷ 2] = $10.94 $755,000 ÷ [(35,000 + 35,000) ÷ 2] = $21.57

The number of shares outstanding equals the balance in Common Stock divided by $10 par value per share.

d. Yes, stockholders are realizing a return on their capital of 168.7% (from Part [a]), while debtholders are realizing only a return on their capital of approximately 10.5% ($195,000 of interest expense ÷ $1,850,000 balance in mortgage payable). This difference in returns is due to the company using debt rather than equity to finance operations. Since the debtholders are only entitled to interest, any earnings from operations in excess of interest accrues to the stockholders. Thus, Yakima Manufacturing has efficiently used debt to benefit its stockholders.

P5–6 In order to consider an investment in Goodyear, let us first compute the following ratios: 1. Return on Equity

=

Net Income ÷ Average Stockholders’ Equity

2011: $ 321 ÷ [($1,505 + $1,624) ÷ 2] = 20.5% 2012: $183 ÷ [($1,624 + $1,159) ÷ 2] = 13.2% 2. Return on Sales 2011: 2012:

=

(Net Income + [Interest Expense (1 – Tax Rate)]) ÷ Net Sales

($321 + [$330 (1 – 0.30)]) ÷ $22,767 = 2.4% ($183 + [$357 (1 – 0.30)]) ÷ $20,992 = 2.1%

24


P5–6

Concluded

3. Current Ratio =Current Assets ÷ Current Liabilities 2011: 2012:

$9,812 $8,498

÷ ÷

$5,929 $5,322

= =

1.65 1.60

4. Debt/Equity Ratio = Total Liabilities ÷ Total Stockholders’ Equity 2011: $16,005 2012: $15,814

÷ ÷

$1,624 $1,159

= =

9.86 13.64

Generally, an equity investor would have much more information available to make a decision than what is provided by Goodyear. However, based on the information provided, an equity investment in Goodyear would be unwise. The company is showing a lower ROE and return on sales, has high leverage on its balance sheet and is generating a drop in cash balances. Until Goodyear can turn around its operations, it is not an attractive opportunity.

P5–7 a. Return on Equity = Net Income ÷ Average Stockholders' Equity Robotronics: Technology:

$610,000 ÷ [($1,005,000 + $1,005,000) ÷ 2] = .607 $1,675,000 ÷ [($1,440,000 + $1,440,000) ÷ 2] = 1.163

Based on return on equity, Technology is almost twice as efficient as Robotronics at managing the stockholders' capital. If unusual items were not considered, return on equity for each company would be: Robotronics: Technology:

$610,000 ÷ [($1,005,000 + $1,005,000) ÷ 2] = .607 ($1,675,000 – $1,300,000) ÷ [($1,440,000 + $1,440,000) ÷ 2] = .260

Technology now appears to be considerably worse than Robotronics at managing the stockholders' capital. Including unusual items in calculating return on equity does provide a more complete measure of how efficiently a company managed its stockholders' equity in the current year. However, since unusual items are, by definition, items that occur infrequently, these items do not indicate a company's continued ability to efficiently manage the stockholders' capital. Thus, unusual items probably should not be used to calculate return on equity.

25


P5–7

Concluded

b. Financial leverage indicates how effectively a company uses debt for the benefit of stockholders. Financial leverage equals return on equity less return on assets. Thus, return on assets must be calculated before calculating financial leverage. Return on Assets = (Net Income + Interest Expense (net of tax)) ÷ Average Total Assets Robotronics: Technology:

($610,000 + $100,000) ÷ [($3,360,000 + $3,360,000) ÷ 2] = .211 ($1,675,000 + $175,000) ÷ [($1,870,000 + $1,870,000) ÷ 2] = .989

Financial Leverage = Return on Equity – Return on Assets Robotronics: Technology:

.607 – .211 = .396 1.163 – .989 = .174

From this analysis, Robotronics is approximately twice as effective as Technology at using debt to generate returns for its stockholders. If unusual items are not considered, the return on assets for each company would be: Robotronics: Technology:

($610,000 + $100,000) ÷ [($3,360,000 + $3,360,000) ÷ 2] = .211 [($1,675,000 – $1,300,000) + $175,000] ÷ [($1,870,000 + $1,870,000) ÷ 2] = .294

Therefore, the financial leverage of the two companies would be: Robotronics: Technology:

.607 – .211 = .396 .260 – .294 = –.034

If unusual items are not considered, Technology has negative financial leverage. That means that Technology is not generating a large enough return on its debt to even cover the interest expense. Thus, Technology is using debt to the detriment of its stockholders. It appears, therefore, that unusual items can affect the conclusions one draws when analyzing a company.

26


P5–8 Return on Sales .08 Net Sales

= = =

Cost of Goods Sold

Net Income ÷ Net Sales $25,000 ÷ Net Sales $312,500 =

Net sales x (1 – Gross Margin Percentage) = $312,500 x (1 – 40%) = $187,500

Net Income $25,000 Expenses =

= Net Sales – Cost of Goods Sold – Expenses = $312,500 – $187,500 – Expenses $100,000

Inventory Turnover 5 Ending Inventory

= = =

Cost of Goods Sold ÷ Average Inventory $187,500 ÷ [($0 + Ending Inventory) ÷ 2] $75,000

Receivables Turnover

= =

Net Credit Sales ÷ Average Accounts Receivable $312,500 ÷ [($0 + Ending Accounts Receivable)÷2) Ending $78,125

Quick Ratio

=

.5 Cash

= =

(Cash + Accounts Receivable + Marketable Securities) ÷ Current Liabilities (Cash + $78,125 + $0) ÷ $200,000 $21,875

8 Accounts Receivable

=

Tumwater Canyon Campsites Income Statement For the Year Ended December 31, 2015 Sales Cost of goods sold Gross profit Expenses Net income

$312,500 187,500 $125,000 100,000 $ 25,000

Tumwater Canyon Campsites Statement of Current Assets and Liabilities December 31, 2015 Current assets Cash Accounts receivable Inventory Total current assets

$ 21,875 78,125 75,000 $175,000

Current liabilities Accounts payable

$200,000

Total current liabilities

________ $200,000

27


P5–9 a. Mountain-Pacific Railroad Common-Size Balance Sheet December 31, 2015 and 2014 2015 Assets Current assets: Cash Short-term marketable securities Accounts receivable Inventory Prepaid expenses Total current assets Long-term investments Property, plant, and equipment Accumulated depreciation Total assets Liabilities and Stockholders' Equity Current liabilities: Accounts payable Wages payable Dividends payable Income taxes payable Current portion of long-term debt Total current liabilities Mortgage payable Common stock ($10 par value) Additional paid-in capital Retained earnings Total liabilities and stockholders' equity

2014

Dollar

%

10,000 125,000 500,000 200,000 50,000 $ 885,000 225,000 430,000 (65,000) $ 1,475,000

0.68% 8.47% 33.90% 13.56% 3.39% 60.00% 15.25% 29.15% (4.40%) 100.00%

$

$

10,000 5,000 125,000 50,000 100,000 290,000 350,000 200,000 135,000 500,000 1,475,000

$

$

$

29

Dollar

%

312,000 120,000 150,000 210,000 75,000 $ 867,000 225,000 540,000 (100,000) $ 1,532,000

20.36% 7.83% 9.79% 13.71% 4.90% 56.59% 14.69% 35.25% (6.53%) 100.00%

0.68% 0.34% 8.47% 3.39% 6.78% 19.66% 23.73% 13.56% 9.15% 33.90%

$

50,000 2,000 5,000 35,000 175,000 267,000 450,000 110,000 95,000 610,000

3.26% 0.13% 0.33% 2.29% 11.42% 17.43% 29.37% 7.18% 6.20% 39.82%

100.00%

$ 1,532,000

100.00%

$


P5–9

Continued Mountain-Pacific Railroad Common-Size Income Statement For the Years Ended December 31, 2015 and 2014 2015

Revenue: Net cash sales Net credit sales Total revenue Cost of goods sold: Beginning inventory Net purchases Cost of goods available for sale Less ending inventory Cost of goods sold Gross profit Selling & administrative expenses: Depreciation expense General selling expenses General administrative expenses Total selling & administrative exp. Income from operations Interest expense Income from continuing operations (before taxes) Income taxes Income before unusual items Unusual loss (net of tax benefit of $60,000) Net income

2014

Dollar

%

Dollar

%

1,955,000 4,150,000 6,105,000

32.02% 67.98% 100.00%

$

2,775,000 1,410,000 4,185,000

66.31% 33.69% 100.00%

210,000 4,005,000 4,215,000 200,000 4,015,000 2,090,000

3.44% 65.60% 69.04% 3.28% 65.76% 34.24%

$

300,000 2,475,000 2,775,000 210,000 2,565,000 1,620,000

7.17% 59.14% 66.31% 5.02% 61.29% 38.71%

$

75,000 575,000 480,000

1.23% 9.42% 7.86%

$

90,000 600,000 420,000

2.15% 14.34% 10.04%

$ $

1,130,000 960,000 50,000

18.51% 15.73% 0.82%

$ $

1,110,000 510,000 65,000

26.53% 12.18% 1.55%

$

14.91% 5.08% 9.83%

$

$

910,000 310,000 600,000

$

445,000 151,000 294,000

10.63% 3.61% 7.02%

$

115,000 485,000

1.88% 7.95%

$

0 294,000

0.00% 7.02%

$ $ $ $ $ $

$

$ $ $

By looking at the common-size balance sheets and income statements, we can observe the following: 1. The proportion of current assets to total assets has increased slightly from 57% to 60%. The composition of current assets has changed dramatically. Cash balance has declined by about 19% and accounts receivables have gone up by about 24%. 2. Since current liabilities are up by 2% and long-term debt is reduced by only 6%, it is not clear, without the statement of cash flows, where the cash went. 3. Such a dramatic increase of 24% in the accounts receivable can be a cause for concern. Is the company relaxing its credit policies or is it having a poor collection year? Depending upon the cause, one may anticipate that net cash flow problems could occur in 2016.

30


P5–9

Concluded

4. Since retained earnings are down by approximately 6%, and the net income is slightly up, one can only conclude, in the absence of other information, that a hefty dividend was probably paid. This could also explain the dramatic decrease in the cash balance. 5. It seems that the relative composition of cash versus credit sale is switching from 2014 to 2015. This corroborates the dramatic increase in accounts receivable. 6. It is also clear that the company’s increased cost of goods sold has cut into its gross profit, which is down by almost the same amount (4%) as the increment in the cost of goods sold. 7. The company has been quite successful in running tight operations, as is evidenced by a reduction in the total selling and administrative expenses. Reductions such as this would add value to its stock in the long run. b. The proportion of credit sales and cash sales to total sales changed dramatically from 2014 to 2015. The company made approximately twice as many credit sales during 2015 as it made during 2014. This shift flowed through to the balance sheet. Fewer cash sales caused (1) the Cash balance to decrease and (2) the Accounts Receivable balance to increase during 2015. c.

Common-size financial statements allow people to make comparisons across time and across companies by providing a benchmark against which to make the comparisons. Standard financial statements allow only absolute comparisons. By providing a benchmark, common-size financial statements allow relative comparisons. Such comparisons allow financial statement users to focus on the relative importance of an account rather than on whether an account simply increased or decreased in absolute terms. Further, common-size financial statements can provide financial statement users with insights as to why an account balance changed or why a certain trend has developed. For example, in the case of Mountain-Pacific Railroad, the shift in the relative importance of cash and accounts receivable can be explained by examining the shift in the relative importance of cash and credit sales.

P5–10 a. Return on Equity = Net Income ÷ Average Stockholders' Equity 2014: $294,000 ÷ $815,000 = .361 2015: $485,000 ÷ [($815,000 + $835,000) ÷ 2] = .588 Current Ratio = Current Assets ÷ Current Liabilities 2014: 2015:

$867,000 ÷ $267,000 = 3.247 $885,000 ÷ $290,000 = 3.052

Quick Ratio = (Cash + Marketable Securities + Accounts Receivable) ÷ Current Liabilities 2014: 2015:

($312,000 + $120,000 + $150,000) ÷ $267,000 = 2.180 ($10,000 + $125,000 + $500,000) ÷ $290,000 = 2.190

31


P5–10

Continued

Return on Assets = (Net Income + [Interest Expense (1 – Tax Rate)]) ÷ Average Total Assets 2014: ($294,000 + [$65,000 (1 - .34)]) ÷ $1,532,000 = .220 2015: ($485,000 + [$50,000 (1 - .34)]) ÷ [($1,532,000 + $1,475,000) ÷ 2] = .345 Receivables Turnover = Net Credit Sales ÷ Average Accounts Receivable 2014: 2015:

$1,410,000 ÷ $150,000 = 9.400 $4,150,000 ÷ [($150,000 + $500,000) ÷ 2] = 12.769

Earnings per Share = Net Income ÷ Average Number of Common Shares Outstanding 2014: 2015:

$294,000 ÷ 11,000 = $26.73 $485,000 ÷ [(11,000 + 20,000) ÷ 2] = $31.29

Price/Earnings Ratio = Market Price per Share ÷ Earnings per Share 2014: 2015:

$45.00 ÷ $26.73 = 1.684 $70.00 ÷ $31.29 = 2.237

Debt/Equity Ratio = Total Liabilities ÷ Total Stockholders' Equity 2014: 2015:

($267,000 + $450,000) ÷ $815,000 = .880 ($290,000 + $350,000) ÷ $835,000 = .766

Return on Sales = (Net Income + [Interest Expense (1 – Tax Rate)]) ÷ Net Sales 2014: 2015:

($294,000 + [$65,000 (1 - .34)]) ÷ $4,185,000 = .081 ($485,000 + [$50,000 (1 - .34)]) ÷ $6,105,000 = .085

Financial Leverage = Return on Equity – Return on Assets 2014: 2015:

.361 – .220 = .141 .588 – .345 = .243

Dividend Yield = Dividend per Share ÷ Market Price per Share 2014: 2015:

($10,000 ÷ 11,000 shares) ÷ $45 = .020 {$595,000 ÷ [(11,000 + 20,000) ÷ 2]} ÷ $70 = .548

Return on Investment = Market Price0 2014: 2015:

(Market Price1 – Market Price0 + Dividends per Share) ÷

($45 – $50 + $0.91) ÷ $50 = –.082 ($70 – $45 + $38.39) ÷ $45 = 1.409

32


P5–10

Continued

Interest Coverage Ratio = (Net Income Before Taxes and Interest Expense) ÷ Interest Expense 2014: 2015:

($445,000 + $65,000) ÷ $65,000 = 7.846 ($910,000 + $50,000) ÷ $50,000 = 19.200

Inventory Turnover = Cost of Goods Sold ÷ Average Inventory 2015: $4,015,000 ÷ [($210,000 + $200,000) ÷ 2] = 19.585 It appears that during 2015 the company became more efficient at using capital provided by all investors and by equity owners. Both return on assets and return on equity increased by over 50% during 2015. Further, as evidenced by the increase in financial leverage, the company was also more efficient at using debt to benefit the equity owners. The dramatic increase in inventory turnover is probably the primary reason the company became more efficient at using capital. The higher number of inventory turns allowed the company to generate more profits, thereby increasing return on equity, return on assets, financial leverage, and earnings per share. The company also became slightly more efficient at managing its costs, as evidenced by the increase in return on sales. The company has more than sufficient current assets to meet its current liabilities, as evidenced by its current ratio. The company's receivable turnover increased dramatically during 2015, which indicates that it is doing a better job of collecting from its customers. Closer inspection of the receivable turnover, however, reveals that Mountain-Pacific may actually be doing a worse job of collecting from its credit customers. If the 2015 receivable turnover is calculated using just the December 31, 2015 receivables balance instead of using the average receivables balance, the receivable turnover falls to 8.3, which is less than the turnover rate in 2014. Further, the increase in the inventory turnover may indicate future solvency problems. As the inventory turnover increases, the company will have to acquire inventory more often. Therefore, increasing the number of inventory turnovers places added pressure on the company to have sufficient cash to meet its debts as they come due. If the company is unable to generate cash from its receivables on a timely basis, and if it continues to suffer a decline in cash sales, it could very well experience severe solvency problems.

33


P5–10

Concluded

b. Based on the average of the company's 2014 and 2015 ratios, Mountain-Pacific's return on equity, current ratio, and return on assets are almost identical to the industry averages. While the absolute levels of these ratios are similar, the trend of Mountain-Pacific's ratios provides additional information on the company's performance. Based on return on equity, the company has become more efficient during 2012 at managing the equity owners' capital, and based on return on assets, the company has also become more efficient at managing the capital provided by both debt and equity investors. These trends imply that Mountain-Pacific has also become more effective at using debt to benefit the equity owners. Further, the company is now more efficient than the average company in the industry. Several other ratios, such as receivables turnover, return on investment, and times interest earned, indicate that Mountain-Pacific is also performing better than the industry average. When Mountain-Pacific’s quick ratio is compared to the industry average, the company's solvency position appears to be better. Without having additional information about Mountain-Pacific's solvency position, it is difficult to conclude how the company is performing relative to the rest of the industry.

P5–11 As a loan officer, I would be concerned with whether a potential borrower has the ability to meet its debts as they come due. Since both companies are requesting only nine-month loans, I would be interested in the potential borrowers' short-term solvency. Therefore, I would examine their current ratios and quick ratios. Further, I would consider the effect of the potential loan on these ratios. The current ratio is calculated as current assets divided by current liabilities. Selig Equipment: Mountain Bike:

$715,000 ÷ ($285,000 + $125,000) = 1.74 $835,000 ÷ ($325,000 + $125,000) = 1.86

It appears that both companies have more than sufficient current assets to meet their current obligations, including the new loan. However, some current assets, such as prepaid expenses and inventory, are not near-cash assets. Thus, a better measure of a potential borrower's ability to meet its current obligations is the quick ratio. This ratio is calculated as the sum of cash, marketable securities, and accounts receivable divided by current liabilities. Again, the effect of the new loan should be considered. Selig Equipment: Mountain Bike:

($15,000 + $215,000) ÷ ($285,000 + $125,000) = .561 ($160,000 + $470,000) ÷ ($325,000 + $125,000) = 1.400

34


P5–11

Concluded

Based on the quick ratio, Mountain Bike, Inc. appears to be a much better risk than Selig Equipment. Mountain Bike has approximately 2.5 times more near-cash assets available than Selig Equipment to meet its current obligations. Therefore, Mountain Bike does not have to rely as heavily on converting other assets to cash as Selig does to meet its obligations. The company that can most readily convert its inventory and receivables to cash might be the better risk. Two possible measures of a company's ability to generate cash from its receivables and inventory are the turnover and number-of-days ratios. Receivables turnover is calculated as net credit sales divided by average accounts receivable, and the number of days for receivables is calculated as 365 divided by the receivables turnover. Receivables turnover: Selig Equipment: $1,005,000 ÷ $215,000 = 4.67 Mountain Bike: $1,625,000 ÷ $470,000 = 3.46 Number of days: Selig Equipment: Mountain Bike:

365 ÷ 4.67 = 78.16 365 ÷ 3.46 = 105.49

These ratios indicate that Selig Equipment, on average, collects its receivables 27 days quicker than Mountain Bike. Therefore, Selig Equipment can more easily convert its receivables to cash than Mountain Bike can. Inventory turnover is calculated as cost of goods sold divided by average inventory, and the number of days is calculated as 365 divided by inventory turnover. Inventory turnover: Selig Equipment: Mountain Bike:

$755,000 ÷ $305,000 = 2.48 $960,000 ÷ $195,000 = 4.92

Number of days: Selig Equipment: Mountain Bike:

365 ÷ 2.48 = 147.18 365 ÷ 4.92 = 74.19

These ratios bode well for Mountain Bike. Mountain Bike sells its inventory, on average, 73 days sooner than Selig Equipment sells its inventory. This difference implies that Mountain Bike generates more sales which, in turn, implies that it generates more accounts receivable. Although Mountain Bike does not turn over its receivables as often as Selig Equipment, it has a larger amount of receivables to turn over. Thus, Mountain Bike potentially has more assets that can easily be converted into cash than Selig Equipment. Based upon Mountain Bike's superior quick ratio and potential ability to generate cash from its larger receivables base, I would recommend that the bank grant the loan to Mountain Bike, Inc.

35


P5–12 a. Watson Metal Products' 2016 income statements under the different financing alternatives would be as follows.

Income from operations* Interest expense Net income before taxes Income taxes Net income * $16,500,000

Alternative 1

Alternative 2

Alternative 3

$ 16,500,000 4,000,000 $ 12,500,000 5,000,000 $ 7,500,000

$ 16,500,000 4,750,000 $ 11,750,000 4,700,000 $ 7,050,000

$ 16,500,000 4,375,000 $ 12,125,000 4,850,000 $ 7,275,000

= $15,000,000 2016 income from operations from non-French operations per the 2015 income statement + $1,500,000 2016 income from operations from French operations.

The formulas for the requested ratios are: Earnings per Share Return on Equity Return on Assets Financial Leverage Debt/Equity Ratio Note:

= = = = =

Net Income ÷ Average Number of Common Shares Outstanding Net Income ÷ Average Stockholders' Equity (Net Income + [Interest Expense (1– Tax Rate)]÷ Average Total Assets Return on Equity – Return on Assets Total Liabilities ÷ Total Stockholders' Equity

Although several of the ratios use averages, ending balances were used as specified in the problem.

Alternative 1 EPS: $7,500,000 ÷ (2,000,000 shares* + 200,000 shares) = $3.41 * 2,000,000 shares = $6,600,000 2015 net income ÷ $3.30 2015 earnings per share ROE: $7,500,000 ÷ ($45,000,000 + $5,000,000a + $7,500,000b) = .1304 a $5,000,000 = 200,000 shares  $25 per share b $7,500,000 = 2016 net income

ROA: ($7,500,000 + [$4,000,000 (1 – .4)]) ÷ ($35,000,000 + $45,000,000 + $5,000,000 $7,500,000) = .1070 Leverage: .1304 – .1070 = .0234 Debt/Equity:

$35,000,000 ÷ ($45,000,000 + $5,000,000 + $7,500,000)

36

= .609

+


P5–12

Continued

Alternative 2 EPS: $7,050,000 ÷ 2,000,000 shares = $3.53 ROE: $7,050,000 ÷ ($45,000,000 + $7,050,000)

= .1354

ROA: ($7,050,000 + [$4,750,000 (1 - .40]) ÷ ($35,000,000 + $45,000,000 + $5,000,000 $7,050,000) = .1076

+

Leverage: .1354 – .1076 = .0278 Debt/Equity:

($35,000,000 + $5,000,000) ÷ ($45,000,000 + $7,050,000) = .768

Alternative 3 EPS: $7,275,000 ÷ (2,000,000 shares + 100,000 shares) = $3.46 ROE: $7,275,000 ÷ ($45,000,000 + $2,500,000* + $7,275,000) = .1328 * $2,500,000 = 100,000 shares  $25 per share ROA: ($7,275,000 + [$4,375,000 (1 - .4)]) ÷ ($35,000,000 + $45,000,000 + $5,000,000 $7,275,000) = .1073

+

Leverage: .1328 – .1073 = .0255 Debt/Equity: ($35,000,000 + $2,500,000) ÷ ($45,000,000 + $2,500,000 + $7,275,000) = .685 b. Alternative 2 prevents a dilution of the stockholders' position. Since this alternative did not require any additional shares of stock to be issued, it provides the largest earnings per share. Alternative 2 allows the company to more effectively manage its stockholders' investment, as evidenced by return on equity, and all investments, as evidenced by return on assets. The only potentially serious drawback of this alternative is that it makes the company more risky, as evidenced by it having the largest debt/equity ratio. Further, Alternative 2 allows the company to use debt to benefit stockholders more effectively than is allowed with either of the other two alternatives. Under Alternative 3, stockholders earn a slightly smaller return on their equity, but incur fewer risks, since the company has issued less debt. Alternative 1 provides a marginally lower return to stockholders, but imposes even less risk on them. Stockholders must trade off the risk from issuing debt against the benefits of issuing debt. If the company is close to violating debt covenants or projects weak future cash flows, then Alternatives 1 or 3 would probably be preferable. Otherwise, Alternatives 2 or 3 would probably be preferable.

37


P5–12 c.

Concluded

Alternative 1 $3.30 = ($6,600,000 + Net income from expansion project) ÷ (2,000,000 shares + 200,000 shares) Net income from expansion project = $660,000 Alternative 2 $3.30 = ($6,600,000 + Net income from expansion project) ÷ 2,000,000 shares Net income from expansion project = $0 Alternative 3 $3.30 = ($6,600,000 + Net income from expansion project) ÷ (2,000,000 shares + 100,000 shares) Net income from expansion project = $330,000

P5–13 Note:

Although some ratios use average balances, year-end balances were used in the ratios as directed in the problem.

a.

Return on Equity = .75 = Total Stockholders' Equity =

b.

Debt/Equity Ratio = .4 = Total Liabilities =

c.

d.

e.

f.

Total Assets

= = =

Return on Assets .65 Interest Expense

Total Liabilities ÷ Total Stockholders' Equity Total Liabilities ÷ $600,000 $240,000 Total Liabilities + Total Stockholders' Equity $240,000 + $600,000 $840,000

= = =

(Net Income + Interest Expense) ÷ Total Assets ($450,000 + Interest Expense) ÷ $840,000 $96,000

Net Income After Taxes = $450,000 = Net Income Before Taxes = Return on Sales = .2 = Net Sales =

Net Income ÷ Total Stockholders' Equity $450,000 ÷ Total Stockholders' Equity $600,000

Income Before Taxes  (1 – Tax Rate) Net Income Before Taxes  (1 – 34%) $681,818

Net Income ÷ Net Sales $450,000 ÷ Net Sales $2,250,000

g. Credit Sales = Net Sales  80% = $2,250,000  80% = $1,800,000 38


P5–13

Concluded

h. Receivables Turnover

= 25 = Accounts Receivable =

i.

Net Credit Sales ÷ Accounts Receivable $1,800,000 ÷ Accounts Receivable $72,000

Net Sales  55% = $2,250,000  55% = $1,237,500

Cost of Goods Sold=

j.

Average Days' Supply of Inventory = 12.167 = Inventory Turnover =

k.

Inventory Turnover 30 Inventory

l.

Current Liabilities =

m.

n.

Current Ratio 3.00 Current Assets

Cost of Goods Sold ÷ Inventory $1,237,500 ÷ Inventory $41,250

= =

Total Liabilities  35% $240,000  35% $84,000

= = =

Current Assets ÷ Current Liabilities Current Assets ÷ $84,000 $252,000

Quick Ratio Liabilities 2.0 Marketable Securities

o. Noncurrent Assets=

p.

= = =

365 ÷ Inventory Turnover 365 ÷ Inventory Turnover 30

=

(Cash + Accounts Receivable + Mkt. Securities) ÷ Current

= =

($68,000 + $72,000 + Marketable Securities) ÷ $84,000 $28,000

Total Assets – Current Assets = $840,000 – $252,000 = $588,000

Earnings per Share $16.00 Common Shares Outstanding =

= Net Income ÷ Number of Common Shares Outstanding = $450,000 ÷ Number of Common Shares Outstanding 28,125 shares

39


P5–14 a. Total Revenues equal $53,395 with the percentage breakdown as follows: Frito Lay North America Quaker Foods North America Latin America Foods Americas Beverages U.K & Europe Middle East, Africa, Asia

25.4% 4.9% 14.6% 40.1% 2.5% 12.5%

b. Ranking the segments on profit margin (Profits/Revenues) yields: 1. Frito Lay North America 26.7% 2. Quaker Foods North America 26.4% 3. U.K. & Europe 24.6% 4. Americas Beverages 13.7% 5. Latin America Foods 13.6% 6. Middle East, Africa, Asia 11.2% c.

Ranking the segments on Return on Assets (Profits/Assets) yields: 1. Quaker Foods North America 72.0% 2. Frito Lay North America 68.4% 3. Latin America Foods 21.2% 4. Middle East, Africa, Asia 13.0% 5. Americas Beverages 9.5% 6. U.K. & Europe 1.7%

d. Quaker Foods is the second smallest of the segments, but it is one of the most profitable. Pepsico’s management should focus efforts on growing the Quaker Oats business in terms of revenue (while maintaining the profit margins). Frito Lay is the second largest segment (behind the beverage operation for North, Central and South America) and its profit margins are the highest; this division clearly distinguishes Pepsico from its rival Coca-Cola. One final conclusion can be seen in the numbers for Europe and the Middle East; the company has dedicated significant investment in these markets (seeing them as future growth areas), but the profits have not yet matched the investment in assets.

40


P5–15 The ratios for the 2012 Unilever statements are: a. ROE Net Income/Average Stockholders’ Equity 4,948/15,318.5 = 32.3% ROA [Net Income + (1 – Tax Rate)x (Interest Expense)]/Average Total Assets [4,948 + (1 - .26)(526)]/46,839 = 11.4%

Return On Sales [Net Income + (1 – Tax Rate)x (Interest Expense)]/Net Sales [4,948 + (1 - .26)(526)]/51,324 = 10.4%

Common Equity Leverage Net Income/[Net Income + (1 – Tax Rate)x (Interest Expense)] 4,948/[4,948 + (1 - .26)(526)] = 0.93

Capital Structure Leverage Average Total Assets/Average Stockholders’ Equity 46,839/ 15,318.5 = 3.06 Debt/Equity Ratio Average Total Liabilities/Average Stockholders’ Equity 31,520.5/15,318.5 = 2.06 Long-term Debt Ratio Long-term Liabilities/Total Assets 14,635/46,166 = 0.32 Current Ratio Current Assets/Current Liabilities 12,147/15,815 = 0.77 Quick Ratio (Cash + Marketable Securities + Accounts Receivables)/Current Liabilities 6,901/15,815 = 0.44 Interest Coverage (Net Income + Tax Expense + Interest Expense)/Interest Expense (4,948 + 6,683 + 526)/526 = 23.1 Accounts Payable Turnover Cost of Goods Sold*/Average Accounts Payable 30,703/11,319.5 = 2.71 365 days/2.71 times = 134.7 days Receivables Turnover Sales/Average Accounts Receivable 51,324/4,474.5 = 11.47 365 days/11.47 times= 31.8 days Inventory Turnover Cost of Goods Sold*/Average Inventory 30,703/4,518.5 = 6.80 365 days/6.80 times = 53.7 days 41


Fixed Asset Turnover Sales/Average Fixed Assets 51,324/9,109.5 = 5.63 Total Asset Turnover Sales/Average Total Assets 51,324/46,839 = 1.10 P5–15 Concluded *Cost of Goods Sold found in footnotes Based on the Current and Quick Ratios, the company will have to look to the conversion of long-term assets to Cash (from either operating those assets or liquidating them) to satisfy current liabilities. The company pays its trade suppliers extremely slowly (thus driving up the balance in Current Liabilities), but it can cover its interest payments very easily. From an earnings power perspective, the high returns are matched by strong and consistent cash flow generated from operations. b. As demonstrated in the ROE Model, Return on Equity is driven by Return on Assets, Capital Structure Leverage and Common Equity Leverage. Although the 2010 numbers are not provided (for 2011 averages), it appears that Equity levels are up, while Liabilities levels are down—meaning that Unilever has deleveraged its position in 2012. At the same time, profits for 2012 are higher than those in 2011. Those two factors (higher profits, higher equity) indicate that the driver for 2012 ROE will be ROA, which is in turn driven by Asset Turnoer and Profit Margin. Since the profit margin (Return on Sales) dropped from 10.8% to 10.4%, the growth in ROA (and therefore the growth in ROE) was driven by the increase in Asset Turnover (the growth in sales relative to the asset base).

42


ISSUES FOR DISCUSSION ID5–1 (1) Bank of America (2) Bed, Bath & Beyond (3) HP

(4) Kelly Services

Bank of America is Company #1. As a commercial bank, B of A generates service revenue (see Income Statement) and carries receivables (loans) that are funded by deposits (current liabilities on the balance sheet). Banks also are highly leveraged; Company #1 has the lowest equity of the four companies shown. Bed, Bath & Beyond is Company #2. As a retailer, BB & B generates sales revenue from providing inventory to its customers. Company #2 is the only company of the four shown with 100% sales revenue. Further, the company has a significant investment in inventories, as do all retailers. Hewlett Packard is Company #3. As a technology and consulting company, HP will generate both sales revenue (from the sale of products such as printers) and service revenue (from its consulting business). Of the four companies, only #3 has a mix of sales and service revenue. The company also has investment in inventories, receivables and long-term assets, common for a manufacturing concern. Kelly Services is Company #4. As a temp agency, Kelly will generate service revenue from placing workers and will also carry accounts receivable from its large clients. The company does not carry inventory and does not need a significant investment in long-term assets to operate.

ID5–2 (1) General Electric

(2) EchoStar

(3) Walgreen’s

(4) Campbell’s Soup

General Electric is Company #1. GE operates as both a manufacturer and a diversified financial services firm, generating both sales and service revenue. The financial services business, similar to Bank of America in ID5-1, will carry a significant amount of receivables. EchoStar is Company #2. The company sells customers satellite dishes and the television services that are received by the dishes. Further, satellite television services are a relatively new business; start-up operations often show losses for a number of years and Company #2 is showing a loss on the income statement. Walgreen’s is Company #3. As a retailer, the company will generate sales revenue and carry heavy investment in inventories. (The inventory line is greater in #3 than it is in #4.) Campbell’s is Company #4. The description indicates that Campbell’s has been growing by acquisition, which usually generates goodwill, an intangible asset carried under “Other Assets”. Further, manufacturers typically carry balances in receivables, inventory and long-term assets such as property, plant and equipment.

43


ID5–3 Book value (stockholders’ equity) is a GAAP number that is derived from accounting values that may not reflect current market prices. For example, the book value of a company might include fixed assets that were purchased many, many years ago and were recorded on the books at historic costs; if market values for those fixed assets have increased, there will be a significant difference between the book value of the company (based on historic costs) and the market value (based on current market prices of the assets). A price-to-book ratio would be high if a company had many assets on its books at historic costs that are well below current market prices; if inflation and market appreciation have caused the market value of fixed assets to grow well above the original cost, and if many years have passed since the purchase of the assets, then the price-to-book ratio of a company will be high. Businesses with assets that have been held for a long time, or businesses with assets that have grown significantly in value (real estate in Manhattan, for example) will have high PB ratios. On the other hand, if a business owns assets that have lost significant market value due to a recession (like banks owning mortgage securities in 2008-2009), then the book values will be above market prices, creating a low PB ratio.

ID5–4 (a) A consensus earnings forecast is the most common or average earnings forecast by Wall Street analysts. Companies wish to consistently beat the estimates because, in general, the stock of their company tends to go higher as they beat the consensus estimate. If a company misses a forecast then analysts could downgrade the stock and the stock price could fall. (b) Companies don’t want to beat the estimates by too much because it may result in future expectations beyond what the company can deliver. So while the stock price might go up significantly now it could lead to a future earnings disappointment which could cause the stock to fall significantly. (c) Management can do two things. The first is to manage the expectations of research analysts. Management can “guide” analysts in terms of sales and expenses. The second is that management may try to “manage earnings” by increasing accruals or taking write-offs in good quarters and reducing accruals or taking gains in weak quarters. In both cases the idea is to keep earnings on a smooth path from quarter to quarter. Analysts have concern about companies who manage earnings because they feel they are not seeing the true picture of a company’s performance if the bottom line is smoothed by management assumptions regarding accruals and estimates.

ID5–5 (a) It is likely that analysts saw Kodak’s move coming. Analysts closely monitor industry conditions and technological changes and therefore understood that digital photography was the future. Pinpointing the exact dollar amount of the charge, however, would be more difficult for the analysts. Management, with its access to all the data and numbers of the company, is in a much better position to estimate dollar amounts for restructuring. The stock market probably applauded the move, as it knew that Kodak’s traditional business had been shrinking in recent years. Financial markets often reward companies with a boost to the stock price when restructuring announcements are made. The fact that management is proactively addressing a problem is often seen as a positive. 44


(b) Management of Kodak was taking all of the bad news at once. By booking the entire restructuring expense at the date of the announcement, and not as the payments are made, is a move companies take to show all the negatives today, holding the positives for the future. The thought is that the negative news is going to hit anyway, so the company might as well take all of the bad publicity at once. Future periods, therefore, will not be burdened with the expenses and will show better results. Earnings management of this type is very common among public companies. If the company overestimates the charges that it takes today it allows the company to “improve” its performance in future periods as it takes the overestimated expenses back into income. (c) It would be most prudent to take the restructuring expense out of the financial statements and calculate your ratios excluding the charge. If the charge is a one-time adjustment, to get a true picture of the company’s trends, the expense should not be included in the analysis. (d) Several cases of companies facing technological obsolescence exist. For example, network television companies such as ABC, NBC and CBS are vulnerable to the advances of cable and satellite television providers. Companies specializing in the delivery of documents such as Federal Express and UPS (as well as the US Postal Service) are vulnerable to new methods of communication like email. Music companies (and musicians) that historically made their profits by selling album and compact discs to consumers are vulnerable to the technological advances of transmitting and sharing music over the internet. Blockbuster’s brick-and-mortar movie rental business is vulnerable to online movie streaming.

ID5–6 Staples might want to separate the losses from its dot-com operations in order to help maintain a high stock price. By excluding the losses from the dot-com operation Staples will be reporting a higher net income. This will tend to have a positive effect on its stock price. This is especially true if its competitors include similar types of losses in their reported income. Management may justify not including the results from the dot-com operation in its reported income because Staples’ reported income in past years did not include this operation and so by including these numbers would reduce the consistency and in some cases the comparability of the numbers. Management may also may the claim that including the dot-com operations would be confusing to investors and not allow investors to evaluate the primary Staples business. The policy of not including the dot-com operations in reported income doesn’t seem to fit with the overall goals of full disclosure and would seem to be shielding investors from valuable information. The dot-com operation is not any different than a company entering into a new market (product or geography) and having losses during the early years. This seems to be a very questionable accounting policy and not an appropriate one.

ID5–7 A stock price that has already fallen dramatically has less risk than one that is trading at its all time high. A number of investors have already sold the stock and as the stock price falls there may be other investors who become interested in the stock. A falling stock price could also be reflecting that short sellers of the stock have been knocking the price down. Short sellers eventually have to buy back the shares that they have shorted. The fact that the company reported positive free cash flow means that the company is able to fund itself through its operations and is not forced to go to the capital markets (sell stock) when the stock price is low. 45


Using conservative accounting methods implies that the company is not inflating its earnings by using questionable accounting methods. Conservative accounting methods allow investors to have more confidence in the reported numbers. A build-up in raw inventory materials and partially-completed inventory would imply that management is preparing for higher sales levels by increasing its inventory in the pipeline. Since finished inventory has not been increasing this would seem to say that sales are in good shape. An increase in discretionary expenditures like research and development would be a strong statement by management that future business seems to be in very good shape. If management did not have a positive view on future sales then it probably would not increase spending in areas that were not critical. Many companies have undervalued assets on the balance sheet because fixed assets are carried at their original cost. Years later many of those assets have appreciated in value (buildings) but yet this increase in value has not been shown on the income statement. Having little or no debt while still having a high return on assets is a very positive sign for the company. This means that the company could borrow money for high return projects or to buyback the company’s stock. Consistency between what the president’s letter says and what actually happens again provides a lot more confidence for investors that the company is going to be able to execute on its strategy. Many companies say they will do a lot of different things and then never actually get to them. Confidence in what management says will draw more investors to the company’s stock.

ID5–8 a. An efficient market implies that all information about a company is reflected in the price of the company's stock as soon as the information becomes available to the public. Since the information in financial reports is available to the public, market efficiency implies that as soon as the financial report is made public, all the information in the report is reflected in the company's stock price. Thus, investors could not use financial reports to identify undervalued securities because the investors would not have any information that is not already reflected in the stock price. b. Although accounting information may not be useful in identifying undervalued or overvalued stocks, it can be useful in assessing a company's solvency and earning power, which are important components of an investment decision. For example, banks use financial statements to determine the creditworthiness of loan applicants.

ID5–9 Intangibles refer to the many assets that do not have physical characteristics. The majority of these are not recorded on the balance sheet of a company. A very valuable intangible asset that is not shown on the balance sheet is the brand name of a company. Examples of these would be the Coca-cola name, the Nike swoosh, and the Mercedes mark. While these assets are not included on the balance sheet they indirectly impact the income statement. The whole idea of a valuable brand name is that consumers want to buy your product and this is reflected in higher sales. In talking about the balance sheet it appears that his comments are exactly correct. Compare the book value of a company to its market value. This gives a good indication of the value of the company that is not reflected on the balance sheet. The income statement reflects that value of a brand name in the reported sales. 46


ID5–10 Buffett is making the case that the reaction of a weak management to a difficult operating environment is to try and slide through by using questionable accounting measures to make up for poor performance. A strong management would be able to work through the difficult times and not have to fudge the accounting numbers. Evaluating how management handles adversity is probably a good indicator of which companies will perform the best during good times. If a company’s customers are suffering in a difficult economic environment a strong management team would work on making sure that sales were made to viable customers while a weak management team would sell to any customer to show higher sales and then would probably understate the potential for bad debts in the financial statements. Eventually some of these sales will not be collected and those disappointments will lead to lower stock performance. Financial statement users should look at the accounting standards employed by companies and determine what this is saying about the quality of the management team and their ability to operate the company. Over time a good management team will figure out a way to make a company successful while a weak management team will almost always underperform its competitors..

ID5–11 a.

Receivables turnover and inventory turnover are the two ratios that probably best capture the suggested indicators.

b.

Inventory turnover is calculated as cost of goods sold divided by average inventory, and receivables turnover is calculated as net credit sales divided by average accounts receivable. A decreasing inventory turnover means that the company is taking longer to sell its inventory. Similarly, a decreasing receivables turnover means that the company is taking longer to collect its receivables. Both ratios have implications for solvency and earning power. If a company is turning over its inventory more slowly, the company will be generating fewer cash and credit sales. That is, the company will be generating fewer assets (such as cash or accounts receivable) through operations, thereby having lower earning power. Lower sales implies lower cash collections from cash or credit sales, which, in turn, implies that it will be more difficult for the company to pay its obligations as they come due (i.e., the company has lower solvency). Less sales also implies that the company will have fewer assets to reinvest in productive assets, which, in turn, implies that the company will be less profitable in the future (i.e., the company has lower earning power). If the company is collecting its receivables more slowly, it will have less cash available to pay its obligations—which implies lower solvency—and to invest in productive assets—which implies lower earning power. Because solvency and earning power are important to investors, any indication that a company is having solvency and/or earning power problems would be expected to result in a drop in the price of the company's stock. The ratios are more likely than reported earnings to pick up this information because management has much more discretion (at least in the short run) over reported earnings than over the items used to calculate the ratios.

ID5–12 47


Theoretically, the price of a company's stock equals the present value of the stock market's expectations of the cash flows an investor would realize from investing in that company's stock. Thus, the price of a company's stock is a function of the market's expectations about future cash flows. If the market receives information that causes it to revise its expectations, the price of the company's stock should change. Earnings information is one source of information about future cash flows. It is possible that an earnings announcement of lowered results can also give the market information about the company’s future that indicates an increase in future cash flows. Likewise, an announcement of higher earnings can contain information that is negative about the company’s prospects. The stock price is determined by the market’s interpretation of future results.

ID5–13 a. Many of the Internet companies that became publicly-traded in the 1990s ended in failure. The researchers were suggesting that a similar problem might happen again in the future with the 2007 companies going public before establishing themselves as profitable businesses. b. The worry is that the investors buying the IPO shares are buying the hype and promise of the company without having any proof of either the company’s viability or the soundness of its business model. In effect, the investors are paying now for profits that will supposedly be earned in the future. c.

Companies (and their investment bankers) are incented to sell shares when the selling price is high. When the overall market is at record levels, those selling stand to receive more cash from the proceeds of the IPO. The temptation to “go public” before profitability is strong when market prices are exceedingly high.

d. Investors should be examining a company’s Income Statement to see revenues versus expenses, but investors should also analyze the Statement of Cash Flow to understand the inflows and outflows of cash due to the company’s operations, investments and financings. Ideally, this analysis should be done on both historic data, as well as on projections of future periods.

ID5–14 The case asks you to assess Eli Lilly’s financial statements using the ROE model. Relevant ratios for Lilly are calculated below: (tax rate equals 24%; interest expense taken from the footnotes equals $171.9 million in 2012 and $167.4 million in 2011) 2012 Return on Equity = 2011 Return on Equity =

28.9% 33.5%

2012 Return on Assets = 2011 Return on Assets =

12.4% 13.8%

2012 Common Equity Leverage 2011 Common Equity Leverage

= =

.969 .972

2012 Capital Structure Leverage 2011 Capital Structure Leverage

= =

2.40 2.49

ROE = ROA X C.E.L. x C.S.L. 2012

.289 = .124 x .969 x 2.40 48


2011

.335 = .138 x .972 x 2.49

2012 Return on Sales = 2011 Return on Sales =

18.67% 18.43%

2012 Asset Turnover 2011 Asset Turnover ID5–14 Concluded

= =

.664 .751

ROA = P.M. x A.T. 2012 2011

.124 = .1867 x .664 .138 = .1843 x .751

From 2011 to 2012, Eli Lilly experienced a drop in Return on Equity from over 33 percent to approximately 29 percent. The ROE model can be used to pinpoint the variable(s) that is(are) driving the change in shareholder return. Three variables directly feed into ROE. As shown above, Eli Lilly experienced a decrease in Return on Assets, as well as small decreases in leverage (from both Capital Structure Leverage and Common Equity Leverage). All three variables pushed down the ROE results, but the decrease in ROA was more significant than either of the leverage declines. Two variables directly affect ROA: return on sales and asset turnover. Return on sales determines how well the company converts sales into profits, while asset turnover measures the company’s ability to generate sales from assets. Lilly’s profit margin actually increased so the drop in ROA was driven by the drop in sales relative to assets. Given the company’s business model, the lack of sales (possibly from drugs losing patent protection, or from a lack of new drugs in the company’s research pipeline) would be of great concern to analysts. However, the ROE levels are high enough that the company has certainly created value for its shareholders (where ROE is compared to Lilly’s cost of capital).

ID5–15 2012 $22,603.4 34,398.9 19,625

Sales Total Assets Total Liabilities

Historic Relationship NA 1.52 times Sales 57.1% of Assets

If Sales are projected to grow at 5% and if expenses are expected to be 85% of Sales (as well as the historic relationships above), the projected 2013 financial statements can be summarized below:

Sales Net income

2013 $23,733.6 3,560.0

Total assets Total shareholders’ equity

36,075.1 15,476.2

ID5–16 Profile #1 – EchoStar

Profile #2 – US Airways

Profile #3 – Wal-Mart

49


EchoStar is a relatively new company with a relatively new product. The company has been growing, as more consumers switch from traditional and cable television services to satellite television. The growth of the company has generated positive cash from operations. However, the company has had to take that cash and invest it back into its business; the negative cash from investing in all three years shows that the company is growing its business by investing in long-term assets. The growth is not sufficiently funded by operations, so the company is raising money from debt and equity sources (financing activities) to fund its business. At the present time, the company is not returning cash to its shareholders or repaying debt. It is borrowing additional funds and raising additional equity to fund its growth. US Airways operates in a difficult industry. The company has been unable to generate a positive cash flow from its operations, a definite red flag for any business. The cash outflow from operations has been paired with negative cash from investing activities, as the company must reinvest in long term assets (aircraft) to remain competitive in its industry. The only source of cash to fund the negative operations and the investments is financing (issuing debt and selling equity). The long term prospects for the company do not look healthy. No business can sustain itself through endless financing activities to fund cash; if the company is going to survive, it will eventually have to generate cash from its operations so that it can eventually repay debt and return some cash to shareholders. Wal-Mart is an excellent example of a successful company that has reached the stage in its maturity that its operations throw off enough cash to fund the company’s still-impressive growth (investing activities) AND to return cash to shareholders and/or repay debt (financing activities). The company can rely on its daily operations to generate cash, which is then used to add more stores and fund dividends and debt reduction.

ID5–17 a) Financial ratios for Google are outlined below: (tax rate = 19%)

Return on Equity

2012 16.5%

Return on Assets

13.0%

Return on Sales

21.5%

Common Equity Leverage

0.99

Capital Structure Leverage

1.28

Debt/Equity

0.31

Long-term Debt Ratio

0.032

Current Ratio

4.22

Quick Ratio Interest Coverage

3.90 159.75 ([10,737 +2,598 +84*]/84*) 50


Accounts Payable Turnover/Days on Hand

18.87 / 23.0 days

Accounts Receivable Turnover/Days on Hand

7.54 / 48.4 days

Inventory Turnover/Days on Hand

12.81 / 28.5 days

ID5–17 Continued

Fixed Asset Turnover Asset Turnover

4.68 0.60

*see Footnote #10 Google has generated impressive profitability numbers in the year ending 2012. Returns on equity exceeded sixteen percent (despite a very large equity base), while the return on sales was over twenty one percent. Debt levels are very low; the company is very solvent, carrying almost four dollars in quick assets for every dollar in current obligations; the company has over $48 billion in cash and marketable securities. b)

ROE = ROA x Common Equity Leverage x Capital Structure Leverage ROA = Return on Sales x Asset Turnover Google 2012 .165 = .130 x .99 x 1.28 .130 = .215 x 0.60 (rounding) Google’s strong return on equity is driven primarily by its ROA. The company’s equity base is relatively large, but the company is still able to generate a significant return on this equity by throwing off a strong return from its assets. The company appears to be debt-averse (low leverage), which means that it carries relatively more equity. To generate a satisfactory return for its shareholders, the company must do an excellent job converting its assets into acceptable returns. Driving the ROA are the two factors: converting sales from assets and converting those sales into profits. The company needs a large asset base to generate revenue; then, once the sale is made, Google converts that revenue into bottom-line earnings at a strong pace. A true analysis would 1) take the company’s financial ratios over the last five years to see the changes and trends in the various performance metrics, and 2) compare Google to its competitors to determine its relative strengths and weaknesses.

51


ID5–17 Concluded c)

Sales Net income Total assets Total shareholders’ equity

2012 $50,175 10,737 93,798 71,715

Historic Relationship NA 21.4% of Sales 1.87 times Sales 76.5% of Assets

If Sales are projected to grow at 8% and if the historic relationships maintain, the projected 2013 financial statements can be summarized below:

Sales Net income

2013 $54,189 11,596

Total assets Total shareholders’ equity

101,333 77,520

52


CHAPTER 6 THE CURRENT ASSET CLASSIFICATION, CASH, AND ACCOUNTS RECEIVABLE BRIEF EXERCISES BE6–1 a. Total Accounts Receivable

= Net Receivables + Allowance for Uncollectibles

2012 Total Accounts Receivable =

$4,983

2012 Total Accounts Receivable =

$5,092

+

$109

2012 Uncollectibles as a Percentage of Total Accounts Receivable = $109/$5,092 = 2.14% 2011 Total Accounts Receivable =

$4,502

2011 Total Accounts Receivable =

$4,606

+

$104

2011 Uncollectibles as a Percentage of Total Accounts Receivable = $104/$4,606 = 2.26% Therefore, the percentage decreased. b. Since Emerson Electric is using the percentage of accounts receivable method (balance sheet approach), bad debt expense for 2012 would be the amount needed to adjust the allowance for doubtful accounts to $109. This number (bad debt expense) is impacted by the balance in the uncollectible account at the beginning of the year and the write-offs taken during the year by Emerson Electric.

BE6–2 a.

The changes to the Allowance account are: Ending Allowance Balance = Beginning Allowance Balance + Bad Debt Charge – Write-Offs + Recoveries

Therefore, the “increases” shown are the annual bad debt expenses. For 2011 the amount was $3,795 million, while the amount in 2012 was a very close $3,793 million. Unlike other time periods, 2011 and 2012 were both economic recovery periods, indicating that the company’s credit issues with customers were similar.

b.

2011: $7,139 million write-offs; $5,524 million write-offs, net of recoveries of previous write-offs 2012: $6,582 million write-offs; $4,998 million write-offs, net of recoveries of previous write-offs 1


c.

The allowance account decreased by 19.5% from 2011 to 2012. Although the annual expense for bad debts and recoveries were very similar, the amount written off (due to specific knowledge of a customer’s situation) decreased more substantially. Perhaps GE has dealt with many of its problempaying customers in the early part of the economic recovery, leaving less issues to be cleaned up.

BE6–3 a.

GE bad debts as a percentage of total revenues = $3.9/$147 = 2.7%; as a percentage of GECS revenues, the calculation is $3.9/$46 = 8.5%. GE overall revenues should be used since the bad debt provision is for GE and not for GECS.

b. On a balance sheet for GECS accounts receivable would be expected to be the largest account. Its primary role is as a financing company, and the receivables from the buyers of appliances would be a large asset of GECS. c.

GE is very large and has many subsidiaries that make it difficult to classify it as just a manufacturing, retail or service company. The overall GE business is best known as a manufacturing company. It does not have its own retail stores, so it is not a retailer. While there are many services offered with its products, its primary focus is as a manufacturer. Services, primarily financial services, were becoming more important for the consolidated operation prior to the economic crisis, but senior management has pledged to reduce the importance of financing revenue in future years.

EXERCISES E6–1 a. Cash. Money held in checking accounts is defined as cash, and there are no restrictions on the account. b. Cash. Checks are considered cash unless the checks cannot be cashed until a later date (i.e., postdated). In this case, the check date has passed, so the checks are considered cash. c. Investment. Certificates of deposit contain penalties for early withdrawal. Since the certificates mature outside the time frame of current assets, this source of cash is not readily available and should not be classified as Cash. d. Cash. Because banks have the right to demand notice prior to a withdrawal from a savings account, the cash in savings accounts is technically not readily available. However, since banks rarely exercise this right, savings accounts are considered cash. e. Cash. Petty cash is always considered cash. f. Restricted cash. Because the company does not have ready access to these funds, the $50,000 should not be reported as cash. The portion of the $50,000 corresponding to short-term loans (i.e., $15,000) should be classified in current assets as restricted cash, and the remaining $35,000 should be classified as a long-term investment or as an other asset. g. Cash. See (a).

E6–2 a. unrestricted cash b. investment c. restricted cash

d. e. f.

investment investment restricted cash

g.

unrestricted cash


E6–3 a. 12/12

1/5

Accounts Receivable (+A)........................................................... Sales (R, +SE) ....................................................................... Made sale on account.

40,000

Cash (+A) .................................................................................... Accounts Receivable (–A) .................................................... Collected cash from customer.

40,000

Cash (+A) .................................................................................... Cash Discount (–R, –SE) ............................................................. Accounts Receivable (–A) ....................................................

39,200 800

40,000

40,000

b. 12/20

40,000

The timing of the cash receipts would affect the income statement due to the cash discount. Cash discount is a contra sales account and is deducted from the sales revenue. Therefore, the net income is reduced by the amount of the cash discount during year 1 in case (b). Under option (a) the statement of Cash Flows will show no inflow during year 1, but during year 2 it will show an inflow from operating activities of $40,000. Under option (b) no inflow from operating activities will be shown in year 2, but during year 1 there will be an increase of $39,200 from operating activities.

E6–4 5/1/15

Accounts Receivable (+A)........................................................... Sales (R, +SE) ....................................................................... Sold lobster on account.

30,000

Accounts Receivable (+A)........................................................... Sales (R, +SE) ....................................................................... Sold cod on account.

20,000

Cash (+A) .................................................................................... Cash Discount (-R, -SE) ............................................................... Accounts Receivable (-A)..................................................... Collected cash from customer.

29,100 900

5/31/15 Cash (+A) .................................................................................... Accounts Receivable (-A)..................................................... Collected cash from customer.

20,000

5/5/15

5/6/15

30,000

20,000

30,000

20,000

E6–5 Allowance for Uncollectibles 2014 Write-Offs

10,000

12/31/14 Bad Debt Expense

30,0001


2015 Write-Offs

22,000

12/31/15 Bad Debt Expense

28,0002

Ending Balance

26,000

___________ 1 $1,500,000  .02 = $30,000 2 $1,400,000  .02 = $28,000 Overall, the bad debts estimates are sufficient to cover the write-offs.

E6–6 a.

Ending Allowance Balance $200,000

= Beginning Allowance Balance + Bad Debt Charge – Write-Offs + Recoveries = Beginning Allowance Balance + $162,500 – $195,000 + $45,000 = $187,500

Beginning Allowance Balance _____________ * $162,500 = Sales of $3,250,000  Estimated uncollectible percentage of 5%

b. [Assume the $4,200,000 cash collections includes the $45,000 recovery] Ending Accounts Receivable = Beginning Accounts Receivable + Credit Sales during the year – Cash Collected on account receivables during the year – Accounts Receivables written off + Written-Off Accounts Receivables Recovered. $7,500,000

= X =

X + $3,250,000 – $4,200,000 – $195,000 + $45,000 $8,600,000

E6–7 a. Bad Debt Charge (E, –SE)........................................................................... Allowance for Doubtful Accounts (–A) ............................................... Recognized bad debt charge. _____________

b. Ending Allowance Balance

6

= Beginning Allowance Balance + Bad Debt Charge – Write-Offs + Recoveries = $12 + $6 – $5 + $0 = $13

E6–8 a. Ending Allowance Balance

= Beginning Allowance Balance + Bad Debt Charge – Write-Offs + Recoveries

6


$1,300 Write-Offs

= $1,000 + (Sales of $75,300 x Estimated uncollectible percentage of 2%) – Write-Offs + $55 = $1,261

b. Accounts Receivable (+A) .......................................................................... Sales (R, +SE)....................................................................................... Made sales on account.

75,300

Accounts Receivable (+A) .......................................................................... Allowance for Doubtful Accounts (-A) ................................................ Recovered accounts previously written off.

55

Allowance for Doubtful Accounts (+A) ...................................................... Accounts Receivable (–A) ................................................................... Wrote off accounts deemed uncollectible.

1,261

Bad Debt Expense (E, –SE)......................................................................... Allowance for Doubtful Accounts (–A) ............................................... Recognized bad debt expense.

1,506

75,300

55

1,261

1,506

Cash (+A) ............................................................................................. 73,894* Accounts Receivable (–A) ................................................................... 73,894 Collected cash from customers. _____________ * $73,894 was calculated as follows: Ending balance in Accounts Receivable of $9,400 = Beginning balance in Accounts Receivable of $9,200 + 2015 Credit sales of $75,300 – 2015 Cash collections + 2015 Recoveries of $55 – 2015 Write-offs of $1,261

E6–9 a. Extending credit to customers (and therefore creating accounts receivable on the balance sheet) is a normal part of business operations. Therefore, the transactions related to receivables, such as uncollectibles and increases/decreases in gross receivables, are treated as operating activities. b. The indirect format of the statement of cash flow is structured around converting net income (a number based in accrual accounting) into a pure cash number. The provision is a non-cash charge, similar to depreciation expense,that reduces net income but does not (by itself) reduce the company’s cash balance. Therefore, the non-cash expense is added back to net income in the calculation of cash from operating activities. The fact that receivables decreased indicates that the company collected more cash than just the amount in its current year sales— effectively, the company collected this year’s sales and some of last year’s (that were sitting as accounts receivable on the balance sheet). If receivables had increased, the company would have deducted that amount in the statement of cash flow (because the company would not have collected as much cash as it had booked in sales)..The decrease in receivables is called a “source” of cash (while an increase would be called a “use” of cash). c. Net cash from operating activities can be higher than net income because of non-cash expenses (as discussed above) and because of decreases in current assets (“sources” of cash as discussed above) or increases in current liabilities (also “sources” of cash). If “uses” of cash (such as


increases in current assets and decreases in current liabilities) had been larger, net cash from operating activities may have been lower than net income.

E6–10 Total receivables equals the sum of the receivable balances for each age classification. Therefore, total receivables equals $290,000 + $110,000 + $68,000 + $40,000, or $508,000. Expected bad debts equals the sum of the estimated uncollectible amounts for each age classification. As shown in the following table, the total expected bad debts associated with the $508,000 currently in accounts receivable is $22,740. Age Current 1–45 days 46–90 days >90 days Total

Account Balance $290,000 110,000 68,000 40,000 $508,000

Noncollection Probability 2% 5% 8% 15%

Uncollectible Amount $ 5,800 5,500 5,440 6,000 $ 22,740

E6–11 1/1/15

Notes Receivable (+A) ................................................................ 10,450* Sales (R, +SE) ....................................................................... Made sale in exchange for a note. _________________ * $10,450 = Sales in Canadian dollars of 11,000 x Exchange rate of $0.95/Canadian dollar Notes Receivable (+A) ................................................................ 440* Exchange Gain (Ga, +SE)...................................................... Recorded foreign currency exchange gain on receivable. _________________ * $440 = Adjusted value of note – Carrying value of note = (Note for 11,000 Canadian dollars x Current exchange rate of $0.99/Canadian – $10,450

10,450

12/31/15

Exchange Loss (Lo, –SE).............................................................. 990* Notes Receivable (–A) ......................................................... Incurred foreign currency exchange loss on receivable. _________________ * $990 = Adjusted value of note – Carrying value of note = (Note for 11,000 Canadian dollars x Current exchange rate of $0.90/Canadian – $10,890

440

dollar)

12/31/16

E6–12 January 1, 2015

990

dollar)


Notes Receivable (+A) ...................................................................................... Sales (R, +SE) ............................................................................................. Made sale in exchange for a note.

10,450* 10,450

Cash (+A) .......................................................................................................... 10,450* Notes Payable (+L)..................................................................................... Recorded borrowing (in Canadian dollars). _________________ * $10,450 = Note balance of 11,000 Canadian dollars x Exchange rate of $0.95/Canadian dollar December 31, 2015 Notes Receivable (+A) ...................................................................................... Exchange Gain (Ga, +SE) ............................................................................ Earned a foreign currency exchange gain on receivable.

440* 440

Exchange Loss (Lo, –SE) .................................................................................... 440* Notes Payable (+L)..................................................................................... Incurred a foreign currency exchange loss on payable. _________________ * $440 = Adjusted value of note – Carrying value of note = (Note for 11,000 Canadian dollars x Current exchange rate of $0.99/Canadian dollar) – $10,450 December 31, 2016 Exchange Loss (Lo, –SE) .................................................................................... Notes Receivable (–A) ............................................................................... Incurred a foreign currency exchange loss on receivable.

10,450

440

990*

Notes Payable (–L) ........................................................................................... 990* Exchange Gain (Ga, +SE) ............................................................................ Earned a foreign currency exchange gain on payable. _________________ * $990 = Adjusted value of note – Carrying value of note = (Note for 11,000 Canadian dollars x Current exchange rate of $0.90/Canadian – $10,890

990

990

dollar)

Exchange gains and losses on debt and receivables for the same amount and expressed in the same foreign currency always offset each other. When the exchange rate increases, both the receivable and the payable increase by the same amount. The increase in the receivable gives rise to a gain (i.e., the company will receive more U.S. dollars), while the increase in the liability gives rise to a loss (i.e., the company must pay out more U.S. dollars). When the exchange rate decreases, both the receivable and the payable decrease by the same amount. The decrease in the receivable gives rise to a loss, while the decrease in the liability gives rise to a gain. Therefore, through hedging Outreach Inc. is able to avoid wide fluctuations in its net income from fluctuations in exchange rates.


PROBLEMS P6–1 a. No. The $285,000 is comprised of the $225,000 in savings and checking accounts and the $60,000 compensating balance. The $225,000 can properly be classified as a current asset, since no restrictions on the access or on the use of these funds exist. However, Print-O-Matic is restricted from using the $60,000 compensating balance. Whether restricted cash should be classified as a current or as a noncurrent asset depends on whether the restriction will end within the time frame of current assets or outside this time frame. In this case the $60,000 is restricted until the loan matures on October 1, 2019. Consequently, the $60,000 will not become available within the time frame of current assets, thereby not qualifying the $60,000 for classification as a current asset. The $60,000 should be classified as a noncurrent asset. b. The concept of interest expense is viewed differently by economists and accountants. Economists would define interest expense as the total cost of borrowing. Such costs would include the actual interest charged on the borrowing plus an opportunity cost incurred from borrowing. In this case, the actual interest charged on the borrowing for 2014 was $6,250. Print-O-Matic also incurred an opportunity cost. By borrowing the money, the company lost the opportunity to invest the $60,000 compensating balance. Assuming that Print-O-Matic would have invested this money in its savings and checking accounts at an annual rate of 6%, the company incurred an opportunity cost of $900 (i.e., $60,000 x 6% x 1/4). The opportunity cost should be considered when making business decisions. Accountants would define interest expense as the outflow of assets or the inflow of liabilities associated with borrowing. Since opportunity costs represent lost opportunities and do not represent outflows of assets or inflows of liabilities, opportunity costs are not properly classified as expenses. Consequently, accountants would classify only the $6,250 as interest expense for 2014.

P6–2 a. 3/3

3/8

3/11

3/28

Accounts Receivable (+A)........................................................... Sales (R, +SE) ....................................................................... Made sale on account.

1,400

Accounts Receivable (+A)........................................................... Sales (R, +SE) ....................................................................... Made sale on account.

800

Cash (+A) .................................................................................... Cash Discount (–R, –SE) ............................................................. Accounts Receivable (–A) .................................................... Collected cash from customer.

1,372 28

Cash (+A) .................................................................................... Accounts Receivable (–A) .................................................... Collected cash from customer.

800

1,400

800

1,400

800


P6–2 3/29

b.

Concluded Accounts Receivable (+A)........................................................... Sales (R, +SE) ....................................................................... Made sale on account.

1,800 1,800

The annual interest rate of forfeiting a cash discount is calculated as follows: Annual rate = Cash discount rate x (365 days ÷ Number of days receipt collected after the end of the discount period) = 2% x (365 ÷ 10 days) = 73% Since BBB can borrow money at an annual interest rate of 9%, BBB should have borrowed money at the 9% rate and paid its obligation to QNI Corporation. BBB would have saved itself some interest costs by borrowing the money rather than forfeiting the cash discount.

P6–3 a. 2013 Allowance for Doubtful Accounts (+A) ...................................................... Accounts Receivable (–A) ................................................................... Wrote off accounts deemed uncollectible. Bad Debt Charge (E, –SE)........................................................................... Allowance for Doubtful Accounts (–A) ............................................... Recognized bad debt charge. _____________ * $5,400 = Credit sales of $180,000 x Estimated uncollectible percentage of 3% 2014 Allowance for Doubtful Accounts (+A) ...................................................... Accounts Receivable (–A) ................................................................... Wrote off accounts deemed uncollectible. Bad Debt Charge (E, –SE)........................................................................... Allowance for Doubtful Accounts (–A) ............................................... Recognized bad debt charge. _____________ * $6,000 = Credit sales of $200,000 x Estimated uncollectible percentage of 3% 2015 Allowance for Doubtful Accounts (+A) ...................................................... Accounts Receivable (–A) ................................................................... Wrote off accounts deemed uncollectible. Bad Debt Expense (E, –SE)......................................................................... Allowance for Doubtful Accounts (–A) ............................................... Recognized bad debt charge. _____________ * $6,150 = Credit sales of $205,000 x Estimated uncollectible percentage of 3%

6,000 6,000

5,400* 5,400

10,000 10,000

6,000* 6,000

11,000 11,000

6,150* 6,150


P6–3

Concluded

b. January 1, 2013 balance .................................................... Write-offs during 2013 ...................................................... 2013 bad debt charge .......................................................

$10,000 (6,000) 5,400

December 31, 2013 balance.............................................. Write-offs during 2014 ...................................................... 2014 bad debt charge .......................................................

c.

$

9,400 (10,000) 6,000

December 31, 2014 balance.............................................. Write-offs during 2015 ...................................................... 2015 bad debt charge .......................................................

$

5,400 (11,000) 6,150

December 31, 2015 balance..............................................

$

550

CNG should consider increasing the percentage of credit sales that is considered uncollectible. From 2013 through 2015, write-offs exceeded bad debt expense, with the difference increasing over time. Write-offs as a percentage of credit sales increased from 3.33% ($6,000 ÷ $180,000) in 2013 to 5.37% ($11,000 ÷ $205,000) in 2015. This trend indicates that the December 31, 2015 balance in Allowance for Doubtful Accounts is probably understated, thereby causing an overstatement of the company's assets. A more appropriate bad debt percentage would be 4.5% to 5.0% of credit sales.

P6–4 a. Bad Debt Charge (E, –SE)........................................................................... Allowance for Doubtful Accounts (–A) ............................................... Recognized bad debt expense. _____________ * $49,500 = Net sales x 3% = ($1,800,000 – $130,000 – $20,000) x 3% b. Ending Allowance balance

c.

49,500

= Beginning Allowance balance + Bad Debt Charge + Recoveries – Write-Offs = $65,000 + $49,500 (from Part [a]) + $0 - $70,000 = $44,500

Bad Debt Expense (E, –SE)......................................................................... Allowance for Doubtful Accounts (–A) ............................................... Recognized bad debt expense. _____________ * $40,500 = Net sales x 3% = ($1,500,000 – $100,000 – $50,000) x 3%

d. Ending Allowance balance

49,500*

40,500*

= Beginning Allowance balance + Bad Debt Expense + Recoveries – Write-Offs = $44,500 (from Part [b]) + $40,500 (from Part [c]) +

$0 – $85,000 =

$0

40,500


P6–5 a. Ending Allowance balance

2013 Ending Allowance Balance

= Beginning Allowance balance + Bad Debt Charge + Recoveries – Write-Offs

= =

$0 + $4,200* + $0 - $3,000 $1,200

= =

$1,200 + $7,600** + $0 - $6,000 $2,800

= =

$2,800 + $9,600*** + $0 - $8,400 $4,000

=

Beginning A/R balance + Credit Sales Cash Collections – Write-Offs

=

A/R balance – Allowance balance

= = =

$0 + $105,000 - $92,000 - $3,000 $10,000 $10,000 - $1,200 = $8,800

Balance

= = =

$10,000 + $190,000 - $161,000 - $6,000 $33,000 $33,000 - $2,800 = $30,200

Balance

= = =

$33,000 + $240,000 - $214,000 - $8,400 $50,600 $50,600 - $4,000 = $46,600

*2013 credit sales x .04 2014 Ending Allowance Balance **2014 credit sales x .04 2015 Ending Allowance Balance ***2015 credit sales x .04 b. Ending A/R balance

A/R (net) 2013 Ending A/R Balance 2013 A/R (net) 2014 Ending A/R

2014 A/R (net) 2015 Ending A/R

2015 A/R (net)

c. In each of the three years shown, Albertson’s Locksmith Corporation has estimated bad debts slightly higher than the annual write-offs. The estimates are adequate and provide some cushion for future years in case of a larger-than-anticpated write-off.


P6–6 2012 2011 2010 January 1, balance ............................................................. $27,609 $32,266 $39,096 Provision for losses............................................................ 3,390 7,580 16,843 Recoveries ..................................................................... 1,843 2,266 2,109 Less: December 31, balance .............................................. ($21,936) ($27,609) ($32,266) Actual Write-offs .............................................................. $10,906 $14,503 $25,782 The 2010 provision (an expense based on an estimate of uncollectible receivables) was significantly higher due to the lingering recession. During poor economic times, banks will have more difficulty collecting loans; the higher provision and the higher balance in the allowance account reflect the economic realities. Since the recovery, the annual provision, write-offs and the ending balances have all declined.

P6–7 a.

Hadley Company Income Statement For the Year Ended December 31, 2014 Sales ...................................................................................... $ 200,000 Cost of goods sold .................................................................. 102,000 Expenses ................................................................................. 115,000* Net income (loss) .................................................................... $ (17,000) _________________ * $115,000 = $65,000 of previously reported expenses + $50,000 of bad debt expense associated with the Litzenberger account Hadley Company Balance Sheet December 31, 2014 Assets Cash .......................................... Accounts receivable, net ............ Other assets ................................ Total assets .................................

$

5,000 35,000 40,000

$ 80,000

Liabilities & Stockholders' Equity Current liabilities .................. $ 13,000 Long-term notes payable ..... 80,000 Stockholders' equity............. (13,000) Total liabilities and stockholders' equity ........... $ 80,000

After considering the adjustment for potential bad debts, Hadley generated a net loss for 2014. Therefore, it appears that Hadley Company did not have a very successful first year of operations. b.

Auditors have their own interests. They must consider factors affecting their own well-being. One item that could adversely affect auditors' well-being is being the defendant in a lawsuit. If the auditors did not require an adjustment for the Litzenberger account, and Litzenberger was subsequently unable to pay its debt, users of Hadley's financial statements could sue the auditors for any losses incurred. Conservatism, which states "when in doubt, understate rather than overstate," applies to this situation. Since the auditors are uncertain as to whether Hadley Company will ever collect the money


from Litzenberger, the auditors would prefer to understate Hadley Company's financial health rather than overstate it.

P6–7 c.

Concluded

While it is true that Litzenberger is still operating, Hadley's CFO is ignoring the revenue recognition principle and the matching principle. Under the revenue recognition principle, revenue should not be recognized if post-sales costs can not be adequately estimated (subject to materiality). In this case, the actual bad debt cost associated with Litzenberger will not occur until a subsequent period. However, if this cost cannot be adequately estimated, Hadley Company should not even recognize the revenue from the sale to Litzenberger. Assume that Hadley Company can adequately estimate the bad debt cost. In this case Hadley Company is allowed to recognize the revenue. But under the matching principle, all costs associated with generating revenue should be matched against that revenue. Hence, any costs associated with making a sale, whether incurred in the current period or in subsequent periods, should be recorded in the period of the sale. Since the bad debt cost is associated with generating revenue, Hadley Company should record the bad debt cost in the current period as an expense.

P6–8 a. The effect of the auditors’ findings on 2014 Fees Earned, Accounts Receivable, Allowance for Doubtful Accounts, current ratio, working capital, and net income can be determined as follows. Fees Earned:

Fees Earned would decrease from $240,000 to $230,000.

Accounts Receivable:

Accounts Receivable would decrease from $68,000 to $58,000.

Allowance for Doubtful Accounts: This account should have a balance equal to 10% of the new Accounts Receivable balance. The correct balance would be $5,800, or an increase of $2,400. Current Ratio: The current ratio before the auditors’ findings was 1.62 ($105,000 ÷ $65,000). Current assets after adjusting for the auditors’ findings would be $92,600 ($105,000 – $10,000 decrease in Accounts Receivable – $2,400 increase in Allowance for Doubtful Accounts). Current Liabilities would be unaffected by the auditors’ findings. Thus, the new current ratio would be 1.42 ($92,600 ÷ $65,000). Working Capital: Working capital would decrease from $40,000 ($105,000 – $65,000) to $27,600 ($92,600 – $65,000). Net Income: Net income would decrease by the reduction in Fees Earned of $10,000 and by the increase in Bad Debt Charge of $2,400. The new net income would be $2,600. b. Prior to the auditors’ findings, Finley, Ltd. was in compliance with its debt covenants. However, after adjusting the books for the auditors’ findings, Finley, Ltd. has violated both requirements of its debt covenants. The company’s current ratio has fallen to 1.42, which is below 1.5 as specified in the loan agreement. Further, the company declared dividends equal to 192.3% of the adjusted net income. It appears that one possible explanation for Finley’s decision to record the sale and not record an adequate amount for bad debts was to avoid violating its debt covenants.


P6–9 a. Current Method Bad debt charge Accounts rec. value Allowance Method Bad debt charge Accounts rec. value Note:

2011

2012

$ 10,000

$ 50,000

950,000

900,000

$ 80,000

$ 78,000

880,000

802,000

2013 $

20,000

2014 $

1,200,000

$

82,000 1,040,000

80,000

2015 $

1,175,000

$

82,560 1,012,440

90,000 1,095,000

$

79,200 943,240

Under the allowance method, the net value of Accounts Receivable equals the balance in Accounts Receivable less the balance in Allowance for Doubtful Accounts.

b. Total bad debt charge

Current Method $250,000

Allowance Method $401,760

The allowance method provides a measure of net income consistent with both the revenue recognition and the matching principles. Over the five-year period Fine Linen Service wrote off a total of $250,000 in Accounts Receivable from sales made in prior years. Under the current method, the $250,000 did not reduce revenue in the year of the sale. Hence, net income in each individual year was misstated. Under the allowance method, bad debt expense is calculated using the year’s sales. Consequently, the allowance method, while an estimate, provides better matching of expenses with the associated revenues.

P6–10 a. Bad Debt Charge (E, –SE)........................................................................... 15,000* Allowance for Doubtful Accounts (–A) ............................................... Estimated bad debt charge. _____________ * $15,000 = $750,000 December credit sales x 2% Estimated uncollectible percentage b. Ending allowance balance

= Beginning allowance balance + Annual bad debt charge + Recoveries – Write-offs = $70,000 + ($11,940,000 x 2%) + $0 – $239,000 = $69,800

c. Ending accounts receivable balance = Beginning accounts receivable balance + Credit sales + Recoveries – Cash collections – Write-Offs $3,250,000 (from the aging) = Beginning accounts receivable balance + $11,940,000 + $0 – $12,126,000 – $239,000 Beginning accounts receivable balance = $3,675,000

15,000


P6–10 Concluded d. Account Age Current 1–30 days 31–75 days > 75 days Total

Account Balance $700,000 1,200,000 550,000 800,000

Noncollection Probability 2.0% 5.5% 10.0% 25.0%

Uncollectible Amount $ 14,000 66,000 55,000 200,000 $335,000

Bad Debt Charge (E, –SE)........................................................................... 280,200* Allowance for Doubtful Accounts (–A) ............................................... Estimated bad debt charge. _____________ * $280,200 = $335,000 – Unadjusted Allowance balance = $335,000 – ($70,000 + $223,800 of monthly bad debt charge adjusting entries for January through November – $239,000 of accounts written off)

280,200

e. Some companies use the percent-of-sales method to estimate bad debts but also periodically use the aging method. Companies would use the two methods for several reasons. The percent-of-sales method is very easy and comparatively inexpensive to use. Further, the percent-of-sales method emphasizes revenues and expenses, since estimated bad debts are a function of revenues. For these reasons a company is more apt to use the percent-of-sales method than the aging method. Additionally, the aging method can be very costly and time-consuming. For all but the smallest companies, preparing an aging of Accounts Receivable requires a computerized accounting system. However, in return for this increased cost and time, the aging method provides several types of useful information. First, the aging method identifies slow-moving accounts, which may require additional collection efforts. Further, the dollar amount of any particular aging classification dictates the maximum amount that the company should expend in trying to collect the accounts. A company would generally not want to spend more to collect an account than it expects to ultimately collect. Second, an aging can help a company define its credit policies. For example, if an aging identifies a particular customer as a consistently slow payer, the company may decide to no longer extend credit to this customer. If an aging indicates a large percentage of old accounts, this information could be used to decide whether to offer cash discounts and the terms of such discounts. Finally, an aging can indicate the accuracy of the percent-of-sales method. A detailed analysis of Accounts Receivable will usually provide an accurate estimate for the required balance in Allowance for Doubtful Accounts. This estimated balance can be compared to the balance obtained using the percent-of-sales method. Significant differences would indicate a need to revise the percent of sales considered uncollectible. To obtain this information, companies will periodically prepare an aging of Accounts Receivable.


P6–11 a. Looking at the net income for the years 2013, 2014, and 2015, it is true that Ticheley has reported profit increases consistently over the last three years. However, what remains to be seen is whether the increase in the profits is due to regular recurring operations or whether it is due to some income manipulation strategy adopted by Ticheley. b. A company would establish such a system of rewarding the management only on the belief that an increase in return on Stockholders’ equity (Net Income as a % of total Stockholders’ Equity) would lead to an increase in the market price of its common stock. This belief is based on the fact that an increase in the market price of the company’s common stock (1) is what the investors are looking for and (2) is the true measure of the success of a company during any one year. c.

It appears that Ticheley is using an “income-smoothing” reporting strategy. Even though we have the data only for 3 years, the fluctuating bad debt change as a percentage of accounts receivables points to that conclusion. 2013: 2014: 2015:

2,100 2,900 1,700

÷ ÷ ÷

23,100 23,200 27,400

= = =

9.09% 12.50% 6.20%

d. Ticheley is probably using the “income-smoothing” reporting strategy to demonstrate to its stockholders that it is steadily growing. The company is setting aside more than normal (i.e., 12.5%) charge for bad debt expenses during a good year (i.e., 2014) and by dipping into the charge to cover the bad debt losses during the not so good year (i.e., 2015).

P6–12 a.

Value of Transaction In Foreign Currency 320,000 pounds 350,000 yen 500 euros 200,000 Canadian dollars

(1) (2) (3) (4) b. (1)

(2)

(3)

Exchange Rate 1 dollar/.50 pound 1 dollar/125 yen 1 dollar/.75 euro 1 dollar/1.10 Canadian dollar

Value of Transaction in U.S. Dollars $640,000.00 2,800.00 666.67 181,818.18

Accounts Receivable (+A) ................................................................ Sales (R, +SE) ............................................................................ Made a sale on account expressed in British pounds.

640,000

Notes Receivable (+A) ..................................................................... Sales (R, +SE) ............................................................................ Sold a note expressed in yen.

2,800

Inventory (+A) ................................................................................. Notes Payable (+L).................................................................... Purchased inventory for a note expressed in euros.

666.67

640,000

2,800

666.67


P6–12 (4)

c.

Concluded Inventory (+A) ................................................................................. Accounts Payable (+L) .............................................................. Purchased inventory on account expressed in Canadian dollars.

181,818.18 181,818.18

Adjusted Value Carrying Value Exchange Gain (Loss) a (1) $533,333.33 $ 640,000.00 ($106,666.67) (2) 3,043.48b 2,800.00 243.48 c (3) 588.24 666.67 (78.43) d (4) 190,476.19 181,818.18 8,658.01 ________________ a $533,333.33 = 320,000 pounds x (1 dollar/.6 pound) b $3,043.48 = 350,000 yen x (1 dollar/115 yen) c $588.24 = 500 euro x (1 dollar/.85 euro) d $190,476.19 = 200,000 Canadian x (1 dollar/1.05 Canadian)

d. Receivables and payables are stated in a particular currency, for example, in British pounds. Assume that money from the receivable/payable is to be converted into U.S. dollars. At a given point in time, one British pound can be converted into a certain number of dollars. These dollars can, in turn, be used to purchase items. At a different point in time, one British pound can be converted into a different number of dollars. Holding everything else constant, these dollars can now be used to purchase either more or less goods than before. This change in purchasing power affects a company's wealth. Changes in wealth are captured by gains and losses. Since these gains or losses arise due to fluctuations in exchange rates, they are aptly called exchange gains or exchange losses.

P6–13 a. Carrying value of receivable

= = =

Receivable in British pounds x Exchange rate 40,000 British pounds x ($1.70/1 British pound) $68,000

b. The current ratio is calculated as current assets divided by current liabilities. If International Services is to maintain a current ratio of at least 1.5, its current assets after considering the effect of exchangerate fluctuations must be 1.5 times its current liabilities. The company's current assets not affected by exchange-rate fluctuations are $12,000 ($80,000 – $68,000 from Part [a]). Therefore, the minimum acceptable exchange rate would be calculated as follows: (Current assets not affected by exchange rates + Current assets affected by exchange rates) ÷ Current liabilities = 1.5 = = =

($12,000 + Current assets affected by exchange rates) ÷ $50,000 = 1.5 [$12,000 + (Receivable in British pounds x Exchange rate)] ÷ $50,000 = 1.5 [$12,000 + (40,000 pounds x Exchange rate)] ÷ $50,000 = 1.5

Exchange rate = 1.575


P6–13 Concluded c.

For International Services to maintain a current ratio of at least 1.5, its current assets after considering the effect of exchange-rate fluctuations must be 1.5 times its current liabilities adjusted for the effects of exchange-rate fluctuations. The company's current liabilities not affected by exchange-rate fluctuations are $48,400 ($50,000 - $1,600 payable to British bank). Therefore, the minimum acceptable exchange rate would now be calculated as follows: (Current assets not affected by exchange rates + Current assets affected by exchange rates) ÷ (Current liabilities not affected by exchange rates + Current liabilities affected by exchange rates) = 1.5 = ($12,000 + Current assets affected by exchange rates) ÷ ($48,400 + Current liabilities affected by exchange rates) = 1.5 = [$12,000 + (Receivable in British pounds x Exchange rate)] ÷ [$48,400 + (Payable in British pounds x Exchange rate)] = 1.5 = [$12,000 + (40,000 pounds x Exchange rate)] ÷ [$48,400 + (1,000 pounds x Exchange rate)] = 1.5 Exchange rate = 1.574 (rounded to 3 decimals)

d. Increases in the exchange rate cause exchange gains when holding receivables and exchange losses when holding payables. Alternatively, decreases in the exchange rate cause exchange gains when holding payables and exchange losses when holding receivables. Therefore, changes in the exchange rate have opposite effects on receivables and payables. By holding a payable in the same amount as a receivable, exchange gains will perfectly offset exchange losses. In this way, a company can hedge against exchange-rate fluctuations. A company never enjoys an exchange gain but also never incurs an exchange loss.


ISSUES FOR DISCUSSION ID6–1 a. A potential investor is interested in the solvency of a company. Since solvency is associated with the availability of cash to pay off debts, an investor would want to know about any existing restrictions on a reported cash balance. b. The difference is due to the length of time the cash is restricted in its use. If the cash will remain restricted for a time period longer than one year, it should be disclosed as noncurrent. If the cash will not be available to cover current obligations, then it would be inaccurate to designate the balance as current. c. Excluding the cash balance will reduce working capital, the current ratio and the quick ratio. A user of the financial statements should rightly calculate that the solvency of the company is lowered due to the inability to use the restricted cash to cover current obligations. d. If the company had not retired the debt, the 2012 statements would have continued to show the restrictions placed on the cash balances and, therefore, the amount of unrestricted cash would be less than what actually was shown.

ID6–2 a. For a long time, IBM rightfully enjoyed the reputation as the “epitome of financial conservatism”. This reputation was built by IBM by using conservative financial reporting practices. Such a reporting strategy underestimates the revenues and overestimates the expenses. The maneuver by IBM to immediately record all the revenue from a long-term lease contract without realizing all the cash associated with the transaction is an example of liberal accounting policy (i.e., policy that tends to record the revenue as soon as possible). Such maneuvers occurring too often will surely affect IBM’s reputation as the “epitome of financial conservatism.” b. The use of accelerated revenue recognition methods has its own costs and benefits to IBM and its management. Accelerated revenue recognition methods not only inflate the sales and net income on the income statement but also tend to inflate receivables on the balance sheet. Therefore, a number of ratios, such as return on stockholders’ equity, working capital, current ratio, and return on total assets are also inflated. As a result of these inflated ratios, IBM may tend to gain in terms of its ability to comply with a number of provisions in its debt covenants. The cost of these benefits to IBM would be its image in the capital markets as the company that engages in the “window-dressing” of its financial statements. Such an impression about a company within the financial markets could lead to a higher level of scrutiny of its financial statements by financial analysts on one hand, and possibly a higher cost of capital to the firm on the other hand. The major benefits to the management of adopting such an aggressive revenue recognition strategy would be (1) higher bonus, if based on net income or any other measure that involves net income or


sales in the calculation; and (2) less pressure to worry about the credit ratings of the debt. The major cost would be increased scrutiny by the auditors to ensure that management is not fraudulent. c.

IBM’s policy of requiring all employees to swear that they have read the company’s “Business Conduct Guidelines” is in direct contrast to its own behavior. On one hand it discourages its employees from organizing financial information in a potentially deceptive way, but on the other hand the management of the company engages in aggressive revenue recognition tactics to boost the bottom-line. This is a classic example of management not “walking the talk.” Such behavior is non exemplary and sends a very subtle signal to the employees that, during hard times, ethics have no place within the company.

ID6–3 a. Working Capital 1992: 1993: 1994:

=

Current Assets

– Current Liabilities

$1,256.20 – $1,087.5 = $168.70 $1,067.60 – $1,105.1 = $ (37.50) $1,253.60 – $1,259.1 = $ (5.50)

According to the covenant, Quaker Oats Co. must maintain a Working Capital of more than $150 million. At the end of 1992, the company came very close to violating this restriction, as its Working Capital was only $168.70 million. Since then, the Working Capital has been negative, leading to the violation of the debt covenant during 1993 and 1994. b. The 1994 annual report changes the restrictions from the Working Capital to the minimum Shareholders’ Equity. The new restrictions require the company to maintain a minimum shareholders’ equity of $300 million. It seems creditors want to protect their debt to the company, and by imposing the requirement of minimum share capital they are, in essence, securitizing their debt.


ID6–4 a.

Allowance for Credit Losses 2012 2011 2010

Allowance for loan losses at beginning of year Provision for credit losses—consumer Provision for credit losses—corporate Consumer credit losses Corporate credit losses Consumer recoveries Corporate recoveries Allowance for loan losses at end of year

$30,115 $40,655 $36,033 9,796 10,254 35,494 120 (756) (13) 16,838 21,164 31,073 640 2,000 3,418 2,485 2,740 2,638 417 386 994 25,455 30,115 40,655

Across the three years, Citi is estimating less of its consumer loans to be uncollectible. Given the timing of the economic collapse and recovery, this trend is reasonable and to be expected. The company has taken into consideration the loans it will not be able to collect and adjusted its books accordingly. As consumers weather the storm of the economy, the company’s loan collection experience should improve. b. Consumer loans Corporate loans

2012 Loans $408,671 246,783

Net Write-Offs % $14,353 3.51% 223 0.09%

The above information indicates that the Consumer loan portfolio is the riskier of the two.

ID6–5 The Reserve for Loan Losses for a bank (or a mortgage company making home loans) is the same as the Allowance for Uncollectibles account for a company carrying accounts receivable from customers. The account is a contra asset account which lowers the carrying value of the loan portfolio to the amount that the company estimates it can actually collect. The account is increased when the lender takes an expense (often called a Provision for Loan Loss) to reflect the fact that addition loans have been granted and that collectability is not estimated to be 100%. A company such as New Century might be reluctant to increase the Reserve because, as noted above, increasing the account is done through an expense to earnings. Increasing the account will have the immediate effect of reducing the company’s profits. In addition, in the specific case of New Century, an increased Reserve is an acknowledgement that the company extended credit to borrowers who are either unable or unwilling to repay the obligation. A sharp increase in the reserve is an admission of a flaw in the company’s business model. Investors in New Century, however, need to know the amount of loans that will not convert into cash. Factors such as personal disposable income, housing prices, overall consumer debt levels and inflation will factor into loan defaults, especially in vulnerable markets such as sub-prime mortgage lending.

ID6–6 The Provision for Loan Loss is the expense that feeds into a bank’s Allowance for Loan Losses, the contra asset account that adjusts downward the carrying value of the bank’s loan portfolio. When banks determine that their loans are less collectible, they bring the portfolio down in value by increasing the Allowance with a higher Provision on the Income Statement.


A weakened housing market directly affects the collateral that supports the bank’s first mortgage home loan. When a homeowner purchases a home by borrowing money from a bank, the bank takes the house as collateral. If the homeowner is unable to repay the loan, the bank has the right to seize the house (in a foreclosure) and sell the house to recover its money. If housing prices have dropped, the bank will receive less money from the sale of the foreclosed home and will therefore be less likely to recoup all of its cash. To deal with this potential loss, prudent accounting calls for an increase in the Allowance (through the Provision on the Income Statement) to properly reflect the value of the loan.

In addition to the effect on the bank’s collateral, a weakened housing market may also affect the bank’s customers and their ability to repay their loans. If the housing market suffers, all the individuals who work in industries supporting that market (construction, real estate sales and development, mortgage lending, insurance, retail, etc.) will suffer and collectively will have less available cash flow for debt service. As discussed above, if bank customers have employment problems that weaken their monthly cash flow, the bank’s loans are less collectible and the bank will need to lower the value of the loans by increasing the Allowance through a Provision on the Income Statement.

Analysts will question a bank’s estimates of Uncollectibility because they understand that bank management has the incentive to keep earnings (and therefore the stock price) high. If a bank acknowledges problem loans, the bank will need to increase its Provision on the Income Statement— meaning the bank will report lower earnings. Analysts fear that bankers will understate problem loans in order to keep earnings high to support stock prices.

ID6–7 a. The Bad Debt Provision is an expense that represents management’s estimate of future uncollectible receivables. Like other expenses, the provision can be found on the Income Statement. b. The statement of cash flow reconciles net income (an accrual accounting number) with cash generated by the various areas of business activities. The operating section deals with those activities involved in the daily running of the business (like granting credit to clients and ultimately collecting cash), so activities related to receivables are included in the operating section. The provision is a non-cash expense which lowered net income for Target and Toyota but did not lower the cash levels for those companies. To convert net income into a cash number, the companies have added back those expenses (like the provision) that reduced earnings but did not reduce cash balances. The change to accounts receivable is included because the company collected a different dollar amount of cash than it booked as sales. In the case of Target and Toyota, the companies had an increase in receivables, meaning that they have not yet collected all the cash due from sales; the amount of the receivable increase is therefore subtracted away to help convert earnings to cash from operations. c. Both companies have large non-cash expenses (such as depreciation and the provision) that will cause cash to be larger than earnings. In addiiton, current assets other than receivables (for these companies, mainly inventory) may have decreased in amount, effectively providing cash for the business. Finally, some current liabilities (such as trade payables) may have increased, effectively providing cash for the companies’ operations. d. GAAP and IFRS treat receivables and bad debts in a very similar manner.


ID6–8 Selling on account to customers who will not ultimately pay cash for the purchase will boost sales and profits in the short-term but cause huge problems in the long-term. Many companies chase sales growth by granting credit (selling today with the promise of receiving cash tomorrow) to customers that have not demonstrated the ability to honor obligations. Sales will grow, but in future periods the benefit will be reduced when a large expense is booked to increase the Allowance for Uncollectibles account to cover the write-off of the bad receivable. At the time of the sale, Assets (in the form of Accounts Receivable) and Equity (from Sales, which is closed into Net Income as part of Retained Earnings) increase. Also, if the company is correctly following the Matching Principle, a portion of the Receivable is booked as an expense and as an increase to the Allowance account. However, if the entire amount of the receivable is ultimately deemed to be uncollectible (because the company was granting credit to unworthy customers), then the Allowance will need to be significantly increased with another Bad Debt Expense (which will decrease Assets and Equity). Accounts Receivable Days can be calculated by dividing the Accounts Receivable Turnover Ratio (discussed in Chapter 5) into 365 days. For example, an Accounts Receivable Turnover Ratio of 6.0 translates into Accounts Receivable Days of 60.83. The “days” represents the average time period it takes the company to collect the cash from the receivable. Low days indicates the company quickly converts the receivable into cash, while higher days implies the company is slow at collecting its cash. Analysts can track a company’s collection period by calculating A/R Days to determine the “quality” of the company’s earnings. Profits that are boosted by receivables that will ultimately be uncollectible are of low quality. Tracking the collection period is an excellent tool to determine if the company’s stated profits will hold up over time.

ID6–9 a. Sales concentration (having a large percentage of credit sales and accounts receivable with one customer) is a risk simply due to the fact that if the customer fails to pay its account then a large bad debt charge will significantly reduce earnings and operating cash flow. Having a large portion of business with one account puts a company in a vulnerable position if that customer fails. A better position would be to have receivables spread across many customers (in many different industries), lessening the impact if one customer fails to pay. b. The failure of one large account would cause a company to increase its Allowance for Uncollectibles by taking a large Bad Debt Expense. Earnings, assets and equity would all decline.


ID6–10 a. The company could reduce its bad debt reserve from $6.5 million to $5.39 million because its actual writeoffs were lower than expected. If there was an increase in the credit quality of its loans it could support a reduction in the reserve for bad debts. It could also be reducing its reserve for bad debts because its reported earnings were going to be lower than expected, and management made a decision to lower its reserve in order to pump up earnings in the reported period. In a year where sales have increased by 30% it is difficult to believe that the credit quality of its portfolio of loans has improved so much that there should be a reduction in the reserve account. If sales were flat in the reporting period this might be reasonable, but often when there is a significant increase in sales this increase may have been the result of lowering the credit standards so that more buyers could qualify to purchase time shares. b. “that’s tomorrow’s problem?” means that in future years Fairfield may have to take very large charges for bad debts. By reducing its provision this year (when sales were up 30%), Fairfield may be setting themselves up to have to take a very large charge in order to increase reserves to the appropriate level relative to sales.

ID6–11 One of the issues in the recent real estate crisis was excess capacity, a situation caused by real estate developers building too many projects (often due to the easy access those developers had to capital). With too many projects built and not enough tenants to rent and occupy spaces, the real estate firms saw lower cash flow (both from vacant properties and from depressed rents in the properties that were occupied). With less cash flow the developers were less solvent, less able to meet their obligations. Firms such as Hill International that had extended credit to their developer clients were often left with a receivable that was uncollectible. As with banks and other companies that extend credit, when a receivable is uncollectible the company takes an expense (a provision for bad debts) that reduces the carrying amount of the receivable and reduces equity (through lowered retained earnings from the expense charged to income). An analyst following the industry could “link the chain” of cash flow problems, starting from tenants who pay lower rents to developers who default on payments (due to lower rent receipts) to support firms (such as Hill International) who have bad receivables and to banks who have extended credit to both developers and those firms that support the developers.

ID6–12 a. If a company holds receivables or payables stated in a foreign currency (as would be the case for companies that operate internationally), the receivables and payables must be converted to U.S. dollars when preparing financial statements. As the value of the U.S. dollar fluctuates relative to other currencies, the value of the receivables and payables in U.S. dollars also fluctuates. This gives rise to foreign currency gains and losses, which can result in substantial variations in income and other reported values from one reporting period to the next. In addition, foreign currency gains and losses have economic consequences in that they can affect a company's stock prices, credit ratings, debt covenants, and so forth. By centralizing the treasury function, a company can better monitor its overall exchange gains and losses. Without a centralized treasury function, a company may not realize it is incurring exchange gains and losses because nobody within the company realizes that it is their responsibility to monitor such items.


b. The main strategy many U.S. companies use to reduce the risks of holding receivables or payables stated in a foreign currency is hedging. c.

Hedging involves taking a position in a foreign currency in an equal and opposite amount to a particular receivable or payable stated in that currency. For example, if IBM had a receivable for 100,000 British pounds, it could hedge its position by creating a payable for 100,000 British pounds. The reason hedging protects a company is that any loss realized from holding a receivable when currency rates fluctuate will be perfectly offset from the gain realized from holding the payable, and vice versa. Hedging is valuable to companies because without it companies would experience large fluctuations in income and reported receivables and payables. Unexpected fluctuations could cause a company to violate an existing debt covenant. Through hedging, companies are able to manage away such unexpected fluctuations and thereby decrease the chance that they will violate a debt covenant.

ID6–13 a. Google’s current ratio in 2012 was 4.22 ($60,454/$14,337), down from the 2011 level of 5.92. The company’s working capital in 2012 was $46,117 which changed from 2011’s working capital of $43,845. The changes that had the biggest impact on these numbers were the increases in Cash and Receivables and the decrease in Accounts Payable. b. Cash and cash equivalents are included. Cash equivalents refer to any short term, highly liquid investments that have an original maturity of less than three months. c.

Google’s net receivables comprise 13.0% (10.3% in 2011) of Current Assets and 8.41% (7.48% in 2011) of Total Assets as of 12/31/2012. Receivables represent a significant investment by the company and are crucial to strong cash flow; the management of those receivables, therefore, is an important part of the company’s focus. The allowance for uncollectible receivables was $581 and $133 in 2012 and 2011, respectively. The balance in the Allowance account represents 6.9% of total accounts receivable in 2012 (2.4% in 2011). The increase in the contra account would be of interest to analysts following the company, as cash collection receivables of ad revenue and Motorola hardware sales are the main source of operating cash for the company.

d. Footnote #15 indicates that 53.2% of Net Revenue comes from international sources. Google enters into forward hedging contracts to protect itself against fluctuations in foreign currencies. Translation gains and losses are reported in accumulated other comprehensive income, as part of stockholders’ equity. e. The increase in accounts receivable is an operating activity (dealing with the daily extension of credit to and collection of cash from customers) that acted as a “use” of cash, meaning the company has yet to collect some of the receivables from the credit sales on the income statement.


CHAPTER 7 MERCHANDISE INVENTORY BRIEF EXERCISES BE7–1 The inventory purchases made by Hewlett-Packard during 2012 can be calculated as follows: Beginning inventory $ 7.5 billion + Purchases X – Cost of Goods Sold 59.5 =Ending Inventory $ 6.3 billion Purchases = $58.3 billion

BE7–2 a. From the footnote it is apparent that Johnson & Johnson is a manufacturer. A retailer or a service company would not have accounts called Raw materials and supplies or a Goods in process within the detail of their inventory. These accounts are only used by manufacturing companies. b. From this disclosure it appears that Johnson & Johnson uses the FIFO inventory cost flow assumption. If a company uses LIFO it must disclose the amount of the LIFO reserve imbedded in the valuation of the inventory.

BE7–3 If General Electric used the FIFO inventory cost flow assumption instead of LIFO, its inventory balance for 2012 would be ($15.4 + 0.398) = $15.798 billion. This disclosure is useful to financial statement users because it can make it easier to compare GE’s results with a company that uses a FIFO assumption. It also tells the reader the financial statement and tax liability impact on GE if it were to switch to a FIFO assumption.

EXERCISES E7–1 (1) Since the goods were shipped FOB shipping point, legal title to the goods passes to the buyer when the goods are shipped on December 30, 2014. Since Dallas is the buyer, Dallas has legal title to the inventory as of December 31, 2014. Further, Dallas rightfully included the items in its inventory. There will be no misstatement on any of the financial statements. (2) The goods were shipped FOB shipping point, so legal title passes to the buyer when the goods are shipped on December 31, 2014. Since Dallas is the seller, not the buyer, legal title passed from Dallas on December 31, 2014. Dallas, wrongfully included the items in its ending inventory. This would result in an overstatement of inventory on the balance sheet. Assuming

1


E7–1

Concluded

that Dallas has properly recorded the sale but did not yet record the COGS, there will be an understatement of COGS on the income statement and an overstatement of net income and retained earnings. (3) Since the goods were shipped FOB destination, legal title to the goods passes to the buyer when the goods reach their destination on January 2, 2015. Since Dallas is the seller, not the buyer, Dallas has legal title to the inventory as of December 31, 2014. Dallas has rightfully included the items in its inventory. Assuming no other entries regarding the sale have been made, there will not be any misstatement on any of the financial statements. (4) The goods were shipped FOB destination, so legal title to the goods passes to the buyer when the goods reach their destination on December 31, 2014. Since Dallas is the buyer, Dallas has legal title to the inventory as of December 31, 2014. Dallas has rightfully included the items in its inventory, and assuming that the goods were correctly included in purchases as of December 31, 2014, there will not be any misstatement on any of the financial statements. (5) The goods were shipped FOB destination, so legal title to the goods passes to the buyer when the goods reach their destination on January 3, 2015. Since Dallas is the buyer, Dallas does not have legal title to the inventory as of December 31, 2014. Dallas has wrongfully included the items in its ending inventory. This would result in an overstatement of inventory on the balance sheet. Assuming that Dallas has also improperly recorded the purchases, there will be no effect on the COGS or the net income.

E7–2 10/10

Inventory (+A) ................................................................................... Accounts Payable (+L) ................................................................ Purchased inventory on account.

76,000

Inventory (+A) ................................................................................... Accounts Payable (+L) ................................................................ Purchased inventory on account.

36,000

Accounts Payable (–L) ....................................................................... Cash (–A) .................................................................................... Inventory (–A) ............................................................................ Paid supplier. _________________ * $1,520 = $76,000  2% discount

76,000

10/30

36,000

10/11

10/20

Accounts Payable (–L) ....................................................................... Cash (–A) .................................................................................... Paid supplier.

76,000

36,000

74,480 1,520*

36,000


E7–3 3/3

Inventory (+A) ................................................................................... Accounts Payable (+L) ................................................................ Purchased inventory on account.

50,000

Inventory (+A) ................................................................................... Accounts Payable (+L) ................................................................ Purchased inventory on account.

140,000

Accounts Payable (–L) ....................................................................... Cash (–A) .................................................................................... Inventory (–A) ............................................................................ Paid supplier. _________________ * $4,200 = $140,000  3% discount

140,000

4/25

50,000

3/10

3/20

50,000

140,000

Accounts Payable (–L) ....................................................................... Cash (–A) .................................................................................... Paid supplier

E7–4 12/31/10: Ending inventory: Cost of Goods Sold $13,831 Ending Inventory

= = =

Goods available for sale – Ending Inventory $16,986 – Ending Inventory $3,155

12/31/11: Goods available for sale: Goods available for sale = Cost of Goods Sold + Ending Inventory Goods available for sale = $15,693 + $3,416 Goods available for Sale = $19,109 Purchases: Purchases Purchases Purchases

= Goods available for Sale – Beginning Inventory* = $19,109 - $3,155 = $15,954

* Beginning inventory for 2011 is the Ending Inventory for 2010 12/31/12: Goods available for sale: Goods available for sale = Goods available for sale = Goods available for sale =

Beginning Inventory** + Purchases $3,416 + $16,106 $19,522

135,800 4,200*

50,000


**Beginning inventory for 2012 is the Ending inventory for 2011

Ending inventory: Ending Inventory Ending Inventory Ending Inventory

= = =

Goods available for sale – Cost of goods sold $19,522 - $15,685 $3,837

E7–5 With the perpetual method, the balance in the Cost of Goods Sold account is perpetually updated for sales of inventory, as is the balance in the Inventory account for sales and acquisitions of inventory. This implies that the balance in Cost of Goods Sold should correspond to a balance in the Inventory account of $52,000, and that no entry is necessary at the end of the year to record Cost of Goods Sold. Ending Inventory $52,000 Cost of Goods Sold

= = =

Beginning Inventory + Net Purchases – Cost of Goods Sold $32,000 + ($85,000 + $4,300) – Cost of Goods Sold $69,300

However, since the physical count indicates that Telly's has $2,000 less inventory than is recorded in its Inventory account, the following adjusting entry is necessary at the end of the year. Inventory Shrinkage (E, –SE) ............................................................. Inventory (–A) ............................................................................ Incurred inventory shrinkage.

2,000 2,000

E7–6 a. Error in Ending Inventory in 2011: The $50 understated error in the Ending inventory means that the Ending Inventory should have been $220 + $50 = $270. This would change the Cost of goods sold to $1,109 - $270 = $839 which would then increase the Gross profit to $530 ($1,369 - $839).

b. Error in Ending Inventory in 2012: = The 2011 error in the Ending Inventory changes the Beginning Inventory in 2012 and the Goods Available for sale to $270 + $983 = $1,253. To calculate the Cost of Goods Sold the Ending Inventory for 2012 is deducted from the revised Goods Available for Sale: $1,253 – ($244 - $50) = $1,059. The gross profit would then be $1,443 - $1,059 = $384.

c. Original Cost of Goods Sold Corrected Cost of Goods Sold

2011 $889 $839

2012 $ 959 $1,059

E7–7 a.

Net cash from operating activities is the sum of all cash inflows and outflows that are related to the daily running of the company’s business operations. Net income is the difference between revenues (which do not have to be collected in cash) and expenses (which do not have to be paid out in cash). It is very possible for a company to show a positive cash from operations even though its net income is negative; it is possible that many of the expenses (which were subtracted from revenue to get net income) did not require a cash outlay,


just as it is possible that cash inflows exceeded sales for the period (because, for example, cash was also collected from sales from the prior period). b. The basic form for the journal entry is: Inventory Write Down Expense (E, -SE) Inventory (-A) As shown, this expense will reduce net income, retained earnings and stockholders’ equity but will NOT reduce cash. To reconcile net income (revenue minus expenses) with cash from operations (all cash flows related to operations), all non-cash expenses should be added back in the statement of cash flow. c.

If Sony’s change to inventory (160,432) is being added back to net income in the statement of cash flow, then the change must represent a “source” of cash, which would mean that the inventory decreased from one year to the next. When assets decrease, they free up cash, while asset increases “use” cash.

E7–8 a. If Marian wants to maximize profits and ending inventory, she should sell the customer the lowest priced coat (i.e., Coat 4). If she sells Coat 4, Marian would report the following gross profit and ending inventory.

Revenues COGS of Coat 4 Gross profit

Gross Profit $ 12,000 6,800 $ 5,200

Ending Inventory Coat 1 Coat 2 Coat 3 Total

$

8,400 7,100 7,600 $ 23,100

Marian may have several reasons to maximize profits and ending inventory. If Marian's Furs has borrowed money and entered into debt covenants, the debt covenants may contain clauses stipulating a certain current ratio, debt/equity ratio, and so forth. By maximizing profits and inventory, Marian can also minimize the probability that she will violate one of these ratios, thereby decreasing the chance that she will violate her debt covenants. Further, if Marian has a bonus linked to accounting earnings, she could maximize her bonus by maximizing profits. b. If Marian wants to minimize profits and ending inventory, she should sell the customer the highest priced coat (i.e., Coat 1). If she sells Coat 1, Marian would report the following gross profit and ending inventory. Gross Profit Revenues COGS of Coat 1 Gross profit

$ 12,000 8,400 $ 3,600

Ending Inventory Coat 2 $ 7,100 Coat 3 7,600 Coat 4 6,800 Total $ 21,500

The most likely reason Marian would want to minimize profits and ending inventory is to minimize taxes. Minimizing profits would minimize current tax payments, thereby minimizing the present value


of tax payments. Further, some states charge taxes on a company's assets, thereby providing an incentive to minimize assets.

E7–9 a. FIFO cost flow assumption: Cost of Goods Sold

Gross Profit

= = =

= = =

(75 units  $450) + (50 units  $500) + (5 units  $600) $33,750 + $25,000 + $3,000 $61,750

Sales – Cost of Goods Sold (130 units  $1,000) – $61,750 $68,250

Ending Inventory

= =

(60 units  $600) $36,000

Averaging cost flow assumption: Cost per Unit

= [(75 units  $450) + (50 units  $500) + (65 units  $600)] ÷ (75 units + 50 units + 65 units) ($33,750 + $25,000 + $39,000) ÷ 190 units = $514.47 per unit (rounded) = (130 units  $514.47) = $66,881.10

Cost of Goods Sold

Gross Profit

= = =

Ending Inventory

Sales – Cost of Goods Sold (130 units  $1,000) – $66,881.10 $63,118.90 = =

60 units  $514.47 $30,868.20

LIFO cost flow assumption: Cost of Goods Sold

Gross Profit

= = =

Ending Inventory

= = =

(65 units  $600) + (50 units  $500) + (15 units  $450) $39,000 + $25,000 + $6,750 $70,750

Sales – Cost of Goods Sold (130 units  $1,000) - $70,750 $59,250 = =

(60 units  $450) $27,000


E7–9

Concluded

b. If the monitors are identical, customers would be indifferent between any two monitors. Hence, Vinnie could simply give a customer the monitor that allows him to either minimize or maximize cost of goods sold, thereby maximizing or minimizing gross profit. If Vinnie wants to maximize net income, he would first sell to customers the lowest-priced monitors, followed by the second lowest-priced monitors, and so forth. Since the cost of the monitors is increasing, this strategy is identical to the FIFO cost flow assumption. Therefore, the highest gross profit Vinnie could report is $68,250 (from part [a]). Vinnie may want to maximize net income for several reasons. First, if Vinnie receives any incentive compensation, such as a bonus that is tied to net income, then he can maximize his compensation by maximizing net income. Second, if Vinnie has any existing debt covenants, they may specify a maximum debt/equity ratio. By increasing net income, Vinnie would increase equity, thereby decreasing his debt/equity ratio. In this manner, Vinnie decreases the probability that he will violate the debt covenant. Finally, if Vinnie is in the process of trying to obtain debt, potential creditors may use net income as a factor in determining whether or not to loan money to Vinnie or what interest rate to charge. If Vinnie wants to minimize net income, he would first sell to customers the highest-priced monitors, followed by the second highest-priced monitors, and so forth. Since the cost of the monitors is increasing, this strategy is identical to the LIFO cost flow assumption. Therefore, the lowest gross profit Vinnie could report is $59,250 (from part [a]). The most likely reason Vinnie would want to minimize net income is for tax purposes. If he uses the same set of books for tax and financial reporting purposes, then by minimizing book income, Vinnie minimizes taxable income. Minimizing taxable income, in turn, minimizes the present value of cash outflows for taxes.

E7–10 2013 Cost of goods sold Gross profit (Sales – COGS) Ending inventory

FIFO

Weighted Average

160 290

170 280 170

2014

FIFO

Weighted Average

Cost of goods sold Gross profit (Sales – COGS) Ending inventory

245

262.5 437.5 262.5

290

180 270

180

455

LIFO

160 LIFO 290 410 225

If the business is growing (inventory levels rising) and the cost of inventory is increasing, then if LIFO is chosen, the company will lower its net income which will reduce its tax liability. This increases the cash flow of the company. Using FIFO will increase its reported net income and tax liability but will also increase its current assets. This choice impacts the company’s operating and liquidity ratios.


E7–11 a. LIFO cost flow assumption: Year 2011 2012 2013 2014 2015

Calculation 5,000 units  $12 (12,000 units  $16) + (4,000 units  $12) 2,000 units  $18 10,000 units  $21 (2,000 units  $23) + (3,000 units  $18) + (1,000 units  $12)

Total

Amount $ 60,000 240,000 36,000 210,000 112,000 $ 658,000

FIFO cost flow assumption: Year 2011 2012 2013 2014 2015 Total

Calculation 5,000 units  $12 (5,000 units  $12) + (11,000 units  $16) (1,000 units  $16) + (1,000 units  $18) (4,000 units  $18) + (6,000 units  $21) (4,000 units  $21) + (2,000 units  $23)

Amount $ 60,000 236,000 34,000 198,000 130,000 $ 658,000


E7–11

Concluded

Averaging cost flow assumption: Year 2011

Cost/unit

2012

COGS Cost/unit

2013

COGS Cost/unit

2014

COGS Cost/unit

2015

COGS Cost/unit COGS

Total

= = = = = = = = = = = = = = =

Calculation $120,000 ÷ 10,000 units $12 per unit 5,000 units  $12 [(5,000  $12) + (12,000  $16)] ÷ 17,000 units $14.82 per unit 16,000 units  $14.82 [(1,000  $14.82) + (5,000  $18)] ÷ 6,000 units $17.47 per unit 2,000 units  $17.47 [(4,000  $17.47) + (10,000  $21)] ÷ 14,000 units $19.99 per unit 10,000 units  $19.99 [(4,000  $19.99) + (2,000  $23)] ÷ 6,000 units $20.99 per unit 6,000 units  $20.99

Amount

$

60,000

237,120

34,940

199,900

125,940 $ 657,900 (rounded)

b. Over the life of a company, Cost of Goods Sold would be the same regardless of the cost flow assumption employed. Over the life of a business, all the units of inventory will be sold. Consequently, all costs associated with inventory will be expensed. The choice of a cost flow assumption affects only the allocation of inventory costs to particular accounting periods; it does not affect total inventory costs. c.

Assume that accounting earnings equals tax earnings. Over the life of a business, a company's total earnings are the same regardless of the cost flow assumption employed. Therefore, a company's total tax liability over the company's life is the same, regardless of the cost flow assumption employed, as long as tax rates are unchanged. The choice of a cost flow assumption does, however, affect the allocation of inventory costs to particular years. These different cost allocations give rise to different earnings in particular years. The different earnings amounts under different cost flow assumptions then give rise to different tax liabilities (i.e., cash outflows) in particular years. Due to the time value of money, the timing of cash flows affects the present value of the total tax payments. In periods of inventory build-up, the LIFO cost-flow assumption will result in lower earnings while FIFO will result in higher earnings. The opposite is true in times of inventory liquidation. Consequently, LIFO results in lower tax payments when a company builds up its inventories and FIFO results in higher tax payments. The timing of the tax payments means that the present value of tax payments under LIFO is less than the present value of tax payments under FIFO. In times of deflation, the opposite situation arises. The present value of tax payments under FIFO is less than the present value of tax payments under LIFO.


E7–12 a. Inventories on LIFO basis ............................................................. Add: Adjustment to LIFO basis ..................................................... Inventories on FIFO basis .............................................................

$15,547 2,750 $ 18,297

b. Accumulated tax savings can be computed by multiplying the tax rate by the total decrease in net income due to LIFO adoption. Accumulated Tax Savings

c.

= = =

Tax Rate  (2012 LIFO Reserve) .31  ($2,750) $852.50

The 2012 reported net income under the FIFO cost flow assumption would be $5.907.32 ($5,681 + ($2,750 – 2,422)(1-.31)) if Caterpillar had chosen to change from LIFO to FIFO years earlier. d. The information generated in parts (a), (b), and (c) could be useful to the users from several perspectives. First, users could use the information to compare Caterpillar with other companies within the industry that use FIFO cost flow assumption. Second, the users can readily see the tax savings that the company has generated as a result of its choice of LIFO cost flow assumption. Thirdly, along with other information, users can use this information to assess the quality of earnings of Caterpillar. e. Under IFRS the last-in, first-out (LIFO) inventory cost flow assumption is prohibited. The cost of inventory generally is determined using the first-in, first-out (FIFO) or averaging assumption. Caterpillar would have to abandon its LIFO method and the related benefits.

E7–13 a. Loss on Inventory Write-down (Lo, –SE) ................................................... Inventory (–A) ..................................................................................... Wrote inventory down to market value. Cash (+A) ............................................................................................. Sales (R, +SE)....................................................................................... Sold Item #1.

12 12

50 50

Cost of Goods Sold (E, –SE) ....................................................................... Inventory (–A) ..................................................................................... Recorded cost of goods sold for Item #1. Cash (+A)

.............................................................................................

28 28 50


Sales (R, +SE)....................................................................................... Sold Item #2. Cost of Goods Sold (E, –SE) ....................................................................... Inventory (–A) ..................................................................................... Recorded cost of goods sold for Item #2.

E7–13

40 40

Concluded

b. Item #1 Item #2 c.

50

2014 $12 loss 0

2015 $22 profit 10 profit

Total $10 profit 10 profit

As demonstrated in Part (b) for Item #1, a company can trade off a loss in one period for increased profits in a later period. This implies that if a company is having a good year, it can hide some of those profits by writing down its inventory and then recognize increased profits in future periods when the company's profits may be lower.

E7–14 a. Unilever is a manufacturer. Manufacturing companies carrying raw materials inventory in addition to finished goods inventory, while retailers only carry finished goods. b. Unilever uses the First-In, First-Out (FIFO) method; under IFRS, the Last-In, First-Out (LIFO) method is prohibited. c. An inventory writedown is an adjustment to the carrying value of inventory when the market value has decreased below the cost of the inventory. An inventory recovery is an adjustment to the carrying value of inventory when the market value of inventory previously written down has increased; the recovery restores some of the amount previously lost due to the writedown. The appropriate journal entries for the writedown and recovery are: Inventory Writedown Expense (E, -SE) Inventory (-A)

131

Inventory (+A) Inventory Recovery (R, +SE)

71

131

71

d. If Unilever used U.S. GAAP instead of IFRS, the company would have the option to use LIFO as an inventory method. Secondly, the company would use a different market valuation for its inventory when determining the amount of any writedown expense. (Under GAAP, the market value is normally the replacement cost; under IFRS, the market value is normally the realizable value, the amount at which the inventory could be sold.) Finally, the company would not be allowed to book the recovery of inventory value; under GAAP, inventory may be written down but is never allowed to be written up in value.


PROBLEMS P7–1 11/15

11/26

12/2

12/2

Inventory (+A) ................................................................................... Accounts Payable (+L) ................................................................ Purchased inventory on account.

8,000

Inventory (+A) ................................................................................... Accounts Payable (+L) ................................................................ Purchased inventory on account.

12,000

Accounts Payable (–L) ....................................................................... Cash (–A) .................................................................................... Paid supplier.

8,000

Accounts Payable (–L) ....................................................................... Cash (–A) .................................................................................... Inventory (–A) ............................................................................ Paid supplier.

12,000

Inventory (+A) ............................................................................ Accounts Payable (+L) ......................................................... Purchased inventory on account.

30,000

Inventory (+A) ............................................................................ Accounts Payable (+L) ......................................................... Purchased inventory on account.

60,000

Accounts Payable (–L) ................................................................ Cash (–A) ............................................................................. Inventory (–A) ..................................................................... Paid supplier.

30,000

8,000

12,000

8,000

11,760 240

P7–2 a.

3/5

3/10

3/13

30,000

60,000

29,400 600


7/18

P7–2 b.

3/10

Accounts Payable (–L) ................................................................ Cash (–A) ............................................................................. Paid supplier.

60,000 60,000

Concluded Inventory (+A) ............................................................................ Accounts Payable (+L) ......................................................... Purchased inventory on account.

60,000

Accounts Payable (–L) ................................................................ Cash (–A) ............................................................................. Inventory (–A) ..................................................................... Paid supplier. _____________ * $800 = ($60,000  2/3)  2% discount

40,000

8/7

20,000

3/19

Accounts Payable (–L) ................................................................ Cash (–A) ............................................................................. Paid supplier.

60,000

39,200 800*

20,000

P7–3 The correct amount that should be reported for Cost of Goods Sold is calculated using the following formula. Error in Ending Inventory = Error in Beginning Inventory + Error in Purchases – Error COGS 2010: $500 = $0 + $0 – Error in COGS Error in COGS = ($500). Therefore, COGS as reported is understated $500. Correct COGS = $4,366 + $500 = $4,866 2011: ($150) Error in COGS Correct COGS

= $500 + $0 – Error in COGS = $650: COGS as reported is overstated $650. = $5,068 – $650 = $4,418

2012: $320 Error in COGS

= ($150) + $0 – Error in COGS = ($470). Therefore, COGS as reported is understated $470.

in


Correct COGS

= $4,796 + $470 = $5,266

The restated income statements follow:. 2012 $ 22,603 5,266 $ 17,337 13,718 $ 3,619

Sales Cost of goods sold Gross profit Expenses Net income

2011 $ 24,287 4,418 $ 19,869 14,871 $ 4,998

2010 $ 23,076 4,866 $ 18,210 13,640 $ 4,570

P7–4 a. Cost of Goods Available for Sale

Number of Units Available for Sale

FIFO: Ending Inventory

Cost of Goods Sold

LIFO: Ending Inventory

Cost of Goods Sold

Averaging: Cost per Unit

= =

= Cost of Goods in Beginning Inventory + Cost of Goods Purchased = (15,000 units x $1) + (6,000 units  $1.30) + (9,000 units  $1.50) + (7,000 units  $1.60) = $47,500 = Number of Units in Beginning Inventory + Number of Units Purchased = 15,000 + 22,000 = 37,000 units

(7,000 units  $1.60) + (4,000 units  $1.50) $17,200 = Cost of Goods Available for Sale – Ending Inventory = $47,500 – $17,200 = $30,300

= (11,000 Units  $1.00) = $11,000 = = =

Cost of Goods Available for Sale – Ending Inventory $47,500 – $11,000 $36,500

= Cost of Goods Available for Sale ÷ Number of Units Available for Sale = $47,500 ÷ 37,000 Units = $1.284 per Unit

Ending Inventory

Cost of Goods Sold

= Number of Units in Ending Inventory  Cost per Unit = 11,000 units  $1.284 per unit = $14,124 = =

Cost of Goods Available for Sale – Ending Inventory $47,500 – $14,124


=

P7–4

$33,376

Continued Lumbermans and Associates Income Statements For the Year Ended December 31, 20XX

Sales Cost of goods sold Gross profit Other expenses Income before taxes Income taxes Net income

FIFO $ 55,000 30,300 $ 24,700 15,000 $ 9,700 2,910 $ 6,790

Averaging $ 55,000 33,376 $ 21,624 15,000 $ 6,624 1,987 $ 4,637

LIFO $ 55,000 36,500 $ 18,500 15,000 $ 3,500 1,050 $ 2,450

b. By using LIFO rather than FIFO, Lumbermans and Associates would save $1,860 ($2,910 – $1,050) in taxes. c.

Ending inventory at market value = 11,000 units  $1.35 per unit = $14,850 Lower-of-cost-or-market value: Cost Market value Excess of cost over market value (cannot be negative)

FIFO $ 17,200 14,850

Averaging $ 14,124 14,850

LIFO $11,000 14,850

2,350

0

0

FIFO method: Loss on Inventory Write-down (Lo, –SE) ........................................................ Inventory (–A) .......................................................................................... Adjusted inventory to LCM. Averaging method: No entry is necessary. LIFO method: No entry is necessary.

2,350 2,350


d. Cost of Goods Available for Sale

=

Cost of Goods in Beginning Inventory + Cost of Goods Purchased = (15,000 units  $1.60) + (6,000 units  $1.40) + (9,000 units  $1.30) + (7,000 units  $1.20) = $52,500

Number of Units Available for Sale

P7–4

= Number of Units in Beginning Inventory + Number of Units Purchased = 15,000 + 22,000 = 37,000 units

Concluded

FIFO: Ending Inventory

Cost of Goods Sold

= (7,000 units  $1.20) + (4,000 units  $1.30) = $13,600 = = =

Cost of Goods Available for Sale – Ending Inventory $52,500 – $13,600 $38,900

LIFO: Ending Inventory

Cost of Goods Sold

= 11,000 units  $1.60 = $17,600 = = =

Cost of Goods Available for Sale – Ending Inventory $52,500 – $17,600 $34,900

Averaging: Cost per Unit

= Cost of Goods Available for Sale ÷ Number of Units Available for Sale = $52,500 ÷ 37,000 units = $1.419 per unit

Ending Inventory

Cost of Goods Sold

= Number of Units in Ending Inventory  Cost per Unit = 11,000 units  $1.419 per unit = $15,609 = Cost of Goods Available for Sale – Ending Inventory = $52,500 – $15,609 = $36,891 Lumbermans and Associates Income Statements


For the Year Ended December 31, 20XX FIFO $ 55,000 38,900 $ 16,100 15,000 $ 1,100 330 $ 770

Sales Cost of goods sold Gross profit Other expenses Income before taxes Income taxes Net income

Averaging $ 55,000 36,891 $ 18,109 15,000 $ 3,109 933 $ 2,176

LIFO $ 55,000 34,900 $ 20,100 15,000 $ 5,100 1,530 $ 3,570

LIFO gives rise to the highest net income in this case. Under FIFO, the oldest costs flow into COGS before the most recent costs. Under LIFO, the most recent costs flow into COGS before the older costs. Under the averaging method, all the costs are averaged to determine COGS. In this case, the cost of the inventory is decreasing, so the LIFO cost flow assumption uses lower, newer costs in computing COGS than the other two methods. Since these lower costs flow into COGS under LIFO, the older, higher costs flow into ending inventory.

P7–5 a. Cost of Goods Available for Sale

= Cost of Goods in Beginning Inventory + Cost of Goods Purchased = (500 units x $70) + (1,000 units  $75) + (3,000 units  $80) + (4,000 units  $82) = $678,000

Number of Units Available for Sale

= Number of Units in Beginning Inventory + Number of Units Purchased = 500 + 8,000 = 8,500 units

Units Sold Units remaining in Inventory

= =

FIFO: Ending Inventory

Cost of Goods Sold

LIFO: Ending Inventory

Cost of Goods Sold

Averaging:

= =

6,000 units 2,500 units

2,500 units  $82 $205,000 = Cost of Goods Available for Sale – Ending Inventory = $678,000 – $205,000 = $473,000

= (500 units  $70) + (1,000 units x $75) + (1,000 units x $80) = $190,000 = = =

Cost of Goods Available for Sale – Ending Inventory $678,000 – $190,000 $488,000


Cost per Unit

= Cost of Goods Available for Sale ÷ Number of Units Available for Sale = $678,000 ÷ 8,500 Units = $79.76 per Unit

Ending Inventory

Cost of Goods Sold

P7–5

= Number of Units in Ending Inventory  Cost per Unit = 2,500 units  $79.76 per unit = $199,400 = = =

Cost of Goods Available for Sale – Ending Inventory $678,000 – $199,400 $478,600

Continued Laundryman’s Corporation Income Statements For the Year Ended December 31, 20XX FIFO $ 900,000 473,000 $ 427,000 125,000 $ 302,000 90,600 $ 211,400

Sales Cost of goods sold Gross profit Other expenses Income before taxes Income taxes Net income

Averaging $ 900,000 478,600 $ 421,400 125,000 $ 296,400 88,920 $ 207,480

LIFO $ 900,000 488,000 $ 412,000 125,000 $ 287,000 86,100 $ 200,900

b. By using LIFO rather than FIFO, Laundryman’s would save $4,500 ($90,600 – $86,100) in taxes. c.

Ending inventory Cost

Market /unit Writedown

FIFO 2,500 units @ $82

$78

Averaging LIFO 2,500 units @ $79.76 500 units @ $70 1,000 units @ $75 1,000 units @ $80 $78 $78

2,500 x ($82 - $78) 2,500 x ($79.76 - $78)

1,000 x ($80 - $78)


FIFO method: Loss on Inventory Write-down (Lo, –SE) ........................................................ Inventory (–A) .......................................................................................... Adjusted inventory to LCM.

10,000

Averaging method: Loss on Inventory Write-down (Lo, –SE) ........................................................ Inventory (–A) .......................................................................................... Adjusted inventory to LCM. LIFO method: Loss on Inventory Write-down (Lo, –SE) ........................................................ Inventory (–A) .......................................................................................... Adjusted inventory to LCM.

10,000

4,400 4,400

2,000

P7–5 Concluded d. Cost of Goods Available for Sale

=

Cost of Goods in Beginning Inventory + Cost of Goods Purchased = (500 units x $80) + (1,000 units  $78) + (3,000 units  $77) + (4,000 units  $75) = $649,000

Number of Units Available for Sale

= Number of Units in Beginning Inventory + Number of Units Purchased = 500 + 8,000 = 8,500 units

Units Sold Units remaining in Inventory FIFO:

= =

Ending Inventory

Cost of Goods Sold

6,000 units 2,500 units

= (2,500 units  $75) = $187,500 = = =

Cost of Goods Available for Sale – Ending Inventory $649,000 – $187,500 $461,500

LIFO: Ending Inventory

Cost of Goods Sold

= (500 units  $80) + (1,000 units x $78) + (1,000 units x $77) = $195,000 = = =

Cost of Goods Available for Sale – Ending Inventory $649,000 – $195,000 $454,000

2,000


Averaging: Cost per Unit

= Cost of Goods Available for Sale ÷ Number of Units Available for Sale = $649,000 ÷ 8,500 units = $76.35 per unit

Ending Inventory

Cost of Goods Sold

= Number of Units in Ending Inventory  Cost per Unit = 2,500 units  $76.35 per unit = $190,875 = Cost of Goods Available for Sale – Ending Inventory = $649,000 – $190,875 = $458,125

Laundryman’s Corporation Income Statements For the Year Ended December 31, 20XX

Sales Cost of goods sold Gross profit Other expenses Income before taxes Income taxes Net Income

FIFO $ 900,000 461,500 $ 438,500 125,000 $ 313,500 94,050 $219,450

Averaging $ 900,000 458,125 $ 441,875 125,000 $ 316,875 95,063 $221,812

LIFO $ 900,000 454,000 $ 446,000 125,000 $ 321,000 96,300 $224,700

Because Cost of Goods Sold is the lowest under LIFO due to deflation, LIFO yields the highest net income in this case. Under FIFO, the oldest costs flow into COGS before the most recent costs. Under LIFO, the most recent costs flow into COGS before the older costs. Under the averaging method, all the costs are averaged to determine COGS. In this case, the cost of the inventory is decreasing, so the LIFO cost flow assumption uses lower, newer costs in computing COGS than the other two methods. Since these lower costs flow into COGS under LIFO, the older, higher costs flow into ending inventory.

P7–6 a. LIFO cost flow assumption: (1)

1/3

Purchases (+A) ......................................................................

140,000


Accounts Payable (+L) .................................................... Purchased inventory on account. (2)

Cash (+A) ............................................................................... Sales (R, +SE) .................................................................. Made cash sales.

100,000

Accounts Receivable (+A)...................................................... Sales (R, +SE) .................................................................. Made sales on account.

200,000

Accounts Payable (–L) ........................................................... Cash (–A) ........................................................................ Purchase Discount (–A) .................................................. Made payment to supplier. _____________ * $2,800 = $140,000  2% discount

140,000

(5)

248,500

(3)

(4)

(6)

1/3

140,000

1/9

1/10

1/15

1/19

Purchases (E, –SE) ................................................................. Cash (–A) ........................................................................ Accounts Payable (+L) .................................................... Purchased inventory. Purchases (+A) ...................................................................... Accounts Payable (+L) .................................................... Purchased inventory.

(7) 1/23

100,000

200,000

137,200 2,800*

73,500 175,000

182,000 182,000

Accounts Payable (–L) ........................................................... Cash (–A) ........................................................................ Purchase Discount (–A) .................................................. Made payment to supplier. _____________ * $1,750 = ($175,000 x ½)  2% discount

87,500

(8)

Purchases (+A) ...................................................................... Cash (–A) ........................................................................ Purchased inventory on account.

112,000

Accounts Payable (–L) ........................................................... Cash (–A) ........................................................................ Made payment to supplier.

87,500

(9)

1/27

1/28

85,750 1,750*

112,000

87,500


P7–6

Continued

(10) 1/28

Accounts Payable (–L) ........................................................... Cash (–A) ........................................................................ Purchase Discount (–A) .................................................. Made payment to supplier. _____________ * $3,640 = $182,000  2% discount

182,000

(11) 1/29

Cash (+A) ............................................................................... Sales (R, +SE) .................................................................. Made cash sales.

360,000

Accounts Receivable (+A)...................................................... Sales (R, +SE) .................................................................. Made sales on account.

300,000

Purchases (+A) ...................................................................... Cash (–A) ........................................................................ Purchased inventory.

60,000

Freight-In (+A) ....................................................................... Accounts Payable (+L) .................................................... Incurred freight costs on inventory.

30,000

(12) 1/30

(13) 1/31

(14) 1/31

a. LIFO cost flow assumption

178,360 3,640*

360,000

300,000

60,000

30,000

....................

Adjusting entry 1/31 Inventory (ending) ................................................................ 393,250* Cost of Goods Sold ................................................................ 466,060 Purchase Discount ................................................................ 8,190 Purchases ....................................................................... Freight-In ........................................................................ Inventory (beginning) ..................................................... Recorded COGS and ending inventory. _____________ * Units in Ending Inventory = Units in Beginning Inventory + Units Purchased – Units Sold 18,000 = 5,000 + 30,000 – 17,000: Cost of units in inventory using LIFO: $393,250 = (5,000 units  $19.00) + (7,000 units  $20.60) + (6,000 units  $25.675) The unit costs used to calculate the $393,250 were taken from the following table. Date Number of Units Beg. Inv. 5,000 1/3 7,000 1/15 10,000 1/19 7,000 1/27 4,000 1/31 2,000 __________________

Unit Cost $19.00 20.00 24.85b 26.00 28.00 30.00

Unit Freighta $0.00 1.00 1.00 1.00 1.00 1.00

Unit Discount $ 0.00 0.40 0.175c 0.52 0.00 0.00

742,500 30,000 95,000

Total Unit Cost $19.00 20.60 25.675 26.48 29.00 31.00


P7–6

Concluded a $1.00 = $30,000 freight bill ÷ 30,000 units purchased b $24.85 unit cost = [(3,000  $24.50) + (7,000  $25.00)] ÷ 10,000 units c

$0.175 unit discount = Total discount of $1,750 ÷ 10,000 units

b. FIFO cost flow assumption: All entries throughout January would be identical under the FIFO and LIFO cost flow assumptions using the periodic method. The only difference would be in the adjusting entry to record COGS and ending inventory. Adjusting entry Inventory (ending)............................................................... 491,735* Cost of Goods Sold .............................................................. 367,575 Purchase Discount ............................................................... 8,190 Purchases ....................................................................... Freight-In ........................................................................ Inventory (beginning) ..................................................... Recorded COGS and ending inventory. _________ * The computations for ending inventory are based upon the table used in part (a) $491,735 = (2,000 units  $31.00) + (4,000 units  $29.00) + (7,000 units  $26.48) (5,000 units  $25.675) 1/31

742,500 30,000 95,000

+

P7–7 a.

Current Assets $22,406a 21,411b

÷ ÷ ÷

Current Liabilities $24,262 24,262

FIFO LIFO Decrease _____________ a $22,406 = $14,911 in cash + $7,495 in inventory b $21,411 = $14,911 in cash + $6,500 in inventory b.

= = =

Current Ratio .92 .88 .04

FIFO Sales Cost of goods sold: Beginning inventory Purchases Cost of goods available Ending inventory Cost of goods sold Gross profit Expenses Income before taxes Income tax Net income

LIFO $ 67,224

$

6,285 22,868 $ 29,153 7,495

$ 67,224 $

6,285 22,868 $ 29,153 6,500 21,658 $ 45,566 31,791 $ 13,775 3,261 $ 10,514

22,653 $ 44,571 31,791 $ 12,780 3,067 $ 9,713


P7–7

Concluded

Change in gross profit = $45,566 – $44,571 = $995 Change in net income = $10,514 – $9,713 = $801 c.

Tax dollars saved = $3,261 – $3,067 = $194

d. Using LIFO can have several disadvantages. First, LIFO requires a company to maintain records for older inventory acquisitions. This practice usually results in higher bookkeeping costs. Second, to avoid "eating into" a LIFO layer, which would result in older, lower inventory costs flowing into COGS and raising the company's net income and associated tax liability, managers may purchase inventory at a time or at a cost that is not advantageous to the company. Third, LIFO can adversely affect a company's and/or manager's contracts. A company's debt covenants may stipulate a minimum current ratio, or level of working capital. These both would be lower under LIFO than under FIFO (assuming inflation). Also, using LIFO reduces net income during inflationary periods. If a manager has an incentive contract linked to net income, the manager's compensation would decrease. Finally, the lower net income achieved under LIFO may mislead current and potential investors into believing that the company is performing poorly (although some current research indicates that this last point is not likely).

P7–8 a. Ending Inventory, 12/31/2014: LIFO layers: 2001 4,000 units x $5 per unit

=

$

20,000

b. Ruhe Auto Supplies Income Statement For the Year Ended December 31, 2014 Revenue ....................................................................................... Cost of goods sold: Beginning inventory ................................................................ $ 112,500 Purchases................................................................................. 902,500a Cost of goods available for sale .............................................. $ 1,015,000 Ending inventory ..................................................................... 20,000b Cost of goods sold ................................................................... Gross profit .................................................................................. Operating expenses ..................................................................... Income before income taxes ....................................................... Income taxes ................................................................................ Net income................................................................................... ___________ a $902,500 = 9,500 units purchased during 2014  $95 per unit b $ 20,000 = 4,000 units from 2001  $5 per unit The company's income tax liability is $361,500, and its net income is $843,500.

$ 3,000,000

995,000 $ 2,005,000 800,000 $ 1,205,000 361,500 $ 843,500


P7–8

Concluded

c. Revenue ....................................................................................... $ 3,000,000 Cost of goods sold: Beginning inventory ................................................................ $ 112,500 Purchases................................................................................. 1,900,000a Cost of goods available for sale ............................................... $2,012,500 Ending inventory ..................................................................... 112,500b Cost of goods sold ................................................................... 1,900,000 Gross profit .................................................................................. $ 1,100,000 Operating expenses ..................................................................... 800,000 Net income before taxes.............................................................. $ 300,000 Income taxes ................................................................................ 90,000 Net income................................................................................... $ 210,000 ___________ a $1,900,000 = (9,500 units + 10,500 units)  $95 per unit b $ 112,500 = (14,000 units  $5) + (500 units  $85) Purchasing an additional 10,500 units of inventory at $95 per unit on December 31, 2014 would cost Ruhe Auto Supplies $997,500. By incurring these costs, the company would save only $271,500 in taxes (i.e., $361,500 from part [b]) – $90,000). So on the face of it, it appears that it would not be a wise decision to acquire these additional units of inventory. However, if Ruhe Auto Supplies was planning to acquire additional inventory early in 2015 anyway, then it might not be a bad decision to acquire the inventory at the end of 2014 to lower the company's taxes.

P7–9 a.

Brady’s 2014 reported income under LIFO .................................................. LIFO Layer Liquidation during 2014 (net of income taxes) .......................... FIFO Based Net Income After Taxes .............................................................

$ 42,700 – 5,200a $ 37,500

a = $8,000 x (1- .35) = $5,200, after tax impact of no LIFO liquidation during 2014.

Brady has gone from reporting higher net income to having lower net income. b. Restatement of Brady’s 2014 reported income, if it had always been a FIFO user, can be computed as follows: Brady’s 2014 reported income under LIFO ................................................ $ 42,700 Decrease in LIFO Reserve (net of income taxes) ..................................... – 845a LIFO Layer Liquidation during 2014 (net of income taxes) ...................... – 5,200b FIFO Based Net Income After Taxes ........................................................ $ 36,655 a = ($4,800 – $3,500)  1 – .35) = $845 b = $8,000  (1– .35) = $5,200

According to the analysis given above, Brady’s restated reported income is $36,655 which is lower than Danner’s reported net income. The reason Brady’s income under FIFO is lower than under LIFO is due to the decline in the LIFO reserve and LIFO layer liquidation.


P7–9 c.

Concluded

As of the end of 2014 Brady had a LIFO reserve of $3,500. A LIFO reserve shows the accumulated benefit derived from the LIFO method. Due to the adoption of LIFO Brady reduced its cumulative pretax income by $3,500. In other words, Brady saved taxes worth $3,500  .35 = $1,225 due to its choice of LIFO. As of the end of 2013, due to LIFO adoption, Brady’s cumulative net income decreased by $4,800 on a pre-tax basis. The related tax savings were $4,800  .35 = $1,680. The impact of a LIFO liquidation shows that adoption of LIFO does not necessarily save taxes in all years. LIFO has adverse effects when the layer liquidation occurs.

d. From an income tax point it is not advisable for Brady to change its cost flow assumption. If it did so, it would have to pay taxes on the $3,500 of additional income that would be created by eliminating the LIFO reserve. However, if the company wishes to report higher income, the change may be desirable.

P7–10 a. and b. IBT Income Statements For the Year Ended December 31, 2014 Sales ................................................................ Cost of sales .................................................. Gross profit ................................................... Other expenses ............................................. Income (loss) before taxes ............................ Income taxes ................................................. Net income (loss) ..........................................

Part (a) $ 67,500 17,700 a 27,000 b $ 49,800 20,000 $ 29,800 8,940 $ 20,860

$

Part (b) 67,500 $ 40,500 20,000 $ 20,500 6,150 $ 14,350

____________ a $17,700 = (350 units  $30) + (200 units  $15) + (350 units  $12) b $27,000 = (900 units  $30) c.

The primary advantage of purchasing the additional 550 units on December 20 is the effect on income taxes. Under part (a), IBT would have to pay $8,940 in income taxes. However, under part (b), IBT would have to pay only $6,150 in income taxes. So the net difference between the income statements of parts (a) and (b) is $2,790 in taxes saved. Since income taxes represent a cash flow, the strategy of acquiring the additional 550 units would save IBT $2,790 in cash from income taxes. This tax savings is not without a cost however. To obtain the savings, IBT had to purchase 550 additional units for $16,500. If IBT was planning on acquiring at least 550 units some time in the near future, then the cost of the tax savings is not $16,500, but is rather the return lost on an alternative use of the $16,500. If IBT was not planning on acquiring additional inventory, then the cost of obtaining the tax savings would be the entire $16,500 plus the opportunity cost of not investing the $16,500.


P7–11 a.

Ending Inventory = 400 x $1.50a = $600 Net Income: Sales Cost of Goods Soldb Gross Profit Inventory Writedown Expensec Net Income a

$30,500 15,000 15,500 400 $15,100

market value of $1.50 per item is less than cost (FIFO) of $2.50 per item

bCOGS (FIFO) = (500 x $2) + (5,600 x $2.50) c($2.50 - $1.50) x 400 remaining items

b. Inventory (+A) 520a Inventory Recovery(R, +SE) 520 aRecovery = ($2.80 - $1.50) x 400 items

If Helio Brothers used U.S. GAAP instead of IFRS, no 2015 entry would be made. Under IFRS previous inventory writedowns may be recovered, but under GAAP inventory recoveries are prohibited.


ISSUES FOR DISCUSSION ID7–1 If investors are solely interested in net income, then the partner is probably correct, and companies should select FIFO if they want to raise capital. However, this view is probably not valid. One must remember that net income is simply a measurement; one must not lose sight of what accountants are measuring. Net income is only valuable if it truly represents an increase in the company's net assets. FIFO will result in higher reported income, but the higher income is an illusion. That is, the increased income under FIFO is due to the difference between the inventory's current market value and the older, "understated" inventory costs matched against it. This is why FIFO results in "paper profits." Alternatively, LIFO matches the most recent, higher inventory costs against revenue, which provides a higher quality measure of the company's underlying economic condition. In addition, the reduced income under LIFO implies lower taxes. The lower taxes, in turn, provide cash that the company can plow back into the business to improve operations. Thus, although LIFO results in lower reported income, LIFO provides a higher quality measure of income and results in lower taxes.

ID7–2 a. The choice of LIFO or FIFO will affect the amounts a company reports both in its balance sheet for inventory and in its income statement for cost of goods sold (and consequently net income). Thus, in order to evaluate a company's financial position and performance, particularly in comparison with other companies' performances, investors and creditors need to know which cost-flow assumption the company is using. In addition, the choice of LIFO or FIFO can have a large effect on the company's cash flows. If inventory costs are rising, a company will have lower taxable income—and hence lower cash outflows for taxes—if it uses LIFO than if it uses FIFO. For some companies the difference can be several million dollars a year in tax savings. b. Obsolete inventory, by definition, is inventory that has no value to the company; due to damage or technological changes or other reasons, the company will not be able to convert this inventory into cash. By deducting this line item from the balance sheet, the company is disclosing the value that it will be able to realize from its inventory. c.

According to the footnote, Harley Davidson’s 2012 ending inventory under FIFO would be $45,889,000 more than under LIFO. Therefore, COGS under FIFO would be lower by the same amount and net income before tax higher by the same amount. Based on a 35% tax rate, therefore, the company would have to pay an additional income tax of $16.1 million.

d. Under IFRS the use of LIFO is prohibited. If IFRS were to be adopted, therefore, Harley Davidson would switch to FIFO as its inventory method and would incur the additional tax expenses discussed above.

ID7–3 In times of rising inventory costs, LIFO allows companies to "hide" the value of their inventory. That is, the inventory value reported on the balance sheet is assumed to consist of "old" inventory costs; the most recent costs of inventory are allocated to cost of goods sold. However, the inventory is really worth its current market value. Thus, the difference between the "old" inventory costs and the current


market value represents a "hidden reserve" of profits. By manipulating its inventory acquisition, a company can dip into this reserve and increase its reported income.

ID7–4 a.

b.

Loss on Inventory Write–down (Lo, –SE) .......................................... Inventory (–A) ............................................................................ Wrote down inventory to market value. Period 1 Loss on Inventory Write–down (Lo, –SE) .......................................... Inventory (–A) ............................................................................ Wrote down inventory to market value. Period 2 Accounts Receivable (or Cash) (+A) .................................................. Sales Revenue (R, +SE) ............................................................... Sold inventory. Cost of Goods Sold (E, –SE) ............................................................... Inventory (–A) ............................................................................ Recorded cost of goods sold.

c.

12,000,000 12,000,000

12,000,000 12,000,000

48,000,000 48,000,000

40,000,000 40,000,000

Because the lower-of-cost-or-market rule gives differential treatment to price decreases and price increases, and because it forces the recognition of losses before they are realized, it may provide inconsistent measures of net income. However, such conservative accounting treatments are employed in response to the liability faced by those who provide and audit financial statements. The costs associated with understating inventories and profits are typically less than the potential costs of overstating them.

ID7–5 a. Valero is using the lower of cost or market exception to the historical cost principle that is applied to inventory. If the market value of inventory is lower than the cost of that inventory, it must be written down to the lower value. b. The write-down will lower reported income, current assets and the equity of Valero. c.

Valero’s current ratio will decrease because inventory will be carried at a lower value, which lowers current assets, while there is no change to current liabilities. Valero’s inventory turnover ratio will increase because average inventory for the year will be lower.

d. By reducing the carrying value of inventory Valero is reducing earnings in the current quarter. As this inventory is sold in future periods Valero will report higher earnings than it would have with no writedown. Valero’s reporting strategy could be to either lower this quarter’s earnings because it produced greater earnings than it anticipated, or Valero could be trying to take a large loss this quarter in order to be able to report better earnings in future quarters.


e. Under U.S. GAAP, inventory is written down, if appropriate, but never written back up. Therefore, Valero would simply leave the inventory at the written-down carrying cost, even if market prices rebound. Under IFRS, on the other hand, the inventory writedowns can be recovered if market prices move back up. In that case, if Valero used IFRS, the company would book an Inventory Recovery (which would increase profits and equity) to balance out the increase in the carrying value of the asset.

ID7–6 a. If Sherwin Williams reported inventories at the end of 2012 based on a FIFO system, the ending inventory balance would have been $1,277,627 ($920,324 + $357,303). b. The following were the tax effects to Sherwin Williams as a result of using LIFO. 2010 2011 2012 (Decrease) in net income due to LIFO (16,394) (62,636) (13,365) Pretax effect on net income (effect /(1-tax rate)) (24,469) (93,487) (19,948) (Decrease) in tax liability ($8,075) ($30,851) ($6,583)

c.

A LIFO inventory system operates on the premise that inventory that is sold is the inventory that was most recently purchased and therefore reflects the most current prices for the inventory. By taking this approach, this gives the best indication as to the future earnings potential of the company. This is particularly true during periods of inflation where the cost of inventory could increase dramatically in short periods of time. A LIFO inventory method most closely approximates the earnings power of buying a new unit of inventory today and selling it in the marketplace today. In times of deflation, LIFO still does a better job of matching current costs with current revenues.

ID7–7 a. A company “thins” its inventory when it reduces the amount of inventory it owns at any given time. Inventory is an asset that has a cost; lower levels of inventory mean lower costs to carry that iventory. Companies that are trimming their operations may opt to reduce their investment in inventory, just like they might decide to own fewer buildings or a smaller number of company cars. b. The balance sheet will show the lower investment in inventory from one time period to the next. Another area in the financial statements to determine if a company is reducing its inventory would be the Statemnt of Cash Flow (Indirect Method). If inventory is being reduced, the change in inventory will show up as a source of Cash in the Operating Activities section. c.

FedEx acts as a transportation company and does not take ownership of the goods being shipped. Although FedEx has physical possession of the item being shipped, it remains on the balance sheet of FedEx’s customer.


ID7–8 The entries on the statement of cash flows are intended to show the impact on cash of the changes in the various balance sheet accounts. A change in an asset account impacts the statement of cash flows because if the asset account increased it reduced the amount of cash, and if the asset account decreased then it increased cash. To increase the inventory account the company has to go out and buy inventory, which reduces cash. If the company sold more inventory than it purchased then this would increase the amount of cash in the company. This data reveals that Macy’s has been consistently increasing its investment in inventories, tying up more of the company’s cash in the inventory line item. Since inventory is an asset account, the increase in the investment in inventory effectively decreases the amount of cash on hand, while a decrease in inventory levels effectively increases cash balances. In 2012, for example, the company had $191 less cash because its inventory had increased over 2011 levels by $191. Similar issues occurred in 2011 and 2010.

ID7–9 a. Supervalu has a much larger difference between LIFO and FIFO inventories than does Safeway. Supervalu’s LIFO Reserve of $211 is over 24 of the value of the inventory of $854, while Safeway’s LIFO Reserve is under 3% of the total ($70.5 ÷ $2,562).

b. Supervalu: LIFO Inventory $854 + $211 = $1,065 FIFO Inventory Safeway: LIFO Inventory $2,562 + $70.5 = $2,632.5 FIFO Inventory c.

Assuming a tax rate of 30%, the benefit would be: Supervalu: .30 x $211 = $63.3 Safeway: .30 x $70.5 = $21.2

d. A grocery store such as Supervalu might adopt FIFO for “highly perishable” items such as milk and produce due to the nature of the inventory. Most grocery stores will sell the gallons of milk that are closest to the expiration date before they put the more recently purchased milk on the shelf. The adoption of FIFO, in this case, more closely matches the actual physical flow of the inventory.

ID7–10 a. When an asset such as inventory decreases, it frees up a company’s cash. A drop in an asset is a “source” of cash for a company. Similarly, when a liability such as accounts payable increases (meaning the company still owes its supplier because it has yet to pay out its cash), it is a “source” of cash balances. Since both inventory and accounts payable are used in the running of a company’s daily business operations, changes to those accounts affect the operating section of the statement of cash flows.


b. From the changes to inventory and accounts payable, it appears that Target was managing its business to downsize inventory levels during a slow economic recovery (2012). The company also appeared to stretch its payables to free up cash, putting the burden of the slow recovery onto its suppliers. As long as the suppliers are not pushed so far that they cut off credit sales to Target, the approach is a sound cash management technique.

ID7–11 a. The results from the policy shift will affect the balance sheet with higher Accounts Payable balances and, therefore, higher Cash Balances; if the company maintains short-term working capital financing (commercial paper or a Line of Credit from a bank group), the company may use that excess Cash to pay down the short-term debt. On the statement of cash flows, the policy will result in an addition to Operating cash due to the increase in Accounts Payable. b. Both the current ratio and the quick ratio would be affected with the larger Accounts Payable balances (in the denominator, under Current Liabilities). c. Delaying a cash outflow for payables will increase Cash balances on the balance sheet. d. P & G is trading off good relationships with its trade suppliers (and possibly advantegous selling terms) for larger cash balances and lower interest expense (if borrowing) or higher interest income (if investing). If P & G, due to its size and volume of purchases, is in a strong negotiating position with its suppliers (who cannot afford to lose the P & G relationship and therefore won’t alter sales and/or shipping terms), the trade-off might be well worth it.

ID7–12 a & b. Inventory is a relatively small investment for Google, due to the company’s predominat business line of advertising sales from internet searches; however, the recent acquisition of Motorola’s mobile phone business has increased the importance of inventory to the consolidated company. In 2011 the account represented only 0.05% of total assets, but by the end of 2012 (due to the acquisition), inventory was up to 0.54% of total assets. c.

As could be expected, with the increase in importance of inventory, the significance of accounts payable also grew ($588 to $2,012).

d. The working capital accounts, net, provided an additional $898 of cash to the company in 2012. (The compares favorably to 2011, where the increase in cash from the working capital accounts, net was $630.)

e. Footnote #1 discloses that Google uses FIFO as its inventory method.


CHAPTER 8 INVESTMENTS IN EQUITY SECURITIES BRIEF EXERCISES BE8–1 a.

Comprehensive income includes all non-owner changes in shareholder equity that do not already appear on the income statement. For example, the change in value of assets that have been sold and in certain assets that have not been sold appear as comprehensive income, but net income only includes the changes in assets that have been sold.

b.

The investments are considered Available-for-Sale Securities, because the Unrealized Gains/Losses are not posted to the Income Statement; instead, as shown in the disclosure the Unrealized Gains/Losses are carried directly to Stockholders’ Equity through Comprehensive Income. In 2012 the Available-for-Sale Securities increased $52 million in market value. In 2011 the Available-for-Sale Securities decreased $10 million in market value. In 2010 the Available-for-Sale Securities dropped $2 million in market value. In each of the three years, Merck did not sell the securities at the year-end value. (An Unrealized Gain/Loss is a holding gain/loss, meaning the securities were not sold but simply marked to the current market value.)

BE8–2 Bristol-Myers Squibb designated its Marketable Securities as Available-for-Sale Securities and as such reflected holding gains/losses (unrealized) in Comprehensive Income, not on the Income Statement. The $12 million reflected a change in the market value of the Securities which were still owned by the company. The asset section of the balance sheet will also show the change, with the carrying value of the Securities adjusted by the $12 million.

BE8–3

a. The excerpts from the financial statements indicate that Pepsi stopped using the equity method for accounting for its investment in bottling companies in 2011. Ownership percentages and perceived control/influence must have changed for the company to discontinue its equity method accounting. b. Investment in Equity Securities (+A) 735 Equity in net income of affiliates (R, +SE) 735

c.

In Pepsi’s case, in 2010 the company received cash of $693 million ($735 - $42).

1


BE8–4 Goodwill is the amount paid above fair value when assets are acquired. P & G, in its business growth strategy, has acquired other companies and has paid, collectively, $53.8 billion above the fair value of the assets acquired.

BE8–5 Assets (+A) Goodwill (+A) Liabilities(+L) Cash(-A)

1,323 1,800 323 2,800

Goodwill is an intangible asset that represents the excess above fair value paid for assets acquired. J & J paid $2.8 billion in cash and assumed liabilities. Balancing the journal entry, the company acquired assets with a fair value of $1.323 billion. Assets and liabilities increased by $323 million with no change to equity.


EXERCISES E8–1 a. (1)

(2)

(3)

(4)

(5)

(6)

(7)

(8)

Trading Securities (+A) .................................................................... Cash (–A) .................................................................................. Invested in IBM.

50,000

Trading Securities (+A) .................................................................... Cash (–A) .................................................................................. Invested in GM.

40,000

Cash (+A) ......................................................................................... Trading Securities (–A) ............................................................. Realized Gain on Sale of Trading Securities (Ga, +SE) ............................................................... Sold IBM stock.

45,000

Cash (+A) ......................................................................................... Dividend Receivable ................................................................ Received dividend.

750

Trading Securities (+A) .................................................................... Cash (–A) .................................................................................. Invested in Xerox.

8,000

Cash (+A) ......................................................................................... Realized Loss on Sale of Trading Securities (Lo, –SE) ................................................................... Trading Securities (–A) ............................................................. Sold IBM stock.

7,500

Cash (+A) ......................................................................................... Trading Securities (–A) ............................................................. Realized Gain on Sale of Marketable Securities (Ga, +SE) ....... Sold Xerox stock.

11,600

Cash (+A) ......................................................................................... Realized Loss on Sale of Trading Securities (Lo, –SE) ...................... Trading Securities (–A) ............................................................. Sold GM stock.

30,000 10,000

50,000

40,000

37,500 7,500

750

8,000

5,000 12,500

8,000 3,600

40,000


E8–1

Concluded

b. The transactions that affected the income statement for Monroe Auto Supplies are the gains and losses Monroe Auto Supplies realized from selling marketable securities for amounts different from their purchase prices. Monroe Auto Supplies realized gains of $7,500 and $3,600, respectively, on the first sales of IBM and Xerox stock and realized losses of $5,000 and $10,000, respectively, on the second sale of IBM stock and on GM stock. Thus, the net effect of the dividends, gains, and losses on net income was to decrease net income by $3,900. The dividends received from the General Motors stock were recognized as income in the previous period.

E8–2 a. Change in the wealth level of each of the four companies can be computed by comparing the beginning and year-end balance in the short-term equity investment account. Therefore, the amounts for each company are as follows. Wearever Fabrics Frames Corp. Pacific Transport Video Magic

$12,000 2,000 (24,000) 13,000

b. The amount that should be reported as holding gains and losses on the income statement is always due to changes in the market value of trading securities. Therefore, the holding gains and losses for each company are as follows. Wearever Fabrics Frames Corp. Pacific Transport Video Magic

$20,000 (10,000) (20,000) 10,000

c.

The reason that the answers to (a) and (b) are not the same is due to the fact that in part (a) the wealth charges are computed for the trading securities and available-for-sale securities together. In part (b) the holding gains and losses that are being reported on the income statement pertain to only trading securities. For the available-for-sale securities, the holding gains and losses are taken directly to the stockholders’ equity on the balance sheet via comprehensive income.

d.

If the company were to choose the fair market value option, the change in market values for the available-for-sale securities would also be included. Therefore, the income change would match the wealth change outlined in part a above.

E8–3 a. (1)

(2)

(3)

1/28

2/18

3/15

Short-Term Investments (+A) ......................................... Cash (–A) ................................................................. Invested in Able Co.

140

Short-Term Investments (+A) ......................................... Cash (–A) ................................................................. Invested in Baker Co.

520

Cash (+A) ........................................................................ Dividend Revenue (R, +SE) ...................................... Received dividends.

10

140

520

10


E8–3 (4)

(5)

(6)

Continued 4/29

5/18

6/1

Cash (+A) ........................................................................ Short-Term Investments (–A) .................................. Realized Gain on Sale of Short-Term Investments (Ga, +SE) .......................................... Sold Able Co. stock.

75

Cash (+A) ........................................................................ Dividend Revenue (R, +SE) ...................................... Received dividends.

40

Cash (+A) ........................................................................ Realized Loss on Sale of Short-Term Investments (Lo, –SE) ................................................. Marketable Securities (–A) ...................................... Sold Baker Co. stock.

110

b. Able (5 shares) Baker (15 shares) Total

Cost

Market Value

70 390 460

85 300 385

70 5

40

20 130

June 30 Adjusting Entry (1) If Able and Baker are both considered trading securities: Unrealized Price Decrease on trading securities (Lo, –SE) .............. Trading Securities (–A) ............................................................ Revalued trading securities to market.

75 75

(2) If Able is a trading security and Baker is an available-for-sale security: Able Trading Securities (+A) ................................................................... Unrealized Price Increase on Trading Securities (Ga, +SE) ................................................................................ Baker Unrealized Price Decrease (–SE) .................................................... Available-for-Sale Securities (–A) .............................................

15 15 90 90

(3) If Able is considered an available-for-sale security and Baker is considered a trading security: Able Available-for-Sale Securities (+A) ................................................... Unrealized Price Increase on Available-for-Sale Securities (+SE)...................................................................... Baker Unrealized Price Decrease on Trading Securities (Lo,–SE) ....................................................................................... Trading Securities (–A) .............................................................

15 15

90 90


E8–3

Concluded

(4) If Able and Baker are both considered available-for-sale securities: Unrealized Price Decrease on Available-for-Sale Securities (–SE) ............................................................................ Available-for-Sale Securities (–A) ............................................. c.

75 75

Since management usually want to keep losses off the income statement, the second combination in (b) will depict the management as the most successful in the current period. All the losses ($90) are shown as unrealized losses in stockholders equity while the gain of $15 goes to the income statement.

E8–4 a.

Trading

Available-For-Sale

$ 401 2002 103 1205 206 $ 70

2014 income 12/31/14 balance sheet investment value 2015 income 2015 balance sheet investment value 2016 income Total income

01 2002 104 1205 607 $ 70 $

1. 20 shares were purchased at $8, and on 12/31/14 their market value has increased to $10 a share. An unrealized price increase of $40 for Trading Securities will go to the income statement, but the same unrealized gain for available-for-sale securities will go directly into the stockholders’ equity. Therefore there is no impact on the 2014 income statement. 2. 20 shares @ $10 a share as of 12/31/14. 3. 2015

Cash (+A)................................................................................. Unrealized Price Increase (–Ga, –SE)....................................... Trading Securities .............................................................. Realized Gain (+Ga, +SE)....................................................

90 20

12/31/15 Trading Securities (+A)............................................................ Unrealized Price Increase (+Ga, +SE) .................................

20

100 10

20

The net value of all income statement-related accounts is $10 of increase in income. 4. For available-for-sale securities, the Journal entries would remain the same, but all the unrealized price increases would go directly to the stockholders’ equity account, leaving only $10 of realized gain on the income statement. 5. Purchased 20 shares @ $8 12/31/14 Mark to Market @ $10 Sold 10 shares @ cost basis of $10 12/31/15 Mark to Market @ $12 12/31/15 balance sheet value

= = = = =

$160 40 (100) _ 20 $120


E8–4

Concluded

6. Cash (+A) ................................................................................................... Unrealized Price Increase (–Ga, –SE)......................................................... Trading Securities (–A)........................................................................ Realized Gain (+Ga, +SE) .....................................................................

140 40 120 60

7. For available-for-sale securities, the entry would remain the same, but the $40 unrealized price increase debit would go directly to the stockholders’ equity section, leaving the $60 realized gain on the income statement. b. Whether a security is classified as a trading security or available-for-sale security, the balance sheet investment value remains the same due to the mark-to-market rule. However, the classification does impact the income statement as can be seen from this problem. Overall, whether the security is a trading security or an available-for-sale security, the total income would remain the same.

E8–5 a. Fair market value of Biomet’s available-for-sale securities:

Cost Gains Losses Fair value

2011 $79.0 0.1 (4.7) $74.4

2012 $16.2 0.0 (0.2) $16.0

b. The effect on the company’s comprehensive income associated with its available-for-sale securities is: 2011 2012 Gains 0.1 0.0 Losses (4.7) (0.2) Net losses $ (4.6) $(0.2)

c. The income that would be realized would be ($0.2) [$4.8 - $4.6]. Cash (+A) ................................................................................................... Realized Loss (Loss, –SE)............................................................................ Available-for-sale Securities (–A)........................................................ Unrealized Price Decrease (-Loss, +SE) ...............................................

16.0 4.8 16.2 4.6


E8–6 a. Tom Miller Balance Sheet December 31, 2014 Assets Cash ....................................... Marketable securities ............ Inventory ...............................

$ 2,880 3,840* 1,500

Total assets ............................

$ 8,220

Liabilities & Stockholders' Equity Accounts payable................. $ 1,500 Contributed capital .............. 6,000 Securities fair value adjustments (availablefor-sale) ............................ 720** Total liabilities & stockholders' equity ......... $ 8,220

* $3,840 = 120 shares  $32 ** 720 = 3,840 – 3,120 Larry Rogers Balance Sheet December 31, 2014 Assets Cash ....................................... Marketable securities ............ Inventory ...............................

$ 2,880b 3,840a 1,500

Total assets ............................

$ 8,220

Liabilities & Stockholders' Equity Accounts payable................. $ 1,500 Contributed capital .............. 6,000 Retained earnings ................ 720c Total liabilities & stockholders' equity ......... $ 8,220

a $3,840 = 120 shares x $32 b $2,880 = $6,000 contributed by Larry – $3,120 for initial investment in Diskette + $3,840 in proceeds c

from sale of Diskette – $3,840 for reinvestment in Diskette. $720 = $3,840 proceeds from sale of Diskette – $3,120 cost of Diskette.

b. Net income Working capital Current ratio c.

Tom Miller

Larry Rogers

$0.00 6,720.00 5.48

$720.00 6,720.00 5.48

Truly speaking no one. Both have the same working capital and current ratio. However, Larry would report a net income of $720 while Tom would not.

d. Tom Miller is actually in better financial position than Larry Rogers. Both of them own the exact same marketable securities, and both own $1,500 in inventory. Tom would have more cash because he chose to simply hold on to his investment and not incur the transaction costs of buying and selling. On the other hand, Larry sold and repurchased his securities, and each transaction cost him brokerage fees.


E8–7 a. (1)

This investment should be classified as a long-term investment and accounted for using the equity method. Hartney Consulting owns 40% of the investee's common stock, which indicates that it can exert some control over the company, and Hartney Consulting intends to hold the investment for longer than the time frame of current assets.

(2)

This investment should be classified as a long-term investment and accounted for using the consolidated method. Hartney owns greater than 50% of the investee, and it appears that Hartney Consulting has no intention of disposing of the investment within the time frame of current assets.

(3)

This investment should be classified as a long-term investment and accounted for using the cost method. To be classified as a short-term investment, a ready market for the securities must exist so that the investor can dispose of the investment at any time. In this case, because the company is closely held, Hartney Consulting could not dispose of the investment when it desires.

(4)

This investment should be classified as a long-term investment and accounted for using the equity method. Hartney Consulting owns 45% of the investee's common stock, which indicates that it desires to exert some control over the company, and Hartney Consulting intends to hold the investment for five years, which is longer than the time frame of current assets.

(5)

This investment should be classified as a long-term investment and accounted for using the mark-tomarket method.

(6)

This investment should be classified as a long-term investment and accounted for using the equity method. Although Hartney Consulting owned less than 20% of the investee for a portion of the year, it owned greater than 20% of the investee as of December 31, 2014. Furthermore, the company intends to hold the investment for four years.

b. To be classified as a short-term marketable equity investment, the equity investment must meet two criteria. First, the investor must intend to dispose of the investment within the time frame of current assets. Second, a ready market for the securities must exist. A ready market for the investment enables the investor to dispose of the investment at any time. In the case of nonmarketable equity securities, the investor may not be able to dispose of the investment when it wishes. Since too much uncertainty exists as to whether the investor could actually dispose of nonmarketable investments within the time frame of current assets, the investment should be classified as a long-term investment. Classifying the investment as long-term is more conservative than classifying the investment as current because classifying the investment as long-term reduces the company's measures of solvency (e.g., working capital, current ratio, and quick ratio). To account for equity investments using the mark-to-market method, the market value of the equity securities must be known. With nonmarketable equity securities, no market value exists. Consequently, nonmarketable equity securities are accounted for using the cost method rather than the mark-to-market method.


E8–8 (1)

(2)

(3)

(4)

(5)

Investment in Equity Securities (+A) .................................................... Cash (–A) ........................................................................................ Invested in Thayers International.

260,000

Investment in Equity Securities (+A) .................................................... Cash (–A) ........................................................................................ Invested in Bayhe Enterprises.

875,000

Dividend Receivable (+A)...................................................................... Dividend Revenue (R, +SE)............................................................. Earned, but did not receive, dividend.

20,000

Cash (+A)............................................................................................... Loss on Sale of Equity Securities (Lo, –SE) ............................................ Investment in Equity Securities (–A).............................................. Sold Bayhe Enterprises stock.

135,000 22,500

Cash (+A)............................................................................................... Investment in Equity Securities (–A).............................................. Gain on Sale of Equity Securities (Ga, +SE) .................................... Sold Thayers International stock.

256,000

Available-For-Sale Securities (+A) ................................................ Cash (–A) ............................................................................... Invested in Thayers International.

260,000

Available-For-Sale Securities (+A) ................................................ Cash (–A) ............................................................................... Invested in Bayhe Enterprises.

875,000

Dividend Receivable (+A) ............................................................. Dividend Revenue (R, +SE) .................................................... Earned, but did not receive, dividend.

20,000

Cash (+A) ...................................................................................... Realized Loss on Available-For-Sale Securities (Lo, –SE) ................................................................... Available-For-Sale Securities (–A) ......................................... Sold Bayhe Enterprises stock.

135,000

Cash (+A) ...................................................................................... Available-For-Sale Securities (–A) ......................................... Realized Gain on Sale of Available-For-Sale Securities (Ga, +SE) ............................................................ Sold Thayers International stock.

256,000

260,000

875,000

20,000

157,500

208,000 48,000

E8–9 a. (1)

(2)

(3)

(4)

(5)

260,000

875,000

20,000

22,500 157,500

208,000 48,000


E8–9

Concluded

b. Securities Bayhe Thayers Total

Shares Held 20,500 2,000

Cost $ 717,500 52,000 $ 769,500

Market Value $ $

656,000 50,000 706,000

Unrealized Loss on Available-For-Sale Securities (–SE) ................................................................................... Available-For-Sale Securities (–A)....................................................... Adjusted investments to market.

63,500 63,500

E8–10 a. 2014 Investment in Equity Securities (+A) ......................................................... Cash (–A)............................................................................................. Invested in Reilly Manufacturing.

190,000 190,000

Cash (+A) ................................................................................................... Investment in Equity Securities (–A) .................................................. Received dividends (equity method).

15,000

Investment in Equity Securities (+A) ......................................................... Income from Equity Investments (R, +SE) .......................................... Earned investment revenue (equity method).

18,750*

15,000

18,750

* $18,750 = $75,000 investee net income  25% 2015 Cash (+A) ................................................................................................... Investment in Equity Securities (–A) .................................................. Received dividends (equity method). Loss on Equity Investments (Lo, –SE) ........................................................ Investment in Equity Securities (–A) .................................................. Incurred investment loss (equity method).

15,000 15,000

1,500* 1,500

* $1,500 = $6,000 investee net loss  25% b. Cost of investment Plus: Portion of investee's 2014 net income Less: Portion of investee's 2014 declared dividends Book value of investment as of December 31, 2014 Less: Portion of investee's 2015 net loss Less: Portion of investee's 2015 declared dividends

$ 190,000 18,750 (15,000) $ 193,750 (1,500) (15,000)


Book value of investment as of December 31, 2015

$ 177,250

E8–11 a. During 2012 Duke Energy reported equity losses of $148 million on its income statement. Since it owns 40% of its affiliates, the affiliates reported net losses of $148 million ÷ 40% or $370 million. b. While the income (losses) claimed from the affiliates increases (decrease) the value of the long-term equity investment, the dividends received from the affiliates decrease the value of the investment. [Beginning Investment + Equity in Affiliate Income – Dividends Received from Affiliate = Ending Investment] Duke shows an increase in balance sheet value of $23 million, but losses from affiliates on the income statement of $148 million. The company must have invested additional funds in affiliates to overcome the drop in balance sheet value from the affilate losses. If dividends were also received (further dropping the asset value), the additional investment in affilates would be that much larger.

E8–12 a. Since Mainmont Industries owns 30% of Tumbleweed Construction, 30% of Tumbleweed Construction's net income "flows through" to Mainmont Industries' income statement. Consequently, Tumbleweed Construction's total net income for 2014 would have been $40,000 ($12,000 ÷ 30%). b. Long-term investment in equity securities: 12/31/13 30% of Tumbleweed Construction's 2014 net income Less: 30% of Tumbleweed Construction's 2014 declared dividends Long-term investment in equity securities: 12/31/14

$25,000 12,000 X $29,000

Mainmont Industries' portion of the dividends declared by Tumbleweed Construction was $8,000. Since this amount represents 30% of the total dividends declared by Tumbleweed, the total dividends declared by Tumbleweed were $26,667 ($8,000 ÷ 30%). c.

Cash (+A) ................................................................................................... Investment in Equity Securities (–A) .................................................. Received dividends (equity method).

8,000

Investment in Equity Securities (+A) ......................................................... Income from Equity Investments (R, +SE) .......................................... Earned investment revenue (equity method).

12,000

8,000

12,000

d. On its statement of cash flows (indirect), Mainmont would deduct from net income the Income from Equity Investments net of the cash received in dividends ($4,000 net, for 2014).

E8–13 a. Cash (+A) ........................................................................................ 90,000 Accounts Receivable (+A) ............................................................... 60,000 Inventory (+A)............................................................................... 160,000 Plant & Equipment (+A)................................................................ 560,000 Goodwill (+A) ................................................................................ 330,000 Payables (+L) ....................................................................................... Cash (-A) .............................................................................................

300,000 900,000


Purchased Lipley Company. b. The net book values of assets and liabilities represent the amounts at which the assets and liabilities are carried on the balance sheet. Some assets are carried at original cost, others at net realizable value or lower-of-cost-or-market value. If the fair market value exceeds these other valuation bases, then the net fair market value of Lipley's assets and liabilities could exceed their book values. Multiplex is purchasing all of Lipley Company. Consequently, Multiplex is not only purchasing the net assets of Lipley, but it is also purchasing the nonquantifiable items that make Lipley a viable company. These nonquantifiable items include customer loyalty, name recognition, and employee skills and loyalty. These items make Lipley more valuable than the sum of its quantifiable assets and liabilities. Therefore, Multiplex is willing to pay more for Lipley because it is more valuable than the sum of its individual assets and liabilities due to synergy.

E8–14 Purchase Price = Net Book Value + Net Market Value in Excess of Book Value + Goodwill Transactions: (1) Purchase Price = $7,000 + $1,000 + $1,000 Purchase Price = $9,000 (2) $6,000 = $6,000 + Net Market Value in Excess of Book Value + $0 Net Market Value in Excess of Book Value = $0 (3) $12,000 = Net Book Value + $4,000 + $3,000 Net Book Value = $5,000 (4) $15,000 = $10,000 + $3,000 + Goodwill Goodwill = $2,000 (5) Purchase Price = $2,000 + $1,000 + $3,000 Purchase Price = $6,000 (6) $12,000 = $4,000 + $8,000 + Goodwill Goodwill = $0

E8–15 a. Book Value per Share

Book Value ÷ Number of Common Shares Outstanding $36,000 ÷ Number of Common Shares Outstanding = $12 Number of Common Shares Outstanding = 3,000 Shares

b. Market Value/Share

c.

= =

= = =

Market Value ÷ Number of Common Shares Outstanding $51,000 ÷ 3,000 Shares $17

Conglomerate would be willing to pay more than the market value per share due to goodwill. Camden has generated goodwill, such as name recognition and employee and customer loyalty, that makes its net assets more valuable taken as a whole than taken as individual net assets. In other words, due to synergy Camden is worth more than the sum of its individual parts. Additionally, combining Camden's assets with Conglomerate's assets may provide economies of scale, thereby making Camden's assets more valuable to Conglomerate than their individual fair market values.


d. Investment in Subsidiary (+A) ................................................................... 66,000* Cash (–A)............................................................................................. Acquired subsidiary. *66,000 = $22 per Share Purchase Price  3,000 Common Shares Outstanding

Cash (+A) ................................................................................................... Receivables (+A) ........................................................................................ Inventories (+A) ......................................................................................... Fixed Assets (+A) ....................................................................................... Goodwill (+A) ............................................................................................. Liabilities ............................................................................................. Investment in Subsidiary .................................................................... Consolidated financial statements.

66,000

15,000 24,000 25,000 47,000 15,000 60,000 66,000

E8–16 a. The consolidating entries are shown below: [Note: assets and liabilities purchased are added at Fair Market Value] Investment in Subsidiary (+A) .......................................................................... 144,000* Cash (–A)............................................................................................. Acquired subsidiary. *144,000 = $18 per Share Purchase Price  8,000 Shares (80% of Outstanding) Current Assets (+A).................................................................................... Non-Current Assets (+A)............................................................................ Goodwill (+A) ............................................................................................. Liabilities ............................................................................................. Minority Interest ................................................................................ Investment in Subsidiary ....................................................................

144,000

150,000 80,000 20,000 70,000 36,000 144,000

b. If Maxwell uses IFRS and assumes that minority shareholders have no interest in goodwill, the entry would be: Current Assets (+A).................................................................................... Non-Current Assets (+A)............................................................................ Goodwill (+A) ............................................................................................. Liabilities ............................................................................................. Minority Interest ................................................................................ Investment in Subsidiary ....................................................................

150,000 80,000 16,000 70,000 32,000* 144,000

*minority interest equals 20% of net asset value (.20 x [230,000-70,000]) c. The minority interest would be disclosed in the shareholders’ equity section of the balance sheet (under both U.S. GAAP and IFRS).


E8–17

Accounts

Glover

Adjustments and Eliminations Ward Debit Credit

Cash Accts. Receivable Inventory Investment in Sub. Fixed Assets Goodwill Total Assets

73,000 10,000 110,000 40,000 220,000 60,000 100,000 0 615,000 120,000 __30,000 _____0 1,148,000 230,000

Accounts Payable Long-Term Notes Common Stock Retained Earnings Total Liabilities & Stockholders' Equity

80,000 450,000 500,000 _118,000

70,000 80,000 70,000 _10,000

70,000 10,000 _____0

1,148,000

230,000

80,000

Consolidated Balance Sheet

10,000 100,000 5,000 _5,000 20,000

______ 100,000

83,000 150,000 290,000 0 740,000 __35,000 1,298,000 150,000 530,000 500,000 _118,000

0

1,298,000

The Adjustments and Eliminations columns adjust assets to market value, eliminate the investment account, eliminate the stockholders' equity section of Ward, and recognize goodwill. The goodwill of $5,000 equals the excess of the $100,000 purchase price over the $95,000 fair market value of Ward's individual net assets (i.e., $10,000 in cash + $40,000 in accounts receivable + $70,000 in inventory + $125,000 in fixed assets – $70,000 in accounts payable – $80,000 in notes payable).


PROBLEMS P8–1 a.

The total gains and losses reported on the income statement are as follows: Realized Gains and Losses on Trading as well as Available-for-Sale Securities +

Unrealized Price Changes for Trading Dividend Income Securities +

Security

# of Shares Sold

Realized G/L per share

Houser Co. Miller, Inc. Letter Books Nordic Equip.

60 90 5 145

$25 – $22 = $3 30 – 40 = ($10) 55 – 48 = $7 95 – 70 = $25

Security

# of Shares

Cost per Share

12/31/14 Market per Share

Houser Miller

30 90

$22 $40

$25 $35

Total unrealized price decrease on trading securities

Total Realized Gain/Loss $

Total Cost

Total Market

$

$

660 3,600 $ 4,260 =

180 (900) 35 3,625 $2,940

750 3,150 $ 3,900

$4,260 – $3,900 = $360

Effect on 2014 reported income from all investment transactions and price changes: Net Realized Gain on Sale of Trading and Available-For-Sale Securities During the Year Net Unrealized Price Decrease on Trading Securities Total Dividends Received during 2014 Total Effect on 2014 Income b.

=

$2, 940

= =

(360) 205 $ 2,785

If O’Leary used the fair market value option, then the unrealized gains and losses on the availablefor-securities (Letter and Nordic) would affect net income as well.

# of

Cost

12/31/14 Market

Total

Total


Security

Shares

per Share

per Share

Cost

Market

Letter Nordic

70 25

$48 $70

$46 $90

$ 3,360 1,750 $ 5,110

$ 3,220 2,250 $ 5,470

Total unrealized price increase on available-for-sale securities = $5,470 – $5,110 = Effect on 2014 reported income: $2,785 (from part a) + $360 = $3,145

$360

P8–2 a. (1)

(2)

(3)

(4)

(5)

(6)

(7)

(8)

(9)

3/10

3/31

5/26

7/10

9/11

9/27

10/19

11/6

12/8

Trading Securities (+A) .......................................................... Cash (–A) ........................................................................ Invested in Arctic Oil & Gas.

28,000

Trading Securities (+A) .......................................................... Cash (–A) ........................................................................ Invested in Humphries Manufacturing.

8,000

Cash (+A) ............................................................................... Dividend Revenue (R, +SE) ............................................. Earned and received dividends.

1,250

Trading Securities (+A) .......................................................... Cash (–A) ........................................................................ Invested in Kingsman Game Company.

18,000

Cash (+A) ............................................................................... Trading Securities (–A) ................................................... Realized Gain on Sale of Trading Securities (Ga, +SE)........................................................ Sold Arctic Oil & Gas stock.

28,000

Cash (+A) ............................................................................... Realized Loss on Sale of Trading Securities (Lo, –SE) ... 1,000 Trading Securities (–A) ................................................... Sold Humphries Manufacturing stock.

4,000

Trading Securities (+A) .......................................................... Cash (–A) ........................................................................ Invested in Quimby, Inc.

25,000

Cash (+A) ............................................................................... Dividend Revenue (R, +SE) ............................................. Earned and received dividend.

250

Cash (+A) ............................................................................... Trading Securities (–A) ................................................... Realized Gain on Sale of Trading

10,500*

28,000

8,000

1,250

18,000

22,400 5,600

5,000

25,000

250

8,600


Securities (R, +SE) .......................................................... Sold Arctic Oil & Gas and Humphries Manufacturing stock.

1,900

* $10,500 = (200 shares of Arctic Oil & Gas  $30) + (300 shares of Humphries Manufacturing  $15)

P8–2 Concluded Computation for adjusting journal entry on 12/31:

Security

Number of Shares

Cost/Share

Market Value/Share

0 0 1,000 1,000

$28 10 18 25

$32 14 15 26

Arctic Oil & Gas Humphries Manufacturing Kingsman Game Company Quimby, Inc. Total 12/31

$

$

Total Cost

Total Market Value

0 0 18,000 25,000 43,000

$

Unrealized Price Decrease on Trading Securities (Lo, –SE) ......................................................... Trading Securities (–A)................................................ Adjusted marketable securities to LCM.

b. Marketable securities

$

2,000

$41,000

c. Transaction (1) (2) (3) (4) (5) (6) (7) (8) (9)

3/10 3/31 5/26 7/10 9/11 9/27 10/19 11/6 12/8 12/31 Net increase

Income Statement Effect 0 0 1,250 0 5,600 (1,000) 0 250 1,900 (2,000) $ 6,000

0 0 15,000 26,000 41,000

Explanation

$

Dividend revenue Gain on sale of trading securities Loss on sale of trading securities Dividend revenue Gain on sale of trading securities Holding loss on investment portfolio

2,000


P8–3 a. 10/18/14

12/31/14

12/31/15

b. 10/18/14

12/31/14

12/31/15

c.

Available-For -Sale Securities (+A) ........................................ Cash (–A) ........................................................................

3,200

Available-For -Sale Securities (+A) ........................................ Unrealized Price Increase (+SE)......................................

800

Trading Securities (+A) .......................................................... Unrealized Price Increase (-SE) ............................................. Realized Price Increase from Reclassification (Ga, +SE)...................................................................... Available-For-Sale Securities ..........................................

5,000 800

Trading Securities (+A) .......................................................... Cash ................................................................................

3,200

Trading Securities (+A) .......................................................... Unrealized Gain (Ga, +SE) ..............................................

800

Available-For-Sale Securities (+A) ......................................... Realized Price Increase on Reclassification (Ga, +SE)...................................................................... Trading Securities (–A) ...................................................

5,000

3,200

800

1,800 4,000

3,200

800

1,000 4,000

Income Effect Transactions 10/18/14 12/31/14 12/31/15

Trading Securities

Purchase Mark-to-Market Reclassification Total

$

0 800 1,000 $1,800

Availablefor-Sale Securities $

0 0 1,800 $1,800

P8–4 a.

___Guyer Books___ Balance Sheet Income Value Effect

_____Levy Co._____ Balance Sheet Income Value Effect

Investment Classified as: Trading Securities Available-for-Sale Securities

$9,000 $9,000

$6,600 0

$9,000 $9,000

$6,600 $6,600

b. Guyer and Levy’s income statements would differ only if the security is classified as available-for-sale. The reason: price increases would flow through directly to the stockholders’ equity section, via comprehensive income. In the case of available-for-sale securities, the gain is a realized one, which would flow through the income statement.


P8–5 a. 6/15/14

12/31/14

1/16/15

10/20/15

Trading Securities (+A) .......................................................... Cash (–A) ........................................................................

7,500

Trading Securities (+A) .......................................................... Unrealized Price Increase (Ga, +SE) ...............................

1,500

Cash (+A) ............................................................................... Unrealized Price Increase from Reclassification (–Ga, +SE) .......................................................................... Trading Securities (–A) ................................................... Realized Gain (Ga, +SE) ..................................................

5,700

Cash (+A) ............................................................................... Realized Loss (Lo, –SE) .......................................................... Unrealized Price Increase (–Ga, –SE) .................................... Trading Securities (–A) ...................................................

2,600 400 600

7,500

1,500

900 5,400 1,200

3,600

b. The entries would remain the same except the following: (1) one will debit or credit the Available-forSale Securities account instead of the Trading Securities account; (2) all the price changes would flow through directly to stockholders’ equity instead of the income statement. The realized gain and loss would be the same. c.

Cash Effect Transactions

Trading Securities

Available-for-Sale Securities

6/15/14 Purchase 12/31/14 Mark-to-Market 1/16/15 Sale 10/20/15 Sale Net Cash Effect

$ (7,500) 0 5,700 2,600 $ 800

$ (7,500) 0 5,700 2,600 $ 800

Trading Securities

Available-for-Sale Securities

d. & e. Income Effect Transactions 6/15/14 12/31/14 1/16/15

Purchase Mark-to-Market Sale—Realized Price Increase —Unrealized Price Increase reversal 10/20/15 Sale—Realized Price Decrease —Unrealized Price Increase Reversal Net Income Statement Effect

$

$

0 1,500

$

0 0

1,200

1,200

(900)

0

(400)

(400)

(600) 800

$

0 800


P8–5 f.

Concluded

Overall income statement effect for both the years (i.e., 2014 and 2015) would be the same whether the securities are classified as trading securities or as available-for-sale securities. However, the individual years’ income would differ due to the chosen classification. For trading securities all the unrealized price changes would flow through the income statement, while those for the available-for-sale securities would flow directly into the stockholders’ equity account.

P8–6 a. and b.

We assume that Orlean’s first investment in these securities occurred on 12/31/13. Cost of the Atwater Investment on a Per-Share Basis

Realized Price Increase During 2014

Per Share Market Value as of 12/31/14

Total Unrealized Price Increase on 1,600 Shares

=

$27,000 ÷ 1,800 Shares

=

$15/Share

=

200 Shares  ($15.50 – $15)

=

$100

=

$25,440 ÷ 1,600 Shares

=

$15.90

=

1,600 Shares  .90 =

$1,440

If the securities were classified as trading securities, Orleans would report a total gain of $1,540, of which $100 would be realized and $1,440 would be an unrealized price increase. However, if the securities were classified as available-for-sale, only $100 would be reported in the income statement. The unrealized price increase of $1,440 would go directly to the stockholders’ equity section of the balance sheet.

P8–7 a. The net change in the investment in trading securities account from 2011 to 2012 was billion minus $444 billion, or an increase of $6 billion.

$450

b. The net change in the available-for-sale securities from 2011 to 2012 was $371.2 billion minus $364.8 billion, or an increase of $6.4 billion. The net increase per the statement of cash flows was $4.96 billion, which means that the unrealized gains on the available-for-sale securities were $1.44 billion. This amount would be shown under comprehensive income, which would affect stockholders’ equity on the balance sheet.


P8–8 a. Under the mark-to-market method, the investment is carried on the books at the market value as of the balance sheet date. Adjustments of the investment portfolio to market value "flow through" to the income statement. Dividends declared by the investee are recognized as income to the investor in the period the dividends are declared. (1) Masonite Tires Balance Sheet January 1, 2014 Noninvestment assets ..................... Investment in .................................. Securities ....................................... Total ................................................. (2)

$ 120,000 Liabilities ...................... Stockholders’ equity ............. 40,000 $ 160,000 Total.............................

$ 70,000 90,000 $ 160,000

During 2014, Masonite Tires would make the following original and adjusting journal entries under the mark-to-market method: Cash (+A) ......................................................................................... Revenues (R, +SE) ..................................................................... Earned noninvestment revenues.

85,000

Expenses (E, –SE)............................................................................. Cash (–A) .................................................................................. Incurred expenses.

50,000

Cash (20%  $10,000) (+A) .............................................................. Dividend Revenue (R, +SE) ....................................................... Earned and received dividends.

2,000

Unrealized Loss on Securities (–SE) .............................................................................. Investment in Securities (–A) ................................................... Adjusted securities to market.

85,000

50,000

2,000

4,000 4,000

Masonite Tires Income Statement For the Year Ended December 31, 2014 Revenues .............................................................................................................. Dividend revenue ................................................................................................. Unrealized Loss………………………………………………………………. 4,000 Expenses............................................................................................................... Net income ...........................................................................................................

$ 85,000 2,000 50,000 $ 33,000


P8–8

Continued Masonite Tires Balance Sheet December 31, 2014 Noninvestment assets ...................... $157,000 Liabilities………………. $ 70,000 Investment in Securities ................. 36,000 Stockholders' equity .......... 123,000* Total……………………………… $ 193,000 Total …………………… $193,000 * $123,000 = $90,000 Beginning balance + Net Income of $33,000

(3)

During 2015, Masonite Tires would make the following original and adjusting journal entries: Cash (+A) ......................................................................................... Revenues (R, +SE) ..................................................................... Earned noninvestment revenues.

75,000

Expenses (E, –SE)............................................................................. Cash (–A) .................................................................................. Incurred expenses.

70,000

75,000

70,000

Cash (20%  $15,000) (+A) .............................................................. Dividend Revenue (R, +SE) ....................................................... Earned and received dividends.

3,000

Investment in Securities (–A) .......................................................... Unrealized Gain on Securities (+SE) .................................................................. Adjusted securities to market.

6,000

3,000

6,000

Masonite Tires Income Statement For the Year Ended December 31, 2015 Revenues .............................................................................................................. Dividend revenue ................................................................................................. Unrealized Gain……………………………………………………………….. 6,000 Expenses............................................................................................................... Net income ...........................................................................................................

$75,000 3,000 70,00 0 $14,000


P8–8

Continued Masonite Tires Balance Sheet December 31, 2015 Noninvestment assets ...................... $ 165,000 Investment in ................................... Securities ....................................... 42,000 Total ................................................. $ 207,000

Liabilities ........... …………… $ 70,000 Stockholders’ equity. ...... 137,000* Total ................................ $207,000

* $137,000 = $123,000 Beginning balance + Net Income of $14,000 b. Under the equity method, an investment is initially recorded at its cost and subsequently adjusted for changes in the investee's net assets. Therefore, any net income or loss generated by the investee and any dividends declared by the investee cause the investment account to be adjusted. (1)

Masonite Tires Balance Sheet January 1, 2014 Noninvestment assets ...................... $ 120,000 Liabilities.......... ………………. $ 70,000 Investment in equity securities ...... 40,000 Stockholders' equity .......... 90,000 Total ................................................. $ 160,000 Total .................................. $160,000

(2)

During 2014, Masonite Tires would make the following original and adjusting entries: Cash (+A) ......................................................................................... Revenues (R, +SE) ..................................................................... Earned noninvestment revenues.

85,000

Expenses (E, –SE)............................................................................. Cash (–A) .................................................................................. Incurred expenses.

50,000

Cash (+A) ......................................................................................... Investment in Equity Securities (–A) ........................................ Received dividends (equity method).

2,000

Investment in Equity Securities (+A) ............................................... Income from Equity Investments (R, +SE) ................................ Earned investment revenue (equity method).

3,000*

* $3,000 = Investee's $15,000 income  20%

85,000

50,000

2,000

3,000


P8–8

Continued Masonite Tires Income Statement For the Year Ended December 31, 2014 Revenues: Revenue ........................................................................................................ Income from equity investments .................................................................. Expenses............................................................................................................. Net income .........................................................................................................

$ 85,000 3,000 50,000 $ 38,000

Masonite Tires Balance Sheet December 31, 2014 Noninvestment assets ...................... $ 157,000 Investment in equity securities ...... 41,000 Total ................................................. $ 198,000

Liabilities…………… $ 70,000 Stockholders’ equity. 128,000* Total………………… $198,000

* 1/1/2014 Retained Earnings of $90,000 (see part A) plus Net Income ($38,000). (3)

During 2015, Masonite Tires would make the following original and adjusting entries: Cash (+A) ......................................................................................... Revenues (R, +SE) ..................................................................... Earned noninvestment revenues.

75,000

Expenses (E, –SE)............................................................................. Cash (–A) .................................................................................. Incurred expenses.

70,000

Cash (+A) ......................................................................................... Investment in Equity Securities (–A) ........................................ Received dividends (equity method).

3,000

Investment in Equity Securities (+A) ............................................... Income from Equity Investments (R, +SE) ................................ Earned investment revenue (equity method).

4,000*

75,000

70,000

3,000

4,000

* $4,000 = Investee's $20,000 net income  20% Masonite Tires Income Statement For the Year Ended December 31, 2015 Revenues: Revenue ........................................................................................................ Income from equity investments .................................................................. Expenses.............................................................................................................

$75,000 4,000 70,000


Net income .........................................................................................................

P8–8

$

9,000

Concluded Masonite Tires Balance Sheet December 31, 2015 Noninvestment assets ...................... Investment in equity securities ...... Total .................................................

$ 165,000 Liabilities…………….. $ 70,000 42,000 Stockholders’ equity. ....... 137,000* $ 207,000 Total………………… .......... $207,000

* 12/31/14 Retained Earnings ($128,000) plus Net Income ($9,000). c.

Using the equity method versus the mark-to-market method may be desirable due to debt covenants and incentive compensation, as long as the investee company is earning Net Income. Under the equity method, a portion of the investee's net income flows through to the investor's income statement and through the closing process into stockholders' equity. This increase in stockholders' equity will, holding everything else constant, decrease the investor's debt/equity ratio. In this problem, Masonite's debt/equity ratio as of December 31, 2015 is the same under the equity method and the mark-tomarket method. Further, increased net income would be desirable to management if they receive incentive compensation based on net income. In some cases owning 20% of a company does not constitute substantial influence over the investee. To be able to substantially influence a company means that Masonite must be able to influence general corporate policy, dividend policy, and so forth. If another investor owns a larger percentage of the company than Masonite, that investor could exert more influence than Masonite. In the extreme, some other investor may own 80% of the investee while Masonite owns the remaining 20%. In such a case, Masonite could not exert a substantial influence on the investee and should not account for the investment using the equity method.

P8–9 a. Investment in Subsidiary (+A) ................................................................... Cash (–A)............................................................................................. Purchased Martin Monthly.

62,000 62,000

Alsop Ltd. Consolidated Balance Sheet December 31, 2014 Tangible assets ............................... $ 204,000a Goodwill ........................................ 40,000b Total................................................ $ 244,000 a $204,000 b $40,000

Liabilities............................. Stockholders’ equity ........... Total....................................

$154,000 90,000 $244,000

= $180,000 of Alsop's assets – $62,000 of cash disbursed for investment + $86,000 fair market value of assets acquired. = $62,000 purchase price – $22,000 fair market value of individual net assets acquired (i.e., $86,000 – $64,000).


b. Investment in Equity Securities (+A) ......................................................... Cash (–A)............................................................................................. Invested in Martin Monthly.

P8–9

62,000 62,000

Concluded

Alsop Ltd. Consolidated Balance Sheet December 31, 2014 Noninvestment assets ....................... $ 118,000 Liabilities..................................... Investment in equity securities ....... 62,000 Stockholders’ equity ................... Total................................................... $ 180,000 Total............................................ c.

$ 90,000 90,000 $180,000

Debt/equity ratio = Liabilities ÷ Stockholders' equity Purchase method: Equity method:

$154,000 ÷ $90,000 ÷

$90,000 = $90,000 =

1.71 1.00

Alsop's debt/equity ratio is considerably lower if the investment is accounted for using the equity method than if it is accounted for using the purchase method. If Alsop has any existing debt covenants that specify a maximum debt/equity ratio, the method used to account for this investment might affect whether Alsop violates its debt covenant. The probability that Alsop will violate any existing debt covenant is greater if the investment is accounted for using the purchase method than if it is accounted for using the equity method.

P8–10 a.

Short-Term Investments at 12/31/14

Trading Securities at 12/31/14

=

Trading Securities at 12/31/14

=

Trading Securities at 1/1/14

= =

$130 $280

+

Availablefor-Sale Securities at 12/31/14

+

Purchases During the Year

Sales During the Year

+

$280

[$240 – $80 –$30]

Total short-term investment as of 12/31/14 is $290, of which $280 is in trading securities. Therefore, the remaining $10 is in Available-for-sale Securities. b. Balance Sheet Carrying Value of Trading Securities Sold During 2014

=

Cash Received from the Sale of Trading Securities During 2014

Realized and Unrealized Gain on Short-Term Investments During 2014

=

$240

[80 + 30]


= c.

$130

Earnings per share dollar amount reported by the affiliate: Total Income of the Affiliate

÷

Total # of Shares Outstanding

Total Income of the Affiliate

= =

$40,000 ÷ .40 $100,000

P8–10

Concluded

Total Number of Shares Outstanding

Earnings per Share of the Affiliate =

= =

50,000 ÷ .40 125,000 shares

$100,000 ÷ 125,000 = $.80/share

d. Per share dividend declared by the affiliate: Income Equity Investment – Equity Income in Excess of Cash Received Number of Shares Purchased from Affiliate =

$40,000 − $30,000 = $.20 per share 50,000

P8–11 a. Investment in Subsidiary (+A) ................................................................... Cash (–A)............................................................................................. Invested in subsidiary.

180,000 180,000

After posting this entry, Rice's Cash account would decrease from $196,000 to $16,000, and the Investment in Subsidiary account would increase from $0 to $180,000. b. Adjustments and Eliminations Rachel Debit Credit

Accounts

Rice

Cash Accounts Receivable Inventory Investment in Sub. Fixed Assets Goodwill Total Assets

16,000 150,000 300,000 180,000 400,000 0 1,046,000

10,000 40,000 40,000 0 130,000 0 220,000

Accounts Payable L-T Liabilities Common Stock APIC, Common Stock

80,000 300,000 400,000 140,000

20,000 50,000 90,000 10,000

30,000 180,000 10,000 10,000 40,000

90,000 10,000

190,000

Consolidated Balance Sheet 26,000 190,000 370,000 0 520,000 10,000 1,116,000 100,000 350,000 400,000 140,000


Retained Earnings Total Liabilities & Stkhlders' Equity

126,000

50,000

50,000

1,046,000

220,000

150,000

126,000 0

1,116,000

The adjustments and eliminations columns (1) adjust assets to market value, (2) eliminate the Investment account, (3) eliminate the stockholders' equity section of Rachel, and (4) recognize goodwill. The goodwill of $10,000 equals the excess of the $180,000 purchase price over the $170,000 fair market value of Rachel's individual net assets (i.e., $10,000 in cash + $40,000 in accounts receivable + $70,000 in inventory + $120,000 in fixed assets – $20,000 in accounts payable – $50,000 in long-term liabilities).

P8–12 a. Investment in Subsidiary (+A) ................................................................... Cash (–A)............................................................................................. Invested in subsidiary.

136,000 136,000

After posting this entry, Rice's Cash account would decrease from $196,000 to $60,000, and the Investment in Subsidiary account would increase from $0 to $136,000. b. Adjustments and __Eliminations___ Rachel Debit Credit

Accounts

Rice

Cash Accounts Receivable Inventory Investment in Sub. Fixed Assets Total Assets

60,000 150,000 300,000 136,000 400,000 1,046,000

10,000 40,000 40,000 0 130,000 220,000

Accounts Payable L-T Liabilities Minority Interest Common Stock APIC, Common Stock Retained Earnings Total Liabilities & Stkhlders' Equity

80,000 300,000 0 400,000 140,000 126,000

20,000 50,000 0 90,000 10,000 50,000

90,000 10,000 50,000

1,046,000

220,000

150,000

30,000

30,000

136,000 10,000 146,000

34,000

34,000

Consolidated Balance Sheet 70,000 190,000 370,000 0 520,000 1,150,000 100,000 350,000 34,000 400,000 140,000 126,000 1,150,000

The adjustments and eliminations columns (1) adjust assets to market value, (2) eliminate the Investment account, (3) eliminate the stockholders' equity section of Rachel, and (4) recognize minority interest. The minority interest of $34,000 equals 20% (the amount not purchased by Rice) of the 100% value of Rachel as determined by the price paid for 80% [($136,000/.80 x .20]


P8–13 a. Investment in Subsidiary (+A) ................................................................... Cash (–A)............................................................................................. Invested in subsidiary.

140,000 140,000

After posting this entry, Rice's Cash account would decrease from $196,000 to $56,000, and the Investment in Subsidiary account would increase from $0 to $140,000.

b. Adjustments and Eliminations Rachel Debit Credit

Accounts

Rice

Cash Accounts Receivable Inventory Investment in Sub. Fixed Assets Goodwill Total Assets

56,000 150,000 300,000 140,000 400,000 0 1,046,000

10,000 40,000 40,000 0 130,000 0 220,000

Accounts Payable L-T Liabilities Minority Interest Common Stock APIC, Common Stock Retained Earnings Total Liabilities & Stkhlders' Equity

80,000 300,000 0 400,000 140,000 126,000

20,000 50,000 0 90,000 10,000 50,000

90,000 10,000 50,000

1,046,000

220,000

150,000

30,000 140,000 10,000 5,000 35,000

150,000

35,000

35,000

Consolidated Balance Sheet 66,000 190,000 370,000 0 520,000 5,000 1,151,000 100,000 350,000 35,000 400,000 140,000 126,000 1,151,000

The adjustments and eliminations columns (1) adjust assets to market value, (2) eliminate the Investment account, (3) eliminate the stockholders' equity section of Rachel, and (4) recognize both goodwill and minority interest. The goodwill of $5,000 equals the excess of the valuation of 100% of Rachel based on the price paid for 80% ($140,000/.80 = $175,000) over the fair market value of the individual net assets controlled by Rice, ($10,000 + $40,000 + $70,000 + $120,000 – $20,000 – $50,000). The minority interest of $35,000 equals 20% (the amount not purchased by Rice) of the 100% value of Rachel as determined by the price paid for 80% [($140,000/.80 x .20]


c.

If Rice uses IFRS and assumes the minority shareholders have no equity in goodwill, then the noncontrolling interest would be $34,000 (20% of the FMV of the net assets [$240,000 - $70,000]) and goodwill would be $4,000.

P8–14 a. Net income would be affected by the values reported for inventory (through cost of goods sold), and fixed assets (through depreciation expense). Thus, any attempt to maximize net income in the following year must consider the effect of inventory, fixed assets, and goodwill. Since inventory is a current asset, the assumption is that the costs in inventory will flow through to the income statement in the following year. Thus, Rice would want to value the inventory at $65,000. Fixed assets are long-term assets and are thus depreciated over a period of time (most likely over a period of time not exceeding 10 or 12 years). Rice would be able to report the lowest amount possible for depreciation expense the following year if it capitalizes the fixed assets at $115,000 instead of at $125,000. The tradeoff of allocating $65,000 to inventory and $115,000 to fixed assets is that Rice will have to report a higher amount for goodwill. That is, if Rice makes the allocations given above, Rice will

P8–14 Concluded have to allocate $20,000 to goodwill (i.e., purchase price of $180,000 less sum of the fair market values of Rachel's individual net assets as follows: $10,000 in cash + $40,000 in accounts receivable + $65,000 in inventory + $115,000 in fixed assets – $20,000 in accounts payable – $50,000 in longterm liabilities). b. To minimize next year's reported income for tax purposes, Rice would like to maximize the value of assets that it will consume next year. Thus, Rice would like to maximize the value allocated to inventory because the inventory would probably be sold next year, and thus be allocated to cost of goods sold next year. Similarly, Rice would like to maximize the amount allocated to fixed assets relative to the amount allocated to goodwill. Thus, Rice would like to allocate $125,000 to fixed assets. These two allocations would result in the sum of the fair market values of Rachel's individual net assets being $180,000 ($10,000 in cash + $40,000 in accounts receivable + $75,000 in inventory + $125,000 in fixed assets – $20,000 in accounts payable – $50,000 in long-term liabilities). Since the purchase price was $180,000, Rice would not have to allocate anything to goodwill.

P8–15 Company X Investment in Subsidiary (+A) .......................................................................... Cash (–A) ................................................................................................... Cash (+A) .......................................................................................................... Accounts Receivable (+A) ................................................................................. Inventory (+A) .................................................................................................. Fixed Assets (+A) .............................................................................................. Goodwill ........................................................................................................... Current Liabilities (+L) ............................................................................... Long-Term Liabilities (+L) .......................................................................... Minority Interest (+L) ................................................................................ Investment in Subsidiary (–A) ................................................................... * $20,000

** $21,200

= Purchase Price of 100% – FMV of Net Assets Purchased = [$84,800/.80] – (FMV of Total Assets – FMV of Total Liabilities) = $106,000 – ($6,000 + $12,000 + $30,000 + $70,000 – $32,000) = Purchase Price of 100%  Percentage Not Owned by Parent

84,800 84,800 6,000 12,000 30,000 70,000 20,000* 7,000 25,000 21,200** 84,800


= [($84,800)/.80] x 20% Company Y Investment in Subsidiary (+A) .......................................................................... Cash (–A) ................................................................................................... Cash (+A) .......................................................................................................... Accounts Receivable (+A) ................................................................................. Inventory (+A) .................................................................................................. Fixed Assets (+A) .............................................................................................. Goodwill (+A).................................................................................................... Accounts Payable (+L) ............................................................................... Long-Term Liabilities (+L) .......................................................................... Minority Interest (+L) ................................................................................ Investment in Subsidiary (-A) .................................................................... Potential consolidating entry. __________________ * $17,000 = Purchase Price of 100% – FMV of Net Assets Purchased

P8–15

Concluded

** $16,000

= [$24,000/.60] – (FMV of Total Assets – FMV of Total Liabilities) = $40,000 – ($4,000 + $9,000 + $12,000 + $30,000 – $32,000) = Purchase Price of 100%  Percentage Not Owned by Parent =

Cash (+A) .......................................................................................................... Accounts Receivable (+A) ................................................................................. Inventory (+A) .................................................................................................. Fixed Assets (+A) .............................................................................................. Goodwill (+A).................................................................................................... Accounts Payable (+L) ............................................................................... Long-Term Liabilities (+L) .......................................................................... Minority Interest (+L) ................................................................................ Investment in Subsidiary (–A) ...................................................................

** $5,500

24,000 4,000 9,000 12,000 30,000 17,000* 12,000 20,000 16,000** 24,000

[($24,000)/.60] x 40%

Company Z Investment in Subsidiary (+A) .......................................................................... Cash (–A) ...................................................................................................

* $3,000

24,000

16,500 16,500 2,000 7,000 18,000 15,000 3,000* 5,000 18,000 5,500** 16,500

= Purchase Price of 100% – FMV of Net Assets Purchased = [$16,500/.75] – (FMV of Total Assets – FMV of Total Liabilities) = $22,000 – ($2,000 + $7,000 + $18,000 + $15,000 – $23,000) = Purchase Price of 100%  Percentage Not Owned by Parent =

[($16,500)/.75] x 25%

P8–16 a. Debt/Equity Ratio = Total Liabilities ÷ Total Stockholders' Equity Prior to the acquisition of Atom, Inc., Mammoth's total liabilities were $230,000 and its total stockholders' equity was $270,000. Thus, Mammoth's debt/equity ratio was .85. By acquiring Atom, Inc., Mammoth would need to add Atom's liabilities to its own. If you assume that the book value of Atom's liabilities equals their fair market value, you can infer that the value of Atom's liabilities is $90,000. That is, the $180,000 fair market value of Atom's assets less $20,000 excess of market value over book value of assets less the $70,000 book value of Atom's net assets (i.e., total assets less total liabilities) equals $90,000. Thus, Mammoth's post-acquisition liabilities would be


$320,000. Minority interest would equal $60,000 (representing the 50% not owned by Mammoth and valuing the entire company at Mammoth’s purchase price of half the company, $60,000/.5 x 50%). Total liabilities including minority interest would therefore equal $380,000. The acquisition of Atom would not affect Mammoth's stockholders' equity. Mammoth's post-acquisition debt/equity ratio would therefore be $380,000/$270,000 = 1.41. b. If Mammoth accounted for this investment using the equity method, it would not have to combine Atom, Inc.'s assets and liabilities with its own. Instead, Mammoth would simply report an asset (i.e., Long-Term Investment in Equity Securities) on its balance sheet valued at $60,000. Thus, Mammoth's liabilities and stockholders' equity would not be affected, which means that its debt/equity ratio would remain at .85. As shown in part (a), however, Mammoth's debt/equity ratio would increase if it accounted for the investment using the purchase method. Therefore, by using the equity method, Mammoth presents a more favorable debt/equity ratio, which allows Mammoth to avoid moving closer to violating its debt covenant. Such a move gives the company more accounting flexibility and decreases the probability that it will eventually violate its debt covenant. For these reasons, Mammoth would probably prefer accounting for the investment in Atom, Inc., using the equity method rather than the purchase method.

ISSUES FOR DISCUSSION ID8–1 a. The market value of H&R Block’s marketable securities decreased in 2010 but increased in 2011 and 2012. b. H&R Block could manage its earnings by selling marketable securities that have either a gain or a loss in order to move its net income up or down. The gains or losses on securities that are not sold are recorded in the stockholders’ equity section of the balance sheet but are not recorded on the income statement until an actual sale has taken place. c.

These gains or losses would be reflected in the statement of comprehensive income. Since1998 companies have been required to prepare a statement of comprehensive income.

ID8–2 a. Trading securities are bought and held principally for the purpose of selling them in the near future with the objective of generating profit on short-term price changes. Investments not classified as trading securities are considered available-for-sale securities. Price increases, or decreases, would flow through directly to the stockholders’ equity section, via comprehensive income, in the case of availablefor-sale securities; a gain or loss for a trading security would flow through the income statement. b. Total comprehensive income includes all realized and unrealized gains and losses. The footnote describes the realized gains/losses; the unrealized gains/losses would also factor into the calculation of comprehensive income. c. Proceeds from sale of securities Minus: Gains on securities sold Cost of securities sold

ID8–3

2012 $8.400 0.014 $ 8.386

2010 $ 2.1 0.0 $ 2.1

.


a. Beginning LT Equity Investment + New Investments + Income from Affiliates – Dividends Received – Investments Sold = Ending LT Equity Investment; $3,718 + X + 752– 137 – 157 = $4,581; x = $405 AT & T must have purchased $405 million in long-term equity investments during 2012. b. The statement of cash flow will deduct the $752 million in “equity in net income of affiliates” (because this amount was not a cash inflow to A T & T), net of the $137 million in cash dividends received from affiliates. The cash amount paid for the new investments, as well as the cash amount received from the sale of investments, will also be disclosed. c.

Equity income is not a good measure of the cash AT & T received from its affiliates because there is no relationship between equity income and cash received from an affiliate. The equity income reported by AT & T represents its percentage of the income of the affiliate but does not represent the amount of cash that may have been paid in dividends to AT & T.

ID8–4 a. “Share of profits from associates” represents EADS’ share of the earnings of the companies in which EADS can assert significant influence. “Investment in associates” is the value that EADS assigns to companies in which it has significant influence; under the equity method approach, companies such as EADS record the initial investment at cost and adjust the investment carrying value with increases from associate income and decreases from associate dividend payments. “Results of companies accounted for by equity method” is a deduction on the statement of cash flows because those associate earnings are recorded on the income statement (see above) even though no cash proceeds are received. “Payments for investments in associates” represents cash outflows to purchase new equity positions in companies over which EADS can exert significant influence. “Proceeds from disposals of associates” represents cash inflows when equity stakes have been liquidated. b. Ending Balance Investments in Associates = Beginning Balance Investments in Associates plus Share of Profits from Associates + Payments for Investments in Associates – Dividends Received from Associates – Balance Sheet Value of Associates Disposed; 2,662 = 2,677 + 241 + 328 - 0 - X; X = 584 million euros c. The cash proceeds from disposal (232) was less than the carrying value of the investments that were sold (584), so the company recorded a 352 million euro loss.

ID8–5 a. The investment in associated companies on the balance sheet decreased in value because Sony claimed a share of the associates’ losses as a decrease to the value of the investments and because Sony may have also received dividends from affiliates during 2012. b. The 138,772,figure is an increase on the statement of cash flows because those associate losses were recorded on the income statement even though no cash was used by the affiliates; any cash dividends received were netted against that loss. c. If Sony were to account for these investments under the fair market value option, any increases or decreases in value would flow through earnings on the income statement. For Sony to claim a change in value, the company would be required to disclose its methodology of calculating its fair value estimate. Market values based on quoted prices in active markets for identical securities are


called Level 1 measurements; market values based on less reliable, observable, indirect inputs are called Level 2 measurements; and market values based on much less reliable, unobservable inputs are called Level 3 measurements. The level disclosed can aid the reader of the financial statements in determining the risks associated with changes in balance sheet value.

ID8–6 a. Under the equity method a substantive economic relationship exists between an investor and an investee. The investor records the original investment at cost. For each subsequent period the investor adjusts the carrying value of the investment by (1) the investor’s proportionate share of the income or loss of the investee, and (2) by the amount of the dividend received. In other words, the investor’s net assets increase due to the investee’s earnings and decrease due to the receipt of dividends. Generally, the amount of the dividends received is much less than the amount of earnings recognized by the investor. The investee company, just like any other business entity, pays only a small fraction of earnings in the form of dividends. Therefore, in the books of the investor there is a wide disparity in the amount of earnings recognized and the cash received due to dividends. Thus, investors must be careful in analyzing financial statements that use the equity method of accounting. b. Under the operating section on Chevron’s statement of cash flows, one will subtract “Equity income in excess of Cash Received” from the total net income.

ID8–7 a. With consolidated financial statements, the assets and liabilities of the investee company (subsidiary) are added to the assets and liabilities of the investing company (parent). Thus, the amount of assets and liabilities reported by the consolidated entity will be greater than the assets and liabilities reported by either the parent or subsidiary. Note, however, that the total amount of stockholders' equity reported by the consolidated entity will be the same as the parent's stockholders' equity. If the parent has any debt covenants, consolidated financial statements could cause the company to violate the covenant. For example, assume that the debt covenant specifies a maximum allowable debt/equity ratio. With consolidated financial statements, as mentioned above, the company's liabilities would increase due to the consolidation while its stockholders' equity would remain the same. Thus, the company's debt/equity ratio would increase, thereby pushing it closer to or over the maximum allowable debt/equity ratio.


Debt covenants are not as likely to be a problem for investments accounted for using the equity method. With the equity method, the parent reports its initial investment in the subsidiary in the account Investment in Equity Securities, and this account is reported on one line on the balance sheet. Over time, the parent increases the investment account when the subsidiary generates net income and reduces this investment account when the subsidiary generates net losses and when the subsidiary declares dividends. The balance in the investment account represents the parent's portion of both the subsidiary's assets and liabilities. This means that the parent's portion of the subsidiary's liabilities are not added to the parent's liabilities, thereby allowing the parent to engage in off-balance-sheet financing. Thus, the parent's portion of the subsidiary's liabilities would not affect the company's "official" debt/equity ratio, which, in turn, implies that the equity method should not cause a company to have "difficulties with bond indenture agreements." b. If users of financial statements are reasonably sophisticated, they should be able to understand more complex business and reporting situations. This implies that as the users' sophistication increases, the FASB has a decreased responsibility for developing standards that encompass and account for every possible business situation that arises. Thus, the FASB would be able to promote broader standards as user sophistication increases, without a lessening of its responsibilities to users of financial statements.

ID8–8 a. Unrealized losses for trading securities are reported on the income statement. If Starbucks’ $57.6 million portfolio of trading securities dropped in value by 50%, the income statement for the month of October, 2012 would include an unrealized loss of $28.8 million, significantly reducing the company’s profitability. The unrealized loss on the available-for-sale securities would not affect the company’s income statement. b. Designating all marketable securities as available-for-sale would have eliminated the effect on the income statement from a stock market crash. On the other hand, designating all securities to be trading would have increased the negative impact on the company’s profits. c.

Protecting earnings from daily fluctuations in the stock market is a sound reason that management might choose to designate securities to be available-for-sale.

ID8–9 a. Realized gains (when shares are sold) are always reported on the income statement, so the two articles do not tell us whether the investments were designated as trading or available-for-sale securities. b. Analysts are interested in earnings that are sustainable and can be repeated in future periods. Profits from operations are repeatable; one-time gains from the sale of investments are not necessarily something a reader of financial statements can count on in future reporting periods. c.

Quality of earnings and sustainability of earnings would be measured by separating operating from other profits.

ID8–10 Unrealized gains/losses are not reported on the income statement for available-for-sale securities. Firms with large investment portfolios, such as banks, can therefore keep the fluctuations in the securities values


from affecting the income statement. Only realized gains/losses will hit earnings, allowing banks to time the sale of the securities to coincide with either positive or negative earnings. For example, if a bank is having an unusually strong year, surpassing analyst expectations, the bank could sell securities that have dropped in value from the purchase date; when the realized losses hit the income statement, they are offset by the higher operating earnings. Had those securities been designated trading securities, the unrealized losses would have been booked on the income statement at the time of the market price decline (which might have happened in a period where earnings were not above expectations). Designating the securities as available-for-sale allows management to time when they want to take the realized gain/loss on the income statement.

ID8–11 Comprehensive income includes all non-owner related changes in stockholders’ equity that do not appear on the income statement and are not reflected in the retained earnings balance. As a global company, Eli Lilly sells its pharmaceutical products in foreign countires for foreign currencies. In 2012, the value of those foregin currencies relative to the U.S. dollar boosted Lilly equity, but the exchange rates moved in the opposite direction in 2011 and 2010, decreasing equity for the company. Those changes are reflected in the statement of comprehensive income (and are not posted to retained earnings through the income statement). Similarly, the unrealized gains and losses of the company’s portfolio of available-for-sale securities in 2010-2012 were posted directly to the equity section of the balance sheet (bypassing the income statement) and were reflected as changes to comprehensive income. Finally, SFAS 158 requires companies to reflect the funding status of pension and other plans through comprehensive income. The large negative adjustments Lilly recorded in 2012 and 2011 reflect a loss of equity due to the funding of its plans, but these adjustments are posted directly to stockholders’ equity and do not affect the income statement; similarly, the positive adjustment in 2010 affects equity through comprehensive income but does not touch annual earnings on the income statement.

ID8–12 a. Minority or noncontrolling interest is recognized on the balance sheet because the acquiring company does not purchase 100% of the target company but does purchasing a controlling stake (greater than 50%) and therefore consolidates financial statements with the target company. The noncontrolling portion must be recognized because that interest in the net assets acquired is held by outsiders (not shareholders of the acquiring company). The dollar amount of the noncontrolling interest is calculated by either multiplying the number of shares owned by the minority shareholder(s) times each share’s value or, in the case of IFRS, multiplying the noncontrolling interest by the net assets acquired (following the assumption that minority shareholders do not have an ownership interest in the goodwill associated with the acquisition). b. The argument concerning the placement of minority interest is between the liability section and the shareholders’ equity section on the balance sheet. Currently, under both U.S. GAAP and IFRS, the noncontrollng interest is listed under shareholders’ equity. c. Verizon would no longer show the noncontrolling interest on its balance sheet; instead, the wireless business would be 100% owned and therefore consolidated with the rest of the company,


without any related adjustment for the portion not owned. Vodafone, if it sells its ownership, will not have any equity investment shown on its balance sheet.

ID8–13 a. Google carries $33.3 billion of short-term investments as of the end of 2012 (versus $34.6 billion in 2011). The first footnote indicates that the investments are designated available-for-sale securities. b. The available-for-sale securities are carried at fair values, which were determined using Level 1 (active markets) and Level 2 (less actively traded) measurements (Note 3).

c.

In May, 2012 Google acquired Motorola, a provider of mobile communication devices (Note 6). The journal entry for the acquisition is summarized: ( dollars in billions) Cash Other Assets Patents Intangible Assets Goodwill Cash

2.9 0.8 5.5 0.7 2.5 12.4

d. Goodwill was $10,537 million and $7,346 million in 2012 and 2011, respectively. The Motorola acquisition, as well as 52 other acquisitions (Note 6), are responsilbe for the increase in Goodwill. e. The balance sheet line item “Non-marketable equity securities” includes investments that the company accounts for under the equity method (Note 1). .


CHAPTER 9 LONG-LIVED ASSETS BRIEF EXERCISES BE9–1 a. The new method, straight-line depreciation, will increase net income in the early years and reduce income in the later years versus using an accelerated method. An accelerated method of depreciation increases the depreciation charges in the early years of the life of an asset and reduces the depreciation charges in the later years. b. Allegheny may have decided that it wanted depreciation charges to be spread evenly over the life of an asset so that the impact on net income in any one reporting period was less. It may also feel that it will make its financial statements easier to compare with its competitors. During periods of high fixed asset investment Allegheny’s results may look unfavorable versus other companies that use a straightline method instead of an accelerated method. c.

In the annual report one could look through the first footnote. This footnote typically highlights all of the significant accounting policies and methods used by the company to prepare the financial statements.

BE9–2 a. The recognition of depreciation and amortization affects the basic accounting equation by reducing assets and reducing retained earnings in the stockholders’ equity section. Fixed assets such as property, plant and equipment are reduced through depreciation charges (which are collected in the contra asset account Accumulated Depreciation) which lower net income. Intangible assets are reduced by amortization charges which reduce the net income of the company. This reduction in net income reduces the retained earnings of the company. b. Boeing recognized a loss of $11 million, computed as follows: Accumulated depreciation 2011 + Depreciation charges for 2012 – Accumulated depreciation 2012 Accumulated depreciation on assets sold

$13,993 million 1,248 million 14,645 million $ 596 million

PP&E 2011 + PP&E purchases for 2012 – PP&E 2012 PP&E sold

$23,306 million 1,703 million 24,305 million $ 704 million

1


Cost of PP&E – Accumulated depreciation on assets sold Net book value of PP&E sold Cash received from sale Loss on sale of PP&E

$704 million 596 million 108 million 97 million $ 11 million

Derived Journal Entry: Cash (+A) 97 Accumulated Depreciation (+A) 596 Loss on Sale (E, -SE) 11 Property, Plant & Equipment (-A)

704

The loss on the sale of property, plant and equipment would be shown in the income statement, usually in an “other gains and losses” section. These transactions would affect the statement of cash flows in the “funds from investing activities section”. Any sales would be a source of funds in the amount of cash received.

BE9–3 a. Johnson and Johnson invested $39 million ($793– $754) of land during 2012. b. Accumulated depreciation increased during 2012 because of depreciation expense taken by Johnson and Johnson. Instead of reducing the asset account directly, depreciation expense is added to accumulated depreciation, which offsets the asset account to show its reduction in value. c.

If the company used an accelerated method of depreciation, the assets would be shown at a lower net value in the early years. Accelerated methods take more depreciation charges in the early years of an asset’s life and less in the later years, when compared to the straight-line method.

d. Johnson and Johnson would show $16,097 million for property, plant and equipment on its financial statement for 2012. The gross amount and the accumulated depreciation would be disclosed in the footnote.

BE9–4 a. The Depreciation & Amortization adjustment and the Impairment adjustment both were recorded by charging an expense to earnings and lowering the carrying value of the long-term assets. The gain was recorded when the long term assets were sold for a price greater than the carrying value; the debit to cash and credit to the sold assets were balanced out with a credit to the temporary account “Gain on sale”. b. The Depreciation & Amortization adjustment and the Impairment charge are both non-cash expenses that need to be added back to earnings on the statement of cash flow. (The expenses reduced earnings but did not reduce cash, so the add-back is necessary in the statements conversion of accrual earnings to actual cash flow.) The gain is a deduction because activities involving long-term assets are not included in the Operating section. c. The gain is adjusted out of the Operating section of the statement of cash flow because the cash received from the sale of long-term assets is an Investing activity and needs to be reflected in a different section of the statement of cash flow; additionally, the Investing section will reflect the entire amount of cash received in the sale, not simply the gain.


d. If the company followed U.S. GAAP, similar adjustments would appear.

EXERCISES E9–1 a. Lowery, Inc., should capitalize all costs associated with getting the equipment in a serviceable condition and location. These costs would be the actual purchase price of $920,000, the transportation cost of $62,000, and the insurance cost of $10,000. Therefore, the total cost of the equipment is $992,000. b. The depreciation base equals the dollar amount of a fixed asset's cost that the company does not expect to recover over the asset's useful life, but instead expects to consume over the asset's useful life. Since the plant equipment's total cost is $992,000 and since Lowery, Inc., expects to sell the equipment for $50,000 at the end of its useful life, Lowery, Inc., does not expect to recover $942,000 of the asset's cost. Therefore, the depreciation base equals $942,000. The depreciation base always equals the capitalized cost of a fixed asset less its estimated salvage value. c.

The amount that will be depreciated over the life of the plant equipment is its depreciation base. The depreciation base equals the amount of the equipment's future benefits that the company will consume. The outflow of future benefits are expenses, in this case depreciation expense. Therefore, the total amount that Lowery, Inc., will depreciate over the equipment's useful life is $942,000.

E9–2 Lot 1

Lot 2

Lot 3

Lot 4

Revenue $ 160,000 $ 120,000 $ 60,000 $ 60,000 Expenses 128,000* 96,000* 48,000* 48,000* Net income $ 32,000 $ 24,000 $ 12,000 $ 12,000 ________________ * Expenses were calculated as follows: 1. Calculate total market value. Total Market value = $160,000 + $120,000 + $60,000 + $60,000 = $400,000 2. Allocate costs to each lot based upon relative market values. Lot 1 = $320,000  (160,000/400,000) = $128,000 Lot 2 = $320,000  (120,000/400,000) = $ 96,000 Lot 3 = $320,000  (60,000/400,000) = $ 48,000 Lot 4 = $320,000  (60,000/400000) = $ 48,000


E9–3 a. All costs that are necessary and reasonable to get an asset ready for its intended use should be capitalized as part of the cost of that asset. In the case of property, plant, and equipment, "ready for its intended use" means that the asset is in a serviceable condition and location.

Item

Land

Tract of land Demolition of warehouse Scrap from warehouse Construction of building Driveway and parking lot Permanent landscaping Total

$90,000 10,000 (7,000)

Land Improvements

Building

$140,000 $32,000 4,000 $ 97,000

$32,000

$140,000

b. Land: Since land is assumed to have an indefinite life, it is never depreciated. Land Improvements: Depreciation Expense—Land Improvements (E, –SE) ............................... Accumulated Depreciation—Land Improvements (–A) ....................... Depreciated land improvements.

1,600 1,600

Building: Depreciation Expense—Building (E, –SE) .................................................. Accumulated Depreciation—Building (–A) .......................................... Depreciated building.

7,000 7,000

E9–4 a. b. c. d. e. f. g. h. i.

Maintenance Maintenance Maintenance Betterment Maintenance Maintenance Betterment Maintenance Betterment

Note:

The classification of these expenditures can be quite subjective. Some accountants might very well classify some of these expenditures differently. For example, one might argue that the cost of the


muffler in (h) is actually a betterment expenditure if the reduced noise allows workers to work more efficiently, thereby increasing the productive capacity of the machine.

E9–5 a. (1)

Expensed immediately: Income Statement Revenues Amortization Other expenses Net income

2017

2016

2015

$ 65,000 0 20,000 $ 45,000

$ 65,000 0 20,000 $ 45,000

$ 65,000 40,000 20,000 $ 5,000

Balance Sheet Assets Current assets Long-lived assets (including land) Total assets Liabilities and Stockholders' Equity Liabilities Stockholders' equity Total liabilities & stockholders' equity

12/31/17

12/31/16

12/31/15

$ 135,000

$ 90,000

$ 45,000

50,000 $ 185,000

50,000 $ 140,000

50,000 $ 95,000

$ 35,000 150,000

$ 35,000 105,000

$ 35,000 60,000

$ 185,000

$ 140,000

$ 95,000


E9–5 (2)

Continued Amortized over two years: Income Statement Revenues Amortization Other expenses Net income

2017

2016

2015

$ 65,000 0 20,000 $ 45,000

$ 65,000 20,000 20,000 $ 25,000

$ 65,000 20,000 20,000 $ 25,000

Balance Sheet 12/31/17 Assets Current assets Long-lived assets (including land) Total assets Liabilities and Stockholders' Equity Liabilities Stockholders' equity Total liabilities & stockholders' equity (3)

12/31/16

12/31/15

$ 135,000

$ 90,000

$ 45,000

50,000 $ 185,000

50,000 $ 140,000

70,000 $ 115,000

$ 35,000 150,000

$ 35,000 105,000

$ 35,000 80,000

$ 185,000

$ 140,000

$ 115,000

2017

2016

2015

$ 65,000 13,334 20,000 $ 31,666

$ 65,000 13,333 20,000 $ 31,667

$ 65,000 13,333 20,000 $ 31,667

Amortized over three years: Income Statement Revenues Amortization Other expenses Net income Balance Sheet Assets Current assets Long-lived assets (including land) Total assets Liabilities and Stockholders' Equity Liabilities Stockholders' equity Total liabilities & stockholders' equity

b. Method 1: Method 2: Method 3:

12/31/17

12/31/16

12/31/15

$ 135,000

$ 90,000

$ 45,000

50,000 $ 185,000

63,334 $ 153,334

76,667 $ 121,667

$ 35,000 150,000

$ 35,000 118,334

$ 35,000 86,667

$ 185,000

$ 153,334

$ 121,667

2017

2016

2015

Total

$45,000 45,000 31,666

$45,000 25,000 31,667

$ 5,000 25,000 31,667

$95,000 95,000 95,000


E9–5 c.

Concluded

The balance sheets under all three methods report identical amounts for each balance sheet account. Since the asset was fully amortized by December 31, 2017, the method used to amortize the asset does not affect the amounts reported on the balance sheet as of December 31, 2017.

E9–6 a. and b. Stork Freight Company Income Statement For the Year Ended December 31 12-Year Useful Life Revenues Expenses: Operating expenses Depreciation expense Total expenses Net income

6-Year Useful Life

$ 50,000,000

$ 50,000,000

$ 25,000,000 1,250,000 26,250,000 $ 23,750,000

$ 25,000,000 2,500,000 27,500,000 $ 22,500,000

The percentage decrease in net income would be approximately 5.26% [($22,500,000 – $23,750,000) ÷ $23,750,000]. c. Net income Dividend payout percentage Dividends

12-Year Useful Life

6-Year Useful Life

$ 23,750,000 30% $ 7,125,000

$ 22,500,000 30% $ 6,750,000

The difference in dividends due simply to using different estimated useful lives for the planes would be $375,000 ($7,125,000 – $6,750,000). However, it should be noted that in the 6-year useful life example, dividends in years 7 – 12 would be higher (due to the lack of depreciation expense).

E9–7 a. An asset's book value equals the asset's initial capitalized value less the associated accumulated depreciation. With straight-line depreciation, accumulated depreciation equals depreciation expense per year times the number of years the asset has been used. Therefore, the asset's book value would be calculated as follows: Depreciation expense per year

= (Cost – Salvage Value) ÷ Useful Life = ($60,000 – $12,000) ÷ 5 years = $9,600 per year

Book Value = Capitalized Cost – Accumulated Depreciation = $60,000 – ($9,600  3 years) = $31,200


E9–7

Concluded

b. Depreciation Expense

= [(Cost – Accumulated Depreciation) – Salvage Value] ÷ Remaining Useful Life = (Book value – Salvage value) ÷ Remaining useful life = ($31,200 – $12,000) ÷ 5 remaining years = $3,840

Depreciation Expense (E, –SE) ................................................................... Accumulated Depreciation (–A) ......................................................... Depreciated asset for 2014.

3,840 3,840

E9–8 Objective (a) (b) (c) (d) (e) (f) (g) (h) 1

2

3

StraightLine

Double-DecliningBalance

Activity Method

x1 x

x1 x x x x

x1 x x2

x x x

x

x3 x

Under certain conditions, all three methods could meet this objective. However, for the straight-line method and the double-declining-balance method, this objective will be met only by chance. The activity method will always meet this objective because depreciation is based upon the actual use of the asset. It is possible that the activity method would generate the largest net income in the last year of an asset's useful life. However, this result would be due to the company's use patterns of the asset and would not be due to the depreciation method per se. See note (2). The same rationale would hold in this case too.

E9–9 a. (1)

Straight-line depreciation: Depreciation per Year = (Cost – Salvage Value) ÷ Useful Life = ($300,000 – $60,000) ÷ 4 years = $60,000 per year for 2014, 2015, 2016, and 2017


E9–9 (2)

Concluded Double-declining-balance depreciation: Date

Depreciation Factor

Depreciation Expense

Cost

Accumulated Depreciation

Book Value

1/1/14 $300,000 $ 0 $300,000 12/31/14 50% $150,000a 300,000 150,000 150,000 12/31/15 50% 75,000 300,000 225,000 75,000 12/31/16 50% 15,000b 300,000 240,000 60,000 12/31/17 50% 0 300,000 240,000 60,000 _______________ a Depreciation Expense = Book Value at Beginning of the Period  Depreciation Factor b Book Value  Depreciation Factor = $75,000  50% = $37,500. If Benick Industries depreciated $37,500 in 2016, the asset's book value would drop below its salvage value. To prevent this from happening, depreciation expense for 2016 can be only $15,000. b. A manager should consider the costs and benefits associated with each depreciation method. The most likely benefit is the impact of depreciation methods on income taxes. An accelerated method decreases the present value of tax payments. However, since there is no requirement that a company use the same depreciation method for financial reporting purposes as it does for tax reporting, tax considerations are not an issue for financial reporting. A manager should also consider the bookkeeping costs associated with each method. However, with computers the bookkeeping costs should be relatively consistent across methods. Finally, since the choice of depreciation methods affects net income, managers might consider the impact of the different depreciation methods on contracts such as debt covenants and incentive compensation contracts. Comparability with other firms in the same industry may also be a factor.

E9–10 a. Computer System (+A) .............................................................................. Cash (–A) .................................................................................... Purchased computer system. Note:

b. (1)

335,000 335,000

Capitalizing the $10,000 of training costs could be debated. But, without incurring these costs, the computer system would not be in a serviceable condition. Hence, the training costs meet the requirement to be capitalized as part of the fixed asset. Straight-line depreciation: Depreciation per Year = (Cost – Salvage Value) ÷ Useful Life = ($335,000 – $70,000) ÷ 5 years = $53,000 per year for 2014, 2015, 2016, 2017, and 2018


E9–10 (2)

Concluded Double-declining-balance depreciation:

Date

Depreciation Factor

Depreciation Expense

Cost

Accumulated Depreciation

Book Value

1/1/14 $335,000 $ 0 $335,000 12/31/14 40% $134,000a 335,000 134,000 201,000 12/31/15 40% 80,400 335,000 214,400 120,600 12/31/16 40% 48,240 335,000 262,640 72,360 12/31/17 40% 2,360b 335,000 265,000 70,000 12/31/18 40% 0 335,000 _____________ a Depreciation expense = Book value at beginning of the period  Depreciation factor b Book value  Depreciation factor = $72,360  40% = $28,944. If Stockton Corporation depreciated $28,944 in 2017, the asset's book value would drop below its salvage value. To prevent this from happening, depreciation expense for 2017 can be only $2,360. c.

Depreciation Expense (E, –SE) ............................................................ Accumulated Depreciation (–A) ................................................. Depreciated fixed asset for 2014.

134,000 134,000

E9–11 1. Activity Method: Depreciation Expense per Mile

= =

($100,000 – $20,000) ÷ 200,000 Miles $0.4/Mile

Depreciation Expense (E, –SE) ................................................................... Accumulated Depreciation (–A) ......................................................... Depreciated asset for 2014.

19,200

Depreciation Expense (E, –SE) ................................................................... Accumulated Depreciation (–A) ......................................................... Depreciated asset for 2015.

14,000

Depreciation Expense (E, –SE) ................................................................... Accumulated Depreciation (–A) ......................................................... Depreciated asset for 2016.

16,000

Depreciation Expense (E, –SE) ................................................................... Accumulated Depreciation (–A) ......................................................... Depreciated asset for 2017.

10,000

19,200

14,000

16,000

10,000


E9–11

Concluded

Depreciation Expense (E, –SE) ................................................................... Accumulated Depreciation (–A) ......................................................... Depreciated asset for 2018.

14,000

Depreciation Expense (E, –SE) ................................................................... Accumulated Depreciation (–A) ......................................................... Depreciated asset for 2019.

4,000

Cash (+A) ................................................................................................ Accumulated Depreciation (+A) ................................................................ Loss on Sale of Truck (Lo, –SE) .................................................................. Truck (–A) ........................................................................................... Sold truck.

12,000 77,200 10,800

2. Straight-line Method: Depreciation Expense per Year

4,000

100,000

= ($100,000 – $20,000) ÷ 5 Years = $16,000/year

Depreciation Expense (E, –SE) ................................................................... Accumulated Depreciation (–A) ......................................................... Depreciated asset. Note:

14,000

16,000 16,000

This entry would be made each year for five years. No entry would be made in Year 6 since the truck's estimated useful life ended at the end of Year 5, which means that the truck would have been depreciated down to its estimated salvage value.

Cash (+A) .............................................................................................. Accumulated Depreciation (+A) ................................................................ Loss on Sale of Truck (Lo, –SE) .................................................................. Truck (–A).......................................................................................... Sold truck.

12,000 80,000 8,000 100,000

E9–12 a. Depletion (E, –SE) ...................................................................................... Oil Deposits (–A) ................................................................................. Depleted oil deposits. ____________ * $1,200,000 = ($4,000,000 ÷ 100,000 barrels)  30,000 barrels extracted

1,200,000*

b. Depletion (E, –SE) ...................................................................................... Oil Deposits (–A) ................................................................................. Depleted oil deposits. ____________ * $2,000,000 = ($4,000,000 ÷ 100,000 barrels)  50,000 barrels extracted

2,000,000*

1,200,000

2,000,000


c.

$800,000

E9–13 a. Year 2014 2015 2016 2017

Depreciation Expense Per Company's Books $120,000 0 0 0

Correct Depr. Exp. $25,000 25,000 25,000 25,000

Annual Difference $95,000 (25,000) (25,000) (25,000)

Cumulative Difference $95,000 70,000 45,000 20,000

b. After adjusting entries are prepared and posted on December 31, 2016, Accumulated Depreciation will be understated by $45,000. c.

After adjusting entries, but before closing entries have been prepared and posted on December 31, 2016, Retained Earnings will be understated by $70,000.

d. After both adjusting and closing entries have been prepared and posted on December 31, 2016, Retained Earnings will be understated by $45,000.

E9–14 a. Cash (+A) .............................................................................................. Accumulated Depreciation—Office Equipment (+A) ................................ Office Equipment (–A) ........................................................................ Gain on Sale of Fixed Assets (Ga, +SE) ................................................ Sold office equipment.

235,000 300,000

b. Cash (+A) ................................................................................................ Accumulated Depreciation—Office Equipment (+A) ................................ Loss on Sale of Fixed Assets (Lo, –SE)........................................................ Office Equipment (–A) ........................................................................ Sold office equipment.

185,000 300,000 15,000

500,000 35,000

500,000


E9–15 Assuming that Paris Company kept the equipment for its entire five-year estimated useful life, the depreciation schedule on the equipment would be as follows.

Date

Depreciation Factor

Depreciation Expense

Cost

Accumulated Depreciation

Book Value

1/1/12 $25,000 $ 0 $25,000 12/31/12 40% $10,000 25,000 10,000 15,000 12/31/13 40% 6,000 25,000 16,000 9,000 12/31/14 40% 3,600 25,000 19,600 5,400 12/31/15 40% 400* 25,000 20,000 5,000 12/31/16 40% 0 25,000 20,000 5,000 __________________ * Because the equipment's book value cannot drop below its estimated salvage value, depreciation expense for 2015 cannot exceed $400. a. Accumulated Depreciation—Equipment (+A) ........................................... Loss on Disposal of Equipment (Lo, –SE) ................................................... Equipment (–A)................................................................................... Disposed of equipment.

19,600 5,400

b. Accumulated Depreciation—Equipment (+A) ........................................... Loss on Disposal of Equipment (Lo, –SE) ................................................... Equipment (-A) ................................................................................... Disposed of equipment.

20,000 5,000

c.

Cash (+A) .............................................................................................. Accumulated Depreciation—Equipment (+A) ........................................... Equipment (–A)................................................................................... Gain on Sale of Fixed Assets (Ga, +SE) ................................................ Sold equipment.

d. Fixed Asset (new) (+A) ............................................................................... Accumulated Depreciation—Equipment (+A) ........................................... Loss on Disposal of Fixed Asset (Lo, –SE) .................................................. Cash (–A)............................................................................................. Equipment (old) (–A) .......................................................................... Exchanged fixed assets.

25,000

25,000

8,000 19,600 25,000 2,600

30,000 20,000 3,000 28,000 25,000


E9–16 a. and b.

First, let us compute the original cost of the equipment that was sold in 2014 as follows:

Equipment at the End of 2013

+

$32,700

+

Equipment Purchased during 2014

Equipment sold during 2014

$12,000 X

– =

X $7,200

=

Equipment at the End of 2014

=

$37,500

Now, let us compute the related accumulated depreciation for the equipment sold during 2014 as follows: Accumulated Depreciation at the End of 2013

+

Depreciation Exp. for 2014

$14,300

+

$7,200

Accumulated Depreciation for the Sold Equipment during 2014 X X

=

= =

Accumulated Depreciation at the End of 2014

$17,600 $ 3,900

Now, we can reconstruct the journal entry. Cash ........................................................................................................... Accumulated Depreciation ........................................................................ Equipment .......................................................................................... Gain on Sale of Equipment ................................................................. ____________ * $7,200 + $2,100 – $3,900 = $5,400

E9–17 Account a. Property, plant & equipment Less: accumulated depreciation Depreciation expense Investments in property, plant & equipment

Financial Statement Balance Sheet Balance Sheet Income Statement Statement of Cash Flows

b. Property, plant & equipment – 2011 Plus: investments in property, plant & equipment Less: property, plant & equipment – 2012 Property, plant & equipment sold in 2012

$58,073 11,027 66,046 $ 3,054

c.

$34,446 6,357 38,063 $ 2,740

Accumulated depreciation – 2011 Plus: depreciation expense – 2012 Less: accumulated depreciation – 2012 Accumulated depreciation – sold property

5,400* 3,900 7,200 2,100


E9–17 Concluded d. Compute the gain on the sale: Cost of property sold $3,054 Less: accumulated depreciation 2,740 Book value of property sold $ 314 Sales price of property Less: book value of property Loss on sale of property

$100 314 $214

This loss on sale of property would appear on the income statement.

E9–18 a. First, let us compute the related accumulated depreciation for the equipment sold during 2014 as follows: Accumulated Depreciation at the End of 2013

+

Depreciation Cap. for 2014

$9,800

+

$3,800

Accumulated Depreciation for the Sold Equipment during 2014 X X

=

= =

Accumulated Depreciation at the End of 2014

$10,500 $ 3,100

Now, we can reconstruct the journal entry. Cash ........................................................................................................... Loss on Sale of Equipment ........................................................................ Accumulated Depreciation ........................................................................ Equipment ..........................................................................................

b.

Equipment at the End of 2013

+

$23,400

+

Equipment Purchased during 2014

Equipment sold during 2014

X X

– =

$8,300 $11,800

____________ Equipment purchased during 2014 = $11,800

4,300 900 3,100 8,300

=

Equipment at the End of 2014

=

$26,900


E9–19 a. Swift Corporation should capitalize these costs. Assets are defined as items that are expected to provide future economic benefits to the entity. Organization costs are costs incurred by an entity prior to starting operations. Such costs include legal fees to incorporate and accountant's fees to set up an accounting system. Without incurring these costs, most companies could not be in business. Consequently, organization costs allow a company to be in business, thereby helping it to generate future benefits. Since these costs help in generating future benefits, they should most definitely be capitalized. b. Theoretically, organization costs should be amortized over their useful life. In the extreme, organization costs provide a benefit over the entire life of a company. Since under the going concern assumption accountants assume that entities will exist indefinitely, it would seem that organization costs should be amortized over an indefinite period. Since this position is not practical, the accounting profession has decided that organization costs should be amortized over a period not to exceed forty years. Assuming that Swift Corporation amortizes its organization costs over the maximum period of forty years, the appropriate adjusting journal entry for a single year would be as follows: Amortization Expense (E, –SE) .................................................................. Organization Costs (–A) ...................................................................... Amortized organization costs. c.

1,125 1,125

As mentioned in part (b), organization costs theoretically provide benefits over the entire life of the company. Under the going concern assumption, the company is assumed to exist indefinitely. If the company is assumed to exist indefinitely and if organization costs provide benefits over the entire life of the company, then these costs should provide an indefinite benefit. Consequently, organization costs should provide a benefit for an indefinite period of time, which implies that they should be reported as an asset (i.e., future benefit) indefinitely. But if organization costs are amortized, the asset will at some point in time have a zero balance, and the cost of the asset cannot be matched against the benefits the asset will help generate in the future. This situation contradicts the matching principle and the concept of an asset.

d. A patent gives a company the exclusive right to use or market a particular product or process, thereby providing the company with an expected future benefit. Consequently, the costs incurred to acquire a patent should be capitalized as an asset and amortized over the patent's useful life. If Swift were to immediately expense the $65,000, the company would be implying that it did not expect to receive any benefits from the patent in the future. If this were the case, one would have to question why Swift purchased the patent in the first place. e. Research and development costs may or may not provide a company with future benefits. The company will not know whether or not a particular R & D expenditure will provide a future benefit until some time in the future. Due to the uncertainty of projecting the usefulness of a given R & D expenditure, the FASB, in Statement of Financial Accounting Standards No. 2, "Accounting for Research and Development Costs," requires companies to expense R & D costs in the year in which they are incurred.


E9–19 f.

Concluded

Engaging in research and development activities can lead companies to develop new products or processes that will provide them with future benefits. In such cases, the R & D costs should, theoretically, be capitalized. The R & D costs would then be allocated to those periods in which the costs help generate a benefit. From a practical standpoint, however, this matching of costs with the associated benefits is not readily possible. For example, consider a company that spends $10,000,000 trying to develop a more efficient manufacturing process. The company's attempts end in failure, but the company acquires some new technology from its R & D activities that permit it to develop a revolutionary new product ten years later. In this case, it is clear that the $10,000,000 eventually provided a future benefit. But this information is available only with hindsight. At the time the $10,000,000 was expended, all the company knew was that the R & D project was a failure. So, while capitalizing R & D costs and then amortizing the costs over their useful lives is theoretically superior to immediately expensing the R & D costs, immediately expensing R & D costs is extremely practical and lessens a manager's ability to manipulate the financial statements.

E9–20 a. (1)

Southern Robotics should report the costs incurred in acquiring the patent as an asset. Therefore, the $50,000 of legal and filing fees should be capitalized as an asset in 2014. Since it is company policy not to amortize intangible assets in the year of acquisition, the company would report the entire $50,000 as an asset as of December 31, 2014.

(2)

Since Southern Robotics successfully defended its patent, the patent is still expected to provide a future benefit to the company. Hence, the company should continue to carry the patent on its books as an asset. The amount it should report for the patent as of December 31, 2015 should be the cost of acquiring and defending the patent less the portion of these costs that have been amortized. Therefore, Southern Robotics should report $200,000 on its balance sheet (i.e., $50,000 in legal and filing fees incurred in 2014 + $200,000 in legal fees incurred in 2015 to defend the patent – $50,000 in amortization).

(3)

Amortization Expense (E, –SE)............................................................ Patent (–A) ................................................................................... Amortized patent.

50,000 50,000

b. (1)

Since the lawsuit did not take place until 2015, the patent still had value to Southern Robotics as of December 31, 2014. Therefore, the company should still report the patent at $50,000 on its books as of December 31, 2014. However, if Southern Robotics was aware of the lawsuit as of December 31, 2014, it might want to disclose the lawsuit and the potential effect on the company's financial statements in a footnote as a contingency.

(2)

Since Southern Robotics was unsuccessful in defending its patent, the company no longer has the exclusive right to use or market its robotics arm. Therefore, the patent no longer provides the company with any future benefits. Since the patent no longer provides any future benefits, it should be written off in 2015.

(3)

Loss on Patent (Lo, –SE)...................................................................... Legal Expenses (E, –SE) .......................................................................

50,000 200,000


Patent (–A) ................................................................................... Cash (–A) ...................................................................................... Incurred legal fees for patent defense and wrote off patent.

50,000 200,000

E9–21 a. The journal entry for the acquisition can be derived from the information provided and appears below: Assets (+A) (at fair market value) ......................................................... 5.1 Goodwill (+A) 3.8 Liabilities (+L) ...................................................................................... Cash (-A) ........................................................................................ 7.8

1.1

Goodwill represents the excess of the purchase price above the fair market value of the assets purchased. b. Assets increased by a net $1.1 billion ($5.1 + 3.8 – 7.8) and liabilities increased by $1.1 billion.

E9–22 a. Under US GAAP, long-lived assets must be carried at original cost less accumulated depreciation (amortization); if the market value of the asset permanently falls below the balance sheet carrying value, an impairment charge must be recorded, and cannot be reversed in later periods if the value of the asset recovers. Under IFRS, companies can either follow the US GAAP method, or they can periodically revalue their long-lived assets to fair market value – recognizing not only impairments, but also increases and recoveries of asset values. b. EADS has accounted for this asset according to U.S. GAAP methods. c. If EADS were to carry the asset at its Fair Market Value (using the IFRS approach), the company would increase the value of the asset by 7 million euros and would record a gain on its income statement of the same amount.


PROBLEMS P9–1 a. Stonebrecker should capitalize all costs that it incurred that were necessary and reasonable to get the equipment in a serviceable condition and location. The capitalizable costs are (1) the $1,000,000 purchase price, (2) the $40,000 transportation costs actually incurred by Stonebrecker, (3) the $8,000 insurance coverage, (4) the $20,000 installation fees, (5) the $15,000 to reinforce the floor, and (6) the $10,000 of employee downtime. Some accountants may disagree with capitalizing the last two items as part of the equipment. However, theoretically, these costs are necessary to get the equipment in a usable condition. Therefore, the total dollar amount that should be capitalized for the equipment is $1,093,000. b. Equipment (+A) ......................................................................................... Cash (–A)............................................................................................. Purchased equipment. c.

1,093,000 1,093,000

The depreciation base represents the capitalized cost of a fixed asset that the company does not expect to recover over the asset's estimated useful life. Since the capitalized cost of the equipment is $1,093,000 and the company expects to sell the equipment for $100,000 after ten years, the company does not expect to recover $993,000 of the capitalized cost. Therefore, the depreciation base of the equipment is $993,000.

d. As discussed in part [c], the depreciation base represents the dollar amount of a fixed asset that the company does not expect to recover from the asset at the end of the asset's estimated useful life. This implies that the depreciation base represents the dollar amount of a fixed asset that the company expects to consume over the asset's estimated useful life. Since the consumption of an asset is an outflow of that asset and since, by definition, outflows of assets are expenses, the depreciation base represents the amount that should be expensed over a fixed asset's useful life. This is true whether the company uses the straight-line method or the double-declining-balance method. Thus, every depreciation method will result in the same total amount being depreciated over a fixed asset's useful life. Although each method gives rise to the same total amount of depreciation, the timing of depreciation charges varies across depreciation methods. The straight-line method allocates depreciation evenly across time, while the double-declining-balance method allocates the depreciation base more rapidly to the early years of the asset's useful life and more slowly to the later years of the asset's useful life. Thus, Stonebrecker will depreciate a total of $993,000 under both depreciation methods.


P9–2 a. 1/1/14 FMV

Relative FMV

300,000 150,000 75,000 75,000 600,000 $ 1,200,000

300/1,200 150/1,200 75/1,200 75/1,200 600/1,200 1,200/1,200

Asset Building Office equip. Crane 1 Crane 2 Land Total

$

Purchase Price $1,000,000 1,000,000 1,000,000 1,000,000 1,000,000

=

Cost Allocation $

250,000 125,000 62,500 62,500 500,000 $ 1,000,000

Building (+A) ............................................................................................. Office Equipment (+A) ............................................................................... Cranes (+A) ............................................................................................. Land (+A) .............................................................................................. Cash (–A)............................................................................................. Purchased basket of assets.

250,000 125,000 125,000 500,000 1,000,000

b. Depreciation Expense—Building (E, –SE) .................................................. Depreciation Expense—Office Equipment (E, –SE) ................................... Depreciation Expense—Cranes (E, –SE) .................................................... Accumulated Depreciation—Building (–A) ......................................... Accumulated Depreciation—Office Equipment (–A) ......................... Accumulated Depreciation—Cranes (–A) ........................................... Depreciated fixed assets. ____________ a $8,750 = ($250,000 – $75,000) ÷ 20 years b $30,000 = ($125,000 – $35,000) ÷ 3 years c $19,000 = [$125,000 – ($15,000 + $15,000)] ÷ 5 years

8,750a 30,000b 19,000c

c.

Property, plant, and equipment: Land ........................................................................................................... Building...................................................................................................... Office equipment ...................................................................................... Cranes........................................................................................................ Less: Accumulated depreciation ............................................................... Total property, plant, and equipment....................................................... ____________ * $201,000 = ($8,750  4 years) + ($30,000  3 years) + ($19,000  4 years)

8,750 30,000 19,000

$500,000 250,000 125,000 125,000 (201,000)* $799,000


P9–3 a. Cost = Purchase Price + Transportation + Installation = $950,000 + $100,000 + $130,000 = $1,180,000 b. (1)

Double-declining-balance method: Depreciation Expense—Equipment (E, –SE) ................................... Accumulated Depreciation—Equipment (–A) .......................... Depreciated fixed asset. _____________ *$590,000 = $1,180,000  50%

(2)

282,500* 282,500

Double-declining-balance method: Cash (+A) ......................................................................................... Accumulated Depreciation: Equipment (+A) .................................. Loss on Sale of Equipment (Lo, –SE) ............................................... Equipment (–A) ........................................................................ Sold equipment.

(2)

590,000

Straight-line method: Depreciation Expense—Equipment (E, –SE) ................................... Accumulated Depreciation—Equipment (–A) .......................... Depreciated fixed asset. _____________ *$282,500 = ($1,180,000 – $50,000) ÷ 4 years

c. (1)

590,000*

250,000 590,000 340,000 1,180,000

Straight-line method: Cash (+A) ......................................................................................... Accumulated Depreciation: Equipment (+A) .................................. Loss on Sale of Equipment (Lo, –SE) ............................................... Equipment (–A) ........................................................................ Sold equipment.

250,000 282,500 647,500 1,180,000


P9–4 a.

Truck (+A) ................................................................................................ Cash (–A)............................................................................................. Purchased a truck.

48,000 48,000

b. Depreciation Per Books 2012 $48,000 2013 0 _____________ *$12,000 = ($48,000 – $12,000) ÷ 3 years

Correct Depreciation $

0 12,000*

Difference $48,000 12,000

Therefore, in 2012 expenses were overstated by $48,000, so net income was understated by $48,000. In 2013 expenses were understated by $12,000, so net income was overstated by $12,000. c. Depreciation Per Books 2012 $48,000 2013 0 _____________ *$32,000 = $48,000  2/3

Correct Depreciation $

0 32,000*

Difference $48,000 32,000

Therefore, in 2012 expenses were overstated by $48,000, so net income was understated by $48,000. In 2013 expenses were understated by $32,000, so net income was overstated by $32,000.


P9–5 a. Dryer (+A) .................................................................................................. Cash (–A)............................................................................................. Purchased a dryer.

100,000 100,000

b. In deciding how to account for service and repair costs, one must consider the effect of the cost on (1) the useful life of the asset, (2) the quality of units produced by the asset, (3) the quantity of units produced by the asset, or (4) the cost of operating the asset. If the costs increase one of the first three items or reduce the last item, they provide a future benefit to the company. Consequently, the costs should be capitalized and amortized over the asset's useful life. If the costs do not increase one of the above items, they do not provide a future benefit, and they should be expensed immediately. In this particular case, the $160,000 overhaul increased both the dryer's efficiency and useful life. Consequently, the $160,000 should be capitalized as follows: Dryer (+A) .................................................................................................. Cash (–A)............................................................................................. Overhauled dryer.

160,000 160,000

The annual service cost of $1,000 and the major repair cost of $5,000 are incurred simply to maintain the dryer's existing service potential and, therefore, do not provide expected future benefits. Consequently, these costs should be expensed as incurred. c.

2011 through 2014: Depreciation Expense

2015 through 2018: Depreciation Expense

= =

(Cost – Salvage Value) ÷ Useful Life ($100,000 – $10,000) ÷ 5 Years = $18,000 per Year

= {[(Cost – Accumulated Depreciation) + Betterments] – Salvage Value} ÷ Remaining Useful Life. = {[($100,000 – $72,000) + $160,000] – $10,000} ÷ 4 Years = $44,500 per Year

P9–6 a. Building (+A) .............................................................................................. Cash (–A)............................................................................................. Purchased a building.

1,500,000

b. Building (+A) .............................................................................................. Cash (–A)............................................................................................. Installed a new roof on building.

200,000

c.

1,500,000

Depreciation Expense (E, –SE) ................................................................... 50,500* Accumulated Depreciation (–A) ......................................................... Depreciated building. ______________ *$50,500 = {[($1,500,000 – $540,000 Accumulated Depreciation) + $200,000] – $150,000 Salvage Value} ÷ Remaining Life of 20 Years.

200,000

50,500


P9–6

Concluded

d. Cash (+A) ................................................................................................... 1,200,000 Accumulated Depreciation (+A) ................................................................ 843,000* Building (–A) ....................................................................................... 1,700,000 Gain on Sale of Building (Ga, +SE) ...................................................... 343,000 Sold building. ______________ *$843,000 = ($54,000  10 years for 2010 through 2019) + ($50,500  6 years for 2020 through 2025) Note:

The above entry assumes that the adjusting entry to record depreciation expense for 2025 had already been recorded. If this entry had not yet been made, the appropriate entry to record the sale would have been as follows:

Cash (+A) ................................................................................................... Accumulated Depreciation (+A) ................................................................ Depreciation Expense (E, –SE) ................................................................... Building (–A) ....................................................................................... Gain on Sale of Building (Ga, +SE) ...................................................... Sold building.

1,200,000 792,500 50,500 1,700,000 343,000

P9–7 a. Depreciation Expense per Year

1/1/14 Accumulated Depreciation

1/1/14 Book Value

= ($180,000 – $30,000) ÷ 10 Years = $15,000 per Year = $15,000 Depreciation Expense per Year  5 Years = $75,000

= Cost – 1/1/14 Accumulated Depreciation = $180,000 – $75,000 = $105,000

b. Depreciation Expense (E, –SE) ................................................................... 9,375 Accumulated Depreciation (–A) ......................................................... Depreciated fixed assets. ______________ *$9,375 = (Book Value of $105,000 – Salvage Value of $30,000) ÷ Remaining Useful Life of 8 Years

9,375

P9–8 When the hand-held instruments were capitalized as a prepaid expense, they were carried as Current Assets and converted to an expense as used. After the accounting change, the asset was carried as a long-lived asset and converted to an expense through a depreciation charge that spread the cost over a five-year period, lowering the amount charged against earnings when compared to the previous method. The asset will remain on the books for a longer period of time (assets and equity are higher) and income will also be higher due to the lower expense. The current ratio is lowered, because the assets are moved to the noncurrent section of the balance sheet. Finally, the fixed asset turnover ratio,


which measures sales to fixed assets, will be lower as the dollar amount assigned to fixed assets is now greater.

P9–9 a. Every depreciation method depreciates the same amount over the useful life of a fixed asset. Depreciation methods only vary the timing of depreciation charges. Therefore, both the straightline method and the double-declining-balance method will give rise to the same total amount of depreciation over the four-year useful life of this equipment. The following table shows that the total depreciation under the two methods is the same. Method

Year 1

Year 2

Year 3

Year 4

Straight-linea $15,000 $15,000 $15,000 $15,000 b Double-declining-balance 40,000 20,000 0 0 ______________ a $15,000 = ($80,000 – $20,000) ÷ 4 years b Depreciation Depreciation Historical Accumulated Date Factor Expense Cost Depreciation 1/1/14 12/31/14 12/31/15 12/31/16 12/31/17

50% 50% 50% 50%

$40,000 20,000 0 0

$80,000 80,000 80,000 80,000 80,000

$

0 40,000 60,000 60,000 60,000

Total $60,000 60,000

Book Value $80,000 40,000 20,000 20,000 20,000

b. Since, as demonstrated in part (a), both depreciation methods give rise to the same total amount of depreciation over the fixed asset’s life, the total amount of net income over the asset’s life must also be the same. Therefore, the total amount of taxes will be the same regardless of which depreciation method a company selects. The following shows that the total amount of net income and taxes are the same under the two methods. Straight-line method: Method Revenues Depreciation exp. Other expenses Pretax income Income taxes Net income

Year 1

Year 2

Year 3

Year 4

Total

$ 100,000 15,000 60,000 $ 25,000 8,750 $ 16,250

$ 100,000 15,000 60,000 $ 25,000 8,750 $ 16,250

$ 100,000 15,000 60,000 $ 25,000 8,750 $ 16,250

$ 100,000 15,000 60,000 $ 25,000 8,750 $ 16,250

$ 400,000 60,000 240,000 $ 100,000 35,000 $ 65,000

Year 1

Year 2

Year 3

Year 4

Total

$ 100,000 40,000 60,000 $ 0 0 $ 0

$ 100,000 20,000 60,000 $ 20,000 7,000 $ 13,000

$ 100,000 0 60,000 $ 40,000 14,000 $ 26,000

$ 100,000 0 60,000 $ 40,000 14,000 $ 26,000

$ 400,000 60,000 240,000 $ 100,000 35,000 $ 65,000

Double-declining-balance: Method Revenues Depreciation exp. Other expenses Pretax income Income taxes Net income c.

The double-declining-balance method is preferred for tax purposes because this method defers tax payments. Under this depreciation method, more depreciation is taken in the early years of an


asset’s life than in later years. Increasing depreciation in an asset’s early life reduces taxable income which, in turn, reduces income taxes in the early years of the asset’s life [see part b]. The reduction in income taxes in the early years of the asset’s life is offset by higher taxes in the later years of an asset’s life. However, due to the time value of money, deferring taxes is beneficial. d. Straight-line method: Present Value = $8,750 from part (b)  Present Value of an Ordinary Annuity Factor for i = 10% and n = 4 = $8,750  3.16987 (from Table 5) = $27,736.36 Double-declining-balance method: Present Value = ($7,000  Present Value Factor for i = 10% and n = 2) + ($14,000  Present Value Factor for i = 10% and n = 3) + ($14,000  Present Value for i = 10% and n = 4) = ($7,000  0.82645) + ($14,000  0.75131) + ($14,000  0.68301) = $5,785.15 + $10,518.34 + $9,562.14 = $25,865.63

Factor

In present value terms, Kimberly Sisters would save $1,870.73 ($27,736.36 – $25,865.63) in taxes on this one asset by selecting the double-declining-balance method over the straight-line method.

P9–10 (a) S-L Depreciation (10-year life)

(b) DDB Depreciation

(c) S-L Depreciation (5-year life)

$ 250,000 (80,000)b (140,000) $ 30,000 (9,600) $ 20,400

$ 250,000 (80,000)c (140,000) $ 30,000 (9,600) $ 20,400

47,600 8% 3,808

$

20,400 8% 1,632

$

47,600 75% 35,700

$

20,400 75% 15,300

$

Tax Payments: Revenues $ 250,000 Depreciation expense (40,000)a Other expenses (140,000) Net income before taxes $ 70,000 Income taxes (22,400) Net income $ 47,600 ________________ a $40,000 = ($400,000 – 0) ÷ 10 years b $80,000 = ($400,000  20%) c $80,000 = ($400,000 – 0) ÷ 5 years Bonus Payment: Net income Bonus percentage Bonus amount Dividend Payment Net income Dividend percentage Dividend amount

$ $

$ $

$

$

$

$

20,400 8% 1,632

20,400 75% 15,300


P9–11 a. Drilling Equipment (+A) ............................................................................. Mobile Home (+A) ..................................................................................... Cash (–A)............................................................................................. Purchased assets for drilling fields. b. 2014: Depletion (E, –SE) ...................................................................................... Drilling Equipment (or Accumulated Depletion) (–A)......................... Depleted drilling equipment. ____________ *$240,000 = ($800,000 ÷ 2,000,000 barrels)  600,000 barrels 2015: Depletion (E, –SE) ...................................................................................... Drilling Equipment (or Accumulated Depletion) (–A)......................... Depleted drilling equipment. ____________ *$300,000 = ($800,000 ÷ 2,000,000 barrels)  750,000 barrels 2016: Depletion (E, –SE) ...................................................................................... Drilling Equipment (or Accumulated Depletion) (–A)......................... Depleted drilling equipment. ____________ *$260,000 = ($800,000 ÷ 2,000,000 barrels)  650,000 barrels

800,000 54,000 854,000

240,000* 240,000

300,000* 300,000

260,000* 260,000

c. 2014: Depreciation Expense (E, –SE) ................................................................... Accumulated Depreciation (–A) ......................................................... Depreciated mobile home. 2015: Depreciation Expense (E, –SE) ................................................................... Accumulated Depreciation (–A) ......................................................... Depreciated mobile home. 2016: Depreciation Expense (E, –SE) ................................................................... Accumulated Depreciation (–A) ......................................................... Depreciated mobile home. ____________ *$7,000 = ($54,000 – $5,000) ÷ 7 years

7,000* 7,000

7,000* 7,000

7,000* 7,000


P9–11

Concluded

Different methods are used to allocate the costs of the drilling equipment and the mobile home based upon the link between the asset and the oil field. The drilling equipment is site-specific. Hence, its useful life is identical to the productive life of the oil field. Under the matching principle, the activity method provides the best matching of the costs with the associated benefits. On the other hand, the mobile home is not site-specific; it has a useful life beyond this oil field. The activity method would not be appropriate for the mobile home because the productive capabilities of future oil fields on which the mobile home may be used are not yet known. Consequently, Garmen Oil Company must select either the straight-line method or an accelerated method to depreciate the mobile home. d. Depletion: 2014: Depletion (E, –SE) ...................................................................................... Drilling Equipment (or Accumulated Depletion) (–A)......................... Depleted drilling equipment.

240,000 240,000

2015: Depletion (E, –SE) ...................................................................................... 300,000 Loss on Oil Field (Lo, –SE) .......................................................................... 260,000* Drilling Equipment (or Accumulated Depletion) (–A)......................... 560,000 Depleted drilling equipment. ____________ * Since the well is dry, the drilling equipment will not provide any future benefits; hence, the remaining cost of $260,000 [($800,000 – ($240,000 + $300,000)] should be written off. 2016: No journal entries are necessary. Depreciation: Since the mobile home is not site-specific, the entries for depreciation would be the same as in part (c).


P9–12 a. Cash (+A) ................................................................................................... Accumulated Depreciation (+A) ................................................................ Machinery (–A) ................................................................................... Gain on Sale of Machinery (Ga, +SE) .................................................. Sold machinery. ____________ *$240,000 = [($500,000 – $100,000) ÷ 5 years]  3 years used

325,000 240,000*

b. Depreciation Expense (E, –SE) ................................................................... Accumulated Depreciation (–A) ......................................................... Depreciated machinery for January 1 – June 30. ____________ *$40,000 = [($500,000 – $100,000) ÷ 5 years]  1/2 year

40,000*

Cash (+A) ................................................................................................... Accumulated Depreciation (+A) ................................................................ Machinery (–A) ................................................................................... Gain on Sale of Machinery (Ga, +SE) .................................................. Sold machinery.

320,000 280,000

c.

FMV of asset received: Land (+A) ................................................................................................... Accumulated Depreciation (+A) ................................................................ Loss on Trade-in (Lo, –SE).......................................................................... Machinery (–A) ................................................................................... Cash (–A)............................................................................................. Exchanged machinery for land. FMV of assets given up: Land (+A) ................................................................................................... Accumulated Depreciation (+A) ................................................................ Loss on Trade-in (Lo, –SE).......................................................................... Machinery (–A) ................................................................................... Cash (–A)............................................................................................. Exchanged machinery for land. ____________ *$250,000 = Cash given up + Appraised value of machinery given up

500,000 65,000

40,000

500,000 100,000

210,000 240,000 225,000 500,000 175,000

250,000* 240,000 185,000 500,000 175,000


P9–13 a. Most assets are reported on the balance sheet at historical cost or at historical cost less accumulated depreciation. The historical cost of a particular asset is constant over time. However, the fair market value of that same asset fluctuates over time. Consequently, the fair market value of assets can be less than, equal to, or greater than the historical cost of the assets at any point in time. b. Diversified would pay more for Specialists due to goodwill (i.e., synergy). Specialists' assets considered as a package are worth more than the sum of their individual values. Goodwill arises because certain "assets" are not included on a company's balance sheet. Items that cannot be given a value (i.e., cannot be quantified) are omitted from a balance sheet. Examples include customer loyalty and the company's name recognition. c.

Assets (+A) ................................................................................................. Goodwill (+A) ............................................................................................. Liabilities (+L) ...................................................................................... Cash (–A)............................................................................................. Purchased Specialists, Inc.

1,350,000 700,000 250,000 1,800,000

d. Until recently under GAAP, goodwill was capitalized at the time of acquisition and then amortized over a maximum of 40 years. The school of thought holding the opposite viewpoint espouses that goodwill should be expensed at the time of acquisition. They maintain that since goodwill is a plug number on the books of the acquired company and its amortization period is totally arbitrary, it need not be put on the balance sheet. Further, goodwill should be periodically tested to see if it has been “impaired” (i.e., if the fair value of the assets acquired has dropped).

P9–14 a. Goodwill would be calculated by taking the purchase price less the fair value of the net assets. In this case $3.4 billion was paid and the fair value of the net assets was $1.3 billion ($2.3 less $1.0). Therefore, Goodwill is $2.1 billion. b. There may be a number of reasons why Zimmer paid over the fair value of Centerplus. There may be assets that are not recorded on Centerplus’s books. This usually would be the value of the brand name or a high quality workforce. Another reason could be that Zimmer foresees that there will be significant synergy between the two companies. It may be that these additional locations will have a significant competitive benefit to Zimmer. Another possible reason would be to block another competitor from buying Centerplus. Other reasons could be that Zimmer management just want to make Zimmer a bigger company (management compensation is sometimes based on the size of the company), or it could be that Zimmer overpaid.


ISSUES FOR DISCUSSION ID9–1 a. Gains and losses resulting from the disposal of fixed assets are based on the difference between the proceeds received from the disposal and the asset's book value. Thus, one would have to know the book value of each individual casino and the land to be able to determine the gain or loss from selling one of the casinos. Book value equals the original cost of the fixed asset less any accumulated depreciation associated with the fixed asset. Since MGM Grand, Inc., held the casino it sold for just a short time, the casino's book value is essentially identical to its cost. Thus, in this case, one would need to know the individual costs of the two casinos and the land. When assets are purchased in a group for one purchase price, as they were in this case, a common method to determine the purchase price for each individual asset acquired is to use the assets' relative fair market values. The fair market values of the individual assets would often be determined by an independent appraiser. b. Cash (+A) ................................................................................................... Casino (–A).......................................................................................... Land (–A)............................................................................................. Gain on Sale (Ga, +SE) ........................................................................ Sold casino and land. c.

110,000,000 75,000,000 17,000,000 18,000,000

For the purchasing company, it would be necessary to allocate the total $110 million cost to the casino and land. This could be done based on an appraised fair market value. The land without the casino would be appraised first; the difference between the $110 million and the appraisal would be the value of the casino.

d. Cost of hotel: $110,000,000 – $43,000,000 = $67,000,000 Depreciation per year: $67,000,000 ÷ 25 years = $2,680,000 per year

ID9–2 One of the underlying goals of an accounting system is to properly match revenues with expenses. There are many marketing costs that will help to produce revenue for the company over multiple periods. If the company expenses all of these marketing expenses in the first year, then net income for the first year will be understated and then overstated in future years when the revenue produced is not matched with the marketing expenses incurred to generate it. At the same time it is very difficult to determine a rational way to allocate marketing costs to the revenue that it produces. Management, separate from any desires to influence the stock price, will generally want to match its marketing expenses with the revenues that these expenses produce. Management wants to be able to evaluate the impact of its marketing efforts. Shareholders may want to see a system of charging marketing expenses that will have the best impact on stock performance. In the early years of a company this may mean capitalizing heavy marketing from the early years and defering the expense until the company has higher levels of revenue to absorb these expenses. Auditors want to make sure that marketing expenses are handled consistently and in a manner that fairly represents the true economic value of these


ID9–2

Concluded

expenditures. Auditors also tend to be conservative when there is uncertainty as to the future value of an asset. Will these marketing costs from this year truly have value in future years? Since this is a subjective estimate, auditors may want to expense all marketing expenses in the year incurred.

ID9–3 a. The main issue to be considered is whether the capital expenditure is a betterment or simply maintenance. To be considered a betterment, the expenditure must (1) increase the asset's useful life, (2) increase the quality of the asset's output, (3) increase the quantity of the asset's output, or (4) reduce the cost associated with operating the asset. If the expenditure meets one of these criteria, the expenditure should be capitalized. Otherwise, the expenditure should be expensed. b. The amount may be immaterial.

c.

Depreciation per year represents the remaining net cost of an asset allocated over the asset's estimated remaining useful life. In this particular case, the remaining net cost equals the sum of the asset's book value at the time of refurbishment and the cost of the refurbishment less the estimated salvage value of the plant. This amount would be depreciated over the estimated useful life of the "new" plant.

ID9–4 a. EADS is expensing a portion of its research and development costs, but the company is not expensing the entire amount. The portion not expensed is capitalized on the balance sheet as an asset (Capitalized Development Costs) and amortized as an expense in future periods. b. Under U.S. GAAP, the entire amount would be expensed.

c.

The R & D expense under US GAAP would have been 3,142 million euros plus 572 million euros.

d. In a comparison of earnings IFRS vs. U.S. GAAP for EADS, the IFRS net income overstates earnings by the 572 million euros that were not expensed (but were instead capitalized to the balance sheet). Had the company used U.S. GAAP, total expenses would have been 572 million euros higher.

ID9–5


One of the underlying goals of an accounting system is to properly match revenues with expenses. There are many advertising and research & development costs that will help to produce revenue for the company over multiple periods. If the company expenses all of these expenses in the first year, then net income for the first year will be understated and then overstated in future years when the revenue produced is not matched with the expenses incurred to generate it. At the same time it is very difficult to determine a rational way to allocate these costs to the revenue that it produces. When these costs are incurred it is extremely difficult to know the revenue, if any, that will be produced in future periods. The capitalization of software development costs has the opposite effect. Expenses that are incurred in the current year will not impact the income statement this year but will in future years. The predictability of the future value of software is higher than for advertising and research and development costs. Management, separate from any desires to influence the stock price, will generally want to match its expenses with the revenues that these expenses produce. Management wants to be able to evaluate the impact of both its advertising and research & development costs. Shareholders may want to see a system of charging marketing expenses that will have the best impact on stock performance. In the early years of a company this may mean capitalizing heavy marketing from the early years and defering the expense until the company has higher levels of revenue to absorb these expenses. Management may also have an incentive to report higher net income, which could cause some managers to want to change accounting policies to work to their benefit. This speaks for the benefit of consistently applying accounting policies from year to year.


ID9–6 a. The effect on profits from increased capital spending will come from increased depreciation charges. Capitalized expenditures for fixed assets will eventually hit the income statement as depreciation expense. b. The balance sheet will reflect growth in the property, plant and equipment, as well future growth in the contra asset accumulated depreciation account. The income statement will show increased depreciation expense. And finally, the statement of cash flow will show greater uses of cash in the investing activities section. c.

The justification of management for the increased capital expenditures will be to remain competitive in the marketplace. If businesses do not reinvest in the long-term assets of their operations, they will not be able to remain competitive. Spending the additional money for fixed assets will affect the financial statements today, but that spending will also keep the business viable into the future. The stockholders have to allow funds to be allocated for investing activities if they want to continue to receive a return on their investment in the company.

ID9–7 a. The most likely scenario causing a restaurant’s value to be impaired is a loss in the desirability of the location. If a McDonald’s restaurant was located at a certain intersection and traffic patterns in the city changed (due to a new interstate highway, for example), the restaurant will no longer be as attractive a location. McDonald’s therefore would have to adjust downward the carrying value for that restaurant. b. McDonald’s will record the impairment by first determining the fair value of the asset. Then, the company will record an impairment expense and reduce the asset from its current carrying value down to the (new) fair value. c. As with other expenses that are somewhat at management’s discretion, shareholders are vulnerable if management decides to take an impairment expense in a year where earnings are otherwise very healthy, eliminating the need to take the expense in future years when earnings are less robust. A management team could lower current earnings (and thus future earnings expectations) by taking impairment charges in current periods.

ID9–8 a. It seems GE is engaging in income-smoothing. Whenever GE has a one-time reporting gain due to any unusual events, it also tries to book a related expense or a charge to offset that gain. This ensures that earnings do not rise so high that they cannot be topped the following year. b. Discretionary restructuring charges are used by GE to offset the one-time gains in order to avoid an abnormal peak in the company’s earnings for the year. It seems that GE’s management strongly believes in modest but consistent earnings growth. Therefore, they engage in below-the-line activities and generate gains for themselves, which they try to offset to a somewhat lesser extent by recording a restructuring charge. GE wants to pursue this strategy because capital markets reward a net increase in earnings with a higher stock price. Therefore, GE hopes to increase the price of its stock. c.

Since the proliferation of total quality movement, restructuring is usually perceived by the investors and the market as a cost-cutting exercise by the company. If the market believes that a company is


becoming leaner and meaner in its operations, it expects the company’s profits to rise in the future. Therefore, in anticipation it rewards the company by increasing its stock prices. FASB is rightfully concerned in limiting such behavior of various companies. The reason is that several times in practice, restructuring charges have had nothing to do with downsizing or rightsizing. No costs are reduced in the future and no real benefits may accrue to the company in the coming years. Since there are no guidelines, all kinds of expenses incurred by the companies are being classified under this broad category of “restructuring charges,” which the market has come to perceive as a favorable charge.

ID9–9 a. The problem with using current costs is trying to determine what the current cost is. That is, how does one determine the current cost of a specialized piece of manufacturing equipment or the current cost of an office building in a slow-moving real estate market? This difficulty in determining current costs gives managers leeway to manipulate the amounts reported in the financial statements. If managers are given the opportunity to manipulate the financial statements through subjective current costs, then financial statement users will be wary of placing any reliance on the financial statements. Thus, current costs could potentially lead to the demise of financial statements. b. Historical costs are sunk costs in that they represent the cost of an asset at the time the asset was acquired; historical costs do not indicate the magnitude of cash or net assets that an asset will generate in the future. Since sunk costs are irrelevant for decision-making purposes, historical costs are not relevant for decision-making purposes. Alternatively, current costs provide a measure of the value of an asset today. For example, the amount reported for Cost of Goods Sold and Depreciation Expense under current cost represents the current values of the inventory sold during the period and of the fixed assets "consumed" during the accounting period, respectively. These values essentially represent what it would cost the company to replace the inventory it sold and the fixed assets it "consumed." In addition, the amounts reported on the balance sheet for inventory and fixed assets essentially represent what it would cost the company to replace its inventory and fixed assets, which is essentially the same value as what the company would realize if it sold the inventory and fixed assets. Thus, current cost information would be very relevant for decision-making purposes because it provides information about cash inflows the company could generate and about cash outflows the company is likely to make. In short, current cost information is very relevant for decision-making purposes. c.

The argument comes down to reliability versus relevancy. Current cost information is more relevant than historical cost information, but it is considerably more difficult to objectively determine current costs than it is to determine historical costs. If individual financial statement users were able to dictate the valuation basis to be used in preparing financial statements, each individual would be able to determine his or her personal decision on the tradeoff between reliability and relevancy. However, financial statements are intended for general use, which means that the same financial statements will be used by a variety of people. Historically, reliability has been given more importance than relevancy because (1) relevant information is not very useful if you are not sure you can rely on the information and (2) managers and auditors are legally liable to financial statement users. That is, they are uneasy about providing information that might be subjective because it could greatly increase their legal exposure.

ID9–10


a. Asset write-downs allow Kellogg to manage earnings by reducing depreciation expenses in future periods. If Kellogg has a good quarter and decides to write down an asset this lowers the book value of the asset and thereby reduces the amount of depreciation expense that will be incurred in future periods. b. The accounting profession in general tends to prefer conservative accounting practices. By carrying the value of assets at a lower value, the auditors reduce their risk that if something goes wrong with the company that they will be sued. So if management makes estimates that reduce the value of assets, the auditors will be less likely to object. c.

The FASB has come out against this policy of “taking a bath” by companies when they have had a really bad quarter to begin with. Some companies will then go ahead and write down assets so that in future periods the amount of depreciation will be reduced thereby improving reported net income. While the write down of assets may be conservative, this approach violates the matching principle. The appropriate costs are not being matched with the related revenues in future periods.

ID9–11 a. The write-off of an outdated technology system would reduce assets and equity; equity is reduced because of the write-off expense, which reduces Retained Earnings through lower profits. b. The most likely factor in determining that a system is overvalued is the introduction of new technology products in the market that better meet the company’s needs. The old system, still carried on the balance sheet, is no longer as valuable because of the technological updates of the new systems. c.

Management could decide to take the write-off expense in a year where earnings are otherwise very healthy, eliminating the need to take the expense in future years when earnings are less robust. A management team could lower current earnings (and thus future earnings expectations) by taking the write-off charge in the current period.

ID9–12 According to U.S. GAAP, long-lived assets are recorded at original cost less accumulated depreciation. If the market value of the asset permanently falls below the balance sheet carrying value, an impairment charge must be recorded, and cannot be reversed in later periods, even if the value of the asset recovers. Under IFRS, companies can either follow the U.S. GAAP method, or they can periodically revalue their long-lived assets to fair market value, recognizing not only impairments but also recoveries of previously impaired assets and increases in asset values. Effectively, U.S. GAAP follows the more conservative “lower-of-costor market” principle, where asset values may be marked down but may never be marked up. IFRS, on the other hand, gives companies the option to value assets according to ever-changing market values, where both market value increases and decreases are recorded. Under U.S. GAAP, development costs must be expensed, while IFRS gives companies the ability, in certain circumstances, to capitalized development costs and amortize those costs over future accounting periods. U.S. GAAP requires immediate expensing, while IFRS can allow costs to be allocated to future periods.


ID9–13 a. Property, plant and equipment make up 12.6% ($11,854/$93,798) of total assets. Other long-lived assets make up 22.9% ($21,490/$93,798) of total assets. b. According to Note 5, Information Technology Assets is the largest category within property, plant and equipment. c.

Depreciation expense (from the Statement of Cash Flow) is 3.96% ($1,988/$50,175) of Net Revenue. Because depreciation is a non-cash expense, it is added back in the Statement of Cash Flow in the calculation of cash from operating activities.

d. According to Note 1, Google depreciates its assets using the straight-line method. The company uses 2 to 5 years for most assets, but up to 25 years for buildings. e. The company’s largest intangible asset is Goodwill. f.

The company evaluates assets for impairment whenever business circumstances or events indicate the carrying value of assets might not be recoverable. If the evaluation determines impairment, the asset is written down to its estimated fair value.

g. According to the statement of cash flow, Google spent $3,273 million on PP & E in 2012.


CHAPTER 10 INTRODUCTION TO LIABILITIES: ECONOMIC CONSEQUENCES, CURRENT LIABILITIES, AND CONTINGENCIES BRIEF EXERCISES BE10–1 a. Dividends declared during a year and the actual cash paid for dividends during the year may be different because dividends declared includes dividends that are accrued but not paid as of year end. The cash paid for dividends will include all payments that relate to dividends during the year, regardless of when those dividends were recognized in the financial statements. b. Dividend payable is a short term liability because it represents dividends that will have to be paid within the next 12 months as of the balance sheet date. c.

Dividend payable – 12/31/11 + Dividends declared – 2012 – Dividends paid during 2012 Dividend payable – 12/31/12

$ 1,281 million 5,173 million 5,116million $ 1,338 million

BE10–2 a. Calculation of inventory purchases during 2012: Inventory @ 12/31/11 $ 7,918 million + Purchases - 2012 x – Inventory @ 12/31/12 7,903 million Cost of Goods Sold – 2012 $ 50,568 million x = $50,553 million ............................................................................. Inventory (+A) .................................................................................................. Accounts Payable (+L)......................................................................... Record inventory purchases made during 2012. b. Calculation of payments made to suppliers during 2012: Accounts Payable @ 12/31/2011 + 2012 Inventory Purchases – Accounts Payable @ 12/31/2012 Payments made to suppliers during 2012

50,553

$ 6,857 million 50,553 million 7,056 million $50,354 million

Accounts Payable (-L) ................................................................................ Cash (-A) ............................................................................................. Record cash paid to suppliers during 2012.

1

50,553

50,354 50,354


BE10–3 a. The accrual for environmental litigation increased the liabilities and reduced stockholders’ equity (because of lowered profits due to the environmental expense). b. Monsanto is actually following a couple of accounting principles with this expense. By recording contingency losses, the company is following the conservatism principle and the matching principle (taking the expense in the time period in which related revenues were recorded). c.

Environmental Liabilities | $ 176 25 | 62 | __ _|____ | $213

(Note: if $25 million was spent to reduce the liability in 2012, Monsanto would have accrued an additional $62 million in environmental remediation expenses in 2012 to show an increase in the liability to the year-end value of $213 million.)

BE10–4 a. The decrease in inventory indicates that Target sold more inventory than it purchased. Therefore, the inventory purchases were COGS – change in inventory ($50,568 – 15 = $50,553). If inventory purchases were made on account, then the $50,553 was credited to accounts payable during the year. If accounts payable increased over the course of the year, Target must have paid less to suppliers than it purchased from them (increasing previous payables). Therefore, the cash paid to suppliers was Purchases + change in accounts payable ($50,553 - 199 = $50,354). b. The increase in Accounts Receivable represented a “use” of cash and therefore decreased Target’s cash flows. The decrease in Inventory and increase in Accounts Payables were “sources” of cash and increased the company’s cash flows.

EXERCISES E10–1 a. A 12% annual discount rate is equivalent to a daily discount rate of .03288%. Present value = $40,000  [(1 + .0003288) -10] = $40,000  .99672 = $39,868.80 b. Although present value probably provides a better economic measure of a company's liabilities than is provided by the liabilities' face value, accounts payable are carried on the balance sheet at their face value. This is due to both materiality and cost/benefit considerations. Because payables are usually paid off relatively quickly, the difference between the present value of a payable and its face value will be quite small (in the problem, the difference is only $131.20). The differences are so small that financial statement users' decisions would be the same whether payables are reported at their present value or at their face value. Thus, the difference is immaterial. In addition, the cost of amortizing the discounts


(i.e., the excess of the face value of a payable over its present value) would exceed the benefits because the discount amount is so small.


E10–2 a. Current Ratio

= Current Assets ÷ Current Liabilities = ($130,000 + $50,000) ÷ $80,000 = 2.25

Current assets cannot fall below 1.5 times current liabilities. Therefore, dividing current assets by 1.5 indicates the maximum level that Darrington and Darling can allow current liabilities to grow to without violating the debt covenant. So current liabilities can grow to $120,000 ($180,000 ÷ 1.5). b. Current Ratio

= ($130,000 + $20,000) ÷ $80,000 = 1.875

Using the same logic as in part (a), the current liabilities can grow to $100,000 ($150,000 ÷ 1.5). c.

Current Ratio

= ($130,000 + $0) ÷ $80,000 = 1.625

Using the same logic as in part (a), the current liabilities can grow to $86,666.67 ($130,000 ÷ 1.5).

E10–3 Reported amounts Adjustments: Rent Wages Interest Adjusted amounts

Current Assets

Current Liabilities

Net Income

$ 24,000

$ 15,000

$ 7,500

1,008 5,000a $ 25,008

50b $ 20,050

1,008 (5,000)a (50)b $ 3,458

a $5,000 = ($7,500 ÷ 15 days per pay period)  10 days left in December b $50 = $10,000  12%  15/360

Current Ratio

= Current Assets ÷ Current Liabilities = $25,008 ÷ $20,050 = 1.25

Net Income = $3,458

E10–4 a. Cash (+A) ................................................................................................... Discount on Notes Payable (–L) ................................................................ Notes Payable (+L) .............................................................................. Borrowed money from First Bank and Trust.

19,250 750 20,000


E10–4

Concluded

b. Interest Expense (E, –SE) ........................................................................... Discount on Notes Payable (+L) .......................................................... Incurred interest expense.

250* 250

* $250 = $750 discount  30/90 days Spencer Department Store should disclose the note payable on its December 31 balance sheet as follows:

c.

Notes payable............................................................................................ Less: Discount on notes payable ............................................................... $19,500 Interest = Principal  Rate  Time $750 = $19,250  Rate  90/360 days Rate = 15.58% (rounded)

$20,000 500

d. The actual, or effective, interest rate is determined by comparing the cash payments for interest to the actual amount of cash received. The cash payment for interest equals the interest rate stated in the note times the face value of the note. Thus, the only way for the stated and effective interest rates to be the same is for the amount of cash received to be the same as the note's face value. In this case, however, the actual amount of cash received is $750 less than the note's face value of $20,000. The $750 essentially represents prepaid interest.

E10–5 a. Lacey Treetoppers has to make a total of fifteen payments of $20,000 each. As of December 31, 2014, the company has made payments for 2010, 2011, 2012, 2013, and 2014. Consequently, Lacey Treetoppers has a total of ten payments remaining. The remaining liability of $200,000 must be allocated on the balance sheet between the amounts that will mature within the time frame of current liabilities and the amounts that will not mature within that time. As of December 31, 2014, only $20,000 will become due within the next year. This $20,000 should be classified on the balance sheet under current liabilities as Current Maturities of Long-Term Debt. The remaining $180,000 should be classified on the balance sheet under long-term debt. b. Current liabilities are defined as those liabilities that will be settled through the use of current assets or through the creation of other current liabilities. If a liability is to be settled through the use of noncurrent assets or through long-term refinancing, then the liability should be classified as long-term debt. In this case, Lacey Treetoppers has basically two options in trying to avoid classifying the upcoming $20,000 installment payment as a current liability. The first option is to negotiate with the creditor to refinance the payment on a long-term basis. The second option is to intend to pay off the $20,000 using noncurrent assets. For example, the company could create a sinking fund to service the entire obligation. Since the sinking fund would be classified as a long-term investment, the corresponding liability would also be classified as long-term. Lacey Treetoppers may also be able to avoid violating its debt covenants by increasing its current assets to offset the increase in current liabilities. Assume that Lacey Treetoppers intends to pay off the upcoming installment payment by selling off investments in marketable securities classified as longterm. Since these securities would be sold off within one year, the marketable securities should be reclassified as a current asset. Consequently, current assets would be increased to offset the increase in current liabilities.


E10–6 a. Cash (+A) ................................................................................................... Deferred Revenue (+L) ....................................................................... Sold gift certificates.

88,000

b. Deferred Revenue (–L) .............................................................................. Sales (R, +SE)....................................................................................... Made sales.

52,000

Cost of Goods Sold (E, –SE) ....................................................................... Inventory (–A) ..................................................................................... Cost of inventory sold.

32,000

c.

2015 Ending balance

88,000

52,000

32,000

= 2015 Beginning balance + Gift certificates sold during 2015 – Gift certificates redeemed during 2015 = ($88,000 – $52,000) + $60,000 – $80,000 = $16,000

E10–7 a. Calculation of payments made to suppliers during 2012: Step One: Inventory – 2011 + Purchases - 2012 – COGS - 2012 = Inventory-2012

$ 2,071,890 thousand x 6,525,830 thousand $ 2,466,214 thousand

x = 6,920,154 thousand Step Two: Accounts Payable – 2011 + Purchases on account - 2012 – Cash Paid to suppliers -2012 = Accounts Payable-2012

$ 752,064 thousand 6,920,154 thousand (x) thousand $ 913,365 thousand

Cash Paid to suppliers – 2012 = $6,758,853 b. If the Statement of Cash Flows is structured in the direct format, the Cash Paid to Suppliers number would be disclosed in the Operating Section. If the Statement is structured in the indirect format, this dollar amount would be disclosed on the statement of cash flows, but it is broken into three different areas. Net income, which includes the cost of goods sold, is the first line item on the statement of cash flows. The other two items on the statement of cash flows that reflect the balance of this amount are the change in accounts payable and the change in inventory.


E10–8 a. Since Zeus Power brought the lawsuit, Zeus Power is facing a gain contingency. Gain contingencies are ordinarily not disclosed in the financial statements or in the footnotes to the financial statements due to conservatism. However, if it is probable that Zeus Power will realize the gain contingency, then it may be acceptable to disclose the contingency in the footnotes to the financial statements to avoid misleading financial statement users.

b. Since Regional Supply is the defendant in the lawsuit, Regional Supply is facing a loss contingency. The appropriate accounting treatment for this lawsuit by Regional Supply depends upon (1) whether an adverse outcome to the lawsuit (from Regional Supply's perspective) is remote, reasonably possible, or probable and (2) whether the amount of the loss, given an adverse outcome, can be reasonably estimated and (3) if it is material to Regional Supply. To record an economic event, a company must be able to quantify the dollar amount of the event. If the company cannot quantify the dollar amount of the event, it is impossible for the company to prepare a journal entry. Thus, if Regional Supply cannot reasonably estimate the amount of the loss, it cannot accrue a contingent liability. At best, Regional Supply could disclose the loss contingency in the footnotes to its financial statements. Alternatively, assume that Regional Supply can reasonably estimate how much it would lose if it lost the lawsuit. If the probability that it will lose the lawsuit is remote, Regional Supply can ignore the lawsuit for financial reporting purposes. If it is reasonably possible that Regional Supply will lose the lawsuit, it should disclose the lawsuit, and the amount of the potential loss, in the footnotes to its financial statements. Finally, if it is probable that Regional Supply will lose the lawsuit, then it should accrue a contingency liability (i.e., it should prepare a journal entry in which it recognizes a loss and related liability for the lawsuit). In this particular case, it appears that the amount of the loss can be reasonably estimated. Regional Supply must decide whether "a greater that 50% chance" of losing the lawsuit means it is reasonably possible or probable that the company will lose the lawsuit. c.

Zeus Power and Regional Supply would account for this lawsuit differently due to conservatism. Under conservatism, the basic rule is "if in doubt on how to record or report an economic event, put your worst foot forward." That is, record or report the event in the way that is least favorable to the company. Since doubt exists as to who will win the lawsuit, this event qualifies for conservatism. For Zeus Power, disclosing or recording the potential gain would put it in a better position than not disclosing or recording it. Consequently, Zeus Power should probably ignore this event for financial reporting purposes. For Regional Supply, the opposite is true. Disclosing or accruing the lawsuit presents the event in the least favorable way.

E10–9 a. The owners of a corporation (i.e., the stockholders) usually want the managers to make operating, investing, financing, and reporting decisions that will maximize the owners' wealth. However, managers have their own goals, and the stockholders of a company typically are unable to observe the day-to-day activities of the company's managers. Consequently, the owners are unsure that the managers are taking actions in the best interests of the owners. If the owners and managers have similar goals, then the managers would be expected to take actions that the owners would approve. Since maximizing net income is positively associated with maximizing stockholders' wealth, one way to help align managers' goals with the owners' goals is to make maximizing net income desirable to managers. If the managers


were to receive bonuses linked to net income, then they would be expected to try to increase net income. So a company would institute a bonus plan to try to align the goals of managers and owners. If Jordan Brothers earned net income of $300,000, then the amount allocated to the bonus pool would be computed as follows. ($300,000 – $200,000)  10% = $10,000 The appropriate journal entry would be: Bonus Expense (E, –SE).............................................................................. Bonus Liability (+L).............................................................................. Incurred, but did not pay, management bonuses.

10,000 10,000

b. The managers of the company are not eligible for a bonus unless net income exceeds $200,000. So if net income is only $180,000, nothing would be allocated to the bonus pool. Assume that Jordan Brothers will eventually lose this lawsuit and that the company will have to pay the entire $60,000. If Jordan Brothers accrues the loss now, net income will be reduced by $60,000, and no additional losses will have to be recorded when the lawsuit is actually settled. Since nothing will be allocated to the bonus pool this year anyway, accruing the loss this year does not affect this year's bonus. Since no additional losses will have to be recorded in future years, accruing the loss this year will not affect future bonuses. If, however, Jordan Brothers simply discloses the loss now, the company will have to record the $60,000 loss when the lawsuit is settled. Since nothing will be allocated to the bonus pool this year anyway, simply disclosing the loss this year does not affect this year's bonus. However, future years' bonuses could be decreased. Assume that in the year the lawsuit is settled, net income, before considering the loss, exceeds $200,000. Any item that reduces net income also reduces the allocation to the bonus pool. Consequently, simply disclosing the lawsuit could have economic consequences to the managers. By accruing the loss this year, the managers are increasing the probability that they will receive a bonus in a future year.

E10–10 a. (1)

(2)

Cash (+A) ........................................................................................ Sales (R, +SE) ........................................................................... Sold outboard engines.

50,000

Warranty Expense (E, –SE) ............................................................ Contingent Warranty Liability (+L) .......................................... Estimated warranty expense.

4,000*

50,000

4,000

* $4,000 = 200 engines  $20 estimated warranty cost per engine (3)

2014 Contingent Warranty Liability (–L) ................................................ Cash (–A) ................................................................................. Made repairs under warranty. 2015

1,400 1,400


Contingent Warranty Liability (–L) ................................................ Cash (–A) ................................................................................. Made repairs under warranty.

b. _2014 Revenue Warranty expense Net income (loss)

Contingency Basis _2015

$50,000 (4,000) $46,000

_2014_

$0 0 $0

2,600 2,600

Cash Basis _2015_ $50,000 (1,400) $48,600

$

0 (2,600) $(2,600)

E10–11 a. Southwest Airlines does not recognize revenue when the cash is received because the revenue has not been earned yet. It is not earned until the passenger takes the flight. There is always the possibility that the passenger will not take the originally scheduled flight and get a refund of their money. b. Air traffic liability is a current liability. Airlines do not sell tickets for flights that are scheduled for more than 12 months in the future. The liabilites are therefore shown as current. c.Calculation of cash receipts from passengers during 2012: Air traffic liabilities @ 2011 $ 1,836 million + cash receipts – 2012 x – revenue recognized – 2012 16.1 billion Air traffic liabilities @ 2012 $ 2,170 million x = $16.434 billion = cash receipts – 2012

E10–12 a. Year 1 Pension Expense (E, –SE) ........................................................................... Cash (–A)............................................................................................. Made pension contribution. Year 2 Pension Expense (E, –SE) ........................................................................... Cash (–A)............................................................................................. Made pension contribution. Year 3 Pension Expense (E, –SE) ........................................................................... Cash (–A)............................................................................................. Made pension contribution.

16,000 16,000

16,000 16,000

16,000 16,000

b. The amount that Seasaw Seasons should report for its pension liability equals the difference between the amount necessary to fund the benefits and the amount already paid into the pension plan. In this case, the liability would be $10,000 ($58,000 – $48,000).


E10–13 a. Income Tax Expense (E, –SE) ..................................................................... Income Tax Liability (+L) ..................................................................... Deferred Income Taxes (+L)................................................................ Incurred, but did not pay, income taxes.

28,000a 22,750b 5,250

a $28,000 = $80,000  35% b $22,750 = $65,000  35%

b. Income Tax Expense (E, –SE) ..................................................................... Income Tax Liability (+L) ..................................................................... Deferred Income Taxes (+L)................................................................ Incurred, but did not pay, income taxes.

24,000a 19,500b 4,500

a $24,000 = $80,000  30% b $19,500 = $65,000  30%

c.

Generally accepted accounting principles differ from the Internal Revenue Code. The two sets of accounting principles/procedures usually yield different income amounts. However, over the life of a company, total net income should, for all practical purposes, be the same as total taxable income. So the total tax expense recognized over the life of the entity should equal the total tax liability incurred over the life of the entity. If in a particular year the tax expense exceeds the tax liability, then in a subsequent year the tax liability must exceed the tax expense to balance out. So the balance in Deferred Income Taxes represents the amount that the entity will have to pay in the future to balance the timing differences between its tax expense and its tax liability. The balance in Deferred Income Taxes is directly linked to the tax rate. The lower the rate, the smaller the difference between the tax expense and tax liability. Consequently, there is a smaller amount that must be balanced out over the life of the entity.

E10–14 a. Conservatism Ratio

=

Reported Income Before Taxes ÷ Taxable Income

= =

$68,000 ÷ $50,000* 1.36

Income Tax Expense .................................................................................. Deferred Income Tax ($9,700 – $8,300) ............................................. Income Tax Payable (Plug).................................................................. *

$19,000 ÷ 38% = $50,000

20,400 1,400 19,000


A conservatism ratio greater than 1.0 indicates the extent to which reported income before taxes exceeds the taxable income. It seems that Busytown Industries does use some aggressive accounting policies. However, a meaningful conclusion can be drawn only with an industry-wide comparison. b. The conservatism ratio provides a reasonably good measure of the extent to which a company uses aggressive versus conservative accounting policies. The numerator of the ratio is income reported to the stockholders, which management is motivated to inflate. The denominator is the income on which the company actually would pay income taxes, which management naturally has the motivation to deflate. Thus, a ratio of the two provides a measure of the divergence between the reported income before taxes and the taxable income. The following table lists a few accounting policies and their effect on the Conservatism Ratio.

Accounting Policy 1. Straight-line method of depreciation X 2. Double-declining or any accelerated depreciation method 3. Accounting estimates, i.e., bad debts, warranties etc.

Conservatism Ratio Increase Decrease

X X

X

E10–15 a. Conservatism Ratio

=

Reported Income Before Taxes ÷ Taxable Income

= =

$145,500 ÷ $162,059* .898

Income Tax Expense .................................................................................. Deferred Income Tax ($19,400 – $18,300)................................................ Income Tax Liability (Plug) .................................................................. *

54,000 1,100 55,100

$55,100 ÷ 34% = $162,059

Since the conservatism ratio is less than 1, it appears that the company is using income-deflating policies on its income statement. b. The conservatism ratio provides a reasonably good measure of the extent to which a company uses aggressive versus conservative accounting policies. The numerator of the ratio is income reported to the stockholders, which management is motivated to inflate. The denominator is the income on which the company actually would pay income taxes, which management naturally has the motivation to deflate. Thus, a ratio of the two provides a measure of the divergence between the reported income before taxes and the taxable income. The following table lists a few accounting policies and their effect on the Conservatism Ratio.

Accounting Policy 1. Straight-line method of depreciation X

Conservatism Ratio Increase Decrease


2. Double-declining or any accelerated depreciation method 3. LIFO inventory valuation 4. FIFO inventory valuation 5. Accounting estimates, i.e., bad debts, warranties etc.

X X X X

X

PROBLEMS P10–1 a., b., and c.

Item (1) (2) (3) (4) (5) (6) (7) (8) Total

Classification Current Long-Term X X X X X

Amount Current Long-Term $170,000

X X

75,000 8,000 25,000 15,000

X X

$ 60,000 425,000

125,000 50,000 $343,000

$610,000

P10–2 The balance sheet of Linton immediately after the bank loan and purchase of equipment would be: Assets Current assets Noncurrent assets

Total assets

Liabilities and Stockholders' Equity $

$ 260,000a Current liabilities 1,860,000b Long-term liabilities

$2,120,000

Capital stock Retained earnings Total liabilities and stockholders' equity

125,000c 775,000d 1,000,000 220,000

$2,120,000

a $260,000 = $120,000 + $140,000 in cash. b $1,860,000 = $1,500,000 + $360,000 in purchased equipment. c

$125,000 = $100,000 + $25,000 in current maturities of the new note.

d $775,000 = $300,000 + $475,000 in long-term maturities of the new note.

The current ratio after recording the bank loan and the purchase of the equipment would be: Current Ratio

= Current Assets ÷ Current Liabilities = $260,000 ÷ $125,000 = 2.08


Declaring a dividend would increase current liabilities. Current assets cannot fall below 2 x current liabilities if Linton is to avoid violating its debt covenant. Since current assets are currently $260,000, Linton could declare and pay a dividend of $5,000. This dividend would increase current liabilities to $130,000 and reduce retained earnings to $215,000.

P10–3 a. (1)

Bad Debt Expense (E, –SE) ............................................................. Allowance for Doubtful Accounts (–A) ................................... Estimated bad debts.

* $1,000

(2)

(3)

(4)

(5)

(6)

1,000* 1,000

= ($50,000 Accounts receivable balance  6% Estimated uncollectible percentage) – $2,000 Balance in the Allowance account

Warranty Expense (E, –SE) ............................................................ Warranty Liabilities (+L) .......................................................... Estimated warranty expense.

7,000

Unearned Revenues (–L) ............................................................... Revenue (R, +SE) ..................................................................... Earned revenue from advance collections.

10,000

Other Current Liabilities (–L) ......................................................... Long-Term Liabilities (+L) ........................................................ Reclassified liabilities.

5,000

Income Tax Expense (E, –SE) ......................................................... Income Tax Payable (+L) ......................................................... Incurred income tax expense.

3,000

Loss on Lawsuit (Lo, –SE) ............................................................... Short-Term Contingent Liability (+L)....................................... Incurred potential loss on a lawsuit.

10,000*

7,000

10,000

5,000

3,000

10,000

* Assumes that 60% is considered probable. If 60% was considered only reasonably possible, the $10,000 would not be accrued, but would, instead, be disclosed in a footnote. b. After considering the journal entries in part (a), except for entry (6), current assets and current liabilities would be as follows. Current Assets

= $40,000 + $50,000 – $3,000 + $52,000 = $139,000

Current Liabilities =

$30,000 + $15,000 + $12,000 + $5,000 + $3,000


= Current Ratio

$65,000

= Current Assets ÷ Current Liabilities = $139,000 ÷ $65,000 = 2.138


P10–3 c.

Continued

After considering the journal entries in part (a), including entry (6), current assets and current liabilities would be as follows. Current Assets

= $40,000 + $50,000 – $3,000 + $52,000 = $139,000

Current Liabilities = $30,000 + $15,000 + $12,000 + $5,000 + $3,000 + $10,000 = $75,000 Current Ratio

= $139,000 ÷ $75,000 = 1.853

d. The company's auditors must first consider the directives of the FASB. SFAS Statement No. 5, "Accounting for Contingencies," addresses contingent liabilities. This pronouncement states that if a contingent liability is both reasonably estimable and probable, then the liability must be accrued. But if the contingent liability is reasonably estimable and only reasonably possible, or if it is not reasonably estimable, then the liability only has to be disclosed in a footnote. Finally, if a contingency is remotely likely, then the contingent liability does not have to be disclosed. Since SFAS Statement No. 5 does not provide a definition of probable, reasonably possible, or remote, judgment must be used in converting a probability estimate to one of these categories. One factor that may greatly influence the auditor's judgment is his or her legal liability regarding each classification. Assume that the company loses the lawsuit. If the liability was considered remotely likely, and hence not disclosed, the auditor would be legally liable to financial statement users. If the liability was considered reasonably possible, and hence disclosed in the footnotes, the auditor could potentially still face legal action from the financial statement users. The users could argue that the auditor should have required the lawsuit to be accrued. Finally, if the liability was considered probable, and hence accrued, the financial statement users were provided with the correct information and the auditor would not face any legal liability. Assume that the company wins the lawsuit. If the liability was considered remotely likely, the financial statement users were provided with the correct information. The managers of the company would also be happy because the company was not made to appear worse off than it actually was. If the liability was considered reasonably possible and only disclosed in the footnotes, it is doubtful that many financial statement users incurred out-of-pocket losses due to the disclosure. Some potential investors or creditors might have foregone transacting with the company, but these people only incurred opportunity losses. People cannot sue for opportunity losses. The managers might be somewhat upset that the potential loss was disclosed in the footnotes. This disclosure might make it more difficult to attract capital. However, since the contingent liability was only disclosed in the footnotes and was not accrued, the liability would not affect any of the manager's contracts such as debt covenants or bonus contracts. Finally, if the contingent liability were accrued, the auditor would probably not face any legal liability because the auditor took the most conservative action. However, the managers would probably be upset. Since the contingent liability was accrued, liabilities would be increased, and the company would have to report a loss on the income statement. Accruing the liability could decrease the manager's bonus and could also place the company in default on its debt covenants.


P10–3

Concluded

The auditor faces two conflicting interests. The auditor must consider his or her legal liability to financial statement users. On the other hand, the manager pays the audit fees. If the auditor demands that the contingent liability be accrued, the manager may fire the auditor. The auditor must trade off these two conflicting interests in trying to decide whether or not to accrue a contingent liability. In most cases, the cost of legal liability will probably be greater than lost audit fees and damage to the auditor's reputation. In this particular case, a 60% probability of losing the lawsuit would probably be considered either reasonably possible or probable. So at the minimum, the auditor should require disclosure of the contingent liability in the footnotes.

P10–4 a. From an accounting perspective, the Floor Wax Shop must consider generally accepted accounting principles. The company would find authoritative guidance in SFAS Statement No. 5, "Accounting for Contingencies." According to this statement, the two factors that the Floor Wax Shop must consider are (1) whether the amount of the loss can be reasonably estimated and (2) the probability that the Floor Wax Shop will eventually incur the loss. From an economic perspective, the Floor Wax Shop must consider the costs and benefits of the different ways to report this lawsuit. If the lawsuit is not disclosed or accrued and the company subsequently loses the lawsuit, financial statement users could sue the company for any losses they incurred from relying on the company's financial statements. If the Floor Wax Shop discloses the lawsuit but does not accrue a loss, then it is less likely that any financial statement users would be able to successfully sue the company. Also, any contracts in place, such as debt covenants or bonuses, would probably not be affected by a footnote disclosure. However, the footnote disclosure may make it more difficult for the company to attract new capital. Some suppliers may also be wary of extending credit, thereby forcing the Floor Wax Shop to pay cash for purchases or sign interest-bearing notes. In either case, the Floor Wax Shop could encounter cash flow problems. If the Floor Wax Shop accrues the lawsuit, the company would report a loss on its income statement and an associated liability on its balance sheet. In this case it would be virtually impossible for a financial statement user to successfully sue the Floor Wax Shop for providing misleading or incorrect information. However, in addition to the problems discussed associated with disclosing the lawsuit, accruing a loss for the lawsuit could affect existing contracts. Accruing a loss might also be considered an admission of guilt to the court, which could cause accruing the loss to become a self-fulfilling prophecy.


P10–4

Concluded

b. The Floor Wax Shop should accrue the lawsuit for two reasons. First, the contingent loss meets the requirements set forth in SFAS Statement No. 5, "Accounting for Contingencies," for accruing a contingent loss. That is, it is both probable that the Floor Wax Shop will lose the lawsuit, and the amount of the loss can be reasonably estimated. Second, accruing a loss minimizes my legal exposure as the company's auditor. c.

The Floor Wax Shop would have made the following entry on December 31, 2014 to accrue the contingent loss. Contingent Loss on Lawsuit (Lo, –SE) ........................................................ Contingent Liability (+L) ...................................................................... Accrued contingent loss.

742,000 742,000

The amount recorded for the contingent loss and liability on December 31, 2014 was simply an estimate. That is, $742,000 was the company's best estimate as of that date about the amount the company would eventually lose. On August 12, 2015, when the lawsuit was settled, the Floor Wax Shop was able to revise its estimate. Since changes in estimates are accounted for prospectively, a prior period adjustment is not necessary. Instead, the Floor Wax Shop should make the following entry on August 12, 2015. Contingent Liability (–L) ............................................................................ Cash (–A)............................................................................................. Recovery of a Contingent Loss (R, +SE) .............................................. Settled lawsuit.

742,000 690,000 52,000

P10–5 a. Cash (+A) ................................................................................................... Sales (R, +SE)....................................................................................... Sold cars. b. 5/30

332,500

Contingent Warranty Liability (–L) ........................................... Cash (–A)............................................................................ Parts (–A) ........................................................................... Made repairs under warranty.

3,000

Contingent Warranty Liability (–L) ........................................... Cash (–A)............................................................................ Parts (–A) ........................................................................... Made repairs under warranty.

5,000

Contingent Warranty Liability (–L) ........................................... Cash (–A)............................................................................ Parts (–A) ........................................................................... Made repairs under warranty.

6,000

Warranty Expense (E, –SE) ........................................................................ Contingent Warranty Liability (+L) ..................................................... Estimated warranty expense.

26,600*

9/2

11/15

c.

332,500

1,200 1,800

2,000 3,000

2,400 3,600

26,600


* 26,600 = 35 cars sold  $760 estimated warranty cost per car

P10–5

Concluded

d. Ending Balance

= Beginning Balance + Warranty Expense for the Year – Cost of Under Warranty = $3,500 + $26,600 [from part (c)] – ($3,000 + $5,000 + $6,000) = $16,100

Repairs

e. Under the matching principle, all costs that are incurred in generating revenue should be matched against those revenues. The matching principle does not distinguish between costs incurred before or after the point of sale. Consequently, both presale and post-sale costs should be matched against the associated revenue. The problem with matching post-sale costs against the associated revenue is that the costs have not yet been incurred by the time the revenue is earned. Thus, the only way to be able to match post-sale costs against the associated revenue at the time the revenue is earned is to estimate the value of the post-sale costs. Since post-sale costs are an example of a contingency, those post-sale costs that are both reasonably estimable and probable are recognized at the time the revenue is recognized. In the case of warranties, a company can usually use company- or industry-specific data to estimate the average cost that will subsequently be incurred to make repairs under warranty. However, if a company expects to incur a material amount of repairs under warranty but cannot reasonably estimate the amount, it is debatable whether the company should recognize the revenue in the current period. One of the criteria for recognizing revenue under the revenue recognition principle is that post-sale costs can be reasonably estimated. If this criterion cannot be met and the amount is material, a company should probably delay recognizing the revenue.

P10–6 a.

2014 Cash (+A) ………………………………… Sales (R, +SE) ……………………….. Made sales.

40,000 ........ 56,000 40,000 ......

2015 56,000

Promotion Expense (E, –SE)……………… .......... 400a Contingent Promotion Liability (+L) …. 400 Estimated promotion cost.

560b 560

= 20,000 boxes sold during the year  Expected redemption rate of 10% ÷ 5 box tops per refund  $1.00 per refund b $560 = 28,000 boxes  10% ÷ 5 box tops per refund  $1.00 per refund 2014 2015 a Contingent Promotion Liability (–L) ................... 300 400b Cash (–A)...................................................... 300 Paid promotional refund. a $400

a $300 = 1,500 box tops redeemed ÷ 5 box tops per refund  $1.00 per refund b $400 = 2,000 box tops redeemed ÷ 5 box tops per refund  $1.00 per refund

400


P10–6

Concluded

b. Ending balance

=

Beginning balance + Promotional expense – Refund payments

2014:

= =

$0 + $400 – $300 $100

2015:

= =

$100 + $560 – $400 $260

P10–7 a. Restructuing Expense (E, –SE) ................................................................... Assets (-A) ........................................................................................... Severance Liability (+L) .......................................................................

425 364 61

b. The $364 million expense was subtracted from earnings to produce net income, but the expense did not reduce the company’s cash balances. Therefore, to convert the net income figure to the cash from operating activities number, the non-cash expense was added back. c.

Reduction in taxable earnings x tax rate = tax savings $425 million x .26 = $110.5 million

P10–8 a.

2010 Pension Expense (E, –SE) ................................. 40,000 Cash (–A).................................................... Pension Liability (+L) .................................. Funded pension.

2011 40,000 32,000 8,000

32,000 8,000

2012 Pension Expense (E, –SE) .................................. 40,000 Cash (–A)..................................................... Pension Liability (+L) ................................... Funded pension.

2013 40,000 36,000 4,000

36,000 4,000

2014 Pension Expense (E, –SE) ................................. 40,000 Cash (–A).................................................... Funded pension. b.

Pension Expense 2010 2011 2012 2013 2014

$ 40,000 40,000 40,000 40,000 40,000 $200,000

Amount Funded $ 32,000 32,000 36,000 36,000 40,000 $176,000

40,000

Pension Liability $

8,000 8,000 4,000 4,000 0 $24,000


Thus, the balance in the Pension Liability account as of December 31, 2014 is $24,000.

P10–9 a. 2010 Income Tax Expense (E, –SE) ..................................................................... Deferred Income Taxes (+L)................................................................ Income Tax Liability (+L) ..................................................................... Incurred income taxes.

30,625a 4,375 26,250b

a $30,625 = ($100,000 – $12,500 in depreciation expense)  35% b $26,250 = ($100,000 – $25,000 in depreciation expense)  35%

2011 Income Tax Expense (E, –SE) ..................................................................... Income Tax Liability (+L) ..................................................................... Incurred income taxes.

30,625* 30,625

* $30,625 = ($100,000 – $12,500 in depreciation expense)  35% 2012 Income Tax Expense (E, –SE) ..................................................................... Deferred Income Taxes (–L) ...................................................................... Income Tax Liability (+L) ..................................................................... Incurred income taxes.

30,625.00 2,187.50 32,812.50*

* $32,812.50 = ($100,000 – $6,250 in depreciation expense)  35% 2013 Income Tax Expense (E, –SE) ..................................................................... Deferred Income Taxes (–L) ...................................................................... Income Tax Liability (+L) ..................................................................... Incurred income taxes.

2010 2011 2012 2013

30,625.00 2,187.50 32,812.50

Income Tax Expense

Income Tax Liability

Change in Deferred Income Taxes

Deferred Income Tax Balance

$30,625 30,625 30,625 30,625

$26,250.00 30,625.00 32,812.50 32,812.50

$ 4,375.00 0 (2,187.50) (2,187.50)

$4,375.00 4,375.00 2,187.50 0

b. The balance in Deferred Income Taxes represents the amount a company will, theoretically, have to pay the government in the future. Amount differences, such as arise when a company uses different depreciation methods for financial reporting and tax purposes, between book and taxable income reverse themselves over time. If the government lowers the tax rate, then the tax liability in future periods when the timing differences reverse themselves will be lower than the associated tax expense recorded on the books when the older tax rate was still in effect. Thus, the amount of tax expense recorded in prior periods was "incorrect" given the new tax rates, and the company should adjust its books for the decrease in the expense. If the misstatement is considered an error, then a prior period adjustment is necessary. But if the misstatement is considered to be a change in estimate (which is the


more likely view), then the correction would be accounted for prospectively and reported as a gain in the current period. In this particular case, the gain from the change in the tax rate is $1,875.

P10–9

Continued

2010

Same as in part (a)

2011

Same as in part (a)

2012 Deferred Income Taxes (–L) ...................................................................... Gain on Deferred Income Taxes (Ga, +SE) .......................................... Adjusted for change in statutory tax rate.

1,875* 1,875

* $1,875 = Excess tax depreciation of $12,500  Decrease in tax rate of 15% Income Tax Expense (E, –SE) ..................................................................... Deferred Income Taxes (–L) ...................................................................... Income Tax Liability (+L) ..................................................................... Incurred income taxes.

17,500 1,250 18,750*

* $18,750 = ($100,000 – $6,250 in depreciation expense)  20% 2013 Income Tax Expense (E, –SE) ..................................................................... Deferred Income Taxes (–L) ...................................................................... Income Tax Liability (+L) ..................................................................... Incurred income taxes.

2010 2011 2012 2012 2013 c.

Income Tax Expense

Income Tax Liability

$30,625 30,625

$26,250 30,625

17,500 17,500

18,750 18,750

Change in Deferred Income Taxes $4,375 0 (1,875) (1,250) (1,250)

17,500 1,250 18,750

Deferred Income Tax Balance $4,375 4,375 2,500 1,250 0

2010 Same as in part (a) 2011 Income Tax Expense (E, –SE) ..................................................................... Deferred Income Taxes (+L)................................................................ Income Tax Liability (+L) ..................................................................... Incurred income taxes.

26,250a 4,375 21,875b

a $26,250 = ($100,000 – $25,000 in depreciation expense)  35% b $21,875 = ($100,000 – $37,500 in depreciation expense)  35%

2012 Income Tax Expense (E, –SE) .....................................................................

26,250.00


Deferred Income Taxes (–L) ...................................................................... Income Tax Liability (+L) ..................................................................... Incurred income taxes. * $28,437.50 = ($100,000 – $18,750 in depreciation expense)  35%

2,187.50 28,437.50*


P10–9

Concluded

2013 Income Tax Expense (E, –SE) ..................................................................... Income Tax Liability (+L) ..................................................................... Incurred income taxes.

21,875* 21,875

* $21,875 = ($100,000 – $37,500 in depreciation expense)  35% Income Tax Expense

Income Tax Liability

$30,625 26,250 26,250 21,875

$26,250.00 21,875.00 28,437.50 21,875.00

2010 2011 2012 2013

Change in Deferred Income Taxes

Deferred Income Tax Balance

$ 4,375.00 4,375.00 (2,187.50) (0)

$4,375.00 8,750.00 6,562.50 6,562.50

If a company uses different depreciation methods for calculating book and taxable income, it is possible that the Deferred Income Tax account will remain on the company's books indefinitely. This would happen if a company continues to acquire fixed assets because the deferred taxes arising from the new assets would exceed the deferred income taxes being reversed from prior accounting periods. Consequently, the balance in Deferred Income Taxes would continue to grow over time.

P10–10 Based on the information provided in this problem, we can compute the conservatism ratio of each company: The lower the ratio, the higher the earning power of the company. Conservatism Ratio

= Reported Income Before Taxes ÷ Taxable Income

Owen-Foley Company Income Tax Expense (I/S) .......................................................................... Deferred Income Tax ($18,400 – $16,600) ......................................... Income Tax Liability (Plug) .................................................................. Taxable Income

= =

Conservatism Ratio

52,000 1,800 50,200

$50,200 ÷ 36% $139,444 = =

$163,000 ÷ $139,445 1.169

Amerton Industries Income Tax Expense (I/S) .......................................................................... Deferred Income Tax ($19,800 – $18,800)................................................ Income Tax Liability (Plug) .................................................................. Taxable Income Conservatism Ratio

= =

53,500 1,000 54,500

$54,500 ÷ 36% $151,389 = $158,500 ÷ $151,389 = 1.047

Amerton Industries’ conservatism ratio is lower than Owen-Foley Company’. Therefore, it has stronger earning power than Owen-Foley.


P10–11 Based on the information provided in this problem, we can compute the conservatism ratio of each company: The lower the ratio, the more conservative the company. The higher conservatism ratio indicates that management is more aggressive with its tax policies and results in higher future tax liabilities. The lower the ratio, the less aggressive management is with tax policies. Conservatism Ratio

= Reported Income Before Taxes ÷ Taxable Income

Walgreen’s Taxable Income = $1,010 ÷ 37.0% = $2,730 Conservatism Ratio

= =

$3,376 ÷ $2,730 1.24

The Limited Taxable Income

Conservatism Ratio

= =

$517 ÷ 41.2% $1,255 = =

$1,281 ÷ $1,255 1.02

The above analysis indicates Walgreen’s is more aggressive than The Limited.


ISSUES FOR DISCUSSION ID10–1 a. Under the terms of its debt covenants, FedEx's current assets must be at least as great as its current liabilities. Since Federal Express had current assets of $9,056 as of May 31, 2012, the maximum amount of liabilities Federal Express could have without violating its debt covenant would also be $9,056. Therefore, FedEx could report additional current liabilities of $3,682 ($9,056 – $5,374) as of May 31, 2012. b. There are many current liabilities where either the creation of or the amount reported for the liability is under the company's control. For example, a company's board of directors decides when to declare a dividend (which results in the current liability Dividend Payable), how much the dividend will be and when the liability will be paid. Management has discretion over the amount reported for contingent liabilities such as Contingent Warranty Liability or Contingent Promotion Liability. By altering their estimates, management can decrease the amount reported for such liabilities. Management could also control to some extent the amount reported for Unearned Revenues. That is, management could simply not take any advances from customers, or management could control when it provides the goods or services for which it collected the advances from customers. c.

Violating a debt covenant results in the borrower being in technical default on the loan. If a company is in default on a loan, the creditor could require the borrower to immediately repay the outstanding balance. Alternatively, the creditor could allow the borrower to renegotiate the loan. However, in such cases the borrower is usually forced to agree to less favorable loan terms such as a higher interest rate, providing more collateral, and so forth.

d. If FedEx purchased the aircraft for cash, its noncurrent assets would increase by $4 billion but its current assets would decrease $4 billion for the cash disbursed. With no change in its current liabilities, funding the aircraft purchase with cash would decrease the company’s current ratio from 1.69 ($9,056/$5,374) to 0.97 ([$9,056 - $4,000]/$5,374). FedEx could not fund the purchase with cash without violating its debt covenant. Alternatively, if FedEx purchased the aircraft using long-term debt, neither current assets nor current liabilities would be affected, leaving the current ratio at 1.69 (in compliance with the debt covenant).

ID10–2 a. The account Customers' Advance Payments represents cash collected from customers in advance of providing desired goods or services to the customers. When Ingersoll-Rand makes these collections, it is implicitly promising to either provide the desired goods or services or refund the customer's money. Thus, the cash collected in advance for which Ingersoll-Rand has not yet provided the desired goods or services represents a liability to Ingersoll-Rand, and it should be reported in the liability section of the balance sheet in the account Customers' Advance Payments. In addition, the cash collections made during the year should be reported in the operating activities section of the statement of cash flows.


ID10–2 Concluded b. Under the revenue recognition principle, a company should not recognize revenue until (1) the company has earned the revenue, (2) the amount of revenue earned can be objectively determined, (3) any post-sale costs can be reasonably estimated, and (4) cash collection is reasonably assured. Although the cash collection is reasonably assured when Ingersoll-Rand collects cash in advance from its customers, it is not really entitled to keep the cash at the time it makes the collections. That is, the company has to provide some goods or services before it is entitled to the cash (i.e., before it has generated some revenue). Thus, Ingersoll-Rand does not earn any revenue simply from making collections in advance from its customers. It is not until the company provides some goods or services that the company should actually recognize the revenue. Once Ingersoll-Rand provides the goods or services and recognizes the revenue, the company should then match all costs—both pre-sale and postsale costs—against the revenue in accordance with the matching principle. c.

Ingersoll-Rand should not recognize revenue for simply collecting customer advances; instead, it generates revenue when it actually ships the goods. Thus, earnings per share would increase when Ingersoll-Rand ships goods (assuming that the company sells its inventory at a profit), and it would not be affected by receiving advance payments. Collecting advance payments would increase both Ingersoll-Rand's current assets (through the cash collected) and current liabilities (through the obligation arising from the advances). The actual effect on the company's current ratio depends on what the ratio was before the company made the advance collections. If the current ratio was less than 1, collecting advance payments would increase IngersollRand's current ratio. Alternatively, if its current ratio was greater than 1, collecting advance payments would decrease Ingersoll-Rand's current ratio. Shipping the related goods would decrease IngersollRand's current liabilities, which means that the company's current ratio would increase. Since collecting advance payments would increase Ingersoll-Rand's current liabilities, its debt/equity ratio would increase. Shipping the related goods would both decrease the company's current liabilities and increase its stockholders' equity (through the increased income that would be closed into Retained Earnings). Thus, the company's debt/equity ratio would decrease when the goods are shipped.

ID10–3 a.

In 2012, 25.0% ($324/$1,297) of the reserve was classified as current; in 2011, 23.1% ($298/$1,291) of the reserve was current.

b.

The journal entries appear below:

2010: Other Operating Expense (E, -SE) Professional Liability Reserve (+L)

222 222

2011: Other Operating Expense (E, -SE) Professional Liability Reserve (+L)

244 244

2012: Other Operating Expense (E, -SE)

331


Professional Liability Reserve (+L)

331

ID10–3 CONTINUED c. Professional Liability Reserve– 12/31/2011 + Professional Liability Expense - 2012 – Cash Payments re: Liability - 2012 = Professional Liability Reserve-12/31/2012

$ 1,291 million 331 million x $ 1,297 million

x = $325 million d. Earnings could be managed in the healthcare industry with manipulation to the professional liability reserve (a liability account) by adjusting the annual charge taken for the expense. If the company were having a very successful year, beating analyst earnings estimates, management could “pad” the reserve by taking larger-than-necessary expenses so that in future years, when earnings might be below targets, management could then take lower-than-necessary expenses (and therefore show higher earnings).

ID10–4 The numerous filed and potential criminal and civil lawsuits against Philip Morris are examples of loss contingencies. SFAS Statement No. 5, "Accounting for Contingencies," provides authoritative guidance in accounting for contingencies. According to this statement, the two factors that should be considered in deciding how to account for a loss contingency are (1) whether the amount of the loss can be reasonably estimated and (2) the likelihood that the company will eventually experience the loss. If the likelihood that the company will eventually experience the loss is considered to be remote, then the company does not need to provide any information about the loss contingency in its financial statements or associated footnotes. A company should disclose a loss contingency in the footnotes to its financial statements under two conditions. The first condition is that the likelihood that the company will eventually experience the loss is considered to be reasonably possible. The second condition is that the likelihood that the company will eventually experience the loss is considered to be probable, but the amount of the loss cannot be reasonably estimated. Finally, a company should accrue the loss if it is both probable that the company will eventually experience the loss and the amount of the loss can be reasonably estimated. Accruing the loss means that the company reports a loss on its income statement and, in the case of a lawsuit, an associated liability on its balance sheet. It is probable that Philip Morris would lose a number of lawsuits associated with smoking, it was probably not possible to determine which lawsuits it would win and which ones it would lose. Philip Morris would have to estimate the probability of which lawsuits it would lose, then Philip Morris has to reasonably estimate the amount of the court settlements because of the magnitude of the damage caused by smoking. Philip Morris then estimated the overall liability and booked this as a contingent liability. By disclosing the lawsuits in its footnotes, Philip Morris has provided its financial statement users with relevant information about events that could affect the company's financial health. Thus, the financial statement users would have access to a discussion about the various lawsuits and management’s estimate as to the potential financial losses. Users can then make their own assessment as to the potential liabilities that Philip Morris may face.


ID10–5 a. There are two reasons why Lifschultz's stock could be highly valued even though the company has a negative book value. First, the stock market expects Lifschultz to win the lawsuits and collect a large settlement. The expected settlement, however, is currently not reflected in the company's financial statements. Thus, the potential settlement essentially represents an unreported asset. If Lifschultz included the amount it expected to win in the lawsuits, its net book value may be positive. In addition, generally accepted accounting principles do not allow a company to report any self-generated goodwill (such as customer loyalty or name recognition). Goodwill is often a company's largest asset. Second, book value and stock prices are based on two different measures. The price of a company's stock should, theoretically, equal the present value of the future cash flows the stock market expects an investment in that company to generate. Alternatively, a company's net book value is based largely on historical cost. A company could have a negative book value, but if the stock market thought that the company could use its assets to generate enough cash to both service its debt and disburse to its stockholders, the stock market would value that company's stock positively. b. SFAS Statement No. 5, "Accounting for Contingencies," provides authoritative guidance in accounting for contingencies. According to this statement, gain contingencies should usually be ignored for financial reporting purposes. If it is highly probable that the company will eventually realize the gain, the company could arguably disclose the gain contingency in a footnote. Alternatively, there are three potential ways to account for a loss contingency. The first way is to ignore the loss contingency. This approach would be appropriate if the likelihood of the company eventually experiencing the loss is remote. The second accounting treatment is to disclose the loss contingency in a footnote to the financial statements. This approach is appropriate (1) if the likelihood that the company will eventually experience the loss is considered to be reasonably possible or (2) if the likelihood that the company will eventually experience the loss is considered to be probable but the amount of the loss cannot be reasonably estimated. The final potential accounting treatment is to accrue a loss. That is, record a loss and associated liability. This approach is appropriate when both the likelihood of the company eventually experiencing the loss is probable and the amount of the loss can be reasonably estimated. Since Lifschultz has a gain contingency, it should not disclose this information in its financial statements or footnotes. Alternatively, the three trucking companies have loss contingencies associated with the lawsuit. The fact that the three companies only disclosed the lawsuit in their footnotes indicates that either the trucking companies thought it was only reasonably possible that they would lose the lawsuit or that they could not reasonably estimate the amount of the loss. c.

The costs of overstating and understating a company's financial position and operations is very different. Assume that a company overstates its financial position and operations in its financial statements. If the users of the financial statements use the information in the financial statements to make investment decisions and then incur a loss on their investments, the financial statement users would probably sue the company and its auditors. The financial statement users could argue that they would not have made the investment if the company had not overstated its financial position and operations. The cost of defending oneself against such lawsuits—both the monetary cost and the loss of reputation—can be very large.


Alternatively, assume that a company understates its financial position and operations in its financial statements. People may invest in the company despite its reported financial position and operations, but once the understatement becomes known, the investment should increase in value. Thus, the people who invested in the company anyway will probably not be upset and pursue legal action against the company or its auditors. Although some people might argue that they incurred a loss because they would have invested in the company if it had not under-reported its financial position and operations, such losses represent opportunity costs. Opportunity costs generally do not provide a sufficient basis for pursuing legal action against a company or its auditors. Thus, the cost to a company and its auditors of overstating the company's financial position and operations is greater than the cost of understating its financial position and operations. Reporting or accruing a gain contingency could result in an overstatement but not an understatement of a company's financial position and operations. Since reporting or accruing a loss contingency could result in an understatement but not an overstatement of the company's financial position and operations, it makes sense economically that plaintiffs and defendants would account for unsettled lawsuits differently.

ID10–6 a. A "technical default" means that the borrower has violated some term of a debt agreement, usually a debt covenant. In this particular case, Citibank reacted to Campeau Corporation's technical default by giving Campeau a few days to correct the problem. If Campeau is unable to correct the problem, Citibank appears to intend to require Campeau to immediately repay the entire outstanding loan balance. Another option Citibank could have pursued would have been to renegotiate the loan with terms more favorable to Citibank. For example, Citibank could have required more collateral from Campeau or increased the interest rate it was charging Campeau. b. Campeau's technical default and Citibank's statement that they may demand immediate full repayment of the loan indicate that Campeau may be experiencing severe cash flow problems. Suppliers would usually be unwilling to ship merchandise to a customer if there is a good chance that the customer will not be able to pay for the merchandise. If the customer is in danger of declaring bankruptcy, the suppliers' unwillingness would be even greater. This is because suppliers are usually unsecured creditors in that the goods were purchased on account, which means that the suppliers have only the customer's promise of payment; the merchandise usually does not serve as collateral. If the customer were to declare bankruptcy, the secured creditors would be paid first, and only if there was any cash left, would the unsecured creditors be paid. The suppliers' unwillingness to ship merchandise only increases Campeau's cash flow problems. Without the new spring merchandise, Campeau's stores will be unable to compete against other stores who have the new fashions, and its stores will probably run low on merchandise. This means that the company will be able to generate less cash from its operations.

ID10–7 a.

Unearned Extended Warranty Revenue |45 (2011 Beginning Balance) | |72 (2011 accruals*) (2011 settlements)67 _ |_______________________

|50 (2011 Ending Balance) *the journal entry would have been Cash or A/R (+A)

72


Unearned Extended Warranty Revenue (+L)

72

Unearned Extended Warranty Revenue |50 (2012 Beginning Balance) | |88 (2012 accruals*) (2012 settlements)78_ |_______________________

|60 (2012 Ending Balance) *the journal entry would have been Cash or Accounts Receivable (+A) 88

Unearned Extended Warranty Revenue (+L)

88

The Revenue is only recognized when the earning process (honoring the requests under the warranty program) is complete. In the time period when Agilent honors the warranty, it will incur expenses (to repair a system, for example) which will then be matched with the Revenue from the Extended Warranty.

b. The cash from the sale of the Extended Warranty is received on the date of the sale of the warranty; the Statement of Cash Flow will reflect an inflow of cash in the period that the cash was received. The profitability of the warranty, however, is recorded as the warranty period expires, when the Revenue from the Extended Warranty is matched with any expenses incurred to honor the warranty.

ID10–8 a.

Deferred Revenue is a liability that represents the obligation that United Continental has to its customers for flights that have been paid for but have not yet been taken. The liability is carried on the balance sheet until the Revenue can be recognized. When the revenue is recognized (with a credit), the liability Deferred Revenue is reduced (with a debit).

b.

From a cash flow perspective, the $5.2 billion was received in prior periods when customers purchased their tickets. When the flight is taken by the customer, the airline simply a converts the liability to revenue but will not receive any additional cash.

c.

An analyst will be concerned with United Continental if there is a sharp drop in Deferred Revenue, as that will most likely mean fewer flights in the future and an adverse effect on profitability. Since most airline customers purchase tickets in advance, a drop indicates a softening business climate. A sharp rise, on the other hand, would indicate that business is picking up as customers purchase more and more flights. Given the high fixed costs in the industry and the correlation between volume and profits, a change in Deferred Revenue is certainly of interest to analysts.

ID10–9 a.

Short-term borrowings: the company issued debt securities that are due within 12 months Trade accounts payable: the company purchased inventory on account, agreeing to pay in the near future Employee compensation: the company owes its workers for labor that was performed since the last pay day; the obligation will be satisfied at the next pay day Other accrued liabilities: the company has miscellaneous obligations, such as utility bills, that have been expensed on the income statement but have yet to be paid in cash Dividend payable: the company has declared but not yet paid dividends to its stockholders Income tax payable: the company has expensed its tax obligations but has yet to settle with cash


Current portion of long-term debt: the company has long-term debt with some principal payments due in the next 12 months; that portion of the long-term debt due after the next 12 months is categorized in the noncurrent liabilities section of the balance sheet b.

If these obligations are categorized as current, then they will be satisfied from the conversion of current assets. For example, the accounts payable obligation will be paid when the company sells inventory (a current asset) and collects cash.

c.

The company might be considering converting the short-term borrowings to long-term. If long-term interest rates are low and the company recognizes that it will be a borrower for some time, it might make sense to restructure the debt to long term. Perhaps the company could address some solvency issues (working capital, the current ratio) by recasting some of its short-term debt to long-term.

d.

Return on equity is the ratio of net income to shareholders’ equity. If a company can responsibly use debt, it can improve its ROE. If the debt is used to add assets that strongly contribute to earnings, then ROE will rise. If Abbott can use its current liabilities to fund inventory that can be sold for higher margins, the increased profitability will transfer to improved returns for the shareholders. On the other hand, if companies overload on debt (in an attempt to drive ROE), interest charges can drag down earnings; in a worst case scenario, if the debt is increased to the point that an interruption to earnings and cash flow prevent principal repayment, ROE can be jeopardized as the company may become insolvent. Proper management of debt allows companies to use leverage to improve earnings available to shareholders while adequately providing cushion so that the debt can be repaid.

ID10–10 A proxy is a device whereby a security holder authorizes another person or group of persons to act for him at a meeting of security holders—in effect a power of attorney. Whenever the solicitation is made on behalf of the management of the issuer and relates to an annual meeting of security holders at which directors are to be elected, a proxy statement must be furnished to the security holders pursuant to the SEC requirements. In essence, proxy statements are disclosure instruments with all kinds of information. They are furnished to the stockholders at a time when the shareholders have to make critical decisions through a proxy to the management. The new SEC required disclosures in the proxy statements regarding the executive compensation can be very informative to the stockholders. If executive compensation is linked to the shareholder returns or stock market performance, then it is most likely that management’s actions, while maximizing their own wealth, will also maximize the wealth of the stockholders. In a way, linking executive compensation to stock market returns or shareholder returns promotes goal congruence. Further, if a company’s executives own a lot of its stock, stockholders can take comfort that executives will make decisions that would really maximize the economic wealth of the company. They would try to increase the market price of each share rather than play with accounting policies to declare more income on the income statement. Any information that indicates that there has been a change in the auditors can also provide useful signals to the stockholders. Such a change can indicate the existence of serious disagreement between the auditor and the management. The shareholders can further investigate through SEC as to what were the reasons leading to such a change before assigning their proxy to the current management.


ID10–11 a. Medical and Life Insurance Expense (E, –SE) .......................... Accrued Insurance Liability (+L) ........................................ Accrued medical and life insurance expense.

2,260,000,000 2,260,000,000

b. As part of the Chapter 11 reorganization, LTV negotiated new credit agreements. These credit agreements contained debt covenants, most likely related to the amount of debt LTV could have. If LTV had waited until after emerging from Chapter 11, the $2.26 billion insurance liability might have caused the company to violate the amount of allowable debt as specified in its debt agreements. Such a violation could have forced the company back into Chapter 11 or to renegotiate its debt agreements at less favorable terms. By recording the $2.26 billion liability prior to negotiating its new debt agreements, the $2.26 billion liability would be considered by creditors in creating the debt covenants. Thus, this liability would not place LTV into an automatic violation of its debt covenants. c.

There are probably two reasons why LTV chose to take several significant charges while it was under bankruptcy proceedings. First, the significant charges would adversely affect LTV's reported results of operations and financial position, and LTV may have been trying to extract more favorable settlement terms from its creditors by demonstrating weakened performance and financial position. Second, LTV may have been positioning itself to show improved performance once it emerged from Chapter 11. By taking the $2.26 billion charge now, not only does LTV avoid having to reduce its earnings in the future, but it also reduces the company's earnings so much that its earnings can only increase next year. This latter strategy is known as "taking a bath."

ID10–12 The conservatism ratio is an excellent measure of a company’s aggressiveness in its tax strategy. The lower the ratio, the more conservative the company. The higher conservatism ratio indicates that management is more aggressive with its tax policies and results in higher future tax liabilities. The lower the ratio the less aggressive management is with tax policies. Conservatism Ratio

= Reported Income Before Taxes ÷ Taxable Income

2010 Taxable Income = $942 ÷ .294 = $3,204 Conservatism Ratio

= =

$2,879 ÷ $3,204 0.90

2011 Taxable Income

= =

Conservatism Ratio 2012 Taxable Income

Conservatism Ratio

$1,017 ÷ .310 $3,281 = =

= =

$3,631 ÷ $3,281 1.11

$1,277 ÷ .350 $3,649 = =

$3,115 ÷ $3,649 0.85


ID10–12 CONCLUDED The trends indicate that the company is becoming more conservative in its tax strategy (with a bump in aggressiveness in 2011).

ID10–13 a. What Mr. Healy really means is that GM earned $200 million in 1997 not by selling autos but by working through an accounting adjustment due to a change in the tax rates in its North American operations. b. Change in the expected tax rate can lead to a positive effect on reported earnings due to the fact that tax liability was accrued at a higher tax rate and eventually paid at a lower tax rate. c.

Not really. It seems that GM estimated its tax liability to be higher, and it had to pay much less lower taxes at the end of the year. It is akin to a change of an accounting estimate.

ID10–14 a. A patent gives the holder the exclusive rights to use an idea (or a technology) and prevents competitors from using the idea (or technology). Patents are considered intangible assets and are carried on the balance sheet. b. From Vonage’s perspective, the lawsuits represent contingent losses. Depending on the evaluation (by Vonage management and legal counsel) of the merits of the lawsuits, the company can ignore the lawsuits in the financial statements (if loss is deemed remotely likely), disclose the lawsuit in the footnotes (if loss is deemed reasonably likely) or accrue an expense and a matching contingent liability (if loss is deemed highly likely). An analysis of Vonage’s balance sheet, income statement and footnotes will determine how the company and its auditors treated the contingencies related to the patent lawsuits.

ID10–15 a. Provisions under IFRS are recorded more frequently than contingent liabilities under U.S. GAAP because under IFRS provisions are accrued when the obligation is “more likely than not,” but GAAP uses a more stringent requirement of “highly probable”. Additionally, with IFRS the long term portion of any provisions are valued at the present value of the future expected cash flows; GAAP, on the other hand, does not apply any discounting when valuing contingent liabilities. Finally, when there is a range of possible outcomes associated with the liability, U.S. GAAP uses the lowest possible value, while IFRS employs the “best estimate”. b. When VW increased its provision account, it booked an expense (reducing profitability and equity) and a liability. The increase may include items such as warranties, restructurings and environmental issues. c. When VW reduced its provision account, it settled obligations by paying cash (reducing assets) and reducing the amount of its liabilities. Severance payments, costs incurred to honor automobile warranties, and costs for environmental clean-up are examples that may be included in VW’s number.


ID10–16 a. Working capital is the excess of current assets over current liabilities. It is one measure of the solvency of a company. Google’s working capital was $43,845 million in 2011 and $46,117 million in 2012, showing a strong increase. b. In 2011 working capital was 60.4% of total assets, decreasing to 49.2% in 2012. c.

Google’s current ratio was 5.92 in 2011 and decreased to 4.22 in 2012. The largest factors in the drop are the increases in accounts payable and accrued expenses. Both could be related to the acquisition of Motorola’s mobile hardware business.

d. The most important current liabilities are the two outlined above (accounts payable and accrued expenses) along with short term debt. e. Accounts payable turnover = COGS ÷ (Average accounts payable) 2012 X = $20,634 ÷ (($2,012 + $588) ÷ 2) X = 15.87* *dividing turnover into 365 days yields 23.0 days on hand for accounts payable 2011 X = $13,188 ÷ (($588 + $483) ÷ 2) X = 24.63* *dividing turnover into 365 days yields 14.8 days on hand for accounts payable The company is paying its suppliers 8.2 days slower, but the company’s business model changed to include a Motorola manufacturing component in 2012, possibly explaining the slowdown. f. Per footnote #11, the company is involved in antitrust, environmental and intellectual property legal issues, but none of the proceedings pose a material risk to Google’s financial condition. g. The largest accrued liabilities are for compensation and benefits. An accrued liability occurs when the company records an expense with a debit and an obligation (a liability) to settle the charges at a later date with a credit. h. Per footnote #13, company employees do not have a pension plan but may participate in a 401(k) savings program, with the company matching contributions under certain circumstances. Google contributions were $180 million, $136 million and $100 million in years 2012, 2011, and 2010, respectively.


ID10–16 Concluded i. Conservatism Ratio = Reported Income Before Taxes ÷ Taxable Income 2012

2011 12,326

2010

Reported income before tax

13,386

10,796

÷ Taxable income

14,799

10,695

10,764

= Conservatism ratio

0.90

1.15

1.00

Current year tax liability*

2,871

2,246

2,282

÷ Effective tax rate

19.4%

21.0%

21.2%

= Taxable Income

$14,799

$10,695

$10,764

*footnote #14 The tax strategy of Google was more aggressive fom 2010 to 2011, but then more conservative from 2011 to 2012.


LONG-TERM LIABILITIES: NOTES, BONDS, AND LEASES BRIEF EXERCISES BE11–1 a. During 2012 Radio Shack paid down $88.1 million of notes payable (long-term debt) and borrowed $175.0 million of other long-term debt. Further, the reduction in the unamortized discount reflects the fact that interest expense was booked at the effective rate while cash paid for interest was calculated at the stated rate of interest. When the debt obligations were repaid, the balance sheet and the statement of cash flows were adjusted. Interest expense affects the income statement. b. Assuming the discount applies to the 6.75% notes and assuming that the debt reduction occurred toward the end of the year, during 2012 approximately $9.375 million of interest expense would be recognized on the 2.5% notes (2.5% x $375 million). c.

If Radio Shack paid $300 million to retire the notes in 2012 the company would have recorded a gain of $25.0 million on the transaction. ($325.0 – $300.0). This amount would be found on the income statement as a gain on the early extinguishment of debt.

BE11–2 a. The life of these bonds is 20 years, from 1997 until 2017. b. The stated interest rate is 0%. c.

The effective rate is 3.2%. (The present value is $968 million, while the future value of the single sum in 20 years is $1.8 billion.)

d. Bonds typically are issued in amounts of $1,000, therefore Hewlett-Packard issued 1.8 million bonds with a total face value of $1.8 billion.

BE11–3 a. The operating lease payments reduced the reported income for the period, reduced the assets on the balance sheet (payment of cash), and impacted the statement of cash flows by reducing the net income of SuperValu for the reporting period. By reducing net income these lease payments are a use of cash from operations. b. The interest portion of capital lease payments and the depreciation on the capitalized lease assets reduce reported net income for the period. They also impact the balance sheet by reducing both the assets and the liabilities. For a capital lease the interest payments is cash from operations, principal reductions is financing and the depreciation has no impact on the statement of cash flows. c.

The $120 million of operating leases is a form of off-balance sheet financing for SuperValu. Supervalu does not have to show these future liabilities on its balance sheet. These future contractual payments are disclosed in the footnotes. One of the advantages of this approach is that these liabilities are not used in the calculation of financial ratios.

1


EXERCISES E11–1 a. Melrose Enterprises' debt/equity ratio is currently 1.25 [($200,000 + $300,000) ÷ $400,000]. The company's loan agreement specifies that debt can be twice the stockholders' equity. Consequently, the company's debt cannot exceed $800,000. Since Melrose Enterprises already has $500,000 in debt, the company can borrow an additional $300,000. b. By definition, Melrose Enterprises will settle its December 31, 2014 current liabilities sometime during 2015. The company will probably also incur new current liabilities as of December 31, 2015. Since no information is provided as to the expected amount of current liabilities as of December 31, 2015, a reasonable assumption is that these liabilities will remain at $200,000. Consequently, Melrose Enterprises would have total debt of $500,000 and total stockholders' equity of $550,000 ($400,000 + $950,000 in revenues – $800,000 in expenses). The company could now borrow a total of $1,100,000 ($550,000  2) without violating the debt covenant. Melrose Enterprises could, therefore, borrow an additional $600,000. c.

At the end of 2015, Melrose Enterprises would have $200,000 in current liabilities and $300,000 in long-term debt for total debt of $500,000, and it would have $450,000 in stockholders' equity ($400,000 + $950,000 in revenues – $800,000 in expenses – $100,000 in declared dividends). The company could now borrow a total of $900,000 (i.e., $450,000  2) without violating its debt covenant. Consequently, Melrose Enterprises could borrow an additional $400,000. If Melrose Enterprises declares, but does not pay, the $100,000 dividend, the company's debt/equity ratio will be affected. Dividends that are declared but not paid are typically classified as current liabilities. Consequently, Melrose Enterprises would have $300,000 in current liabilities and $300,000 in long-term liabilities for total liabilities of $600,000, and it would have $450,000 in stockholders' equity, which means that the company could borrow a total of $900,000 without violating its debt covenant. Consequently, Melrose Enterprises could borrow an additional $300,000. Declaring but not paying the dividend, as opposed to declaring and paying the dividend, reduced the amount of money that the company could borrow on a dollar-for-dollar basis.


E11–2 a. 1/1/14

1/1/15

1/1/16

1/1/17

1/1/18

1/1/19

$30,000

$30,000

$30,000

$30,000

$30,000 $300,000

b. All dollar amounts on the time line below are in thousands of dollars. 1/14

c.

1/15

7/15

1/16

7/16

1/17

7/17

1/18

7/18

1/19

$15

$15

$15

$15

$15

$15

$15

$15

$15

$15 $300

Total Present Value

= Present Value of Face Value + Present Value of Periodic Interest Payments

(1)

Annual interest payments: Total Present Value = ($300,000  Present Value Factor for i = 10% and n = 5) + ($30,000  Present Value Factor of an Ordinary Annuity Factor for i = 10% and n = 5) = ($300,000  .62092 from Table 4, Appendix A) + ($30,000  3.79079 from Table 5, Appendix A) = $186,276 + $113,724 = $300,000

(2)

Semiannual interest payments: Total Present Value = ($300,000  Present Value Factor for i = 5% and n = 10) + ($15,000  Present Value Factor of an Ordinary Annuity Factor 5% and n = 10) = ($300,000  .61391 from Table 4, Appendix A) + ($15,000  7.7218 from Table 5, Appendix A) = $184,173 + $115,827 = $300,000

E11–3 1 2 3 4

7/14

Par value Discount Premium Premium

for i =


E11–4 Present Value = Present Value of Face Value + Present Value of Interest Payments Note 1 Present Value

Note 2 Present Value

Note 3 Present Value

Note 4 Present Value

Note 5 Present Value

= ($1,000  Present Value Factor for i = 8% and n = 4) + [($1,000  0%)  Present Value of an Ordinary Annuity Factor for i = 8% and n = 4] = $1,000  .7350 (from Table 4 in Appendix A) + $0 = $735.00 = ($5,000  Present Value Factor for i = 6% and n = 6) + [($5,000  0%)  Present Value of an Ordinary Annuity Factor for i = 6% and n = 6] = $5,000  .7050 (from Table 4 in Appendix A) + $0 = $3,525.00 = ($8,000  Present Value Factor for i = 12% and n = 6) + [($8,000  4%)  Present Value of an Ordinary Annuity Factor for i = 12% and n = 6] = ($8,000  .5066 (from Table 4 in Appendix A) + ($320  4.1114 (from 5 in Appendix A) = $4,052.80 + $1,315.65 = $5,368.45 = ($3,000  Present Value Factor for i = 8 % and n = 7) + [($3,000  8%)  Present Value of an Ordinary Annuity Factor for i = 8% and n = 7] = ($3,000  .5835 (from Table 4 in Appendix A) + ($240  5.2064) (from Table 5 in Appendix A) = $1,750.50 + $1,249.54 = $3,000.00 = ($10,000  Present Value Factor for i = 6 % and n = 10) + [($10,000  10%)  Present Value of an Ordinary Annuity Factor for i = 6% and n = 10] = ($10,000  .5584 (from Table 4 in Appendix A) + ($1,000  7.3601 (from Table 5 in Appendix A) = $5,584.00 + $7,360.10 = $12,944.10

Table


E11–5 a.

Present Value $11,348

Present Value Factor

= Present Value of Face Value + Present Value of Interest Payments = ($20,000  Present Value Factor for i = ? and n = 5) + [($20,000  0%)  Present value of an ordinary annuity factor for i = ? and n = 5] = ($20,000  Present Value Factor) + $0 = 0.5674

Looking in the n = 5 row of Table 4 in Appendix A reveals that a present value factor of 0.5674 corresponds to an annual effective interest rate of 12%. b. Equipment (+A) ......................................................................................... Discount on Notes Payable (–L) ................................................................ Notes Payable (+L) .............................................................................. Purchased equipment by issuing a note. c.

Interest Expense

d. Balance Sheet Value

11,348 8,652 20,000

= Book Value of Debt  Effective Interest Rate = ($20,000 – $8,652)  12% = $1,361.76 = = =

Face Value of Note – Discount on Notes Payable $20,000.00 – ($8,652.00 – $1,361.76) $12,709.76

e. Interest expense is computed as the debt's book value times the effective interest rate. For a note issued at a discount, the book value will increase over time until the book value equals the face value immediately prior to the note maturing. Since the book value is greater at the beginning of Year 2 than it was at the beginning of Year 1, and the effective interest rate is constant, the interest expense recognized by Tradewell in the second year will be greater than the interest expense recognized in the first year. Intuitively this makes sense. During Year 1, Tradewell only "borrowed" $11,348. Although Tradewell has to compensate the creditor for using the creditor's money during Year 1, Tradewell did not make any such payment to the creditor during Year 1 because the stated rate on the note is 0%. Thus, the amount that Tradewell should have compensated the creditor (i.e., interest expense) is simply added on to what Tradewell "borrowed" from the creditor. During Year 2, therefore, Tradewell has to pay interest not only on the initial $11,348 it "borrowed," but also on the interest that it incurred, but did not pay, during Year 1. As proof, Interest Expense for Year 2

= $12,709.76 [from part (d)]  12% = $1,525.17

This amount exceeds the interest expense for Year 1 computed in part (c).


E11–5 f.

Concluded

Since the note has not yet matured, the same logic used in part (e) can be applied to this question. Consequently, the interest expense recognized by Tradewell in the third year will be greater than the interest expense recognized in the second year. As proof, Interest Expense for Year 3

= Book Value at Beginning of Year 3  12% = [$20,000.00 – ($8,652.00 – $1,361.76 – $1,525.17)]  12% = $1,708.19

This amount exceeds the interest expense for Year 2 computed in part (e).

E11–6 a. Stated interest rate = 8% Cash (+A) ................................................................................................... Notes Payable (+L) .............................................................................. Issued notes payable.

8,000 8,000

Interest Expense (E, –SE) ........................................................................... Cash (–A)............................................................................................. Incurred and paid interest.

640

Interest Expense (E, –SE) ........................................................................... Cash (–A)............................................................................................. Incurred and paid interest.

640

Notes Payable (–L)..................................................................................... Cash (–A)............................................................................................. Repaid notes payable.

8,000

b. Stated interest rate = 0% Face value ...................................................................................... Present value (i = 8%, n = 2) Present value of face value $8,000  .8573 (from Table 4 in Appendix A)......................... Discount on notes payable ............................................................

640

640

8,000

$ 8,000.00

6,858.40 $ 1,141.60

Cash (+A) ................................................................................................... Discount on Notes Payable (–L) ................................................................ Notes Payable (+L) .............................................................................. Issued notes payable.

6,858.40 1,141.60

Interest Expense (E, –SE) ........................................................................... Discount on Notes Payable (+L) .......................................................... Incurred interest.

548.67

Interest Expense (E, –SE) ........................................................................... Discount on Notes Payable (+L) .......................................................... Incurred interest.

592.93

8,000.00

548.67

592.93


E11–6

Concluded

Notes Payable (–L)..................................................................................... Cash (–A)............................................................................................. Repaid notes payable. c.

Stated interest rate = 6% Face value .................................................................................................. Present value (i = 8%, n = 2) Present value of face value $8,000  .8573 (from Table 4 in Appendix A) ................................... Present value of interest payments ($8,000  6%)  1.7833 (from Table 5 in Appendix A) ...................... Total present value.................................................................................... Discount on notes payable ........................................................................

8,000.00 8,000.00

$8,000.00

$6,858.40 855.98 $

Cash (+A) ................................................................................................... Discount on Notes Payable (–L) ................................................................ Notes Payable (+L) .............................................................................. Issued notes payable.

7,714.38 285.62

Interest Expense (E, –SE) ........................................................................... Discount on Notes Payable (+L) .......................................................... Cash (–A)............................................................................................. Incurred and paid interest.

617.15a

7,714.38 285.62

8,000.00

137.15b 480.00c

a $617.15 = Book Value  Effective Interest Rate = 7,714.38  8% b $137.15 = Interest Expense – Interest Payment c

$480.00 = Face Value  Stated Interest Rate = $8,000  6%

Interest expense (E, –SE) ........................................................................... Discount on Notes Payable (+L) .......................................................... Cash (–A)............................................................................................. Incurred and paid interest.

628.47a 148.47b 480.00

a $628.47 = Book Value  Effective Interest Rate = (7,714.38 + 137.15)  8% b $148.47 = $8,000.00 – [$7,714.38 + $137.15 (from prior entry)]

Notes Payable (–L)..................................................................................... Cash (–A)............................................................................................. Repaid notes payable.

8,000 8,000


E11–7 a. Present value = Present value of face value + Present value of periodic interest payments (1) Discount rate = 8% Present value of face value (i = 8%, n = 2) ($2,500  0.85734 from Table 4 in Appendix A) Present value of periodic interest payments (i = 8%, n = 2) ($200  1.78326 from Table 5 in Appendix A) Total present value (2)

(3)

Discount rate = 10% Present value of face value (i = 10%, n = 2) ($2,500  0.82645 from Table 4 in Appendix A) Present value of periodic interest payments (i = 10%, n = 2) ($200  1.73554 from Table 5 in Appendix A) Total present value Discount rate = 12% Present value of face value (i = 12%, n = 2) ($2,500  0.79719 from Table 4 in Appendix A) Present value of periodic interest payments (i = 12%, n = 2) ($200  1.69005 from Table 5 in Appendix A) Total present value

$ 2,143.35 356.65 $ 2,500.00

$ 2,066.13 347.11 $ 2,413.24

$ 1,992.98 338.01 $ 2,330.99

b. The effective interest rate is the interest rate that equates the undiscounted future cash flows with the present value of the future cash flows. In this case, the undiscounted future cash flows are (1) the $2,500 face value due in two years and (2) the interest payments of $200 due at the end of each year for two years, while the present value of the note is the proceeds of $2,413. From part (a), a discount rate of 10% equates the future cash flows and the proceeds. Therefore, the effective interest rate is 10%. c.

If Wilmes Floral Supplies originally borrowed $2,500, the $2,500 would be the present value of the future cash flows. From part (a), a discount rate of 8% equates the future cash flows with $2,500. The effective interest rate would, therefore, be 8%. Anytime the proceeds equal the face value, the effective interest rate equals the stated interest rate.

E11–8 a. The building should be capitalized at the cash value of the transaction. In this particular case, the cash value of the transaction would be assumed to equal the building's appraised value. Therefore, the building should be recorded at $550,125.


E11–8

Concluded

b. The total present value of a note equals the sum of the present value of the note's face value and the present value of the periodic interest payments specified in the note. Since the note signed by Morrow Enterprises is non-interest-bearing, there are no periodic interest payments, and the present value of the note would equal just the present value of the note's face value.

c.

(1)

Discount Rate Present Value

= = = =

6% ($693,000  Present Value Factor for i = 6% and n = 3) $693,000  0.83962 from Table 4 in Appendix A) $581,856.66

(2)

Discount Rate Present Value

= = = =

8% ($693,000  Present value factor for i = 8% and n = 3) $693,000  0.79383 from Table 4 in Appendix A) $550,124.19

(3)

Discount Rate Present Value

= = = =

10% ($693,000  Present value factor for i = 10% and n = 3) $693,000  0.75131 from Table 4 in Appendix A) $520,657.83

The effective interest rate is the rate that equates the undiscounted future cash flows with the present value of the future cash flows. In this case, the undiscounted future cash flow is the $693,000 face value due in three years, and the present value of the note is the value of the building, or $550,125. From part (b), a discount rate of 8% equates the future cash flows and the proceeds. Therefore, the effective interest rate is 8%.

d. Since the note is non-interest-bearing, the only cash flow is the face value of $693,000. Dividing the present value of $550,125 by the face value of $693,000 yields a present value factor of .79383. Looking across the n = 3 row of the present value of $1 table (i.e., Table 4) in Appendix A reveals that the annual effective interest rate on this note is 8%.

E11–9 a.

Interest Expense $16,400 Effective Interest Rate

= Effective Rate  Book Value of Debt at Beginning of the Period = Effective Rate  ($200,000 – $14,400) = 8.8% (rounded)

b. Interest Expense (E, –SE) ........................................................................... Discount on Notes Payable (+L) .......................................................... Cash (–A)............................................................................................. Incurred and paid interest. * $2,400 = Change in the balance of Discount on Notes Payable

16,400 2,400* 14,000


E11–10 a. The ten-year notes call for annual interest of $25.025 million (stated rate of 6.5% X face value of $385 million) and the repayment of $385 million in principal. The proceeds of the notes were $380 million. If the present value of the contract is $380 million and the future values are represented in the interest (ordinary annuity) and the principal (single sum), then the effective interest rate is the rate that discounts the future values to the present value of $380 million. The effective interest rate is approximately 6.7%. (The general present value formula of 1/[(1 + r) to the nth] was used in this calculation.) b. The interest expense for 2002 will be the effective rate of 6.7% multiplied by the proceeds of $380 million, or $25.46 million. This amount is made up of $25.025 million of cash paid plus $.435 million of non-cash interest from the amortization of the bond discount.

c.

The market paid less than $385 million for these bonds because the market demands 6.7% interest for their investment dollars for the risk posed by the company at the time of issuance. The notes only pay a cash interest rate of 6.5% and so the only way that investors can make their desired return is to pay less for the notes. This allows the investors to make the market rate of 6.7%.

E11–11 a. Bond A Face value ........................................................................................... Present value (i = 3%, n = 20) PV of face value ($100,000  0.55368 from Table 4 in Appendix A)........................ PV of periodic interest payments ($3,000 x 14.87747 from Table 5 in Appendix A) .......................... Total present value (i.e., proceeds) .................................................... Discount/premium ............................................................................. Bond B Face value ........................................................................................... Present value (i = 3%, n = 20) PV of face value ($400,000  0.55368 from Table 4 in Appendix A)........................ PV of periodic interest payments ($16,000  14.87747 from Table 5 in Appendix A)........................ Total present value (i.e., proceeds) .................................................... Premium ............................................................................................. Bond C Face value ........................................................................................... Present value (i = 4%, n = 10) PV of face value ($600,000  0.67556 from Table 4 in Appendix A)........................ PV of periodic interest payments ($18,000  8.11090 from Table 5 in Appendix A)..........................

$ 100,000

$ 55,368 44,632 $

100,000 0

$ 400,000

$ 221,472 238,040 459,512 $ 59,512

$ 600,000

$ 405,336 145,996


Total present value (i.e., proceeds) .................................................... Discount .............................................................................................

551,332 $ 48,668

E11–11 Concluded b. Immediately before a bond matures, its carrying value on the balance sheet must equal its face value. Thus, discounts and premiums must be amortized over time so that the carrying value approaches the bond's face value over time. For bonds that are issued at their face value, such as Bond A, the bond is already stated at its face value and there is no discount or premium to amortize. Consequently, the carrying value of the bond will remain equal to its face value over the life of the bond. Thus, the carrying value of Bond A will remain constant over its life. For bonds issued at a discount, such as Bond C, the carrying value on the date the bond is issued is less than its face value. Consequently, over the life of the bond, its carrying value must increase as the discount is amortized. Remember that a discount on a bond is deducted from the bond's face value to determine the carrying value. Thus, any reduction in the discount balance, such as when the discount is being amortized, will decrease the amount being deducted from the face value which thereby increases the carrying value of the bond. The carrying value of Bond C will, therefore, increase over its life. Alternatively, the carrying value of Bond B will decrease over its life. For bonds issued at a premium, such as Bond B, the carrying value on the date the bond is issued is greater than its face value. Consequently, over the life of the bond, its carrying value must decrease as the premium is amortized. Remember that a premium on a bond is added to the bond's face value to determine the carrying value. Thus, any reduction in the premium balance, such as when the premium is being amortized, will decrease the amount being added to the face value which thereby decreases the carrying value of the bond. Thus, the carrying value of Bond B will decrease over its life. c.

Interest expense is computed as the bond's book value at the beginning of the accounting period times the effective interest rate per period. Since accountants use the effective interest rate on the date a bond is issued to calculate interest expense, the effective interest rate is constant over the bond's life. This implies that the only factor that could affect whether the interest expense recognized each period increases, decreases, or remains constant over the life of the bond is the book value. As discussed in part (b), the book value of Bond A will remain constant over the life of the bond issue. Consequently, the interest expense recognized in each accounting period will remain constant over the life of Bond A. Alternatively, the interest expense recognized for Bonds B and C will vary across periods. Since the book value of Bond B will decrease over the life of the bond issue [see part (b)], interest expense associated with Bond B will also decrease from one period to the next. The interest expense associated with Bond C will increase from one period to the next because the book value of Bond C will increase each period [see part (b)].


E11–12 a. 1/1/14

b. 6/30/14

Cash (+A)............................................................................ Bonds Payable (+L)...................................................... Issued bonds for cash.

30,000

Interest Expense (E, –SE) ................................................... Cash (–A) ..................................................................... Incurred and paid interest.

1,500a

30,000

1,500b

a $1,500 = Book Value  Effective Interest Rate per Period = $30,000  5% b $1,500 = Face Value  Stated Interest Rate per Period = $30,000  5%

12/31/14

c.

Interest Expense (E, –SE) ................................................... Cash (–A) ..................................................................... Incurred and paid interest.

1,500 1,500

Balance Sheet Value

= Face Value – Associated Discount + Associated Premium = $30,000 – $0 + $0 = $30,000 d. Present value (i = 5%, n = 18) PV of face value ($30,000  .4155) ................................................................................. $12,465 PV of interest payments ($1,500  11.6896) ............................................................................... 17,534 Total present value.................................................................................... $30,000 e. Balance Sheet Value as of 12/31/15 = Face Value – Associated Discount + Associated Premium = $30,000 – $0 + $0 = $30,000 Present value as of 12/31/15 (i = 5%, n = 16) PV of face value ($30,000  .4581) ................................................................................. PV of interest payments ($1,500  10.8378) ............................................................................... Total present value....................................................................................

$13,743 16,257 $30,000

Notice that the balance sheet value of $30,000 is identical to the present value just calculated. Amortizing premiums and discounts using the effective interest rate results in bonds being carried on the balance sheet at an amount equal to the present value of the future cash flows of the bonds, using the effective interest rate on the date the bonds were issued as the discount rate.


E11–13 a. Face value ........................................................................................ Present value (i = 4%, n = 10) PV of face value ($500,000  0.6756 from Table 4 in Appendix A)....................... PV of interest payments ($15,000  8.1109 from Table 5 in Appendix A)......................... Total present value.......................................................................... Discount ..........................................................................................

$ 500,000

$ 337,800 121,664 $

Cash (+A) ................................................................................................. Discount on Bonds Payable (–L) .............................................................. Bonds Payable (+L) ........................................................................... Issued bonds.

459,464 40,536

459,464 40,536 500,000

18,378.56a

b. Interest Expense (E, –SE) ......................................................................... Discount on Bonds Payable (+L) ....................................................... Interest Payable (+L) ......................................................................... Accrued interest payable.

3,378.56c 15,000.00b

a $18,378.56 = Book Value  Effective Rate per Period = $459,464  4% b $15,000.00 = Face Value  Stated Rate per Period = $500,000  3% c

c.

$3,378.56 = $18,378.56 – $15,000.00

Balance sheet value as of 12/31/15

= = =

Face value – Discount as of 12/31/15 $500,000.00 – ($40,536.00 – $3,378.56) $462,842.56

d. Present value (i = 4%, n = 9) PV of face value ($500,000  0.7026 from Table 4 in Appendix A)............................... PV of interest payments ($15,000  7.4353 from Table 5 in Appendix A)................................. Total present value..................................................................................

$ 351,300.00 111,530.00 $ 462,830.00

Notice that the $462,842.56 from part [c] is essentially identical to the $462,830.00 just calculated. Amortizing discounts and premiums using the effective interest rate results in bonds being carried on the balance sheet at an amount equal to the present value of the bonds, using the effective interest rate on the date the bonds were issued as the discount rate.

E11–14 a. Face value ........................................................................................ Present value (i = 3%, n = 20) PV of face value ($100,000  0.55368 from Table 4 in Appendix A)..................... PV of interest payments ($4,000  14.87747 from Table 5 in Appendix A).......................

$ 100,000

$

55,368 59,510


Total present value.......................................................................... Premium ..........................................................................................

E11–14

$

114,878 14,878

Concluded

Cash (+A) ................................................................................................. Premium on Bonds Payable (+L)....................................................... Bonds Payable (+L) ........................................................................... Issued bonds.

114,878 14,878 100,000

3,446.34a 553.66c

b. Interest Expense (E, –SE) ......................................................................... Premium on Bonds Payable (-L) .............................................................. Interest Payable (+L) ......................................................................... Accrued interest payable.

4,000.00b

a $3,446.34 = Book Value  Effective Rate per Period = $114,878  3% b $4,000.00 = Face Value  Stated Rate per Period = $100,000  4% c

c.

$553.66 = $4,000.00 – $3,446.34

Balance sheet value as of 12/31/15

= = =

Face value + Premium as of 12/31/15 $100,000.00 + ($14,878.00 – $553.66) $114,324.34

d. Present value (i = 3%, n = 19) PV of face value ($100,000  0.57029 from Table 4 in Appendix A)............................. PV of interest payments ($4,000  14.3238 from Table 5 in Appendix A)................................. Total present value..................................................................................

$

57,029.00

57,295.20 $ 114,324.20

Notice that the $114,324.34 from part [c] is essentially identical to the $114,324.20 just calculated. Amortizing discounts and premiums using the effective interest rate results in bonds being carried on the balance sheet at an amount equal to the present value of the bonds, using the effective interest rate on the date the bonds were issued as the discount rate.

E11–15 a. Since it is one year later, two interest periods have passed. Thus, there are only eight remaining interest periods. Present value (i = 3%, n = 8) PV of face value ($20,000  0.7894 from Table 4 in Appendix A)................................ PV of interest payments ($800  7.0197 from Table 5 in Appendix A)..................................... Total present value.................................................................................

$15,788.00 5,615.76 $21,403.76

To determine whether Treadway has experienced an economic gain or loss, we need to know both the market value and the carrying value of the bonds. The market value of the bonds on December 31, 2014 should equal the present value of the bond's future cash flows discounted using the prevailing market interest rate. Thus, the market value of Treadway's bonds on December 31, 2014 is $21,403.76.


Since the bonds were issued at face value, the effective interest rate on the date of issue equaled the stated interest rate, and there was no discount or premium associated with the bonds. When bonds are issued at face value, the bonds are carried on the books at face value until the bonds mature. Consequently, the book value of these bonds is $20,000.00. Since the market value of the bonds now exceeds $20,000.00, Treadway has experienced an economic loss. That is, if Treadway wanted to retire the bonds, it

E11–15 Concluded would cost the company $21,403.76 rather than $20,000.00. The amount of the loss is the excess of the bond's market value over the bond's book value, or $1,403.76. b. Present value (i = 5%, n = 8) PV of face value ($20,000  0.6768 from Table 4 in Appendix A)................................... PV of interest payments ($800  6.4632 from Table 5 in Appendix A)........................................ Total present value....................................................................................

$13,536.00 5,170.56 $18,706.56

The market value of the bonds is now less than their book value. If Treadway Company wanted to retire the bonds through the bond market, it would have to pay less than the value of the bonds per the company's financial records. Therefore, the effective liability of the company has decreased, which implies that the company has experienced an economic gain. The gain would be the excess of the bonds' book value over the bonds' market value, or $1,293.44. c.

Companies experience economic gains and losses when their wealth changes. In the case of bonds, their market value indicates the company's effective obligation on the bonds at that particular point in time. If the market value exceeds the bonds' book value, then the company has experienced a decrease in wealth; if the market value is less than book value, then the company has experienced an increase in wealth. Such gains and losses, however, are not usually reflected in a company's financial statements because it is assumed that when a company issues bonds, the bonds will remain outstanding until they mature. That is, the company will not retire the bonds before they mature. On the date that the company issued the bonds, the company locked into the market rate of interest on that day. This means that the effective interest rate on the date the bonds are issued is the interest rate the company expects to incur over the life of the bonds. Thus, the economic gains and losses due to fluctuations in the market interest rate will essentially "wash out" over the life of the bonds because the bonds are expected to remain outstanding until they mature. Economic gains or losses associated with changes in the market interest rate are only reflected in a company's financial statements when a company retires some bonds prior to their maturity date. If you were analyzing Treadway's financial statements, you might want to adjust the amounts reported for notes and bonds to reflect the prevailing market interest rate. In this manner, the statements would more accurately reflect the company's economic liability—and hence associated gains and losses— compared to the amounts reported on the balance sheet.


E11–16 a. Cash paid to redeem the bonds

= = =

Face value  101% $500,000  101% $505,000

Bonds Payable (–L) .................................................................................... Loss on Redemption (Lo, –SE) ................................................................... Discount on Bonds Payable (+L) ......................................................... Cash (-A) ............................................................................................. Redeemed bonds.

500,000 15,000 10,000* 505,000

* $10,000 = Face Value – Book Value = $500,000 – $490,000 b. Bonds Payable (–L) .................................................................................... Premium on Bonds Payable (–L) ............................................................... Cash (–A)............................................................................................. Gain on Redemption (Ga, +SE) ........................................................... Redeemed bonds.

500,000 7,000 505,000 2,000

E11–17 a. Lilly paid $47 million to retire this debt. This is comprised of $35 million (book value of bonds), $7.2 million (after tax loss) plus $4.8 million (tax benefit of loss). b. The $4.8 million is a benefit to the company because the loss offsets against the operating income of Lilly. As a result it reduces the amount of tax that Lilly will have to pay. Therefore the loss on retiring this debt provides a tax benefit to Lilly. c.

The loss would be shown as part of the net income that is shown on the first line item on the statement of cash flows. It would be added back to compute cash from operating activities.

d. The loss would be shown as an extraordinary item on the income statement.

E11–18


a. American Greetings paid cash of $181.2 million to retire the debt (book value of $142.2 million plus loss on debt repurchase of $39.0 million). b. The company, in effect, paid extra to make the debt go away. The debt was on the books for $142.2 million, but the company—through negotiations with the holders of the debt—agreed to pay a total of $181.2 million to retire the obligations. The additional amount paid, above the book value of the debt, is the amount of the loss recorded on the income statement. c.

American Greetings refinanced the debt at much lower interest rates. A company would be willing to incur a loss on its income statement if it can refinance debt at much lower rates. Over the life of the new debt the company figures it will more than make up for the amount booked on the income statement as a loss by lowering its interest expense in all the years that the debt is outstanding.

E11–19 a. Interest Expense (E, –SE) ........................................................................... Cash (–A)............................................................................................. Discount on Bonds (+L) ....................................................................... Incurred and paid interest.

4,822.70a 4,000.00b 822.70

a $4,822.70 = Book Value  Effective Interest Rate per Period = $96,454  5% b $4,000.00 = Face Value  Stated Interest Rate per Period = $100,000  4%

b. Bonds Payable (–L) .................................................................................... Cash (–A)............................................................................................. Discount on Bonds Payable (+L) ......................................................... Gain on Retirement of Bonds Payable (Ga, +SE) ................................ Retired bonds.

100,000.00

* $2,723.30 = $3,546 Discount balance as of 12/31/14 – $822.70 Discount from 1/1/15 to 7/1/15 [from part (a)]

91,700.00 2,723.30* 5,576.70

amortized

E11–20 a. The effective interest rate can be calculated in two ways. The first way is by solving for i in the following equation where n=2 since there are two periods until maturity (12/31/14, the balance sheet date and maturity at 12/31/16). $94,650 = [($100,000  (1 + i )-2] + {$5,000  [(1 – [(1 + i)-2 + i ]} The second way is by trial and error. Simply plug an interest rate into the equation above until the right-hand side of the equation equals the left hand side. Since the bond is issued at a discount, we start with the knowledge that the effective rate is greater than the stated rate of 5%. The annual effective interest rate for the bonds is 8%. b. To determine the effective rate an investor would be earning if the bonds were purchased on 12/31/14 at the market value of $98,167, perform the same procedure using the equation. $98,167 = [($100,000  (1 + i )-2] + {$5,000  [(1 – [(1 + i)-2 + i ]}


The annual effective interest rate for the bonds is 6%. c.

The book value of the bonds on Beasley Brothers’ books at December 31, 2014, is $94,650. The market value of the bonds as of December 31, 2014, is $98,167. The difference represents a loss of $3,517. It is a loss because if Beasley Brothers were to repurchase these bonds on the market in order to retire them, it would have to pay $3,517 more than the book value.

Net income Unrealized holding loss on Bonds Payable Adjusted net income

$27,000 (3,517) $23,483

The loss is not a decrease in the wealth of the company if it intends to keep the bonds outstanding until maturity. However, if Beasley intends to retire this debt, the loss represents a decrease in wealth because Beasley will sacrifice net assets of $3,517 to do so. d. Extraordinary Realized Loss on Retirement of Debt .............................. Bonds Payable ........................................................................................ Discount on Bonds Payable ............................................................. Cash .................................................................................................

3,517 100,000 5,350 98,167

Once the repurchase has occurred, the loss has been realized because a reduction in net assets has occurred. Before this occurs, however, the loss is unrealized and may be misleading if the company intends to keep the bonds until maturity.

E11–21 a. The effective interest rate can be calculated in two ways. The first way is by solving for i in the following equation where n=2 since there are two periods until maturity (12/31/15, the balance sheet date and maturity at 12/31/17). $193,059 = [($200,000  (1 + i )-2] + {$10,000  [(1 – [(1 + i)-2 + i ]} The second way is by trial and error. Simply plug an interest rate into the equation above until the right-hand side of the equation equals the left hand side. Since the bond is issued at a discount, we start with the knowledge that the effective rate is greater than the stated rate of 5%. The annual effective interest rate for the bonds is 6.92%. b. To determine the effective rate an investor would be earning if the bonds were purchased on 12/31/15 at the market value of $186,479, perform the same procedure using the equation. $186,479 = [($200,000  (1 + i )-2] + {$10,000  [(1 – [(1 + i)-2 + i ]} The annual effective interest rate for the bonds is 8.83%. c.

The book value of the bonds on Cohort Enterprises’ books at December 31, 2015 is $193,059. The market value of the bonds as of December 31, 2015 is $186,479. The difference represents a gain of $6,580. It is a gain because if Cohort Enterprises were to repurchase these bonds on the market in order to retire them, it would have to pay $6,580 less than the book value.


Net income Unrealized holding gain on Bonds Payable Adjusted net income

$38,500 6,580 $45,080

The gain is not an increase in the wealth of the company if it intends to keep the bonds outstanding until maturity. However, if Cohort intends to retire this debt, the gain represents an increase in wealth because Cohort will acquire additional net assets of $6,580.

d. Bonds payable .......................................................................................... Discount on Bonds Payable ................................................................ Cash .................................................................................................... Extraordinary Realized Gain on Retirement of Debt ..........................

200,000 6,941 186,479 6,580

Once the repurchase has occurred, the gain has been realized because an increase in net assets has occurred. Before this occurs, however, the gain is unrealized and may be misleading if the company intends to keep the bonds until maturity.

E11–22 a. Lease Expense (E, –SE) .............................................................................. Cash (–A)............................................................................................. Incurred and paid lease expense for 2014.

10,000

Lease Expense (E, –SE) .............................................................................. Cash (–A)............................................................................................. Incurred and paid lease expense for 2015.

10,000

Lease Expense (E, –SE) .............................................................................. Cash (–A)............................................................................................. Incurred and paid lease expense for 2016.

10,000

Lease Expense (E, –SE) .............................................................................. Cash (–A)............................................................................................. Incurred and paid lease expense for 2017.

10,000

Lease Expense (E, –SE) .............................................................................. Cash (–A)............................................................................................. Incurred and paid lease expense for 2018.

10,000

10,000

10,000

10,000

10,000

10,000

b. The effective interest rate on the lease is 8%. The following entries would be recorded on Q-Mart’s books. Facility (A) .................................................................................................. Lease Liability (L) ................................................................................ Record the capitalized lease. ($10,000*3.9927)

39,927 39,927


c.

Depreciation Expense (E, –SE) ................................................................... Accumulated Depreciation (–A) ......................................................... Record depreciation of capitalized asset for 2014. ($39,927/5)

7,985.40

Lease Liability (-L) ...................................................................................... Interest Expense (E, -SE)............................................................................ Cash (-A) ............................................................................................. Made lease payment for 2014.

6,805.84 3,194.16

Depreciation Expense (E, –SE) ................................................................... Accumulated Depreciation (–A) ......................................................... Record depreciation of capitalized asset for 2015.

7,985.40

Lease Liability (-L) ...................................................................................... Interest Expense (E, -SE)............................................................................ Cash (-A) ............................................................................................. Made lease payment for 2015.

7,350.32 2,649.68

Depreciation Expense (E, –SE) ................................................................... Accumulated Depreciation (–A) ......................................................... Record depreciation of capitalized asset for 2016.

7,985.40

Lease Liability (-L) ...................................................................................... Interest Expense (E, -SE) ..................................................................... Cash (-A) ............................................................................................. Made lease payment for 2016.

7,938.32 2,061.68

Depreciation Expense (E, –SE) ................................................................... Accumulated Depreciation (–A) ......................................................... Record depreciation of capitalized asset for 2017.

7,985.40

Lease Liability (-L) ...................................................................................... Interest Expense (E, -SE)............................................................................ Cash (-A) ............................................................................................. Made lease payment for 2017.

8,573.36 1,426.64

Depreciation Expense (E, –SE) ................................................................... Accumulated Depreciation (–A) ......................................................... Record depreciation of capitalized asset for 2018.

7,985.40

Lease Liability (-L) ...................................................................................... Interest Expense (E, -SE)............................................................................ Cash (-A) ............................................................................................. Made lease payment for 2018.

9,260.00 740.00

7,985.40

10,000

7,985.40

10,000

7,985.40

10,000

7,985.40

10,000

7,985.40

10,000

Classifying the lease as an operating lease would give rise to both higher net income and a lower debt/equity ratio. By classifying the lease as an operating lease, net income would be reduced during 2014 by $10,000 [from part (a)] for rent expense. Alternatively, classifying the lease as a capital lease


would reduce net income by a total of $11,179.56 [from part (b)] for the interest expense associated with the lease and for the depreciation associated with the capitalized asset. Future obligations under operating leases are not disclosed in a company's financial statements as a liability. Consequently, an operating lease would not affect a company's total liabilities. On the other hand, the present values of future lease obligations are reported as liabilities under capital leases, which means that a capital lease results in increased liabilities compared to an operating lease. In addition, the differential effect of capital and operating leases on net income will affect total stockholders' equity through Retained Earnings. Specifically, the balance in Retained Earnings, and thus total stockholders' equity, will be higher by classifying the lease as an operating lease as opposed to classifying it as a capital lease. Therefore, QMart’s total liabilities would be lower and its stockholders' equity higher if the lease were classified as an operating lease rather than as a capital lease. This means that classifying the lease as an operating lease would yield a lower debt/equity ratio. At the end of the useful life both will be equal.

E11–23 = Rental Expense per Car  Number of Cars = $10,000  100 cars = $1,000,000

a. Annual Rental Expense

= $10,000 per Car  100 Cars x Present Value of an Ordinary Annuity Factor for i = 10%, n = 5 = $1,000,000  3.7908 from Table 5 in Appendix A = $3,790,800 Automobiles (+A).................................................................................... 3,790,800 Lease Liability (+L) ........................................................................... 3,790,800 Leased automobiles.

b. Present Value of Lease Payments

c.

Interest Expense

= Lease Obligation  10% = $3,790,800  10% = $379,080

Depreciation Expense

= = =

Cost of Automobiles ÷ 5 Years $3,790,800 ÷ 5 $758,160

Total Rental Expense

= Interest Expense + Depreciation Expense = $379,080 + $758,160 = $1,137,240

d. Classifying the lease as an operating lease would give rise to both higher net income and a lower debt/equity ratio. By classifying the lease as an operating lease, net income would be reduced during 2014 by $1,000,000 [from part (a)] for rent expense. Alternatively, classifying the lease as a capital lease would reduce net income by a total of $1,137,240 [from part (c)] for the interest expense associated with the lease and for the depreciation associated with the capitalized asset. Future obligations under operating leases are not disclosed in a company's financial statements as a liability. Consequently, an operating lease would not affect a company's total liabilities. On the other hand, the present values of future lease obligations are reported as liabilities under capital leases, which means that a capital lease results in increased liabilities compared to an operating lease. In


addition, the differential effect of capital and operating leases on net income will affect total stockholders' equity through Retained Earnings. Specifically, the balance in Retained Earnings, and thus total stockholders' equity, will be higher by classifying the lease as an operating lease as opposed to classifying it as a capital lease. Therefore, Tradeall's total liabilities would be lower and its stockholders' equity higher if the lease were classified as an operating lease rather than as a capital lease. This means that classifying the lease as an operating lease would yield a lower debt/equity ratio.


E11–23 Concluded e. Off-balance sheet financing refers to financing agreements that require future payments, yet are structured so that the financing arrangement does not meet any of the criteria for the financing arrangement to be reported as a liability. The substance of an operating lease is considered to be a rental agreement. This implies that a company does not incur an obligation under the lease until it actually uses the item being leased. Thus, the future obligations under the lease should not be reported as a liability. Alternatively, the substance of a capital lease is considered to be a purchase agreement. This implies that the company has, in substance, acquired an asset and that the lease is simply a note payable for the acquisition of that asset. Thus, the present value of the future cash outflows specified in the lease agreement should be reported as a liability. The difference between operating and capital leases provides a way for companies to engage in off-balance-sheet financing. That is, by structuring a lease agreement as an operating lease, the lessee can engage in off-balance sheet financing.

E11–24 a. Since the face value of the bank loan equals the proceeds of the loan (i.e., $149,388), the effective interest rate is equal to the stated interest rate. Therefore, the appropriate effective interest rate for Watts Motors for a ten-year borrowing arrangement is 12%. This rate should also be used for the lease. The annual lease payments would be an ordinary annuity for i = 12% and n = 10. Setting up the following formula and solving for the payment amount gives us the annual lease payment that would equate the two financing options. $149,388 = Lease Payment  Present Value of an Ordinary Annuity Factor for i = and n = 10 $149,388 = Lease Payment  5.65022 (from Table 5 in Appendix A) Lease payment = $26,439.32 (rounded)

12%

b. With the lease payment, Watts Motors would pay $26,439.32 at the end of each year for ten years. With the bank loan, Watts Motors would make interest payments of $17,926.56 ($149,388  12%) at the end of each year for ten years and a payment of $149,388 at the end of Year 10. The essential difference between the two financing arrangements is that a portion of every lease payment is applied against the outstanding principal balance while the annual payments under the bank loan do not reduce the principal balance. c.

Option 1 Building (+A) .............................................................................................. Notes Payable (+L) .............................................................................. Purchased a building. Option 2 Assets Acquired Under Capital Leases (+A) ............................................... Obligations Under Capital Leases (+L) ................................................ Acquired a building under a capital lease.

149,388 149,388

149,388 149,388

Option 3 Under the operating lease, the building would not be capitalized. Instead, on every lease payment date, Watts Motors would debit Lease Expense or Rent Expense for $26,439 and credit Cash for the same amount.


E11–24 Concluded d.

Interest Expensea

Payment $26,439.32 26,439.32

$17,926.56 16,905.03

Principal Reductionb

Principal $149,388.00 140,875.24 131,340.95

$8,512.76 9,534.29

a Interest Expense = Principal  Effective Interest Rate of 12% b Principal Reduction = Payment – Interest Expense

e. Present Value

= $26,439.32  Present Value of an Ordinary Annuity Factor for i = 12% and n = 8 = $26,439.32  4.96764 (from Table 5 in Appendix A) = $131,341.02

The present value of the future lease payments equals the amount reported on the balance sheet calculated in part (d).

E11–25 Present Value

= Present Value of Face Value + Present Value of Interest Payment = (Face Value  Present Value Factor) + (Periodic Interest Payment  Present Value of an Ordinary Annuity Factor)

Note 1 Since the proceeds (i.e., present value) equal the face value, we know that the effective rate equals the stated rate. Consequently, the effective rate for Note 1 is 8%. As proof: Present value (i = 8%, n = 6) PV of face value ($10,000  .63017 from Table 4 in Appendix A).............................. PV of interest payments [($10,000  8%)  4.62288 from Table 5 in Appendix A] ................ Total present value (i.e., proceeds) ...................................................... Note 2 $35,056 PV Factor PV Factor

$

6,301.70

3,698.30 $ 10,000.00

= ($100,000  Present Value Factor) + [($100,000  0%)  Present Value of an Ordinary Annuity Factor] PV factor = $35,056 ÷ $100,000 = .35056

Examining Table 4 in Appendix A (i.e., present value of $1 table) for n = 8, we find that the effective rate is 14%.


E11–25 Concluded Note 3 $922 = ($1,000  Present Value Factor) + [($1,000  7%)  Present Value of an Ordinary Annuity Factor] Since the proceeds (i.e., present value) are less than the face value, we know that the note was issued at a discount. Consequently, the effective rate must be more than the stated rate. Try i = 9% for n = 5: ($1,000  .64993 from Table 4 in Appendix A) + [($1,000  7%)  3.88965 from Table 5 in Appendix A)] = $649.93 + $272.28 = $922 (rounded) Therefore, the annual effective interest rate must be 9%. Bond 1 $11,635 = ($10,000  Present Value Factor) + [($10,000  3%)  Present Value of an Ordinary Annuity Factor] Since the proceeds are greater than the face value, we know that the bond was issued at a premium. Consequently, the effective rate is less than the stated rate. Try i = 2% for n = 20: ($10,000  .67297 from Table 4 in Appendix A) + [($10,000  3%)  16.35143 from Table 5 in Appendix A] = $6,729.70 + $4,905.43 = $11,635 (rounded) The effective rate per period is 2%. Since there are two interest periods per year, the annual effective interest rate is 4%. Bond 2 $54,323 = ($50,000  Present Value Factor) + [($50,000  4.5%)  Present Value of an Ordinary Annuity Factor] Since the proceeds (i.e., present value) are greater than the face value, we know that the bond was issued at a premium. Consequently, the effective rate must be less than the stated rate. Try i = 4% for n = 30: ($50,000  .30832 from Table 4 in Appendix A) + [($50,000  4.5%)  17.29203 from Table 5 in Appendix A] = $15,416.00 + $38,907.07 = $54,323 (rounded) The effective rate per period is 4%. Since there are two interest periods per year, the annual effective interest rate is 8%.


E11–26 a. Since the bonds have a face value of $1,000 and they are selling for 89.16, an individual bond would have a present value of $891.60 ($1,000 x 89.16%). For these bonds to be attractive to an investor who requires an annual rate of return of 12%, the present value of the bonds' future cash flows discounted using a discount rate of 6% semiannually must be greater than or equal to $891.60. If the present value is less, the bonds would not provide an annual rate of return of at least 12%. The present value of the bonds using a discount rate per six-month period of 6% is: Present value (i = 6%, n = 16) PV of maturity receipt ($1,000  .39365) ............................................................................ PV of interest receipts [($1,000  4%)  10.10590)............................................................. Total present value....................................................................................

$ 393.65 404.24 $ 797.89

Since the present value of the future cash flows is less than $891.60, the bonds are providing an annual rate of return of less than 12%. Thus, the bonds do not provide the required rate of return, and they should not be considered for investment purposes. b. The annual effective interest rate that would make an investor indifferent to purchasing the bonds at 89.16 would be 10%, which implies a six-month rate of 5%. As proof: Present value (i = 5%, n = 16) PV of maturity receipt ($1,000  .45811) ............................................................................ PV of interest receipts [($1,000  4%)  10.83777)............................................................. Total present value (i.e., proceeds) ...........................................................

$ 458.11 433.51 $ 891.62

At an annual effective rate of 10%, an investor would be indifferent to purchasing the bonds.

E11–27 a. Bonneville issued the debt at a fixed rate, meaning that its outlay for interest will not change even if market interest rates change. A drop in market rates would not lower the interest paid by Bonneville; therefore, such a drop would increase the liability on the books of the company. In effect, the company would be paying above-market rates for its debt due to the fixed nature of the contract. b. Bonneville could manage the risk of fluctuating interest rates (and debt values) by entering into a hedging agreement known as an interest rate swap. Under a swap arrangement, Bonneville would agree with a counter party, most likely a large commercial bank, to receive periodic payments at a fixed rate of interest while also agreeing to pay out periodic payments that are tied to a floating rate of interest (Treasury Bills, for example). If market rates increase over the time period, Bonneville will make larger periodic payments under the swap agreement, while a drop in market rates will reduce the amount that Bonneville remits in periodic interest payments; regardless of changes in market rates, Bonneville will receive the same fixed payment from the counter party.


The fixed payment received under the swap agreement will match what the stated rate of interest requires Bonneville to pay in the long term debt contract. The effect of the interest rate swap is to hedge against changes in debt values due to changes in market rates; the variable interest payments from the swap agreement effectively eliminate those market value changes.

PROBLEMS P11–1 a. The present value of the future cash flows of this note equals $20,000. Since the effective rate of 10% equals the stated rate of 10%, the note will be issued at par value. Consequently, the face value of this note would be $20,000. b. The present value of the future cash flows of this note equals $20,000. Since the effective rate of 10% exceeds the stated rate of 0%, the note will be issued at a discount. The task is to determine what amount of cash paid at the end of two years discounted at 10% would equal $20,000 today. In other words, $20,000 $20,000 Face value

= (Face Value  Present Value Factor for i = 10%, n = 2) = Face Value  .8264 (from Table 4 in Appendix A) = $24,202 (rounded)

Consequently, the face value of this note would be $24,202. c.

Note A Cash (+A) ................................................................................................... Notes Payable (+L) .............................................................................. Issued note payable for cash. Note B Cash (+A) ................................................................................................... Discount on Notes Payable (–L) ................................................................ Notes Payable (+L) .............................................................................. Issued note payable for cash.

d. Note A Interest Expense (E, –SE) ........................................................................... Cash (–A)............................................................................................. Made interest payment. *

20,000 20,000

20,000 4,202 24,202

2,000* 2,000

$2,000 = $20,000  10%

Notes Payable (–L)..................................................................................... Cash (–A)............................................................................................. Made principal payment on note payable. Note B Interest Expense (E, –SE) ...........................................................................

20,000 20,000

1,782*


Discount on Notes Payable (+L) .......................................................... Amortized discount on notes payable. * $1,782

1,782

= $4,202 Initial discount – $2,420 discount amortized during 2015

Notes Payable (–L)..................................................................................... Cash (–A)............................................................................................. Made principal payment on note payable.

24,202 24,202

P11–2 a. The bonds will be issued at a discount. The bond market has determined that purchasers of Hartl Enterprises' bonds should earn an annual return on their investment of 10%. However, Hartl Enterprises is offering interest equal to only 8%. Since the stated interest rate cannot be changed, the only way that the investors can earn their 10% return is to invest a smaller amount in Hartl Enterprises. They will still receive the same future cash flows. Consequently, the bonds will be issued at a price that allows the investors to earn a return of exactly 10% on their investment. b. Face value ............................................................................................... Present value (i = 5%, n = 20) PV of maturity receipt ($10,000  .3769 from Table 4 in Appendix A) ............................ PV of interest receipts ($400  12.4622 from Table 5 in Appendix A) ............................. Total present value................................................................................. Discount ................................................................................................. Cash (+A) ................................................................................................... Discount on Bonds Payable (–L) ................................................................ Bonds Payable (+L) ............................................................................. Issued bonds. c.

Interest Expense (E, –SE) ........................................................................... Discount on Bonds Payable (+L) ......................................................... Interest Payable (+L) ........................................................................... Accrued interest payable. a $218.85 b $18.85 c $200.00

= = = = =

$ 10,000

$ 3,769 4,985 $

8,754 1,246

8,754 1,246 10,000

218.85a 18.85b 200.00c

Book value  Effective rate per period  Portion of period outstanding $8,754  5%  3/6 Interest expense – Interest payable Face value  Stated rate per period  Portion of period outstanding $10,000  4%  3/6

d. Interest Expense (E, –SE) ........................................................................... Interest Payable (–L).................................................................................. Discount on Bonds Payable (+L) ......................................................... Cash (–A)............................................................................................. Made interest payment.

218.85 200.00 18.85 400.00


P11–3 a. L-T Debt/Equity Ratio

= Total Long-Term Liabilities ÷ Total Stockholders' Equity = $40,000 ÷ $100,000 = 0.4

b. Proceeds = Present Value of Future Cash Flows Discounted at 11% for 5 Periods = $40,000  .59345 (from Table 4 in Appendix A) = $23,738 If Manheim Corporation borrows this $40,000, its long-term debt/equity ratio would be .637 [($40,000 + $23,738) ÷ $100,000]. c.

Proceeds = Present Value of Future Cash Flows Discounted at 4% for 40 Periods = Present Value of the Face Value + Present Value of Interest Payments = ($40,000  .20829 from Table 4 in Appendix A) + [($40,000  5%)  19.79277 (from Table 5 in Appendix A)] = $8,332 + $39,586 = $47,918 If Manheim Corporation issues these bonds, its long-term debt/equity ratio would be .879 [($40,000 + $47,918) ÷ $100,000].

P11–4 a. Note A Face value ........................................................................................... Present value (i = 10%, n = 5) PV of face value ($20,000  .6209 from Table 4 in Appendix A) ........................ PV of interest receipts ($0  3.7908 from Table 5 in Appendix A) ............................... Total present value (i.e., proceeds) .................................................... Discount ............................................................................................. Note B Face value ........................................................................................... Present value (i = 10%, n = 8) PV of face value ($35,000  .4665 from Table 4 in Appendix A) ........................ PV of interest receipts ($2,800  5.3349 from Table 5 in Appendix A) ........................ Total present value (i.e., proceeds) .................................................... Discount .............................................................................................

$ 20,000

$ 12,418 0 $

12,418 7,582

$ 35,000

$ 16,328 14,938 31,266 $ 3,734


P11–4

Concluded

Note C Face value ........................................................................................... Present value (i = 4%, n = 20) PV of face value ($50,000  .4564 from Table 4 in Appendix A) ........................ PV of interest receipts ($2,000  13.5903 from Table 5 in Appendix A) ...................... Total present value (i.e., proceeds) .................................................... Discount/premium ............................................................................. b. Note A Cash (+A) ................................................................................................... Discount on Notes Payable (–L) ................................................................ Notes Payable (+L) .............................................................................. Issued notes payable for cash. Note B Cash (+A) ................................................................................................... Discount on Notes Payable (–L) ................................................................ Notes Payable (+L) .............................................................................. Issued notes payable for cash. Note C Cash (+A) ................................................................................................... Notes Payable (+L) .............................................................................. Issued notes payable for cash. c.

Interest Expense (E, –SE) ........................................................................... Cash (–A)............................................................................................. Incurred and paid interest.

d. Note B Interest Expense (E, –SE) ........................................................................... Discount on Notes Payable (+L) .......................................................... Cash (–A)............................................................................................. Incurred and paid interest. a $3,126.60 b $326.60 c

$2,800.00

$ 50,000

$ 22,820 27,181 50,000 $ 0

12,418.00 7,582.00 20,000.00

31,266.00 3,734.00 35,000.00

50,000.00 50,000.00

2,000.00 2,000.00

3,126.60a

= Book Value  Effective Rate per Period = ($35,000 – $3,734)  10% = Interest Expense – Interest Payment = Face Value  Stated Rate per Period = $35,000  8%

326.60b 2,800.00c


P11–4

Continued

Note C Interest Expense (E, –SE) ........................................................................... Cash (–A)............................................................................................. Incurred and paid interest.

2,000.00 2,000.00

e. Interest Expense (E, –SE) ........................................................................... 1,241.80* Discount on Notes Payable (+L) .......................................................... Amortized discount on notes payable. _________________ * $1,241.80 = Book Value  Effective Rate per Period = ($20,000 – $7,582)  10%

1,241.80

P11–5 a. The effective interest rate can be calculated in two ways. The first way is by solving for i in each of the following equations. Note A: Note B: Note C:

$37,566 = [$50,000  (1 + i)-3] $50,000 = [$50,000  (1 + i)-3] + {$5,000  [(1 - (1+ i)-3) ÷ i]} $45,027 = [$50,000  [(1 + i) -3] + {$3,000  [(1 - (1+ i)-3) ÷ i]}

The second way is by trial and error. Plug an interest rate into the equations until the right-hand side of the equation equals the left-hand side. The annual effective interest rate is 10%. b.

Interest Expense (Note A) Year 1 Year 2 Year 3 Total

c.

3,756.60a 4,132.26b 4,545.14c $ 12,434.00 $

Interest Expense (Note B) $

5,000.00 5,000.00 5,000.00 $ 15,000.00

Interest Expense (Note C) 4,502.70d 4,652.97e 4,817.33f $ 13,973.00 $

Interest Expense = Book Value at Beginning of Period  Effective Rate a $3,756.60 = Initial Book Value of $37,566  10% b $4,132.26 = ($37,566 + $3,756.60)  10% c $4,545.14 = ($50,000 – $37,566) – ($3,756.60 + $4,132.26) d $4,502.70 = Initial Book Value of $45,027  10% e $4,652.97 = [$45,027 + ($4,502.70 – $3,000.00)]  10% f $4,817.33 = ($50,000 – $45,027) – ($4,502.70 – $3,000.00) – ($4,652.97 – $3,000.00) + $3,000 Note A


Return

Expense

4,507.92a 5,048.88b 5,654.73c $ 15,211.52

Year 1 Year 2 Year 3 Total

$

Income

$

3,756.60 4,132.26 4,545.14 $ 12,434.00

$

751.32 916.61 1,109.59 $ 2,777.52

a $4,507.92 = $37,566  12% b $5,048.87 = ($37,566 + $4,507.92)  12% c

$5,654.73 = ($37,566 + $4,507.92 + $5,048.87)  12%

P11–5

Concluded

Note B Year 1 Year 2 Year 3 Total

Return

Expense

6,000.00a 6,720.00b 7,526.40c $ 20,246.40 $

Income

$

5,000.00 5,000.00 5,000.00 $ 15,000.00

$ 1,000.00 1,720.00 2,526.40 $ 5,246.40

a $6,000.00 = $50,000  12% b $6,720.00 = ($50,000 + $6,000)  12% c

$7,526.40 = ($50,000 + $6,000 + $6,720)  12%

Note C

Return

Expense

Income

Year 1 Year 2 Year 3 Total

5,403.24a

$ 4,502.70 4,652.97 4,817.33 $ 13,973.00

$

$

6,051.63b 6,777.82c $ 18,232.69

900.54 1,398.66 1,960.49 $ 4,259.69

a $5,403.24 = $45,027  12% b $6,051.63 = ($45,027 + $5,403.24)  12% c

$6,777.82 = ($45,027 + $5,403.24 + $6,051.63)  12%

d. Total Debt = Current Liabilities as of 12/31/15 + (Long-Term Liabilities as of 12/31/15 + Face Value of Notes Payable – Discount Balance) Total Stockholders' Equity = Stockholders' Equity as of 12/31/15 + Net Income Debt/Equity Ratio = Total Debt ÷ Total Stockholders' Equity

12/31/16 12/31/17 12/31/18

Debt/Equity (Note A)

Debt/Equity (Note B)

Debt/Equity (Note C)

4.270 4.277 4.271

4.516 4.279 3.972

4.418 4.278 4.086

e. Boyton must consider at least four factors in deciding which note to issue. First, the company must consider the income that can be earned from the proceeds. Since Note B provides the largest proceeds,


this note provides the highest net income. Second, the company must consider the cash outflow effects of each note. If the company did not have sufficient cash on hand to meet an interest or principal payment, it could be forced into bankruptcy. Since each note requires a payment at maturity of $50,000, the only difference between the notes is the periodic interest payments. In this case, Note A requires the lowest interest payments. Third, the company must consider the immediate effects on its debt/equity ratio. If the company has any existing debt with a debt covenant that specifies a maximum debt/equity ratio, one of the notes may cause the company to violate the debt covenant. In this case, Note A results in the lowest debt/equity ratio in the current year. Finally, the company must consider the trend in the debt/equity ratio over time. If the company needs or desires to issue additional debt in the future, it might be constrained by its future debt/equity ratio. Creditors might be wary of a company with too high of a debt/equity ratio. In this case, the decrease in the debt/equity ratio is greatest for Note B.

P11–6 a. The Amount of Interest Payments

= Face Value of Debt  Stated Interest Rate = $800,000  10% = $80,000

b. When the note payable was issued, the stated interest rate did not equal the effective interest rate; the effective interest rate exceeded the stated interest rate. Consequently, the proceeds from the note were less than face value, so that the entire loan to Rix Driving Range and Health Club would actually earn the effective interest rate on its money. The excess of the face value over the proceeds gave rise to the Discount on Notes Payable, and from Rix's viewpoint, this account effectively represents prepaid interest. Over the life of the note, this discount will be amortized to Interest Expense. Consequently, the difference between the balance in Interest Expense and the cash paid out for interest payments represents the amortization of the Discount on Notes Payable. c.

Interest Expense $95,000 Rate

= Book Value at Beginning of the Period  Effective Interest Rate = ($800,000 – $70,000)  Effective interest rate = 13% (rounded)

d. Interest Expense (E, –SE) ........................................................................... Discount on Notes Payable (+L) .......................................................... Cash (–A)............................................................................................. Incurred and paid interest.

95,000 15,000 80,000

P11–7 a. Face value .......................................................................................... Present value (i = 4%, n = 12) PV of cash payment at maturity ($20,000  0.6246 from Table 4 in Appendix A) ..................... PV of cash interest payments ($600  9.3851 from Table 5 in Appendix A) .......................... Total present value............................................................................ Discount on bonds............................................................................. b. Face value .......................................................................................... Present value (i = 4%, n = 11)

$ 20,000.00

$ 12,492.00 5,631.06 18,123.06 $ 1,876.94 $ 20,000.00


PV of cash payment at maturity ($20,000  0.6496 from Table 4 in Appendix A) ..................... PV of cash interest payments ($600  8.7605 from Table 5 in Appendix A) .......................... Total present value............................................................................ Discount on bonds.............................................................................

$ 12,992.00 5,256.30 $

18,248.30 1,751.70

The present value of the cash flows on these bonds as of December 31, 2015, using the effective interest rate on the date the bonds were originally issued, represents the book value of the bonds as of December 31, 2015. c.

The difference of $125.24 in present values from June 30, 2015 and December 31, 2015 represents the change in book value of these bonds for this six-month period. The change in book value would be captured by the amortization of the Discount on Bonds Payable account.

P11–7

Concluded

d. Interest Expense (E, –SE) ........................................................................... Discount on Bonds Payable (+L) ......................................................... Cash (–A)............................................................................................. Incurred and paid interest. a $724.92 b $124.92 c

$600.00

724.92a 124.92b 600.00c

= Book Value  Effective Rate per Period = $18,123.06  4% = Interest Expense – Interest Payment = Face Value  Stated Rate per Period = $20,000  3%

The amount of discount on bonds payable is essentially the same as the amount in part (c); the difference of 32¢ is due to rounding. Under the effective-interest method, bonds are carried on the balance sheet at their present value (based upon the effective rate at the initial date of issue) at that particular point in time. Hence, it makes no difference if one computes the present value of the cash outflows associated with bonds or applies the effective-interest method; both methods will yield essentially identical financial statements.

P11–8 a. To compute the amount of money that Ross Running Shoes must invest on June 30, 2015, the future cash flows must be discounted at the investment rate of 8%. Since the investment rate is an annual rate, and interest is paid semiannually, the rate must be adjusted to a six-month rate of 4%. Therefore, i = 4% and n = 6. Present Value

= Present Value of Face Value + Present Value of Interest Payments = ($10,000  .79031 from Table 4 in Appendix A) + [($10,000  5%)  5.24214 from Table 5 in Appendix A] = $7,903.10 + $2,621.07 = $10,524.17

b. Interest Expense (E, –SE) ........................................................................... Premium on Notes Payable (–L) ................................................................

420.97a 79.03b


500.00c

Cash (–A)............................................................................................. Incurred and paid interest. a $420.97 = Book Value  Effective Rate per Period = ($10,000 + $524.17)  4% b $79.03 = Interest Expense – Interest Payment c

c.

$500.00 = Face Value  Stated Rate per Period = $10,000  5%

Interest Expense (E, –SE) ........................................................................... Premium on Notes Payable (–L) ................................................................ Cash (–A)............................................................................................. Incurred and paid interest.

412.64a 87.36b 500.00c

a $412.64 = Interest Payment – Premium Amortization b $87.36 = Total Premium ÷ Number of 6-Month Periods = $524.17 ÷ 6 periods c

P11–8

$500.00 = Face Value  Stated Rate per Period = $10,000  5%

Concluded

d. Under the effective-interest method, the company will recognize interest expense during 2015 of $420.97 [from part (b)]. Under the straight-line method, the company will recognize interest expense during 2015 of $412.64 [from part (c)]. Thus, the straight-line method results in lower expenses and higher net income in the early periods of a note issued at a premium. e. Over the life of a note or bond, both the effective-interest and straight-line methods will amortize the entire discount or premium balance. Consequently, over the life of a note or bond, both methods will amortize exactly the same amount of discount or premium. As noted in part (d), for notes issued at a premium, the straight-line method will recognize lower interest expense than the effective-interest method in the early years of the note's life. The lower interest expense recognized under the straightline method will eventually have to be offset if both methods are to recognize the same amount of interest expense over the life of the note. Consequently, the straight-line method will have to recognize “relatively” higher interest expense and, hence, lower net income in the later years of a note issued at a premium.

P11–9 a. Note A 1/1/15 Present value (i = 6%, n = 3) PV of face value ($1,000  .8396) PV of interest payment ($100  2.6730) Total present value 12/31/16 Present value (i = 6%, n = 1) PV of face value

$

839.60

267.30 $ 1,106.90

12/31/15 Present value (i = 6%, n = 2) PV of face value ($1,000  .8900) PV of interest payment ($100  1.8334) Total present value

$

890.00

183.34 $ 1,073.34


($1,000  .9434) PV of interest payment ($100  .9434) Total present value Note B 1/1/15 Present value (i = 10%, n = 3) PV of face value ($1,000  .75132) PV of interest payment ($100  2.48685) Total present value 12/31/16 Present value (i = 10%, n = 1) PV of face value ($1,000  .90909) PV of interest payment ($100  .90909) Total present value

$

943.40

94.34 $ 1,037.74

$

751.32

248.69 $ 1,000.00

$

909.10

90.91 $ 1,000.00

12/31/15 Present value (i = 10%, n = 2) PV of face value ($1,000  .82645) PV of interest payment ($100  1.73554) Total present value

$

826.45

173.55 $ 1,000.00


P11–9

Continued

Note C 1/1/15 Present value (i = 10%, n = 3) PV of face value ($1,000  .75132) PV of interest payment ($60  2.48685) Total present value 12/31/16 Present value (i = 10%, n = 1) PV of face value ($1,000  .90909) PV of interest payment ($60  .90909) Total present value

$ 751.32 149.21 $ 900.53

12/31/15 Present value (i = 10%, n = 2) PV of face value ($1,000  .82645) PV of interest payment ($60  1.73554) Total present value

$ 909.09 54.55 $ 963.64

b. Interest Expense

Payment Amount

Disc./Prem. Amortization

Face Value

Note A 1/1/15 12/31/15 12/31/16 12/31/17

$66.42 64.40 62.26

$100.00 100.00 100.00

$33.58 35.60 37.74

$1,000.00 1,000.00 1,000.00 1,000.00

$106.90 $1,106.90 73.32 1,073.32 37.71 1,037.73 ($0.00) 1,000.00

Note B 1/1/15 12/31/15 12/31/16 12/31/17

$100.00 100.00 100.00

$100.00 100.00 100.00

$0.00 0.00 0.00

$1,000.00 1,000.00 1,000.00 1,000.00

$0.00 $1,000.00 0.00 1,000.00 0.00 1,000.00 0.00 1,000.00

Note C 1/1/15 12/31/15 12/31/16 12/31/17

$90.05 93.06 96.36

$60.00 60.00 60.00

$30.05 33.06 36.37

$1,000.00 1,000.00 1,000.00 1,000.00

$99.48 69.43 36.37 ($0.00)

Date

Disc./Prem. Balance

Book Value

$900.52 930.57 963.63 1,000.00

$ 826.45 104.13 $ 930.58


P11–9

Concluded

c. Date

Interest Expense

Payment Disc./Prem. Amount Amortization

Face Value

Disc./Prem. Balance

Book Value

Note A 1/1/15 12/31/15 12/31/16 12/31/17

$64.37 64.37 64.37

$100.00 100.00 100.00

$35.63 35.63 35.63

$1,000.00 1,000.00 1,000.00 1,000.00

$106.90 $1,106.90 71.27 1,071.27 35.63 1,035.63 0.00 1,000.00

Note B 1/1/15 12/31/15 12/31/16 12/31/17

$100.00 100.00 100.00

$100.00 100.00 100.00

$0.00 0.00 0.00

$1,000.00 1,000.00 1,000.00 1,000.00

$0.00 $1,000.00 0.00 1,000.00 0.00 1,000.00 0.00 1,000.00

Note C 1/1/15 12/31/15 12/31/16 12/31/17

$26.84 26.84 26.84

$60.00 60.00 60.00

$33.16 33.16 33.16

$1,000.00 1,000.00 1,000.00 1,000.00

$99.48 66.32 33.16 ($0.00)

$900.52 933.68 966.84 1,000.00

d. Compare parts (b) and (c) to part (a). The effective interest method maintains the net book value of the liability equal to the present value of the future cash flows of the liability throughout the liability's life. Alternatively, the straight-line method does not maintain this equality. Further, under the effective interest method, interest expense is always the same percentage of the outstanding debt throughout the life of the liability. This constant relationship arises because interest expense is computed as the book value times the effective interest rate, and since the effective interest rate is assumed to be constant, interest expense remains a constant percentage of the liability. The straight-line method does not result in this constant relationship between interest expense and the outstanding liability, as evidenced by the amounts reported under interest expense in part (c).

P11–10 a. Book Value of Debt

= Face Value of $500,000 + Premium Balance of $12,600 = $512,600

Cash Paid to Retire Debt

= Face Value  104% = $500,000  104% = $520,000

Loss = Excess of Cash Paid Over Book Value = $512,600 – $520,000 = $7,400 b. Cash Paid to Retire Debt

= Face Value  108% = $500,000  108% = $540,000


P11–10 Concluded Loss = Excess of Cash Paid Over Book Value = $540,000 – $512,600 = $27,400 c.

Ginny and Bill Eateries is required to make an interest payment on June 30, 2015 under the terms of the debt agreement. The entry to record this payment would be: Interest Expense (E, –SE) ........................................................................... Premium on Bonds Payable (–L) ............................................................... Cash (–A)............................................................................................. Incurred and paid interest.

15,378a 4,622b 20,000c

a $15,378 = Book Value  Effective Rate per Period = $512,600  3% b $4,622 = Interest Expense – Interest Payment c

$20,000 = Face Value  Stated Rate per Period = $500,000  4%

Book Value of Debt = Face Value + Premium Balance = $500,000 + ($12,600 – $4,622) = $507,978 Cash Paid to Retire Debt

= Face Value  110% = $500,000  110% = $550,000

Loss = Excess of Cash Paid Over Book Value = $550,000 – $507,978 = $42,022

P11–11 a. Face value ........................................................................................... Present value (i = 7%, n = 10) PV of face value ($5,000  .5083 from Table 4 in Appendix A) .......................... PV of interest payments ($300  7.0236 from Table 5 in Appendix A) ........................... Total present value (i.e., proceeds) .................................................... Discount ............................................................................................. Cash (+A) ................................................................................................... Discount on Bonds Payable (–L) ................................................................ Bonds Payable (+L) ............................................................................. Issued bonds.

$ 5,000.00

$ 2,541.50 2,107.08 4,648.58 $ 351.42 4,648.58 351.42 5,000.00


P11–11 Continued b. Interest Expense (E, –SE) ........................................................................... Discount on Bonds Payable (+L) ......................................................... Cash (–A)............................................................................................. Incurred and paid interest.

325.40a 25.40b 300.00c

a $325.40 = Book Value  Effective Rate per Period = $4,648.58  7% b $25.40 = Interest Expense – Interest Payment. c

c.

$300.00 = Face Value  Stated Rate per Period = $5,000  6%

As of June 30, 2017, the bonds have a remaining life of five six-month periods until they mature. Option 1: Repurchase the bonds through the bond market. Present value (i = 5%, n = 5) PV of face value ($5,000  .7835 from Table 4 in Appendix A) .......................... PV of interest payments ($300  4.3295 from Table 5 in Appendix A) ........................... Total present value (i.e., repurchase price) .......................................

$ 3,917.50 1,298.85 $ 5,216.35

Option 2: Repurchase the bonds using the call provision. Repurchase Price

= Face Value  103.5% = $5,000  103.5% = $5,175.00

In this case, Ficus Tree Farm would have to use less cash to redeem the bonds using the call provision than to repurchase them through the bond market. Consequently, the company should use the call provision to redeem the bonds. d. Assume that a company wishes to redeem all outstanding bonds prior to maturity. It is unlikely that it could accomplish this goal by repurchasing the bonds through the bond market. Some bondholders would simply be unwilling to sell the bonds. It is costly for bondholders to sell their bonds and reinvest. They incur transaction costs (i.e., brokerage fees, etc.) when selling investments. It is also timeconsuming (for example, the opportunity cost of researching new investment opportunities). Bondholders would also consider the tax implications of selling their bonds. If the bondholder would have to recognize any gains on the sale of the bond, these gains would be considered taxable income. To avoid these taxes, the bondholder may prefer to simply hold the bond. By exercising a call provision, a company can compel all bondholders to surrender their bonds. Consequently, if a company wishes to retire all outstanding bonds, the company will have to resort, at least partially, to exercising any relevant call provisions.


P11–11 Concluded e. Bonds Payable (–L) .................................................................................... Extraordinary Loss on Redemption (E, –SE) .............................................. Discount on Bonds Payable (+L) ......................................................... Cash (–A)............................................................................................. Redeemed bonds. *see table Date

Interest Expense

1/1/15 6/30/15 1/1/16 6/30/16 1/1/17 6/30/17

Payment Disc./Prem. Amount Amortization

$325.40 327.18 329.08 331.12 333.30

$300.00 300.00 300.00 300.00 300.00

$25.40 27.18 29.08 31.12 33.30

Face Value

5,000.00 380.35 205.35* 5,175.00

Disc./Prem. Balance

$5,000.00 5,000.00 5,000.00 5,000.00 5,000.00 5,000.00

Book Value

$351.42 $4,648.58 326.02 4,673.98 298.84 4,701.16 269.76 4,730.24 238.64 4,761.36 205.35 4,794.65

P11–12 a. Face value ........................................................................................ Present value (i = 5%, n = 8) PV of face value ($100,000  .67684 from Table 4 in Appendix A) ................. PV of interest payments ($3,000  6.46321 from Table 5 in Appendix A) ................... Total present value.......................................................................... Discount .......................................................................................... Date

Interest Expensea

Cash Paymentb

12/31/15 6/30/16 12/31/16 6/30/17 12/31/17 6/30/18 12/31/18 6/30/19

$4,353,70 4,421.39 4,492.45 4,567.08 4,645.43 4,727.70 4,814.09 4,904.79

$3,000.00 3,000.00 3,000.00 3,000.00 3,000.00 3,000.00 3,000.00 3,000.00

$ 100,000

$ 67,684 19,390

Amortized Discountc

$1,353.70 1,421.39 1,492.45 1,567.08 1,645.43 1,727.70 1,814.09 1,904.79

87,074 $ 12,926 Book Valued

$ 88,427.70 89,849.09 91,341.54 92,908.62 94,554.05 96,281.75 98,095.84 100,000.00

a Interest Expense = Book Value at the Beginning of the Period  Effective Rate per Period of 5% b Cash Payment = Face Value of $100,000  Stated Rate per Period of 3% c

Unamortized Discount = Unamortized Discount at the Beginning of the Period – Excess of Interest Expense Over the Cash Payment d Book Value = Face Value of $100,000 – Unamortized Discount


P11–12 Concluded b. Cash outflows Total interest payments

= =

$3,000  8 payments $24,000

Total principal payment

=

$100,000 on maturity of the bonds

Total cash outflow

= =

$24,000 + $100,000 $124,000

= =

Proceeds received upon issuing the bonds $87,073

Cash inflows Cash inflows

Therefore, cash outflows exceed cash inflows by $36,927. c.

Cash outflows Post-tax interest payments

= = =

[$3,000  (1 – tax rate)]  8 payments [$3,000  (1 – 34%)]  8 payments $15,840

Total principal payment

=

$100,000 on maturity of the bonds

Total cash outflow

= =

$15,840 + $100,000 $115,840

= =

Proceeds received upon issuing the bonds $87,073

Cash inflows Cash inflows

Therefore, cash outflows exceed cash inflows by $28,767. d. Cash outflows Individual post-tax interest payments = $3,000  (1 – tax rate) = $1,980 = $1,980  Present value of an ordinary annuity

Present value of post-tax payments = 5% and n = 8)

= $1,980  6.4632 from Table 5 in Appendix A = $12,797 Total principal payment

= $100,000 on maturity of the bonds

Present value of principal payment Total cash outflow

= $67,684 [from part (a)] = $12,797 + $67,684 = $80,481

Cash inflows Cash inflows

= =

Proceeds received upon issuing the bonds $87,073

Therefore, cash inflows exceed cash outflows by $6,592.

for i


P11–13 a. On the financial statements a capital lease is treated like the company had purchased the fixed assets. The asset and the related liability are recorded on the balance sheet and interest and depreciation are recorded on the income statement. An operating lease is treated like a recurring expense each month but nothing is recorded on the balance sheet. The amount of the lease payment is shown as an expense each month. b. A company may want to treat leases as operating leases because there is no debt that is recorded on the balance sheet. This treatment impacts a number of financial ratios (debt-to-equity, for example) and so may be to the company’s advantage to treat it like an operating lease. c.

total liability ÷ total asset ratio if Wal-Mart treats these leases as: currently recorded: $121 ÷ $203 = 59.6% if all leases are capital leases: $130.2 (121 + 9.2) ÷ $212.2 (203 + 9.2) = 61.4% $3.4 ÷ $6.2 = 54.8%, 54.8% of $16.8 billion = $9.2 billion

d. An analyst needs to be able to compare companies that use different methods for accounting for leases. If an analyst does not do this additional analysis there is a good chance that the analyst will be misled as to the relative performance of the companies. The more leases that the companies have on their books, the more this difference between operating leases and capital leases could affect the analysis of the companies. A review of “off-balance sheet financing” is always a prudent step in financial analysis.

P11–14 a. The initial balance sheet value of the equipment and the initial leasehold obligation both equal the present value of the lease payments. This amount can be determined in the following ways. Present value of lease payments

= FMV of equipment = $119,782

or Present value of lease payments

= Present value of lease payments = $30,000  Present value of an ordinary annuity factor for i = 8% and n = 5 = $30,000  3.99271 (from Table 5 in Appendix A) = $119,781.30


P11–14

Concluded

Balance Sheet Value of Leasehold Date Equipmenta Obligationb 1/1/14 $119,781.30 $119,781.30 12/31/14 95,825.04 99,363.80 12/31/15 71,868.78 77,312.91 12/31/16 47,912.52 53,497.94 12/31/17 23,956.26 27,777.78 12/31/18 (0.00) (0.00) Total

Interest Expensec

Depr. Expensed

Total Expense

$9,582.50 7,949.10 6,185.03 4,279.84 2,222.22 $30,218.70

$23,956.26 23,956.26 23,956.26 23,956.26 23,956.26 $119,781.30

$33,538.76 31,905.36 30,141.29 28,236.10 26,178.48 $150,000.00e

a Balance Sheet Value of Equipment = Value of Equipment on 1/1/14 – Accum. deprec. b Leasehold Obligation = Leasehold Obligation at Beginning of the Period – ($30,000

Lease Payment – Interest Expense for the Period) Interest Expense = Leasehold Obligation at Beginning of the Period  8% d Depreciation Expense = $119,781.30 ÷ 5 years e Total has penny discrepancy due to rounding to even cents throughout lease term. c

= Annual Rent Payments  Number of Years of the Lease = $30,000  5 years = $150,000

b. Total Rent Expense

c.

If the lease is treated as a capital lease, total expenses would be $150,000 [from part (a)]. If the lease is treated as an operating lease, total expenses would still be $150,000 [from part (b)]. Although total expenses would be the same under either approach, different expense accounts are affected under the two approaches. With a capital lease, the $150,000 is allocated between interest expense and depreciation expense, while with an operating lease, the entire $150,000 is allocated to rent expense.

P11–15 a. If the lease is treated as an operating lease, Thompkins Laundry would not have to report any liability associated with the lease. Therefore, its debt/equity ratio would be as follows. Debt/Equity Ratio

= = = =

Total Liabilities ÷ Stockholders' Equity (Current Liabilities + Long-Term Liabilities) ÷ Stockholders' Equity $30,000 ÷ $40,000 0.75

b. If the lease is treated as a capital lease, Thompkins Laundry would have to report a liability equal to the present value of the future lease payments. Therefore, its debt/equity ratio would be affected. Present value of lease payments

= $5,000  Present value of an ordinary annuity factor for i = 12% and n = 5 = $5,000  3.60478 (from Table 5 in Appendix A) = $18,023.90


Debt/equity ratio

P11–15

= ($30,000 + $18,023.90) ÷ $40,000

=

1.20

Concluded

c.

Rent Expense Operating lease Capital lease

$5,000.00 0.00

Interest Expense $

0.00 2,162.87

Depreciation ___Expense_ $

0.00 3,604.78

Total Expenses $5,000.00 5,767.65

d. There are two primary reasons why Thompkins Laundry might want to arrange the terms of the lease agreement so that the lease would be classified as an operating lease rather than as a capital lease. First, lease obligations under an operating lease are not disclosed on the face of the balance sheet. Consequently, operating leases are essentially off-balance-sheet financing and will not affect any existing debt covenants that are based on reported liabilities. Second, in this case the capital lease classification results in higher expenses and, hence, lower net income in 2015 than the operating lease classification. Decreased net income would adversely affect any contracts, such as the manager's incentive contract, written on the basis of reported net income. To avoid classifying this lease as a capital lease, Thompkins Laundry would have to arrange the terms so that the lease did not meet any of the criteria for capital leases. Consequently, the company would have to arrange the terms so that: (1) (2) (3) (4)

the present value of the lease payments is less than 90% of the fair market value of the leased property; the term of the lease is less than 75% of the leased property's life; the lessee does not have the right either during or at the expiration of the lease agreement to purchase the property from the lessor at a nominal amount; or ownership of the property is not transferred to the lessee from the lessor by the end of the lease term.

P11–16 a. Equipment (+A) ......................................................................................... Discount on Notes Payable (–L) ................................................................ Notes Payable (+L) .............................................................................. Purchased equipment in exchange for a note. b. Present Value $17,604

17,604 2,396 20,000

= Present Value of Maturity Payment + Present Value of Periodic Payments = ($20,000  Present Value Factor) + ($1,000  Present Value of an Ordinary Annuity Factor)

Since the present value of $17,604 is less than the face value, we know that the note was issued at a discount. Consequently, the effective rate is greater than the stated rate. We also know that the stated rate is 5% ($1,000 ÷ $20,000 face value). Try i = 6% for n = 5: ($20,000  .74726 from Table 4 in Appendix A) + ($1,000  4.21236 from Table 5 in Appendix A) = $14,945 + $4,212 = $19,158 Try i = 8% for n = 5 ($20,000  .68058 from Table 4 in Appendix A) + ($1,000  3.99271 from Table 5 in Appendix A) = $13,612 + $3,992 = $17,604


Therefore, the effective interest rate on the note is 8%.

P11–16 c.

Concluded 1,408a

Interest Expense (E, –SE) ........................................................................... Discount on Notes Payable (+L) .......................................................... Cash (–A)............................................................................................. Incurred and paid interest.

408b 1,000

a $1,408 = Book Value  Effective Rate per Period = ($20,000 – $2,396)  8% b $408 = Interest Expense – Interest Payment

d. 12/31/15 Net Book Value

= = =

Face Value – 12/31/15 Discount on Notes Payable $20,000 – ($2,396 – $408) $18,012

P11–17 a. Since the bonds are selling at par value, the effective interest rate must be equal to the stated interest rate of 9%. The effective interest rate is the sum of two components: a risk-free component and a risk premium. It is given in the problem that the risk-free rate is 7%, which implies that the risk premium on Hodge Sports, bonds must be the difference between the effective interest rate of 9% and the risk-free rate of 7%, or 2%. b. If the risk premium increased from 2% to 5%, the effective interest rate would increase to 12%. A single bond would now be worth $889.59 to you, as calculated below. (Remember that bonds usually have a face value of $1,000 and pay interest semiannually.) Present value (i = 6%, n = 10) Present value of face value ($1,000  .55839 from Table 4 in Appendix A) ............................... Present value of interest payments ($45  7.36009 from Table 5 in Appendix A) .................................. Total present value.................................................................................... c.

$ 558.39 331.20 $ 889.59

A decrease in the prime interest rate would probably result in a drop in the effective interest rate used to discount the future cash flows of Hodge Sports’ bonds. As the effective interest rate drops, the stated interest rate looks relatively more attractive to investors. Thus, demand for the bonds should increase, which, in turn, should drive up the selling price of the bonds. A single bond would now be worth $1,040.55, as calculated below. Present value (i = 4%, n = 10) Present value of face value ($1,000  .67556 from Table 4 in Appendix A) ............................... Present value of interest payments ($45  8.11090 from Table 5 in Appendix A) .................................. Total present value....................................................................................

$

675.56

364.99 $ 1,040.55


P11–18 a. The effective interest rate on the bonds is 7.85%. The future value of the bond payments are $2,000 (semi-annual interest payment based on the stated rate of 4%) for four periods and $100,000 (principal due at maturity); the present value is the purchase price of $92,994. The effective rate of 8% discounts the future values to the present value. (The general present value formula of 1/[(1 + r) to the nth] was used in this calculation.) b. Cash 2,000 Bond Investment 1,650 Interest Revenue 3,650 Receipt of interest payment on 11/30/2014 (3,650 = Eff. Rate per period of 3.925% X $92,994)

Cash 2,000 Bond Investment 1,715 Interest Revenue 3,715 Receipt of interest payment on 5/31/2015 (3,715 = Eff. Rate per period of 3.925% X [92,994 + 1,650]) c. On May 31, 2015 the book value of the investment is $96,359 (92,994 + 1,650 + 1,715). On the same date, if market interest rates are 6% the market value of the investment is $98,087 (PV of a $2,000 ordinary annuity, n=2, r = 3 plus PV of a single sum of $100,000, n = 2, r = 3).


ISSUES FOR DISCUSSION ID11–1 a. A debenture is an unsecured bond. That is, there is no collateral supporting the bond. Thus, should the company not repay the bonds, investors do not have security in any of the company's assets that could be sold to repay the bonds. For this reason, unsecured bonds are riskier than secured bonds. Investors are compensated for this increased risk on debentures through a higher return (i.e., effective interest rate). Accordingly, these bonds would be priced lower than a secured bond in the same company. b. There are three general reasons why a company would repurchase its outstanding debt. First, the company may no longer need the money it borrowed. By repurchasing the debt, the company could avoid incurring interest. Second, due to a decrease in interest rates, the company may have repurchased its debt with the intent of issuing new debt at the lower prevailing interest rates. Finally, the company may repurchase some of its debt in an effort to improve its balance sheet. This would generally be in an effort to improve some financial ratios specified in debt covenants. c.

Repurchasing debt would decrease both a company's liabilities (due to the amount of debt repurchased) and its assets (due to the cash paid out to repurchase the debt). For Sun Company, its stockholders' equity would also decrease because it paid out $957.50 for each bond when the book value of a bond was only $875. Thus, Sun Company would have a loss of $82.50 on each bond repurchased, which would decrease stockholders' equity through closing the loss into Retained Earnings.

d. Sun Company would not have recognized any loss if it had not repurchased its debt. Unless there is evidence to the contrary, such as a company repurchasing its debt, accountants assume that when a company issues debt, the debt will remain outstanding until it matures. This implies that changes in the market value of the debt are irrelevant to the financial position of the company as reported in its financial statements.

ID11–2 a. The stated interest rate affects only the magnitude of periodic interest payments. What is important to investors is the rate of return on their investments. Thus, if an investor is not in need of periodic cash payments, a non-interest-bearing obligation that provides a competitive rate of return is an attractive investment option. b. The rate that discounts $200 million due in eight years to a present value of $66.48 million is 14.75%. c.

If bonds have a stated rate, the company has to have sufficient cash flow to make the periodic interest payments. Thus, if a company does not expect to have sufficient cash flows to support periodic interest payments, it is to the company's advantage to issue bonds with a stated interest rate of zero.


ID11–2 Concluded d. To simplify the calculations, the effective interest rate of 14.75% [see part (b)] is rounded to 15%. 5% stated rate Present value of $200 million paid in 8 years $200 million  .32690 (from Table 4 in Appendix A) ................................... Present value of periodic interest payments ($200 million  5%)  4.48732 (from Table 5 in Appendix A) ...................... Issue price........................................................................................................ 18% stated rate Present value of $200 million paid in 8 years $200 million  .32690 (from Table 4 in Appendix A) ................................... Present value of periodic interest payments ($200 million  18%)  4.48732 (from Table 5 in Appendix A) .................... Issue price........................................................................................................

$

65,380,000

44,873,200 $ 110,253,200

$

65,380,000

161,543,520 $ 226,923,520

ID11–3 a. The effective interest rate is the interest rate that equates the undiscounted future cash flows with the present value of the future cash flows. For both alternatives, the undiscounted cash flows are only the fifteen annual payments of $6 million each, and the present value of both alternatives is the $45,636,480 price of the jet plane. The equation to equate the undiscounted future cash flows and the present value is as follows. $45,636,480 = $6,000,000  {[1– (1 + i)-15] ÷ i]} Solving for i mathematically or by trial and error indicates that the annual effective interest rate is 10%. b. Cash (+A) ........................................................................................ Note Payable (+L) .................................................................... Issued note payable.

45,636,480

Airplane (+A)................................................................................... Cash (–A).................................................................................. Purchased airplane.

45,636,480

Airplanes Capitalized Under Leases (+A) ........................................ Lease Liability (+L) ................................................................... Leased airplane.

45,636,480

c.

45,636,480

45,636,480

45,636,480

d. If Southwest Airlines borrows the necessary funds and then purchases the airplane, Southwest's fixed assets and liabilities would both increase by $45,636,480. In addition, Southwest would have to depreciate the airplane. The effect on the financial statements would be identical if Southwest leases the airplane, and the lease is considered to be a capital lease. e. Southwest Airlines would not have to prepare any journal entry when it signs the lease if the lease is considered to be an operating lease.


ID11–3 Concluded f.

Structuring the leasing arrangement as an operating lease would be an example of off-balance sheet financing. With an operating lease, the substance of the lease arrangement is that Southwest is renting the airplane from the Boeing Company. Thus, Southwest is not considered to have any obligation to Boeing until Southwest actually uses the airplane. As Southwest uses the airplane, Southwest should record rent expense. This means that Southwest would never report any liability on its balance sheet associated with the future payments required under the lease agreement. Thus, Southwest would be able to finance the "acquisition" of an airplane without having to report any liability associated with acquiring the airplane. Southwest would most likely engage in off-balance sheet financing in order to prevent an increase in the amount of reported debt. By limiting any increases in reported debt, the company decreases the chance it would violate any debt covenants that specify a maximum debt/equity ratio. In addition, reporting less debt would make Southwest appear to be a less risky investment option. Since the return a company must pay on investment capital is positively associated with the risks of the company, anything that would make Southwest appear to be less risky should decrease its cost of capital. The real question, however, is whether potential investors, in evaluating investment alternatives, focus solely on the debt reported on the balance sheet or, instead, focus on the company's obligations reported in the footnotes to the financial statements. The footnotes would usually disclose any major obligations under operating leases.

ID11–4 a. The current portion of Long Term Debt ($1,512 million) appeared in the Current Liabilities section of the balance sheet; the rest of the Long Term Debt, totaling $11,489 million, appeared in the long-term liabilities section of Johnson & Johnson’s balance sheet. b. A zero coupon debenture is a debt instrument that has a stated rate of interest of 0%. The debenture contract only requires the repayment of the face amount at maturity. However, because no company borrows at zero percent, the debentures are sold at a discount depending on the effective rate of interest. The zero coupon debenture that are due in 2020 carry an effective interest rate of 3.00, indicating that Johnson & Johnson did not receive the face value of the debentures at funding but will have to repay the face value at maturity (meaning that the company effectively is paying a 3% interest rate to borrow the money). c.

A bond contract with a stated rate of interest equal to the effective rate of interest will be sold at par (no discount or premium). The 2017, 2018, 2023 and 2033 debentures carry effective rates equal to their stated rates, therefore selling at par.

d. The 6.95% notes due in 2029 were issued with an effective rate of interest of 7.14%, which is in excess of the stated rate of 6.95%. Therefore, the notes were sold at a discount, meaning that the face amount is greater than the balance sheet amount of $296 million.


ID11–5 a. Investors are motivated by risk and return. If investors determine that the low returns available on high quality bond issues are not sufficient for their income needs, those investors might be tempted to take on additional risk from stock investments in order to earn higher returns. As the economy moved away from the Great Recession, investors became more comfortable assuming risk, especially as returns on bond issues remained low. b. In the case of a company’s failure, the debt holders are paid prior to the equity holders; therefore, the risk of the debt holders is less and the required return of those debt holders is similarly lower. Equity providers, last in line in the order of being paid, assume more risk and therefore require a higher return. c.

The laws of supply and demand determine the price of a bond. If investors move to sell their bond holdings, the supply of bonds in the market will increase at the same time that the demand for those bonds from other investors decreases. These two moves, a decrease in demand and an increase in supply, will push the equilibrium price down.

d. Since the cash flow receipts of a bond holder are locked in place (the coupon interest payments and the face value are fixed in the contract terms), the price paid to own those future cash inflows will determine the yield earned. If an investor buys those fixed cash flows for a lower market price, the return will naturally be higher; conversely, paying more to get the same (fixed) future inflows will drive down the yield. ID11–6 a. A large amount of debt forces a company's management to place greater emphasis on generating cash so that it has sufficient cash to make the required interest and principal payments. Thus, a company may alter its operating, investing, and financing decisions to allow it to generate the cash it needs when it needs it. b. The massive borrowing activity during the 1980s would have manifested itself as increased liabilities on the companies' balance sheets. By analyzing different companies' current ratios and debt/equity ratios, which are measures of a company's solvency, potential investors may have been able to identify those companies that were taking on an excessive amount of debt. However, even this type of analysis may not have been sufficient to identify overly risky companies. Companies will often engage in off-balance sheet financing, such as structuring leasing arrangements as an operating lease. Companies are most likely to engage in off-balance sheet financing when they are close to violating existing debt covenants that specify a maximum debt/equity ratio or when the company already has a large amount of debt. Since, by definition, off-balance sheet financing does not show up on the balance sheet as a liability, it will not be reflected in either the current ratio or the debt/equity ratio. An alternative analysis strategy investors could have used was to examine the statement of cash flows to determine whether the company was consistently generating enough cash from operating activities to service its debt. This approach is good in that any cash payments associated with off-balance sheet financing will be reflected on the statement of cash flows. The negative aspect of this analysis approach is that the analysis cannot be adequately performed until the company is making interest and principal payments. By this time it may be too late! c.

A debenture is an unsecured bond. That is, there is no collateral supporting the bond. Thus, should the company not repay the bonds, investors in debentures, unlike investors in secured bonds, do not have security in any of the company's assets that could be sold to repay the bonds. In other words, in the event a company liquidates, the secured creditors are paid before the unsecured creditors. This means


that if the company does not have sufficient cash available after liquidating to repay both the secured creditors and the holders of debentures, it is the latter group that will not receive full payment.

ID11–7 a.

Home Depot

Liabilities Total assets

$23 billion $41 billion

Liabilities/total assets ratio

0.56

b. If all leases are capital leases: Home Depot Liabilities + Lease liabilities Total liabilities Assets + Lease liabilities Total Assets

$23 billion 5.4 billion $28.4 billion

$19 billion $33 billion 0.58

Lowe’s $19 billion 3.5 billion $22.5 billion

$41 billion $33 billion 5.4 billion 3.5 billion $46.4 billion $36.5 billion

Adjusted liabilities/total assets ratio 0.61

c.

Lowe’s

0.62

Both companies saw their adjusted debt ratio increase. In both cases, the adjustment doesn’t make much of a difference since the companies already have relatively high ratios. However, adding the leases to the balance sheet does increase the leverage of the companies.

d. An analyst may wish to make the adjustment required above so he/she can make equal comparisons between large retailers. By making the adjustment, the analyst knows that differences due to accounting method choices or lease reporting choices have been removed.

ID11–8 a. A ratings agency is a supposedly independent expert charged with the responsibility of analyzing the risks associated with debt (and equity) securities. Once the risks have been analyzed and quantified, the ratings agency assigns a grade (a “rating”) to the security. The lower the risk, the higher the rating; the higher the risk, the lower the rating. b. Investors interested in purchasing the security will perform their own analysis of the underlying risks but will also look to the rating as guidance. If the ratings agency assigns a low rating (implying greater risk), the investor will demand a higher interest rate to compensate for the risks involved. On the other hand, if the agency assigns a high rating (implying less risk), the investor does not require as high a rate of interest. As discussed in this chapter, the rate demanded by the investor (the effective rate of interest) determines the price the investor will pay to purchase the security and receive the stated rate of interest. If the rate demanded by the borrower (due to the rating of the security), exceeds the stated rate in the security, the investor will purchase the security at a discount (that is to say, the price of the security will be below the face value).


c.

The ratings agency should look at a number of areas, including: the income and credit history of the individual borrowers (the homeowners), the value of the homes pledged as collateral, the amount of downpayment made by the borrowers (the initial equity in the house), and the overall health of the housing market, both nationally and in the local markets where the loans were made.

ID11–9 a. The covenant limits the company’s borrowing capacity by stating that funded debt can be no more than three times EBITDA (a rough estimation of annual cash flow). Since EBITDA was $1,604 million, funded debt could be no more than $4,812 million. With existing debt at $3,505 million, the covenant limits additional debt to no more than $1,307 million. b. The creditors are trying to control the amount of debt that J.C. Penney has on its balance sheet by limiting that debt to a multiple of annual cash flow. The thinking is that the debt will be repaid from that cash flow, so a reasonable limit on debt would be one that correlates the amount of debt with the amount of cash flow the company generates. If the company wants to borrow more, it needs to generate more cash flow; if the company (due to a recession or to poor operations) generates less cash flow, then it will be allowed to borrow less. c.

If J.C. Penney violates the financial covenant, the creditors have the right to immediately call the loans, requiring the company to pay them off at once. Most likely, however, there would be a renegotiation of the credit agreements between the creditors and the company. Contract issues such as interest rates, fees, and future financial covenants might be changed by the lenders who would feel that they are carrying more risk due to the covenant violation.

ID11–10 a. On the financial statements a capital lease (a lease that is “equivalent to purchasing an asset”) is treated like the company had purchased the fixed asset. The asset and the related liability are recorded on the balance sheet and interest and depreciation are recorded on the income statement. An operating lease (a lease in the nature of short-term hire) is treated like a recurring expense each month but nothing is recorded on the balance sheet. The amount of the lease payment is shown as an expense each month. The result is that a company with an operating lease will show higher net income (in early years) and a lower debt/equity ratio than a company with a capital lease. b. The rule passed in 1981 was unpopular for 2 primary reasons. The first was that it forced companies to capitalize some leases. This would have the impacts as described above. Capitalized leases would tend to lower net income and increase the debt/equity ratio. The second reason it was unpopular was because it was a relatively complex set of rules that had to be followed. These rules required a fair amount of time and effort for companies to implement. c.

Financial engineers have sought to keep debt off the balance sheet by structuring contracts in such a manner that the contract will qualify as an operational lease when the reality is that the fixed asset has all other characteristics of an asset that would normally be capitalized. This is done by making sure that the lease contract does not meet any of the four criteria that would force the company to capitalize the lease.

d. Mr. Holgate makes a good point. When two transactions, that are substantially the same, can be recorded in significantly different ways on the financial statements then there is a problem. This is


confusing to users of the financial statements and can lead to non-productive decisions being made. Companies will structure transactions to work around accounting rules as opposed to structuring the transaction in the best way for the company.

ID11–11 a.

During the severe economic recession, the likelihood that companies would not be able to meet their obligations (the companies’ “default risk”) increased substantially. Investors, sensing this increased risk level, reacted by shunning debt from companies with heightened default risk. In the financial markets, this investor reaction increased the supply of corporate bonds and simultaneously decreased demand for these financial contracts. Prices of the debt fell considerably.

b.

A company that was able to purchase its own debt at a steep discount would be reducing its liabilities without having to pay full price to do so. Effectively, companies could decrease a dollar’s worth of debt by paying less than a dollar to do so.

c.

On the financial statements, any company retiring debt by paying less than the current carrying cost of the liabilities would experience a Gain on Retirement of Debt (along with a cash outflow and a reduction of the liability).

ID11–12 Bristol-Myers Squibb is concerned that changes in interest rates could adversely impact the company’s financial condition. To protect against this possibility—to manage its interest rate risk—the company has entered into interest rate swaps with another party. In essence, an interest rate swap allows the two parties to exchange interest payments. One party would have interest rates on debt that is locked in at a fixed rate, meaning that no matter what market rates of interest do, the fixed rate will not change. The other party to the swap agreement would have interest rates on debt that float with changes in market interest rates. The party with the fixed rate swaps interest payments with the party with floating rates, effectively giving the company the other rate arrangement.

ID11–13


a. The long-term debt / total asset ratio for Google was 7.6% in 2011 ($5,516/$72,574) and 8.3% in 2012 ($7,746/$93,798). The ratio increased only slightly over this time period. b. Footnote #4 indicates, “The effective interest yields of the 2014, 2016, and 2021 Notes were 1.258%, 2.241%, and 3.734%, respectively.”

c.

The proceeds from long-term debt issuances increased from over $5 billion to nearly $11 billion to over $16 billion (from 2010 to 2012).

d. The carrying value of the long-term debt on the balance sheet was $2.986 billion and $2.988 (2011 and 2012), while “the total estimated fair value of the Notes was approximately $3.2 billion at December 31, 2011 and December 31, 2012.” (Footnote #4)

e. Footnote #3 indicates that the company uses interest rate swaps to hedge its exposure to interest rate fluctuations.


CHAPTER 12 STOCKHOLDERS' EQUITY BRIEF EXERCISES BE12–1 a. 38.4% of net income was paid in dividends during the year ($1,743/$4,535). b. The issuance of common stock affected the basic accounting equation by increasing assets (cash) and increasing stockholders equity by the same amount (common stock and additional paid-in capital). c.

The purchase of treasury stock affected the basic accounting equation by reducing assets (cash) and reducing stockholders equity (treasury stock) by the same amount.

d. The total dollars distributed to the company’s shareholders during the year totaled $5,743 ($1,743 + $4,000). This is comprised of dividends paid and the treasury stock that was purchased. e. The balance in retained earnings as of the end of the year was $20,038. ($17,246 + $4,535 – $1,743).

BE12–2 a. The number of shares outstanding after the split would be 194 million shares (97 million x 2) and the price per share would be approximately $50 ($100/2). b. The company’s overall value or market capitalization is $9.7 billion ($50 x 194 million shares). The company’s overall market should not change simply because of the share split. The number of shares will double but the price per share will be cut in half. Sometimes companies that announce a share split see their stock price rise because many investors see a share split as a positive sign from management but this shareholder action is inferring positive news that has not been announced.

BE12–3 a. During 2010 the company paid an average of $30.21 per share in its repurchase program ($2,961/98). b. During 2011 the company paid an average of $35.54 per share in its repurchase program ($4,513/127). c.

With no information regarding 2013 treasury purchases, the balance in the treasury stock account will be $33,809 ($35,009 - $1,200) .

EXERCISES

1


E12–1 a.,b.,c. Accounts

Effect on Account

Effect on Total Stockholders' Equity

Increase Increase N/A Increase Increase Increase Decrease Decrease Increase N/A

Increase

(6)

Common Stock Additional Paid-In Capital, C/S None Treasury Stock Common Stock Additional Paid-In Capital, C/S Retained Earnings Treasury Stock Additional Paid-In Capital, T/S None

(7)

Retained Earnings

(1) (2) (3) (4)

(5)

Increase

No effect Decrease No effect

Increase No effect

Increase

E12–2 a. Debt = Total Liabilities = $52,000 + $35,000 = $87,000 Contributed Capital

Earned Capital

= Preferred Stock + Common Stock + Additional Paid-In Capital, Preferred Stock + Additional Paid-In Capital, Common Stock – Treasury Stock = $50,000 + $80,000 + $50,000 + $100,000 – $80,000 = $200,000

= Retained Earnings = $113,000

The portions of Lamont's assets provided by debt, contributed capital, and earned capital are, therefore, 21.75%, 50%, and 28.25%, respectively. b. Debt/Equity

Debt/Equity

c.

= = = =

Total Liabilities ÷ Stockholders' Equity Total Liabilities ÷ (Contributed Capital + Earned Capital) $87,000 ÷ ($200,000 + $113,000) .278

= Total Liabilities ÷ Total Stockholders' Equity = (Total Liabilities + Contributed Preferred Capital) ÷ (Contributed Common Capital + Earned Capital – Treasury Stock) = ($87,000 + $50,000 + $50,000) ÷ ($80,000 + $100,000 + $113,000 – $80,000) = .878

Most states restrict the dollar amount of dividends to either the balance in Retained Earnings or the balance in Retained Earnings less any treasury stock. So in this case, Lamont Brothers, either would be restricted to $113,000 or $33,000, depending upon the state.


E12–3 (1)

No entry is necessary.

(2)

Cash (+A)............................................................................................... Common Stock (+SE)...................................................................... Additional Paid-In Capital, Common Stock (+SE) ........................... Issued common stock.

300,000

Cash (+A)............................................................................................... Preferred Stock (+SE) ..................................................................... Additional Paid-In Capital, Preferred Stock (+SE) .......................... Issued preferred stock.

3,750,000

Cash (+A)............................................................................................... Preferred Stock (+SE) ..................................................................... Issued preferred stock.

2,500,000

(3)

(4)

50,000 250,000

2,500,000 1,250,000

2,500,000

Effect of each event on basic accounting equation: Transaction

Assets

Liabilities

1. 2. 3. 4.

NE + + +

NE NE NE NE

Owner’s Equity NE + + +

Since par value of a share of stock has no relationship to market value, it has very little economic significance. At one time, par value was construed to be legal minimum capital to protect creditors in times of dissolution or bankruptcy, but over time the concept has lost its appeal as creditors have found better ways to protect themselves.


E12–4 a. Treasury Stock (–SE) .................................................................................. Cash (–A)............................................................................................. Acquired treasury stock. (Dollars in millions) b. Debt/Equity

c.

1,000 1,000

= Total Liabilities ÷ Total Stockholders' Equity

Before

= $1,048 ÷ $2,012 = 0.52

After

= $1,048 ÷ ($2,012 – $1,000) = 1.04

Earnings per Share

= Net Income ÷ Outstanding Shares

Before

= $94 ÷ 125.5 Shares = $0.75 per Share

After

= $94 ÷ (125.5 Shares Issued – 26 Treasury Shares) = $0.94 per Share

d. A company might choose to purchase treasury stock at year end in order to reduce the number of shares outstanding as of year-end. A smaller number of shares outstanding will increase the earnings per share (based on average # of shares outstanding during the year).

E12–5 a. (1)

(2)

(3)

(4)

(5)

Cash (+A) ................................................................................................ Common Stock (+SE) ....................................................................... Additional Paid-In Capital, Common Stock (+SE) ............................ Issued common stock.

500,000

Cash (+A) ................................................................................................ Preferred Stock (+SE) ...................................................................... Issued preferred stock.

60,000

Treasury Stock (–SE)............................................................................... Cash (–A) ......................................................................................... Repurchased treasury stock.

45,000

Cash (+A) ................................................................................................ Treasury Stock (+SE) ........................................................................ Additional Paid-In Capital, Treasury Stock (+SE) ............................. Reissued treasury stock.

18,000

Cash (+A) ................................................................................................ Additional Paid-In Capital, Treasury Stock (–SE) .................................... Retained Earnings (–SE) ......................................................................... Treasury Stock (+SE) ........................................................................ Reissued treasury stock.

5,000 3,000 7,000

125,000 375,000

60,000

45,000

15,000 3,000

15,000


E12–5

Concluded

b. Preferred stock ($8 par value, 5,000 shares outstanding) ........................ Common stock ($5 par value, 25,000 shares issued, 24,000 shares outstanding) ..................................................................... Additional paid-in capital .......................................................................... Retained earnings...................................................................................... Treasury stock ........................................................................................... Total stockholders' equity .........................................................................

$

60,000

125,000 375,000 493,000 (15,000) $ 1,038,000

E12–6 a. (1)

(2)

Treasury Stock (–SE) ...................................................................... Cash (–A) ................................................................................. Purchased treasury stock.

60,000

Cash (+A) ........................................................................................ Retained Earnings (–SE) ................................................................. Treasury Stock (+SE) ............................................................... Reissued treasury stock as compensation.

10,000 20,000

60,000

30,000

Note:

Depending upon the terms of the employees' compensation package, it may be more appropriate to debit Compensation Expense for $20,000 rather than debiting Retained Earnings for $20,000.

(3)

Cash (+A) ........................................................................................ Treasury Stock (+SE) ............................................................... Additional Paid-In Capital, Treasury Stock (+SE)..................... Reissued treasury stock.

33,000

Common stock........................................................................................... Additional paid-in capital (common and treasury stock) .......................... Retained earnings ($60,000 - $20,000 +$20,000) ..................................... Total stockholders' equity .........................................................................

$ 80,000 13,000 60,000 $153,000

30,000 3,000

b. A total of $3,000 of additional paid-in capital is attributable to treasury stock. This amount would be recorded in the account Additional Paid-In Capital, Treasury Stock. Under the cost method, this amount represents the amount of proceeds received in excess of the acquisition cost of the treasury stock reissued.

E12–7 a. Treasury Stock (–SE) .................................................................................. Cash (–A)............................................................................................. Purchased treasury stock.

2,850,000 2,850,000


E12–7

Concluded

b. Common stock........................................................................................... Additional paid-in capital, common stock ................................................. Retained earnings................................ ($4,500,000 +$350,000 - $50,000) Treasury stock ........................................................................................... Total stockholders' equity ......................................................................... c.

$

100,000 2,400,000 4,800,000 (2,850,000) $ 4,450,000

When common stock is initially issued, it is recorded at the value of the assets received. In this case Stuart Corporation received $25 per share for 100,000 shares of common stock, for a total of $2,500,000. This $2,500,000 was allocated between the accounts Common Stock and Additional Paid-in Capital, Common Stock. Under the cost method of accounting for treasury stock, acquiring treasury stock does not cause the amount recorded for the initial issue of common stock to be adjusted. The treasury stock is simply recorded at the value of the assets given up. So if the value of the assets given up to acquire the treasury stock exceeds the value of the assets received when the common stock was initially issued, then the Treasury Stock balance will exceed contributed capital.

E12–8 a. Book Value per Share

= Common Stockholders' Equity ÷ Common Shares Outstanding = $13,065 ÷ 782 Shares = $16.71 per Share

b. Book Value per Share

= [$13,065 + (50 Shares  $32)] ÷ 832 Shares = $17.63 per Share

c.

Book Value per Share

= [$13,065 + (50 Shares  $20)] ÷ 832 Shares = $16.91 per Share

d. Book Value per Share

= [$13,065 – (50 Shares  $32)] ÷ 732 Shares = $15.66 per Share

e. Book Value per Share

= [$13,065 – (50 Shares  $20)] ÷ 732 Shares = $16.48 per Share

f.

Issuing stock can either increase or decrease the book value of a company's common stock. Whether issuing stock increases or decreases the book value depends upon the issue price of the new stock. If new stock is issued at a price above the pre-issue book value, then issuing the stock increases the book value. Alternatively, if new stock is issued at a price below the pre-issue book value, then issuing the stock decreases the book value.

g. Purchasing treasury stock can either increase or decrease the book value of a company's common stock. Whether purchasing treasury stock increases or decreases the book value depends upon the acquisition price of the treasury stock. If treasury stock is acquired at a price above the pre-issue book value, then purchasing treasury stock decreases the book value. Alternatively, if treasury stock is acquired at a price below the pre-issue book value, then purchasing the stock increases the book value.


E12–9 a. Cash (+A) ................................................................................................... Preferred Stock (+SE) .......................................................................... Issued preferred stock.

300

b. Cash (+A) ................................................................................................... Common Stock (+SE) .......................................................................... Additional Paid-In Capital, Common Stock (+SE)................................ Issued common stock.

120

c.

30

Cash (+A) ................................................................................................... Treasury Stock (+SE) ........................................................................... Additional Paid-In Capital, Treasury Stock (+SE) ................................ Reissued treasury stock.

300

100 20

20 10

E12–10 =

Common Stock + Additional Paid - in Capital Number of Shares Issued

=

$10,000 + $25,000 2,000 *

=

$17.50

*Total Par Value of Common Stock

÷

Par Value per Share

=

Number of Shares Issued

$10,000

÷

$5

=

2,000

a. Issue Price per Share

b. Purchase Price of Treasury Stock

c.

=

Treasury Stock Carrying Value on Balance Sheet Number of Treasury Stock Purchased

=

$8,000 400

=

$20/Share

To acquire Timeco, Zielow issued 1,000* shares and the market price of Timeco at the time of acquisition was $28,000.** * $5,000 c/s ÷ $5 P/V ** $5,000 c/s + $23,000 AP/C

d. Since the common stock accounts is always credited with total par value of shares issued to stock option holders, the company issued 200 shares (i.e., $1,000 c/s ÷ $5 a share P/V). The stock options were exercised at a price of $9* a share. Most certainly the market price of Zielow’s shares would be more than $9 at that time. * ($1,000 c/s + $800 AP/C) ÷ 200 shares.


E12–10 Concluded e. Per Share Dividend Rate

=

Total Dividends Paid During 2015 Common Stock Outstanding

=

$3,520 2,600 *

=

$1.354 per share

* ($16,000 ÷ $5) – [(8,000 + 4,000) ÷ $20] In this problem it is assumed that during the year 2015, treasury-stock was acquired at $20 a share.

E12–11 =

Common Stock + Additional Paid - in Capital Number of Shares Issued

=

$8,000 + $32,000 8,000 shares *

=

$5 a share

*Number of Shares Issued

÷

Total Par Value of Common Stock

=

Par Value per Share of C/S

8,000

÷

$8,000

=

$1

a. Issue Price per Share

b. Purchase Price of Treasury Stock

c.

=

Treasury Stock Carrying Value on Balance Sheet Number of Treasury Stock Purchased

=

$18,000 1500 , Shares

=

$12/Share

Since $3,000 worth of treasury stock was used to satisfy the stock options, at a price of $12 a share, a total of 250 shares were issued through stock options. Since retained earnings is debited at $2,750, it means that stock options were sold at par value or $1 a share resulting in the following entry. Cash (+A) ................................................................................................... Retained Earnings (–SE)............................................................................. Treasury Stock (+SE) ...........................................................................

250 2,750 3,000


E12–11 Concluded d. Per Share Cash Dividend Rate

=

Total Cash Dividends Number of Common Shares Outstanding

=

$3,500 6,750 *

=

$.519 per share

* 6,750 = 8,000 Shares – 1,500 Treasury Shares Purchased + 250 Treasury Reissued

E12–12 a. Only those shares that are both issued and outstanding are eligible to receive dividends. Since Enerson has 375,000 shares of common stock issued, but 50,000 of those shares are held in treasury, the total number of common shares outstanding and, hence, eligible for a dividend are 325,000 shares. b. Date of declaration Cash Dividend (–SE) .................................................................................. Dividend Payable (+L) ......................................................................... Declared cash dividend.

3,900,000 3,900,000

Date of record No journal entry is necessary. Date of payment Dividend Payable (–L) ................................................................................ Cash (–A)............................................................................................. Paid cash dividend.

3,900,000 3,900,000

E12–13 a. Each year the preferred stockholders are entitled to $5 for each share of preferred stock outstanding. Since 5,000 shares are outstanding, total dividends to preferred stockholders should be $25,000 per year. Year 2011

Dividends Declared $

0

2012

30,000

2013

80,000

2014 2015

15,000 40,000

Dividends to Preferred $

0 25,000 5,000 20,000 25,000 15,000 10,000 25,000

Dividends to Common $

(for 2011) (for 2012) (for 2012) (for 2013) (for 2014) (for 2014) (for 2015)

0 0 35,000 0 5,000


E12–13 Concluded b. Dividends in arrears are the dividends preferred stockholders are entitled to if and when the company's board of directors declares a dividend. The amount of dividends in arrears equals the cumulative total of dividends not declared and not paid to preferred stockholders in the current and past years. The amount of dividends in arrears at the end of each year would be as follows. December 31, 2011 December 31, 2012 December 31, 2013 December 31, 2014 December 31, 2015 c.

$25,000 $20,000 $0 $10,000 $0

Dividends in arrears should not be considered a liability. A liability represents the probable future sacrifice of assets. A company may choose to reinvest its profits back into the company, or the company may not be financially secure enough to pay a dividend. This uncertainty is only resolved when the company's board of directors actually declares a dividend. Preferred stockholders are only entitled to receive their dividend when the company declares a dividend. If the board of directors never declares a dividend, then the preferred stockholders are not entitled to receive one; thus, no liability exists until the dividend is actually declared.

E12–14 a. Stock Dividend (–SE).................................................................................. Common Stock (+SE) .......................................................................... Additional Paid-In Capital, Common Stock (+SE)................................ Declared and issued 2% stock dividend.

11,200* 960 10,240

* $11,200 = (10,000 shares issued – 2,000 shares in treasury)  2%  $70 per share b. No journal entry is necessary. However, the company should prepare a memorandum entry stating that the par value has decreased from $6 to $4 per share and that there are now 15,000 shares issued and 12,000 shares outstanding. c.

Stock Dividend (–SE).................................................................................. Common Stock (+SE) .......................................................................... Additional Paid-In Capital, Common Stock (+SE)................................ Declared and issued 10% stock dividend. * $64,000

64,000* 4,800 59,200

= (10,000 shares issued – 2,000 shares in treasury)  10%  $80

d. No journal entry is necessary. However, the company should prepare a memorandum entry stating that the par value has decreased from $6 to $3 per share and that there are now 20,000 shares issued and 16,000 shares outstanding.


E12–14 Concluded e. Ratio = (Common Stk. + Additional Paid-In Capital – Treasury Stock) ÷ Retained Earnings Prior to entries ($60,000 + $100,000 – $24,000) ÷ $60,000 = 2.27 After (a) [($60,000 + $960) + ($100,000 + $10,240) – $24,000] ÷ ($60,000 – $11,200) = 3.02 After (b) ($60,000 + $100,000 – $24,000) ÷ $60,000 = 2.27 After (c) [($60,000 + $4,800) + ($100,000 + $59,200) – $24,000] ÷ ($60,000 – $64,000) = .00* After (d) ($60,000 + $100,000 – $24,000) ÷ $60,000 = 2.27 * Since retained earnings account is –$4,000, no ratio can be computed. In a stock split the number of outstanding shares is simply “split” into smaller units, which require the corporation to distribute additional shares. However, in a stock dividend additional shares, usually expressed as a percentage of the outstanding shares, are issued to stockholders. Large stock dividends have essentially the same effect as stock splits.

E12–15 a. Option 1 Stock Dividend (–SE).................................................................................. Common Stock (+SE) .......................................................................... Additional Paid-In Capital, Common Stock (+SE)................................ Declared and issued 10% stock dividend. Option 2 Stock Dividend (–SE).................................................................................. Common Stock (+SE) .......................................................................... Additional Paid-In Capital, Common Stock (+SE)................................ Declared and issued 20% stock dividend.

42,500 5,000 37,500

85,000 10,000 75,000

When a company declares an ordinary stock dividend, the fair market value of the new shares issued is transferred from Retained Earnings, via the Stock Dividend account, to the contributed capital accounts. Since in most states dividends are restricted to the balance in Retained Earnings (or Retained Earnings less Treasury Stock), any decrease in its account balance decreases the amount of potential dividends. So from the stockholders' viewpoint a stock dividend is not attractive because it decreases potential future dividends. The primary reason that a company would declare an ordinary stock dividend is as a publicity gesture. Some companies take great pride in being able to "promote" the company's dividend-paying history. If a company finds itself short of cash and still wants to be able to claim that it paid a dividend, then the company can maintain its dividend-paying streak by declaring a stock dividend.


E12–15 Concluded b. Option 3 would have no effect on any of the account balances reported in the financial statements because a stock split does not affect a company's financial position. However, a stock split does affect both the par value per share of the company's common stock and the number of shares outstanding, and as a result earnings per share. In this particular case, the 2-for-1 stock split would result in the par value per share of Railway Shippers' common stock decreasing from $10 to $5 and in the number of common shares outstanding increasing from 5,000 shares to 10,000 shares. c.

Declaring a stock split does not inherently increase or decrease a company's value. After a stock split, a company's value is allocated over a larger number of shares, so each share is worth less. If each share is worth less, then each share should sell for a lower price. If a company's stock is trading at too high a price, the average investor will not be able to invest in the company. If the price of the company's stock was lowered, trading in the company's stock would be stimulated. Consequently, a company can lower the price of its stock and stimulate trading in its stock by declaring a stock split.

E12–16 a. Appropriating retained earnings serves to restrict a portion of retained earnings from the payment of future dividends. Appropriations of retained earnings usually arise for two reasons. First, a creditor may require the borrower to appropriate retained earnings. Such appropriations prevent the borrower from paying "excessive" dividends to the stockholders, thereby reducing the amount of cash available to repay creditors. Second, a company may decide to restrict future dividends and use the cash that would have otherwise been used to pay dividends to finance plant expansion and so forth. In this particular case, the company appropriated retained earnings for both of these reasons. Auditors would require that appropriations of retained earnings be disclosed in the financial statements because they affect the magnitude of a company's future dividends. The magnitude of future dividends is information that current and potential investors desire. Failure to adequately disclose such information could cause investors to lose money on their investments and, consequently, to sue the auditor. b. Common stock........................................................................................... Additional paid-in capital .......................................................................... Retained earnings Restricted ........................................................................................... Unrestricted........................................................................................ Total stockholders' equity ......................................................................... c.

XX XX 350,000 450,000 $ XX

The company can only declare a dividend equal only to the portion of retained earnings which exceed $500,000. Since the debt covenant requires a minimum balance of $500,000 in the retained earnings account, the board is constrained by that clause and could possibly declare dividends up to a maximum of $300,000.


PROBLEMS P12–1 a. Cash (+A) ................................................................................................... Preferred Stock (+SE) .......................................................................... Issued preferred stock. Debt/Equity

b. Debt/Equity

c.

100,000 100,000

= Total Liabilities ÷ Stockholders' Equity = $250,000 ÷ ($330,000 + $100,000) = .58 = ($250,000 + $100,000) ÷ $330,000 = 1.06

If management classifies the stock as stockholders' equity, then the company will not be in violation of its debt agreement. However, if management classifies the stock as debt, then the company will be in violation of its debt agreement. Since violating debt agreements can be quite costly to both the company and managers, managers have incentives to classify the stock as stockholders' equity. The terms of the preferred stock make it appear to be more similar to debt than to equity. For example, the stock has a specified rate, does not participate in the benefits of ownership (i.e., does not vote and does not participate in profits), and has a fixed life. Based upon these factors, it appears that the stock is in substance actually debt and should be classified as such. Auditors will typically be guided by generally accepted accounting principles (GAAP) in deciding how to report an item. However, in most cases, GAAP does not provide clear-cut guidance, and auditors must apply their judgment. In applying their judgment, auditors are likely to consider the costs and benefits of alternative reporting options to themselves, the financial statement users, and the company in deciding how to classify the item. Auditors are normally better off by having events recorded in the most conservative manner. In this case, the most conservative manner of recording the stock would be to classify it as debt. Classifying the stock in this way decreases the probability that financial statement users will suffer out-of-pocket losses and be able to sue the auditors. Consequently, the auditors would probably prefer that the stock be classified as debt. It should be noted that some securities are such a hybrid of debt and equity that GAAP requires such securities to be reported as neither debt nor equity. Instead, GAAP requires such securities to be reported on the balance sheet in a special section between long-term debt and stockholders' equity.

P12–2 a. The balance in the Common Stock account represents the number of shares of common stock issued times the par value per share. Since the balance of $300,000 represents 50,000 shares, the par value per share must be $6. b. Book Value = (Total Stockholders' Equity – Contributed Capital from Preferred Stockholders) ÷ Number of Common Shares Outstanding = [($840,000 – $300,000) - $50,000] ÷ 45,000 Shares = $10.89


P12–2 c.

Concluded

Since 50,000 shares have been issued, and only 45,000 shares are outstanding, 5,000 shares are held in treasury. The balance of $40,000 in treasury stock represents the cost of these 5,000 shares. Consequently, the average price of the treasury stock was $8.

d. If the company reissues the treasury stock at $10 per share, stockholders' equity would increase by $50,000 and the company would make the following entry. Cash (+A) ................................................................................................... Treasury Stock (+SE) ........................................................................... Additional Paid-In Capital, Treasury Stock (+SE) ................................ Reissued treasury stock.

50,000 40,000 10,000

Debt/equity ratio The company's debt/equity ratio is calculated as Total Debt ÷ Total Stockholders' Equity. This ratio would decrease, because the numerator would be unchanged, while the denominator would increase. Book value The book value is calculated as Total Common Stockholders' Equity ÷ Number of Common Shares Outstanding. In this case, the numerator would increase by $50,000, and the denominator would increase by the 5,000 shares reissued. The book value would now be $10.80 [($540,000 – $50,000 + $50,000) ÷ (45,000 + 5,000)]. Since the book value prior to the reissue was $10.89 (see part b), the company's book value would decrease. Earnings per share Earnings per share is calculated as Net Income ÷ Number of Common Shares Outstanding. If the company did not reissue the stock, its EPS would be $1 per share ($45,000 ÷ 45,000 shares). The reissue of treasury stock has no effect on net income, but it does increase the number of common shares outstanding. So the company's EPS would decrease. Its EPS would now be $0.90 per share [$45,000 ÷ (45,000 + 5,000)].

P12–3 (1)

(2)

(3)

Cash (+A) ................................................................................................ Common Stock (+SE) ....................................................................... Issued common stock.

500,000

Cash (+A) ................................................................................................ Common Stock (+SE) ....................................................................... Additional Paid-in Capital, Common Stock (+SE) ............................ Issued common stock.

400,000

Cash (+A) ................................................................................................ Common Stock (+SE) ....................................................................... Additional Paid-in Capital, Common Stock (+SE) ............................ Issued common stock.

300,000

500,000

10,000 390,000

100,000 200,000


P12–3 (4)

b.

Concluded

Cash (+A) ................................................................................................ Preferred Stock (+SE) ...................................................................... Issued preferred stock.

400,000 400,000

Par value has accounting significance only in that it is used to determine the amount that is allocated to the paid-in capital accounts associated with common stock or preferred stock accounts when stock is issued. Economically, par value has little significance. Par values were initially established to protect creditors. Par value represents the amount that stockholders are liable to creditors. However, because par values are usually very small amounts, such as $1.00 or even $0.01 and because debt covenants now exist to protect creditors, par value no longer has much economic significance.

P12–4 a. Dividends are paid only on the shares that are both issued and outstanding. In this case, 55,000 shares have been issued, but 8,000 of these shares are held as treasury stock. Thus, only 47,000 shares are eligible to receive a dividend. b. Date of declaration Cash Dividend (–SE) ................................................................................... Dividend Payable (+L) ......................................................................... Declared divided.

705,000 705,000

Date of record No journal entry is necessary. Date of payment Dividend Payable (–L) ................................................................................ Cash (–A)............................................................................................. Paid dividend. c.

Stock Dividend (–SE).................................................................................. Common Stock (+SE) .......................................................................... Additional Paid-in Capital, Common Stock (+SE) ................................ Declared and issued stock dividend.

705,000 705,000

235,000a 47,000b 188,000

a $235,000 is calculated as follows:

= Number of Shares Outstanding  10% = 47,000  10% = 4,700 Shares 2. Value of Dividend = $50 Fair Market per Share  4,700 Shares to Be Distributed b $47,000 = 4,700 shares to be distributed  par value of $10 per share 1. Number of Shares to Be Distributed

d. The overall impact of cash dividends is a decline in the Retained Earnings account. Since retained earnings is a part of equity, the debt/equity ratio will increase. Issuance of stock dividends results in no change in the overall stockholders’ equity of the company. However, the balance in the retained earnings account goes down and common stock and additional paid-in capital account balances go up. Thus, the debt/equity ratio remains unchanged after the issuance of stock dividends.


P12–4

Concluded

e. Stockholders would generally prefer a cash dividend over a stock dividend. Assume that you own 1,300 shares of Royal Company's common stock prior to any dividend. Since there are 47,000 shares outstanding, you own 3% of the company. In addition, since each share is worth $50, the total value of your investment is $65,000. Since the company's financial position would not be expected to improve or worsen simply from declaring a stock dividend, the total value of your investment should still be worth $65,000. Thus, it appears that receiving a stock dividend has not improved your wealth. To the extent that Royal Company cannot declare and pay dividends in excess of its balance in Retained Earnings, a stock dividend may even decrease your wealth. By declaring a stock dividend, Royal Company has capitalized part of Retained Earnings, which means that it will never be available for cash dividends. Alternatively, with a cash dividend you would receive something of value, namely cash, while still maintaining a 3% ownership interest in the company. The trade-off, however, is that the value of the company would decrease by the value of the cash dividend.

P12–5 a. Each preferred stockholder is entitled to 10% of the par value, or $5.00. Thus, the 15,000 preferred stockholders are entitled to a total of $75,000 in any particular year.

Year

Total Dividends

Preferred Dividends

2009 2010 2011 2012 2013 2014 2015

$ 65,000 100,000 70,000 50,000 125,000 110,000 99,000

$65,000 75,000 70,000 50,000 75,000 75,000 75,000

b. Year

Total Dividends

2009 2010

$ 65,000 100,000

2011 2012

70,000 50,000

2013

125,000

2014 2015

110,000 99,000

Preferred Dividends $65,000 10,000 75,000 70,000 5,000 45,000 30,000 75,000 75,000 75,000

(for 2009) (for 2009) (for 2010) (for 2011) (for 2011) (for 2012) (for 2012) (for 2013) (for 2014) (for 2015)

Common Dividends

Preferred per Share

Common per Share

$

0 25,000 0 0 50,000 35,000 24,000

$4.33 5.00 4.67 3.33 5.00 5.00 5.00

$0.00 0.50 0.00 0.00 1.00 0.70 0.48

Common Dividends

Preferred per Share

Common per Share

0 15,000

$4.33 5.67

$0.00 0.30

0 0

4.67 3.33

0.00 0.00

20,000

7.00

0.40

35,000 24,000

5.00 5.00

0.70 0.48

$


P12–6 a. The maximum cash dividend that Cotter Company could declare given its current financial position is $25,000. This amount represents the cash that the company currently has on hand. If the company wants to declare a larger cash dividend, it would either have to sell some of its assets or borrow cash, thereby changing its financial position. b. In most states a company can not legally declare a dividend that exceeds the balance in Retained Earnings. Since Cotter Company has a balance in Retained Earnings (after closing entries) of $288,000, this amount is the maximum amount that the company could legally declare as a stock dividend. To be considered an ordinary stock dividend, however, the stock dividend cannot exceed 20% to 25% of the common shares already outstanding. The company's common stock is currently selling for $40 per share, which means that the company could distribute an additional 7,200 common shares ($288,000 ÷ $40 per share) in a stock dividend. Since the 7,200 shares represent only 14.4% of the 50,000 shares already outstanding, the distribution of the 7,200 shares would be considered an ordinary stock dividend. c.

Cash Dividend Cash Dividend (–E)..................................................................................... Cash (–) ............................................................................................... Declared and paid cash dividend. Stock Dividend Stock Dividend (–E).................................................................................... Common Stock (+SE) .......................................................................... Additional Paid-In Capital, Common Stock (+SE)................................ Declared and issued stock dividend.

25,000 25,000

288,000 72,000 216,000

d. If Cotter Company sold its marketable securities, it would receive the market value of $50 per share. Since Cotter Company owns 2,500 shares, it would receive $125,000. Combining this $125,000 with the $25,000 of cash already on hand would allow Cotter Company to declare and pay a cash dividend of $150,000.

P12–7 a. Stevenson Enterprises would make the following journal entry for the stock dividend. Stock Dividend (–SE).................................................................................. Common Stock (+SE) .......................................................................... Additional Paid-In Capital, Common Stock (+SE)................................ Declared and issued 10% stock dividend. * $150,000 = (60,000 shares outstanding  10%)  $25 per share

150,000* 36,000 114,000


P12–7

Continued

Stockholders’ equity: Common stock ($6 par value, 650,000 shares authorized, 76,000 shares issued, 66,000 shares outstanding, and 10,000 shares held as treasury stock) ................................................ Additional paid-in capital (C/S) ................................................................. Retained earnings...................................................................................... Treasury stock ........................................................................................... Total stockholders' equity .........................................................................

$

456,000 639,000 545,000 (100,000) $ 1,540,000

b. If Stevenson Enterprises declares a 2-for-1 stock split, no journal entry is necessary. Stockholders’ equity: Common stock ($3 par value, 1,300,000 shares authorized, 140,000 shares issued, 120,000 shares outstanding, and 20,000 shares held as treasury stock) ................................................ Additional paid-in capital (C/S) ................................................................. Retained earnings...................................................................................... Treasury stock ........................................................................................... Total stockholders' equity ......................................................................... c.

$

420,000 525,000 695,000 (100,000) $ 1,540,000

If Stevenson Enterprises declares the stock dividend and then a stock split, the only journal entry the company would have to make would be the journal entry given in Part (a). Stockholders’ equity: Common stock ($3 par value, 1,300,000 shares authorized, 152,000 shares issued, 132,000 shares outstanding, and 20,000 shares held as treasury stock) ................................................ Additional paid-in capital (C/S) ................................................................. Retained earnings...................................................................................... Treasury stock ........................................................................................... Total stockholders' equity .........................................................................

$

456,000 639,000 545,000 (100,000) $ 1,540,000

d. Stevenson Enterprises does not need to prepare any journal entry for the stock split. If Stevenson Enterprises subsequently declares and pays a 10% stock dividend, the company would have to make the following entry. Stock Dividend (–SE).................................................................................. Common Stock (+SE) .......................................................................... Additional Paid-In Capital, Common Stock (+SE)................................ Declared and issued 10% stock dividend. a $150,000 b $36,000

150,000a

= [(60,000 shares initially outstanding + 60,000 shares issued in stock split)  10%]  $12.50 per share = 12,000 new shares issued  $3 par value per share

36,000b 114,000


P12–7

Concluded

Stockholders’ equity: Common stock ($3 par value, 1,300,000 shares authorized, 152,000 shares issued, 132,000 shares outstanding, and 20,000 shares held as treasury stock) ................................................ Additional paid-in capital (C/S) ................................................................. Retained earnings...................................................................................... Treasury stock ........................................................................................... Total stockholders' equity .........................................................................

$

456,000 639,000 545,000 (100,000) $ 1,540,000

P12–8 a.

2011

Dollar Amount / Shares

Exercised stock options $312.5 Sale of common stock Repurchase of stock

19.1 555.9 $887.5

17.3

=

Average price $ 18.06

0.5 15.6

$ 38.20 $ 35.63

33.4

$ 26.57

2012 Exercised stock options $ 326.1 Sale of common stock Repurchase of stock

16.5

19.5 501.9 $847.5

$ 19.76 0.3

12.3

$ 65.00 $ 40.80

29.1

$ 29.12

b. The common stock shares were issued at a higher price than the options. Stock options typically have a life of 5-10 years. So it is very likely that the stock options that were exercised this year were granted in previous years. If the stock price has risen over the last few years then the options would have been granted at lower prices in previous years. The trend in the stock price from 2011 to 2012 has been up rather significantly.


P12–9 a. (1)

(2)

(3)

(4)

Treasury Stock (–SE) ...................................................................... Cash (–A) ................................................................................. Purchased treasury stock.

1,000

Cash (+A) ........................................................................................ Preferred Stock (10%) (+SE) .................................................... Additional Paid-In Capital, 10% Preferred Stock (+SE) ........... Issued preferred stock.

3,800

Cash (+A) ........................................................................................ Treasury Stock (+SE) ............................................................... Additional Paid-In Capital, Treasury Stock (+SE)..................... Reissued treasury stock.

660

10% Preferred Cash Dividend (–SE) ............................................... 12% Preferred Cash Dividend (–SE) ............................................... Common Stock Cash Dividend (–SE) .............................................. Dividends Payable (+L) ............................................................ Declared cash and stock dividend.

400a 180 170

1,000

2,000 1,800

360 300

750

a Cumulative preferred dividends are paid first.

Since no dividends have been paid since 2013, dividends in arrears in the amount of $200 (2 years of 10% of par value) on the 10% cumulative Preferred stock must be paid before any dividends can be paid on any other shares. The new shares of the 10% issue also receive dividends on a priority basis. The $750 payment goes to the cumulative shares first, with the remainder spread over the other preferred issue and then the common shares.

(5)

(6)

Dividends Payable (–L)................................................................... Cash (–A) ................................................................................. Paid and issued dividend.

750 750

No journal entry is needed. A memorandum entry should be made stating that the company's common stock now has a par value of $.50 per share and that 7,000 shares are now issued and 840 shares [after the events underlying entries (1) and (3)] are held in treasury.


P12–9

Concluded

b. Preferred stock (10%, $10 par value, cumulative) .................................... Preferred stock (12%, $10 par value, noncumulative) .............................. Common stock ($0.50 par value, 10,000 shares authorized, 7,000 shares issued, and 840 shares held in treasury) ....................... Additional paid-in capital: Preferred stock (10%) ......................................................................... Preferred stock (12%) ......................................................................... Common stock .................................................................................... Treasury stock .................................................................................... Retained earnings...................................................................................... Treasury stock ($5,750 + $1,000 - $360) ................................................... Total stockholders' equity ......................................................................... * $4,405

=

$

3,000 1,500 3,500

2,850 1,275 2,345 300 4,405* (6,390) $ 12,785

$4,256 – $750 (dividends declared) + $899 (net income)

P12–10 a. Number of Shares Issued

Average Issue Price

= Increase in Contributed Capital ÷ Number of Shares = (Increase in Par Value + Increase in Additional Paid-in Capital) ÷ 900 Shares = [($200,000 – $110,000) + ($150,000 – $35,000)] ÷ 900 Shares = $227.78 per Share

b. Number of Shares Issued

Average Issue Price

c.

= Increase in Par Value ÷ Par Value per Share = ($200,000 – $110,000) ÷ $100 per Share = 900 Shares

= ($900,000 – $750,000) ÷ $10 per Share = 15,000 Shares

= [($900,000 – $750,000) + ($465,000 – $298,000)] ÷ 15,000 Shares = $21.13 per Share

Treasury Stock (–SE) .................................................................................. Cash (–A)............................................................................................. Purchased treasury stock. Average Repurchase Price

110,000 110,000

= Total Repurchase Price ÷ Number of Shares Held in Treasury = $110,000 ÷ 5,000 Shares Held in Treasury = $22.00 per Share


P12–10 Concluded d. Book value equals total common stockholders' equity divided by the total number of common shares outstanding. To determine total common stockholders' equity prior to the acquisition of the treasury stock, the value of the 5,000 shares of treasury stock must be added back to total stockholders' equity, and the 5,000 shares of treasury stock must be considered to be outstanding. Thus, prior to the acquisition of the treasury stock, the book value of outstanding common shares was: Total Common Stockholders Equity ÷ Number of Common Shares Outstanding = [(Total Stockholders' Equity + Treasury Stock) – Preferred Stock – Additional Paid-In Capital, Preferred Stock] ÷ Number of Common Shares Outstanding = ($2,180,000 + $110,000) – $200,000 – $150,000) ÷ 90,000 Shares = $21.56 per Share After the repurchase of the treasury stock, the book value of outstanding shares was $21.53 per share ($1,830,000 ÷ 85,000 shares outstanding). Thus, repurchasing the treasury stock decreased book value per share by $0.03.

P12–11 a. Number of Shares Issued

b. Average Issue Price

c.

= Change in Total Par Value ÷ Par Value per Share = ($110,000 – $70,000) ÷ $10 per Share = 4,000 Shares

= Cash Received ÷ Number of Shares Issued = [Change in Total Par Value + Change in Additional Paid-In Capital, Common Stock] ÷ 4,000 Shares [from part (a)] = [($110,000 – $70,000) + ($625,000 – $500,000)] ÷ 4,000 Shares = $41.25 per Share

Ending Retained Earnings = Beginning Retained Earnings + Net Income – Dividends Declared – Treasury Stock Adjustments $975,000 = $250,000 + $2,000,000 – Dividends Declared – 0* Dividends declared = $1,275,000 * Since Additional Paid-in Capital, Treasury Stock increased during 2014, we know that, on average, Tracey Corporation reissued the treasury stock for more than what it paid for the stock. Since Retained Earnings is adjusted for treasury stock transactions only when the stock is reissued for less than its acquisition cost, there is no treasury stock adjustment to Retained Earnings during 2014.

d. Total dividends declared = $1,275,000 [from Part (c)] Dividends to Preferred Stockholders

= Dividends in Arrears + Current Dividends Dividends for 2013 + Dividends for 2014 = ($400,000  10%) + ($400,000  10%) = $80,000

Dividends to Common Stockholders

= Total Dividends – Dividends to Preferred = $1,275,000 – $80,000 = $1,195,000

=


P12–11 Concluded e. Treasury Stock (–SE) .................................................................................. Cash (–A)............................................................................................. Purchased treasury stock.

105,000

f.

90,000

Cash (+A) ................................................................................................... Treasury Stock (+SE) ........................................................................... Additional Paid-in Capital, Treasury Stock (+SE) ................................ Reissued treasury stock.

g. Reissue Price per Share

105,000

21,000 69,000

= Cash Received ÷ Number of Shares Reissued = $90,000 [from part (f)] ÷ 300 Shares* = $300.00 per Share

* The 300 shares reissued is computed as follows: 1. Cost of shares = $105,000 ÷ 1,500 shares = $70 per share 2. Number of shares reissued = Change in treasury stock balance ÷ $70 per share = ($105,000 – $84,000) ÷ $70 per share = 300 shares

P12–12 a. (1)

(2)

Cash (+A) ........................................................................................ Common Stock (+SE) ............................................................... Additional Paid-in Capital, Common Stock (+SE) .................... Issued common stock.

750,000

Stock Dividend (–SE) ...................................................................... Common Stock (+SE) ............................................................... Additional Paid-in Capital, Common Stock (+SE) .................... Declared and issued 10% stock dividend.

90,000*

300,000 450,000

30,000 60,000

* $90,000 = 50,000 shares outstanding  10%  $18 per share (3)

(4)

(5)

(6)

Cash (+A) ........................................................................................ Common Stock (+SE) ............................................................... Additional Paid-in Capital, Common Stock (+SE) .................... Issued common stock.

1,320,000

Treasury Stock (–SE) ...................................................................... Cash (–A) ................................................................................. Purchased treasury stock.

375,000

Cash (+A) ........................................................................................ Treasury Stock (+SE) ............................................................... Additional Paid-in Capital, Treasury Stock (+SE) ..................... Reissued treasury stock.

135,000

360,000 960,000

375,000

125,000 10,000

The company should make a memorandum entry stating that par value is now $2 per share and the total number of shares issued is now 345,000 shares, of which 315,000 shares are outstanding and 30,000 shares are held in treasury at an adjusted cost of $8.33 per share.


P12–12 Concluded (7)

Cash (+A) ........................................................................................ Treasury Stock (+SE) ............................................................... Additional Paid-In Capital, Treasury Stock (+SE)..................... Reissued treasury stock.

80,000 66,640* 13,360

* $66,640 = 8,000 shares  Adjusted acquisition price of $8.33 per share (8)

(9)

(10)

Cash Dividend (–SE) ....................................................................... Dividend Payable (+L) ............................................................. Declared a cash dividend.

50,000

Dividend Payable (–L) .................................................................... Cash (–A) ................................................................................. Paid cash dividend.

50,000

Cash (+A) ........................................................................................ Treasury Stock (+SE) ............................................................... Additional Paid-In Capital, Treasury Stock (+SE)..................... Reissued treasury stock.

30,000

50,000

50,000

16,660* 13,340

* $16,660 = 2,000 shares  Adjusted acquisition price of $8.33 per share b. Common stock ($2 par value, 1,000,000 shares authorized, 345,000 shares issued, and 20,000 shares held in treasury) .............. Additional paid-in capital: Common stock .................................................................................... Treasury stock .................................................................................... Retained earnings...................................................................................... Treasury stock ........................................................................................... Total stockholders' equity ......................................................................... a $480,000

b $166,700

$690,000 1,470,000 36,700 480,000a (166,700)b $ 2,510,000

= Cumulative net income – Cumulative declared dividends = ($400,000 + $100,000 + $100,000 + $20,000) – [$90,000 from entry (2) + $50,000 from entry (8)] = $375,000 [from entry (4)] – $125,000 [from entry (5)] – $66,640 [from entry (7)] – $16,660 [from entry (10)]

P12–13 a. Only investors of shares that are both issued and outstanding are eligible to vote to elect a board of directors. If the current board could reduce the number of shares held by investors other than themselves, then they would control a greater proportion of the shares outstanding and could decrease the probability that Vadar, Inc., would take over Edmonds. One way that the board could consolidate its ownership position is to reacquire the company's common stock from other investors.


P12–13 Concluded If the current board could consolidate its ownership so that it controlled 50% plus one share of the outstanding shares, they could completely block Vadar, Inc.'s takeover attempt. In this particular case, the current board members own 140,000 shares (35%  400,000 shares outstanding). To allow the board to completely control the company without the board members personally acquiring any additional shares, the total number of shares outstanding could not exceed 279,999 [(140,000 shares  2) – 1 share]. Since 260,000 shares are currently held by non-board investors, and the non-board members can control a total of only 139,999 shares (279,999 shares – 140,000 shares held by board members), the company would have to repurchase 120,001 shares of the company's stock. b. The company will need to pay $50 per share for 120,001 shares [from part (a)]. Consequently, the company will need at least $6,000,050 in cash. Edmonds currently has insufficient cash to repurchase all 120,001 shares. If the board wants to block Vadar's takeover attempt by repurchasing some of its common stock, the company will have to borrow additional cash. c.

Assets Cash Other current assets Noncurrent assets

$ 1,149,950* 4,200,000 8,220,000

Total assets

$ 13,569,950

Liabilities and Stockholders' Equity Liabilities $ 5,250,000 Common stock 8,000,000 Retained earnings 6,320,000 Less: Treasury stock (6,000,050) Total liabilities and stockholders' equity $ 13,569,950

* $1,149,950 = $3,150,000 cash on hand + $4,000,000 cash borrowed –$6,000,050 cash paid out for stock d. Debt/Equity Ratio = Total Liabilities ÷ Total Stockholders' Equity Before $1,250,000 ÷ ($8,000,000 in Common Stock + $6,320,000 in Retained Earnings) = .087 After $5,250,000 ÷ ($8,000,000 in Common Stock + $6,320,000 in Retained Earnings – $6,000,050 in Treasury Stock) = .631 The purchase of the treasury stock had a large effect on the company's financial position, as evidenced by the large increase in the company's debt/equity ratio. The actual purchase of the treasury stock reduced Edmonds' stockholders' equity by the value of the stock purchased. Since Edmonds financed the purchase by issuing debt, the purchase directly increased Edmonds' long-term debt by the $4,000,000 borrowed.


P12–14 a. Loss on Write-Down of Fixed Assets (Lo, –SE) .......................................... Fixed Assets (–A)................................................................................. Wrote down obsolete fixed assets. b. Alternative 1 Cash (+A) ................................................................................................... Current Assets (–A) ............................................................................. Fixed Assets (–A)................................................................................. Liquidated assets.

50,000 50,000

650,000 200,000 450,000

Cash Dividends (–SE) ................................................................................. Cash (–A)............................................................................................. Declared and paid cash dividend.

250,000

Liabilities (–L)............................................................................................. Cash (–A)............................................................................................. Repaid liabilities.

400,000

Alternative 2 Cash (+A) ................................................................................................... Current Assets (–A) ............................................................................. Fixed Assets (–A)................................................................................. Liquidated assets.

250,000

400,000

650,000 200,000 450,000

Cash Dividends (–SE) ................................................................................. Cash (–A)............................................................................................. Declared and paid cash dividend.

400,000

Liabilities (–L)............................................................................................. Cash (–A)............................................................................................. Repaid liabilities.

250,000

Alternative 3 Cash (+A) ................................................................................................... Current Assets (–A) ............................................................................. Fixed Assets (–A)................................................................................. Liquidated assets.

400,000

250,000

650,000 200,000 450,000

Treasury Stock (–SE) .................................................................................. Cash (–A)............................................................................................. Purchased treasury stock.

250,000

Liabilities (–L)............................................................................................. Cash (–A)............................................................................................. Repaid liabilities.

400,000

250,000

400,000


P12–14 Concluded Alternative 4 Cash (+A) ................................................................................................... Current Assets (–A) ............................................................................. Fixed Assets (–A)................................................................................. Liquidated assets. Treasury Stock (–SE) .................................................................................. Cash (–A)............................................................................................. Purchased treasury stock. c.

650,000 200,000 450,000

650,000 650,000

In the event of a liquidation, creditors have the first claim on the company's assets. Consequently, obligations to creditors should be settled in full before the stockholders receive any residual assets. In alternatives (2) and (4), the board of directors has proposed circumventing the law and has planned to distribute assets to the stockholders first and then distribute the residual assets, if any, to the creditors. The creditors could sue the company and probably force the stockholders to return the "excess" dividends, which would then be used to satisfy the creditors' claims.


ISSUES FOR DISCUSSION ID12–1 a. A 100% stock dividend effectively means that the number of shares will be doubled. Therefore, if 149.5 million shares were outstanding before the stock dividend, then 299 million shares were outstanding after the dividend. b. Stock Dividend (–SE)............................................................................... Common Stock (+SE) ....................................................................... Declared and issued stock dividend.

149,500a 149,500

a

Because the stock dividend can be considered a 2:1 stock split par value for the 149.5 million shares is used. c.

Total Value of Hershey

Post dividend Value/Share

= Number of Shares Outstanding  Market Value per Share = 149.5 Million Shares  $46 per share = $6.877 Billion = Total Value of Hershey ÷ Number of Shares Outstanding = $6.877 Billion ÷ 299 Million Shares = $23 per Share

d. Prior to the stock dividend, Mr. Jones owned .669% of Hershey (1 million shares ÷ 149.5 million shares outstanding), and Mr. Jones' investment was worth $46 million (1 million shares  $46 market value per share). After the stock dividend, Mr. Jones would own 2,000,000 shares, and the total number of shares outstanding would increase to 299 million shares. Thus, Mr. Jones would still own .669% of Hershey (2 million ÷ 299 million shares outstanding). Because the market price would be expected to drop to $23 per share [see part (c)], Mr. Jones' investment would still be worth approximately $46 million. Thus, neither the percentage of shares owned nor the total value of an equity investment would be expected to be affected by a stock dividend, provided that the market price decreases proportionately to the increase in the number of shares outstanding. e. As demonstrated in part d, a stock dividend does not represent an economic exchange between a corporation and its shareholders. A stock dividend merely splits the shareholders’ interests into smaller pieces. The total value of the company and its assets and liabilities remain unchanged. f.

A company may issue a stock dividend for several reasons. These reasons could include wanting to give the appearance of paying a dividend when there is limited cash on hand, wanting to decrease the market price of the shares outstanding, or wanting to expand the number of shares outstanding without selling new stock.

ID12–2 There are two general situations that would lead a company to cut its dividend. The first scenario is when a company is no longer able to generate enough cash to pay the dividend that it has been paying in the past. This usually reflects a downturn in the company’s business and the stock price does not act well after this announcement. The second scenario is when a company has been paying a dividend and is still generating enough cash to pay the dividend but decides not to. This is usually because the company feels that it has


very good investment opportunities and wants to use the money to pursue them. This is usually a positive sign and the stock price may rise.

ID12–3 Since the stock price of a company is affected by the expected future cash inflows due to dividend payments and capital appreciation, the price of Philip Morris (now known as Altria Group) stock rose in response. After the $6 billion stock buy-back is completed, the company will have fewer shares outstanding in the market, and it will increase the future EPS and the book value of the company’s stock. Further, the company is raising the quarterly dividend by almost 20%, which means higher-cash inflow to the existing shareholders. All of these are positive signs and would lead to an increase in the price of Philip Morris’s common stock. However, the company’s credit rating will go down, primarily due to the fact that after paying an increased dividend and executing a $6 billion stock buy-back plan, Philip Morris may be low on cash, which could affect its short-term solvency position. Therefore, the rating agencies would perceive Philip Morris as a risky company. The higher the risk, the lower the credit rating.

ID12–4 a. Companies could use their cash to invest in the latest technology to improve the productivity of their manufacturing operations. Companies could use their excess cash to perform research and development to find new products for their markets. Companies could use their excess cash to acquire other companies to take advantage of economies of scale as a larger entity with less competition. Companies could use their cash to repay debt to lower interest costs and improve their performance during cyclical downturns. If any of the above alternate uses of cash increase the profitability and cash flow of the company, the shareholders benefit through capital appreciation (an increased stock price, for example), as well as the potential increase in dividend payments made possible by the improved performance of the company. If the alternate use of cash also improves the company’s standing in its industry, and positions it for future success, the shareholders benefit from the improve long term prospects of the company. b. Fewer shares in the market would imply that the market capitalization per share is higher, driving up the stock price. If the company’s performance (profitability, cash flow, leverage, etc.) is not changed but there are fewer shares of ownership, then each share would command a higher price as the total value of the company is divided among the equity providers. Driving up the share price strictly by reducing the number of shares outstanding is an artificial change as the company itself is no different. c.

A company purchasing its own stock is in essence no different than an investor purchasing stock in the company. “Buy low, sell high” makes sense for any investor. If, due to current market levels, the stock price is high, there is always the risk that the buyer—the company in this case—would be buying high with the possibility that a market correction could drive the stock price down to low levels. The company, similar to any investor, would then be holding a stock with a market value below the cost paid to acquire the stock. If a company’s stock is at a historically low point, repurchasing stock does not pose the same risk. However, if the stock price is very high, the possibility exists that the company would be purchasing an asset that will drop in value.

ID12–5


a. A hybrid security carries some of the properties of debt and some of the properties of equity. A determination must be made as to which properties dominate the characteristics of the security. If it is determined that the hybrid carries mainly equity characteritists, then it should be reported in the Stockholders’ Equity section of the Balance Sheet and should not affect the Income Statement. In either case, the hybrid security will be treated in the Financing Activities section of the Statement of Cash Flow. On the other hand, if the debt characteristics dominate, then the security should be classified as a liability on the Balance Sheet and will affect the Income Statement. b. Equity securities typically carry more risk than debt securities but consequently offer higher rates of return. If an investor was interested in accepting more risk than offered by a debt security, but required more of a return to justify that risk, then the investor might have an interest in a hybrid security. If the risk profile of a pure equity security was too great for the investor, but the returns offered by debt securities insufficient for the investor’s needs, then a hybrid security would offer a perfect compromise. A company might be interested in offering a hybrid security if it did not want to give up all that would be required by a traditional equity security or if it had no additional capacity to raise funds through traditional debt securities. An issue of preferred stock, for example, would allow the company to raise financing without giving up the voting rights associated with common stock; further, if the company could borrow no additional money from its banks and the capital markets, the preferred stock issue might be marketable as it would not carry the interest payments required each year under a debt issue.

c. Characteristics associated with debt include a higher priority toward receiving payments in case of a company liquidation and some type of fixed requirement of periodic payments. Characteristics associated with equity include a vote for the directors of the company and no fixed maturity date for the financial arrangement.

ID12–6 The case describes a typical scenario that usually occurs while enforcing the debt covenant restrictions. Many times such agreements or decrees explicitly specify how changes in the key performance indicators will be treated due to an unanticipated change in the accounting standards. In the current case it is not clear whether any such clause exists that deals with the effect of change in net worth due to an unforeseen accounting standard. From the perspective of an executive of Westinghouse, the company should not be forced to place the $325 million in the escrow account due to the following reasons: (1) Westinghouse’s net worth dipped below $1.9 billion due to a new method of accounting mandated by FASB. The decline was entirely due to factors beyond Westinghouse’s control. (2) The new method of accounting merely creates an additional expense and an additional liability on the books of the company. It does not impact the current cash flow situation. Therefore, there is no change in the economic situation of Westinghouse. Therefore, it should not be subjected to setting aside $325 million in an escrow account. From the perspective of a representative of the federal government, Westinghouse should place $325 million in an escrow account. The terms of the decree are very clear, which say that if the company’s net worth falls below $1.9 billion, it should place $325 million in an escrow account. As

P12–6 Concluded


a federal government representative, he has to ensure that the terms of the decree are complied with. From the perspective of a resident of Bloomington, Indiana, Westinghouse should place $325 million in an escrow account to build the incinerator to destroy the PCB-Contaminated materials. As a resident, one must be very concerned about the impact of PCB-contaminated materials on the health and safety of the town’s residents as well as the impact on the town’s environment. Several unanticipated factors may occur that may be out of the company’s control. These risks are inherent in doing business. The company has no right to transfer those risks to the community by not complying with the decree. Further, technically, the company was always liable for employee retirement costs, but it ignored that liability by adopting a liberal accounting policy of a pay-as-you-go basis. It is only fair that Bloomington, Indiana wants to ensure that the company pays for the hazardous waste cleanup, irrespective of the games that the company can play with the accounting numbers.

ID12–7 c.

An investor can make a return through 1) capital appreciation (the stock moving up in value), 2) receipt of cash dividends from the company, or 3) both capital appreciation and dividends.

d. Companies that pay dividends are not necessarily under the same pressure to improve earnings and cash flow as companies who opt not to pay dividends. Investors who are confident they can rely on a steady stream of dividends are less likely to pressure management to take steps to continually boost future cash flows to drive the stock price ever higher. Companies, on the other hand, who reinvest all earnings in their operations must provide a return to their shareholders in the form of higher stock prices; these companies will be under the gun to continually improve operations, grow their businesses, and improve cash flow and earnings in order to move the stock price. e. Investors will monitor sales, profits and earnings-per-share on the income statement, as well as cash flow from operations on the statement of cash flows. In addition to financial statement clues, investors can look for industry information, such as market share and consumer surveys to predict future financial results for the company.

ID12–8 a. Book value of a stock is simply the equity on the balance sheet divided by the number of equity shares outstanding. This figure is driven by Generally Accepted Accounting Principles. Market value is the value placed on a share of stock by all the buyers and sellers in the free market. This figure is driven by company results, but it is also influenced by factors such as general macroeconomic trends, stock market trends, and very often the subjective mood of investors. b. The price-to-book ratio is the market price (per share) divided by the book value (per share). Prior to the buyout offer, the price-to-book ratio of Dow Jones was 6.0 ($36 per share divided by $6 per share). After the buyout offer, the ratio jumped to 9.8 ($58.60 per share divided by $6 per share).

ID12–8 Concluded


c.

Two reasons might explain the high offer for Dow Jones. First, News Corporation might see some specific synergies of operation between its media businesses and those of Dow Jones (including the Wall Street Journal and the popular WSJ website); the combined businesses might be worth more to News Corporation than they would be to other investors. Other investors might only see $36 in value based on the projected cash flow of Dow Jones, but News Corporation might see $60 in value because of future business plans combining and growing the two companies together. Secondly, many market watchers speculated that News Corporation offered such a premium for Dow Jones simply to scare off other potential buyers in order to make the deal close faster. And, in retrospect, since no other bidders emerged, the $60 strategy may have been a sound approach to the acquisition.

d. External events (such as technological change) can cloud the future for companies and, since stock prices are driven by valuing a company’s future, affect the stock valuation. When events beyond the control of management change the prospects for a business, the market will react by pricing in those expected changes. If the internet will drive advertisers away from traditional newspapers, the future cash flow of those newspapers will drop; the market reacts by simply pricing the stock today in anticipation of those lowered expectations.

ID12–9 a. “Net” proceeds mean the actual cash received by the company after commissions to underwriters and expenses are paid. “Gross” proceeds would equal all the cash paid by investors buying the offered shares. b. Dividing the proceeds by the number of shares issued indicates that the stock price has been climbing rapidly. The proceeds per share issued climbed from $68.29 to $287.72 and then leveled off at $268.05 for the three years shown. c.

Google has been one of the major success stories for companies going public in the last decade. As internet traffic has increased, Google has captured a dominant share in search engine activity. The company’s business model sells advertising to companies, allowing the advertiser to directly target internet users based on search requests. More searches will mean more advertising dollars to Google. Technological changes and a very profitable business model have brightened the future for Google. The stock market has reacted to those heightened expectations for cash flow and profits by driving the stock price higher.

ID12–10 a. Both Rhodia and Segway issued equity because of losses that would be evident on the Income Statement. The influx of funds would be evident in the Financing Activities section of the Statement of Cash Flows. The Balance Sheet would also show an immediate increase in the Cash asset, but that would soon be converted to other assets as the company employed the funds. b. When companies issue new stock, the concept of ownership dilution must always be considered. If Aventis wants to maintain its 15.3 percent ownership of Rhodia, it must purchase 15.3 percent of the new shares issued. A large equity stake in another company, such as what Aventis has with Rhodia, is purchased for strategic control reasons. If Aventis wants to continue to significantly influence the operations of Rhodia, it must maintain its large ownership share in the company. If new shares are sold to the public, and if Aventis does not purchase its pro rata share, its 15.3 percent ownership stake will be diluted to something lower and Aventis’ influence might follow suit.


The original equity providers of Segway might have a similar interest in maintaining their control position of that company. If they do, they will have to purchase the percentage of the $31 million offering that matches their current ownership percentage in the company. However, if the investors are not concerned with maintaining the amount of influence in the company, they do not need to buy into the new offering. c.

Since both companies are experiencing profitability problems, before purchasing additional equity in the companies, a sound investor would need to understand the steps taken by management to correct the profitability problems. To assure that “good money was not chasing bad money”, a sound investor would meet with management, discuss the causes of past problems, and come to a firm understanding of and agreement with the remediation steps taken to restore the companies to profitability. Without sustained profits, no business can remain viable. A sound investor would need to believe that the companies will soon be in a position to show that revenues exceed operating expenses.

ID12–11 a. A 4:1 stock split means that the corporation will replace every share of outstanding stock with 4 shares of the stock. Because the corporation is simply splitting the ownership shares into smaller pieces, declaring or issuing a stock split does not affect the dollar amounts Walt Disney would report in its financial statements. However, a 4:1 stock split would result in (1) the number of shares of common stock Disney had outstanding to increase by a factor of four and (2) the par value per share to drop to 25% of its pre-stock split value. b. The stock market often views the declaration of a stock split as a positive signal about the corporation. For example, the market often perceives a stock split as a signal from management that they believe that they can maintain the price of the stock despite the stock split. In addition, some companies increase their dividend per share soon after splitting their stock. Such positive signals, in turn, increase demand for the company's stock, which drives up the price of the stock. In addition, stock splits would be expected to cause the market price per share to drop once the stock split actually takes place to reflect the additional shares of stock outstanding. The lower market price would make the stock more affordable to more investors, which should increase demand for the stock and drive up the price. c.

Theoretically, the value of a company's stock equals the present value of the cash flows the stock market expects an investor to generate from an equity investment in the company. Any information that would cause the stock market to change its expectations--either positively or negatively—about the future cash flows that would be realized from an investment would result in a change in the price of the company's stock. Thus, if the market perceives the declaration of a stock split as positive new information about the company's future prospects, as suggested by the quote from The Wall Street Journal, then the market would be expected to use this information and increase the price of the company's stock.

ID12–12 a. When a stock price is discussed in terms of a multiple of earnings, say 45 times, it simply means that the market price per share is 45 times the size of the earnings per share. Or, every penny of earnings is currently valued by the market at 45 cents. This “multiple” is often cited as a basis for how over- or under-priced a stock is by the market. When stock markets highly value a stock, its multiple will be very high (say 45 times), but when markets turn cold to a stock, that multiple drops, meaning every penny of earnings is worth less to investors in the market. b. A volatile market will challenge a company that is intent on repurchasing its own shares. As with any investor, companies buying their own shares do not wish to overpay for their purchases. If a volatile market exists and stock prices are jumping all around, the company with a committed


buyback program will be paying many different prices (some high, some low) for the shares it repurchases. c. Stock repurchases will reduce the company’s cash balances (affecting both liquidity and solvency) and will lower the company’s equity base (increasing leverage). Subsequent stock re-issuances will increase cash balances and equity levels, but the reissue price will determine if those equity levels will be fully restored. If the stock is reissued for a lower price than the acquisition price, a permanent hit to equity will occur (adjusted through Additional Paid In Capital, assuming APIC balances are sufficient). If, on the other hand, the stock is re-issued for a higher price, then an increase in equity balances will occur. d. Borrowing to repurchase shares has a double effect on a company’s leverage. The borrowing, of course, increases the companies liabilities, while the Treasury Stock accounting decreases the company’s equity levels. Both treatments (+L, -SE) increase a company’s leverage. e. Reissuing Treasury Stock is considered to be a capital transaction and, as such, does not affect company’s income statement and its level of profitability.

a

ID12–13 a. Stock-based compensation expense is the expense related to the granting (and subsequent exercise) of stock options to employees. If an employee is given the option to purchase shares of company stock from the company, the cost of those shares to the company has to be included as a form of compensation expense, just as cash paid to an employee for services rendered is considered compensation expense. The calculation of the actual cost of the shares to the company is still subject of much debate and controversy, but the fact that options are a form of compensation is generally accepted. b. As the shares are issued (or re-issued from treasury), the statement of stockholders’ equity must reflect the change in shares outstanding. Since the options are now considered an expense, the income statement is also affected, as is the statement of cash flows (and balance sheet) for any cash received by the company when the options are exercised by the employee.

c.

If the options were not considered an expense, Microsoft’s net income would have been considerably higher in the years shown, so the expense is definitely material to the company. Net income would have been higher by 11.6%, 8.5% and 11.4% for 2009, 2008 and 2007, respectively, if the cost of the options had not been expensed.

d. Many companies resisted the expensing of options due to the negative effect the move would have on earnings. Many start-up and technology companies relied heavily on stock options for employee compensation. These companies knew that the large amount of options exercised by their employees would dramatically increase expenses and reduce earnings.

ID12–14 The Statement of Stockholders’ Equity for Emerson Electric for three years is shown at the end of Chapter 12 in the textbook. Under the Retained Earnings section, the dividend history for 2010 – 2012 can be seen.


The company has consistently paid cash dividends in each of the years. On a per share basis, the dividend payment is rising from 2010’s $1.34 per share to 2011’s $1.38 per share to 2012’s $1.60 per share. As a percentage of net income, dividends represented 46.6% (2010), 41.9% (2011), and 59.5% (2012). The large increase in percentage of earnings paid out in dividends in 2012 is driven by a slight increase in dividend dollars but more so by the drop in dollar profits (from $2.48 billion to $1.97 billion). In 2010, $100 million of shares were repurchased, but that amount grew to $958 million in 2011; in the latest year the amount fell to $787 million. Even though the company is reissuing shares annually, due to stock option plans, due to the size discrepancy of repurchases versus re-issues, the net effect is to remove shares from the marketplace.

ID12–15 a. Companies can spend their cash balances on a number of areas: they can expand physical capacity, they can acquire other businesses, they can invest in new technologies, they can pay cash dividends, or they can use their cash to repurchase their own shares. Companies that repurchase shares are saying that the best use of cash to build value for the shareholder is to use that cash to reduce the number of shares in the market. Companies that do not repurchase their shares feel that there is a better use for cash balances, a better way to build value for the shareholder. b. HP and Apple have been very successful companies, using cash balances to acquire other businesses (HP) and to reinvest in new product launches (Apple). Bear Stearns, on the other hand, was forced into a fire sale, selling itself to another financial services firm in order to avoid a bankruptcy filing. c.

Companies can create wealth for shareholders by building and running successful businesses (leading to higher share value), by paying cash dividends from strong earnings, and by repurchasing stock from shareholders.

ID12–16 a. Net income is the difference between revenue and expenses for a given time period. Comprehensive income includes net income but also includes any other changes to shareholders’ equity (such as unrealized gains/losses on available-for-sale securities and changes to currency exchange rates). b. Comprehensive income is for a given time period, such as a year, while accumulated other comprehensive income is a running total of past comprehensive incomes. The difference between comprehensive income and accumulated other comprehensive income can be thought of as the difference between the income statement and the balance sheet: one is for a period of time while the other is an accumulation as of a point in time. c.

Under U.S. GAAP, the information must be provided as either a separate statement or part of the Statement of Shareholders’ Equity. Under IFRS, the information must be provided as a separate statement or an extension of the Income Statement.

ID12–17 a.


As of 12/31/2012: Liabilities ÷ Total Assets = 23.5% Contributed Capital ÷ Total Assets = 9.1% Retained Earnings ÷ Total Assets = 67.4%

b. Common Stock: 21 billion shares authorized and 654.874 million shares issued and outstanding. c.

Other than shares repurchased in an acquisition, the company spent no cash over the last three years on either dividends or share buybacks.

f.

Although preferred stock has been authorized, no shares have been issued.

g. For the year ending 12/31/2012 Google added back stock-based compensation expense of $2.7 billion. This expense reduced earnings but has not yet cost the company cash, so the amount is added back in the computation of cash from operations. h. Over the time period, Google’s stock has traded from $474.88 per share to $768.05 per share. i.

The Statement of Cash Flows indicates that the company has issued debt in the last three years but not equity. Footnote #12 indicates that shares granted in stock option plans have been exercised at an average price of $305.81 in 2012.

j.

Google’s accumulated other comprehensive income as of 12/31/2012 was $538 million. This balance consists primarily of unrealized gains on available-for-sale securities. Comprehensive Income was $10,999 million (slightly higher than net income of $10,737 million).


CHAPTER 13 THE COMPLETE INCOME STATEMENT BRIEF EXERCISES BE13–1 To accurately compare a company’s performance from one year to the next, non-recurring items such as debt retirements and accounting changes, as well as the activities of discontinued operations, should be excluded from results. An analyst reviewing a company’s performance would want to see those items that should occur in every year of a company’s operations, not the one-time, special expenses and gains. In Goodrich’s case, an analyst would compare 2011 and 2010, focusing on recurring revenues and expenses only; those items in the two years that could not be expected to happen in future years would be excluded from the analysis. A comparison of Goodrich’s operating results would then give the anlayst an idea toward the company’s future results.

BE13–2 ASSETS

=

Closure of Tampa Breweries: Write down of plant assets –113.7 Employee severence costs –19.4 Disposal costs

LIABILITIES +

= = =

Sale of St. Louis Cardinals: Sale of team (+cash, – assets) +95.3, –95.3 Gain on sale of team +54.7

=

+26.9

OWNERS’ EQUITY

–113.7 –19.4 –26.9

+54.7

BE13–3 Glancing only at Merck’s net income trend might give an analyst an incorrect impression regarding the company’s operations. However, an analyst would like to exclude the special items that cannot be counted on to recur in future periods. The large amount in “other income” (from $1.3 billion in 2010 to $1.1 billion in 2012) contributes greatly to the company’s bottom line, but an analyst would certainly question if the billion dollar level will happen again in 2013. Another one-time charge that affects the profit trend line is the restructuring charge. The equity income from affiliates is a more difficult read for an analyst. Given Merck’s business strategy, long term investments (that exert significant influence but not control) in other companies will be a recurring part of the business; however, the equity income shown on the income statement does not represent a cash inflow to Merck as it is simply Merck’s share of the other company’s profitability.

1


BE13–3 Concluded An analyst would certainly be interested in computing the recurring revenues and expenses from Merck’s core business: (amount in billions) 2012 2011 2010 Sales $47.3 $48.0 $46.0 Materials, Production Expenses 16.5 16.9 18.4 Marketing, Administrative Expenses 12.8 13.7 13.1 R & D Expenses 8.2 8.5 11.1 Operating Income $ 9.8 $ 8.9 $ 3.4 Op Income/Sales 20.7% 18.5% 7.4% This analysis actually shows an improving trend in profitability based on the revenues and expenses that are recurring in nature. .

BE13–4 a. Comprehensive Income can be calculated as: Net Income $266 million Add: Unrealized Gain AfS Securities 3.1 million Less: Foreign Exchange Loss 2.2 million Comprehensive Income $266.9 million b. The balance for Accumulated Other Comprehensive Income is included in the Stockholders’ Equity section of the balance sheet, to reflect the changes in equity that do not originate from profitability. Comprehensive Income (the change to Accumulated Other Comprehensive Income) is reflected at the bottom of the Income Statement, below the calculation of Net Income. Finally, the Statement of Stockholders’ Equity will contain a column showing all the changes to Accumulated Other Comprehensive Income (with the ending balance matching what is shown on the Balance Sheet).


EXERCISES E13–1 a. Statement 1.

IS

IS

6.

IS

7.

IS

8. 9. 10.

SE SE IS

11.

IS

12.

IS

c. Explanation

Other This is an example of a bookkeeping entry without an underlying economic event. Some are involuntary. N/A N/A Usual and frequent Wages are a normal, recurring part of operations. Usual and frequent Bad debts are a normal, recurring part of operations. Unusual or infrequent Sales of equipment are secondary to normal operations. Unusual and infrequent Expropriation against a company occurs rarely in the U.S. N/A N/A Unusual or infrequent Dividend Revenue is secondary to normal buying and selling activities of most businesses. Usual and frequent Cost of goods sold is part of normal daily operations. Usual and frequent Rent expense is part of normal operations.

voluntary, some are 2. N 3. N 4. IS 5.

b. Classification

E13–2 a.

1. 2. 3. 4. 5. 6. 7. 8.

1 4 5 5 4 5 1 1

9. 10. 11. 12. 13. 14. 15.

2 5 5 5 1 5 3

b. As the transactions move from 1 to 5, they move away from exchanges with owners toward exchanges with third parties involving the company's central, ongoing activities. Stockholders and creditors are considered providers of capital, and this financing provides capital that is necessary for the company to be able to operate. However, the company does not exist to draw capital from the equity and debt investors; instead, it exists to provide goods and/or services to other entities. Providing goods and/or services is considered the company's operations.


E13–3 a. (1) (2) (3) (4) (5)

Financing Operating Operating Financing Financing Smedley Company Income Statement For the Year Ended December 31, 2016 Fees earned ......................................................................................................... Expenses ............................................................................................................ Net income ..........................................................................................................

b. Comprehensive Income

$ 50,000 24,000 $ 26,000

= Change in Equity from Nonowner Sources = Revenues – Expenses + Gains – Losses + Cumulative Accounting Adjustments = $50,000 – $24,000 + 0 – 0 = $26,000

* Fees and expenses represent Smedley’s only nonowner changes in equity. c.

The two are equal because Smedley’s economic events/activities do not differ from those described in the income statement.

E13–4 a. (1)

(2)

(3)

(4)

Net income Maximum Dividend

= 20%  $182,800 = $36,560 Income before change in accounting method Maximum Dividend = 20%  $130,800 = $26,160 Income before extraordinary items Maximum Dividend = 20%  $108,800 = $21,760 Net operating income Maximum Dividend = 20%  $150,000 = $30,000

b. The bank requires restrictions on dividend payments to increase the probability that Morton Manufacturing will have sufficient cash to meet its interest and principal payments. More stringent restrictions on dividends decrease the amount that Morton Manufacturing can potentially pay out as dividends, thereby increasing the probability that the company will have sufficient cash to meet its obligations to the bank. In this particular case, basing the dividend restriction on net operating income provides the most stringent dividend restriction. Further, managers have greater ability to manipulate items disclosed after net operating income than those items disclosed before it. For example, the decision to sell assets and realize gains is completely at a manager's discretion, as is a change in accounting principle resulting in an increase in net income.


Therefore, basing the dividend restriction on net operating income limits management's ability to manipulate the restriction.

E13–5 a. Losses from equity investments: AT&T invested excess cash in the equity of other companies; the value of these investments dropped during the course of the year; Loss from discontinued operations: AT&T closed down or sold off business units during the year; the business units had generated losses (expenses exceeded revenues), but those units are no longer operating as part of the company; Gain on disposition of discontinued operations: The discontinued business units were sold for a price that exceeded the book value of the units; Gain/Loss from accounting changes: AT&T made a one-time change to its accounting treatment of particular transactions and the change resulted in either a gain or loss.

b.

Reported net income

($ in billions)

Loss on equity investments Loss from discontinued operations Gain from sale of discontinued operations Gain from accounting changes

Year 1

Year 2

Year 3

$7.7

($13)

$1.9

7.5

0.4 4.0

(1.3)

14.5

0.01

(1.3)

(0.9)

Loss from accounting changes Operating income

0.01

(.02) 0.9

$17 .0

$1.5

$1.9

Each of these special items are not part of operating income, which represents the recurring activities of the business. These special charges are one time gains or losses and so can mislead the users of financial statements as to the future prospects of the company. The schedule above shows that the operating income of AT&T dropped significantly from Year 1 levels, but that the performance between Year 2 and Year 3 is more consistent than shown by the net income figures. Adjusting net income for the nonrecurring items gives a clearer picture of the company’s performance; business has changed significantly from Year 1 levels, but performance in the last two years has been more consistent.

E13–6 a. Cash (+A) ................................................................................................... Liabilities (–L)............................................................................................. Assets (–A) .......................................................................................... Gain on Disposal of Business Segment (Ga, +SE) ............................... Sold business segment.

625,000 1,400,000 1,850,000 175,000


E13–6 Concluded Gain on Disposal of Business Segment (–Ga, –SE) .................................... Income Tax Liability (+L) ..................................................................... Recognized income tax liability on disposal of business segment.

61,250* 61,250

$61,250 = Gain on disposal of $175,000  Tax rate of 35%

Income From Operations of Discontinued Segment (–Ga, –SE)................ Income Tax Liability (+L) ..................................................................... Recognized income tax benefit on net loss of discontinued segment. *

5,250* 5,250

$5,250 = Net income of $15,000  Tax rate of 35%

b. Discontinued operations: Income from operations of discontinued segment (net of tax expense of $5,250) ................................................................. Gain on disposal of business segment (net of $61,250 in taxes) ....... Discontinued operations .............................................................

$ 9,750 113,750 $ 123,500

E13–7 a.

Mallory Services Income Statement For the Year Ended December 31, 2015 Income from continuing operations (before taxes) ................... Income tax expense .................................................................... Income from continuing operations........................................... Discontinued operations: Income earned by discontinued segment (net of tax expense of $4,200)......................................................... $ 7,800b Gain on disposal of discontinued segment (net of tax expense of $13,300)....................................................... 24,700c Discontinued operations .................................................. Net income ................................................................................. _________________ a $332,500 = $950,000  35%

$ 950,000 332,500a $ 617,500

32,500 $ 650,000

b Income = Revenues – Expenses  (1 – tax rate)

$7,800 = ($35,000 – $23,000  (1– 35%) c Gain = Sale price – Book value  (1– tax rate)

$24,700 = [$105,000 – ($93,000 – $26,000)]  (1 – 35%) Earnings persistence reflects the extent to which a particular dollar of earnings can be expected to continue in the future. A user would expect the income from continuing operations of $617,500 to continue in the future, but interpret the income and gain from the discontinued division to reflect a one-time increase in earnings that is not expected to be realized in the future.


E13–7 Concluded b.

Mallory Services Income Statement For the Year Ended December 31, 2015 Income from continuing operations (before taxes) ............................................ Income tax expense ............................................................................................. Net income ..........................................................................................................

$ 962,000a 336,700b $ 625,300

______________________________

a Income from continuing operations = Income excluding clerical division + Activities

of clerical division $962,000 = $950,000 + ($35,000 – $23,000) b $336,700 = $962,000  35%

A user would expect the entire amount of net income to persist in the future. No items are separated out as one-time unusual and/or infrequent. c.

Sharon Sowers faces a number of trade-offs as she decides whether to complete the sale in 2015 or 2016. First, income is already positive and strong in 2015. The gain from the sale increases net income by only $24,700. If Mallory Services expects a weaker 2016, it may be more beneficial to the income statement to wait until 2016 in order to offset poorer performance. Also, by selling in 2015, recurring income from operations isn’t as large as it could be. Users would interpret the financial statements as $617,500 persisting into the future versus $625,300 if the division isn’t sold until 2016. On the other hand, Sharon may receive bonuses on net income and may prefer to receive the bonus now or may even need the gain caused by the disposal to increase net income to some minimum amount for a bonus. Also Mallory Services may face debt convenants that need to be covered by a larger amount of income. Finally, Susan may be able to use the cash from the sale to invest in alternative projects that immediately earn a higher return than produced by the clerical division.

E13–8 a.

Carmich Industries Income Statement For the Year Ended December 31, 2015 Income from continuing operations (before taxes) ................... Income tax expense .................................................................... Income from continuing operations........................................... Discontinued operations: Income earned by discontinued segment (net of tax expense of $8,750)......................................................... $ 16,250b Gain on disposal of discontinued segment (net of tax expense of $45,850)....................................................... 85,150c Discontinued operations .................................................. Net income ................................................................................. a $675,500 = $1,930,000  35%

$ 1,930,000 675,500a $ 1,254,500

101,400 $ 1,355,900


b

Income = Revenues – Expenses  (1 – tax rate) $16,250 =($145,000 – $120,000  (1 – 35%)

c Gain = Sale price – Book value  (1 – tax rate)

$85,150 = [$350,000 – ($437,000 – $218,000]  (1 – 35%)

E13–8

Concluded Earnings persistence reflects the extent to which a particular dollar of earnings can be expected to continue in the future. A user would expect the income from continuing operations of $1,254,500 to continue in the future, but interpret the income and gain from the discontinued division to reflect a one-time increase in earnings that is not expected to be realized in the future.

b. Carmich Industries Income Statement For the Year Ended December 31, 2015 Income from continuing operations (before taxes) ......................................... Income tax expense .......................................................................................... Net income .......................................................................................................

$ 1,955,000a 684,250b $ 1,270,750

a Income from continuing operations = Income excluding clerical division + Activities of clerical

division $1,955,000 = $1,930,000 + $25,000 b $684,250 = $1,955,000  35%

A user would expect the entire amount of net income to persist in the future. No items are separated out as one-time, unusual, and/or infrequent. c.

Rob Blandig faces a number of trade-offs as he decides whether to complete the sale in 2015 or 2016. First, income is already positive and strong in 2015. The gain from the sale increases net income by only $85,150, only 6.7%,. If Carmich Industries anticipates a weaker 2016, it may be more beneficial to the income statement to wait until 2016 in order to offset poorer performance. Also, by selling in 2015, recurring income from operations isn’t as large as it could be. Users would interpret the financial statements as $1,254,500 persisting into the future versus $1,270,750 if the division isn’t sold until 2016. Also, Rob and other top management receive a 20% bonus on income from continuing operations. This means that top management earns an additional 20% on the income from the division if it is not sold in 2015. On the other hand, Carmich Industries may face debt convenants that need to be covered by a larger amount of income. Also, Rob may be able to use the cash from the sale to invest in alternative projects that immediately earn a higher return than produced by the chemical division and increase income and his bonus even higher next year.


E13–9 a. Net Sales is the revenue generated by selling products/services to customers. Operating expenses are those charges that make the revenue possible. Operating income is the profitability associated with the basic (recurring) operations of the company. Net cost of debt is the interest expense net of any interest income. Income taxes is the expense related to the taxes incurred on the company’s profits. Net income of equity-accounted affiliates is Group Danone’s share of the income of companies in which Danone owns a significant (but not controlling) portion; this income statement item is a non-cash addition to profitability. Net income from continuing operations is the bottom-line profitability, after taxes, of all the operations of the company that will recur in the following fiscal period. Net income from discontinued operations is the profitability from companies owned during the year but sold or closed by the end of the year; in other words, the discontinued operations will not recur in the next year for Group Danone. b. During the year Group Danone sold or closed operations that generated 3.3 million euros in profitability; this level of earnings represented 76% of Danone’s total profits for the year, as the company’s continuing operations (those that will occur again in the next year) earned only 1.0 million euros. An analyst interested in evaluating the future performance of Group Danone would look to the continuing operations as the guide for future results, not the total operations; the 3.3 million euros earnings will not recur for Group Danone in the next year and beyond.

E13–10 a. Loss on Destruction of Inventory (Lo, –SE)................................................ Inventory (–A) ..................................................................................... Recognized inventory loss from earthquake.

2,325,000

b. Extraordinary Loss on Destruction of Inventory (Lo, –SE) ......................... Inventory (–A) ..................................................................................... Recognized inventory loss from earthquake.

2,325,000

Income Tax Liability (–L) ............................................................................ Extraordinary Loss on Destruction of Inventory (–Lo, +SE) ................ Recognized income tax benefit on inventory loss from earthquake.

813,750*

2,325,000

2,325,000

813,750

* $813,750 = Loss of $2,325,000  Tax rate of 35% c.

If the earthquake is considered both an unusual and infrequent event, then the loss should be classified as an extraordinary loss. However, if the loss is considered unusual or infrequent, but not both, then it should be classified as other revenues and expenses. If the loss is considered both usual and frequent, then it should be disclosed as part of operations. Since Paxson Corporation's plant is located in San Francisco, then the magnitude of the earthquake would have to be considered. To be considered an extraordinary loss, the events that gave rise to the loss must be both unusual in nature and occur infrequently. In San Francisco, minor earthquakes are not infrequent; they are expected to occur occasionally. Consequently, if this loss was due to a minor earthquake, it would not be considered extraordinary. Since earthquakes are not usual (i.e., part of the company's normal operations), the loss should not be considered as part of operations. Instead, the loss should be classified as another expense. Alternatively, if the damage was due to a major


earthquake, it might qualify as an extraordinary loss. Major earthquakes, such as the 1906 and 1989 earthquakes that devastated the Bay area, are both unusual and infrequent. d. If Paxson’s plant was located in Miami, Florida, then any earthquake is considered both unusual and infrequent and would qualify as an extraordinary loss.

E13–11 a. If a lawsuit is considered unusual but not infrequent, then it would be classified under other expenses and losses. Consequently, the loss from the lawsuit would be used to compute net income from continuing operations. Bonus = 12%  [($800,000 – $480,000)  (1 – Tax Rate)] = 12%  [$320,000  (1 – 35%)] = $24,960 b. If the loss from the lawsuit is considered extraordinary, then the loss would not be used to compute net income from continuing operations. Bonus = 12%  [$800,000  (1 – Tax Rate)] = 12%  [$800,000  (1 – 35%)] = $62,400 c.

Gain not considered extraordinary Bonus = 12%  [($800,000 + $480,000)  (1 – Tax Rate)] = 12%  [$1,280,000  (1 – 35%)] = $99,840 Gain considered extraordinary Bonus = 12%  [$800,000  (1 – Tax Rate)] = 12%  [$800,000  (1 – 35%)] = $62,400

d. Income figures are often used as the basis for awarding incentive compensation to managers, or included in ratios which are, in turn, incorporated into debt covenants. If the income figures included in incentive contracts or in debt covenants stipulate between income from continuing operations and net income, then the decision of whether or not to classify an event as extraordinary or not can have economic consequences. As demonstrated in parts (a) through (c), how a loss or gain is classified can have a profound effect on the magnitude of a manager's bonus.

E13–12 a. Income from continuing operations ........................................ Disposal of business segment* ................................................ Extraordinary loss ..................................................................... Changes in accounting method ................................................ Net earnings per share .............................................................

$ 0.73 0.33 (0.46) 0.40 $ 1.00

b. Income from continuing operations ........................................ Disposal of business segment* ................................................ Extraordinary loss ..................................................................... Changes in accounting method ................................................

$ 0.44 0.20 (0.28) 0.24


c.

Net earnings per share .............................................................

$ 0.60

Income from continuing operations ........................................ Disposal of business segment* ................................................ Extraordinary loss ..................................................................... Changes in accounting method ................................................ Net earnings per share .............................................................

$ 0.37 0.16 (0.23) 0.20 $ 0.50

* The EPS disclosure for the disposal of the business segment includes both the income from the disposed segment and the gain on the sale of the disposed segment.

E13–13 a. Rothrock Consolidated Income Statement For the Year Ended December 31, 2015 Revenue: Operating revenues ............................................................... $ 87,000 Total revenue......................................................................... Expenses: Operating expenses ............................................................... Other revenue ............................................................................ Income from continuing operations (before taxes) ................... Income tax expense .................................................................... Income from continuing operations........................................... Discontinued operations: Income earned by discontinued segment (net of taxes of $1,050) ................................................................... $ 1,950 Loss on disposal of discontinued segment (net of tax benefit of $7,350) .......................................................... (13,650) Discontinued operations ....................................................... Income before extraordinary items ........................................... Extraordinary loss (net of tax benefit of $1,750) ....................... Income before change in accounting method ........................... Effect of change in accounting method (net of taxes of $4,375) ..................................................................... Net income .................................................................................

$ 87,000 32,500 5,200 $ 59,700 20,895 $ 38,805

(11,700) $ 27,105 (3,250) $ 23,855 8,125 $ 31,980

b. Rothrock Consolidated Statement of Retained Earnings For the Year Ended December 31, 2015 Retained earnings, January 1, 2015..................................................................... Plus: Net income.................................................................................................. Less: Dividends declared .....................................................................................

$ 72,000 31,980 (18,000)


Retained earnings, December 31, 2015 ..............................................................

$ 85,980


E13–14 a. Watson Company Income Statement For the Year Ended December 31, 2015 Sales revenues ............................................................................ Cost of goods sold ...................................................................... Gross profit ................................................................................. Operating expenses: Administrative expenses ....................................................... $ Depreciation expense ............................................................ Selling expenses..................................................................... Total expenses.................................................................. Income from operations ............................................................. Other revenues and expenses: Rent revenue ......................................................................... $ Loss on sale of fixed assets .................................................... Total other revenues and expenses ................................. Income from continuing operations (before taxes) ................... Income taxes .............................................................................. Income from continuing operations........................................... Extraordinary loss (net of tax benefit of $70,700) ..................... Net income .................................................................................

$1,385,000 475,000 $ 910,000 100,000 250,000 189,000 539,000 $ 371,000 360,000 (105,000) 255,000 $ 626,000 219,100 $ 406,900 (131,300) $ 275,600

b. Watson Company Statement of Retained Earnings For the Year Ended December 31, 2015 Retained earnings, January 1, 2015..................................................................... Plus: Net income.................................................................................................. Less: Dividends .................................................................................................... Retained earnings, December 31, 2015 .............................................................. c.

$ 847,000 275,600 (460,000) $ 662,600

If Watson Company had no tax liability as of January 1, 2015, and made no tax payments during 2015, then the company's tax liability as of December 31, 2015 would equal the sum of the intraperiod tax allocations. Watson Company incurred $219,100 in taxes associated with income from continuing operations. The extraordinary loss provided a tax benefit, thereby reducing income taxes by $70,700. Therefore, the company's total tax liability as of December 31, 2015 would be $148,400.


E13–15 a. Kennington Company Income Statement For the Year Ended December 31, 2015 Income from continuing operations (after taxes) ...................... Loss on lawsuit (net of tax benefit of $21,700) .......................... Income from continuing operations........................................... Gain on disposal of discontinued segment (net of tax expense of $8,750)............................................... $ 16,250 Extraordinary loss on early retirement of debt (net of tax benefit of $13,300) .............................................. (24,700) Net income .................................................................................

$ 235,000 (40,300) $ 194,700

(8,450) $ 186,250

Maximum dividends Kennington can declare = Income from continuing operations  15% $29,205 = $194,700  15% b. Kennington Company Income Statement For the Year Ended December 31, 2015 Income from continuing operations (after taxes) ...................... Gain from sale of short-term investment (net of tax expense of $8,750)............................................... Income from continuing operations........................................... Gain on disposal of discontinued segment Extraordinary loss on lawsuit, (net tax benefit of $21,700)................................................... $ (40,300) Extraordinary loss on early retirement of debt (net of tax benefit of $13,300) .............................................. (24,700) Net income .................................................................................

$ 235,000 16,250 $ 251,250

(65,000) $ 186,250

Maximum dividends Kennington can declare = Income from continuing operations  15% $37,687.50 = $251,250  15%


E13–16 a. Madigan International Income Statement For the Year Ended December 31, 2015 Income from continuing operations (before taxes) ................... Gain on sale of subsidiary ........................................................... $ 42,000 Gain due to change in accounting principle ............................... 25,000 Income from continuing operations........................................... Income tax expense .................................................................... Income from continuing operations........................................... Extraordinary items: Loss of inventory due to earthquake, (net tax benefit of $18,550) ............................................. $ (34,450) Loss due to write-off of accounts receivable, (net tax benefit of $13,300) ............................................. (24,700) Extraordinary items............................................................ Net income .................................................................................

$ 865,000 67,000 $ 932,000 326,200a $ 605,800

(59,150) $ 546,650

a$326,200 = $932,000  35%

b. Madigan’s accountants are not presenting income statement items in accordance with generally accepted accounting principles. Instead, they are presenting all the gain items as part of income from continuing operations and all the loss items as extraordinary. The intention is to represent to investors and other users a higher income from continuing operations than actually exists. The greater is income from continuing operations, the more income investors and users expect will exist in the future. The gain on the sale of the subsidiary should be reported as a discontinued operation separately after income from continuing operations. The gain from the change in accounting principles should be reported separately after income from continuing operations. The loss due to the earthquake may be extraordinary as reported by Madigan, depending on the location of Madigan and the magnitude of the earthquake. The loss due to a write-off of accounts receivable, however, occurs in the ordinary course of business and should not be reported as an extraordinary item.


E13–16 Concluded The income statement that is consistent with GAAP follows. Madigan International Income Statement For the Year Ended December 31, 2015 Income from continuing operations (before taxes) ................... Loss due to write-off of accounts receivable ............................. Income from continuing operations........................................... Income tax expense .................................................................... Income from continuing operations........................................... Gain on discontinued operations, (net of tax expense of $14,700)............................................. $ 27,300 Extraordinary loss on inventory due to earthquake, (net of tax benefit of $18,550) .............................................. (34,450) Gain due to change in accounting principle, (net of tax expense of $8,750)............................................... 16,250 Net income .................................................................................

$ 865,000 (38,000) $ 827,000 289,450a $ 537,550

9,100 $ 546,650

a $289,450 = $827,000  35%

c. Madigan’s original treatment makes future income look more favorable. Stock prices tend to follow a multiple of earnings, so higher future income would lead to a higher future stock price.

E13–17 a. 2014

2015

2016

2017

2018

Net Income 250,000 -200,000* 50,000 250,000 -200,000* 50,000 250,000 -200,000* 50,000 250,000 -200,000* 50,000 250,000 -200,000* 50,000

Balance Sheet Value

800,000**

600,000

400,000

200,000

0

*straight-line depreciation expense = $1,000,000/5 years **Book Value = Cost – Accumulated Depreciation


E13–17 Concluded b. Net Income Balance Sheet Value 2014 250,000 -400,000* (150,000) 600,000** 2015 250,000 -240,000* 10,000 360,000 2016 250,000 -144,000* 106,000 216,000 2017 250,000 - 86,400* 163,600 129,600 2018 250,000 -129,600* 120,400 0 *double declining balance depreciation expense = 2 x Book Value/5 years **Book Value = Cost – Accumulated Depreciation c.

Net Income Balance Sheet Value 2014 250,000 -210,000* 40,000 790,000** 2015 250,000 -168,000* 82,000 622,000 2016 250,000 -190,000* 60,000 432,000 2017 250,000 - 205,000* 45,000 227,000 2018 250,000 -227,000* 23,000 0 *depreciation expense = change in market value **Book Value = Cost – Accumulated Depreciation

d. All three approaches yield the same total profitability over five years (each method totals $250,000 in profitability), but the methods allocate those profits differently. The straight-line method produces equal profits of $50,000 per year, while the double-declining balance method results in lower net income in the earlier years (due to the acceleration of the depreciation expenses) and higher net income in the later years. Finally, the market value approach ties the fleet expense to the drop in market value of the fleet assets, producing a fluctuating annual net income (that nonetheless totals the same $250,000 over the life of the fleet).


PROBLEMS P13–1 a. 1. 2. 3. 4. 5. 6. 7. 8. 9.

Operating Operating Operating Financing Operating Operating Operating Financing Operating & Investing

b. 1. 2. 3. 4. 5. 6. 7. 8. 9.

Normal and recurring Normal and recurring Other revenues and expenses Issues and payments of debt Normal and recurring Other revenues and expenses Changes in accounting principles Exchanges with stockholders The sale of the securities would be classified as purchases, sales, and exchanges of assets, and the loss would be classified as other revenues and expenses.

c.

Items (2), (3), (5), (6), (7), and (9) would all be disclosed on the income statement as follows. (2)

The $500,000 would be disclosed as operating revenues, and the $375,000 would be disclosed as cost of goods sold, which is an operating expense.

(3)

Minor earthquakes are not infrequent in San Francisco, although they are unusual. Thus, the $100,000 would be disclosed as part of other revenues and expenses.

(5)

The $143,000 would be disclosed as operating expenses.

(6)

Lawsuits are unusual, but it appears that the company is frequently sued. Thus, the $10,000 would be disclosed as part of other revenues and expenses.

(7)

The effect of the change in accounting methods would be disclosed, net of any tax effect, after extraordinary items and before net income.

(9)

The amount of the loss would be disclosed as part of other revenues and expenses.


P13–2 a. Bonus = 25%  (Income from Operations Before Interest Expense – Interest Expense) = 25%  [$1,200,000 – ($1,000,000  8% Interest Rate)] = $280,000 b. Bonus = 25%  (Income from Operations Before Interest Expense – Interest Expense) = 25%  ($1,200,000 – $0) = $300,000 c.

The decision whether to finance the plant expansion through an equity issue or through a debt issue is worth $20,000 to the managers because the managers will receive an additional $20,000 in bonuses if the company issues equity instead of debt. Although issuing equity is in the best interest of the managers, this option may not be in the best interest of the existing stockholders. It is possible that issuing additional stock would dilute the ownership interests of the existing stockholders. Further, the interest payments on debt are tax deductible. Hence, issuing debt would reduce cash outflows for taxes.

d. If interest expense was not considered to be an operating expense, the managers' bonus would be the same whether the company issued debt or equity. In either case the bonus would be 25% of $1,200,000, or $300,000. In this situation, the managers would, hopefully, base their decision on factors that are more relevant to the stockholders.

P13–3 a. 1. 2. 3. 4. 5. 6. 7.

Financing Operating Financing Operating Investing No cash involved Investing & Financing

b. Raleigh Corporation Income Statement For the Year Ended December 31, 2015 Fees earned ................................................................................ Expenses: Wage expense ....................................................................... $ 125,000 Supplies expense ................................................................... 35,000 Depreciation expense ............................................................ 80,000 Miscellaneous expense.......................................................... 75,000 Total expenses.................................................................. Net income .................................................................................

$ 580,000

315,000 $265,000


P13–3 c.

Concluded

Comprehensive Income

= Change in Equity from Non-owner Sources = Revenues – Expenses + Gains – Losses + Cumulative Accounting Adjustments = $580,000 – $315,000 + 0 – 0 = $265,000

* Fees and expenses represent Raleigh’s only non-owner changes in equity. The two measures are equal because Raleigh’s economic events/activities do not differ from those described in the income statement.

P13–4 a. Sales Sales returns Cost of goods sold Dividends Rent expense Wages payable Loss on sale of food services division Loss incurred by food services division Depreciation expense Cumulative effect on income of change in depreciation methods Gain on land appropriated by government Insurance expense Inventory Administrative expenses Prepaid insurance Gain on sale of short-term investments

Income Statement Yes Yes Yes No Yes No Yes segment Yes segment Yes

Classification . Usual and frequent Usual and frequent Usual and frequent N/A Usual and frequent N/A Disposal of business

Yes method

Change in accounting

Yes Yes No Yes No Yes

Unusual and infrequent Usual and frequent N/A Usual and frequent N/A Unusual or infrequent

Disposal of business Usual and frequent


P13–4

Concluded

Note:

The loss on sale of food services division and the loss incurred by food services division would be classified as a disposal of a business segment only if the food services division meets the requirements of being a business segment as defined under GAAP. If the food services division does not meet these requirements, then the loss on sale of food services division would be classified as part of other revenues and expenses. The loss incurred by food services division would be broken down into its components (i.e., revenues and expenses) and reported as part of operating revenues and expenses.

b. Crozier Industries Income Statement For the Year Ended December 31, 2015 Revenue: Sales revenue......................................................................... $ 977,000 Less: Sales returns ................................................................. (9,000) Gain on sale of short-term investments ................................ 142,000 Total revenue.............................................................................. Expenses: Cost of goods sold ................................................................. $ 496,000 Rent expense ......................................................................... 90,000 Depreciation expense ............................................................ 100,000 Insurance expense ................................................................. 12,000 Administrative expenses ....................................................... 109,000 Total expenses.................................................................. Income from continuing operations (before taxes) ................... Income taxes .............................................................................. Income from continuing operations........................................... Discontinued operations: Loss on operation of discontinued segment (net of tax benefit of $3,500)............................................... $ (6,500) Loss on disposal of discontinued segment (net of tax benefit of $700).................................................. (1,300) Discontinued operations .................................................. Income before extraordinary items ........................................... Extraordinary gain on appropriated land (net of taxes of $32,200) ................................................................... Income before effect of accounting changes ............................. Cumulative effect of change in accounting principle (net of tax benefit of $45,500) .............................................. Net income .................................................................................

$ 1,110,000

$

807,000 303,000 106,050 196,950

$

(7,800) 189,150

$

59,800 248,950

$

(84,500) 164,450

$

Sales revenues and all the expenses (cost of goods sold, rent, depreciation, insurance, and administrative) are expected to persist into the future. The gain on the sale of short-term investments is expected to occur infrequently, as a secondary activity to the primary business operations. The losses due to the discontinued operations loss and its sale are one-time amounts, not expected to persist into the future. The extraordinary gain on appropriated land is both unusual and infrequent and is not expected to occur again. Finally, the cumulative effect of a change in accounting principle may occur, but only infrequently. Future income statements should not be affected by many accounting changes.


P13–5 a. 1. Hurricanes are unusual in that they are not part of a company's normal operations; however, they occur relatively frequently in Florida. Thus, this loss probably should not be classified as an extraordinary item. Instead, it should be disclosed gross of taxes as part of other revenues and expenses. 2. A loss on the disposal of a business segment is not considered an extraordinary item. This item should be disclosed net of taxes as a separate item on the income statement after income from continuing operations, but before extraordinary items. The classification of this item is the disposal of a business segment. 3. This loss appears to be both unusual and infrequent; consequently, it should be classified as an extraordinary item on the income statement. Extraordinary items should be disclosed net of any tax effect. 4. Writing off an open account receivable as uncollectible should not be disclosed on the income statement. Under GAAP, a company should use the allowance method to account for bad debts. With the allowance method, write-offs of uncollectible accounts affect only balance sheet accounts. 5. Floods are unusual in that they are not part of a company's normal operations. Although Arizona does get flooding, particularly flash floods, the flooding would probably be considered to be infrequent. Consequently, this loss should be classified as an extraordinary item on the income statement. Extraordinary items should be disclosed net of any tax effect. b. Extraordinary items: Loss to employee destruction (net of tax benefit of $78,750) ........................... 146,250 Loss due to flood (net of tax benefit of $31,500) ............................................... 58,500 Total Loss on Extraordinary Items $204,750

P13–6 a. Income from continuing operations (before taxes) Income taxes Income from continuing operations Extraordinary loss (net of tax benefit of $47,250) Net income b. Income tax expense

$ $ $

850,000 297,500 552,500 (87,750) 464,750

= Income taxes on net income generated during 2015 = Income taxes on income from continuing operations + Income tax effect of extraordinary loss = $297,500 tax expense [from part (a)] – $47,250 tax benefit [from part (a)] = $250,250


P13–6

Concluded

c. MTM Company Statement of Retained Earnings For the Year Ended December 31, 2015 Retained earnings, January 1, 2015.................................................................... Plus: Net income................................................................................................. Less: Dividends declared during 2015 ................................................................ Retained earnings, December 31, 2015 .............................................................

$ 1,259,000 464,750 (175,000) $ 1,548,750

d. The balance in the Income Tax Liability account would equal the beginning balance plus the entire amount owed to the government for income taxes as of December 31, 2015 less tax payments made during 2015. MTM has two sources of income taxes during 2015. The first source is the income tax on income from continuing operations in the amount of $297,500 [from part (a)]. The second source is the income tax benefit on the extraordinary loss in the amount of $47,250 [from part (a)]. Therefore, MTM owes the beginning balance of $70,000 plus the total of the two sources of 2015 taxes of $250,250 less the 2015 tax payments of $200,000. Therefore, the tax liability balances of December 31, 2015 equals $120,250.

P13–7 a. This allows the user of the financial statements to try and make estimates of the future potential results of the company. Discontinued operations will not impact future revenues and expenses so this gives more information to the user. Continuing operations gives users of the financial statements the best information for estimating the company’s future performance. b. An estimate of the number of shares outstanding is calculated by taking net earnings ($3,544.2) and dividing it by the basic net earnings per share ($2.87) or 1,235 million shares. c.

The distinction between basic net earnings per share and diluted net earnings per share is that basic is calculated using the weighted average number of shares outstanding and diluted is calculated using the outstanding weighted average number of shares plus the number of shares that would be outstanding if all convertible instruments were converted to common shares. The most common financial instruments that are convertible to common shares are stock options, preferred stock and some bonds.

d. An estimate of the number of shares that would be outstanding if all other instruments were converted to common shares is calculated by taking net earnings ($3,544.2) divided by diluted net earnings per share ($2.83) or 1,252 million shares.

P13–8 a.

The readers of the financial statements are interested in what operations and profitability are likely to continue into the following year. Line items such as discontinued operations and extraordinary events will not occur in the following year and thus should be separated from continuing operations that will be likely to recur in the upcoming year. Further, owners of Duke Energy would like a distinction between income from operations that the company controls and income from companies over which Duke has no control.


b.

Earnings were $1,768 million and earnings per share were $3.07, therefore the number of shares would be $1,768/$3.07 = 575.896 million.

P13–8 Concluded c.

Earnings per share represents the net income that can be attached to each share that is currently outstanding. If, however, additional shares become outstanding (due to, for example, the conversion of preferred shares into common shares), then the net income needs to be spread among a larger number of shares. Diluted EPS shows the net income spread among the total number of shares that could be outstanding.

d.

Since the EPS basic and diluted is the same, there is no opportunity for securities to be converted into common shares.

P13–9 a. 1. Extraordinary losses are disclosed on the income statement net of the tax benefit after discontinued operations but before the effect of changes in accounting methods. 2. Extraordinary gains are disclosed on the income statement net of taxes after discontinued operations but before the effect of changes in accounting methods. 3. The sale of inventory would be disclosed on the income statement (gross of taxes) as part of operating revenues. The associated cost of inventory sold would be disclosed on the income statement (gross of taxes) as part of operating expenses. 4. The loss on disposal of a business segment would be disclosed on the income statement net of taxes as part of discontinued operations, which is disclosed after income from continuing operations but before extraordinary items. 5. The income effect due to change in accounting method would be disclosed net of the tax effect on the income statement as the last item before net income. 6. Advertising expense would be disclosed on the income statement (gross of taxes) as part of operations expenses. 7. Income earned by the disposal of a business segment would be disclosed on the income statement net of taxes as part of discontinued operations, which is disclosed after income from continuing operations but before extraordinary items. b. Extraordinary items: Extraordinary loss (net of tax benefit of $87,500) ............................... Extraordinary gain (net of taxes of $19,250)........................................ Discontinued operations: Loss on disposal of business segment (net of tax benefit of $35,000) ........................................................... Income earned by disposed business segment

(162,500) 35,750

(65,000)


(net of taxes of $52,500) .................................................................... Effect of change in accounting method (net of taxes of $28,000) ....................................................................

97,500 52,000

P13–9 Concluded c.

Income from continuing operations ................................... Disposal of business segment ............................................. Extraordinary items ............................................................. Effect of changes in accounting method ............................. Net earnings per share ........................................................ a $0.16

= = b $(0.63) = =

$ 3.00 0.16a (0.63)b 0.26 $ 2.79

(Income from disposed segment – Loss on disposal) ÷ 200,000 shares ($97,500 – $65,000) ÷ 200,000 shares (Extraordinary gain – Extraordinary loss) ÷ 200,000 shares ($35,750 – $162,500) ÷ 200,000 shares

P13–10 Microbiology Labs Income Statement For the Year Ended December 31, 2015 Sales revenue......................................................................... Cost of goods sold ................................................................. Gross profit ............................................................................ Operating expenses ............................................................... Income from operations ........................................................ Loss on sale of office equipment ........................................... Income from continuing operations (before taxes) .............. Income taxes ......................................................................... Income from continuing operations...................................... Disposal of business segment: Gain on disposal of assets (net of taxes of $87,500) ......$ Income from operations of disposed segment (net of taxes of $52,500) ........................................................ Disposal of business segment ..................................... Income before extraordinary items ...................................... Extraordinary items: Loss on retirement of bonds (net of tax benefit of $28,000) ...................................................................... Loss due to insect infestation (net of tax benefit of $280,000) .................................................................... Extraordinary losses .................................................... Net income ............................................................................

$ 10,000,000 2,500,000 $ 7,500,000 750,000 $ 6,750,000 (60,000) $ 6,690,000 2,341,500 $ 4,348,500 162,500 97,500* 260,000 $ 4,608,500

$

(52,000) (520,000) (572,000) $ 4,036,500


Earnings per share: Income from continuing operations.................................................................... Disposal of business segment.............................................................................. Extraordinary items ............................................................................................. Net earnings per share ........................................................................................ * $97,500

$ 2.17 0.13 (0.29) $ 2.01

= [Discontinued segment's sales – Discontinued segment's cost of goods sold – Discontinued segment's operating expenses  (1 – Tax rate of

35%)

The objectives of financial accounting are to allow prediction of future cash flows. Earnings numbers are useful because they reflect changes in a company’s resources. Some measures such as income from operations reflect amounts expected to persist in the future from the normal business operations. Other measures, such as income from continuing operations, reflect an infrequent or unusual item with less persistence. Finally, extraordinary items, disposal gains and losses, and cumulative effects of accounting method changes have low persistence and are less useful for prediction of future cash flows.

P13–11 a. The sale of the credit card division (disposal of a business segment) would be shown on the income statement after net income from continuing operations; the segment would be separated into two line items, the income from the discontinued operations up to the date of disposal and the gain from the disposal (both shown net of tax). The restructuring charges would be shown in “Other revenues and expenses”, on the income statement after the calculation of net operating income. b. Sears’ performance over the two-year period is better measured when the one-time, non-recurring items are removed from the analysis. The profitability of $309 million is not as strong as it appears, because it includes a gain from the sale of a division (that will not recur in the following year); likewise, the following year’s profits of $53 million should actually be viewed more positively because the restructuring expense (which is presumably a non-recurring item) dragged down that year’s bottom line.

P13–12 a. Recognized income and expense under IFRS is similar to comprehensive income under U.S. GAAP. b. The SORIE would be presented as: Net profit Fair value gains(losses) on available-for-sale securities and currency translation gains(losses) Total recognized income and expense c.

1,800 (531) 1,269

The statement of shareholders’ equity will show the effects from currency and securities transactions that affect equity but do not affect profitability. (The balance sheet will also show the end result in the shareholder equity section.)


P13–13 a. Adjusting journal entries (1)

(2)

Inventory (ending) (+A) ................................................................. Cost of Goods Sold (E, –SE) ............................................................ Purchases (–A) ........................................................................ Inventory (beginning) (–A) ...................................................... Recorded cost of inventory sold.

480,000 737,000

Bad Debt Expense (E, –SE) ............................................................. Allowance for Doubtful Accounts (–A) ................................... Estimated bad debt expense.

25,000*

* (3)

(4)

(5)

25,000

$25,000 = $75,000 – $50,000 Balance in Allowance for Doubtful Accounts

Depreciation Expense (E, –SE) ....................................................... Accumulated Depreciation (–A).............................................. Depreciated fixed assets.

85,000

Insurance Expense (E, –SE) ............................................................ Prepaid Insurance (–A) ........................................................... Recognized expiration of insurance policy.

20,000

Interest Expense (E, –SE) ............................................................... Interest Payable (+L) ............................................................... Incurred, but did not pay, interest.

25,000*

* (6)

750,000 467,000

20,000

25,000

$500,000  10% = $50,000 less 25,000 Already Recognized

Income Tax Expense (E, –SE) ......................................................... Income Tax Liability (+L) ......................................................... Accrued income taxes on continuing operations. *

85,000

$21,000 = =

21,000* 21,000

Income from continuing operations  Tax rate of 35% {($1,256,000 + $76,000) – [$737,000 + $100,000 + $255,000 + $25,000 + $25,000 + $25,000 + $85,000 + $20,000]}  35%

Income Tax Liability (–L) ............................................................................ Extraordinary Loss (–Lo, +SE) ............................................................. Recognized income tax benefit from extraordinary loss.

12,250* 12,250

* $12,250 = Extraordinary loss of $35,000  Tax rate of 35% Income Tax Liability (–L) ............................................................................ Cumulative Loss from Accounting Change (–Lo, +SE) ........................ Recognized income tax benefit from accounting change.

21,000 21,000


P13–13 Continued Closing entries (c1) Income Summary ........................................................................... Sales ............................................................................................... Gain on Sale of Land ..................................................................... Cost of Goods Sold ................................................................. Administrative Expenses......................................................... Selling Expenses ...................................................................... Interest Expense .................................................................... Bad Debt Expense .................................................................. Depreciation Expense ............................................................. Insurance Expense .................................................................. Income Tax Expense ............................................................... Extraordinary Loss .................................................................. Cumulative Loss from Accounting Change ............................. Closed revenues and expenses into Income Summary. (c2)

(c3)

22,750 1,256,000 76,000 737,000 100,000 255,000 50,000 25,000 85,000 20,000 21,000 22,750 39,000

Retained Earnings ......................................................................... Income Summary .................................................................... Closed Income Summary into Retained Earnings.

22,750

Retained Earnings .......................................................................... Dividends ................................................................................ Closed Dividends into Retained Earnings.

135,000

22,750

135,000


P13–13 Concluded b. Laidig Industries Income Statement For the Year Ended December 31, 2015 Revenues: Sales ....................................................................................... $ 1,256,000 Gain on sale of fixed assets ................................................... 76,000 Total revenues .................................................................. Expenses: Cost of goods sold ................................................................. $ 737,000 Administrative expenses ....................................................... 100,000 Selling expenses..................................................................... 255,000 Depreciation expense ............................................................ 85,000 Bad debt expense .................................................................. 25,000 Insurance expense ................................................................. 20,000 Interest expense .................................................................... 50,000 Total expenses.................................................................. Income from continuing operations (before taxes) ................... Income taxes .............................................................................. Income from continuing operations........................................... Extraordinary loss (net of tax benefit of $12,250) ..................... Income effect due to change in accounting principle (net of tax benefit of $21,000) .............................................. Net loss .......................................................................................

$ 1,332,000

1,272,000 $ 60,000 21,000 $ 39,000 (22,750)

$

(39,000) (22,750)

Earnings per share: Income from continuing operations ........................................... Extraordinary loss ....................................................................... Income effect due to change in accounting principle ................ Net earnings per share ...............................................................

$ 0.20 (0.11) (0.20) $(0.11)

c. Laidig Industries Statement of Retained Earnings For the Year Ended December 31, 2015 Retained earnings, January 1, 2015.................................................................. Net loss ........................................................................................................... Less: Dividends declared .................................................................................. Retained earnings, December 31, 2015 ...........................................................

$ 673,000 (22,750) (135,000) $ 515,250

d. The objectives of financial accounting are to allow prediction of future cash flows. Earnings numbers reflect changes in a company’s resources. Some measures such as income from continuing operations are highly persistent, reflecting amounts expected to persist in the future due to normal business operations. Net income includes an extraordinary loss and income from a change in accounting principle, items that have low persistence and are less useful for the prediction of future cash flows.


ISSUES FOR DISCUSSION ID13–1 a. The net income and comprehensive income for CVS are very similar due to the relatively low dollar amount of changes to equity that did not also affect profitability. Caterpillar, on the other hand, saw a more significant change (a drop of $229 million) from net income to comprehensive income due to changes from foreign exchange, pension obligations and securities holdings. b. Caterpillar operates globally, so it must deal with foreign currency exposure; as shown, if exchange rates move the company will see a change to its equity. CVS is mainly focused on its domestic retail operations, so it does not have this same exposure. Caterpillar has negotiated labor contracts with its production employees that specify some defined benefit pension obligations for the company, which bring into effect market and interest rate risks that can create equity losses for the company. CVS employs a different type of workforce and does not offer its employees the same type of retirement benefits and therefore does not face the same level of risks to its equity base. c. An analyst would focus on the companies’ profit levels, but would also focus on what other factors could harm the company’s equity cushion. A certain level of expertise and understanding in foreign exchange markets and trends would be required for an analyst following Caterpillar; an analyst focused on CVS would not be as concerned with the FX markets. Similarly, an understanding of pension accounting and assumptions appears more important for an analyst following Caterpillar than it would for someone following CVS.

ID13–2 a. Tightened credit markets will mean less volume in debt issuances. Since each issue is rated by an agency, the revenue of the ratings agencies will decline as the number of debt issuances declines. b. The income statement will be directly affected with a drop in revenue and a consequent drop in profits. Stockholders’ equity will be affected due to the reduced net income shown on the income statement. To balance the lowered equity, the ratings agencies will either reduce asset balances or increase liabilities. c.

It is very possible that the companies will see less volume in the future, even if the credit markets return to their previous levels. It is possible that we will see a restructuring in the markets, with less emphasis placed on the ratings assigned by outside agencies. Given the fact that the agencies assigned inaccurate grades on the mortgage-backed securities, and the fact that these inflated grades caused investors to lose principal, it is very possible that investors will look to a different mechanism to help understand risk. This potential would permanently depress revenue and profit levels for the ratings agencies.

ID13–3


a. For an event to be classified as extraordinary, the event must have been both unusual in nature and infrequent in occurrence. b. If the eruptions continue periodically, then such eruptions would probably not be viewed as being infrequent in occurrence. Thus, any losses resulting from future eruptions probably would not be considered to be extraordinary. c.

The entire loss would have been $69.231 million. That is, the after-tax loss of $36 million divided by (1 – tax rate of 48%). The journal entries would be: Extraordinary Loss from Volcano Eruption (Lo, –SE) ............................ Timberland (–A) ............................................................................. Recognized extraordinary loss from volcano eruption.

69,231,000

Income Tax Liability (–L) ....................................................................... Extraordinary Loss from Volcano Eruption (–Lo, +SE) ................... Recognized tax benefit from volcano eruption.

33,231,000

69,231,000

33,231,000

ID13–4 a. Standard & Poor's may have viewed the charge as different from previous years and felt including it would distort comparisons from year to year. Value Line may have felt that the exclusion of the charge would be a distortion of the income for the year. b. The income statement is comprised of several categories. Revenues and expenses that are both usual and frequent are classified as operations. Revenues and expenses that are either usual or frequent, but not both, are classified as other revenues and expenses and considered part of continuing operations. Following continuing operations are special items. Such items, in order of their disclosure, are discontinued operations, extraordinary items (which are both unusual and infrequent), and the cumulative effect of accounting changes. All items listed after continuing operations are disclosed net of the associated tax effect. c.

All items listed on a company's income statement are important in that these items affect a company's actual financial position and/or the amounts reported on the balance sheet for assets and liabilities. Thus, financial analysts do not care how or where the information is recorded as long as the information is available. Alternatively, the location of items on the income statement can have important economic consequences. For example, if a company's management receives incentive compensation based on income, it is important whether the incentive compensation contract defines income as income from operations, income from continuing operations, or net income. Whether something is classified as an operating expense, an other expense, or an extraordinary loss, therefore, would affect management's incentive compensation, depending on how income is defined. In addition, stockholders are interested in a company's earning power, i.e., the company's ability to generate net assets on an ongoing basis. If the company were to include items as part of income from continuing operations that were not expected to continue on into the future, stockholders would be receiving distorted information that could cause them to misvalue the company's stock. Thus, the classification of items on the income statement could affect the magnitude of management's incentive compensation as well as the valuation of the entire company.


ID13–5 a. Analysts are many times looking for companies that will be the dominant company in different business categories. The internet has provided an alternative method for selling goods and services to the public. Since this is a new industry many analysts were not sure how much market share this channel would take away from the traditional retailing industry. As a result analysts were trying to pick the future winners in this new industry channel. Since few of these companies are profitable analysts had to develop alternative measures to try and evaluate the relative and absolute performance of these companies. b. The specific effect that this FASB rule had on the income statements on companies like Amazon.com was to increase the cost of good sold and decrease the gross profit percentage. These companies lobbied against this change because these companies were promoting the idea that their gross profit percentages were much higher than traditional retailers. After this change it became very apparent that their cost structure was not a competitive advantage against traditional retailers. At that point their stock prices started to decline. c.

There is no impact on the reported cash flows of these companies as a result of this accounting change. This change moves where these costs are reported on the income statement but has no impact on the physical operations of the company.

d. Theoretically, the price of a company's stock equals the present value of the cash flows the stock market expects a stockholder in that company to realize from holding that stock. Thus, the stock price of these companies should not be impacted by this accounting change. While the cash flow of a company will not change as a result of this accounting change it does impact the expectations of investors as to the future cash flows of these companies. This change highlighted information in a way that reduced investors’ future estimates of the profitability or cash flow of these companies.


ID13–6 a. Although it could be argued that lawsuits are a normal part of conducting business in the United States and that lawsuits may not be that infrequent, the settlement was probably disclosed as an extraordinary loss. The determining factor in each case would be if the event was infrequent and unusual, if the event is both of these then it should be reported as an extraordinary. Extraordinary items should be disclosed net of the associated tax effect. With a tax rate of 34% and a loss of $909.5 million, Kodak would receive a tax benefit of $309.23 million. Thus, Kodak should have reported a net loss of $600.27 million on its 1990 income statement for the settlement. b. The patent infringement case is an example of a contingency. SFAS No. 5, "Accounting for Contingencies" states that a contingency is "an existing condition, situation, or set of circumstances involving uncertainty as to possible gain or loss to an enterprise that will ultimately be resolved when one or more future events occur or fail to occur." In this particular case, Polaroid had a gain contingency and Kodak had a loss contingency. Under the guidelines set forth in SFAS No. 5, the most that Polaroid could have done in its 1989 annual report was to disclose the lawsuit in a footnote. Alternatively, Kodak could have either disclosed nothing about the lawsuit, disclosed the lawsuit in a footnote, or recorded a loss and associated liability. The appropriate course of action depended upon (1) whether it was remote, reasonably possible, or probable, given the information available in 1989, that Kodak would eventually lose the lawsuit and (2) whether the amount of the loss could be reasonably estimated in 1989. If the probability that Kodak would eventually lose the lawsuit was remote, then Kodak could ignore the lawsuit for financial reporting purposes. If the probability of the loss was reasonably possibly or if the amount of the loss could not be reasonably estimated, Kodak should have disclosed the lawsuit in a footnote only. Finally, if both the probability of the loss was probable and the amount of the loss could be reasonably estimated, Kodak should have recorded a loss and associated liability. Due to the magnitude of the case and the associated publicity, it is doubtful that Kodak ignored the lawsuit for financial reporting purposes. In addition, if Kodak knew that it was probable that it would lose the lawsuit, it probably would have settled the case out of court. Thus, Kodak most likely did not think it was probable that it would lose the lawsuit, and therefore only disclosed the lawsuit in a footnote. c.

There are at least two reasons why Kodak's stock increased in value. First, the settlement provided unexpected "good" news about Kodak. The stock market may have expected Kodak to lose the lawsuit and have to pay out close to the amount being asked for by Polaroid. The fact that Kodak had to pay considerably less than expected caused the stock market to positively reevaluate the prospects of investing in Kodak. Second, Kodak reported a large increase in operating earnings. Stockholders are interested in a company's earning power. That is, stockholders are interested in a company's ability to generate net assets from operations on an ongoing basis. The large increase in Kodak's operating earnings provided information to the stock market that Kodak has strong earning power. Alternatively, the lawsuit settlement is a one-time payout that should not adversely affect Kodak's ongoing operations, although it may have some short-term effects on Kodak's earning power.


ID13–7 a. Impairment charge – is shown as an other expense item on the income statement. Research and development costs – are shown as an expense below cost of goods sold and before income from continuing operations. Foreign currency translation – recognized gains and losses are shown gross on the income statement as an other gain or loss. Any unrecognized gains or losses are included in comprehensive income. Net gain on disposal of business – is shown net of tax below income from continuing operations on the income statement. Provision for restructuring – is shown gross on the income statement in the other gain or loss section. Unrealized gain on available for sale securities – is shown not on the income statement but as an item under comprehensive income. Cost of products sold – is shown gross in the cost of goods section of the income statement. Litigation recoveries – are shown gross in the other gains and losses on the income statement. Equity in net income of affiliates— is shown gross on the income statement and before income from continuing operations. b. The items that would be expected to be persistent or be repeated in future years are research and development costs, foreign currency translation, cost of products sold and equity income from affiliates. Each of these items is an expense (or gain) that Bristol-Myers Squibb would expect to incur in future years in the normal operation of the business. The other items are not expected to occur again in future years.

ID13–8 a. Federal Express should report this as an extraordinary gain on its income statement. This assumes that aircraft being destroyed by fire is both unusual and infrequent. This is a one-time impact on the wealth of Federal Express and would not be expected to persist in the future. b.

Motorola would report the gains related to these asset sales below the gross profit line in a section called “Other Gains and Losses”. The sale of assets is not Motorola’s primary business and so these gains should not be shown in the revenue section at the top of the income statement. At the same time these sales are not extraordinary items since they are not unusual and infrequent. These sales represent one-time increases in wealth to the company and would not be expected to persist into the future. c.

Owens-Corning would report the $68 million restructuring charge below the income from operations line, the $11 million of equity in net income of affiliates in the revenue section, and the $15 million accounting change below the income from operations line. Both the restructuring charge and the accounting change would not be expected to persist in the future but the equity in net income of affiliates would be expected to persist in the future.

d. Owens-Corning would report the $875 million charge below the income from operations line on the income statement. The class action lawsuits related to asbestos are (as a group) unusual and infrequent and should not be reported in income from operations. This is a one-time reduction in the wealth of OCF and would not be expected to persist into the future.


ID13–9 a. Rising raw material costs would be reflected in Cost of Goods Sold on the income statement, as well as in Inventory (Raw Materials) on the balance sheet. Currency fluctuations would be reflected in comprehensive income under currencey translation adjustments. A gain would be reflected on the income statement under “other gains/expenses”. Increased costs to introduce new models would be reflected on the income statement under operating expenses (engineering, marketing, etc.). b. Rising material costs and costs to introduce new models might be expenses that will permanently affect BMW’s business model. c.

Comprehensive income measures all changes in a company’s equity due to nowowner transactions. Foreign currency adjustments (as seen by global operations such as BMW) and certain holding gains/losses are not thought to affect a company’s profits but do indeed affect the amount of equity shown on the financial statements. Comprehensive income captures all nonowner changes to equity.

d. Analysts will review a number of factors in addition to a company’s earnings, including: macroeconomic events, overall stock market pricing, a company’s guidance about future earnings, management changes, and a company’s market share.

ID13–10 a. Carrefour defines operating income to come before (to not include) income from affiliated companies, while VW and Sony include the equity income from affiliates in its operating income. VW includes interest (finance) costs in its calculation of operating income, while Carrefour and Sony both deduct interest expense after operating income is calculated. b. It is possible that the financing function for Volkswagen is more aligned with that company’s daily operations of its business. It is possible that VW and Sony consider the work of its affiliated companies to be so closely aligned with its own core operations that it includes the equity income from those affiliates in the calculation of operating profits. c. An analyst following all three of these companies would strive for consistency in the analysis of the companies’ profits. An analyst would reconstruct the three income statements to treat all the line items in the same manner; for example, the analyst may follow Carrefour’s lead and take the affiliate income and interest expense for Carrefour and Sony and move them lower on the income statement so that they are not included in the operating profit line.


ID13–11 a. The significant item shown below operation income is interest and other income, but these amounts total less than 1.5% of total revenues for the company. Only in 2012 did Google show operations that had been discontinued and, again, the dollar amount is relatively small. The vast majority of operations are still continuing. b. Google’s EPS increased from $26.69 in 2010 to $32.81 in 2012. If all potential shares are converted to outstanding, EPS would drop from 2012’s $32.81 to $32.31. c.

Other comprehensive income items are shown in the Statement of Comprehensive Income. For the year ending 12/31/2012, comprehensive income was $10,999 million, $262 million higher than net income. The difference was due mainly to unrealized gains on available for sale securities; other adjustments were related to cash flow hedges.


CHAPTER 14 THE STATEMENT OF CASH FLOWS BRIEF EXERCISES BE14–1 a. Depreciation expense is shown as an adjustment to net income to calculate cash flow. Depreciation expense is added back to net income because it is a non-cash expense. This means that it is deducted in the calculation of net income but there is no cash expenditure related to depreciation. b. Net income plus depreciation does not equal net cash provided by operating activities because all of the changes in current assets and current liabilities are also shown in the cash provided by operating activities section. Increases in current asset accounts represent a use of cash and increases in current liabilities present a source of cash. c.

The estimated net change in current assets and current liabilities is $0.2 billion. This is calculated by taking $10.6 billion net cash provided by operating activities minus (net income of -$12.7 billion plus $5.1 billion of depreciation and amortization plus the $18.0 non-cash impairment expense).

BE14–2 During 2012 Pier One collected $1,694 million from its customers. This can be calculated as follows: Accounts receivable, beginning balance Add: 2012 Sales Less: Accounts receivable, ending balance Cash collected during 2012

$ 16.3 million 1.7 billion 22.3 million $1,694 million

BE14–3 a. Cost of inventories purchased during 2012 equals: Inventory, ending balance Add: 2012 Cost of sales Less: Inventory, beginning balance Inventory purchased in 2012

$356 million 962 million 322 million $996 million

b. Cash payments made to suppliers during 2012 equals: Accounts payable, beginning balance Add: 2012 inventory purchases Less: Accounts payable, ending balance Cash payments to suppliers made in 2012

BE14–4 1

$

$

64 million 996million – 59 million 1,001 million


a. Cash from operations Cash from investing Cash from financing Change in cash

Agilent $1,228 (2,372) (31) $(1,175)

AMD $ (338) (19) 37 $ (320)

b. AMD is generating cash from financing activities—that is, the company is raising cash from equity and/or debt issuances. Agilent, on the other hand, is using cash to return to shareholders and/or repay debt. Both companies are investing cash in long-term assets. Finally, Agilent is generating cash from operations, while AMD’s core business is losing cash balances. Overall, AMD is only generating cash through financing, while Agilent is increasing cash balances from its daily operations. c.

Cash from operations exceeds the net income(loss) figure due to depreciation, other non-cash expenses and the change to operating accounts (such as receivables, payables). The difference in Cash from Financing Activities tells you that AMD is having to supplement its cash balance deficits (from operations) through borrowings and/or equity issuances while Agilent is able to use the positive cash generated by its operations to pay dividends and/or repurchase shares and/or reduce debt.

BE14–5 a. In 2011 profit was below operating cash flows because the change in operating accounts (such as receivables and payables) acted as a source of cash balances. Receivables decreased and/or payables increased, increasing cash balances as the cash was freed up from the operating account line(s). In 2012, when profits exceeded operating cash flows, the change in account balances reversed from what happened in the prior year. The operating accounts in 2012 were a use of cash, with receivables increasing and/or payables decreasing (which tied up cash in the operating account line). b. In 2011 the company used its cash flow from operations and combined it with the proceeds from long term asset assets to fund a rather large return of cash to debt and equity providers. In 2012, the company sold off more long term assets and used that cash to cover shortfalls from operations and to pay dividends, repurchase shares, and/or reduce debt.

EXERCISES


E14–1 1. 2. 3. 4. 5. 6. 7. 8. 9. 10.

Investing activity Financing activity Operating activity Financing activity Investing activity Investing activity (for the cash paid for the building) Financing activity (for the mortgage payable) Financing activity (for the principal payment) Operating activity (for the interest payment) Operating activity Financing activity Operating activity

E14–2 1.

Not included on the statement of cash flows because it does not affect cash. As proof, the entry for this event would be: Allowance for Doubtful Accounts (+A) .................................. Accounts Receivable (–A) ............................................... Wrote off uncollectible accounts.

XX XX

2.

Investing activity

3.

Financing activity

4.

Any gain or loss would be adjusted out of operating activities. The exchange of notes for the building would have no net impact on the statement of cash flows because it does not affect cash. As proof, the entry for this event would be: Notes Receivable (+A) ........................................................... Accumulated Depreciation: Building (+A) ............................. Building (–A) ................................................................... Sold a building in exchange for a five year note.

5.

XX

Not included on the statement of cash flows because it does not affect cash. As proof, the entry for this event would be: Dividend (–SE)........................................................................ Dividend Payable (+L) ..................................................... Declared a dividend.

6.

XX XX

XX XX

Not included on the statement of cash flows because it does not affect cash. As proof, the entry for this event would be: Bonds Payable (–L) ................................................................ Common Stock (+SE)....................................................... Retired bonds with common stock.

XX XX


E14–2

Concluded

7.

Investing activity

8.

Not included on the statement of cash flows because it does not affect cash. As proof, the entry for this event would be: Dividend (–SE)........................................................................ Common Stock (+SE)....................................................... Additional Paid-in Capital, Common Stock (+SE) ............ Declared and issued a stock dividend.

9.

XX XX XX

Not included on the statement of cash flows because it does not affect cash. As proof, the entry for this event would be: Depreciation Expense (E, –SE) ............................................... Accumulated Depreciation (–A)...................................... Depreciated fixed assets.

XX XX

10.Operating activity 11.

Not included on the statement of cash flows because it does not affect cash. As proof, the entry for this event would be: Inventory (+A) ........................................................................ Accounts Payable (+L) ..................................................... Purchased inventory on account.

XX XX

12.

Operating activity

13.

Any gain or loss would be included in operating activities only to adjust Net Income. The balance of the transaction would not be included on the statement of cash flows because it does not affect cash. As proof, the entry for this event would be: Land (+A)................................................................................ Accumulated Depreciation: Building (+A) ............................. Building (–A) ................................................................... Exchanged a building for land.

14.

Financing activity

15.

Operating activity

XX XX XX


E14–3 Company AAA BBB CCC DDD EEE

Cash Provided (Used) by Operations Investments 320 (178) 219 (450) 197 (414) 120 (130) 290 (120)

Financing $(180) 190 80 430 (100)

Net Increase (Decrease) $ (38) (41) (137) 420 70

AAA This company appears to be following a policy of maintaining a relatively constant cash balance. The company also appears to be relying primarily on operating activities to provide cash to finance the acquisition of nonoperating assets and to finance the repayment of debt and/or acquisition of treasury stock. BBB Similar to AAA, this company appears to be following a policy of maintaining a relatively constant cash balance. This company also appears to be using cash from operating activities and from borrowings to purchase nonoperating assets. CCC CCC appears to be using large amounts of cash generated in both the current and prior periods to acquire nonoperating assets. This company also appears to disdain borrowing money. DDD DDD appears to be following a policy of acquiring large amounts of cash in the current period. The company acquired this cash primarily through borrowings. EEE EEE appears to be acquiring moderate amounts of cash in the current period through operating activities. Similar to Company AAA, Company EEE used some of this cash to acquire nonoperating assets and to repay existing debts and/or to acquire treasury stock.

E14–4 Kraft Foods – Cash from Investments ($422): the company was able to generate cash from operations and used much of this increase in cash to repay debt, pay dividends, and/or repurchase equity. The company also used cash to purchase long-term assets and build up its cash position. Kellogg’s – Net Change in Cash ($170): the company was able to generate cash from operations and from debt and equity issuances. The company used that cash to reinvest heavily in long-term assets. General Mills– Cash from Operations $2,384: the company was able to generate a significant amount of cash from operations and used that cash to reinvest in long-term assets and to repay debt, pay dividends, and/or repurchase equity.


E14–5 Assets 1. Depreciation expense

–170,000

2. Issue of common stock

+180,000

= = =

3. Purchase IBM stock

375,000 (securities) –375,000 (cash)

4. Purchase insurance

+27,000 (ppd insurance) –27,000 (cash)

5. Purchase of building

Liabilities

+ Owners’ Equity –170,000 +180,000

+200,000 (building) – 40,000 (cash) = +160,000 (mortgage)

1. Under the direct method, depreciation expense is not included on the statement of cash flows because it does not affect cash. However, under the indirect method, depreciation expense is included under operating activities as an adjustment to net income to arrive at net cash flows from operating activities. 2. Issuance of common stock, $180,000, increases cash. The $180,000 would be included in the financing activities section. 3. Purchase of marketable securities, $375,000, decreases cash. Marketable securities are nonoperating assets for most businesses. Consequently, the purchase of these securities for cash would be disclosed in the investing activities section. 4. Prepaid insurance, $27,000, decreases cash. The $27,000 would be included in the operating activities section. 5. Down payment on building, $40,000, decreases cash. The $40,000 would be included under investing activities. (The mortgage would be disclosed in the footnotes to the financial statements or in a supplemental schedule to the statement of cash flows.)

E14–6 a.

1. 2. 3. 4. 5. 6. 7. 8.

9. 10.

Merchandise Inventory and Accounts Payable Prepaid Insurance Unearned Sales Revenue, Accounts Receivable and Allowance for Doubtful Accounts Rent Payable and Prepaid Rent Dividends Receivable Wages Payable Supplies Inventory Interest Payable, Prepaid Interest, Premium on Bonds Payable, and Discount on Bonds Payable. Note that if a company does not use a Premium on Bonds Payable account for bonds issued at a premium or a Discount on Bonds Payable account for bonds issued at a discount, it would be necessary to use the Bonds Payable account to analyze cash flows associated with interest. Unearned Rent Accumulated Depreciation


E14–6

Concluded

b. Sales Revenue 1. Unearned Sales Revenue would increase when a company collects cash from a customer in advance of providing goods or services. Since this amount would not be reflected in Sales Revenue until the company provided goods or services, the amount of the increase in Unearned Sales Revenue would have to be added to accrual-basis sales. 2. Accounts Receivable would increase when a company makes credit sales. The amount of the increase would be reflected in Sales Revenue, yet the company would not have collected any cash. Thus, the amount of the increase in Accounts Receivable would have to be deducted from accrualbasis sales. 3. An increase in Allowance for Doubtful Accounts by itself has no effect on accrual-basis sales. However, one must analyze the account to determine whether the company had any write-offs or recoveries of previously written off accounts during the year. Write-offs would be deducted from accrual-basis sales, while recoveries would be added to accrual-basis sales to arrive at cash collections from sales. Cost of Goods Sold Adjusting the balance in Cost of Goods Sold for changes in Merchandise Inventory and Accounts Payable will convert COGS from the accrual-basis to the cash outflow for inventory. An increase in Merchandise Inventory means that the company purchased more inventory than it sold during the year. An increase in the balance of Merchandise Inventory is added to COGS because it is assumed that the company paid cash for the inventory it acquired during the year. However, an increase in the balance of Accounts Payable indicates that this assumption is not valid. That is, an increase in Accounts Payable implies that the company purchased some of its inventory on account rather than for cash. Thus, the increase in Accounts Payable has to be deducted from COGS to offset the increase in Merchandise Inventory added to COGS that did not really reflect cash outflows. Interest Expense 1. An increase in Interest Payable would be deducted from accrual-basis interest expense because the increase implies that the company incurred interest expense, but has not yet paid it. Thus, the balance in Interest Expense overstates the amount of cash disbursed during the year for interest. 2. An increase in Prepaid Interest implies that the company disbursed cash during the year to cover future interest expense. Since the cash disbursement is not reflected in the current period's interest expense, the increase in Prepaid Interest would be added to accrual-basis interest expense to arrive at cash outflows associated with interest. 3. An increase in Discount on Bonds Payable does not affect interest expense per se; it indicates that the company issued additional bonds at less than face value. However, the net increase in the discount is comprised of two components. First, the discount balance increases for the discount associated with the new bonds issued. Second, the discount balance decreases for the amount of the discount balance amortized during the accounting period. Since the amount of the amortized discount flows into interest expense, the amount of the discount amortized would be deducted from accrual-basis interest expense. Consequently, the discount account would have to be


analyzed in depth to determine the magnitude of these two components. Similarly, the balance in Premium on Bonds Payable would have to be analyzed in depth.

E14–7 a.

Hamilton Direct method Cash collections from customers Cash paid for inventory Cash paid for other expenses Net cash flow from operating activities Indirect method Net income Adjustments: Depreciation expense Net cash flow from operating activities

$

$

900,000 (400,000) (200,000) 300,000

Watson $

$

900,000 (400,000) (200,000) 300,000

$ 250,000

$ 200,000

50,000 $ 300,000

100,000 $ 300,000

b. Cash flows from operating activities measures all the cash inflows and cash outflows associated with a company's operating assets and liabilities. Alternatively, net income measures the inflows and outflows of operating assets and liabilities, not just the cash associated with operating assets and liabilities. Thus, cash flows from operating activities can differ from net income due to items affecting net income that do not affect cash and due to cash flows that do not affect net income. In this particular case, the difference is due to depreciation expense. Depreciation is the systematic allocation of the cost of fixed assets. Since depreciation is simply the allocation of the asset's cost, depreciation does not affect cash flows. The indirect method more clearly shows that net income is different from cash flows because this method explicitly reconciles the difference between the two. c.

Disagree. Many people think that depreciation expense represents a fund established to finance future acquisitions of fixed assets. If this were true, it would follow that companies using accelerated depreciation methods would have more cash available than companies that use straight-line depreciation. However, as demonstrated in part (a), depreciation expense has absolutely no effect on the cash flows from operating activities. Both companies have the same cash flows, even though each company uses a different method to compute depreciation. One must remember that depreciation is simply the allocation of the net cost of fixed assets. Cash flows associated with fixed assets arise when fixed assets are acquired or sold, not when the cost of the fixed asset is allocated to expenses.

E14–8 a. 1.

2.

3.

4.

Cash (+A) ........................................................................................ Contributed Capital (+SE) ....................................................... Owner contributed capital.

20,000

Cash (+A) ........................................................................................ Notes Payable (+L) .................................................................. Borrowed money from the bank.

60,000

Property, Plant, & Equipment (+A) ................................................ Cash (–A) ................................................................................. Purchased long-lived assets.

25,000

Inventory (+A)

40,000

20,000

60,000

25,000


Cash (–A) ................................................................................. Accounts Payable (+L) ............................................................. Purchased inventory.

E14–8 5.

6.

7.

25,000 15,000

Continued Cash (+A) ........................................................................................ Accounts Receivable (+A) .............................................................. Sales (R, +SE) ........................................................................... Made sale.

20,000 60,000

Cost of Goods Sold (E, –SE) ............................................................ Inventory (–A) ......................................................................... Recognized cost of inventory sold.

25,000

Operating Expenses (E, –SE) .......................................................... Payable to Bank (–L) ...................................................................... Dividend (–SE)................................................................................ Cash (–A) ................................................................................. Made cash disbursements.

18,000 5,000 2,000

Operating Expenses (E, –SE) .......................................................... Operating Expenses Payable (+L) ............................................ Incurred, but did not pay, expenses.

15,000

80,000

25,000

25,000

15,000

b. Tony’s Business Income Statement For the Year Ended December 31, 2015 Sales ........................................................................................................... Cost of goods sold ..................................................................................... Operating expenses ................................................................................... Net income ................................................................................................

$ 80,000 (25,000) (33,000) $ 22,000

Tony's Business Statement of Retained Earnings For the Year Ended December 31, 2015 Beginning retained earnings balance: January 1, 2015 ............................. Plus: Net income........................................................................................ Less: Dividends .......................................................................................... Ending retained earnings, December 31, 2015 .........................................

$

0 22,000 (2,000) $ 20,000


E14–8

Continued Tony's Business Balance Sheet December 31, 2015

Assets Cash ....................................................... Accounts receivable .............................. Inventory ............................................... Property, plant & equipment ................

Total assets............................................ c.

$

25,000 60,000 15,000 25,000

$ 125,000

Liabilities & Stockholders' Equity Accounts payable...................... $ 15,000 Operating expenses payable .. 15,000 Payable to bank ........................ 55,000 Contributed capital ................... 20,000 Retained earnings ..................... 20,000 Total liabilities and stockholders' equity ................ $ 125,000

Beginning balance Owner's contribution Proceeds from bank loan Proceeds from sale

Cash 0 20,000 Purchase of fixed assets 25,000 60,000 Purchase of inventory 25,000 20,000 Operating expenses 18,000 Principal payment 5,000 Dividend payment 2,000

Ending balance

25,000

Tony's Business Statement of Cash Flows For the Year Ended December 31, 2015 Cash from operating activities: Cash collections from sales ..................................................... Cash paid for inventory ........................................................... Cash paid for expenses ............................................................ Net cash increase (decrease) due to operating activities ..........................................................................

$ 20,000 (25,000) (18,000) $ (23,000)

Cash from investing activities: Purchase of fixed assets .......................................................... Cash from financing activities: Proceeds from owner's contribution ......................................

(25,000)

$ 20,000


Proceeds from bank loan ........................................................ Principal repayment on debt ................................................... Payment of dividend ............................................................... Net cash increase (decrease) due to financing activities .......................................................................... Net increase (decrease) in cash....................................................

60,000 (5,000) (2,000) 73,000 $ 25,000

Beginning cash balance, January 1, 2015 ..................................... Ending cash balance, December 31, 2015....................................

E14–8

0 $ 25,000

Concluded

d. Tony's Business Statement of Cash Flows For the Year Ended December 31, 2015 Cash from operating activities: Net income ................................................................ Adjustments: Increase in accounts receivable ........................... Increase in inventory............................................ Increase in accounts payable ............................... Increase in operating expense payable................ Total adjustments ........................................... Net cash increase (decrease) due to operating activities ...........................

$ 22,000 $ (60,000) (15,000) 15,000 15,000 (45,000) $ (23,000)

Cash from investing activities: Purchase of fixed assets ............................................ Cash from financing activities: Proceeds from owner's contribution ........................ Proceeds from bank loan .......................................... Principal repayment on debt ..................................... Payment of dividend ................................................. Net cash increase (decrease) due to financing activities .............................................. Net increase (decrease) in cash......................................

(25,000)

$ 20,000 60,000 (5,000) (2,000) 73,000 $ 25,000

Beginning cash balance, January 1, 2015 ....................... Ending cash balance, December 31, 2015......................

0 $ 25,000

E14–9 a.

1.

2.

Cash (+A) ........................................................................................ Common Stock (+SE) ............................................................... Issued common stock.

6,000

Inventory (+A) ................................................................................ Accounts Payable (+L) ............................................................. Purchased inventory on account.

6,000

6,000

6,000


3.

4.

Equipment (+A) .............................................................................. Cash (–A) ................................................................................. Purchased equipment.

5,000

Cash (+A) ........................................................................................ Accounts Receivable (–A) ....................................................... Collected cash from customers.

10,000

5,000

10,000


E14–9 5.

6.

7.

8.

9.

10.

Continued Accounts Payable (–L).................................................................... Cash (–A) ................................................................................. Made payment to suppliers.

5,000

Dividend (–SE)................................................................................ Cash (–A) ................................................................................. Declared and paid cash dividend.

2,000

Rent Expense (E, –SE) .................................................................... Prepaid Rent (+A)........................................................................... Cash (–A) ................................................................................. Disbursed cash for rent.

6,000 6,000

Cash (+A) ........................................................................................ Accounts Receivable (+A) .............................................................. Sales (R, +SE) ........................................................................... Made sales.

65,000 35,000

Miscellaneous Expenses (E, –SE) ................................................... Cash (–A) ................................................................................. Incurred and paid miscellaneous expenses.

40,000

Cash (+A) ........................................................................................ Marketable Securities (–A) ..................................................... Gain on Sale of Marketable Securities (Ga, +SE) .................... Sold marketable securities.

25,000

b.

Cash (B.B.) (1) (4) (8) (10)

25,000 6,000 10,000 65,000 25,000

(E.B.)

67,000

(3) (5) (6) (7) (9)

5,000 5,000 2,000 12,000 40,000

5,000

2,000

12,000

100,000

40,000

20,000 5,000


E14–9

Concluded

c. Driftwood Shipbuilders Statement of Cash Flows For the Year Ended December 31, 2015 Cash flows from operating activities: Cash collections from customers ........................................ Cash payments for rent ....................................................... Cash payments for miscellaneous expenses ....................... Cash payments for inventory .............................................. Net cash increase due to operating activities ................

$ 75,000 (12,000) (40,000) (5,000)

Cash flows from investing activities: Proceeds from sale of marketable securities ...................... Purchase of equipment ....................................................... Net cash increase due to investing activities .................

$ 25,000 (5,000)

Cash flows from financing activities: Proceeds from issue of common stock ............................... Dividend payment ............................................................... Net cash increase due to financing activities ................. Net increase in cash ................................................................. Beginning cash balance, January 1, 2015 ................................. Ending cash balance, December 31, 2015................................

$ 18,000

20,000

$

6,000 (2,000) 4,000 $ 42,000 25,000 $ 67,000

E14–10 Insurance 2015 Ending prepaid insurance $7,000 Insurance purchases Wages 2015 Ending wages payable $6,000 Wages paid

= 2015 Beginning prepaid insurance + Insurance purchases during 2015– 2015 Insurance expense = $4,200 + Insurance purchases – $3,000 = $5,800

= 2015 Beginning wages payable + 2015 Wage expense – Wages paid during 2015 = $0 + $8,500 – Wages paid = $2,500


E14–11 a. 2015 Ending machinery $45,000 Machinery purchased

= 2015 Beginning machinery + Cost of machinery purchased during 2015 – Cost of machinery sold during 2015 = $20,000 + Machinery purchased – $8,000 = $33,000

b. When the machinery was sold during 2015, Dylan’s Toys, would prepare the appropriate entry using the following format. Cash (+A) ............................................................... Accumulated Depreciation (+A) ............................ Machinery (–A) ............................................... Gain on Sale of Machinery (Ga, +SE) ..............

XX XX XX XX

We can find the cash collected for the sale of the machinery by first calculating the other three amounts. Machinery It is given in the exercise that the cost of the machinery sold was $8,000. Gain on Sale of Machinery It is given in the exercise that the gain on the sale was $2,000. Accumulated Depreciation 2015 Ending accumulated depreciation

$15,000 Accumulated depreciation on items sold

= 2015 Beginning accumulated depreciation + 2015 Depreciation expense – Accumulated depreciation on items sold = $10,000 + $7,000 – Accumulated depreciation on items sold = $2,000

From the entry given above, Cash = Cost of machinery sold + Gain on sale of machinery – Accumulated depreciation on machinery sold = $8,000 + $2,000 – $2,000 = $8,000 c.

Cash (+A) ................................................................................................... Accumulated Depreciation (+A) ................................................................ Machinery (–A) ................................................................................... Gain on Sale of Machinery (Ga, +SE) .................................................. Sold machinery.

8,000 2,000 8,000 2,000

E14–12 Cash Receipts 499 Change in payables (‘12-‘11)

Cash Payments

12/31/11 accounts receivable

$

$

79

2012 Sales

17,097 2012 cost of sales

14,803

12/31/12 accounts receivable

(466) Change in inventory (‘11-’12)

(54)


2012 cash receipts

$17,130 2012 cash payments

$ 14,828


E14–13 1. Direct method The first step in calculating the cash flows from operating activities is to calculate the cash inflows and outflows associated with each income statement account. These calculations are given below. Cash collections from customers: Cash collections from customers

Cash paid for inventory: a. Inventory purchased 2015 Ending inventory $9,000 Net purchases

= 2015 Beginning inventory + Net inventory purchased during 2015 – 2015 Cost of goods sold = $11,000 + Net purchases – $30,000 = $28,000

b. Disbursements for inventory 2015 Ending accounts payable

$3,000 Payments Cash paid for wages: 2015 Ending wages payable $1,800 Wages paid Cash paid for advertising: 2015 Ending prepaid advertising $3,000 Advertising paid

= Sales – Ending accounts receivable + Beginning accounts receivable + Ending deferred revenues – Beginning deferred revenues = $48,000 – $4,000 + $5,000 + $0 – $3,000 = $46,000

= 2015 Beginning accounts payable + Net inventory purchased during 2015 – Payments for inventory during 2015 = $4,000 + $28,000 – Payments = $29,000

= 2015 Beginning wages payable + 2015 Wage Expense – Wages paid during 2015 = $900 + $4,000 – Wages paid = $3,100

= 2015 Beginning prepaid advertising + Advertising paid – 2015 Advertising expense = $1,200 + Advertising paid – $1,000 = $2,800

Cash flows from operating activities: Cash collections from customers........................................................ Payments for inventory ...................................................................... Payments for wages ........................................................................... Payments for advertising .................................................................... Net cash increase due to operating activities ....................................

$ 46,000 (29,000) (3,100) (2,800) $ 11,100


E14–13 Concluded 2. Indirect method Cash flows from operating activities: Net income ............................................................................ Adjustments: Depreciation ..................................................................... Decrease in accounts receivable ...................................... Decrease in inventory ....................................................... Increase in wages payable ................................................ Increase in prepaid advertising ........................................ Decrease in deferred revenues ........................................ Decrease in accounts payable .......................................... Total adjustments........................................................ Net cash increase due to operating activities ..............................................................

$ 11,000 $

2,000 1,000 2,000 900 (1,800) (3,000) (1,000) 100 $ 11,100

E14–14 Grimes Pools Statement of Cash Flows For the Year Ended December 31, 2015 Cash from operating activities: Cash collections from sales ..................................................... Cash paid on operating liabilities ............................................ Cash paid for expenses ............................................................ Net cash increase (decrease) due to operating activities ..........................................................................

$ 35,000 (6,000) (34,000) $ (5,000)

Cash from investing activities: Proceeds from sale of nonoperating assets ............................ Cash from financing activities: Proceeds from issuing debt ..................................................... Payment of dividends .............................................................. Repurchase of contributed capital .......................................... Net cash increase (decrease) due to financing activities .......................................................................... Net increase (decrease) in cash.................................................... Beginning cash balance, January 1, 2015 ..................................... Ending cash balance, December 31, 2015....................................

8,000a

$

2,000b (3,000) (4,000)c (5,000) $ (2,000)

$

6,000 4,000

a Proceeds from sale of nonoperating assets equals the decrease in nonoperating assets. Since no gain

on sale of assets or loss on sale of assets is reported, one must assume that the book value of the assets sold equaled the proceeds from the sale. b Proceeds from issue of debt = Increase in nonoperating liabilities c Repurchase of contributed capital = Decrease in contributed capital


E14–15 Romora Supply House Statement of Cash Flows For the Year Ended December 31, 2015 Cash from operating activities: Cash collections from sales ..................................................... Cash from decrease in noncash operating assets ................... Cash from increase in operating liabilities .............................. Cash paid for expenses ............................................................ Net cash increase (decrease) due to operating activities ..........................................................................

$ 64,000 5,000 5,000 (61,000) $ 13,000

Cash from investing activities: Purchase of nonoperating assets ............................................ Cash from financing activities: Proceeds from contributed capital.......................................... Payment on debt ..................................................................... Payment of dividends .............................................................. Net cash increase (decrease) due to financing activities .......................................................................... Net increase (decrease) in cash....................................................

(4,000)a

$

3,000b (2,000)c (3,000)

Beginning cash balance, January 1, 2015 ..................................... Ending cash balance, December 31, 2015....................................

(2,000) $ 7,000 5,000 $ 12,000

a Purchase of nonoperating assets = Increase in nonoperating assets b Proceeds from contributed capital = Increase in contributed capital c

Payment on nonoperating debt = Decrease in nonoperating liabilities

E14–16 Accrual-Basis Sales 2015 Sales = Cash inflows from sales made during 2015 + 2015 Ending accounts receivable – 2015 Beginning accounts receivable – 2015 Ending deferred revenue + 2015 Beginning deferred revenue = $65,000 + $3,000 – $9,000 – $4,000 + $1,000 = $56,000 Accrual-Basis COGS 2015 COGS = Cash disbursements during 2015 for inventory – 2015 Ending inventory + 2015 Beginning inventory + 2015 Ending accounts payable – 2015 Beginning accounts payable = $40,000 – $18,000 + $10,000 + $7,000 – $3,000 = $36,000


E14–16 Concluded Accrual-Basis Wage Expense 2015 Wage expense = Cash disbursements during 2015 for wages + 2015 Ending wages payable – 2015 Beginning wages payable = $6,000 + $2,100 – $1,300 = $6,800 Accrual-Basis Advertising Expense 2015 Advertising expense = Cash disbursement during 2015 for advertising – 2015 Ending prepaid advertising + 2015 Beginning prepaid advertising = $1,000 – $5,000 + $8,000 = $4,000 Depreciation Expense 2015 Depreciation expense

= 2015 Ending accumulated depreciation – 2015 Beginning accumulated depreciation = $8,000 – $5,000 = $3,000 Schlee and Associates Income Statement For the Year Ended December 31, 2015

Sales .............................................................................................................. Cost of goods sold ........................................................................................ Wage expense .............................................................................................. Advertising expense ..................................................................................... Depreciation expense ................................................................................... Net income ...................................................................................................

$ 56,000 (36,000) (6,800) (4,000) (3,000) $ 6,200

E14–17 L.L. Beeno Operating Section – Statement of Cash Flows (Direct Method) For the Year Ended December 31, 2015 Cash from operating activities: Cash receipts from revenues ................................................... Cash payments for inventory .................................................. Cash payments for wages........................................................ Cash payments for insurance .................................................. Cash payments for interest ..................................................... Cash payments for taxes ......................................................... Net cash increase (decrease) due to operating activities ..........................................................................

$ 45,900 (26,400) (5,100) (3,900) (1,600) _(1,200) $

7,700


E14–17 Concluded

L.L. Beeno Operating Section – Statement of Cash Flows (Indirect Method) For the Year Ended December 31, 2015 Cash flows from operating activities: Net income ............................................................................ Adjustments: Depreciation expense ....................................................... Increase in accounts receivable ........................................ Decrease in inventory ....................................................... Decrease in prepaid insurance ......................................... Decrease in accounts payable .......................................... Increase in wages payable ................................................ Total adjustments........................................................ Net cash increase due to operating activities ..............................................................

$ $

5,500

3,300 (1,100) 300 300 (1,700) 1,100 2,200 $

7,700

E14–18 Martland Stores Operating Section – Statement of Cash Flows (Direct Method) For the Year Ended December 31, 2015 Cash from operating activities: Cash receipts from revenues ................................................... Cash payments for inventory .................................................. Cash payments for wages........................................................ Cash payments for insurance .................................................. Cash payments for interest ..................................................... Net cash increase (decrease) due to operating activities ..........................................................................

$ 97,500 (59,200) (16,500) (8,900) _(2,100) $ 10,800


E14–18 Concluded

Martland Stores Operating Section – Statement of Cash Flows (Indirect Method) For the Year Ended December 31, 2015 Cash flows from operating activities: Net income ............................................................................ Adjustments: Depreciation expense ....................................................... Decrease in accounts receivable ...................................... Decrease in inventory ....................................................... Decrease in prepaid insurance ......................................... Decrease in accounts payable .......................................... Increase in wages payable ................................................ Total adjustments........................................................ Net cash increase due to operating activities ..............................................................

$ (3,600) $

5,700 1,500 5,300 300 (500) 2,100 14,400 $ 10,800

E14–19 Mako Retail Operating Section – Statement of Cash Flows (Direct Method) For the Year Ended December 31, 2015 Cash from operating activities: Cash receipts from revenues ................................................... Cash payments for inventory .................................................. Cash payments for wages........................................................ Cash payments for rent ........................................................... Cash payments for interest ..................................................... Cash payments for taxes ......................................................... Net cash increase (decrease) due to operating activities ..........................................................................

$ 108,700 (58,800) (13,000) (8,400) (3,600) _ (4,400) $ 20,500


E14–19 Concluded Mako Retail Operating Section – Statement of Cash Flows (Indirect Method) For the Year Ended December 31, 2015 Cash flows from operating activities: Net income ............................................................................ Adjustments: Depreciation expense ....................................................... Loss on sale of equipment ................................................ Increase in accounts receivable ........................................ Decrease in inventory ....................................................... Decrease in prepaid rent .................................................. Decrease in accounts payable .......................................... Increase in wages payable................................................ Decrease in interest payable ............................................ Increase in unearned revenue .......................................... Total adjustments........................................................ Net cash increase due to operating activities ..............................................................

$ 11,200 $

6,200 4,200 (2,200) 600 600 (3,400) 2,200 (700) _ 1,800 9,300 $ 20,500

E14–20 Steeler and Jones Operating Section – Statement of Cash Flows (Direct Method) For the Year Ended December 31, 2015 Cash from operating activities: Cash receipts from revenues ................................................... Cash payments for inventory .................................................. Cash payments for wages........................................................ Cash payments for rent ........................................................... Cash payments for interest ..................................................... Cash payments for taxes ......................................................... Net cash increase (decrease) due to operating activities ..........................................................................

$ 86,300 (50,600) (15,500) (11,400) (1,700) _(4,800) $

2,300


E14–20 Concluded Steeler and Jones Operating Section – Statement of Cash Flows (Indirect Method) For the Year Ended December 31, 2015 Cash flows from operating activities: Net income ............................................................................ Adjustments: Depreciation expense ....................................................... Gain on sale of equipment ............................................... Decrease in accounts receivable ...................................... Decrease in inventory....................................................... Increase in prepaid rent ................................................... Decrease in accounts payable .......................................... Decrease in wages payable............................................... Increase in interest payable ............................................. Decrease in unearned revenue ........................................ Total adjustments........................................................ Net cash increase due to operating activities ..............................................................

$

9,300

$

5,700 (5,200) 1,100 1,500 (400) (5,400) (2,300) 200 _ (2,200) (7,000) $

2,300

E14–21 Harbaugh Auto Supply Operating Section – Statement of Cash Flows (Direct Method) For the Year Ended December 31, 2015 Cash from operating activities: Cash receipts from sales revenue............................................ Cash receipts from service revenues....................................... Cash receipts from interest revenue ....................................... Cash payments for inventory .................................................. Cash payments for wages........................................................ Cash payments for rent ........................................................... Cash payments for interest ..................................................... Cash payments for taxes ......................................................... Net cash increase (decrease) due to operating activities ..........................................................................

$ 47,000 32,400 9,600 (26,700) (19,000) (16,500) (5,600) (10,300) $ 10,900


E14–21 Concluded Harbaugh Auto Supply Operating Section – Statement of Cash Flows (Indirect Method) For the Year Ended December 31, 2015 Cash flows from operating activities: Net income ............................................................................ Adjustments: Depreciation expense ....................................................... Gain on sale of investments ............................................. Increase in accounts receivable ........................................ Increase in inventory ........................................................ Decrease in interest receivable ........................................ Increase in prepaid rent ................................................... Decrease in accounts payable .......................................... Decrease in wages payable............................................... Increase in interest payable ............................................. Decrease in taxes payable ................................................ Decrease in unearned revenue ........................................ Total adjustments........................................................ Net cash increase due to operating activities ..............................................................

$ 24,300 $ 11,500 (13,200) (500) (2,900) 300 (1,200) (2,600) (1,400) 600 (1,200) (2,800) (13,400) $ 10,900

E14–22 Standard Center Manufacturing Operating Section – Statement of Cash Flows (Direct Method) For the Year Ended December 31, 2015 Cash from operating activities: Cash receipts from sales revenue............................................ Cash receipts from service revenue ........................................ Cash receipts from interest revenue ....................................... Cash payments for inventory .................................................. Cash payments for wages........................................................ Cash payments for rent ........................................................... Cash payments for interest .....................................................

$ 73,500 25,800 9,500 (14,600) (28,800) (24,200) (7,500)


Cash payments for taxes ......................................................... Net cash increase (decrease) due to operating activities ..........................................................................

_(3,400) $ 30,300

E14–22 Concluded Standard Center Manufacturing Operating Section – Statement of Cash Flows (Indirect Method) For the Year Ended December 31, 2015 Cash flows from operating activities: Net income ............................................................................ Adjustments: Depreciation expense ....................................................... Loss on sale of investments .............................................. Decrease in accounts receivable ...................................... Decrease in inventory ....................................................... Decrease in interest receivable ........................................ Increase in prepaid rent ................................................... Increase in accounts payable ........................................... Decrease in wages payable............................................... Decrease in interest payable ............................................ Decrease in taxes payable ................................................ Decrease in unearned revenue ........................................ Total adjustments........................................................ Net cash increase due to operating activities ..............................................................

E14–23 An estimate of net cash from operations would be: Income before tax Less: Tax Plus: Provision for Dep. & Amortization Less: Net gains on sales of assets Plus: Provision for impairment Less: Net increase in working capital Cash from operations

552 (543) 1,610 (186) 719 (42) 2,110

$2,800 $

4,300 17,900 6,000 1,000 2,200 (2,700) 3,900 (400) (300) (2,000) _(2,400) 27,500 $30,300


Taxes are deducted because they represent an operating cash outflow, while the non-cash charges for depreciation/amortization and impairment are added back. The sale of assets is an investing activity, so it should not be included in the operating section, while the increase in working capital represents a use of cash and is therefore deducted.

PROBLEMS P14–1 a., b., and c. Transaction 1. 2. 3. 4. 5. 6. 7. 8. 9. 10. 11.

Section Operating N/A Operating Investing Operating Financing N/A Financing N/A Financing Investing N/A

Inflow

Outflow

Amount

X

$ 60,000

X

40,000 94,000 15,000 75,000

X X X X

150,000

X

475,000 100,000

X

P14–2 a., b., and c. Transaction 1. 2. 3. 4. 5. 6. 7. 8. 9.

Section Operating Investing Operating N/A Operating Investing Operating Financing Financing Operating

Inflow X

Outflow X X

Amount $ 52,000 12,000 30,000

X X X

10,000 90,000 45,000 50,000 40,000 25,000

X X X


10. 11. 12. 13.

Financing N/A N/A N/A

X

300,000

P14–3 a. Transaction 1. 2. 3. 4. 5. 6. 7. 8. 9. 10. 11. 12. 13.

b.

1. 2. 3. 4. 5.

6. 7. 8. 9. 10. 11. 12.

Cash Affected Yes Yes Yes Yes Yes Yes No Yes Yes No No No No

Type of Effect Provided Used Used Provided Provided Provided Used Used

Dollar Amount $ 1,200 13,000 9,000 7,000 2,500 3,000 7,000 5,000

Investing Investing Operating Financing Operating (for the interest) Investing or Operating (for the principal) depending on how the company acquired the note. If it was accepted in a sales transaction, it could be argued that collecting the principal is an operating activity. If the company acquired the note in any other way, the collection of the principal would be considered an investing activity. Operating N/A Financing Operating N/A N/A N/A


13.

N/A

P14–4 The two most notable similarities among all three companies are that they have generated increasing amounts of cash from their operating activities and that they all have used cash to invest in long-term assets. Two of the three companies also look very similar from a financing perspective. Priceline and Amazon typically generated cash from financing activities, meaning the firms issued equity and/or debt to raise cash. On the other hand, eBay used cash in its financing activities, which indicates either dividends have been paid, debt has been retired, or equity has been repurchased.


P14–5 Transaction

Effect on Cash

1.

$50,000

2.

(55,000)

3.

100,000

Section of Statement

Explanation

Operating inventory activity. Operating expenses. Investing

Operations is defined in terms of Cash payments for operating Sale of a nonoperating asset.

Note: If the company uses the indirect method to prepare its statement of cash flows, the $15,000 loss on sale of fixed assets would be included in the operating activities section as an adjustment to net income to arrive at net cash flow from operating activities. 4. 5. 6.

(70,000) (500,000) (100,000)

Financing Financing Operating

7. 8.

No effect (500,000)

N/A Investing in equity securities,

9.

$202,000

Financing

10.

No effect

Operating

Dividend payment. Stock repurchase. Cash payment associated with operating assets. No effect on cash. Cash payment for an investment a nonoperating asset. Issuing debt provides financing to the company. Net income would be lower but would be offset by an increase in rent payable. The net of these two is a zero effect on cash.

P14–6

a. Cash from operating activities Cash from investing activities Cash from financing activities Increase (decrease) in cash Cash balance at beginning of year Cash balance at end of year Missing values

2015

2014

2013

$

X 160 (150) $ X 86 $ 176

$

(202) X 280 (24) X 86

$

$

$

(102) 110

$

80 90

$ $

$ $

X (500) 900 110 0 X (290) 110

b. Ruttman Enterprises began operations during 2013. As part of its start-up phase, the company appears to have issued large amounts of stock and/or debt to finance the acquisition of nonoperating assets, to cover the cash used by operating activities during the start-up phase, and to provide a cash surplus for future years. During 2014, the company once again issued large amounts of stock and/or debt to finance the acquisition of nonoperating assets and to cover the cash used by operating activities. However, the company acquired only enough additional financing to meet these needs; it did not acquire an excessive amount of capital through financing activities. By 2015 the company was generating cash from its operating activities and was able to dispose of some of its nonoperating assets. These cash inflows were then used to retire some of the company's debt and/or to reacquire some of its outstanding stock.


P14–7 a.

2012 Cash provided (used) by operating activities Cash provided (used) by investing activities Cash provided (used) by financing activities Increase (decrease) in cash Cash balance at beginning of year

2011

2010

$10,571 $12,639 $11,922 (3,453) (13,959) (11,359) (3,860) (1,566) (2,913) 3,258 (2,886) (2,350) 8,043 10,929 13,279

Cash balance at end of year

$

11,301

$ 8,043

$10,929

b. Cash from operations has been consistently strong over the three-year period. H-P uses this cash flow to invest in its growing business (negative cash from investing activities in all three years) and is still able to return cash to shareholders and repay any debt (negative cash from financing activities in all three years). The company’s strong cash flow from operations allows it to continue to fund its growth and to return cash to the sources of funding (debt and equity providers).

P14–8 a. Case 1: Based on the $820,000 beginning balance in the Buildings account and the purchase during 2015 of a building for $60,000, one would expect the Buildings account to have a balance of $880,000 at the end of 2015. The fact that its balance is only $750,000 implies that Webb Industries must have sold a building that originally cost $130,000. Similarly, based on the beginning balance of $80,000 in the Accumulated Depreciation: Buildings account and the $40,000 of depreciation taken on the building during 2015, one would expect the Accumulated Depreciation: Buildings account to have a balance of $120,000 at the end of 2015. The fact that its balance is only $100,000 implies that the accumulated depreciation associated with the building that Webb Industries sold during 2015 must have been $20,000. This information is summarized in the following T accounts. Buildings B.B. Purchase

820,000 60,000

E.B.

750,000

Sale

Accumulated Depreciation X

Sale

Y

B.B. Depr. Exp.

80,000 40,000

E.B.

100,000

X = Cost of building sold = $130,000 Y = Accumulated depreciation on building sold = $20,000 Case 2: Based on the $380,000 beginning balance in the Equipment account and the sale during 2015 of equipment that originally cost $50,000, one would expect the Equipment account to have a balance at the end of 2015 of $330,000. The fact that its balance is $500,000 implies that Webb Industries must have purchased some equipment for $170,000. Similarly, based on the beginning balance of $85,000 in the Accumulated Depreciation: Equipment account and the $15,000 of depreciation taken on the equipment during 2015, one would expect the Accumulated Depreciation: Equipment account to have a balance at the end of 2015 of $100,000. The fact that its balance is only $75,000 implies that the accumulated depreciation associated with the equipment that Webb Industries sold during 2015 must have been $25,000. This information is summarized in the following T accounts.


P14–8 Continued Equipment B.B. Purchase

380,000 X

E.B.

500,000

Sale

Accumulated Depreciation 50,000

Sale

Y

B.B. Depr. Exp.

85,000 15,000

E.B.

75,000

X = Cost of equipment purchased = $170,000 Y = Accumulated depreciation on equipment sold = $25,000 Case 3: Based on the $250,000 beginning balance in the Land account and the sale of land during 2015, one would expect the balance in the Land account to decrease. The fact that its balance is still $250,000 at the end of 2015 implies that (1) Webb Industries must have purchased some land during 2015 and (2) the cost of the land purchased exactly equaled the original cost of the land that was sold. Since a gain on the sale of land equals the excess of the proceeds over the cost of the land, it can be inferred that the land that Webb Industries sold during 2015 originally cost $225,000 (i.e., proceeds of $300,000 less gain of $75,000). Thus, the cost of the land that Webb Industries purchased during 2015 was $225,000. This information is summarized in the following T account. Land B.B. Purchase

250,000 Y

E.B.

250,000

Y = Cost of land purchased = $225,000

Sale

X

X = Cost of land sold = $225,000

Case 4: In exchange between two independent parties, one would expect the fair market value of the item given up by one of the parties to equal the fair market value of the item that party is to receive. Thus, it is probably safe to assume that the fair market value of the new building is $600,000. Based on the $820,000 beginning balance in the Buildings account and the acquisition during 2015 of a new building for $600,000, one would expect the Buildings account to have a balance of $1,420,000 at the end of 2015. The fact that its balance is only $750,000 implies that Webb Industries must have disposed of a building during 2015 that originally cost $670,000. Similarly, based on the beginning balance of $80,000 in the Accumulated Depreciation: Buildings account and the $40,000 of depreciation taken on buildings during 2015, one would expect the Accumulated Depreciation: Buildings account to have a balance at the end of 2015 of $120,000. The fact that its balance is only $100,000 implies that the accumulated depreciation associated with the building that Webb Industries sold during 2015 must have been $20,000. Further, based on the $250,000 beginning balance in the Land account and the disposal during 2015 of land that had a book value of $150,000, one would expect the Land account to have a balance of $100,000 at the end of 2015. The fact that its balance is $250,000 implies that Webb Industries must have acquired some land during 2015 that cost $150,000. This information is summarized in the following T accounts.


P14–8 Continued Buildings B.B. Purchase

820,000 600,000

E.B.

750,000

Sale

Accumulated Depreciation X

Sale

Y

B.B. Depr. Exp.

80,000 40,000

E.B.

100,000

X = Cost of building sold = $670,000 Y = Accumulated depreciation on building sold = $20,000 Land B.B. Purchase

250,000 Z

E.B.

250,000

Sale

150,000

Z = Cost of land purchased = $150,000 b. Case 1: Proceeds from sale

Note:

= Cost of building sold – Related accumulated depreciation = $130,000 – $20,000 = $110,000

This solution assumes that there was no gain or loss on the sale of the building, since no such information was given in the problem.

In the statement of cash flows for 2015, Webb Industries would report the following items under cash flows from investing activities. Proceeds from sale of building Purchase of building Case 2: Proceeds from sale

$ 110,000 (60,000)

= Book value of equipment sold + Gain on sale = ($50,000 – $25,000) + $5,000 = $30,000

In the statement of cash flows for 2015, Webb Industries would report the following items under cash flows from investing activities. Proceeds from sale of equipment Purchase of equipment

$

30,000 (170,000)

Case 3: In the statement of cash flows for 2015, Webb Industries would report the following items under cash flows from investing activities. Proceeds from the sale of land

$300,000


Purchase of land

(225,000)

P14–8 Concluded Case 4: Since Webb Industries exchanged land for a building, this transaction did not affect cash and would not be disclosed in the body of the statement of cash flows. However, Webb Industries did collect $650,000 from the building it sold ($670,000 cost of building sold – $20,000 accumulated depreciation on the building), which Webb Industries would report under cash flows from investing activities.

P14–9 Total number of shares issued during 2015

Number of shares issued for cash

= Change in balance of common stock account ÷ Par value per share of common stock = ($128,000 – $100,000) ÷ $1 per share = 28,000 shares

= Total number of shares issued – (Shares issued as stock dividend + Shares issued in exchange for land) = 28,000 shares – [(100,000 shares outstanding on 1/1/15  20%) + 6,000 shares exchanged for land] = 28,000 shares – (20,000 shares + 6,000 shares) = 2,000 shares

Cash received = Change in common stock account due to issue of common stock for cash + Change in additional paid-in capital, common stock account due to issue of common stock for cash = (2,000 shares  $1 par value per share) + [($95,000 – $12,000) – $40,000a + $12,000b] = $2,000 + $31,000 = $33,000 a $40,000 represents the additional paid-in capital from the 20% stock dividend. The company distributed

20,000 shares, and the fair market value at the time was $3 per share. One dollar was allocated to the Common Stock account, and the remaining $2 was allocated to the Additional Paid-in Capital, Common Stock account. b $12,000 represents the additional paid-in capital from exchanging stock for land.

P14–10 a. Ending accounts receivable

2014 $95,000 Gross sales 2015

= Beginning accounts receivable + (Gross sales – Sales returns) – Cash collections

= $0 + (Gross sales – $20,000) – $350,000 = $465,000


$150,000 Gross sales

= $95,000 + (Gross sales – $25,000) – $500,000 = $580,000

P14–10 Concluded b. 2015 Ending inventory $110,000 Net purchases

= 2015 Beginning inventory + Net purchases of inventory during 2015 – 2015 Cost of goods sold = $130,000 + Net purchases – $375,000 = $355,000

2015 Ending accounts payable

$115,000 Cash payments c.

Ending prepaid insurance

= 2015 Beginning accounts payable + Net purchases of inventory during 2015 – Cash payments for inventory during 2015 = $105,000 + $355,000 – Cash payments = $345,000 = Beginning prepaid insurance + Insurance purchased during the year – Insurance expense

2014 Ending prepaid insurance

= $0 + $65,000 – $35,000 = $30,000

2015 Ending prepaid insurance

= $30,000 + $90,000 – $50,000 = $70,000

P14–11 Accrual sales

= = =

Collections from customers + Increase in accounts receivable $26,000 + $3,000 $29,000

Accrual COGS

= Payments to suppliers – Increase in inventory + Increase in accounts payable = $13,000 – $3,000 + $1,000 = $11,000

Accrual operating expenses

= Payments for expenses – Decrease in accrued payables = $10,000 – $2,000 = $8,000 Battery Builders, Inc. Income Statement

Sales .................................................................................................................. Cost of goods sold ............................................................................................ Depreciation expense ....................................................................................... Other operating expenses ................................................................................ Gain on sale of equipment ............................................................................... Net income .......................................................................................................

$ 29,000 (11,000) (3,000) (8,000) 2,000 $ 9,000


P14–12 a. Pendleton Enterprises Statement of Cash Flows from Operating Activities For the Years Ended December 31, 2013, 2014, and 2015 Cash collections from customers and sales* ............................. Cash payments for expenses and to suppliers** ....................... Net cash increase (decrease) due to operating activities ................................................................................ *Revenues minus increase in Accounts Receivable **Expenses minus increase in Accounts Payable

$

2015 9,000 (12,000)

2014 $ 13,000 (8,000)

2013 $ 4,000 (4,000)

$

(3,000)

$

$

5,000

0

b. Pendleton Enterprises Statement of Cash Flows from Operating Activities For the Years Ended December 31, 2013, 2014, and 2015 Cash collections from customers and sales ............................... Cash payments for expenses and to suppliers ........................... Net cash increase (decrease) due to operating activities ................................................................................

$

2015 9,000 (12,000)

2014 $ 9,000 (8,000)

2013 $ 8,000 (4,000)

$

(3,000)

$

$

1,000

4,000

c. Pendleton Enterprises Statement of Cash Flows from Operating Activities For the Years Ended December 31, 2013, 2014, and 2015 Cash collections from customers and sales ............................... Cash payments for expenses and to suppliers ........................... Net cash increase (decrease) due to operating activities ................................................................................

2015 $ 9,000 (12,000)

2014 $ 13,000 (11,000)

2013 $ 4,000 (1,000)

$

$

$

(3,000)

2,000

3,000

d. Managers can manipulate cash flows from operating activities by manipulating the timing of cash collections and cash payments associated with operating activities. By comparing parts (b) and (c) with part (a), it is obvious that these types of manipulations offset themselves in the next period. So if a manager wants to continue manipulating cash flows from operating activities, the manager will have to manipulate the timing of cash inflows and outflows every year.


P14–13 Watson and Holmes Detective Agency Statement of Cash Flows – Direct Method For the Year Ended December 31, 2015 Cash flows from operating activities: Cash collections from customers ............................................ Cash paid for inventory ........................................................... Cash paid for interest .............................................................. Cash paid for other expenses .................................................. Net cash increase (decrease) due to operating activities .......................................................................... Cash flows from investing activities: Purchase of long-lived assets .................................................. Net cash increase (decrease) due to investing activities .......................................................................... Cash flows from financing activities: Cash paid for dividends ........................................................... Proceeds from issuance of common stock .............................. Net cash increase (decrease) due to financing activities .......................................................................... Net increase in cash ..................................................................... Beginning cash balance, January 1, 2015 ..................................... Ending cash balance, December 31, 2015.................................... * $35,500

$ 34,500* (23,000) (2,800) (9,000) $

$

(300)

(1,000) (1,000)

$

(700) 6,000

$

5,300 4,000

6,000 $ 10,000

= $42,000 revenues – $5,000 increase in accounts receivable + $500 increase in allowance for doubtful accounts – $2,000 bad debt expense – $1,000 decrease in deferred revenues


P14–13 Concluded Watson and Holmes Detective Agency Statement of Cash Flows – Direct Method For the Year Ended December 31, 2015 Cash flows from operating activities: Net income ................................................................ Adjustments: Depreciation......................................................... Increase in allowance for doubtful accounts ....... Decrease in accounts payable .............................. Decrease in discount on note payable ................. Increase in accounts receivable ........................... Decrease in inventory .......................................... Decrease in deferred revenues ............................ Total adjustments ........................................... Net cash increase (decrease) due to operating activities.............................

$ 2,000 $

2,000 500 (1,000) 200 (5,000) 2,000 (1,000)

Cash flows from investing activities: Purchase of long-lived assets .................................... Net cash increase (decrease) due to investing activities ............................................ Cash flows from financing activities: Cash paid for dividends ............................................. Proceeds from issuance of common stock ................ Net cash increase (decrease) due to financing activities ............................................ Net increase in cash .......................................................

(2,300) $

(300)

$ (1,000) (1,000) $

(700) 6,000

$

Beginning cash balance, January 1, 2015 ....................... Ending cash balance, December 31, 2015......................

5,300 4,000

6,000 $ 10,000

P14–14 a.

1.

2.

3.

4.

Cash (+A) ........................................................................................ Common Stock (+SE) ............................................................... Issued common stock.

60,000

Inventory (+A) ................................................................................ Accounts Payable (+L) ............................................................. Purchased inventory on account.

20,000

Prepaid Rent (+A)........................................................................... Cash (–A) ................................................................................. Prepaid rent.

7,000

Furniture (+A) ................................................................................ Cash (–A) ................................................................................. Long-Term Note Payable (+L) ................................................. Purchased furniture.

30,000

60,000

20,000

7,000

20,000 10,000


P14–14 Continued 5a.

5b.

6.

7.

8.

9.

10.

11.

12.

Accounts Receivable (+A) .............................................................. Sales (R, +SE) ........................................................................... Made sales on account.

35,200

Cost of Goods Sold (E, –SE) ............................................................ Inventory (–A) ......................................................................... Recognized cost of inventory sold.

8,800

Accounts Payable (–L).................................................................... Cash (–A) ................................................................................. Made payment to suppliers.

10,000

Cash (+A) ........................................................................................ Accounts Receivable (–A) ....................................................... Collected cash from customers.

12,000

Miscellaneous Expenses (E, –SE) ................................................... Cash (–A) ................................................................................. Incurred and paid miscellaneous expenses.

10,000

Depreciation Expense (E, –SE) ....................................................... Accumulated Depreciation (–A).............................................. Depreciated furniture.

5,000

Interest Expense (E, –SE) ............................................................... Accrued Interest Payable (+L) ................................................. Incurred, but did not pay, interest.

1,000

Dividends (–SE) .............................................................................. Dividends Payable (+L) ............................................................ Declared dividends.

3,000

Rent Expense (E, –SE) .................................................................... Prepaid Rent (–A) .................................................................... Used portion of prepaid rent.

3,000

35,200

8,800

10,000

12,000

10,000

5,000

1,000

3,000

3,000

b. ISS, Inc. Income Statement For the Year Ended December 31, 2015 Sales .................................................................................................................. Cost of goods sold ............................................................................................ Depreciation expense ....................................................................................... Rent expense .................................................................................................... Interest expense ............................................................................................... Miscellaneous expenses ................................................................................... Net income .......................................................................................................

$ 35,200 (8,800) (5,000) (3,000) (1,000) (10,000) $ 7,400


P14–14 Continued ISS, Inc. Statement of Retained Earnings For the Year Ended December 31, 2015 Beginning retained earnings balance: January 1, 2015 .................................... Plus: Net income............................................................................................... Less: Dividends ................................................................................................. Ending retained earnings balance: December 31, 2015...................................

$

0 7,400 (3,000) $ 4,400

ISS, Inc. Balance Sheet December 31, 2015 Assets Cash .............................................. Accounts receivable ..................... Inventory ...................................... Prepaid rent ................................. Furniture ...................................... Accumulated depreciation ...........

$ 25,000 23,200 11,200 4,000 30,000 (5,000)

Total assets...................................

$ 88,400

Liabilities & Stockholders' Equity Accounts payable ...................... Accrued interest payable .......... Dividend payable....................... Long-term note payable............ Common stock .......................... Retained earnings ..................... Total liabilities and stockholders' equity ................

$ 10,000 1,000 3,000 10,000 60,000 4,400 $ 88,400

ISS, Inc. Statement of Cash Flows – Indirect Method For the Year Ended December 31, 2015 Cash flows from operating activities: Net income ..................................................................... Adjustments: Depreciation ............................................................ Increase in accounts receivable............................... Increase in inventory ............................................... Increase in prepaid rent .......................................... Increase in accounts payable .................................. Increase in accrued interest payable....................... Total adjustments ............................................. Net cash increase (decrease) due to operating activities ....................... Cash flows from investing activities: Purchase of furniture ..................................................... Net cash increase (decrease) due to investing activities ................................................ Cash flows from financing activities: Proceeds from issuance of common stock..................... Net cash increase (decrease) due to financing activities ............................................... Net increase (decrease) in cash ............................................ Beginning cash balance, January 1, 2015 .............................. Ending cash balance, December 31, 2015 ............................

$ $

7,400

5,000 (23,200) (11,200) (4,000) 10,000 1,000 (22,400) $ (15,000) $ (20,000) (20,000) $ 60,000 60,000 $ 25,000 0 $ 25,000


P14–14 Concluded c.

Working capital

Current ratio

= Current assets ÷ Current liabilities = $63,400 ÷ $14,000 = 4.53

d. Working capital

Current ratio

= Current assets – Current liabilities = ($25,000 + $23,200 + $11,200 + $4,000) – ($10,000 + $1,000 + $3,000) = $63,400 – $14,000 = $49,400

= ($63,400 – $10,000 cash) – ($14,000 – $10,000 accounts payable) = $53,400 – $4,000 = $49,400 = $53,400 ÷ $4,000 = 13.35

Net cash flow used by operating activities

= ($15,000) from part (b) + ($10,000) = ($25,000)

P14–15 a. Marketing revenue 2015 Ending accounts receivable $150,000 Cash collections

= 2015 Beginning accounts receivable + 2015 Marketing revenue – Cash collections during 2015 = $105,000 + $1,000,000 – Cash collections = $955,000

Salary expense Since no related balance sheet account exists as of December 31, 2014 or as of December 31, 2015, it is safe to assume that the entire balance in Salary Expense was paid in cash. Therefore, the cash paid for salaries equals $250,000. Supplies expense 2015 Ending office supply inventory

$75,000 Office supply purchases

= 2015 Beginning office supply inventory + Office supplies purchased during 2015 – 2015 Office supplies expense = $85,000 + Supplies purchased – $175,000 = $165,000

Depreciation expense Depreciation is the allocation of the cost of a fixed asset. Depreciation does not provide cash or use cash; hence, the cash flow associated with depreciation is zero. Insurance expense 2015 Ending prepaid insurance

= 2015 Beginning prepaid insurance + Insurance purchased during 2015 – 2015 Insurance expense


$50,000 Insurance purchases

= $10,000 + Insurance purchased – $60,000 = $100,000


P14–15 Concluded Rent Expense 2015 Ending rent payable $20,000 Rent paid

= 2015 Beginning rent payable + 2015 Rent expense – Rent paid during 2015 = $8,000 + $120,000 – Rent paid = $108,000

This method would be similar to directly computing net cash flow from operating activities since the actual cash flow for each component of operating activities is being computed. Under the indirect method, net cash flow from operating activities is computed by adjusting net income. b. Net income ................................................................................... Adjustments: Depreciation expense .............................................................. Decrease in office supplies inventory ..................................... Increase in rent payable .......................................................... Increase in accounts receivable .............................................. Increase in prepaid insurance ................................................. Total adjustments .............................................................. Net cash increase due to operating activities .............

$ 295,000 $ 100,000 10,000 12,000 (45,000) (40,000) 37,000 $ 332,000

This method is similar to the indirect method since net income is adjusted for the change in each operating account on the balance sheet to arrive at cash flows from operating activities. The magnitudes of these adjustments are the same as in part (a). However, in part (a) each of these adjustments was related to an individual operating activity to determine the actual cash inflow or actual cash outflow associated with the individual operating activity; whereas in part (b) each adjustment is related to net income to arrive at the overall net cash flow associated with all operating activities. c.

Operating Cash Flows Income Statement

Adjustments

Marketing revenue Salary expense Office supplies exp.

$ 1,000,000 (250,000) (175,000)

Depreciation exp. Insurance expense Rent expense

(100,000) (60,000) (120,000)

Net income

$

295,000

Increase in accts. receivable None Decrease in office supplies inventory Depreciation expense Increase in prepaid insurance Increase in rent payable Cash provided by operating activities

Direct Method $(45,000) 0

$

10,000 100,000 (40,000) 12,000

955,000 (250,000) (165,000) 0 (100,000) (108,000)

$

332,000


P14–16 Direct method Bower Manufacturing Industries Statement of Cash Flows For the Year Ended December 31, 2015 Cash flows from operating activities: Cash collections from sales and accounts receivable.............. Cash paid to suppliers for inventory ....................................... Cash paid for wages................................................................. Cash paid for supplies.............................................................. Cash paid for interest .............................................................. Net cash decrease due to operating activities ...................

$ 90,000a (90,000) (12,000) (5,000) (5,000)

Cash flows from investing activities: Proceeds from sale of fixed assets .......................................... Proceeds from sale of marketable securities .......................... Net cash increase due to investing activities .....................

$ 125,000b 51,000c

$

(22,000)

176,000

Net cash flow from financing activities ........................................ Net increase in cash .....................................................................

0 $ 154,000

Beginning cash balance, January 1, 2015 ..................................... Ending cash balance, December 31, 2015....................................

593,000 $ 747,000

a Cash collections include changes in both Accounts and Notes Receivable. b Proceeds from sale of fixed assets is computed as follows:

Cost of machinery and equipment sold ............................................... Less: Related accumulated depreciation.............................................. Book value of machinery and equipment sold ..................................... Less: Loss on sale of fixed assets (per income statement) ................... Proceeds from sale of fixed assets ....................................................... c Proceeds from sale of marketable securities is computed as follows: Decrease in marketable securities ....................................................... Less: Loss on sale of marketable securities (per income statement) ........................................................................................ Proceeds from sale of marketable securities .......................................

$ 150,000 15,000 $ 135,000 10,000 $ 125,000

$ 55,000 4,000 $ 51,000


P14–16 Concluded Indirect method Bower Manufacturing Industries Statement of Cash Flows For the Year Ended December 31, 2015 Cash flows from operating activities: Net income ..................................................................... Adjustments: Decrease in inventory .............................................. Depreciation ............................................................ Decrease in discount on bonds payable .................. Decrease in supplies inventory................................ Loss on sale of fixed assets ...................................... Loss on sale of marketable securities ...................... Increase in accounts receivable............................... Increase in notes receivable .................................... Decrease in accounts payable ................................. Total adjustments ............................................. Net cash decrease due to operating activities ................................................... Cash flows from investing activities: Proceeds from sale of fixed assets ................................. Proceeds from sale of marketable securities ................. Net cash increase due to investing activities ..........

$

37,000

$ 25,000 30,000 5,000 2,000 10,000 4,000 (50,000) (50,000) (35,000) (59,000) $

(22,000)

$ 125,000 51,000 176,000

Net cash flow from financing activities ................................. Net increase in cash ..............................................................

0 $ 154,000

Beginning cash balance, January 1, 2015 .............................. Ending cash balance, December 31, 2015 ............................

593,000 $ 747,000


P14–17 Direct method Price Restaurant Supply Company Statement of Cash Flows For the Year Ended December 31, 2015 Cash flows from operating activities: Cash collections from customers ............................................ Cash paid to suppliers for inventory ....................................... Cash paid for interest .............................................................. Net cash decrease due to operating activities ................... Cash flows from investing activities: Proceeds from sale of plant equipment .................................. Purchase of plant equipment .................................................. Net cash increase due to investing activities ..................... Cash flows from financing activities: Proceeds from common stock issue........................................ Payment of dividends .............................................................. Net cash increase due to financing activities ..................... Net increase in cash ..................................................................... Beginning cash balance, January 1, 2015 ..................................... Ending cash balance, December 31, 2015....................................

$ 165,000 (199,000) (13,000) $

$

(47,000)

90,000a (25,000)a 65,000

$

65,000b (30,000)c $

35,000 53,000

120,000 $ 173,000

a Explanation of activity involving plant equipment:

Ending plant equipment $275,000 Equipment purchased

= Beginning plant equipment + Equipment purchased – Equipment sold = $350,000 + Equipment purchased – $100,000 = $25,000

Proceeds from sale of equipment

= Book value of assets sold + Gain on the sale = [(Asset cost – Accumulated depreciation on asset sold) + Gain on the sale] = [($100,000 – $20,000) + $10,000] = $90,000

b Proceeds from issue of common stock

c

Dividends

= Increase in common stock + Increase in additional paid-in capital (common stock) = $35,000 + $30,000 = $65,000

= Beginning retained earnings + Net income – Ending retained earnings = $204,000 + $37,000 – $211,000 = $30,000


P14–17 Concluded Indirect method Price Restaurant Supply Company Statement of Cash Flows For the Year Ended December 31, 2015 Cash flows from operating activities: Net income ........................................................... Adjustments: Depreciation.................................................... Decrease in accounts receivable ..................... Decrease in prepaid insurance ........................ Increase in accounts payable .......................... Increase in inventory....................................... Decrease in premium on bonds payable ........ Gain on sale of plant equipment..................... Total adjustments ...................................... Net cash decrease due to operating activities .......................... Cash flows from investing activities: Proceeds from sale of plant equipment ............... Purchase of plant equipment ............................... Net cash increase due to investing activities ....................................... Cash flows from financing activities: Proceeds from common stock issue..................... Payment of dividends ........................................... Net cash increase due to financing activities ....................................................... Net increase in cash .................................................. Beginning cash balance, January 1, 2015 .................. Ending cash balance, December 31, 2015.................

$ $

37,000

12,000 5,000 10,000 1,000 (100,000) (2,000) (10,000) (84,000) $

$

(47,000)

90,000 (25,000) 65,000

$

65,000 (30,000)

$

35,000 53,000

$

120,000 173,000


P14–18 a. 2014 Cash collections from customers: Sales for cash .................................................................... Cash collections from accounts receivable ...................... Total cash collections from customers ...................... Cash payments for: Salary ................................................................................ Advertising........................................................................ Administrative expenses .................................................. Janitorial expense ............................................................. Supplies ............................................................................ Total cash paid for expenses ..................................... Net cash increase due to operating activities .......................

$ 3,633,500a 4,936,500b $ 8,570,000c $ 1,649,000d 710,000 832,000 120,000 232,000e (3,543,000) $ 5,027,000

a $3,633,500 = 2014 sales of $5,590,000  65% b Cash collections = Sales on account + Decrease in accounts receivable

= ($5,590,000  35%) + $2,980,000 = $4,936,500 c An alternative approach to calculating the $8,570,000 is to add the decrease in Accounts Receivable during 2014 of $2,980,000 to the 2014 sales of $5,590,000. d $1,649,000 = Salary expense of $1,794,000 – Salary payable of $145,000 e $232,000 = Supplies expense of $299,000 – Supply inventory payable of $67,000 2015 Cash collections from customers: Sales for cash ......................................................................... Cash collections from accounts receivable ........................... Total cash collections from customers ........................... Cash payments for: Salary ..................................................................................... Advertising............................................................................. Administrative expenses ....................................................... Janitorial expense .................................................................. Supplies ................................................................................. Total cash paid for expenses .......................................... Net cash increase due to operating activities ............................

$ 1,491,750a 2,803,500b $

4,295,250c

$

(4,228,000) 67,250

$ 2,145,000d 705,000e 898,000 132,000 348,000f

= 2015 sales of $5,967,000  25% = Sales on account – Increase in accounts receivable = ($5,967,000  75%) – $1,671,750 c This amount can also be calculated as 2015 sales of $5,967,000 less the increase in Accounts Receivable of $1,671,750 during 2015. d $2,145,000 = Salary expense of $2,025,000 – Ending salary payable of $25,000 + Beginning salary payable of $145,000 e $705,000 = Advertising expense of $755,000 – Advertising payable of $50,000 a $1,491,750 b $2,803,500


f

$348,000

= Supplies expense of $281,000 + Beginning supply inventory payable of $67,000

P14–18 Concluded b. The first thing that must be explained to the stockholders is the nature of dividends. Dividends are paid out of assets, not out of net income. If a company has insufficient assets or has alternative uses for its assets, it will be unable to declare a cash dividend. Although net income is a measure of the net assets that have flowed into the company during the year from operations, these net assets may be in a form, such as inventory or accounts receivable, that cannot be easily distributed to stockholders. Thus, net income provides only a low-quality indication of potential dividends. A better indication of a company's potential ability to declare and pay cash dividends is the net cash flows from operating activities. In this particular case, 2014 was a good year for the Lynch Engineering Firm. The company generated $597,000 in net income and $5,027,000 in cash flows from operating activities [see part (a)]. The large cash flows are due primarily to the collection of receivables that were outstanding at the beginning of the year and were not associated at all with 2014 net income. 2015 was also a good year with respect to net income; the company generated income of $638,000. However, the company realized only a net cash flow from operating activities of $67,250. The low cash flows, in comparison to 2014's net cash flows, are due to (1) a larger proportion of sales being on account in 2015 and (2) customers not paying their accounts promptly. As a board member, you felt that the 2015 cash flows were insufficient to both fund future operations and pay a dividend.

P14–19 a. Original entries 1a. Cash (+A) ...................................................................................... Common Stock (+SE) ............................................................. Additional Paid-In Capital, Common Stock (+SE) .................. Issued common stock. 1b.

2.

3.

750,000 750,000

Cash (+A) ...................................................................................... Preferred Stock (+SE) ............................................................ Additional Paid-In Capital, Preferred Stock (+SE) ................. Issued preferred stock.

102,000

Fixed Assets (+A) .......................................................................... Cash (–A) ............................................................................... Purchased fixed assets.

750,000

Cash (+A) ...................................................................................... Bonds Payable (+L) ................................................................ Premium on Bonds Payable (+L) ........................................... Issued bonds.

29,200a

a b

4.

1,500,000

100,000 2,000

750,000

20,000b 9,200

$29,200 = 20 bonds  $1,000 face value per bond  146% $20,000 = 20 bonds  $1,000 face value per bond

Land (+A) ...................................................................................... Common Stock (+SE) .............................................................

40,000 15,000


Additional Paid-In Capital, Common Stock (+SE) .................. Purchased land in exchange for common stock.

25,000


P14–19 Continued 5a.

5b.

6a.

6b.

7.

8.

9a.

9b.

9c.

10.

11.

12.

Inventory (+A) .............................................................................. Accounts Payable (+L) ........................................................... Purchased inventory on account.

2,000,000

Accounts Payable (–L) .................................................................. Cash (–A) ............................................................................... Made payment to suppliers.

1,075,000

Cash (+A) ...................................................................................... Sales (R, +SE) ......................................................................... Made sales.

2,050,000

Cost of Goods Sold (E, –SE) .......................................................... Inventory (–A) ....................................................................... Recognized cost of inventory sold.

875,000

Prepaid Insurance (+A) ................................................................ Cash (–A) ............................................................................... Purchased two-year insurance policy.

80,000

Marketable Securities (+A) .......................................................... Cash (–A) ............................................................................... Purchased marketable securities.

250,000

Accounts Receivable (+A) ............................................................ Sales (R, +SE) ......................................................................... Made sales on account.

880,000

Cost of Goods Sold (E, –SE) .......................................................... Inventory (–A) ....................................................................... Recognized cost of inventory sold.

490,000

Cash (+A) ...................................................................................... Accounts Receivable (–A)...................................................... Collected cash from customers.

500,000

Miscellaneous Expenses (E, –SE) ................................................. Cash (–A) ............................................................................... Incurred and paid miscellaneous expenses.

500,000

Dividends (–SE) ............................................................................ Dividends Payable (+L) .......................................................... Declared dividends.

100,000

Interest Expense (E, –SE) ............................................................. Premium on Bonds Payable (–L) .................................................. Cash (–A) ............................................................................... Incurred and paid interest.

1,460a 140

_____________________

a b

2,000,000

1,075,000

2,050,000

875,000

80,000

250,000

880,000

490,000

500,000

500,000

100,000

$1,460 = Book value of $29,200  Effective rate per six-month period of 5% $1,600 = Face value of $20,000  Stated rate per six-month period of 8%

1,600b


P14–19 Continued Adjusting entries (a) Depreciation Expense (E, –SE) ..................................................... Accumulated Depreciation (–A) ............................................ Depreciated fixed assets. * (b)

(c)

(d)

(e)

140,000

$140,000 = ($750,000 Cost – $50,000 Salvage value) ÷ 5 year useful life

Insurance Expense (E, –SE) .......................................................... Prepaid Insurance (–A).......................................................... Expiration of a portion of insurance policy.

20,000

Unrealized Loss on Marketable Securities (Lo, –SE) .................... Allowance for Unrealized Loss on Marketable Securities (–A) ................................................... Adjusted short-term portfolio to LCM.

25,000

Miscellaneous Expenses (E, –SE) ................................................. Miscellaneous Payables (+L) ................................................. Incurred, but did not pay, miscellaneous expenses.

75,000

Bad Debt Expense (E, –SE) ........................................................... Allowance for Doubtful Accounts (–A) ................................. Estimated bad debts.

70,400*

* (f)

140,000*

20,000

25,000

75,000

70,400

$70,400 = Credit sales of $880,000  Uncollectible percentage of 8%

Loss on Inventory Write-down (Lo, –SE)...................................... Inventory (–A) ....................................................................... Adjusted inventory to LCM.

Cash B.B. (1a) (1b) (3) (6a) (9c)

0 1,500,000 102,000 29,200 2,050,000 500,000

E.B.

1,524,600

Marketable Securities

(2) (5b) (7) (8) (10) (12)

750,000 1,075,000 80,000 250,000 500,000 1,600

Accounts Receivable B.B. (9a)

0 880,000

E.B.

380,000

(9c)

5,000

Allow. for Unr. Loss on M. S.

B.B. (8)

0 250,000

B.B. (c)

0 25,000

E.B.

250,000

E.B.

25,000

Allowance for D. A.

500,000

5,000

Inventory

B.B. (e)

0 70,400

B.B. (5a)

0 2,000,000

E.B.

70,400

E.B.

630,000

(6b) (9b) (f)

875,000 490,000 5,000


P14–19 Continued Prepaid Insurance B.B. (7)

Land

0 80,000 (b)

E.B.

B.B. (4)

0 40,000

B.B. (2)

0 750,000

E.B.

40,000

E.B.

750,000

20,000

60,000

Accumulated Depreciation

Accounts Payable

B.B. (a)

0 140,000

E.B.

140,000

(5b)

Dividends Payable

E.B.

925,000

E.B.

75,000

Premium on Bonds Pay.

E.B.

100,000

E.B.

20,000

(12)

APIC: Preferred Stock

140

B.B. (3)

0 9,200

E.B.

9,060

Common Stock

B.B. (1b)

0 100,000

B.B. (1b)

0 2,000

B.B. (1a) (4)

0 750,000 15,000

E.B.

100,000

E.B.

2,000

E.B.

765,000

Retained Earnings

B.B. (1a) (4)

0 750,000 25,000

E.B.

775,000

(c4)

100,000

B.B. (c3)

E.B.

0 2,050,000 880,000

B.B. (6b) (9b)

0

E.B.

628,140

B.B. (11)

E.B.

B.B. (e)

0 875,000 490,000

B.B. (10) (d)

(c2) E.B.

0

E.B.

Depreciation Expense

0

B.B. (f)

E.B.

0

20,000

0

Unrealized loss on Mkt. Sec. B.B. (c)

(c2)

0 20,000 (c2)

E.B.

0 5,000

140,000

0

70,400

Loss on Inv. Write-down

0 140,000

575,000

Insurance Expense B.B. (b)

(c2)

0 500,000 75,000

1,365,000

0 70,400

1,460

100,000

0

Bad Debt Expense

0 1,460

0 100,000

Miscellaneous Expenses

0

Interest Expense

(c2)

0 728,140

Cost of Goods Sold

(c2)

(c2)

Dividends

(c4)

2,930,000

0

0 75,000

0 20,000

E.B.

E.B.

B.B. (d)

B.B. (3)

B.B. (6a) (9a)

B.B. (a)

0 2,000,000

0 100,000

Sales

E.B.

B.B. (5a)

B.B. (11)

APIC: Common Stock

B.B. (12)

1,075,000

Miscellaneous Payables

Bonds Payable

Preferred Stock

(c1)

Fixed Assets

0 25,000

5,000

(c2) E.B.

0

25,000


P14–19 Continued b. Closing entries (c1) Sales ............................................................................................. Income Summary .................................................................. Closed revenues into Income Summary. (c2)

(c3)

(c4)

2,930,000 2,930,000

Income Summary ......................................................................... Miscellaneous Expenses ....................................................... Cost of Goods Sold ................................................................ Interest Expense ................................................................... Bad Debt Expense ................................................................. Depreciation Expense ........................................................... Insurance Expense ................................................................ Loss on Inventory Write-down ............................................. Unrealized Loss on Marketable Securities ............................ Closed expenses into Income Summary.

2,201,860

Income Summary ......................................................................... Retained Earnings ................................................................. Closed Income Summary into Retained Earnings.

728,140

Retained Earnings ........................................................................ Dividends .............................................................................. Closed Dividends into Retained Earnings.

100,000

575,000 1,365,000 1,460 70,400 140,000 20,000 5,000 25,000

728,140

100,000

c. Mick’s Photographic Equipment Income Statement For the Year Ended December 31, 2014 Sales revenue .......................................................... Cost of goods sold ................................................... Gross profit ............................................................. Operating expenses: Depreciation expense....................................... Bad debt expense ............................................. Insurance expense............................................ Miscellaneous expenses ................................... Total operating expenses .......................... Income from operations ......................................... Interest expense...................................................... Loss on write-down of inventory ............................ Unrealized loss on marketable securities ............... Net income ..............................................................

$

2,930,000 1,365,000 $ 1,565,000

$

140,000 70,400 20,000 575,000 $

$

805,400 759,600 (1,460) (5,000) (25,000) 728,140


P14–19 Continued Mick’s Photographic Equipment Balance Sheet December 31, 2014 Assets Current assets: Cash .............................................................................................. Marketable securities (net of allowance for unrealized losses of $25,000) ................................................ Accounts receivable (net of allowance for doubtful accounts of $70,400) ............................................................. Inventory ...................................................................................... Prepaid insurance ......................................................................... Total current assets ............................................................... Land…. ............................................................................................... Fixed assets (net of accumulated depreciation of $140,000) ............. Total assets.......................................................................................... Liabilities and Stockholders' Equity Current liabilities: Accounts payable ......................................................................... Miscellaneous payables ............................................................... Dividends payable ........................................................................ Total current liabilities........................................................... Bonds payable (including associated premium of $9,060) ................. Stockholders' equity: Preferred stock ............................................................................. Common stock.............................................................................. Additional paid-in capital: Preferred stock ...................................................................... Common stock ....................................................................... Retained earnings......................................................................... Total stockholders' equity ..................................................... Total liabilities and stockholders' equity .............................................

55

$ 1,524,600 225,000 309,600 630,000 60,000 $ 2,749,200 40,000 610,000 $ 3,399,200

$

925,000 75,000 100,000 $

$

1,100,000 29,060

100,000 765,000 2,000 775,000 628,140 $

2,270,140 3,399,200


P14–19 Continued d. Direct method Mick’s Photographic Equipment Statement of Cash Flows For the Year Ended December 31, 2014 Cash flows from operating activities: Cash collections from customers ........................................ Cash paid for inventory ....................................................... Cash paid for insurance ....................................................... Cash paid for interest .......................................................... Cash paid for miscellaneous expenses ................................ Net cash increase (decrease) due to operating activities ...................................................................... Cash flows from investing activities: Purchase of marketable securities ...................................... Purchase of fixed assets ...................................................... Net cash increase (decrease) due to investing activities ...................................................... Cash flows from financing activities: Proceeds from bond issue ................................................... Proceeds from preferred stock issue .................................. Proceeds from common stock issue.................................... Net cash increase (decrease) due to financing activities ...................................................................... Net increase in cash ................................................................. Beginning cash balance, January 1, 2014 ................................. Ending cash balance, December 31, 2014................................

$

2,550,000 (1,075,000) (80,000) (1,600) (500,000) $

$

893,400

(250,000) (750,000) (1,000,000)

$

29,200 102,000 1,500,000

$

1,631,200 1,524,600

$

0 1,524,600


P14–19 Concluded Indirect method Mick’s Photographic Equipment Statement of Cash Flows For the Year Ended December 31, 2014 Cash flows from operating activities: Net income ..................................................................... Adjustments: Increase in accounts receivable ................................ Increase in inventory ................................................. Increase in prepaid insurance ................................... Decrease in premium on bonds payable ................... Increase in accounts payable .................................... Increase in miscellaneous payables .......................... Depreciation expense ................................................ Bad debt expense ...................................................... Unrealized loss on mkt. securities ............................. Total adjustments ................................................ Net cash increase (decrease) due to operating activities .................................... Cash flows from investing activities: Purchase of marketable securities ................................. Purchase of fixed assets ................................................. Net cash increase (decrease) due to investing activities ................................................. Cash flows from financing activities: Proceeds from bond issue .............................................. Proceeds from preferred stock issue ............................. Proceeds from common stock issue .............................. Net cash increase (decrease) due to financing activities ................................................. Net increase in cash .............................................................. Beginning cash balance, January 1, 2014 .............................. Ending cash balance, December 31, 2014 ............................

$ 728,140 $ (380,000) (630,000) (60,000) (140) 925,000 75,000 140,000 70,400 25,000 165,260 $

893,400

$ (250,000) (750,000) (1,000,000)

$

29,200 102,000 1,500,000

$

1,631,200 1,524,600

$

0 1,524,600


ISSUES FOR DISCUSSION ID14–1 a. Quality of earnings and earnings persistence have to do with the sustainability of the earnings as well as nearness of the earnings to the cash generated by operating activities. The indirect method of preparing the statement of cash flows is especially helpful in spotting the noncash adjustments to net income. This information helps investors identify the companies that use aggressive accounting choices, since the magnitude of such adjustments, in general, would be higher in these cases. b. In the case of Mattel, the changes in accounts receivable were certainly positive, which indicated that the majority of the sales were on credit with no contributions to cash. The change in the inventory could have been attributed to either: 1. matching the old production and currents sales or 2. purchasing new inventory on credit. Similarly, an increase in deferred taxes and translation gains would indicate no related cash effect except an adjustment to earnings due to (i.e., changes in tax rates) and foreign currency exchange rates. c.

Wall Street firms employ various models to identify over- and under-valued stocks. The choice of accounting policies by a firm is just one variable in the whole equation. Therefore, the choice of accounting policy may have some explanatory power, but alone it would not provide sufficient information to differentiate among the stocks.

ID14–2 a. The Washington Post will use the equity method to record the accounting transactions related to the other companies because it holds “significant influence” over those companies. The equity method means that the Washington Post will record its ownership percentage times the companies’ net income as equity income on the Washington Post’s income statement; if the other companies lose money, the Washington Post will show a net loss from equity investments on its income statement (its ownership percentage times the companies’ net losses). If the companies pay dividends, then the Washington Post would record this amount as an increase in cash and a reduction to the investment account. b. Losses in these investments are shown as losses on the income statement of the Washington Post. However, these are non-cash losses, so these losses are added back to the net income of the Washington Post to determine the cash changes for the year. (The treatment is similar to that of depreciation expense, another non-cash deduction to earnings.) c.

“Net of distributions” means the amount of income or loss that did not involve the exchange of cash between the Washington Post and the investment companies. The companies may have paid cash dividends to their owners, including the Washington Post, so these cash payments are deducted from the non-cash losses that are added back to net income to determine the Post’s operating cash flow.

d. Again, these transactions were of a non-cash nature. The Washington Post Company determined that the value of property, plant & equipment was permanently different from that shown on its balance sheet and recorded a loss to more correctly disclose the balance sheet value. However, this loss was not an outflow of cash, so the adjustment on the statement of cash flow was necessary to convert accrual net income into the Washington Post Company’s cash generation. The balance sheet now is more accurate in its value disclosure of the long term assets, the income statement shows a deduction in profitability due to the loss to reflect the new balance sheet value, and the statement of cash flow shows the adjustment for the conversion of net income into cash flow.

ID14–3


a. The capitalization and matching process is not very useful for assessing the cash available to a company because disbursing cash and consuming an item are two very different concepts. In order to measure performance, it is necessary to match the costs that were incurred to generate benefits against the benefits. Under accrual accounting, costs should be capitalized if they are expected to provide benefits in the future. Thus, when a company disburses cash to acquire an item expected to provide a future benefit, the company should capitalize the cost of the item as an asset. As the company uses the item over time to help generate revenue, the item will be expensed and matched against the revenue it helped generate. Thus, through the process of capitalizing costs and then expensing them when the cost helps generate revenue, revenues and expenses are closely matched. In this manner, the capitalization and matching process make it difficult to assess a company’s cash performance. b. Solvency refers to a company’s ability to pay its obligations as they come due, and earning power refers to a company’s ability to generate net assets through operating activities. The two concepts are closely related in that a company will not remain solvent if it is unable to generate net assets through operations. That is, a company cannot stay in business indefinitely if its operations fail to generate sufficient net assets to allow the company to pay back debt or pay dividends. Similarly, a company will not have very good earning power if it is not solvent. If a company is having solvency problems, it will have to divert cash away from operations and toward paying creditors. Information that is useful for assessing a company’s solvency can be found in both the balance sheet and statement of cash flows. The balance sheet lists the company’s obligations and the assets currently available to the company to use to pay off the liabilities. In addition, the balance sheet indicates which liabilities are expected to mature in the near future and which assets are more liquid. The statement of cash flows provides information about where the company is getting its cash and how it is using it. Of particular interest is the net cash generated or used by operating activities. The income statement is the primary source to assess a company’s earning power. Net income represents the net assets the company generated during the year from operations. By comparing net income to certain balance sheet amounts, such as average stockholders’ equity or average total assets, one can assess how effectively a company is using its assets to generate returns. c.

Bankruptcies usually imply that a company was unable to pay its debts. Because cash is the most common medium of exchange in the United States, creditors expect to receive interest and principal payments in cash. Thus, a wave of bankruptcies implies that companies were having cash flow problems, which, in turn, increases interest in assessing nonbankrupt companies’ cash flows to avoid other problem loans.

ID14–4


a. SuperValu paid out cash when it acquired fixed and intangible assets. Depreciation and amortization of these assets simply represent the allocation of the assets' cost to particular accounting periods. There is no cash outflow or inflow associated with depreciation and amortization. SuperValu is adding depreciation and amortization back to net income to arrive at net cash flow from operating activities because it uses the indirect method to report cash flows from operating activities. With the indirect method, a company begins with net income and then reconciles (i.e., explains) why net income is different from cash flows from operating activities. Since depreciation and amortization reduced SuperValu's net income but did not have any effect on cash, SuperValu needs to add depreciation and amortization back to net income to adjust net income to net cash flow from operating activities. b. Impairment charges are recorded when the company determines that the carrying value of an asset is greater than that asset’s fair value. If an asset has changed from the time of acquisition, or if business conditions have changed the effectiveness of that asset, the company will determine that the asset is overvalued. The charge is added back to net income for the same reason SuperValu added back depreciation and amortization. That is, recording the impairment expense reduces net income but has no effect on cash. c.

The “gain on the sale of assets” is subtracted from net earnings and “loss on sale” is added back because both of these items are non-recurring in nature and so the intent is to eliminate these from the calculation of cash from operating activities. (The cash flow related to the sale of assets belongs in the Investing section.) To reverse these out from net income, they have to be offset by the opposite of how they affect net income on the income statement. Therefore losses are added back and gains are subtracted away.

d. Receivables, in the two earliest years shown, were a use of cash for SuperValu, meaning that the change in accounts receivable had been an increase each year, which is tying up the company’s cash. More receivables (as compared to the prior year) indicate less cash flowing into the company. In the latest year, though, the receivable balance declined, indicating that more cash is flowing into the company (as SuperValu collects cash from this year’s sales, as well as last year’s.) Inventory grew in 2011, using up company cash balances, but then declined in the latest two years, freeing up cash that had formerly been tied up in groceries on the shelf. e. One of the first tipoffs that there may be some problems with the quality of earnings is if cash provided by operating activities is lower than net income. This is not the case with SuperValu and so there is no indication that earnings are being managed by the timing of transactions. SuperValu has shown significantly higher cash from operations than net income in all three years. Even if the large impairment charge in 2013 is adjusted out, earnings would have been less than cash from operations.

ID14–5 Shortcuts to “cash flow”, such as adding back depreciation and amortization (non-cash expenses) to net income, do not take into consideration all of the changes that affect cash. The indirect statement of cash flow starts by adding back these non-cash expenses, but the statement continues with the cash flow adjustments. Changes to working accounts are also included in the calculation to come up with the operating cash flow number. The most prominent example of the shortcoming of simply adding depreciation expense to earnings to calculate cash flow is the case of slowing receivables. If a company is growing rapidly because of additional credit sales, net income should also grow (holding all other factors


ID14–5 Concluded constant). However, if those credit sales were made to companies without the ability or willingness to pay, net income will overstate the company’s ultimate cash position. As receivables become stagnant and build up on the balance sheet, the statement of cash flow would deduct this increase from net income to arrive at the cash flow number. The shortcut of simply adding back depreciation to net income will not reflect the problem with collections. Not including the changes to the current accounts (such as accounts receivable) runs the risk that the reader of the financial statements will not get a true picture of the company’s cash position.

ID14–6 a. The annual depreciation expense is decreasing, which would imply that the company has less long lived assets that require annual cost allocation. The company appears to not be investing in its fixed asset base, with long term assets becoming fully depreciated and not being replaced with new assets. b. The issuance of stock to employees as a form of compensation does not require the outlay of cash. There is a theoretical cost to the company in the form of the opportunity cost of not being able to sell those shares on the open market, but the granting of shares does not directly cost the company cash. Therefore, the expense associated with issuing those shares is a non-cash charge, and similar to depreciation expense, is added back to net income in the calculation of changes in cash balances. c. Inventory decreased somewhat in 2010, giving the company over $16 million of cash. In 2011 the trend reversed, draining over $12 million of cash as the company saw increased investment in its inventories. But 2012 changed again, back to a source of cash from inventory (much larger this time). No consistent trend can be documented. d. The asset impairment charges are non-cash expenses (debit expense, credit asset to lower the carrying value of the asset and decreases earnings/equity) and, similar to depreciation expense, are added back to net income in the calculation of changes in cash balances. The treatment of inventory that is no longer fit for sale is similar; the asset is adjusted down to zero, an expense is recorded, but there is no outflow of cash (so an adjustment on the statement of cash flows is necessary).

ID14–7 The statement of cash flows for Danone lists the company’s various sources of net income with an adjustment for non-cash charges, such as depreciation expense (similar to the setup in U.S. GAAP for the indirect format). Then the statement deals with changes in operating accounts that affect operating cash flow, again in a similar approach to U.S. GAAP. After totaling operating cash flow (from income, non-cash expenses and other changes, and working capital adjustments), Danone lists its investing activities (mainly the purchase and sale of long-term assets). Finally, sources and uses of debt and equity capital are totaled in the financing section. Of note, the company lists changes to marketable securities (short term, operating investments) in its financing section. The changes in cash from the three areas are tabulated to show the overall increase/decrease to company cash balances from the previous year. Company cash management practices were very similar in 2011 to 2012. The company generated over 2.5 billion euros from operations and then spent nearly a billion euros on new fixed assets (investing activities) and still had approximately 1.5 billion euros to pay down liabilities, repurchase shares and make dividend payments to shareholders.


ID14–8 a. Over the three-year time period Eli Lilly’s general cash management profile has remained similar, but the results are trending in the wrong direction. The company generates cash from its operations (in excess of its profit levels) and uses that cash to invest in long term assets and to repay debt and return cash to shareholders. But in 2012 the cash from operations is down significantly from prior years (as is profitability), which seems to have limited the cash the company can invest in long term assets; however, the amount of cash used by financing activities jumped, draining the company cash balances by year end (a $1.9 billion decline). b. More than likely the decline in profitability affected the drop in operating cash, which in turn affected the capacity with which the company could make long-term investments for its future. The increased move to return cash to shareholders and retire debt, however, occurred in the same time period as the drop in operating cash, reducing the company’s cash position (in a time when other companies have been bulking up cash balances). To address the cash management concerns, the company might first have to address the profitability issues (as well as the policies regarding share buybacks and dividends, as well as debt repayments).

ID14–9 a. Starbucks has been incredibly consistent in its generation of cash from operating activities over the time period. It has used this cash to open additional stores (cash from investing activities) and has still had excess balances allowing it to repay debt and return cash to shareholders. b. The heavy use of cash for investing signals this profile as one of a growing company. Starbucks differs from many other growth companies in that it does not have to look to financing sources (debt and/or equity) to fund the investments; the operations are strong enough to provide the funding for the growth.

c.

At this point the company is quite strong. Fast growth can often hurt companies, but Starbucks is not leveraging up its balance sheet or diluting its existing owners to fund the growth. The consistent cash flow from operations given the economic climate of the Great Recession and the relatively soft economic recovery is quite impressive.

d. At some point, the company’s growth will slow, meaning that the use of cash for investing will not be as much a drag for the company. If operations remain strong, the company might divert the cash (that in other years would have gone to new stores) to shareholders in the form of larger dividends or increased share repurchases. On the negative side, if the operations face increased competition (from competitors such as McDonald’s, from new coffee chains or from substitution products if the coffee fad fades), cash flow could decrease and limit the company’s ability to grow and return cash to its financiers.

ID14–10 When an acquisition is recorded, any cash outflow (credit) is considered an investing activity, with the assets acquired booked (debit) onto the balance sheet. If in subsequent periods (as happened to Hewlett Packard), the value of the assets acquired is deemed to be less than the carrying value on the balance sheet, the company lessens the carrying value (credit) and books an accompanying impairment expense (debit). No cash flow is involved in the correction of the carrying amount of the acquired business, so the statement of cash flows remains the same, even though the income statement is affected (dramatically in HP’s case) by the impairment expense.


ID14–11 a. Google’s major sources of cash are cash from operations. This cash is primarily used to fund investing activities as the company purchases property, plant and equipment and acquires other companies (especially the large Motorola acquisition in 2012). In addition the company generates cash from financing activities, mainly debt issuances due to the extremely low interest rate environment in 20102012. b. Depreciation and amortization expenses and stock-based compensation expenses appear to be the largest transactions that were non-cash in nature. c.

The gain on divestitures in 2012 is shown as a negative number in the operating section because the sale of long term assets is an investing activity and the associated cash inflow is accounted for in the investing section (see “proceeds received from divestiture”). The gain increased profits and therefore needs to be backed out in the calculation of operating cash flow.

d. In each of the three years shown, accounts receivable have grown, acting as a drain (a “use”) on company cash balances. The growth in receivables can indicate that some of the company’s sales growth (29% in 2011, 32% in 2012) is coming in the form of receivables that might not be collected, or from receivables that will be very slow to be collected. It is possible that the company’s top line in sales contains receivables that will not convert in the near future to cash collections. e. Over the three-year period, Google’s incredibly strong operating cash flow has been able to fund the company’s capital spending (additions to property, plant & equipment) and acquisitions. Like other technology growth companies, Google has not used cash to pay dividends to its shareholders, as it feels it can generate superior returns by investing those amounts into the growing businesses it operates. Cash balances, therefore, are very high ($48 billion between cash and marketable securities), giving the company massive funds to build existing and new businesses. At some stage in the future, investors may demand a return of cash in the form of dividends and share buybacks, but for now all are satisfied with Google’s investment strategies for the cash.


APPENDIX A THE TIME VALUE OF MONEY EXERCISES EA–1 Time Periods (Years) Compound Interest Rates

5

10

15

5%

$150  1.27628 = $191.44

$150  1.62889 = $244.33

$150  2.07893 = $311.84

10%

$150  1.61051 = $241.58

$150  2.59374 = $389.06

$150  4.17725 = $626.59

15%

$150  2.01136 = $301.70

$150  4.04556 = $606.83

$150  8.13706 = $1,220.56

EA–2 Time Periods (Years) Compound Interest Rates

5

10

15

5%

$10,000 = $7,835.26 1.05^5

$10,000 = $6,139.15 1.05^10

$10,000 = $4,810.17 1.05^15

10%

$10,000 = $6,209.21 1.10^5

$10,000 = $3,855.44 1.10^10

$10,000 = $2,393.92 1.10^15

15%

$10,000 = $4,971.76 1.15^5

$10,000 = $2,471.85 1.15^10

$10,000 = $1,228.94 1.15^15

The above problem has also been attempted in an alternate way to demonstrate the use of formulas.

1


EA–3 Time Periods (Years) Compound Interest Rates

5

10

15

5%

$150  5.52563 = $828.84

$150  12.57789 = $1,886.68

$150  21.57856 = $3,236.78

10%

$150  6.10510 = $915.77

$150  15.93743 = $2,390.61

$150  31.77248 = $4,765.87

15%

$150  6.74238 = $1,011.36

$150  20.30372 = $3,045.56

$150  47.58041 = $7,137.06

EA–4 Time Periods (Years) Compound Interest Rates

5

10

15

5%

$150  5.80191 = $870.29

$150  13.20679 = $1,981.02

$150  22.65749 = $3,398.62

10%

$150  6.71561 = $1,007.34

$150  17.53117 = $2,629.68

$150  34.94973 = $5,242.46

15%

$150  7.75374 = $1,163.06

$150  23.34928 = $3,502.39

$150  54.71747 = $8,207.62

EA–5 Time Periods (Years) Compound Interest Rates

5

10

15

5%

$10,000  4.32948 = $43,294.80

$10,000  7.72173 = $77,217.30

$10,000  10.37966 = $103,796.60

10%

$10,000  3.79079 = $37,907.90

$10,000  6.14457 = $61,445.70

$10,000  7.60608 = $76,060.80

15%

$10,000  3.35216 = $33,521.60

$10,000  5.01877 = $50,187.70

$10,000  5.84737 = $58,473.70

2


EA–6 Time Periods (Years) Compound Interest Rates

5

10

15

5%

$10,000  4.54595 = $45,459.50

$10,000  8.10782 = $81,078.20

$10,000  10.89864 = $108,986.40

10%

$10,000  4.16987 = $41,698.70

$10,000  6.75902 = $67,590.20

$10,000  8.36669 = $83,666.90

15%

$10,000  3.85498 = $38,549.80

$10,000  5.77158 = $57,715.80

$10,000  6.72448 = $67,244.80

EA–7 a. ($50  .85734) + ($100  .68058) = $42.87 + $68.06 = $154.15 b. ($100  3.31213) = $331.21 = $385.24 c.

+ +

($100  .54027) $54.03

($60  .68058) + ($60  .63017) = $40.83 + $37.81 = $192.39

d. ($90  .58349) = $52.51 = $146.15

+ ($80  .54027) + $43.22

+ ($60  .58349) + ($60  .54027) + + $35.01 + $32.42 +

+ ($90  .54027) + ($90  .50025) + $48.62 +

($100  .46319) $46.32

$45.02

EA–8 a. ($50  .85734) + = $42.87 = $157.61

($100  .68058) + $68.06

b. ($100  3.57710) = $357.71 = $416.06

+ +

c.

+ +

($80  .58349) $46.68

($60  .73503) + ($60  .68058) + ($60  .63017) + = $44.10 + $40.83 + $37.81 = $207.78

($60  .58349) + $35.01

($100  .58349) $58.35

d. ($90  .63017) + ($90  .58349) = $56.72 + $52.51 = $157.85

+ ($90  . 54027) + $48.62

3

+ ($100  .50025) + $50.03


EA–9 a. Dollar amount

= $25,000  Future value factor for i = 10% and n = 4 = $25,000  1.46410 (from Table 1) = $36,603

Dollar amount

= $36,603  Future value factor for i = 12% and n = 3 = $36,603  1.40493 (from Table 1) = $51,425

Dollar amount

= $51,425  Future value factor for i = 15% and n = 5 = $51,425  2.01136 (from Table 1) = $103,434

b. Ben should not accept $36,000 for $25,000 at the end of 4 years. Why not? Because if he invests the initial $25,000 at 10 percent per annum compounded annually, he will have a total of $36,603, $603 more than the amount the person offered him.

EA–10 a. Dollar amount

= ($40,000  Present value factor for an ordinary annuity factor for i = 10% and n = 10) + ($500,000  Present value factor for i = 10% and n = 10) = ($40,000  6.14457 from Table 5) + ($500,000  .38554 from Table 4) = $245,782.80 + $192,770.00 = $438,552.80

b. There are two different ways to calculate the dollar amount. The two ways are shown below. Dollar amount

= ($40,000  Present value factor for an annuity due for i = 10% and n + ($500,000  Present value factor for i = 10% and n = 10) = ($40,000  6.75902 from Table 6) + ($500,000  .38554 from Table 4) = $270,360.80 + $192,770.00 = $463,130.80

Dollar amount

= $40,000 + ($40,000  Present value factor for an ordinary annuity factor for i = 10% and n = 9) + ($500,000  Present value factor for i = 10% and n = 10) = $40,000 + ($40,000  5.75902 from Table 5) + ($500,000  .38554 from Table 4) = $40,000 + $230,360.80 + $192,770.00 = $463,130.80

4

= 10)


EA–11 Option 1 Present value = = = Option 2 Present value =

$500,000  Present value factor for an ordinary annuity for i = 10% and n = 20) $500,000  8.51356 (from Table 5) $4,256,780

$4,500,000

Option 3 Present value = $1,000,000 + [($2,100,000  Present value factor for an ordinary annuity for i = 10% and n = 3)  Present value factor for i = 10% and n = 4] = $1,000,000 + [($2,100,000  2.48685 from Table 5)  .68301 from Table 4] = $1,000,000 + $3,566,941 = $4,566,941 Option 3 should be chosen because it has the highest present value. In other words, if receiving the equivalent amounts for each of the 3 payment patterns, alternative 3 would yield the largest payout today.

EA–12 Ordinary Annuity

a. $700  2.48685 (from Table 5) $700  2.73554 (from Table 6)

$1,740.80

b. $700 + ($700  1.73554 from Table 5) ($700  1.10000 from Table 1) + $700 + ($700  .90909 from Table 4)

1,914.88

c.

Annuity Due

$1,914.88

2,106.36

($700  1.10000 from Table 1) + $700 + ($700  .90909 from Table 4) $700  2.31000 (from Table 3) + $700

2,106.36 2,317.00

d. $700  3.31000 (from Table 2) $700  3.64100 (from Table 3)

2,317.00 2,548.70

e. The present value is the value of future cash flows at the current point in time. Thus, the values in Part (a) represent the present value of the two different annuities. f.

The future value is the value of future cash flows at a future point in time. Since the ends of Periods 1, 2, and 3 are all in the future, the value of the cash flows at those points in time all qualify as future values.

g. Annuity due is most valuable. The present value of annuity due is $174.08 more than the present value of ordinary annuity. In other words, if we were to receive $700 each year for the next 3 years, the payment pattern of the annuity due (payment to be received at the beginning of each year) should be

5


more preferable to us than the payment pattern of the ordinary annuity (payment to be received at the end of each year).

EA–13 a. Option 1 Present value = $240,000 Option 2 Present value = $500,000  Present value factor for i = 12% and n = 8 = $500,000  .40388 (from Table 4) = $201,940 Option 3 Present value = $600,000  Present value factor for i = 12% and n = 10 = $600,000  .32197 (from Table 4) = $193,182 Option 4 Present value = $50,000  Present value factor for an annuity due for i = 12% and n = 6 = $50,000  4.60478 (from Table 6) = $230,239 b. By computing the present value of each option's future cash flows, the cost of each option is comparable. Since Option 3 has the lowest present value, it appears to the best deal for Dunn Drafting Company. c.

Option 1: Present value = $240,000 Option 2: Present value = $500,000  Present value factor for i = 8% and n = 8 = $500,000  .54027 (from Table 4) = $270,135 Option 3: Present value = $600,000  Present value factor for i = 8% and n = 10 = $600,000  .46319 (from Table 4) = $277,914 Option 4: Present value = $50,000  Present value factor for an annuity due for i = 8% and n = 6 = $50,000  4.99271 (from Table 6) = $249,636 Option 1 now minimizes the present value of future cash flows. Thus, it appears that Option 1 is now the best option for Dunn Drafting Company.

6


EA–14 a. Since the Croziers plan to invest a lump sum today and then withdraw the money in the form of an annuity, two steps are required to determine how much the Croziers must invest today to pay for Ryan's college education. The first step is to calculate how much money they will need fifteen years from now when Ryan enters college to make the four payments at the beginning of each year Ryan is in college (i.e., the value of the annuity). The second step is to calculate how much they would have to invest now so that it will grow to the value calculated in the first step over the next fifteen years. The calculations are shown below. Present value of college expenses fifteen years in the future: Value = $40,000  Present value factor for an annuity due for i = 10% and n = 4 = $40,000  3.48685 (from Table 6) = $139,474.00 Present value of college expenses today: Value = $139,474.00  Present value factor for i = 10% and n = 15 = $139,474.00  .23939 (from Table 4) = $33,389.00 b. The present value of fourteen annual payments must equal the present value of $33,389 calculated in part (a). By using the following formula, the amount of the annual payments can be calculated. Present value = Annuity payment  Present value factor for an ordinary annuity for i =10% and n = 14 $33,389 = Annuity payment  7.36669 (from Table 5) Annuity payment = $4,532.43 c.

Current investment Present value of college expenses fifteen years in the future: Value = $40,000  Present value factor for an annuity due for i = 8% and n = 4 = $40,000  3.57710 (from Table 5) = $143,084 Present value of college expenses today: Value = $143,084  Present value factor for i = 8% and n = 15 = $143,084  .31524 (from Table 4) = $45,106 Annuity payment Present value = Annuity payment  Present value factor for an ordinary annuity for i = 8% and n = 14 $45,106 = Annuity payment  8.24424 (from Table 5) Annuity payment = $5,471.21

7


EA–15 a. ($30,000  .46319) + ($30,000  .42888) = $13,895.70 + $12,866.40 = $49,706.40

+ ($30,000  .39711) + $11,913.30

+ +

($30,000  .36770) $11,031.00

+ +

($30,000 x .46884) $14,065.20

b. $49,706.40  6.24689 = $7,956.98 c.

($30,000  .55839) + ($30,000  .52679) = $16,751.70 + $15,803.70 = $61,529.70

+ ($30,000  .49697) + $14,909.10

The yearly installment under 6% will be $61,529.70  6.8017 = $9,046.22

8


PROBLEMS PA–1 The price that Christie is willing to pay for the stock is comprised of two components: the present value of the dividends she expects to receive from holding the investment and the present value of the proceeds she will receive when she sells the investment. The total present value is calculated as follows.

Present value

= Present value of dividends + Present value of proceeds = [($5  .89286 from Table 4) + ($6  .79719 from Table 4) + ($7  .71178 from Table 4) + ($8  .63552 from Table 4)] + ($100  .63552 from Table 4) = $4.46 + $4.78 + $4.98 + $5.08 + $63.55 = $82.85

PA–2 a. Investment 1 Future value

Investment 2 Future value

= ($1,000  Future value factor for an ordinary annuity for i = 10% and n = 5)  Future value factor for i = 12% and n = 5 = ($1,000  6.10510 from Table 2)  1.76234 from Table 1 = $10,759.26 = $3,000  Future value factor for an ordinary annuity for i = 15% and n = 7 = $3,000  11.06680 from Table 2 = $33,200.40

Therefore, Wharton's total investment at the end of ten years will equal $43,959.66. b. Current investment

= = =

Future value  Present value factor for i = 12% and n = 10 $43,959.66  .32197 from Table 4 $14,153.69

Therefore, Wharton would have to invest $14,153.69 for ten years earning 12% compounded annually to have an amount equivalent to the two investments.

PA–3 a. Contract 1 Present value

Contract 2 Present value

Contract 3 Present value

= $8,000  Present value factor for an annuity due for i = 6% and n = 10 = $8,000  7.80169 (from Table 6) = $62,413.52 = $8,000 + ($20,000  Present value factor for i = 12% and n = 10) = $8,000 + ($20,000  .32197 from Table 4) = $14,439.40 = ($8,000  Present value factor for an ordinary annuity for i = 10% and n = 3)  Present value factor for i = 10% and n = 3 = ($8,000  2.48685 from Table 5)  .75131 from Table 4 9


= $14,947.16

10


PA–3 Concluded b. (1) Equivalent values at the end of Year 5: Contract 1 Present value

= ($8,000  Future value factor for an annuity due for i = 6% and n = 5) ($8,000  Present value factor for an annuity due for i = 6% and n = 5 = ($8,000  5.97532 from Table 3) + ($8,000  4.46511 from Table 6) = $47,802.56 + $35,720.88 = $83,523.44

+

Proof: $83,523.44  .74726 = $62,413 = Present value of Contract 1 in Part (a) Contract 2 Present value

= ($8,000  Future value factor for i = 12% and n = 5) + ($20,000  Present value factor for i = 12% and n = 5) = ($8,000  1.76234 from Table 1) + ($20,000  .56743 from Table 4) = $14,098.72 + $11,348.60 = $25,447.32

Proof: $25,447.32  .56743 = $14,439 = Present value of Contract 2 in Part (a) Contract 3 Present value

= ($8,000  Future value factor for i = 10% and n = 1) + $8,000 + ($8,000  Present value factor for i = 10% and n = 1) = ($8,000  1.10000 from Table 1) + $8,000 + ($8,000  .90909 from Table 4) = $24,072.72

Proof: $24,072.72  .62092 = $14,947 = Present value of Contract 3 in Part (a) (2)

Equivalent values at the end of Year 10:

Contract 1 Present value

= $8,000  Future value factor for an annuity due for i = 6% and n = 10 = $8,000  13.97164 from Table 3 = $111,773.12

Proof: $111,773.12  .55839 = $62,413 = Present value of Contract 1 in Part (a) Contract 2 Present value

= = = =

($8,000  Future value factor for i = 12% and n = 10) + $20,000 ($8,000  3.10585 from Table 1) + $20,000 $24,846.80 + $20,000.00 $44,846.80

Proof: $44,846.80  .32197 = $14,439 = Present value of Contract 2 in Part (a) Contract 3 Present value

= ($8,000  Future value factor for an ordinary annuity for i = 10% and  Future value factor for i = 10% and n = 4 = ($8,000  3.31000 from Table 2)  1.46410 from Table 1 = $38,769.37 11

n = 3)


Proof: $38,769.37  .38554 = $14,947 = Present value of Contract 3 in Part (a)

12


PA–4 Option 1 Present value Option 2 Present value

Option 3 Present value

= $25,000 = $60,000  Present value factor for i = 9% and n = 8 = $60,000  .50187 (from Table 4) = $30,112.20 = $5,000 + ($27,000  Present value factor for i = 9% and n = 3) + ($20,000  Present value factor for i = 9% and n = 20) = $5,000 + ($27,000  .77218 from Table 4) + ($20,000  .17843 from Table 4) = $5,000 + $20,848.86 + $3,568.60 = $29,417.46

Hartney should accept bonus option 2 because it has the highest present value. In other words, in terms of today’s dollars, bonus option #2 gives Hartney the most amount of money.

PA–5 a. Value

= = = =

b. Value

=

= = = c.

Value

=

= = = d. Value

= = = =

$5,000 + ($10,000  Present value factor for an ordinary annuity for i = 10% and n = 5) + ($15,000  Present value factor for i = 10% and n = 5) $5,000 + ($10,000  3.79079 from Table 5) + ($15,000  .62092 from Table 4) $5,000 + $37,908 + $9,314 $52,222 ($5,000  Future value factor for i = 10% and n = 2) + ($10,000  Future value factor for i = 10% and n = 1) + $10,000 + ($10,000  Present value factor for an ordinary annuity for i = 10% and n = 3) + ($15,000  Present value factor for i = 10% and n = 3) ($5,000  1.21000 from Table 1) + ($10,000  1.10000 from Table 1) + $10,000 + ($10,000  2.48685 from Table 5) + ($15,000  .75131 from Table 4) $6,050 + $11,000 + $10,000 + $24,869 + $11,270 $63,189 ($5,000  Future value factor for i = 10% and n = 4) + ($10,000  Future value factor for an ordinary annuity for i = 10% and n = 4) + [($10,000 + $15,000)  Present value factor for i = 10% and n = 1)] ($5,000  1.46410 from Table 1) + ($10,000  4.64100 from Table 2) + ($25,000  .90909 from Table 4) $7,321 + $46,410 + $22,727 $76,458 ($5,000  Future value factor for i = 10% and n = 5) + ($10,000  Future value factor for an ordinary annuity for i = 10% and n = 5) + $15,000 ($5,000  1.61051 from Table 1) + ($10,000  6.10510 from Table 2) + $15,000 $8,053 + $61,051 + $15,000 $84,104

13


PA–5 Concluded Proof: Value of each equivalent value today Option 1 1. 2. 3. 4.

$52,222  1.00000 $63,189  0.82645 $76,458  0.68301 $84,104  0.62092

Option 2

Option 3

$52,222 $52,222 $52,222 $52,222

PA–6 Present values a. Value = $10,000 b. Value

= = =

$2,000  Present value factor for an ordinary annuity for i = 8% and n = 8 $2,000  5.74664 from Table 5 $11,493.28

c.

Value

= = =

$5,000  Present value factor for an annuity due for i = 8% and n = 3 $5,000  2.78326 from Table 6 $13,916.30

d. Value

= = =

$3,000  Present value factor for an ordinary annuity for i = 8% and n = 5 $3,000  3.99271 from Table 5 $11,978.13

e. Value

= = =

$25,000  Present value factor for i = 8% and n = 7 $25,000  .58349 from Table 4 $14,587.25

f.

Value

= = =

$3,000  Present value factor for an ordinary annuity for i = 8% and n = 2 $3,000  1.78326 from Table 5 $5,349.78

g. Value

= = =

$4,000  Present value factor for an annuity due for i = 8% and n = 3 $4,000  2.78326 from Table 6 $11,133.04

Future values a. Value = = =

$10,000  Future value factor for i = 8% and n = 4 $10,000  1.36049 from Table 1 $13,604.90

b. Value

= = =

$2,000  Future value factor for an ordinary annuity for i = 8% and n = 8 $2,000  10.63663 from Table 2 $21,273.26

c.

= = =

$5,000  Future value factor for an annuity due for i = 8% and n = 3 $5,000  3.50611 from Table 3 $17,530.55

Value

Option 4

14


PA–6 Concluded

= =

($3,000  Future value factor for an ordinary annuity for i = 8% and n = 5)  Future value factor for i = 8% and n = 5 ($3,000  5.86660 from Table 2)  1.46933 from Table 1 $25,859.91

e. Value

=

$25,000

f.

= = =

$3,000  Future value factor for an ordinary annuity for i = 8% and n = 2 $3,000  2.08000 from Table 2 $6,240.00

d. Value

Value

=

g. Value = $4,000  Future value factor for an annuity due for i = 8% and n = 3 = $4,000  3.50611 from Table 3 = $14,024.44

PA–7 a. To determine whether the offer of $110,000 today is a good deal, the future cash flows must be converted into equivalent values in present dollars (i.e., present values). The contract specifies two types of future cash flows: $2,000 at the beginning of each year for ten years and a lump-sum receipt of $300,000 in ten years. The present value of the two types of cash flows are calculated below. (1) Present value of annual receipts: Value = $2,000  Present value factor for an annuity due for i = 10% and n = 10 = $2,000  6.75902 (from Table 6) = $13,518.04 (2) Present value of lump-sum receipt: Value = $300,000  Present value factor for i =10% and n = 10 = $300,000  .38554 (from Table 4) = $115,662.00 (3) Total present value: Value = $13,518.04 + $115,662.00 = $129,180.04 Since the present value of the future cash flows exceeds $110,000, it would not be wise for Joy Don Corp. to accept $110,000 in cash today in place of the note. By accepting the cash of $110,000 now, Joy would be worse off by more than $19,000. b. As the discount rate increases, the present value of future cash flows decreases. Since the present value of the future cash flows discounted at 10% exceeds $110,000, the discount rate at which Joy Don would be wise to accept $110,000 in cash instead of the note must be greater than 10%. Try i = 12%. Present value

= ($2,000  6.32825 from Table 6) + ($300,000  .32197 from Table 4) = $12,656.50 + $96,591.00 = $109,247.50

15


With a discount rate of 12%, the present value of the future cash flows is slightly less than $110,000, which implies that Joy Don would be better off accepting $110,000 in cash today rather than accepting the note.

16


APPENDIX B QUALITY OF EARNINGS CASES: A COMPREHENSIVE REVIEW CASE 1: LIBERTY MANUFACTURING The analysis that can be done on Liberty is limited because there is no access to macroeconomic information, industry data, or the company’s financial statements from the past several years. The following comments are based solely on the information provided and must be viewed as limited for that reason.

Ratio Analysis at Face Value Liberty’s reported profits of $159,000 appear to be relatively high. Return on equity is 20 percent ($159,000/$799,000), return on assets is approximately 6.6 percent ($159,000/ $2,399,000), return on sales is 14.5% ($159,000/$1,100,000). [Note: ratios are calculated without balance sheet averages so that two years can be analyzed.] The company turns over its inventory, on average, about every 90 days (4 times per year). Moreover, the significant decrease in the inventory balance probably increased the company’s turnover from the previous year. These measures suggest that Liberty has relatively strong earning power. There is also preliminary evidence that Liberty’s solvency position is satisfactory. The company’s current ratio increased slightly from 2.5 to 2.6, and the quick ratio increased to a strong 2.2. There is some concern, however, about Liberty’s accounts receivable management. Average days receivable is approximately 141, indicating that the company turns over its receivables only 2.59 times per year. This relatively slow turnover will slow down cash collections and may reduce the company’s solvency position. The debt/equity (total liabilities/stockholders’ equity) ratio represents another source of concern, because it indicates that the company is carrying a much higher percentage of liabilities as of the end of the second year. Specifically, the ratio increased from 1.24 to 2.00. The increase was caused primarily by increases in the outstanding accounts payable balance and a $600,000 increase in long-term liabilities. However, the interest coverage ratio is 3.2 [($199,000+$90,000)/$90,000], suggesting that the company’s profits were more than enough to cover the interest charges on its long-term debt. In summary, the ratio analysis conducted above suggests that Liberty’s earning power and solvency positions are reasonable. Significant debt has been added to its capital structure recently, but the company’s earnings power appears to be sufficient to cover the necessary debt payments if receivables are collected on a timely basis.

Earnings Persistence A closer look at the income statement reveals some very troubling aspects about Liberty. First, many of the “gains” reported on the income statement are not essential to the operations of the business. A realized gain on the sale of short-term investments accounted for $80,000 of the net profit; $40,000 was generated from Packer, a company whose stock price fell from $5.00 to $4.00 as of the end of the year; $25,000 came from a translation gain because the dollar fell relative to the Swiss franc. The exchange rate between Swiss francs and dollars fell from 2:1 to 1.6:1 [200,000/($100,000 + $25,000)] as of year-end. Book gains recognized on the sale of land and the plant accounted for an additional $14,000. Without these items 1


Liberty’s operating income before taxes is an unimpressive $20,000, only 3.6 percent of sales. Most of these items cannot be counted on to continue in the future, and only a small portion of Liberty’s earning power can be considered persistent.

Quality of Earnings The quality of Liberty’s earning number is also questionable. The allowance for uncollectibles account dropped dramatically as a percentage of outstanding accounts receivable (from 6.7 percent to only 1.8 percent). This drop suggests Liberty’s management is protecting the income statement from bad debt expense. Second, with respect to short-term investment activity, it appears as if Liberty sold its shares of Fredericks in order to report a $80,000 realized gain on the income statement (the unrealized gains go to the balance sheet rather than the income statement). This is evident since it repurchased the shares within 4 days of the sale of the original shares. Third, Liberty reduced its inventory significantly between the years. Reviewing the inventory footnote shows that Liberty reduced its LIFO layers in year two. This had the impact of increasing income by $210,000. This is calculated by taking the current cost per unit ($16) and subtracting the inventory carrying cost for the items that were sold out of inventory. During the year Liberty sold 50,000 units and purchased 25,000 units during the year. Therefore, 25,000 units were sold out of inventory. These 25,000 units had a carrying value as follows: 10,000 units @ $12 each, 10,000 units @ $6 each, and 5,000 units @ $2 each. Since the replacement cost is higher than the carrying cost of these units, a one-time increase in income is created. The total amount of this increase was $210,000 ([($16$12)*10,000] +[ ($16-$6)*10,000] + [($16-$2)*5,000]).

Recommendation Based on this analysis, I would not recommend that the applicant be considered further for a loan. Its solvency position is weak, and its reported earnings contain a number of items that are not expected to persist in the future. In addition, the analysis suggests that the quality of earnings is low and can’t be relied upon.


CASE 2: MICROLINE CORPORATION MEMO To: Sharon Sonneborn Re: Evaluation of Microline Corporation Microline is a potential acquisition candidate for Mega Industries. As such, Mega Industries requires an assessment of Microline’s financial condition and performance. I have prepared an evaluation of Microline’s solvency, earnings power, and earnings quality. The amounts in parentheses are stated in thousands of dollars. Microline’s reported profits of $3.25 million appear to be high, particularly in comparison with the prior year’s results. Profits in the second year are over three times the level of year-one profits. Return on equity has increased from 4% ($900/$22,500) to almost 14% ($3,250/$23,750). Profits have also grown faster than assets, as the return on assets has increased to 6% ($3,250/$51,250) from only 2% ($900/$42,500). While sales are increasing, profits are increasing even faster. The return on sales has increased from 2% ($900/$52,500) to 5% ($3,250/$60,000). Finally, inventory is turning over about 5 2/3 times ($34,000/avg. inventory of $6,000) a year or every 64 days. This is important given that Microline’s inventory may be subject to obsolescence as suggested by its prior year inventory write-down. These results indicate that Microline’s earnings power is strong. However, without industry data or data on Microline’s past financial statements, the assessment may be limited. With respect to solvency, Microline also appears to be strong. Microline’s current ratio of 1.27 ($16,500/$13,000) increased from 0.94 ($11,750/$12,500). The quick ratio also increased significantly from 0.46 to 0.72. The increases in these ratios are due to an increase in current assets rather than a decrease in current liabilities. One reason for the improvement in the current ratio is that Microline issued a long-term note that contains a debt covenant requiring a current ratio greater than one. The increases in the receivables, inventory, and short-term investments may all have been done in order to improve this ratio and meet the current ratio requirement. The result, however, may be overstated receivables. Receivables may not be turning over as quickly as they could be. While still a strong 11.9 ($60,000/average gross receivables of $5,050) or about every 31 days, the receivables turnover has likely decreased as a result of the current ratio requirement. Due to the issuance of additional notes totaling $7,000, Microline’s total liabilities to equity ratio increased from 0.89 ($20,000/$22,500) to 1.16 ($27,500/$23,750). This means that Microline is financed with slightly more debt than equity. Based on its reported profits, the interest coverage ratio is strong at 5.75 ($4,750 + $1,000/$1,000), up from 3.8 ($1,700 + $600/$600). However, since the two new notes equal to $7,000 were issued, the year’s interest expense is not reflective of expected interest expense in the future. If both notes are at an 8% rate, interest expense will be almost $500 higher in the next year. Without additional income, the interest coverage ratio will decrease significantly and possibly indicate solvency problems. Although the earnings power and solvency indicators suggest that Microline is financially strong and has improved significantly, several issues suggest that its earnings may not be expected to persist in the future and that its earnings power is artificially high. First, Microline’s latest profits have been increased by the sale of land and certain short-term investments. For example, Microline sold land for a $1.5 million gain and immediately repurchased a similar parcel of land. Another $500,000 of income was generated by unrealized and realized gains on its short-term investments. Without these gains, Microline’s net income would have been only $1.25 million or only slightly greater than its prior year profits. Second, its investment in Ellery, Inc. produced income of $1.25 million. Finally, a favorable fluctuation in the Canadian dollar produced a


gain of $500,000. None of these items represent income from Microline’s normal operations. Without all of them, Microline would have had $3.75 million less in income and would have, in fact, reported a loss for the year. In addition to poor earnings persistence, earnings quality also may be poor for several reasons. First, the sales on the land and short-term investments appear to have been executed solely for the gains achieved. Immediately after the sale of the land, similar land was purchased back. In essence, Microline transferred excess cash into gains on the income statement. In addition to a stronger income statement, these sales may have been motivated by the bonuses Microline’s management receives if return on equity exceeds 10%. Without these gains on the sale of land and investments, reported income would only be about $1.25 million or 5% of equity. The prior year results indicate no gains or losses from sales of land, equipment, or investments. Clearly, management entered into these transactions for their financial statement effects, as they were not in the shareholders’ best interest. Second, although Microline’s sales have increased 14% and its bad debts allowance has increased 14%, consistent with its policy, the increase in receivables suggests that it may have more bad debts than allowed for. Finally, selling and administrative expenses have increased 30% while sales have only increased 14%. This suggests that some expenses or losses incurred may be hidden in this line item (for example, the bonus earned by management in the prior year). Overall, Microline appears to be strong. However, closer examination reveals that Microline’s own operations contribute little to its net income. A limited portion of the latest year net income is expected to persist. In addition, a number of actions by Microline suggest that the earnings quality is poor.


CASE 3: TECHNIC ENTERPRISES AND SONAR-SUN, INC. MEMO To: Recommendation Team Re: Analysis of Technic Enterprises and Sonar-Sun, Inc. To support Timken Brother’s buy/sell recommendation with respect to Technic Enterprises and Sonar-Sun, I have analyzed their financial statements along three dimensions—solvency, earning power and persistence, and earnings quality. Solvency and Liquidity Position Technic Enterprises Year 2 Year 1 Current ratio Quick ratio Liabilities/equity Receivables turnover

2.14 1.92 2.19 2.44

2.50 1.64 1.26

Sonar-Sun, Inc. Year 2 Year 1 1.04 0.39 1.82 9.39

1.16 0.36 1.59

Technic Enterprises is experiencing decreasing liquidity and solvency. Its current ratio fell between the two years, while the quick ratio increased. Current liabilities almost doubled at the same time inventory balances fell, creating the fall in the current ratio. Increases in current assets other than inventory, however, have helped Technic avoid a dangerously low current ratio. At 2.44, Technic’s receivables turnover is fair. For improved liquidity, receivables should be turning over more quickly than every 150 days. Recently, however, the receivables balance has grown while its bad debts allowance has not, suggesting that part of the receivables balance may be overstated. Another indicator of Technic’s declining solvency is its increasing liabilities to equity ratio. Even excluding current liabilities, the debt to equity ratio is a high 1.54. This means more debt finances Technic than equity. Part of the increase is due to the issuance of a new note for $18 million without sufficient increases in income or equity issuances. Sonar-Sun, Inc. is also experiencing a decreasing current ratio. However, unlike Technic’s, Sonar-Sun’s ratio is dangerously low. At the end of Year 1, its current ratio was only 1.16. By the end of Year 2, it had fallen even further to 1.04. Given that most of Sonar-Sun’s current assets are in the form of inventory, it is not surprising that the quick ratio is also extremely low, at 0.39 in the latest year. These ratios are extremely weak and suggest that Sonar-Sun may soon face a liquidity crisis. Current liabilities have increased almost 50% while current assets have increased by only 30%. Despite additional common stock issued of $5,000,000, debt increased when a $5,000,000 note was issued to buy Wallingford Atlantic. This has only further weakened an already poor solvency position. Unlike Technic, however, Sonar-Sun does maintain a very strong receivables turnover ratio of 9.39, or turns over every 38 days. While Sonar-Sun does not separately report on its allowance accounts, there are no signs that it is experiencing collection problems. While Technic paid almost 50% of its latest year net income in the form of a cash dividend to its investors, Sonar-Sun paid almost 100% of its net income as cash dividends. Given their weak solvency positions, neither payment appears to be prudent. Summary: Given its declining liquidity and solvency and potential additional liabilities, based on a scale of 1 (very weak) to 10 (very strong), I rate Technic’s solvency position a 4. Technic’s liquidity position is strong;


however, it appears to be increasing its debt, spending its cash on investments not immediately necessary for improving its financial position, and not significantly extinguishing its current obligations. Since it is not yet in default, I did not believe a lower rating appropriate. Also, since its receivables turnover is fair, obtaining cash from its operations does not seem to be a problem yet, but could be if improvements are not made. In contrast to Technic, Sonar-Sun has an even weaker liquidity position. Sonar-Sun has a balance sheet consisting mainly of inventory and debt. While its solvency is not as weak as Technic’s, it is nevertheless weak. Sonar’s choice to pay a large cash dividend rather than improve its solvency position is also not a favorable indicator. Its strong receivables turnover ratio, however, suggests it has the ability to generate cash more quickly than Technic. Sonar needs to improve its liquidity position and work to reduce its debts. Based on these considerations, I give Sonar a 3 on liquidity and solvency. It is lower than the rating for Technic because Sonar is in more immediate liquidity danger. Earning Power and Persistence

ROA ROE 0.25 Return on Sales Times interest earned Inventory turnover

Technic Enterprises

Sonar-Sun, Inc.

0.09 0.36 0.17 2.53 3.37

0.13 0.12 5.64 1.75

Technic Enterprises performed well relative to its level of assets, equity, and sales. Each of these ratios suggests Technic earns profits efficiently. Further, at 3.37 or every 108 days, its inventory turnover is good. Further, the reduction in inventory has helped Technic achieve a good inventory turnover ratio. However, given the nature of its inventory and a recent history of a write-down, Technic would face less risk of obsolescence if it turned over its inventory even faster. Technic’s times-interest-earned ratio is fair and should improve as Technic retires its debt. Based on these indicators, it appears that Technic’s earnings power is strong. However, closer examination of the income statement and notes suggests otherwise. First, net income benefited by a $750,000 income effect from a change in the inventory costing method. Second, examination of miscellaneous gains and losses reveals various gains due to sales of investments, buildings, and land. Together the gains from these realized and unrealized gains amounted to $4,140 or 87% of the net income before the accounting change. In addition, a favorable translation gain added another $750,000 to income. Together these items account for all of Technic’s earnings. There is reason to be concerned because none of these items is essential to Technic’s operations or can be expected to persist in the future. Accounting methods cannot be changed annually nor can sales of assets occur frequently. Finally, the translation gain may easily turn into losses next year. Given that the net income would have decreased to almost nothing without these items unrelated to the normal business, earning power is extremely poor. Clearly, without the income from these other activities, Technic may have trouble meeting its obligations. Similar to Technic, the profitability ratios suggest that Sonar-Sun has a good level of earning power. All three earning power ratios (ROA, ROE, and ROS) are strong. In addition, the times-interest-earned ratio suggests Sonar is financially strong and is utilizing its assets effectively. The only weakness is in Sonar’s inventory turnover ratio. At 1.75, it turns over its inventory only every 209 days. This low rate of turnover risks Sonar for significant inventory obsolescence. Given its weak solvency position, the ability to generate sales quickly hurts Sonar’s financial position. Further, this long period is resulting in inventory obsolescence and unnecessary write-off losses, such as the $5,000,000 reduction. In contrast to Technic’s foreign currency translation gain on foreign currency, Sonar-Sun’s net income includes a loss of $1,500,000 due to


the unfavorable translation of the peso. With the exception of the write-downs of inventory and equipment equal to $5,000,000, which are part of normal operations, most of Sonar’s income of $20,700,000 should persist in the future. Only $4,000,000 of the $20,700,000 was earned by EDM Suppliers, an affiliate not under control of Sonar. Note that unlike Technic, Sonar does not have other revenue and expense items hidden in a miscellaneous income statement items. Compared with Technic, operating expenses for Sonar appear to be larger (45% of sales versus 24% of sales). Disaggregation of the general operating expenses may provide information about the content of these expenses. This would help users decide whether these expenses are expected to persist in the future or not. Finally, Sonar’s equity investments and affiliate investment appear to be performing well. Sonar earned $4,000,000 on its $12,000,000 affiliate investment (beginning balance) in EDM Suppliers. Technic’s investment in Lehmon Financial Services, however, has not done as well. After purchasing the shares at $300 each, Lehmon is now selling for $25.50 per share. This investment returns only $1,200 out of a $15,000 investment. Given liquidity and solvency problems, this investment may not have been good use of its assets. Summary: Given the portion of income generated from activities outside its normal operations, Technic does not appear to have good earnings power or persistence of positive income. Based on these factors, I rate Technic a 3 on earning power and persistence. In contrast, Sonar-Sun earned positive income mostly due to its own primary operating activities. Also, some of its losses or expenses are not expected to persist in the future. For these reasons, Sonar-Sun earns a 7. Earnings Quality Technic Enterprises has presented its income statement and made decisions that favor the presentation of its performance. First, hidden within miscellaneous gains and losses is an inventory write-down of $450,000. This is an ordinary part of business and should be reported in the operations section. Further, a significant number of other gains are included in this miscellaneous category that are not an ordinary part of business nor are expected to persist in the future. For example, the sale of land appears to have been motivated in order to recognize a gain. Immediately after the sales, Technic repurchases 7 or the 10 parcels sold. Similar activity occurred with respect to its building and short-term investment. Second, operating expenses are a significant $7,200,000, but are not described further. Clearly, this expense may contain some questionable items. Third, Technic has adopted a change in an accounting method that results in a favorable credit to its income. Fourth, depreciation policies are liberal up to 40 years in order to reduce depreciation expense each year. Fifth, allowances for uncollectible accounts may not be sufficient. The reported allowance decreased in half even though the receivables balance almost doubled. Unlike Technic, Sonar’s accelerated depreciation methods result in an earnings amount that reflects expenses in the most conservative way. Neither Technic nor Sonar appear to manage their short-term investment portfolios for reporting purposes. Further, Technic uses FIFO inventory cost flow assumption, while Sonar uses a LIFO assumption. The result is lower income for Sonar. Also, Sonar reports a bad debt allowance of 4.4%, compared with only 2% for Technic, again suggesting that the quality of Sonar’s reported income is excellent and that of Technic is poor. Summary: Sonar-Sun’s earnings quality is not overly inflated by items at the discretion of management. Estimated expenses do not appear to be understated and appear to be of good quality, earning Sonar-Sun a rating of 8. Technic has significant income statement items that suggest reported income is artificially high. For this reason, I rate Technic’s earning quality a 5.


CASE 4: AVERY CORPORATION Evaluation of the CEO’s letter Immediately below is a chart containing financial ratios computed on the financial statements of Avery Corporation. The following discussion assesses earning power and solvency and, in so doing, addresses the specific questions posed by Sellers. The solution concludes by summarizing the evaluation of the CEO’s letter to the shareholders. Financial Ratios for Avery Corporation Return on equity Return on assets Earnings per share Return on sales Times interest earned Current ratio Quick ratio Receivables turnover (days) Receivables turnover (times) Inventory turnover (days) Inventory turnover (times) Debt/equity Financial leverage

Year 3 .063 .037 1.90 .056 2.76 3.79 1.95 110 3.32 302 1.21 .66 .026

Year 2 .078 .040 3.75 .046 1.95 6.22 4.19 89 4.12 228 1.60 .73 .038

Year 1

4.23 2.04

1.58

Earning Power Assessment Although reported net income appears to have increased ($5,626 to $7,333), earning power dropped off substantially. First, three return ratios (ROE, ROA, and EPS) using reported net income show a decrease of earning power since Year 2. All three have dipped, and EPS has decreased by almost 50 percent. (The EPS decrease is due primarily to the addition of 2.35 million outstanding shares—1 million for exercised stock options, 1 million for the acquisition of Buckeye, and .35 million for the stock dividend). Second, return on sales and times interest earned have shown considerable increases. However, there is reason to be concerned because the increase in both ratios occurred primarily because the latest net income includes a gain on the sale of equipment ($6 million) and special service revenue ($8 million), neither of which are essential to the operations of Avery and can be expected to persist in the future. (The $6 million gain on the sale of equipment involved equipment with a cost of $20 million and accumulated depreciation of $6 million being sold for $20 million in cash.) Moreover, inventory turnover slowed to only 1.21 times per year, indicating that there are a number of slowmoving inventory items, and additional inventory write-offs may be necessary in the future. Proper income statement disclosure, which is not followed by Avery, should highlight net income from operations, excluding the inventory write-down, income from the affiliate, the gain on the sale of equipment, and special service revenue, for both years. Avery’s net income from operations (before interest and taxes) would then be $6 million ($21.8 million – $1.8 million – $6 million – $8 million) and $11.55 million ($13.05 million – $1.5 million), respectively. It seems, therefore, that “persistent earnings” decreased considerably, and in that time period the company issued 2.35 million common shares, further diluting its outstanding common stock. This disclosure format would also clearly disclose that the company’s operating sales actually decreased from $120 million to $115 million. Moreover, the $5,000 inventory write-off, claimed to be “extraordinary” by Tennenden, would not normally be


considered as such under generally accepted accounting principles. Rather, it is a normal and recurring risk of business operations.

Solvency Assessment The solvency, activity, and capitalization ratios point to a potential solvency problem. The current and quick ratios have decreased from the first years shown, receivables turnover has slowed (suggesting that there may be collection problems), the company has taken on additional debt, and financial leverage has decreased even though reported earnings have increased. In addition, if the nonrecurring items are eliminated from reported net income, Avery’s times-interest-earned ratio plummets. It appears that profits are not sufficient to cover the company’s annual interest payments, despite Tennenden’s claim that “our profits are more than adequate to cover debt interest payments.” A closer look at Avery’s receivable and payable activity indicates that cash collections from receivables totaled $105 million ($35 million + $115 million – $45 million), while sales totaled $115 million. This combination increased net accounts receivable by $9.95 million for the second year in a row. Cash payments for accounts payable totaled $67.7 million ($8.2 million + $75.5 million – $16 million), while purchases totaled $75.5 million. This combination increased payables by $7.8 million. Thus, receivables appear to be growing at a faster pace than are payables, indicating possible collection problems and potential cash flow problems in the future. Another interesting aspect about receivables is that the allowance for uncollectibles only increased by 9 percent, while outstanding accounts receivables increased by almost 30 percent. It is possible that the $500,000 bad debt expense reported on Avery’s income statement is not sufficient to cover the company’s uncollectibles and adding to this allowance would further reduce Avery’s reported net income. Avery also appeared to cut back on its pension contribution. While an accrued pension expense of $10 million was recognized, the company only paid $8 million to the fund. This combination increased the company’s unfunded pension liability to $7.339 million, an increase of 37 percent. Such a cutback may also indicate potential solvency problems. The issuance of a common stock dividend is also troublesome. It appears to have been issued in lieu of a common stock cash dividend. The 10 percent dividend totaled a common stock issuance of 350,000 shares because 3.5 million common shares (5 million – 2 million – 1.5 million + 1 million + 1 million) were outstanding at the time of the declaration. The issuance had no effect on the assets or liabilities of the company, and it had essentially no effect on the wealth levels of the shareholders. It may have been a ploy by management to encourage the shareholders to believe that they were actually receiving something of value. It seems that the acquisition of Buckeye Corporation was a very risky transaction for Avery. The company paid $50 million in cash and issued 1 million common (treasury) shares with a total value of $15 million. Goodwill of $15 million was recognized on the transaction, indicating that the market value of Buckeye’s property, plant, and equipment was $50 million. This transaction drained the company of much of its cash and further diluted the common share value. Perhaps management believes that Buckeye can generate future profits capable of rescuing what appears to be a company with serious financial difficulties.

Returning to Tennenden’s letter to the shareholders and summarizing: 1. In the latest year Avery did not demonstrate strong earning power. 2. The inventory write-off is not considered “extraordinary.”


3. The acquisition of Buckeye was financed primarily by a preferred stock (debt?) issuance and the issuance of 1 million shares of common stock, not cash from operations. 4. The shareholders have not prospered and, in fact, received only a stock dividend in the last year. 5. The company’s debt/equity ratio is grossly understated. 6. Persistent earnings are not sufficient to cover the company’s annual interest payments.

Questions 1. Net income increased from $5,626,000 to $7,333,000 while EPS decreased from $3.75 to $1.90. Even without the extraordinary loss of $1.30, EPS from continuing operations was only $3.20, still less than EPS in the prior year. The reason for the drop is that the number of shares outstanding increased. Avery executives exercised options for 1,000,000 shares. Also, Avery purchased Buckeye Corporation for cash and 1,000,000 shares of stock. Finally, Avery declared and issued a 10% stock dividend on the outstanding shares. Between the two years, the outstanding shares increased by 2,350,000. Number of Outstanding Shares Balance, Beginning Less: Treasury shares—note 9 Treasury shares—note 9 Balance Options exercised—note 9 Buckeye acquisition—note 4 10% Stock dividend Balance, Ending

5,000,000 (2,000,000) (1,500,000)a 1,500,000 1,000,000 1,000,000 3,500,000 350,000 3,850,000

a $18,000,000/$12 share = 1,500,000 shares

2. As outlined above, the 10% stock dividend resulted in the issuance of 350,000 shares. Stock dividends do not affect assets or liabilities. This stock dividend resulted in a decrease to retained earnings of $5,250,000 and an increase in contributed capital of $5,250,000. Thus, total stockholder’s equity is also unchanged. 3. The company issued $5,000,000 in bonds for $4,385,500 resulting in an effective market yield of 10%. The call feature of the bond allows the bonds to be called beginning in the fourth year for 2% above the face value or $5,100,000 (note 6). With a current market rate of 8%, the bonds have a market value of $5,000,000 (par value, since the stated rate is also 8%). Therefore, since Avery can buy its own bonds back in the open market for $5,000,000, it is not wise to exercise the call option for $5,100,000. Also, since the company may be facing solvency difficulties, it may be unwise to use its cash for this purpose. 4. Based on Avery’s latest consolidated balance sheet, Avery has total liabilities of $85,626,000 and total equity of $129,959,000. This results in a debt/equity ratio of 0.65. The ratio is not an accurate measure of the company’s actual debt position because it does not include a number of other debt-like events.


The company’s debt/equity ratio could be grossly understated. First, Avery makes $10 million in lease payments each year. Footnote (8) suggests that the lease contract may meet the criteria for a capitalized lease. If we assume a 10 percent discount rate and that these lease payments are required for the remainder of the firm’s life, a $100 million ($10,000/.10) liability could be added to Avery’s balance sheet. Second, the characteristics of the preferred stock issuance appear to resemble debt in many ways, suggesting that an additional $50 million liability could be added to the company’s balance sheet. Third, the bond payable liability is undervalued on the balance sheet because interest rates have fallen and the market value of the liability ($50 million) is greater than the book value ($45.132 million). Revaluing the liability would add another $4.868 million of debt to the balance sheet. Finally, Avery owns a 30 percent interest in a highly levered financial institution, Spartan Savings, and some would argue that adding 30 percent of Spartan’s assets and liabilities to Avery’s balance sheet would further increase the company’s debt/equity ratio. The capital structures of financial institutions are often comprised of 80 to 90 percent debt. In conclusion, Avery reports liabilities of $85.626 million as of the end of the year. That number could be increased by $155 million or more, which, in turn, would increase the debt/equity ratio from 0.65 to as much as 3.0. It is also not likely that Avery will exercise its options to call its outstanding bonds. First, the call price ($5.1 million = $1,020  5,000 bonds) exceeds the market value of the debt ($5.0 million = $1,000  5,000 bonds), indicating that it would be economically unwise to call the bonds. Second, the company appears to be facing solvency problems, and it may not wish to use its cash for this purpose. 5. The footnotes disclose sales of $115,000,000 in the year. This represents sales earned, and since all sales are made on credit, it does not necessarily measure cash collected from customers. The actual amount of cash collected from customers can be calculated as follows: Sales Less: Increase in net accounts receivable Cash collected from customers

$115,000 9,950 $105,050

Likewise, the amount of cash paid to suppliers for inventory can be calculated as follows: Cost of goods sold Plus: Increase in inventory (before write-down) Purchases

$60,500,000 15,000,000 $75,500,000

Purchases Less: Increase in accounts payable Cash paid to suppliers

$75,500,000 7,800,000 $67,700,000

6. While current assets have increased due to inventory and accounts receivable, it is not necessarily a good sign. Increases in inventory may mean sluggish sales and high inventory storage costs. The inventory write-down (income statement) is a signal that Avery is not selling its inventory in a timely manner, resulting in obsolescence. Increases in accounts receivable may signal poor collections from customers and/or a change in the credit approval criteria. The failure to turn over inventory quickly and collect cash from customers ties up funds that could be reinvested into the company. 7. Latest year revenues consist of sales that have fallen from the prior year, affiliate income, a gain on the sale of equipment, and revenue from special services (note 1). The latter three sources of income


accounted for almost $16,000,000 of Avery’s revenue but are not expected to persist in the future as part of the primary operations. The income from affiliate will not exist in the next year, because Avery sold its interest in Spartan Savings (note 4). Likewise, the gain on the equipment sale and special services revenue are not Avery’s primary sources of revenue. However, by aggregating these items in revenues, Avery is suggesting that they all are usual and frequent. Only the sales revenue is usual and frequent and may persist into the future. However, if the trend continues, sales revenue may not persist in the future either. 8. Purchase price (note 4) Less: Goodwill purchased (balance sheet) Value of Buckeye’s PPE acquired

$65,000,000

($50

million

cash

&

$15

15,000,000 $50,000,000

(primarily PPE)

9. Cash (statement of cash flows) 20,000,000 Accumulated Depreciation ? Equipment (note 3) Gain on Sale of Equipment (note 1)

20,000,000 6,000,000

Accumulated Depreciation must equal $6,000,000. Thus, the book value of the equipment was $14,000,000 ($20,000,000 – $6,000,000). 10. Pension expense Less: Increase in unfunded pension liability Cash contribution

$10,000,000 2,000,000 $ 8,000,000

million

stock)


CASE 5: ZENITH CREATIONS Re: Evaluation of Zenith’s financial condition Zenith Services is a potential acquisition candidate of several companies. Offers from buyers will reflect their assessments of Zenith’s financial condition and performance. So that Zenith can evaluate offers received and to understand how these users may be evaluating Zenith, I have prepared (1) an evaluation of Zenith’s financial condition and performance and (2) an analysis of the financial statement effects of several proposed actions. Financial Condition Zenith Services’ current ratio of 2.15 has declined slightly from 2.94 but remains strong. A greater drop occurred in the quick ratio, from 1.95 to 1.13. Part of this decline is due to the use of cash to retire part of a note, and to purchase $15,000,000 in equipment. Given Zenith’s other obligations, it can clearly better utilize its liquid assets. For example, as of the end of the latest year, Zenith reports payables and other accrual balances of almost twice that at the end of the previous year. At the same time, Zenith is reporting sufficient cash balances to settle these liabilities. While it may be important to settle some obligations, potential buyers of Zenith are also likely to require a much stronger cash position. The size of Zenith’s liabilities is growing faster than its equity, resulting in large debt/equity ratios. Despite the retirement of half a note issuance, the net loss has caused its debt/equity ratio to increase from 0.83 to 0.90. This means that Zenith is financed with approximately equal amounts of debt and equity. However, this ratio may grossly understate Zenith’s debt position. First, Zenith has an operating lease for $8,000,000 each year. Capitalized at a 10% rate results in an additional $80,000,000 of debt. If these leases were capitalized, the debt/equity ratio would approach 2.0. Also, in the latest year Zenith issued net $15,000,000 of preferred stock (500,000 shares at $30 each), which resembles debt but is not classified as debt on the balance sheet. Finally, due to a falling interest rate, the market value of its bond payable is actually higher than stated on the balance sheet. In addition, the debt/equity ratio is even higher because Zenith chose to buy back some of its own stock, thus decreasing total stockholders’ equity. Most of these treasury shares have not been subsequently reissued. Clearly, Zenith is carrying too much debt relative to its equity and current profitability. Another part of the increase in the debt/equity ratio was due to an increase in the pension liability by $2,000,000. This growth in the pension liability indicates that Zenith may be cutting back on its pension funding to conserve cash and improve its liquidity position. The ability of Zenith to turn over its receivables is particularly strong. Its receivables turnover was 5.58 and rose to 7.21. This ratio indicates that it takes an average of only 50 days to collect on sales. Receivables appear to be stable, and there are no signs of significant collection problems. The 50% increase in bad debts and the need to write off an uncollectible account suggest only limited problems with granting credit to customers. Zenith may have a problem with the quick sale of inventory. Its inventory turnover ratios of 1.93 and 2.04 are satisfactory but need to improve. A turnover ratio of 2.0 indicates that it takes 182 days to turn over inventory. Inventory is not turning over as quickly as it should be. The need to write off obsolete inventory is further evidence that Zenith carries too much inventory. Without a change in its inventory or sales policies, it is likely that future inventory write-offs are probable.


With the exception of inventory turnover, Zenith’s immediate liquidity is strong. However, its solvency is dangerously poor due to a large amount of liabilities. One area in which it can utilize its liquid assets is in the reduction of its liabilities. Performance Reported net income has dropped significantly from $19,529,000 to a loss of $11,510,000. One of the largest contributors to this drop was the fall in operating revenue of $16,000,000, including a decline in sales of $10,000,000. While the special services revenue and the loss on the sale of equipment (note 1) may not persist into the future, the decline in sales indicates that Zenith Services’ primary source of revenue and earnings may not persist into the future. The other primary contributor to the decline in net income is the restructuring charge of $15,000,000. While this charge is not extraordinary, it is not part of the normal operations and should not persist. In addition, users may positively view the charge if the restructuring activities contribute to improved operations. Other contributors to the net loss are an extraordinary loss on the extinguishment of part of a note and an inventory write-down of $2,000,000. While the inventory write-down may occur in the future, the note retirement is not expected to occur repeatedly in the future. While much of the net loss is due in part to nonrecurring items, earning power has dropped off significantly. Without the extraordinary loss and the restructuring charge, net income would have been a positive $8,914 and EPS $0.84. This still results in a return on assets of only 0.05 and a return on equity of only 0.10, down from 0.14 and 0.28 in the prior year, respectively. Income from operations fell 58% between the years. Zenith’s operating income return on assets fell from 18.3% to 7.1%. As a result of its poor performance Zenith’s times-interest-earned ratio fell from a strong 6.77 to a low 2.20. Unless earnings improve, Zenith may have trouble meeting its interest obligations. Also, despite the net loss, Zenith paid a stock dividend. Some users may view this dividend in lieu of a cash dividend as a ploy to appear as a healthy company issuing dividends. Stock dividends, however, do not add to the wealth of the investor. As they dilute potential ownership and reduce retained earnings, potential investors may, in fact, not view the stock dividends favorably. Also, Zenith’s investment in Lyon Real Estate through the issuance of common stock significantly diluted the common share value and has not generated significant returns. A $200 loss and only $700 in dividends result in less than a 7% return on the cash investment of $10,000. It is important that Zenith Services’ income statement disclose sales revenue separate from the other sources of revenue and expense (income from affiliate, loss on sale of equipment, special services), particularly if Zenith wants users to focus on those items expected to persist in the future. This would highlight the sales and expenses expected to persist into the future. Earnings quality may be weak. First, Zenith has experienced LIFO liquidations in the past three years. LIFO liquidations result in inflated profits. Sales are not actually greater; rather, the costs of the inventory liquidated are lower than the inventory costs in prior years. While a preliminary analysis suggests Zenith’s liquidity is strong, examination of its solvency and earnings power indicates otherwise. Overall, Zenith is unhealthy because of its debt position. Also, its weakening profitability is contributing to a declining ability to pay its debt obligations or secure additional financing.


Management Actions Zenith’s management is considering a number of actions, some of which significantly affect the financial statements and assessments of financial health. Below are discussed the effects of each of these actions. (1)

Purchase treasury stock at current market price.

The purchase of treasury stock requires the use of cash. While Zenith has sufficient cash available, it would be wiser to use the cash to pay off outstanding payables. On the other hand, if Zenith anticipates improving its health and its stock price, it may be able to secure additional cash by purchasing stock now at a low price and reselling it later for a higher price. The effect of a purchase on the financial statements would be to reduce cash and stockholders’ equity. By reducing current assets and equity, the current ratio will fall and the debt/equity ratio will increase. Earnings per share will increase (if earnings are positive) and decrease (if earnings are negative) as the number of outstanding shares decreases. Further, if retained earnings are declining, further net losses may place Zenith in a deficit retained earnings situation. Outsiders may wonder whether treasury stock was purchased in an attempt to drive up the stock price or increase earnings per share. The purchase will not improve the financial health of Zenith, and the cash reserves may be more wisely used to help improve Zenith’s financial health.

(2)

Write off a relative large uncollectible accounts receivable.

An accounts receivable write-off does not affect net accounts receivable, total assets, or income. The effect is only to remove the account from the allowance and receivable accounts. Outsiders shouldn’t notice a significant difference due to the write-off. (3)

Issue a 20% stock dividend.

A 20% stock dividend would amount to 2,311,000 shares and reduce retained earnings by market value of the shares. Since retained earnings is only $23,488,000, a stock dividend would reduce retained earnings to zero if the current market price is much over $8 per share. The stock dividend does not change assets, liabilities, or total stockholder’s equity. Thus, ratios will not be affected. Retained earnings is effectively capitalized. Unless current shareholders are demanding a dividend, Zenith should save its retained earnings as much as possible, particularly if future net losses are anticipated. Investors’ wealth levels do not benefit from most stock dividends. Also, outsiders may question the reason for the stock dividend, given Zenith’s declining health, and view it as a ploy to convince investors of the financial health of the company.


(4)

Redeem the remaining notes payable for $23,200,000.

Currently, notes payable has a book value of $19,665,000. If Zenith redeems the note for $23,200,000, it will realize an extraordinary loss of $3,535,000. The cash paid will contribute to a decreased current ratio. However, the retirement of the debt will reduce the debt/equity ratio to a more attractive 0.71 ($82,185,000 – $19,665,000)/($91,648,000 – $3,535,000). Because the loss is extraordinary, it will not affect income from continuing operations. However, the overall return on assets will worsen due to the loss on the retirement. While the reduced debt/equity ratio improves Zenith’s flexibility to take advantage of future opportunities and users would not expect the resulting extraordinary loss to persist in the future, the reduction in cash and overall profitability may suggest that it is economically unwise to redeem the note right now.

(5)

Sell the real estate received in the acquisition of Lyon Real Estate for $12,000,000.

When Zenith acquired Lyon Real Estate, it acquired real estate worth approximately $5,010,000. Price Goodwill Value of real estate

$25,010,000 20,000,000 $ 5,010,000

Thus, if Zenith sells the real estate for $12,000,000, it will realize a gain of $6,990,000. By increasing equity by $6,990,000 (before taxes), the gain will reduce the debt/equity ratio. At the same time, the cash received will improve the current ratio. Unlike the debt retirement action, the sale of the real estate does not result in an extraordinary item. The sale of real estate is not a primary operation of Zenith, but it is also not usual and infrequent. Thus, users are more likely to perceive the gain from this sale as more persistent than the gain from the retirement of debt. Given the small return currently being earned on this investment, the sale may be a wise action.

(6)

Change the inventory cost flow assumption from LIFO to FIFO.

A change in the accounting method from LIFO to FIFO requires retroactive adjustments to the comparative balance sheets. This retroactive restatement will result in an increase in retained earnings for the lower income than was reported in the past, due to the LIFO assumption relative to using the FIFO assumption. By increasing retained earnings, debt/equity ratios in the restated years will decrease. Depending on how other firms in the same industry treat inventory, outsiders may not favorably view the change. However, they also may view the change as window-dressing and poorly evaluate Zenith’s earnings quality.


CASE 6: PIERCE AND SNOWDEN

MEMO To: Recommendation Team Re: Analysis of Wellington Mart and Wagner Stores To assess whether Pierce and Snowden should extend credit to Wellington Mart and Wagner, I have analyzed their financial statements along three dimensions—solvency, earning power and persistence, and earnings quality. Solvency position

Current ratio Quick ratio Debt/equity Receivables turnover

Wellington Mart Year 2 Year 1 0.47 1.09 0.39 0.58 0.80 0.62 9.85

Wagner Stores Year 2 Year 1 1.97 3.43 0.82 1.80 1.80 1.51 8.65

Wellington Mart is experiencing decreasing liquidity and solvency. Both its current and quick ratios have fallen significantly between the years shown. As of the end of the most current year, Wellington’s current and quick ratios are a dangerously low 0.47 and 0.39, respectively. A large contributor to the fall in the liquidity is Wellington’s acquisition of Marilyn Real Estate. Marilyn cost $80,000,000, $50,000,000 of which was in cash. Also, Wellington made plant and equipment acquisitions of $23,000,000. Altogether Wellington made significant drains on its cash, given its already weak liquidity and solvency positions. At the same time, its total debt/equity ratio has increased from 0.62 to 0.80. Part of the increase is due to the issuance of a new note for $13,900,000. While the $6,200,000 increase in debt is due to its adoption of a defined pension plan, the ratios may, in fact, be grossly understated. First, Wellington has an operating lease, which requires annual payments of $12,000,000. Given that it has an option to buy the facilities at the end of the lease term for 25% of the market value, this lease may be more properly classified as a capital lease. At a discount rate of 10%, an additional $120,000,000 in debt would be included on Wellington’s balance sheet. However, it appears that Wellington management may be using some discretion to classify the lease in order to avoid violating a bond covenant to keep its liability/asset ratio below 0.60. Also, Wellington has $50,000,000 of outstanding preferred stock, which resembles debt but is not classified as debt on the balance sheet. Finally, Wellington may be subject to certain environmental liabilities in the future. These also are not yet reflected on the balance sheet as future obligations. On the positive side, due to an increasing interest rate, the market value of its bond payable is actually lower than stated on the balance sheet. If Wellington calls these bonds, it will realize an extraordinary gain on the retirement of the bond. The ability of Wellington to turn over its receivables is particularly strong. With a turnover ratio of 9.85, it takes, on average, only 37 days to collect on sales. While receivables are growing, the allowances may be understated. Bad debt expense was $1,550,000, while the allowance only increased $650, indicating possible write-offs of additional uncollectible accounts.


Also, despite its net loss, Wellington paid a stock dividend. This dividend in lieu of a cash dividend may be a ploy to appear as a healthy company issuing dividends. Stock dividends, however, do not add to the wealth of the investor. As they dilute potential ownership and reduce retained earnings, potential investors may, in fact, not view the stock dividends favorably. Wagner Stores is also experiencing a decreasing current ratio, from a strong 3.43 to a much lower 1.97. Also, its quick ratio has fallen from 1.80 to a much weaker 0.82. These ratios are still strong and may only reflect a decision to better use its assets. One reason for the strong ratios is that Wagner issued debt and new common stock for a total of $20,800,000, so that the purchase of additional property, plant, and equipment for $34,500,000 did not severely drain its liquid assets. Due to the additional bond issuance, the debt/equity ratio of Wagner also increased slightly from 1.51 to 1.80. Unlike Wellington, Wagner’s liabilities include capital leases of $4,771,000. In addition, however, Wagner also has operating lease payments of $6,200,000 per year. If capitalized at a discount rate of 10%, an additional $62,000,000 in debt would be included on Wagner’s balance sheet. Also, unlike Wellington, Wagner has only $5,000,000 in preferred stock, which may have features and obligations similar to debt. On the positive side, due to an increasing interest rate, the market value of its bond payable is actually lower than stated on the balance sheet. Like Wellington, if Wagner calls these bonds, it will realize an extraordinary gain on the retirement of the bond. Like Wellington, Wagner maintains a strong receivables turnover ratio of 8.65. Wagner does not separately report on its allowance accounts. However, there are no signs that Wagner has collection problems. Unlike Wellington, Wagner was able to pay a cash dividend of $2,800,000, or 35% of net income. Summary: Given its declining liquidity and solvency and potential additional liabilities, I rate Wellington’s solvency position a 3. Wellington appears to be increasing its debt, spending its cash on investments not immediately necessary for improving its financial position, and not significantly extinguishing its current obligations. Since it is not yet in default, I did not believe a 1 or 2 rating appropriate. Also, since its receivables turnover is strong, obtaining cash from its operations does not seem to be a problem yet. In contrast to Wellington, Wagner has a stronger solvency position, although it is weakening slightly. Wagner has a strong balance sheet, a strong receivables turnover, and spends its cash more conservatively than does Wellington. Wagner needs to be careful to not overload itself with debt. Based on these strengths, I give Wagner an 8 on solvency.

Earning Power and Persistence


Inventory turnover ROA ROE Return on sales Times interest earned (based on operating income [loss])

Wellington Mart 9.00 –0.04 –0.10 –0.07 1.56

Wagner Stores 1.93 0.09 0.26 0.11 8.91

Wellington experienced a significant net loss of $10,878,000. A large contributor to this loss was a restructuring charge of $9,000,000. While this charge is not extraordinary, it is not part of the normal operations and is not expected to persist in the future. In addition, users may positively view the charge if the restructuring activities contribute to improved operations. The loss would have been even greater, however, without other sources of income or gains included in other revenues and expenses on the income statement. Wellington realized a $2,000,000 gain from the sale of securities, $2,500,000 in affiliate income, and $1,500,000 from a translation adjustment. There is reason to be concerned because none of these items is essential to Wellington’s operations or can be expected to persist in the future. Based on their income statement, it appears as if Wellington’s selling and administrative expenses are contributing to the loss situation. There is not evidence that these significant expenses are not expected to persist in the future. The loss of $1,600,000 is more appropriately classified as a normal recurring part of operations. Given that the net loss would have been even greater without some of these items unrelated to the normal business, earning power is extremely poor. A return on assets of –0.04 suggests that Wellington is not utilizing its assets effectively at all. Although interest expense is not excessive, the loss on operations alone results in a negative times-interest-earned ratio. Clearly, Wellington will have trouble meeting its interest obligations. Wagner has a good level of earning power. All three earning power ratios (ROA, ROE, and EPS) are strong. In addition, the return on sales and times-interest-earned ratio suggests Wagner is financially strong and is utilizing its assets effectively. Note that Wagner has profitability on operating income of $13,250,000, versus $3,622 for Wellington on almost twice as much sales The only apparent weakness is Wagner’s inventory turnover of 1.93. A turnover of 2 means that it is taking 182 days to get rid of the inventory. This long period is resulting in inventory obsolescence and unnecessary write-off losses. Note that like Wellington, Wagner has an “other revenue and expense items” on the income statement. Wagner appears to put a number of significant and important expenses in this item. For example, interest expense, loss on inventory write-down, and loss from discontinued operations should all be disclosed separately. The interest and the inventory write-down are part of normal recurring operations and should be included in operating income. The loss on the discontinued equipment, while not part of recurring operations, should be separately disclosed, given its magnitude of $3,200,000. If these items had been properly classified, operating income would have been lower and users would realize that the $3,200,000 loss from the discontinued equipment is not expected to persist. Based on their income statement, it appears that Wagner’s expenses, including general operating and depreciation expense, are reasonable. Like Wellington, disaggregation of the general operating expenses may provide information. This would help users decide whether these expenses are expected to persist in the future or not.


Finally, Wagner’s equity investments appear to be performing well also. In the latest year, Wagner earned $1,500,000 on an investment totaling about $10,000,000 for a return of about 15%. Summary: Given its poor operating performance, Wellington does not appear to have good earning power or persistence of positive income sources. Based on these factors, I rate Wellington a 2 on earning power and persistence. Wagner earned positive income mostly due to its primary operating activities. Also, some of the losses or expenses are not expected to persist in the future. For these reasons, Wagner earns a 7. Earnings Quality Wellington has presented its income statement and made decisions that favor the presentation of its performance. First, hidden within “other revenues and expenses” is the inventory write-down of $1,600,000. This is an ordinary part of business and should be reported in the operations section. Second, selling and administrative expenses are a significant $45,000,000 but are not described further. Clearly, this expense may contain some questionable items. Third, any of the expense due to current-period pension expense should be included as a component of normal operating income. Fourth, Wellington may have reclassified a security to trading in order to recognize the unrealized gain. Wagner also appears to have good earnings quality. Like Wellington, some items are hidden in the “other revenue and expense” line that would be more appropriately disclosed as a component of income from operations (interest expense, inventory write-down). Also, like Wellington, general operating expenses may contain estimated expenses that hurt earning quality. Unlike Wellington, trading securities do not appear to be managed and goodwill is not a significant drain on net income. Note that Wagner uses a FIFO inventory cost flow assumption; Wellington uses a LIFO assumption. The result is a lower income for Wellington. However, the footnotes indicate that price levels are relatively stable so that this difference is not significant. Summary: Wellington’s earnings quality is not overly inflated by items at the discretion of management. Estimated expenses do not appear to be understated, and except for a single gain on a security, some unrealized gains and losses, and a translation gain, the reported loss amount appears to be of good quality, earning it a rating of 6. There is the matter of the contingent liability related to possible environmental cleanup bill. At this point it is not clear as to the magnitude of the cost and therefore it is difficult to assess its impact on the quality of earnings of Wellington. Wagner does not appear to have any artificially high amounts or have write-off earnings. For this reason, I rate Wagner’s earning quality an 8.


Part: 2 CHAPTER 1

Financial Accounting and Its Economic Context

SYNOPSIS In this introductory chapter, the author discusses the economic context of financial accounting including: 1) the demand for financial information; 2) the environment of financial accounting; 3) the controls associated with the financial accounting process; and 4) generally accepted accounting principles. The author illustrates and discusses the roles that equity investors, debt investors, managers, and auditors have in the financial accounting environment, providing a basis for the discussion of both a user orientation and an economic consequence perspective. The author introduces the auditor’s report, the management letter, the financial statements and the footnotes. The author also presents the basic concepts of financial statement analysis, solvency and earning power. The basic forms of investment, debt and equity are described. The economic environment in which financial reports are prepared and used is discussed and key elements of the financial environment are introduced. An international perspective of the general state of accounting practices and standards throughout the world is also considered. The ethics vignette considers the independence of accounting firms who also serve as middlemen in the sale of their audit client’s products. The Internet research exercise examines subsequent results for IBM a company that has performed well during recent years, but fell short of analysts’ expectations in Q1 of 2013. The author compares and contrasts the four different types of accounting (i.e., financial, not-forprofit, managerial, and tax) along six dimensions in Appendix 1A. The following key points are emphasized in Chapter 1: 1.

The economic role of financial accounting statements.

2.

The four financial statements and the kind of financial information each provides.

3.

The standard audit report, management letter, and footnotes to the financial statements.

4.

The two forms of investment—debt and equity—and how information on the financial statements relates to them.

5.

The nature and importance of corporate governance and the role of financial statements.

6.

The current status of accounting standard setting – both in the U.S. and internationally.


2

Chapter 1 TEXT/LECTURE OUTLINE Financial accounting and its economic context. I.

Financial reporting and investment decisions. A.

Investment decisions. 1.

Profit-seeking companies – Managers prepare reports for owners.

These

reports include the four financial statements: the balance sheet, income statement, statement of shareholders’ equity and statement of cash flows. 2.

Owners and other interested parties including potential investors, bankers, government agencies, customers, and suppliers use publicly available financial reports to assess the financial condition and performance of the company and its managers.

3.

User decisions are often based on information provided by financial reports.

4.

Effects of user decisions may impact the financial condition of the company, its managers and owners.

B.

Economic consequences. The use of financial statements by outsiders leads to economic consequences for managers and companies. Knowledge of how business decisions effect the financial statements is useful in assessing how management decisions might be perceived by outsiders. Considering and understanding how such decisions affect the financial statements is an economic consequence perspective.

C. User orientation. Managers need to know how financial statements are used. They also need to know how to read, evaluate, and analyze financial statements. This perspective is called user orientation. See also Appendix 1A introducing managerial, tax and notfor-profit accounting. II.

The demand for financial information: a user’s orientation (Mary Jordan / Microline vignette). A.

Consumption vs. investment.

B.

Where to invest?

C. Demand for documentation. D. Demand for independent audit. E.

Martin and the CPA: different incentives.

F.

The auditor’s report, management letter, financial statements, and footnotes.


3

Chapter 1 1.

Balance sheet.

a. Assets. b. Liabilities. c. Shareholders’ equity. i.

Common stock.

ii. Retained earnings. 2.

Income statement.

a. Revenues. b. Expenses. c. Net income. 3.

Statement of shareholders’ equity – retained earnings and dividends.

4.

Statement of cash flows.

a. Operating activities. b. Investing activities. c. Financing activities. G. Analysis of financial statements. 1.

Solvency.

2.

Earning power.

H. Forms of investment.

I.

1.

Debt.

2.

Equity.

Summary and wind up of Microline vignette.

III. The economic environment in which financial reports are prepared and used. A.

Key elements of the financial accounting environment.

1.

Providers of capital. a)

Equity investors. (1)

Equity investors purchase ownership shares (i.e., equity) in a company.

(2)

Equity investors are most interested in acquiring information about a company's earning power.

b)

Debt investors. (1)

Debt investors loan capital to a company.


4

Chapter 1 (2)

Debt investors are most interested in acquiring information about a company's solvency position.

B.

2.

Managers.

3.

Contracts. a)

Debt contracts.

b)

Compensation contracts.

4.

Financial accounting statements.

5.

Independent auditors.

6.

Legal liability.

7.

Ethics and professional reputation.

Reporting entities and industries. 1.

Consolidated financial statements.

2.

Subsidiaries.

3.

Industries. a. Manufacturing. b. Retailing. c. Services. d. SIC codes.

IV. Corporate governance. A.

B.

C.

D.

Financial information users and capital markets. 1)

Equity investors.

2)

Debt investors.

3)

Management and other users.

4)

Capital markets.

Contracts. 1)

Debt covenants.

2)

Management compensation.

Regulations and standards. 1)

SEC.

2)

GAAP.

3)

Political process.

Independent auditors. 1)

“Big Four”.

2)

Regional and local firms.


5

Chapter 1

V.

E.

Board of directors and audit committee.

F.

Sarbanes-Oxley Act (2002). 1)

Certification by principal executive and financial officers.

2)

Additional responsibilities to assure adequate internal control.

3)

Annual report on internal controls over financial reporting.

G.

Legal liability.

H.

Professional reputation and ethics.

International Perspective: Movement Toward a Single Global Financial Reporting System.

VI. Appendix 1A Three Other Kinds of Accounting: A. Not-for-profit accounting. Not-for-profit accounting generates financial information for those entities that do not exist solely to generate profits. B. Managerial accounting. Managerial accounting generates financial information useful to managers for making day-to-day operating decisions. C. Tax accounting. Tax accounting generates financial information in compliance with the Internal Revenue Code. VII. Ethics in the real world. VIII. Internet research exercise. LECTURE TIPS 1.

Students often expect accounting to be a boring “cookbook” course. Discussion of Chapter 1 can set the tone for the rest of the course through a relevant, interesting discussion of the economics of accounting and an early introduction of real world examples.

2.

Although students usually understand the motives underlying the economic consequences arguments, they often do not apply these motives to accounting.

OUTSIDE ASSIGNMENT OPPORTUNITIES Article search: current developments at Google. 1.

Search the financial press (The Wall Street Journal, Barrons, Fortune, Forbes, etc.) for an article reporting a current development that relates to Google. Consider the impact on Google’s financial statements (presented in Appendix C), from both a user orientation and an economic consequence perspective and present your findings in writing and/or orally to the class.


Chapter 1

6

Article search: current developments in accounting standards 2.

Search the financial press (The Wall Street Journal, Barrons, Fortune, Forbes, etc.) for an article that reports a current development in accounting standards, for example, a new or proposed pronouncement by the FASB or IASB. Consider its impact from both a user orientation and an economic consequence perspective and present your findings in writing and/or orally to the class.

ANSWERS TO IN-TEXT DISCUSSION QUESTIONS Page No. 4. The first statement reflects an economic consequences perspective. Management of high-tech companies knows that the capital markets make decisions based on the level of a company’s reported profits. Therefore, management would prefer to use accounting methods that would tend to maximize their reported profits. The second statement takes a user orientation. Analysis of financial statements is important to users in making investment and credit decisions. As will be learned throughout the text, financial statements that do not reflect the true costs associated with reported earnings mislead investors who depend on the information contained in financial statements in making investment decisions. 12.

Sales and net income would be reported in the income statement. Net income would also be reported in the statement of shareholders’ equity. Total assets and total liabilities would be reported in the balance sheet. Net cash flows from operating activities would be reported in the statement of cash flows. Total expenses would be $8,510 million, the difference between sales and net income. Shareholders’ equity would be $3.9 billion, the difference between total assets and total liabilities. The net income to sales ratio would be 10.4%, or net income divided by sales.

13.

A financial services company such as Bank of New York is especially interested in assets that are liquid and have contractual terms that specify the future amounts of cash flows, which can be matched with funding sources such as deposits and borrowings. A bank’s loan portfolio turns over fairly rapidly. A consumer products company such as Coca Cola Company makes investments in bottling companies and other entities to further its production and marketing of soft drinks and other beverages. Such investments would be relatively more permanent and less liquid than those of a financial services company, and future cash flows would be less easily determinable.

15.

The reported earnings of over $13.8 billion leads one to conclude that GE has significant earning power and strongly suggests the possibility of similar earnings in the future. Past business decisions by GE’s management resulted in earnings of this magnitude. A comparison of these earnings against management’s goals, or against earnings for other years, or against their competitors’ earnings would tell us a lot more about management’s performance. Compensation contracts with managers would ordinarily


7

Chapter 1 contain provisions that enhanced managers’ earnings when earnings targets and other specified criteria are met.

16.

Boeing Tommy Hilfiger DuPont American Express General Electric Microsoft eBay Southwest Airlines Sprint

manufacturer manufacturer (outsources actual manufacturing) manufacturer financial services manufacturer manufacturer service service service

17.

Private companies, that is, those companies whose shares are not traded on the public stock exchanges are not subject to oversight by the Securities and Exchange Commission or the Public Company Accounting Oversight Board. Private companies avoid the costs of complying with regulations issued by these authorities. These costs have risen in recent years due largely to compliance with the requirements of the Sarbanes Oxley Act. The managers of private companies are less burdened by the restrictions and costs of regulation. The investing public largely is unable to invest in private companies. Those who do invest in private companies do not benefit from the protections regulation affords, nor do they suffer the costs of regulatory compliance.

18.

Because stock market analysts were less optimistic about IBM’s future revenues and earnings, the market price of the company’s stock decreased. This was because the desire to own the company’s stock decreased. More investors wanted to own the stock because the future of the company looked weaker than it had before publication of the 2013 first quarter results. Share prices are related to the market’s expectations of the probability of future cash flows. When the prospect of future cash flows decreases, share prices respond negatively. Management compensation is often tied to profitability and/or to the market price of the company’s stock. These incentives are designed to impact management behaviour by encouraging management to work to improve profitability.

18.

Motorola’s employee incentive plan awards bonuses to eligible employees that achieve specified business goals. Tying management’s financial goals to employee compensation increases the likelihood that employees will address corporate financial goals with enthusiasm, thereby increasing the likelihood that financial goals will be met.

19.

Perhaps the current controversy over accounting standards for measuring management compensation rages because the benefits of improving financial reporting are not readily susceptible to measurement while the perceived negative consequences are apparent to those who fear they would be negatively impacted. This is true with respect to all kinds of change.

21.

Dell, Inc. and PricewaterhouseCoopers would have had a conversation about the magnitude and scope of the accounting irregularities, the impact on the financial statements and the extent to which the errors may have affected decisions made (or to be made) by persons relying on the accuracy of the financial statements. If it was determined that the financial statements were materially in error, corrections to the


8

Chapter 1

financial statements would have been discussed. Possible restatement of prior years’ financial statements may also have been discussed. 21.

Audit committee members share the responsibility for oversight of management’s choice of auditor and monitoring the audit to ensure it is thorough, objective and independent. Since the auditor’s job is to audit the financial statements, audit committee members need to have a pretty good understanding of both audits and financial statements.

22.

Sarbanes-Oxley was intended to force a tightening of corporate governance. The responsibilities of corporate directors, audit committees, corporate executives, financial officers and auditors have been dramatically increased. With the new emphasis on responsibility auditors and managers are on high alert and are less willing to overlook errors in financial reporting and weaknesses in internal controls. This new, higher level of diligence has lead to an increase in incidents of accounting errors being admitted and corrected.

23.

“Sound principles of corporate governance are critical to obtaining and retaining the trust of investors -- and to GE's overarching goal of performance with integrity”, - as quoted from the web site cited.

24.

A single set of global standards would provide uniformity across international borders and to provide investors and creditors credible information for comparison and decision making and improve transparency in global financial markets. Having two different sets of financial reporting standards results in costs associated with duplicate reporting and reconciling two sets of information. Having a single set of standards creates interesting and unresolved international legal issues. For instance, can the SEC impose its authority to determine accounting standards to international standards setters? Also, should small businesses across the world be forced to adopt international accounting standards even if they poorly serve the needs of their exclusively local users?

CHARACTERISTICS OF END-OF-CHAPTER ASSIGNMENTS Item

Difficulty

Issues for discussion: ID1–1 E ID1–2 E ID1–3 E ID1–4 M ID1–5 M ID1–6 E ID1–7 E ID1–8 E ID1–9 E ID1–10 E ID1–11 M ID1–12 E ID1–13 E ID1–14 M

Description

Financial statement users Auditors and management fraud Audit committees Banks, the credit crunch, and the financial statements Financial statement relationships Debt covenants Managing reported profits Audit report Corporate governance Ethics and accounting Global reporting standards International accounting standards Managerial vs. Financial accounting The annual report of Google


CHAPTER 2

The Financial Statements

SYNOPSIS The author opens the chapter with an illustration of the flow of capital through a business. Operating, investing and financing activities are introduced. The author then presents overviews that include the various components of each financial statement: the classified balance sheet, the income statement, the statement of shareholders’ equity, and the statement of cash flows. The main part of the chapter ends with a discussion of the relationships among the financial statements. An international perspective is presented on how financial statements differ in other countries. This discussion includes an examination of the financial statements of Unilever, a leading supplier of consumer brands registered in the Netherlands. The ethics vignette considers issues faced by a Swiss manager who is hesitant to adopt IFRS. The Internet research exercise considers the financial results and corporate strategy of JCPenney. The following key points are emphasized in Chapter 2: 1.

The three basic activities of a business and how they are reflected in the financial statements.

2.

The balance sheet, income statement, statement of shareholders’ equity, and statement of cash flows and how these financial statements are used.

3.

Non-U.S. financial statement format and terminology, similarities and differences.

TEXT/LECTURE OUTLINE The financial statements. I.

How businesses are conducted. A.

Flow of capital through a business.

B.

Three activities of a business. 1.

Financing activities: those activities involving the collection of capital through debt or equity issuances and any associated payments.

2.

Investing activities: those activities involving the acquisition and the sale of producing assets (i.e., the assets used to produce and support the goods and services provided).


2

Chapter 2

3 3.

II.

Operating activities: those activities involving the sale of goods and services.

The four fundamental financial statements. A.

Classified balance sheet. 1.

The classified balance sheet is a statement of the company’s financial position as of a point in time. The balance sheet is a statement of the fundamental accounting equation.

2.

In a classified balance sheet, assets and liabilities are grouped into subclassifications; in an unclassified balance sheet, assets and liabilities are not grouped into sub-classifications, they are simply listed.

3.

Assets and liabilities are listed on the balance sheet in order of liquidity.

4.

Asset groupings. a)

5.

(1)

Current assets are assets that are expected to be realized or converted into cash in the near future, usually within one year.

(2)

Current assets typically include cash, short-term investments, accounts receivable, inventory, and prepaid expenses.

b)

Long-term investments.

c)

Property, plant, and equipment.

d)

Intangible assets.

Liability groupings. a)

b) 6.

Current assets.

Current liabilities. (1)

Current liabilities are liabilities that are expected to be settled through the use of current assets.

(2)

Current liabilities typically include accounts payable, miscellaneous other payables (such as wages payable, interest payable, and so forth), and current maturities of long-term debts.

Long-term liabilities.

Shareholders’ equity groupings. a)

Contributed capital.


32

Chapter 2

B.

b)

Retained earnings (i.e., earned capital).

c)

Effect of organizational form (i.e., corporate versus partnership) on the equity section.

Income statement. 1.

The income statement explains the change in the company’s net assets (i.e., total assets less total liabilities) during the accounting period due to operating activities.

2.

The change in net assets from operating activities is called net income (or loss) and is computed as revenues less expenses.

3.

a)

Revenues are defined as inflows of assets or outflows of liabilities over a period of time due to a company’s operating activities.

b)

Expenses are defined as outflows of assets or inflows of liabilities over a period of time due to a company’s operating activities.

Since revenues and expenses are defined by the changes in assets and liabilities, every transaction affecting the income statement also affects at least one asset or liability account on the balance sheet.

C. Statement of shareholders’ equity. 1.

The statement of shareholders’ equity explains the change in the retained earnings and contributed capital balances during the accounting period.

2.

The change in the retained earnings balance is due to the company’s net income or loss and distributions to the company’s owners (i.e., dividends).

3.

The change in the contributed capital balance is due to capital collected from the sale of equity securities.

D. Statement of cash flows. 1.

The statement of cash flows explains (i.e., summarizes) the change in the cash balance during the accounting period. a)

Cash flows from operating activities.

b)

Cash flows from investing activities.

c)

Cash flows from financing activities.

III. Relationships among the financial statements. IV. International perspective.


Chapter 2

4

3 V.

Review Problem: Bed Bath & Beyond.

VI. Ethics in the real world. VII. Internet research exercise.

LECTURE TIPS 1.

The illustration of the flow of capital through a business introduces the concept of operating, investing and financing activities.

2.

Students should be encouraged to read the text carefully so as to thoroughly understand what each account measures. A useful in-class drill requires students to explain what the amount reported on the balance sheet for a particular account represents.

3.

Many introductory students have difficulty differentiating between cash collections and revenue and between cash disbursements and expenses. It is critical that these differentiations are made when discussing the income statement versus the operating activities section of the statement of cash flows.

4.

It is critical to students’ understanding of accounting and how to use financial statement information that they grasp how the four financial statements relate. However, most students often view each financial statement as independent and do not appreciate how the statements relate. Therefore, explain and demonstrate how the financial statements are linked together and complement one another.

5.

When first discussing how to use the information in the financial statements, provide a general overview of how a potential investor would ascertain the company’s solvency (i.e., compare its assets to its obligations) and earning power. Then provide a more detailed discussion by using a classified balance sheet (i.e., using current assets versus current liabilities), the income statement, the statement of cash flows, and ratio analysis.

OUTSIDE ASSIGNMENT OPPORTUNITIES Study of annual reports: evaluation of profitability and solvency using financial statements and other information. 1.

Obtain a recent annual report for a local publicly held company. Read the financial statements and notes and evaluate profitability and solvency using concepts developed in the chapter. Read the letter to shareholders and other information in the annual report and compare with the results of your analysis; especially note instances where the letter to shareholders or the other information reads more favorably than indicated in the financial statements. Report your findings in writing and/or in a brief presentation to the class.

Comparative study of annual reports: a foreign company compared to a U.S. company. 2.

Obtain a recent annual report for a publicly held foreign company. Compare the form, content, and terminology used in its financial statements to that of a U. S. public company,


52

Chapter 2 preferably one in the same industry. Report your findings in writing and/or in a brief presentation to the class.

ANSWERS TO IN-TEXT DISCUSSION QUESTIONS Page No. 31.

Cash provided by operating activities includes cash collected from the sale products and cash payments for related costs and expenses. Investing activities consist of capital expenditures for additional plant and equipment and other assets that relate to the productive capacity of the business. Financing activities include proceeds from the issuance of debt and/or equity, and related repayments of debt and/or payment of dividends.

34.

McDonald’s Corporation’s total assets consist mostly of property, plant, and equipment because McDonald’s is a traditional “brick and mortar” business, whereas Google is an internet company whose presence exists mostly in cyberspace and which is valued mostly for it’s intangible assets and intellectual property.

34.

The principal revenue producing asset of a retailer such as Sears is its inventory of merchandise, which is included in current assets. The amount of inventory would be high relative to property, plant and equipment, especially given that many retailers lease, rather than own, their premises, and leasehold interests and obligations are not shown on the balance sheet. Current assets also include cash, short-term investments, receivables, and prepaid expenses. AT&T, a legacy telephone company, has a huge investment in equipment which is spread throughout its network. All this equipment is a non-current asset. AT&T has little inventory and its current assets are mostly cash and receivables.

35.

Short-term investments are made in readily marketable debt and equity securities to earn income on otherwise idle funds. An insurance company such as WellPoint maintains a high level of short-term investments to insure that they will be able to pay claims, make refunds, and so on, on a timely basis.

35.

The principal activities of banks such as JP Morgan Chase are to earn net interest income by gathering deposits and in turn loaning those funds out to others, as well as providing fee-based services. The largest asset category involved in this process is the loan portfolio, a category of receivable. Wal-Mart is a retailer. It’s main business involves buying and re-selling inventory. Receivables are a relatively minor facet of WalMart’s business.

35.

Retailers such as Home Depot and manufacturers such as Goodyear Tire & Rubber are in the business of selling products, and carry significant amounts of inventory. Service enterprises such as Yahoo and financial institutions such as Bank of America do not deal in products and therefore do not have inventories.

36.

These are the production and/or acquisition costs of television programming that has not yet been licensed or sold. These costs will be expensed when the programs are licensed or sold.


Chapter 2

6

3 37.

Patents are intangible assets that are reflected on a company’s balance sheet when it purchases them from another company. The costs of developing patents internally are generally expensed when incurred. The costs of registering a patent, however, are capitalized. Patents are intellectual property of great value and can confer a monopoly on a patent holder. Patent rights must be zealously protected to prevent their value from being diminished. Patents are reported in the balance sheet at cost, net of accumulated amortization.

38.

The intangible asset goodwill is reflected on a company’s balance sheet when it purchases another company for an amount greater than the market value of its identifiable assets and liabilities. A large portion of goodwill on Cisco System’s balance sheet indicates that Cisco Systems is growing through acquisitions of other companies as opposed to growth through internal expansion.

39.

The primary source of funding for banks such as Bank of America is through deposits gathered from its diverse customer base. Banks do not present classified balance sheets, but most deposits are relatively short-term (current). Borrowings are a secondary source of funding, and are not as great as the deposit liability. Banks are highly leveraged, i.e., shareholders’ equity represents a smaller portion of total assets (usually less than 10%).

40.

A deficit in retained earnings means that Amazon.com has not been profitable on a cumulative basis since its inception, net of any distributions to shareholders. This is not an uncommon situation for young companies, especially those in Internet related businesses.

42.

Cost of goods sold represents the original cost of inventory items sold by manufacturers, wholesalers, and retailers. This inventory cost is reported separately from other operating expenses. Service businesses such as H & R Block provide services and do not sell inventory, thus there is no cost of goods to report.

43.

If the profits were the result of expense reductions, and continued expense reductions were in doubt, continued profits would be doubtful as well.

43.

An excess of other (non-operating) expenses over other (non-operating) revenues would cause net income to be lower than net income from operating activities. In AMR’s case, there is an overall net loss. Investors would be more concerned about net income from operating activities which would be expected to recur while (significant) other revenues and expenses tend to be one-time items. Analysts may see potential for on-going profitability if non-operating expenses can be controlled, but there should be caution given the overall loss shown by the company.

44.

Dividends are reported in the statement of shareholders’ equity as a reduction in retained earnings. Net income is reported as an increase in retained earnings in the statement of shareholders’ equity. Retained earnings represent the portion of net income not paid to shareholders as dividends. The difference–60% to 70% of net income in the case of Johnson & Johnson–represents the portion of earnings normally reinvested in the business.


72

Chapter 2

45.

Cash flows from operating activities generally are usually be expected to continue, whereas cash flows from investing activities are understood to vary considerably from year to year. Furthermore, cash flows from operating activities are more closely tied to profitability than are investing cash flows, which tend to be negative when a company is healthy and growing. Reclassifying cash outflows from operating to investing would make the company’s continuing operations appear more profitable while the increase cash outflows for investing would make the company appear to be enjoying positive growth.

46.

Cash flow from (or used) in operations is shown in the first section of the statement of cash flows under the caption “operating activities”. Analysts do not necessarily react negatively to negative cash flows from operations for emerging businesses. Negative cash flows during the introductory phase of a company’s life cycle are not uncommon. A favorable trend in cash flows, and prospects for positive cash flows as the business grows, would be important to the analyst.

46.

Cash flows from investing activities include cash inflows and outflows associated with the purchase and sale of productive assets (property, plant and equipment, and intangibles), including the acquisition of other businesses, and long-term investments in securities. Yahoo’s use of cash for investing activities indicates additional investment in assets, while Amazon appears to have sold more assets than it purchased.

46.

Cash flows from financing activities include proceeds from the issuance of common stock, so both companies would likely have had positive cash flows from financing activities during their early years.

CHARACTERISTICS OF END-OF-CHAPTER ASSIGNMENTS Item

Difficulty

Description

Brief Exercises: BE2–1 E BE2–2 E BE2–3 E BE2–4 E BE2–5 E

Dividends as a percentage of net income Financing assets Assessing solvency The statement of cash flows across time Financial statement formats - International

Exercises: E2–1 E2–2 E2–3 E2–4 E2–5 E2–6 E2–7 E2–8 E2–9 E2–10

Identifying financing, investing, and operating transactions Identifying financing, investing, and operating transactions Balance sheet or income statement account? Financial statements and the lending decision Retained earnings changes across time Retained earnings changes across time Using working capital to assess solvency Solvency, financing and debt covenants The statement of cash flows across time The statement of cash flows across time

M M E M M M M H M M


8

Chapter 2

3 E2–11 E2–12 E2–13 E2–14

H H H H

Preparing a statement of cash flows Preparing a statement of cash flows Preparing financial statements from simple transactions Preparing financial statements from simple transactions

Problems: P2–1 P2–2 P2–3 P2–4 P2–5 P2–6 P2–7 P2–8 P2–9 P2–10

E E M H M M H M M H

Classifying balance sheet accounts Classifying income statement accounts Financial statement interrelationships Preparing a balance sheet in proper form Balance sheet and income statement relationships across time Using financial statements to assess solvency and earning power Balance sheet value and the fair market values of the assets Analyzing financial statements Analyzing financial statements Debt covenants and financial statements

Issues for discussion: ID2–1 M ID2–2 H ID2–3 E ID2–4 H ID2–5 H ID2–6 H ID2-7 M ID2-8 M ID2–9 M ID2–10 M

Relationships among cash flows, income, and dividends Income statement analysis Debt covenants Statement of cash flow patterns across companies Statement of cash flow patterns across time Balance sheet—IFRS Net income vs. cash flow from operations Transactions and the financial statements IFRS versus GAAP The annual report of Google


CHAPTER 3 The Measurement Fundamentals of Financial Accounting

SYNOPSIS In this chapter, the author discusses the measurement fundamentals of financial accounting which consist of the basic assumptions, valuation issues, principles, and exceptions underlying the financial statements. Fundamental Differences between US GAAP and IFRS are discussed. The ethics vignette considers the case of an industry leader (Microsoft) which formerly used ultra-conservative accounting for revenue recognition and has recently switched to more aggressive revenue recognition accounting. The Internet research exercise directs the student to the SEC Edgar Database to obtain information about filings in general and specific information for a particular company, Microsoft. The following key points are emphasized in Chapter 3: 1.

Four basic assumptions of financial accounting.

2.

The markets in which business entities operate and the valuation bases used on the balance sheet.

3.

The principle of objectivity and how it determines the dollar values that appear on the financial statements.

4.

The principles of matching, revenue recognition, and consistency.

5.

Two exceptions to the principles of financial accounting measurement: materiality and conservatism.

6.

Fundamental differences between US GAAP and IFRS.

TEXT/LECTURE OUTLINE The measurement fundamentals of financial accounting. I. Assumptions of financial accounting. A.

The assumptions concern the business environment and provide a foundation for creating an accounting system.

B.

The basic assumptions of financial accounting.


Chapter 3

1.

2.

3.

4.

II.

Economic entity assumption. a)

Investors are interested in obtaining information about a particular profitseeking entity. Consequently, the economic activities of that entity must be able to be identified and separated from the economic activities of its owners and all other entities.

b)

Economic units versus legal units.

Fiscal period assumption. a)

The life of the entity can be broken into arbitrary time periods (fiscal periods) over which its performance and financial position can be measured.

b)

This assumption is necessary to provide timely performance measures to investors. However, there is a tradeoff between timely and objective financial information.

Going concern assumption. a)

The life of the economic entity is assumed to extend indefinitely beyond the current fiscal period.

b)

This assumption allows accountants to adhere to accrual accounting and to record assets and liabilities.

Stable dollar assumption. a)

An entity's performance and financial position is measured in monetary units. The monetary unit is assumed to possess stable purchasing power over time.

b)

The assumption is inconsistent with the reality of inflation. Consequently, this assumption leads to financial statements that "distort" a company's actual performance.

Valuations on the balance sheet. A. Four valuation bases to use in measuring an entity's performance and financial position. 1.

Present value—represents the discounted future cash flows associated with a particular financial statement item.

2.

Fair market value—represents the sales value in the output market.

3.

Replacement cost—represents the current prices paid in the input market.


Chapter 3

4.

Original cost—represents the input price paid when originally purchased by the company.

III. The principles of financial accounting measurement. A.

B.

Principle of objectivity. 1.

Financial accounting information must be verifiable and reliable. The value of a transaction (and the assets and liabilities arising from that transaction) must be objectively determined; individuals with conflicting incentives would agree on the value of the transaction.

2.

Evaluation of present value, fair market value, and original cost on the dimension of objectivity.

3.

Requirement for objective measures excludes items of value, such as accrued goodwill, from an entity's financial statements.

Matching principle. 1.

Costs are matched against the benefits that result from them.

2.

The first step in applying the matching principle is deciding when to recognize the benefit (i.e., revenue).

C. Principle of revenue recognition. 1.

Addresses the question of when to recognize revenue.

2.

Criteria for recognizing revenue. a) The company must have completed a significant portion of the production and sales effort. b) The amount of revenue can be objectively measured. c) The major portion of the costs has been incurred and the remaining costs can be reasonably estimated. d) The eventual collection of cash is reasonably assured.

D. Principle of consistency. 1.

Consistency refers to the use of the same accounting policies over time.

2.

GAAP encourages a company to use the same accounting policies to promote comparability of a company's performance over time.

3.

Accounting methods can and do change from time to time.


Chapter 3 IV. Two exceptions to the basic principles. A.

B.

Materiality. 1.

If the cost of providing financial information to investors exceeds the expected benefit they would derive from the information, then the company may record the event in the least costly method. In other words, an event is immaterial if the way in which it is recorded would not affect investors' evaluation and control decisions.

2.

Applying materiality requires a great deal of judgment.

Conservatism. Conservatism states that, when in doubt, financial statements should understate assets, overstate liabilities, accelerate the recognition of losses, and delay the recognition of gains; when there is significant uncertainty about a transaction’s value, the conservative alternative should be chosen.

V.

International perspective: Fundamental differences between US GAAP and IFRS

VI. Ethics in the real world. VII. Internet research exercise.

LECTURE TIPS 1.

It is important to tie the principles and assumptions back into the objective of financial accounting and to illustrate how the principles and assumptions affect accounting procedures. A highly recommended approach is to have students evaluate different companies using financial statements based on different assumptions and valuation bases.

2.

The use of illustrative cases may make it easier for students to understand the specific principles and assumptions.

OUTSIDE ASSIGNMENT OPPORTUNITIES Comparative study of annual reports: U.S. companies 1.

Obtain a recent annual report for two publicly held companies in the same industry. Read the financial statements and footnotes (particularly the accounting policy footnote) and compare the accounting principles employed by each in terms of the fundamental concepts developed in the chapter. Evaluate which of the two appears most conservative. Report your findings in writing and/or in a brief presentation to the class.


Chapter 3

Comparative study of annual reports: a continental European company compared to a U.S. company 2.

Obtain a recent annual report for both a continental European company and for a U. S. public company (preferably both in the same industry). Read the financial statements and disclosures and compare the accounting principles employed by each in terms of the fundamental concepts developed in the chapter. Explain the rationale behind differences between the two. Report your findings in writing and/or in a brief presentation to the class.

Research of accounting literature: international standards 3.

Form groups (3–5 students) to research current accounting literature for articles explaining the accounting model employed in a country other than the United States. Each group will research a different instructor-selected country. Compare the principles employed with those used in the United States and explain differences in terms of likely socio-economic levels in the country studied. Report the results in a class presentation. Follow up with an overall comparative summary by each group for all the selected countries.

Research of accounting literature: fair value accounting 4.

Research current accounting literature for articles covering fair value accounting and its expanding place in our accounting model. Focus on the impact on users of financial statements as well as the economic consequences effect on management decisions. For example, much has been written about the financial services industry, which has especially been affected by recent fair value accounting pronouncements. Prepare a written and oral report of the findings.

ANSWERS TO IN-TEXT DISCUSSION QUESTIONS 75.

Each of the various businesses within Illinois Toolworks, Inc. are companies within a company and have distinct needs to report separate measures of performance and financial position for any number of management, credit, tax, and regulatory purposes. Most firms operate with some degree of integration, so it may be difficult to account for transactions between the various businesses and to account for corporate costs and assets common to all members of the consolidated group.

76.

There is a trade-off between the timeliness of accounting information and its objectivity and credibility. Certain accounting information becomes less reliable and more prone to error because more arbitrary and subjective as the reporting period becomes shorter, and it has not yet been subjected to independent verification. Timeliness is often more important than precision to managers.

76.

Going concern is a fundamental assumption underlying our accounting model which uses historical costs as a valuation basis and classifies assets and liabilities and current and long-term. In making an evaluation of going concern status, auditors consider numerous factors such as negative trends (recurring operating losses, capital deficiencies, adverse financial ratios), other indicators of possible financial difficulties (default on loan agreements), and internal and external matters (labor difficulties, loss of a key customer or supplier). Not being able to obtain needed financing negatively impacts the businesses' prospects for remaining in business over the short term. Auditors are concerned with the viability of a business as a going concern, because if a company were not expected to stay


Chapter 3 in business beyond the current year, then liquidation values would be a more appropriate valuation basis, and current and long-term classifications would be meaningless. 77.

Proctor & Gamble’s claim of a 3 percent increase in sales is legitimate because of the stable dollar assumption.

78.

Financial statements are prepared on the stable dollar assumption, which assumes that the purchasing power of the dollar remains constant across fiscal periods, ie, no inflation. If inflation was significant to Wal-Mart, a user of the financial statements would want to assess the degree to which the financial statements were in fact distorted because inflation is ignored in the financial statements.

78.

An analyst attempting to compare companies using different currencies in their financial statements would need to convert the financial statements of one of the two companies into the currency of the other using the applicable exchange rates. This is easier said than done.

80.

The $9.3 billion value reported as vehicles on Avis’s balance sheet would represent original cost to Avis, ie, the input price paid when the vehicles were originally purchased. Present value of the vehicles could be computed by projecting future cash flows from operation and ultimately disposal of the vehicles, and discounting those cash flows back to a present value using time value of money techniques. Fair market value of the vehicles would be determined by what Avis could sell the vehicles for in the output market today. Replacement cost would be the cost Avis would have to pay in today’s input market to purchase the vehicles. Original cost is useful because it is objectively measurable and verifiable. Present value involves a great deal of estimation; in concept it does indicate the true economic value to the firm of the vehicles. Fair market value and replacement cost both represent established prices in the output and input markets, respectively, which would be obtainable from buyers and sellers. Fair market value and replacement cost would be most useful if Avis planned to sell off the fleet or replace it. However, neither possibility coincides with the planned use by Avis of the vehicles.

80.

Because there was no market for these assets, there was no market value. Cost is irrelevant. Management’s best estimate of the net realizable value of the assets would be more meaningful than either zero or cost.

82.

When one company buys another company (A buys B), the price A pays is based in large part on the financial statements of Company B including all of the assets and liabilities on the balance sheet of the company. Company B’s financial statements are materially influenced by the account valuations determined by Company B so the price paid by Company A will be impacted by the valuation of B. Company A should also complete due diligence and review Company B’s valuation to ensure the financial statements are correct and all valuations are understood.

82.

The greatest dollar amount of Google’s assets consists of current assets – cash and cash equivalents, marketable securities, receivables, inventory, deferred income taxes and prepaids and other assets. Cash is valued at face, marketable securities at market, receivables at net realizable value, inventory at the lower of cost or market, deferred income taxes at net value, prepaids are carried at their remaining carrying value which is


Chapter 3

based on their cost. For long term assets, Non-marketable equity securities are carried under the equity or cost method. Property, plant and equipment, intangible assets and goodwill are all valued at historical cost (net of amortization or depreciation). 83.

Any attempt to value intellectual property rights would be inherently subjective. An important principle of accounting measurement is the principle of objectivity, which requires that for accounting information to be reliable, it must be verifiable (ie, determined, usually in an arm’s-length transaction, and backed by documentary evidence). Intellectual property rights that are purchased are valued at cost (net of amortization) but those developed internally do not appear on balance sheets.

84.

The proposed new standard would use uncertain estimates of future cash flows to estimate fair value where observable market prices are not available. Such a standard has the advantage of providing economic value information consistent with an entity’s expectations of future use of the asset, but does involve complex judgments and gives imprecise results.

84.

Under US GAAP the principal of objectivity demands that property, plant and equipment be recorded at historical cost. Fair market value is a subjective determination and is useful information and has not been adopted by GAAP for incorporation into the financial statements because of appraisal costs and concerns about manipulation of net income with the effects of valuation changes. Because IFRS allows the use of fair market value in certain situations for valuation for intangible assets and for property, plant and equipment, one can infer that the principle of objectivity is less important under IFRS than it is under US GAAP.

85.

Eli Lily will keep this property and equipment at cost on the balance sheet and depreciate each item over its useful life. These items will benefit the company and help it produce revenue over a period of time so the related expenses should be recognized over this time period as well (matching principle). Expenses are matched to the time period where they are incurred, they help produce revenue.

86.

The change had the effect of spreading revenue over several future years rather than all in the first year when the contracts were signed, thereby reducing current income and increasing future income. Spreading the revenue is a better example of the matching process. Certainly the company will have costs in future years to service the contracts and spreading the revenues forward to those future years provides for a better matching of the cost of the efforts with the benefits.

87.

The cumulative effect of such an accounting change would be shown in the year of the change as a separate line item in the income statement. Additionally, the change would be described in footnotes to the financial statements and highlighted in the auditor’s report.

88.

A quantitative analysis of materiality is based solely on the size of the item in dollar terms relative to some base. The SEC noted that many companies and auditors consider 5% of income as a materiality threshold, i.e., items greater than that amount would affect the judgment of a reasonable user of the financial statements. The SEC implored companies and auditors to also consider the qualitative effect of an item, even if otherwise small. Such qualitative effects include changes in earnings or other trends, compliance with loan covenants or contractual agreements, unlawful acts, and so on.


Chapter 3

CHARACTERISTICS OF END-OF-CHAPTER ASSIGNMENTS Item

Difficulty

Description

Brief exercises: BE3–1 E Accounting assumptions, principles, and exceptions Exercises: E3–1 E E3–2 E E3–3 E E3–4 M E3–5 M E3–6 M E3–7 E E3–8 E E3–9 H E3–10 M

The effects of inflation on cash The effects of inflation on land Valuation bases on the balance sheet Fundamentals of inventory valuation Revenue recognition Different methods of revenue recognition Assets and depreciation The concept of materiality Changing accounting methods and net income Adjusting for market value – GAAP vs. IFRS

Problems: P3–1 E P3–2 E P3–3 M P3–4 M P3–5 H P3–6 M P3–7 M P3–8 H P3–9 H P3–10 H P3–11 M P3–12 M P3–13 M

The effects of inflation on reported profits Inflation and bank loans The irrelevance of original cost Stable Dollar Assumption Comparing across currencies and accounting systems Economic Value versus Book Value Economic Value versus Book Value--and income Present Value vs. Book Value vs. Liquidation Value Three different measures of income Comparing companies using different accounting methods Different methods of recognizing revenue Revenue recognition and net income Conservatism Exception

Issues for discussion: ID3–1 M Revenue recognition and matching ID3–2 E Aggressive revenue recognition ID3–3 M Revenue recognition ID3–4 H Consistency and accounting changes ID3–5 M Comparability across time ID3–6 E Comparability ID3–7 M Matching and mismatching ID3–8 M Earnings restatements and fiscal period, matching, and consistency ID3–9 M Inventory recoveries under IFRS ID3–10 M Income management and conservatism ID3–11 M Economic entity ID3-12 H Valuation bases ID3-13 M Fair Value accounting ID3-14 H The annual report of Google


CHAPTER 4

The Mechanics of Financial Accounting

SYNOPSIS This chapter covers the mechanics underlying preparation of financial statements and how they help to ensure that a company’s transactions are accurately and completely accounted for. After completing this chapter, students should be able to construct financial statements from economic events. The author discusses economic events and the criteria necessary before such events can be reflected in the financial statements. The author describes the fundamental accounting equation and explains its relationship to the economic events that are reflected in the financial statements. The author explains the use of journal entries and T-accounts to capture the effects of economic events on the accounting equation and the financial statements. The author also describes how and why periodic adjustments to financial statements are made to reflect economic events that are not represented by transactions. Some of the reporting issues faced by multinationals are discussed. This chapter covers the mechanics of the financial accounting process from an economic consequences perspective, emphasizing that managers must understand the mechanics that link their choice of transactions to the financial statements. Appendix 4A further covers the mechanics from a financial statement user’s perspective, explaining how managers in their role as users can infer from financial statements the events and transactions that occurred during the accounting period. Appendix 4A describes how the statement of cash flows can be prepared by using of T-account analysis. The ethics vignette addresses the subjectivity involved in decisions to capitalize or expense costs. The Internet research exercise explores accounting for the United States federal government. The following key points are emphasized in Chapter 4: 1.

Two criteria necessary for economic events to be reflected in the financial statements.

2.

The accounting equation and how it relates to the balance sheet, income statement, statement of shareholders’ equity, and statement of cash flows.

3.

Journal entries (and T-accounts) and how they express the effect of economic events on the basic accounting equation and the financial statements.

4.

Why managers need to understand how economic events affect the financial statements.

5.

Why the financial statements are adjusted periodically to reflect certain economic events.

LECTURE/TEXT OUTLINE


Chapter 4– Financial Accounting in an Economic Context – Instructor’s Manual

The mechanics of financial accounting. I.

II.

Economic events—must be both relevant and objectively measurable to be included in the financial statements. A.

Relevant means that the event has economic significance to the company.

B.

Objectively measurable means that parties with differing incentives reach agreement on the value of the event and that those events are backed by documentary evidence which can be audited.

The fundamental accounting equation. A.

Assets = Liabilities + shareholders' equity.

B.

Business transactions, the accounting equation, and the financial statements. 1.

Transactions and the accounting equation.

2.

The accounting equation and the financial statements.

III. The journal entry. A.

A journal entry records the economic event in a journal that thereby enters the event in the company's accounting records.

B.

The double entry system. 1.

Debit is defined as the left-hand side.

2.

Credit is defined as the right-hand side.

C. All journal entries have three components: 1.

The accounts affected.

2.

The direction of the effect.

3.

The dollar value of the transaction.

D. T-accounts are useful for keeping running tallies of balances for each account. E.

Financial statements can be prepared from T-accounts.

IV. Recognizing gains and losses. V.

Periodic adjustments. A.

Periodic adjustments are necessary because not all economic events that occur during the accounting period are captured by an exchange transaction in the same accounting period. Accrual accounting requires that all economic events that have occurred during the accounting period be recognized to achieve a proper matching of revenues and expenses.


Chapter 4– Financial Accounting in an Economic Context – Instructor’s Manual

B.

Periodic adjustments take one of three forms. 1.

Accruals. a)

Accrue means to build up gradually. Accrual adjustments recognize economic events that have occurred in the current accounting period but have not yet been captured in an exchange transaction.

b)

Accrued expenses.

c)

2.

3. VI.

(1)

Accrued expenses are expenses that have been incurred but not yet paid.

(2)

The appropriate journal entry would be to debit an expense account and credit a liability account.

Accrued revenues. (1)

Accrued revenues are revenues that have been earned but cash collection has not yet occurred.

(2)

The appropriate journal entry would be to debit a receivable account and credit a revenue account.

Deferrals (or cost expirations). a)

Asset capitalization and the matching principle.

b)

Expense or capitalize examples.

c)

Current assets. (1)

Supplies inventory.

(2)

Merchandise inventory.

(3)

Prepaid expenses.

(4)

Unearned (deferred) revenues.

(5)

Property, plant and equipment.

(6)

Intangible assets

(7)

Capitalizing and matching: examples

Revaluation adjustments.

Reporting difficulties faced by multinational companies.


Chapter 4– Financial Accounting in an Economic Context – Instructor’s Manual

VII. T-Account analysis and preparing the statement of cash flows (Appendix 4A). A.

Operating items.

B.

Non-operating items.

C. Indirect form of presentation.

VIII. Review problem.

IX. Ethics in the real world. X.

Internet research exercise.

LECTURE TIPS

1.

Particular attention should be given to helping students in deciding whether an economic event has accounting significance and should be recorded. It would be worthwhile to spend some time covering the concepts of relevant and objectively measurable. Further, discussing different economic events that would be recorded (such as purchasing inventory) or would not be recorded (such as signing a new union contract) in a journal entry would be worthwhile.

2.

Often students need extra help understanding how to record economic events. In particular, students need help to understand (1) which accounts have debit and credit balances and (2) how debits and credits affect particular accounts. A useful approach is to break the recording process down into two distinct steps. The first step is to have students record economic events on the accounting equation (similar to Figure 4–2) expanded to include revenue, expense, and dividend accounts. This step prepares them (1) to work with revenue, expense, and dividend accounts, which are difficult for many students to grasp, and (2) to address which accounts are affected and how the accounts are affected without having to worry about debits or credits. The second step is to have students prepare journal entries for the transactions they recorded in the first step. This step allows students to focus exclusively on the debits and credits.

3.

Students need help in understanding what each account represents and measures to be able to analyze economic events and prepare the appropriate journal entry. A written assignment or inclass presentation that requires them to explain what a particular account balance represents is recommended.

4.

Many introductory students have difficulty differentiating between cash collections and revenue, and between cash disbursements and expenses. It is critical that these differentiations are made when discussing revenues and expenses so that students will be able to correctly analyze economic events and prepare journal entries.

5.

Extra effort is often needed to help students understand adjusting entries. The key to this is the students’ understanding of the relationship of specific income statement accounts to specific balance sheet accounts. A suggested method for helping students understand how specific accounts on these two statements fit together is to have students create T-accounts for balance sheet and income statement accounts. They should then indicate what the beginning and ending balances in each account represent, what economic events cause various accounts to increase or decrease, and which economic events affect more than one account. It is this last step that


Chapter 4– Financial Accounting in an Economic Context – Instructor’s Manual

should help students better understand the relationships between income statement and balance sheet accounts. This exercise is also very useful in helping students understand which accounts have permanent balances and which ones have temporary balances; why some accounts have permanent balances; and what each account balance represents. Other potential ways to help students understand the relationships between specific income statement and balance sheet accounts are listed below: a. Presenting several examples of each type of adjustment. b. Discussing the effects on the financial statements of not making a required adjusting journal entry. c. Converting from accrual accounting amounts to cash accounting amounts or vice versa. It is also important for students to understand why adjusting entries are necessary. This understanding comes when students truly comprehend the difference between accrual accounting and cash-basis accounting. Spending time illustrating the differences using examples relevant to the students is strongly recommended. 6.

Students need help in grasping the rationale for closing entries. Time should be spent discussing why temporary accounts are used rather than recording events affecting revenues, expenses, and dividends directly in the retained earnings account. A particularly useful approach is to have students record economic events on the accounting equation (similar to Figure 4–2) expanded to include revenue, expense, and dividend accounts. After recording the events, point out that assets do not equal the sum of liabilities and shareholders' equity unless the balances in the temporary accounts are considered.

OUTSIDE ASSIGNMENT OPPORTUNITIES Additional assignments covering the accounting cycle 1.

Develop the understanding of the mechanics of the financial accounting cycle through additional outside self-study and assignments available in supplements which complement the text. INTACT, by Dasaratha Rama, is an accounting cycle on-line and/or CD-ROM product that contains tutorials, practice assignments, and student instructions. The Accounting System: A Self-Study Primer for Introductory Accounting, by William Ruland, is a self-paced workbook that helps those with little or no prior exposure to the accounting system and accounting mechanics. Such outside assignments could be spaced over several weeks following coverage of Chapter 4, and would leave class time free to focus on the conceptual issues without sacrificing an understanding of the mechanics. It also meets the objective of utilizing computer technology.

Simulation game 1

2.

Obtain the Albrecht article (referenced below) that describes an accounting and investment simulation game. Play the game and complete the written assignments as instructed therein.

1

David W. Albrecht, “A Financial Accounting and Investment Simulation Game,” Issues in Accounting Education, Spring 1995, pp. 127–141.


Chapter 4– Financial Accounting in an Economic Context – Instructor’s Manual

ANSWERS TO IN-TEXT DISCUSSION QUESTIONS Page No. 106.

Yes, the pharmaceuticals and the software companies become more valuable when critical events occur such as those described. Nonetheless, those events are not recorded in the financial statements, because they have not yet reached the point where their impact on the financial statements can be objectively measured.

107.

The purchase of inventory by a retailer from a supplier ordinarily results in an increase in the current asset, inventory, and a corresponding increase in the current liability, accounts payable.

108.

The basic accounting equation applied to the balance sheet of Amazon is: Assets = Liabilities + Contributed Capital + Retained Earnings $32,555 = $24,363 + $6,515 + $1,677.

109.

Coca Cola’s purchase of $2.8 billion of property, plant, and equipment would both increase and decrease assets by the same amount, because property, plant, and equipment would increase and cash would decrease. The borrowing of $4.2 billion would increase an asset, cash, and increase a liability, notes payable.

112.

Target’s balance sheet, with total assets of $48,163 (million) and total liabilities and shareholders’ equity of $48,163 (million) epitomizes the accounting equation (Assets = Liabilities + Shareholders’ Equity) The statement of cash flows summarizes the increases and decreases in cash for a period of time but has little direct relationship to the accounting equation. The accounting equation, however, is used extensively in the analysis necessary to prepare the statement of cash flows. The income statement summarizes the effects of transactions for a period of time on the retained earnings as reflected in the equity section of the balance sheet. The statement of shareholders’ equity summarizes the increases and decreases in the equity accounts for a period of time. It is useful to understand that the balance sheet which reflects the accounting equation as of a specific point in time, is linked to other balance sheets (as of other specific points in time – i.e. the beginning and end of an accounting period), by the income statement, the statement of cash flows and the statement of shareholders’ equity. The four statements together provide a pretty complete picture of all the economic activity of the entity for a period of time.

114.

Revenue of $46.1 (billion) at Cisco would increase assets, specifically accounts receivable, and correspondingly increase shareholders’ equity, specifically retained earnings. The journal entry (in billions) would be:

Accounts receivable (+A) Revenue (R, +SE)

DR 46.1

CR 46.1


Chapter 4– Financial Accounting in an Economic Context – Instructor’s Manual

119.

The loss arises because the (adjusted) cost of the asset sold was greater than the amount realized from the sale.

120.

The transaction could be summarized (dollars in millions) as follows: DR CR Cash (+A) 53.3 Investment in subsidiary 27.6 Gain on sale of subsidiary (R, +SE) 25.7

123.

Accrued liabilities of $7.2 billion at Honeywell represent expenses incurred, but not yet paid by year-end 2012. Typical examples include wages and interest.

124.

Capitalization of routine maintenance expenses violates the matching principle because this type of expense should be matched against the current revenues of the period and should reduce net income. Capitalization of costs suggests they will benefit future periods and should be matched against future revenues. By capitalizing such expenses management seriously (and fraudulently) overstated net income. Routine maintenance expenses are always expensed (under GAAP) in the period in which they are incurred.

128.

The use of different accounting principles, US GAAP as opposed to IFRS, means that in order to compare apples to apples, an analyst would need to convert one set of financial statements into the accounting method used by the other.

129. Journal entries would be as follows (billions): Accounts receivable (+A) Sales (R, +SE) To record revenues

DR 22.4

CR 22.4

Interest expense (E,–SE) Cash(-A) Interest costs on debt (current period expense)

.462

Inventory (+A) Accounts payable (+L) Inventory purchases on account

4.4

Property, plant, and equipment (+A) Cash (-A) Purchase PPE (future period asset)

.388

Depreciation and amortization (E,–SE) Accumulated depreciation and amortization (–A) Adjust for useful life expired

1.3

.462

4.4

.388

1.3


Chapter 4– Financial Accounting in an Economic Context – Instructor’s Manual

CHARACTERISTICS OF END-OF-CHAPTER ASSIGNMENTS Item Difficulty Brief exercises: BE4–1 E BE4–2 E BE4–3 E Exercises: E4–1 E E4–2 E E4–3 E E4–4 M E4–5 E E4–6 E E4–7 M E4–8 E E4–9 M E4–10 M E4–11 M E4–12 M E4–13 M E4–14 M E4–15 M E4–16 M E4–17 M E4–18 E4–19 E4–20 E4–21 E4–22 E4–23

H H H H H H

Problems: P4–1 P4–2 P4–3 P4–4 P4-5 P4–6

E E M M H M

P4–7 P4–8 P4–9 P4–10 P4–11 P4–12 P4–13 P4–14 P4–15 P4–16

M M H H M H H H H H

Description Effects of transactions on the accounting equation Effects of transactions on the accounting equation Effects of transactions on the accounting equation Effects of transactions on the accounting equation Effects of transactions on accounts Preparing the financial statements Preparing the financial statements Accounting significance Preparing financial statements Preparing financial statements Preparing statement of cash flows from the cash ledger Preparing statement of cash flows from journal entries Preparing the financial statements Preparing statement of cash flows from the cash T-account Classifying adjusting journal entries Classifying transactions Recognizing accrued wages Depreciating a fixed asset The difference between accrual and cash accounting The difference between net income and net cash flow from operations Cash versus Accrual Accounting Preparing a statement of cash flows from original transactions Assessing economic consequences T-account analysis Depreciation and cash flows Preparing a statement of cash flows—direct and indirect method of presentation Journal entries and the accounting equation T-accounts and the accounting equation Journal entries and preparing the four financial statements Preparing the four financial statements Comprehensive problem-preparing the financial statements Effects of transactions on the income statement and statement of cash flows Effects of adjusting journal entries on the accounting equation Preparing adjusting journal entries Inferring adjusting journal entries from changes in T-accounts Reconciling accrual and cash flow dollar amounts Revenue recognition, cost expiration, and cash flows Effects of transactions on financial ratios Effects of financing on financial ratios Effects of events on financial ratios Effects of events on financial ratios Effects of events on financial ratios


Chapter 4– Financial Accounting in an Economic Context – Instructor’s Manual

Item Difficulty Problems continued: P4–17 H P4–18 M P4–19 M P4–20 M Issues for discussion: ID4–1 M ID4–2 E ID4–3 E ID4–4 E ID4–5 M ID4–6 E ID4–7 M ID4–8 M ID4–9 M ID4–10 M ID4–11 M ID4–12 M

Description Comprehensive problem-preparing the financial statements T-accounts analysis T-accounts analysis T-accounts analysis Transaction’s effect on the accounting equation and balance sheet Effects of transactions on the accounting equation Effects of transactions on the accounting equation Cash flows and business failures Corporate frauds and the auditor Cash flow Income statement classification-IFRS Problems with the federal government’s accounting systems Debt transactions and the basic accounting equation Consolidated financial statements and multinationals Real-time accounting The annual report of Google


CHAPTER 5

Using Financial Statement Information

SYNOPSIS In this chapter, the author discusses ways in which information contained within financial statements may be used to evaluate a company. The author discusses how financial statements can be useful in (1) helping investors and creditors influence and monitor the business decisions of a company's managers, and (2) helping to predict a company's future earnings and cash flows. The discussion on using the financial statements focuses on the elements of financial statement analysis: (1) assessing the business environment; (2) reading and studying the financial statements and footnotes; (3) assessing earnings quality; (4) analyzing the financial statements; and (5) predicting future earnings and/or cash flows. The author discusses ways to analyze a company’s financial statements (through ratio analysis) and emphasizes the importance of comparisons across time and within the industry. Comparisons within the financial statements are covered in detail and include (1) common-size financial statements; (2) profitability ratios; (3) leverage ratios; (4) solvency ratios; (5) asset turnover ratios; and (6) other ratios. The chapter includes a thorough discussion on the limitations of financial statements and an overview of financial statement analysis in an international setting. Appendix 5A contains a framework designed to help in the analysis of ratios as a package (called the DuPont model). It also describes the basics of cash flow analysis. The ethics vignette considers four business practices in the internet world that are possible violations of ethical business behavior. The Internet research exercise directs the student to consider the usefulness of information provided to analysts by financial information services. The following key points are emphasized in Chapter 5: 1.

Using financial accounting numbers to influence management decisions and predict future events.

2.

Five steps of financial statement analysis.

3.

Assessing the business environment.

4.

Assessing earnings quality and persistence.

5.

Analyzing financial statements.

6.

Difficulties involved in using annual report information to identify mispriced securities.

7.

Difficulties involved in using financial statements to compare the performance of companies operating in different countries.


Chapter 5

LECTURE/TEXT OUTLINE Using financial statement information. I.

Control and prediction -usefulness of financial accounting numbers. A.

B.

Influencing and monitoring business decisions of a company's managers. 1.

Equity investors can influence management’s business decisions by basing a large portion of management’s compensation on reported profits.

2.

Debt investors can influence management’s business decisions through debt covenants.

Framework for using financial statements to predict future earnings and cash flows. 1.

Equity investors use financial information to predict future earnings and cash flows in their efforts to identify securities that will provide high returns.

2.

Creditors use financial information to predict whether companies can generate enough cash in the future to cover debt payments.

3.

Future cash flows are the heart of a company's true value, which is of interest to both investors and creditors. The following three reasons can cause a company's reported book value and true value to differ: a)

Business environment.

b)

Unrecorded events.

c)

Management bias.

II. Elements of financial statement analysis. A.

B.

Assessing the business environment. 1.

Learn about the company, its industry, and how these relate to the overall economy.

2.

Other experts (Moody’s, Value Line, Dun & Bradstreet, Standard & Poors, The Wall Street Journal) provide ready insight.

Reading and studying the financial statements and footnotes. 1.

The audit report.

2.

Significant transactions and important segments.

3.

The financial statements and footnotes.


Chapter 5 C. Assessing earnings quality. 1.

Quality of earnings—the degree to which a company’s reported earnings diverge from its “true” operating earnings because management exercised its discretionary influence over reported accounting numbers.

2.

Ways for management to “manage” financial accounting numbers.

3.

a)

Overstating operating performance.

b)

Taking a bath.

c)

Creating hidden reserves.

d)

Employing off-balance-sheet financing.

Assessing earnings quality also includes considering unrecorded events and the other inherent limitations of financial statements. a)

The value of human resources (i.e., human capital) is not included in the financial statements.

c)

The value of intangibles in technology-oriented companies is not reported in the financial statements.

d)

Few market values are used in the financial statements.

D. Analyzing the financial statements.

E.

1.

Comparisons across time.

2.

Comparisons within the industry.

3.

Comparisons within the financial statements: common-size statements and ratio analysis.

Common-size financial statements. 1.

Financial statement numbers are expressed as percentages of other financial statement numbers.

2.

Benefits of common-size financial statements. a) Provides relative rather than absolute comparisons. b)

F.

Helps indicate why changes occur in financial performance and condition.

Financial ratios. 1.

Ratio analysis is based on the comparison of two or more financial statement numbers.


Chapter 5 2.

Categories of ratios. a)

Profitability ratios. (1)

Profitability ratios are useful in assessing a company’s earning power.

(2)

Different profitability ratios. (a) Return on equity. i)

Ratio of net income to average shareholders’ equity.

ii)

Measures the efficiency with which a company manages its shareholders’ investments.

(b) Return on assets. i)

Ratio of the sum of net income and after-tax interest expense to average total assets.

ii)

Measures the efficiency with which a company manages both its shareholders’ and its creditors’ investments.

(c) Return on sales (profit margin).

b)

i)

Ratio of the sum of net income and after-tax interest expense to net sales.

ii)

Provides information on a company’s ability to generate and market profitable products and control its costs. It reflects the number of cents in profit for every dollar of sales.

Leverage ratios. (1)

Leverage ratios provide information useful in evaluating a company’s capital structure.

(2)

Different leverage ratios. (a) Common equity leverage. i)

Ratio of net income to the sum of net income and after-tax interest expense.

ii)

Compares the return available to the shareholders to the returns available to all capital providers.

(b) Capital structure leverage.


Chapter 5 i)

Ratio of average total assets to average shareholders' equity.

ii)

Measures the extent to which a company relies on borrowings (liabilities).

(c) Debt/equity ratio. i)

Ratio of average total liabilities to average shareholders' equity.

ii)

Another way to measure capital structure leverage.

(d) Long-term debt ratio.

c)

i)

Ratio of long-term liabilities to total assets.

ii)

Indicates the relative importance of long-term liabilities as a source of asset financing.

Solvency ratios. (1)

Solvency ratios are useful in assessing a company’s ability to meet its debts.

(2)

Different solvency ratios. (a) Current ratio. i)

Ratio of current assets to current liabilities.

ii)

Provides a measure of a company’s ability to cover its current liabilities with current assets.

(b) Quick ratio. i)

Ratio of quick assets (i.e., cash, marketable securities, and accounts receivable) to current liabilities.

ii)

Provides a measure of a company’s ability to cover its current liabilities with cash-like assets.

(c) Interest coverage ratio. i)

Ratio of income before interest and taxes plus tax expense and interest expense to interest expense.

ii)

Measures a company’s ability to meet its interest charges through operations.

(d) Accounts payable turnover.


Chapter 5

d)

i)

Ratio of cost of goods sold to average accounts payable.

ii)

Measures how quickly, on average, suppliers are paid off.

iii)

Dividing this ratio into 365 days indicates the number of days, on average, that accounts payable balances remain outstanding.

Asset turnover ratios. (1)

Asset turnover ratios provide a measure of the speed with which assets move through operations.

(2)

Different asset turnover ratios. (a) Receivables turnover. i)

Ratio of net credit sales to average accounts receivable.

ii)

Indicates the number of times receivables are recorded, collected, and recorded again each year.

(b) Inventory turnover. i)

Ratio of cost of goods sold to average inventory.

ii)

Indicates the number of times inventory is replaced each year.

(c) Fixed assets turnover. i)

Ratio of sales to average fixed assets.

ii)

Indicates the speed with which fixed assets are used up.

(d) Total asset turnover.

(3)

e)

i)

Ratio of sales to average total assets.

ii)

Indicates the speed with which all assets are used up in operations, aggregating the turnover measures of the component assets in (a) to (c).

Each turnover ratio can be converted to "days" by dividing it into 365 days.

Other ratios.


Chapter 5 (1)

Other ratios provide measures of returns to common shareholders due to changes in the market price of common stock and the receipt of dividends.

(2)

Most important other ratios. (a) Earnings per share. i)

Ratio of net income to the average number of common shares outstanding.

ii)

Standardizes a company’s income, which makes it easier to make comparisons across companies. It provides a measure of a company’s profitability strictly from the common shareholders’ viewpoint.

(b) Price/earnings (P/E) ratio. i)

Ratio of market price per share to earnings per share.

ii)

Indicates the sensitivity of a company’s stock price to changes in its earnings.

(c) Dividend yield ratio. i)

Ratio of dividends per share to market price per share.

ii)

Measures the cash return on shareholders’ investments.

(d) Stock price return. i)

Ratio of the sum of price appreciation per share and dividends per share to market price per share at the beginning of the period.

ii)

Measures the rate of return on common shares for the period.

III. Predicting future earnings and/or cash flow. IV. Annual report information and predicting stock prices. V. Financial statement analysis in an international setting. VI. Shareholder value, ROE, and cash flow analyses (Appendix 5A). A.

Cost of equity and return on equlty.

B.

Determinants of value creation: analyzing return on equity.

C. Shareholder value creation and the ROE model: Macy’s vs. Kohls.


Chapter 5

D. Cash flow analysis (solvency assessment). 1.

Operating performance.

2.

Financial flexibility.

3.

Liquidity.

E.

Cash flow profiles.

F.

Predicting future financial statements.

VII. Review problem VIII. Ethics in the real world. IX. Internet research exercise.

LECTURE TIPS 1.

Students must avoid getting bogged down in computing ratios and must understand what the ratios mean. The purpose of each ratio should be thoroughly discussed and illustrated. Further, it should be stressed that there are no “correct” formulas; the actual formula for a ratio will vary depending upon the needs of the person performing the analysis.

2.

Students often try to analyze a company using a single company’s ratios at a particular point in time. It should be stressed that analyzing a company only makes sense across time, across companies, or both.

OUTSIDE ASSIGNMENT OPPORTUNITIES Financial analysis project (longer term assignment) 1.

Form groups (3–5 students) and select a “target” public company and a comparable company in the same industry; obtain the annual reports and 10-Ks for both. Compute the ratios covered in the text (summarized in Figure 5–3) for both companies for as many periods as possible with the most recent reports. Prepare graphic presentations of selected ratios. Obtain and summarize two substantive articles on the “target” company from the financial press. Obtain comparative market statistics for the companies selected, as well as for the entire industry and the market as a whole.


Chapter 5 Use the information gathered to evaluate the target company as a candidate for (1) a shortterm loan; (2) a bond investment; and (3) an equity investment. Prepare a written report and make an oral presentation to the class. Submit the written report for instructor feedback before the final report and presentation. Use the computer for all assignments—the internet to access annual reports, 10-Ks, market data, etc.; programs for word-processing and spreadsheets for text, computations, and graphic presentations. Article search: “managed” financial statements 2.

Search the current financial press (The Wall Street Journal, Barrons, Fortune, Forbes, etc.) for an article relating to a company that may have “managed” its financial statements. Report your findings in writing and/or in a brief presentation to the class.

Application of other analytical techniques: bankruptcy prediction model 3.

1

Obtain and read the Altman article on bankruptcy prediction referenced below. Using the financial statements in Appendix A, compute the Z-score (discussed in the referenced article) for Wal-Mart and interpret the result in a written report.

Application of other analytical techniques: cash flow ratios 4.

2

Obtain and read the Giacomino-Mielke article on cash flow ratios referenced below. Using the financial statements in Appendix C, compute the nine ratios (presented in the referenced article) for Google on a comparative basis for two years and evaluate the results.

ANSWERS TO IN-TEXT DISCUSSION QUESTIONS 165.

The short term business plans of start-up companies normally recognize the fact that it may take some time for a new venture to become profitable. Nevertheless, financial accounting numbers are still as important for control and prediction for start up companies as they are for established companies. Investors with appetites for high-risk investments offering potential high returns are often attracted to start-ups. Investors' interest in an unprofitable start-up will diminish if the anticipated revenues and cash flows do not appear. Comparisons of planned to actual revenues and cash flows are of major importance to start-ups because they demonstrate whether the plan to become profitable is working or not.

166.

The stock market is always forward-looking; it considers the future prospects in the valuation of a company. Historical earnings are used to help predict future earnings and cash flows. The marketplace considers the business environment in which companies operate. Many companies such as Amazon.com, during the introductory phase of their life cycle, reported losses which were not representative of their expected future profits. In the introductory phase, companies generate new products and services through

1 2

E. I. Altman, “Financial Ratios, Discriminant Analysis and the Prediction of Corporate Bankruptcy,” The Journal of Finance, September 1968, pp. 589–609. Don E. Giacomino and David E. Mielke, “Cash Flows: Another Approach to Ratio Analysis,” Journal of Accountancy, March 1993, pp. 55–58.


Chapter 5 research and development, develop markets through advertising and marketing, and build capacity through capital expenditures. Costs associated with those efforts may result in current losses, but lead the way to future profits and cash flows which speak to the true value of the company. 166.

Intellectual capital (intangible assets) drives many of today’s service and technology businesses, which constitute an increasingly greater portion of our economy. Our accounting model excludes the value associated with intellectual capital, simply because it is not measurable in an objectively verifiable transaction. Some observers believe traditional financial accounting sacrifices the relevancy of information about intellectual capital in favor of objectivity and reliability. Our transactions-based accounting model was designed years ago to measure tangible capital in an industrial-based economy, and not intellectual capital in today’s knowledge-based economy. Consequently, the book value of these companies has little relationship to their market values. The present value of projected future cash flows usually provides a more relevant indication of the price an investor would be willing to pay for an investment in a company having significant intellectual capital. Time will tell whether Rupert Murdoch's opinion of value was more correct than the market's. There has been much speculation about Murdoch's motivations for offering such a premium for the stock in Dow Jones. Some of his motives may not have been purely economic.

167.

The present value of projected future cash flows provides relevant indication of the value of an investment in a company. Predictions about future cash flows are outside the scope of generally accepted accounting principles. GAAP is rooted in historical, verifiable fact, where future predictions are only that – guesses about what may happen in the future. Pro-forma numbers based on reasonable expectations are very useful but need to be taken with a grain of salt.

168.

Apple Computer's strategic shift toward providing services in addition to manufacturing traditional hardware will, over time, result in a higher return on assets, more revenues and expenses on the income statement, and better operating cash flows. If the growth into the new areas can be accomplished without incurring debt or raising capital, Apple should see improvements in earnings per share, debt to equity, return on equity as well.

170.

Analysts are very much concerned with earnings persistence, which is the extent to which an income statement item reported in the current period can be expected to reflect future earnings. The “icing on the cake” represents significant one-time transactions, such as Motorola’s gain on the sale of its automotive division, which may not be expected to recur. The “cake itself” represents core earnings from operations which may be more indicative of future earnings. An analyst is probably more interested in the net income generated by the company’s recurring operations.

171.

The term “aggressive accounting” suggests overstatement of revenues and assets and/or understatement of expenses and liabilities. In other words, “aggressive accounting” is the opposite of conservatism. Because the application of generally accepted accounting principles necessarily involves a great deal of discretion and judgment, it is often difficult to draw the line between that which violates GAAP and that which conforms to GAAP. Accounting that flirts with the idea of defying this imaginary line is what is meant by “aggressive accounting”.


Chapter 5 171.

“Front loading expenses” is a form of “taking a bath” (see p 172). Expenditures that benefit future periods are mis-matched against current revenues. This distorts current year’s net income by making it look worse than it really is. Management hopes that by not recognizing these expenses in future years, the financial statements for those years will be improved. The Business Week article includes several examples including items described in financial statements as “restructuring charges”, and write-offs of “inprocess” research.

172.

“Earnings persistence” is an important concept that analysts use to refer to the extent to which an income statement item reported in the current period, such as net income, can be expected to relate closely to future income amounts and be useful in predicting them. A company producing consistent levels of net income over a period of years is more highly valued than an equally profitable company whose net income fluctuates wildly from year to year because the market places a premium on predictability. Management frequently uses it’s discretion to “smooth” earnings from one year to the next to make them look less volatile. The most common areas for the use of management discretion in financial reporting are areas where the numbers in the financial statements reflect management’s estimates. Some examples include allowances for uncollectible accounts, reserves for warranty costs, impairments of goodwill, contingent liabilities, depreciation, and amortization.

172.

Although the legal form of a lease commitment may indicate a rental transaction, the lessee may take on enough of the risks and rewards of ownership, that the lease may in substance be viewed as an asset purchase and related financing. In such a case, the property right under the lease would be recorded as an asset, and the obligation would be recorded as a liability. The income statement would reflect depreciation of the asset and interest on the obligation, rather than rent expense. Issues involved with this important example of off-balance-sheet financing are discussed in Chapter 11.

173.

The information about the banks returning to the labor market to hire back laid off investment bankers and traders would not have had a significant immediate impact on their financial statements. Perhaps an increase in compensation expense would cause net income to decline, but it would take some time for this to have an impact. More significant is the message being sent to the market that the banks were targeting new growth. This information would have already been available to the market and priced into the future prospects for the banking industry, on the basis of the previously reported return to profitability. If the market thinks this new growth and profitability is likely to result in sustained success there may be a positive impact on the market price of banks' stocks. A change in the stock price would not affect the financial statements.

176.

Eli Lilly’s earning power has declined over this period. Return on equity, return on assets and return on sales all went down during the three year period.

177.

Both Eli Lilly’s long term debt ratio and capital structure leverage have decreased. Because the percentage decrease in long term debt ratio was higher than the decrease in the leverage, one might conclude that the relative amounts of long term debt, short term debt and equity (as a percentage of total assets) have shifted away from long term debt. In relation to total assets overall debt is down, and the biggest piece of that decrease is in long term debt.


Chapter 5 178.

Eli Lilly’s current ratio has declined, meaning solvency is decreasing. The interest coverage ratio has decreased, meaning that the availability of funds to meet interest expenses has declined. The slowdown in accounts payable turnover means that Lilly is relying more on suppliers as providers of financing.

179.

The increase in the number of days to collect accounts receivable means that Lilly’s customers are taking longer to pay for their purchases. The reduction in the number of days inventory on hand means that the inventory is moving through the operating cycle faster. The decrease in the number of days to generate sales covering the cost of fixed assets, means that fixed assets are lasting longer, not in terms of time, but in terms of sales volume. The increase in the number of days for total asset turnover, means that total assets are being employed less efficiently in terms of sales volume.

180.

The chart indicates that the value of Lilly stock has performed poorly against its peer group, but better than the S&P Index. All three groups have seen good growth since 2010, probably indicating improving general economic conditions in the market. Eli Lilly took longer to rebound from the economic downturn in 2010 than its peer group.

181.

Some of the variations in the ratios can be explained by the industry and type of business. Some ratios are “NA”, that is, unable to be calculated because one of the components of the ratio is less than zero. Kroger’s accounts receivable turnover ratio is so much higher than Bank of America’s because: 1) Kroger’s terms with their customers require them to pay for their purchases in a relatively short period of time, and 2) Bank of America is in the business of lending money, so they are more willing in providing financing to their customers and allow their customers a longer payment period for the fees they are charged. Bank of America’s capital structure leverage is higher than the other companies represented because they are the only bank represented. Banks borrow money in the form of deposits, then turn around and re-lend their depositors’ money profiting on the spread between the rates they pay and the rates they charge. Banks also derive significant revenue from fees. Most banks have no inventory.

CHARACTERISTICS OF END-OF-CHAPTER ASSIGNMENTS Item

Difficulty

Description

Brief exercises: BE5–1 M Analyzing financial statements BE5–2 M Segment analysis Exercises: E5–1 E E5–2 E E5–3 M E5–4 H E5–5 M E5–6 E E5–7 M E5–8 M

Analyzing financial statements Analyzing financial statements Analyzing financial statements Computing ratios and preparing common-size financial statements Solvency and ratios Solvency and statement of cash flows Solvency and ratios Return on equity and inventory turnover


Chapter 5 E5–9 H E5–10 H E5–11 M E5–12 E E5–13 H E5–14 M E5–15 M E5–16 M E5–17 M E5–18 M

Ratios and statement of cash flows--assessing solvency and earning power Effects of transactions on financial ratios Ratios as debt covenants Examining market ratios over time Ratios and the effect of transactions on return on equity Segment analysis Interpreting financial statement ratios Interpreting financial statement ratios ROE model Projected financial statements

Problems: P5–1 E P5–2 M P5–3 M P5–4 H P5–5 M P5–6 H P5–7 M P5–8 H P5–9 M P5–10 M P5–11 M P5–12 H P5–13 H P5–14 M P5–15 H

Computing ratios Effects on financial ratios Common-size financial statements Comprehensive ratio analysis Comparing companies on earning power Analysing financial statements Unusual items and financial ratios Preparing the financial statements from financial ratios Common-size financial statements Comparing ratios to industry averages Assessing the loan risk of a potential bank customer Issuing debt or equity: effects on ratios Preparing financial statements from financial ratios Segment analysis Analyzing an IFRS-based set of financial statements

Issues for discussion: ID5–1 H Linking company characteristics to the financial statements ID5–2 H Linking company characteristics to the financial statements ID5–3 M Price to book ratios ID5–4 E Analyst forecasts ID5–5 H Analyzing a restructuring ID5–6 M Reporting losses from Internet operations in the retail industry ID5–7 H Characteristics of a good investment ID5–8 M Financial accounting information in an efficient market ID5–9 M Company value, intangibles, and the new economy ID5–10 M Weak accounting ID5–11 M Financial ratios, earning power, solvency, and stock prices ID5–12 M Stock price reactions to earnings announcements ID5–13 M Initial public offerings (IPOs) yet to show profits ID5–14 H Analysing the financial statements of Eli Lilly ID5–15 M Projecting financial statements ID5–16 H Cash flow profiles ID5–17 M The SEC Form 10-K of Google


CHAPTER 6 The Current Asset Classification, Cash, and Accounts Receivable

SYNOPSIS In this chapter, the author discusses the uses and limitations of the current asset classification, the measurement and recording of cash and accounts receivables, and the major concerns of financial statement users in accounting for these items. The specific topics covered include the nature of cash, cash controls, cash discounts, and bad debts using the allowance method. The international perspective segment discusses accounting for receivables and payables expressed in foreign currencies, and hedging. The ethics vignette presents the case of a regional bank with admittedly overstated bad debt expense in good periods and understated bad debt expense in poor periods. In this manner, the bank can achieve consistent increases in reported net income across time. The ethical conduct of both the bank executives and the outside auditors is considered. The Internet research exercise directs the student to access and analyze the activity in the allowance for credit losses account for a major bank, JPMorgan Chase. The following key points are emphasized in Chapter 6: 1.

Current assets, working capital, current ratio, and quick ratio, and how these measures can be used to assess the solvency position of a company.

2.

"Window dressing" and the reporting of current assets, working capital, and the current ratio.

3.

Techniques used to account for and control cash.

4.

Accounts receivable and how they are valued on the balance sheet.

5.

The allowance method for uncollectible receivables.

6.

Major concerns of financial statement users in the area of receivables reporting.

TEXT/LECTURE OUTLINE The current asset classification, cash, and accounts receivable. I.

Current assets. A.

A current asset is any asset that is intended to be converted into cash within one year or the company's operating cycle, whichever is longer. An operating cycle is the time that it takes a company to begin with cash, convert the cash to inventory, sell the inventory, and collect the cash from the sale.


Chapter 6 B.

The relative size of current assets across industries.

C. Measures using current assets: working capital, current ratio and quick ratio. D. The economic consequences of working capital, the current ratio and the quick ratio. E. Limitations of the current asset classification F. A movement toward cash flow accounting. II.

Cash A.

Cash is defined as coin, currency, checking accounts, and negotiable instruments such as personal checks, money orders, certified checks, cashiers' checks, and bank drafts.

B.

Restrictions on the use of cash.

C. Proper management of cash. D. Control of cash. III. Accounts receivable. A.

An account receivable is an amount owed to a company from selling goods or services to customers on account. The agreement between the company and the customer is usually informal.

B.

Importance of accounts receivable

C. Net realizable value: The valuation base for accounts receivable 1.

Net realizable value is the net expected future benefit arising from accounts receivable. NRV represents the amount of cash the company expects to realize from accounts receivable.

2.

NRV equals the face value of the receivable less adjustments for the following items: a)

Cash discounts.

b)

Bad debts.

c)

Sales returns.

D. Cash discounts 1.

A discount offered by a company to provide incentives to its customers to pay their open accounts promptly.


Chapter 6

2.

Methods to account for cash discounts. (A)

Gross method. (i)

The sale and related receivable are recorded at the gross amount of the transaction.

(ii) Recognition is given to the discount only if the customer takes the discount. If the customer takes the discount, Cash Discount is debited for the amount of the discount. This account is a contra revenue account to Sales. (iii) The gross method is the most common method because it is the easiest to use. E.

The allowance method of accounting for bad debts (uncollectibles) 1.

Bad debts are amounts sold to customers on account that the company does not expect to convert into cash.

2.

Companies would prefer to have no bad debts, but it would be extremely costly to eliminate all bad debts. Hence, from a cost/benefit perspective, some bad debts are inevitable.

3.

Bad debts should be recognized in accordance with both the revenue recognition principle (i.e., bad debts provide after-the-fact evidence that cash collection was not reasonably assured) and the matching principle.

4.

Allowance method. (a) Bad debts are estimated and recognized in the period in which the underlying credit sale took place. That is, the allowance method results in an amount being estimated and recognized for both Bad Debt Charge (which is offset against the company's sales) and Allowance for Doubtful Accounts (which is offset against the balance in accounts receivable in the period the underlying credit sale took place). (b) The allowance account is a contra asset account that offsets Accounts Receivable. (c) The balance in the allowance account represents the amount reported in Accounts Receivable that the company does not expect to eventually collect. (d) The balance in Bad Debt Charge represents the amount reported in that period's sales that the company does not expect to eventually collect. (e) Percentage-of-credit-sales approach—bad debt charge is estimated as a percentage of the accounting period's credit sales. This approach takes an income statement approach.


Chapter 6 (f)

Aging approach. i)

A company decomposes its accounts receivable balance into different ages and uses this aging to compute the balance necessary in Allowance for Doubtful Accounts.

ii)

Bad Debt Charge represents the change from the unadjusted to the necessarily adjusted balance in the allowance account. This approach takes a balance sheet valuation focus.

iii)

Companies will often use the aging approach to verify the accuracy of the balance in the allowance account computed using the percentage-of-credit-sales approach.

(g) Regardless of the approach used, the balances in both Allowance for Doubtful Accounts and Bad Debt Charge are based on estimates. i)

With the percentage-of-credit-sales approach, the ending balance for Bad Debt Charge is estimated directly and the ending balance for Allowance for Doubtful Accounts is estimated indirectly.

ii)

With the aging method, the ending balance for Allowance for Doubtful Accounts is estimated directly and the ending balance for Bad Debt Charge is estimated indirectly.

5.

Accounting for sales returns.

6.

Accounts receivable from a user’s perspective. a)

Discretion in recognizing receivables and related revenues can result in manipulation of the financial statements.

b)

Judgment in estimating bad debts and sales returns can result in manipulation of the financial statements.

c)

Balance sheet valuation of receivables.

d)

Financial institutions and uncollectible loans.

IV.

International perspective: receivables, foreign currencies, and hedging.

V.

Review problem

VI. Ethics in the real world. VII. Internet research exercise. LECTURE TIPS 1. Students are often puzzled by the computational difference in estimating bad debt expense between the percentage-of-sales and the aging approach. The numerical example below uses


Chapter 6

the same data set as a basis to demonstrate the alternative approaches. he example can be expanded (Case II) to illustrate the effect of a debit balance in the allowance account before adjustment. The data was constructed so as to produce results that are different but reasonable, reflecting the inherent subjectivity in the estimation process and the appropriateness of alternative approaches. The example emphasizes that the primary difference between the two methods is which balance is being estimated directly and which balance is being estimated indirectly. Data Sales Estimated % of sales uncollectible Accounts receivable balance Estimated % of accounts receivable uncollectible based on an aging analysis

$1,050,000 1/2 % $ 75,000 71/2 %

Case I—Allowance for doubtful accounts Case II—Allowance for doubtful accounts

$250 (Cr.) $250 (Dr.)

Demonstration Examples Using the percentage-of-sales approach and, alternatively, the aging approach, prepare the entry to record bad debt expense and compute the final balance in Allowance for Doubtful Accounts for each case. Treat each case independently and compare the results. Check Numbers Bad Debt Allowance Expense

Balance

Case I—Percentage-of-sales approach Case I—Aging approach

$5,250 $5,375

$5,500 $5,625

Case II—Percentage-of-sales approach Case II—Aging approach

$5,250 $5,875

$5,000 $5,625

OUTSIDE ASSIGNMENT OPPORTUNITIES Group study of current assets, cash, and receivables both across time and within and across industries (continuing assignment for Chapters 6–14) 1.

Form groups (3–5 students) and obtain the most recent annual report of a major public company in one of the four general industry groupings (manufacturing, retailing, general services, and financial services). This report will be used for this assignment and for similar assignments for each succeeding chapter. Depending on class size, each industry group will be represented by several companies. Research the economic characteristics of and current conditions in the industry (e.g., competitive market; capital and labor intensity and other production characteristics; regulatory status; growth profile; and sensitivity to technological, demographic, and macroeconomic trends) and the company’s strategy for competing. Prepare a written summary of those background factors. This summary will be


Chapter 6 used as a framework for understanding and interpreting financial statement information to be studied in these assignments. Identify or compute the items listed below for the two most recent years. Compare the items across time. Relate your findings to the industry and company background factors. Report findings in a class discussion session in which comparisons will be made both across time and within and across industries. Current assets as a percentage of total assets Current ratio Quick ratio Balance sheet caption for cash and equivalents Cash as a percentage of total assets and current assets Balance sheet caption for receivables and the related allowance for uncollectibles Receivables as a percentage of total assets and current assets Bad debts as a percentage of outstanding receivables Average accounts receivable collection period Inferring bad debt reporting strategies for a commercial bank 2.

Obtain the most recent annual report for a commercial bank. Identify the trend in net income for as many periods as possible. (The 10-year summary may provide enough information for a long-term analysis.) Identify any strategy the bank may be using to report bad debts and support your position with appropriate computations. Explain why the bank’s management may be using the strategy you identified.

ANSWERS TO IN-TEXT DISCUSSION QUESTIONS 227.

Tommy Hilfiger, a clothing manufacturer, designs and markets fashion clothing ultimately sold by retailers. It buys raw materials and converts them to finished products and/or outsources production of its merchandise. Finished goods are then sold to retailers on open account. The operating cycle–the time it takes the company to convert cash to inventory, sell the inventory, and collect cash from the sale–would likely be fairly long, but certainly less than a year. Toyota, an automobile manufacturer, designs and manufactures automobiles. It buys raw materials and converts them to finished products. Finished goods are then sold to dealers on open account. The operating cycle–the time it takes the company to convert cash to inventory, sell the inventory, and collect cash from the sale–would likely be fairly long, but probably less than a year. Young & Rubicam, an advertising agency, does not manufacture or sell goods, but rather provides services to its clients. The operating cycle consists of paying salaries to employees who render the services followed by billing and cash collections. The time it takes the company to convert the services rendered by its employees into cash is probably a month or two.


Chapter 6

Yahoo!, an internet portal, does not manufacture or sell goods, but rather provides services to its customers. The operating cycle here is probably a monthly cycle where customers pay for monthly service.

229.

RadioShack’s primary revenue producing asset is inventory, compared to Simon Property Group which generates revenue primarily from its mall properties. Accordingly, the major asset category on RadioShack’s balance sheet is inventory, a current asset, while Simon’s major asset is property, plant and equipment, a non-current asset category.

230.

Recognizing revenue before the sale was complete would create an overstatement of assets, probably in accounts receivable. The overstatement of accounts receivable enhances the appearance of working capital, thus it fits the description of “window dressing.”

231.

Current and quick ratios are two simple measures of liquidity, neither of which reflects Kroger’s true liquidity position. Kroger’s has relatively little in the way of current assets because of the nature of its operations. Kroger’s buys it and sells it and collects for it all very quickly. Inventories are kept low and turn over quickly. Kroger’s has a short operating cycle. Current and quick ratios are static measures. Kroger’s liquidity is better assessed with a dynamic measure, such as cash flow from operations (CFFO) compared to current liabilities.

232.

The correct amount of Cash that a company should keep is a function of its needs, its future plans for the usage of cash (cash budget), and its risk appetite. The company will also react to the needs and expectations of its shareholders who are the owners of the company. Holding excessive cash carries the opportunity costs of all of the other options that are available to the organization in terms of investing the money including putting money back into growing the business. Carrying excessive cash is not an effective use of assets of from the investors’ perspective. If they wanted an investment in cash, they would not have bought stock in the company to begin with. If a company does not have plans for excess cash, then it should be returned to investors who can invest in other opportunities, or the company can increase value to shareholders by reducing the number of shares outstanding by buying treasury stock.

233.

The restricted cash would not be included with the other current assets in the calculation of working capital because it is not available to pay current liabilities.

233.

Cash management requires managers to maintain enough cash to ensure that the business remains liquid and is able to meet payment obligations, and to invest cash in excess of those needs in assets that produce a higher return. Identifying and estimating cash inflows and cash needs, and determining the amount and where to invest excess cash, are important concerns to company managers.

234.

Company managers desire to keep cash balances at a minimum, and therefore the absolute amount of cash at any one point in time is minimal compared to other assets. Yet, cash transactions are very voluminous and are central to every accounting system. Most liabilities, revenues, expenses, and other assets flow through the cash account. The mere number of transactions leads to a higher likelihood of errors. Also, cash is the most liquid of assets and the most susceptible to theft, embezzlement, and


Chapter 6 misappropriation. Accordingly, record and physical controls of cash are critical to the accountant. 235.

Paying attention to the relationship between sales and accounts receivable might alert an analyst to a problem. Accounts receivable turnover is a ratio based on this relationship. An unexplained increase or decrease in receivables turnover may be an indication of an accounting problem.

236.

Large companies generally have more economic power than smaller companies and are better able to control the terms of contracts with both their vendors and their customers than are smaller companies. For this reason, smaller companies will usually have more difficulty collecting their receivables and extending the time they have to pay their vendors than the larger companies, especially in an economic downturn.

236.

Accounts receivable management and control is likely most important to General Electric. Wal-Mart is a cash and carry business without significant accounts receivable. Walgreens has few receivables. General Electric is a large diversified manufacturer, which also has a finance operation. Receivables are very material to its operations and financial statements.

237.

The net realizable value of the 11.02 billion in accounts receivable was only $10.93 billion. The difference is the allowance for doubtful accounts.

240.

A company must evaluate the creditworthiness of potential customers before making sales on account to them. This is a costly, and necessarily subjective (and therefore risky) undertaking, especially when it involves a new customer-base in a new market. Managers must balance the profit from the incremental sales gained from credit sales, against a realistic estimate of the bad debt losses that are inevitable if credit is granted.

241.

Because Citigroup’s core business is financial services (lending) it may be better at managing the credit card business than Sears was. The “increasingly large reserves” were allowances for doubtful accounts, which are often referred to as “reserves.”

243.

The $.1 million charged to expense was the amount of bad debt expense for the year. The $.3 million of uncollectible receivables written off were written off under Radio Shack’s policy because of doubts as to collectability. Because the write-offs exceeded the expense, the allowance decreased.

244.

Collection of the receivables appeared unlikely since no payments had been made. An allowance should have been established for those amounts that would not be collected and a large bad debt expense should have affected the income statement. Users of the financial statements might have detected the problem by comparing the amount in the allowance account to such numbers as sales and accounts receivable across time. A/R ratios such as A/R turnover would have reflected these issues. Unsatisfactory explanations for unusual deviations may have revealed a problem.

245.

There is a trend in American business to outsource functions to others that can perform them more effectively and efficiently, and collection of delinquent receivables is no exception. Companies are willing to sell delinquent receivables for an assured smaller amount now, rather than wait to possibly receive a larger amount later. Mr. Passen is so confident in his new firm’s collection skills, that he believes he can achieve higher


Chapter 6

returns by actually buying the delinquent receivables and keeping the full amount recovered, than by simply taking a percentage of amounts recovered. 246.

The indictment alleges that certain (named) executives kept Computer Associate's books open at the end of fiscal periods. In the week following the end of fiscal periods, while the books were held open, they directed sales managers and salespeople to finalize and backdate license agreements. Revenue from those falsely dated license agreements was then improperly recognized in the quarter just ended. The executives met routinely and conferred with each other during the week following the end of fiscal periods to determine whether Computer Associate's had generated sufficient revenue to meet the quarterly projections, and closed Computer Associate's books only after they determined that Computer Associate's had generated enough revenue to meet the quarterly projections.

247.

Revenue recognition on product sales prior to shipment is not appropriate because the earnings process is not complete until the goods are shipped. Such a practice could result in a receivables write-off if the goods were not in fact subsequently shipped or from a change in accounting to recognize revenue in the proper period.

248.

Macy’s decided that they did not want to be in the customer credit business. Rather, they spun that part of the business off, and sold it. Because of the widespread use of credit cards, merchants can sell on credit without carrying the risks involved in financing consumer purchases.

249.

Receivables may have increased because sales grew, collections slowed, or because of acquisitions of other companies, or a combination of those factors. An investor’s interpretation of the company’s current ratio and cash flow statement should include an evaluation of the factors underlying the increase to help determine whether the receivables are in fact realizable. A favourable current ratio does not mean much unless the receivables are in fact collectible. Cash flow measures should be used together with the current ratio. A large difference between net income and net cash from operations because of an increase in receivables could be a problem, but not necessarily. If the increase in receivables is commensurate with an increase in sales, it may not be.

250.

The increases in accounts receivable of $60, $714, and $774 million represent earnings that were not collected in the form of cash. Because the operating section of the statement of cash flows starts with net income, the fact that accounts receivable increased needs to be taken into consideration when computing cash provided by operations. That much less cash was provided than would have been the case had accounts receivable not increased.

250.

Losses on receivables reduce a bank’s retained earnings, a component of their capital. These impairments of banks’ capital pose a threat to the solvency of individual banks and on a macro-level to our entire banking system and our economy as a whole.

CHARACTERISTICS OF END-OF-CHAPTER ASSIGNMENTS Item

Difficulty

Brief exercises:

Description


Chapter 6 BE6–1 E BE6–2 E BE6–3 E

Analysis of accounts receivable Uncollectible accounts Uncollectible accounts

Exercises: E6–1 E E6–2 E E6–3 E E6–4 E E6–5 M E6–6 M E6–7 M E6–8 H E6–9 M E6–10 M E6–11 M E6–12 M

Classifying cash on the balance sheet Classifying cash on the balance sheet Accounting for cash discounts Accounting for cash discounts Bad debts under the allowance method Accounting for uncollectibles Accounting for doubtful accounts: the allowance method Inferring bad debt write-offs and reconstructing related journal entries Accounting for uncollectibles - IFRS Preparing an aging schedule Exchange gains/losses on outstanding receivables Hedging to reduce the risk of currency fluctuations

Problems: P6–1 M P6–2 E P6–3 M P6–4 M P6–5 M P6–6 M P6–7 H P6–8 H P6–9 M P6–10 H P6–11 H P6–12 M P6–13 M

Classifying cash on the balance sheet Cash discounts Bad debts over time Accounting for uncollectibles over two periods Accounting for uncollectibles over three periods Analysing the activity in the allowance account Ignoring potential bad debts--overstatements Estimating uncollectibles: financial ratios and loan agreements Uncollectibles: ignoring an allowance Accounting for uncollectibles and the aging estimate Inferring reporting strategies Exchange gains and losses Fluctuating exchange rates, debt covenants, and hedging

Issues for discussion: ID6–1 M Restricted cash and solvency ratios ID6–2 E Revenue recognition, ethics, and reputation ID6–3 M Working capital, debt covenants, and restrictions on management decisions ID6–4 M Analyzing the allowance account ID6–5 M Managing reserves for uncollectible receivables ID6–6 M Provisions for loan losses and profits ID6–7 M Bad debts, statement of cash flows: GAAP vs. IFRS ID6–8 M Earnings restatement due to misstated loan losses ID6–9 E Concentrations of credit risk ID6–10 M Boosting earnings with bad debt estimates ID6–11 M Macroeconomic conditions and uncollectibles ID6–12 E Accounting for foreign currencies—an economic consequence ID6–13 M The annual report of Google


CHAPTER 7

Merchandise Inventory

SYNOPSIS In this chapter, the author discusses the economic consequences faced by managers in accounting for inventory. Specifically discussed are the effects of inventory accounting on (1) financial statement users; (2) existing contracts; (3) income taxes; (4) bookkeeping costs; and (5) the usefulness of resulting information. The author also presents the conceptual and practical issues in accounting for inventory, including (1) the acquisition of inventory, covering which units to include as part of inventory and what costs to capitalize as part of inventory; (2) the perpetual method of accounting for inventory; (3) the potential cost flow assumptions for allocating inventory costs to cost of goods sold, FIFO, LIFO, and averaging; and (4) the lowerof-cost-or-market rule, as it applies to inventory. The ethics vignette considers the pressure on auditors to cut hours spent auditing such critical areas such as inventories due to competition. The Internet research exercise explores the inventory policies of Ann Taylor Stores and its competitors. The following key points are emphasized in Chapter 7: 1.

Inventory and how it affects the financial statements.

2.

Four issues that must be addressed when accounting for inventory.

3.

General rules for including items in inventory and attaching costs to these items.

4.

The three cost flow assumptions—average, FIFO, and LIFO.

5.

The lower-of-cost-or-market rule.

TEXT/LECTURE OUTLINE Merchandise inventory. I. Merchandise inventory refers to items held for sale in the ordinary course of business. A. Shareholders, creditors, managers, and auditors are all interested in a company’s inventory. B. The value and marketability of inventory can also provide an indication of a company’s ability to continue as a going concern. II.

The relative size of inventories.

III. Accounting for inventory: four important issues.


Chapter 7 A. Acquiring inventory: what costs to capitalize. 1.

Items to include in inventory. a)

As a general rule, items held for sale and for which the company has complete and unrestricted ownership should be included in inventory. Purchases should be recorded when legal title to the items passes from the seller to the buyer.

b)

Sometimes it is more difficult to determine the appropriate number of inventory units. (i)

Goods on consignment. (a) Items owned by one party (the consignor), but physically held for sale by another party (the consignee). (b) Goods on consignment should be included in the inventory of the consignor, not the consignee, since the consignor retains legal title to the goods.

(ii) Goods in transit. (a) Inventory items not in the physical possession of either the buyer of the seller as of a point in time because the goods are in-transit from the seller to the buyer. (b)

2.

Ownership is determined by the shipping terms. i)

FOB shipping point—legal title to the inventory items passes from the seller to the buyer when the seller delivers the goods to the shipping point.

ii)

FOB destination—legal title to the inventory items passes from the seller to the buyer when the goods reach their destination.

Costs to attach to inventory items. a)

As a general rule, all costs associated with manufacturing, acquiring, storing, or preparing inventory items should be capitalized as inventory.

b)

Cash discounts.

c)

(1)

A discount offered by suppliers for prompt payment on purchases made on account.

(2)

Cash discounts are accounted for using the gross method.

Potential costs that could be capitalized for manufacturing companies.


Chapter 7

B.

(1)

Direct costs (such as direct labor and raw materials) should be capitalized as part of inventory.

(2)

Overhead (such as indirect materials, indirect labor, utility costs, depreciation, and so forth) must be allocated to inventory. Allocating overhead to inventory is subjective and allows for potential management manipulation of financial statements.

Carrying inventory: the perpetual method 1.

The inventory account is directly increased for every inflow of inventory.

2.

The inventory account is directly decreased for every outflow of inventory.

C. Errors in the inventory count. 1.

2.

Inventory errors misstate both inventory on the balance sheet and net income on the income statement for the period. a)

An overstatement of inventory overstates net income by the same amount; an understatement of inventory understates net income by the same amount.

b)

Net income for the next period is misstated by the same amount in the opposite direction.

Inventory errors are not unusual, often quite significant, and sometimes made intentionally by management to manipulate reported income.

D. Selling inventory: which cost flow assumption? 1.

2.

Specific identification. a)

The cost of the actual inventory item sold is allocated to cost of goods sold.

b)

Only practical for companies that sell large-ticket, easily-tracked items.

c)

Allows manipulation of net income and inventory.

Three inventory cost flow assumptions. a)

A cost flow assumption is used to determine the inventory cost allocated to cost of goods sold and inventory. The cost flow assumption does not refer to the physical flow of inventory.

b)

Averaging assumption: a weighted average cost of all inventory units available is used to allocate inventory costs.

c)

First-in, First-out (FIFO) assumption: the oldest inventory costs are assumed to be the first inventory costs sold.


Chapter 7

d)

3.

4.

5.

Last-in, First-out (LIFO) assumption: The most recent inventory costs are assumed to be the first inventory costs sold.

Inventory cost flow assumptions: effects on the financial statements. a)

Inventory costs increasing.

b)

Inventory costs decreasing.

c)

LIFO liquidation boosts profits.

Inventory cost flow assumptions: effects on federal income taxes. a)

LIFO conformity rule.

b)

Inventory costs increasing – LIFO minimizes taxes.

b)

Inventory costs decreasing – FIFO minimizes taxes.

c)

LIFO liquidation increases taxes because it increases profits.

Tradeoffs among the cost flow assumptions. a)

b)

Income and asset measurement tradeoffs. (1)

LIFO matches the most current inventory costs to current revenues, whereas FIFO matches older inventory costs to current revenues. Consequently, LIFO provides "better" matching than FIFO. The averaging assumption falls between LIFO and FIFO.

(2)

FIFO reports the most current inventory costs as ending inventory, whereas LIFO reports older inventory costs as ending inventory. LIFO can report an inventory balance that is grossly out-of-date. The averaging assumption falls between LIFO and FIFO.

Economic tradeoffs. (1)

Income taxes and liquidity.

(a) LIFO conformity rule. (b) In times of rising prices, LIFO results in lower net income than does FIFO. This lower net income results in lower current taxes. The tax effect has cash flow implications and can also have liquidity implications. (2)

Bookkeeping costs: LIFO requires more detailed accounting records so bookkeeping costs are higher under LIFO than FIFO.


Chapter 7

(3)

LIFO liquidations and purchasing practices: Unexpected increases in sales or unexpected decreases in purchases or production can deplete LIFO inventory layers. LIFO liquidations can give rise to substantially increased taxes (with rising prices) and poor inventory purchasing practices.

(4)

Debt and compensation contracts: If debt covenants specify a minimum debt/equity ratio or a minimum working capital requirement, management can minimize the probability of violating the covenant by maximizing inventory and net income via FIFO. Further, if management is compensated, at least partially, with a bonus contract, management can maximize the value of its bonus by maximizing net income via FIFO.

(5)

The capital market: If managers believe that investors cannot "see through" accounting methods but, rather, focus on net income, then managers may prefer to use FIFO to increase income. Research in accounting and finance generally supports the view that investors "see through" accounting methods and value a company on the basis of its underlying cash flows.

(6)

The LIFO reserve: Financial statements of companies using LIFO include footnote disclosure of "LIFO reserves," which reflect the difference between inventories computed using LIFO and FIFO. This disclosure is helpful to financial statement users in assessing the tax and income effects of using LIFO versus FIFO, and for making more valid comparisons with other companies that use FIFO.

IV. Ending inventory: applying the lower-of-cost-or-market rule.

V.

A.

Determine the original cost of the inventory based on the cost flow assumption selected.

B.

Compare the original cost to market value. 1.

If the market value is less than the original cost, adjust the original cost down to the market value, thereby resulting in an unrealized loss.

2.

If the market value exceeds the original cost, then no adjustment is necessary.

The lower-of-cost-or-market rule and hidden reserves. A.

Conceptually, the lower-of-cost-or-market-value (LCM) rule is inconsistent, since decreases, but not increases, in value are recognized.

B.

Although this inconsistent treatment can create hidden reserves that managers can use to manipulate income, the LCM rule makes economic sense due to conservatism.


Chapter 7 VI. International perspective: Japanese business and inventory accounting VII. Review problem VIII. Ethics in the real world. IX. Internet research exercise.

LECTURE TIPS 1.

Students often have difficulty understanding goods in transit, particularly (1) deciding on in whose books goods in transit should be recorded, and (2) how to adjust the books if they have already been adjusted to the physical count and goods in transit are then detected. End-of-chapter exercise 7–1 is useful to demonstrate how to account for goods in transit. The exercise can be extended to illustrate the correcting entries required and how not correcting the errors affects the current and following period.

2.

Students struggle with LIFO because the accounting assumptions used in LIFO are the reverse of how goods normally flow through an inventory. Students understand LIFO best when it is presented as a tax savings technique approved by congress via the political process rather than as a system based on logic. Stress that LIFO is a cost flow assumption, not a “goods flow” assumption.

OUTSIDE ASSIGNMENT OPPORTUNITIES Group study of inventories both across time and within and across industries (continuing assignment for Chapters 6–14) 1.

Using the most recent annual report of a major public company in one of the four general industry groupings, identify or compute the items listed below for the two most recent years. Compare the items across time. Relate your findings to the economic characteristics and the current conditions in the industry and the company’s strategy for competing. Prepare a written summary of your findings. Report findings in a class discussion session in which comparisons will be made both across time and within and across industries. Captions used for inventories Inventories as a percentage of total assets and total current assets Cost flow assumption(s) used Cost of goods sold as a percentage of total revenues LIFO reserves

Article search: inventory fraud 2.

Search the recent financial press (The Wall Street Journal, Barrons, Fortune, Forbes, etc.) for an article reporting inventory fraud. Evaluate the situation in terms of valuation principles violated, methods used, management's motivation, the auditor's role, and ethical considerations. Present your findings in writing and in a class discussion.


Chapter 7 ANSWERS TO IN-TEXT DISCUSSION QUESTIONS 269.

Where inventories are a material income producing factor, such as for manufacturers and retailers, management must ensure that inventories are manufactured or acquired at reasonable costs. Also, management must be efficient concerning quantities on hand, i.e., the cost of carrying inventory and the risk of obsolescence must be balanced against having enough goods on hand to meet demand. “Uncontrolled inventories” are those where management fails to meet those cost and efficiency goals.

271.

IBM sells a tangible product and therefore has substantial inventories. Yahoo is a service provider which does not sell goods, but rather uses its infrastructure to generate revenues.

272.

Consignment inventories are those inventories which are physically on the premise of a consignee (receiver) who holds the goods for sale, in this case Saks, but are actually owned by another, the consignor. Ownership, not physical possession, determines the balance sheet upon which consigned inventory is carried. Therefore the inventories belonging to others held by Saks on consignment are not reflected on Saks’ balance sheet.

273.

Kellogg’s accounting policy of recognizing sales upon the delivery of its product to customers implies that goods are shipped FOB (free on board) destination, which means that Kellogg is responsible for the goods to the point from which they are delivered to the customer. The buyer has the responsibility for the goods after that point. Revenue is recognized at the point of delivery because the risk of ownership has been passed to the buyer, thereby completing the earnings process. Under the matching principle, the cost of the goods sold is removed from inventory at the point of delivery, and charged to the income statement.

273.

Lowe’s costs of assembling light fixtures before displaying them for sale is a cost of getting the inventory ready to be sold and should be capitalized as a cost of inventory. These costs would then be included in the cost of goods sold when the light fixtures are sold.

274.

Dell’s incredibly rapid inventory turnover has a positive impact on Dell’s cash flow. Dell doesn’t need to tie up very much cash in order to carry 13.9 days worth of inventory. Dell could very well sell most of its inventory (and collect the cash) before they have even paid their vendors for it. If these are all cash sales, Dell’s operating cycle would revolve every 13.9 days, generating more and more cash. The term “cash cow” comes to mind.

275.

Intel Corporation is obviously a manufacturer because it has three categories of inventories: raw materials, work in process, and finished goods. Raw materials represents the acquisition cost of materials to be processed into a finished product through the application of labor and other costs, including overhead. Work in process includes accumulated costs of materials, labor and overhead for those goods in process, but not complete at the balance sheet date. Finished goods includes all costs linked to the production process for those goods which are complete, but not sold as of the balance sheet date. The amount of inventory shown on the balance sheet is the total of all three categories, or $4.7 billion at its year-end 2012.


Chapter 7 277.

The nature of most retail businesses is that inventory is an important component of their balance sheet. CVS’s investment in inventory at any given time runs into many millions of dollars. Physical control of the inventory is important if losses from spoilage, theft, and obsolescence are to be avoided. Management has a stewardship responsibility for inventory and each store manager spends a great deal of his or her time attending to inventory related tasks. An increase in inventory turnover can help improve profits and management needs accurate and up to date information in order to properly manage the inventory. They need to purchase and stock the right goods and run promotions that make sense. Without accurate information about inventory managers don’t know what is going on and cannot control inventory, sales or cash flow. Keeping track of inventory is a messy and error prone process. Regular counts of inventory are necessary to verify the information being generated by the inventory management system. The counts are useful in the correcting errors that continuously arise in the process of keeping track of inventory. The accuracy of the monthly financial statements that management uses depends on the accuracy of the accounting records for inventory. The accuracy of the annual financial statements are also dependent correct values for inventory.

278.

If The Gap had incorrectly reported ending inventory at $1,900 million, the overstatement of inventory would be $142 million ($1,900 million inventory versus the correct amount of $1,758 million). The overstatement of ending inventory would have caused an understatement of cost of goods sold, and an overstatement of gross profit and net income by a like amount ($142 million). 2012 net income would have been erroneously reported at $1,277 million.

280.

Specific identification is difficult to use where there are a significant number of items being sold. The difficulties of using this method create additional record-keeping costs which become more and more burdensome as volume increases. The benefit of using this method is the ability of management to manipulate gross profit by selling the least costly item (or the most costly item where profits are not desired) when inventory includes identical items that were acquired at varying costs. In the early days, the ability to manipulate income may have been important to Amazon. However, as the company grew and matured, the costs of using this method of accounting would have increased, while at the same time the ability to manipulate income probably became less important.

282.

As prices rise, the difference between the book value of an inventory under FIFO compared to the book value of that same inventory under LIFO will increase. Assuming there are no liquidations of LIFO layers, the difference will continue to grow every year as long as prices are rising. Since prices usually rise every year, companies that have been in business for a long time will see a large disparity between the LIFO cost of their inventory compared to its cost computed under FIFO. This is not the case in industries where prices are declining, such as computers. For a chain of drug stores, however, with inventories that include expensive prescription drugs, the costs of which are skyrocketing, one would expect there to be a significant LIFO reserve

284.

At a 30% income tax rate, and a $1.9 billion LIFO reserve, Walgreen’s tax savings from using LIFO would be $570 million. Theoretically, this savings is only a deferral of taxes because eventually all the inventory will be sold and all the profits recognized. In reality however, if Walgreen’s continues to be a successful business indefinitely, these taxes are deferred indefinitely.


Chapter 7 285.

In a period of rising prices, LIFO produces a lower income number than FIFO. It results in a more realistic matching of income and costs because it assigns the most recent, higher costs to the income statement. FIFO matches older, lower costs against income, thereby creating so-called paper profits on which taxes have to be paid. The lower income under LIFO would result in lower cash payments required for income taxes, thus improving liquidity for companies with cash flow problems.

286.

It looks like the reduction in LIFO inventory caused a liquidation of LIFO layers. This happens when LIFO is used and inventory levels decline from one year to the next. The liquidation of a LIFO layer has an affect on the cost of goods sold - the costs of the goods sold corresponding to the inventory reduction are understated. The costs are understated because the inventory cost associated with last year’s LIFO layer is lower than the current year’s inventory cost. This understatement of cost of goods sold results in an overstatement of income. Analysts will find this of interest because the portion of the income reported for the year that is attributable to the reduction of LIFO layers would not be expected to recur. This type of income does not generate any cash flow; it actually generates negative cash flow because income taxes have to be paid on this “phantom income”. The adverse tax and cash flow consequences associated with liquidation of LIFO layers is one of the disadvantages of using LIFO.

287.

The “Less adjustments to LIFO value”, would represent the “LIFO reserve” for Deere & Company. It is the difference between the value of inventory under FIFO and the value of inventory under LIFO. Under LIFO, inventories are carried on the books at lower costs and it is important for users of financial statements to understand the amount of potential understatement of inventory values. If Deere & Company liquidated any of this LIFO inventory moving forward, cost of goods sold would be understated and gross profits would be overstated. This is due to higher revenue values being matched against artificially low costs of goods sold.

289.

The weak market caused a decline in steel prices and a reduction in the value of the inventory. Although the inventory write down is an expense, it doesn’t affect cash. Consequently, the write down is a non-cash expense (like depreciation and amortization) that is added back to net income in the operating section of the statement of cash flows.

CHARACTERISTICS OF END-OF-CHAPTER ASSIGNMENTS Item Difficulty Description Brief exercises: BE7–1 E Inventory analysis BE7–2 E Inventory analysis BE7–3 E FIFO vs. LIFO Exercises: E7–1 E E7–2 E E7–3 E E7–4 E E7–5 E E7–6 M

Goods in transit Accounting for inventory purchases Accounting for inventory purchases Inventory equation Cost of Goods Sold and adjusting entries Effects of inventory errors


Chapter 7 E7–7 M E7–8 E E7–9 M E7–10 M E7–11 H E7–12 H E7–13 H E7–14 M

Inventory and statement of cash flows Income manipulation under specific identification Income manipulation under specific identification Inventory cost flow assumptions Inventory flow assumptions over several periods and income taxes Using the LIFO reserve The lower-of-cost-or-market rule and hidden reserves Inventory accounting under GAAP and IFRS

Problems: P7–1 E P7–2 E P7–3 M P7–4 H P7–5 H P7–6 M P7–7 H P7–8 M P7–9 H P7–10 M P7–11 M

Inventory purchases and cash discounts The gross method and partial payments Effects of inventory errors The financial statement and income tax effects of averaging, FIFO, and LIFO Inventory accounting, earnings, taxes, and lower-of-cost or market The gross method and the LIFO and FIFO cost flow assumptions Using LIFO and saving tax dollars LIFO liquidations, income tax implications, and year-end purchases Using the LIFO reserve Avoiding LIFO liquidations Inventory accounting under IFRS

Issues for discussion: ID7–1 M Choosing FIFO or LIFO ID7–2 M LIFO reporting ID7–3 M LIFO liquidation and hidden reserves ID7–4 M The lower-of-cost-or-market rule and the recognition of loss/income ID7–5 M Inventory write-down ID7–6 M Interpreting the inventory footnote ID7–7 M “Thinning” inventories during slow times ID7–8 H Inventory and statement of cash flows ID7–9 H Different inventory disclosures ID7–10 M Analyzing inventory management on statement of cash flows ID7–11 M Accounts payable management and cash flows ID7–12 M The annual report of Google


CHAPTER 8

Investments in Equity Securities

SYNOPSIS In this chapter, the author discusses investments in equity securities. The discussion is divided into equity securities classified as current and long-term investments. The specific topics associated with investments in equity securities are (1) the criteria for equity investments to be classified as current; (2) accounting for investments in trading and available-for-sale securities, including the mark-to-market rule; and (3) selected economic and theoretical issues. The specific topics associated with long-term equity investments are (1) accounting for long-term equity investments under the cost method, the mark-to-market method, the equity method, consolidations, goodwill, special purpose entities, and (2) the economic consequences associated with the different methods. The author provides a discussion of the detailed accounting procedures for consolidating financial statements in Appendix 8A. The ethics vignette examines the liberal accounting rules for recognition and amortization of goodwill in countries outside the United States. It then considers whether it is ethical for the government or standard-setting body in a particular country to set accounting standards that are designed to provide international economic advantages enjoyed solely by the companies and capital markets in that country. The Internet research exercise examines Cisco System’s strategy of growth through acquisitions. The following key points are emphasized in Chapter 8: 1.

Criteria that must be met before a security can be listed in the current assets section of the balance sheet.

2.

Trading and available-for-sale securities and how the mark-to-market rule is used to account for them.

3.

Why companies make long-term investments in equity securities.

4.

The mark-to-market method, the cost method, and the equity method of accounting for long-term equity investments, and the conditions under which each method is used.

5.

Consolidated financial statements, when they are prepared, and how they differ from financial statements that account for equity investments using the equity method.

LECTURE/TEXT OUTLINE Investments in equity securities. I. Equity investments are an investment in the equity securities of other companies. These investments are made (1) to earn investment income through dividends and stock


Chapter 8 appreciation, and (2) to exert influence or control over the board of directors and management of the investee company. II.

Equity securities classified as current. A.

Short-term investments in equity securities are reported as current assets.

B.

Criteria to be classified as current. 1.

The securities must be readily marketable. This criterion implies that the security can be sold and converted into cash at will.

2.

Management must intend to convert the investment to cash within the time period of current assets.

3.

a)

Management's incentives to "window dress" current assets through the classification of equity securities.

b)

Auditors can examine the company's past practices and the nature and size of the investment to determine management's intentions.

If neither criterion is met, the investment must be included in the long-term investment section.

III. Trading and available-for-sale securities. A.

Accounting for readily marketable equity securities is governed by SFAS No. 115, "Accounting for Certain Investments in Debt and Equity Securities."

B.

Investments in readily marketable equity securities are classified into one of two categories. 1.

Trading securities—securities bought and held principally for the purpose of selling them in the near future with the objective of generating profit on shortterm price changes. These securities are always listed in the current section of the balance sheet.

2.

Available-for-sale securities—those investments not classified as trading securities. These securities are listed on the balance sheet as current or longterm, depending on management’s intention.

C. Events related to accounting for trading and available-for-sale securities. 1.

2.

Purchase of securities. a)

Equity securities are always recorded at cost when they are acquired.

b)

Cost includes any incidental acquisition costs such as brokerage commissions and taxes.

Declaration and receipt of cash dividends.


Chapter 8

3.

4.

a)

On the date the dividend is declared, dividend income and the corresponding dividend receivable are recorded.

b)

When the cash from the dividend is received, Cash is debited and Dividend Receivable is credited. Receiving the dividend is simply an exchange of assets.

Sale of securities. a)

The balance sheet value of the securities is removed from the books.

b)

The difference between the balance sheet value of the securities and the proceeds of the sale is recognized as either a realized gain or a realized loss.

Price changes of securities. a)

The FASB mandated in SFAS No. 115 that readily marketable equity securities be carried on the balance sheet at current market value.

b)

The mark-to-market rule gives rise to unrealized (or holding) gains or losses.

c)

End of period adjustments to reflect current market values differ for trading securities and available-for-sale securities. (1) For trading securities, holding gains or losses are considered temporary accounts. They appear in the income statement and are reflected in retained earnings. (2) For available-for-sale securities, the unrealized price changes are considered permanent accounts and are carried in the shareholders' equity section of the balance sheet.

5.

6.

Reclassifications and permanent market value declines. a)

When management changes the classification of investments from trading to available-for-sale, or vice versa, unrealized holding gains or losses should be recognized immediately in the income statement.

b)

When permanent market value declines (i.e., the price declines and is not expected to recover) the security should be written down to its market value and a realized loss should be recognized immediately.

Mark-to-market accounting and comprehensive income. a)

While unrealized price changes in available-for-sale securities are not reflected in net income, the FASB requires that such changes be reported on a statement of comprehensive income.


Chapter 8

7.

b)

No specific form is required, but the statement of comprehensive income must be displayed with the same prominence as the other financial statements.

c)

Comprehensive income provides the statement user with an estimate of the overall change in a company's wealth during the period.

Practical and theoretical issues associated with the mark-to-market rule. a)

Current market values provide more useful information about the wealth of the firm.

b)

Market value accounting, in general, is criticized because market values are subjective and can result in large fluctuations from period to period (which are not necessarily relevant). Mark-to-market accounting for investments in equity securities avoids these criticisms. (1) The rule applies only to readily marketable equity securities, i.e., those that have readily determinable and objective market prices. (2) Market value changes of only those securities that are intended to be sold in the very near future (trading securities) are reflected in current income. Unrealized price changes related to securities classified as available-for-sale are not included in the income statement.

c)

IV.

Management's subjective judgment may influence the reported dollar amount of net income because the classification of securities as trading or available-for-sale depends heavily on management's intention.

Long-term equity investments. A.

The accounting treatment used for long-term equity investments depends upon the investor's potential to influence the investee company. 1.

Mark to market or cost method: If the investor has little potential to influence the investee company (defined as owning less than 20 percent of the investee's outstanding stock), then the marketability of the investee's stock determines the appropriate accounting treatment. a)

If the investee's stock is not readily marketable, then the cost method is used to account for the equity investment. (1) The investment is initially recorded at cost, which includes any incidental costs like brokerage fees. (2) Dividends declared by the investee are recognized as revenue. (3) The investment is carried on the books at cost until it is sold or until it suffers a permanent impairment in value.


Chapter 8 b)

2.

If the investee's stock is readily marketable, then the securities are classified as available-for-sale and the mark-to-market rule is applied. The investment is accounted for as outlined in [II.–C.] above.

Equity method: If the investor can exert significant influence over the investee company (defined as owning between 20 percent and 50 percent of the investee's outstanding stock), then the investment, regardless of whether the investee's stock is marketable, should be accounted for using the equity method. a)

The investment is initially recorded at cost, which includes any incidental costs like brokerage fees.

b)

The amount reported for the investment is adjusted for changes in the investor's proportionate share of the investee's net assets. (1) The carrying value of the investment is adjusted for the investor's proportionate share of the investee's net income or loss. (a) Net income increases the investment, and the investor's share of the investee's net income flows through to the investor's income statement. That is, the investment account is debited, and an investment income account is credited for the investor's proportionate share of the investee's net income. (b) Net loss decreases the investment, and the investor's share of the investee's net loss flows through to the investor's income statement. That is, the investment account is credited, and an investment loss account is debited for the investor's proportionate share of the investee's net loss. (2) The carrying value of the investment is adjusted for the investor's proportionate share of dividends declared by the investee. (a) Dividends declared by the investee decrease the investor's investment and do not flow through to the investor's income statement. That is, Cash is debited and the investment account is credited for the investor's proportionate share of dividends declared by the investee. (b) Dividends declared by the investee are simply an exchange of one asset (the investment) for another asset (cash).

c)

Some cautions to financial statement users about the equity method.

3.

The fair market value option: disclosures are required relating to the basis for the market value estimate when market values are used.

4.

Consolidated financial statements: used when more than 50% of an entity is controlled. The assets and liabilities of the acquired company (subsidiary) are added to those of the parent corporation. The acquired company’s revenues


Chapter 8 and expenses are included in the income statement of the parent. See appendix 8A for further details. 5.

Goodwill. a)

Asset on the balance sheet that arises when there is a business acquisition.

b)

Goodwill is not amortized but is subject to being written-off under the impairment test. (1) Highly subjective test. (2) Permits “earnings management”. (3) Difficult to audit.

6.

The equity method or consolidated statements? a)

The method used will effect financial statements (1) Under equity method the liabilities of the acquired company are not included in the financial statements of the acquirer. (2) Effects ratios, especially debt to equity. (3) Equity method is considered by some to be a method of off-balancesheet financing.

b)

7.

Effect on financial statements may effect decision about what percentage interest should be acquired.

Special purpose entites (SPEs). a)

A separate entity created (1) The SPE raises capital by issuing debt or equity to investors and purchases property. (2) Property is leased to the sponsor. (3) Should the SPE be consolidated into the financial statements of the sponsor? (a) Not if sponsor doesn’t have control. (b) Can be difficult to determine who actually contols the SPE.

b) V. ROE Exercise

Difficult and controversial area – Enron fraud utilized SPEs.


Chapter 8 VI. Consolidated financial statements. (Appendix 8A) A.

B.

If the investor controls the investee company (defined as the investor owning greater than 50 percent of the investee's outstanding stock), then consolidated financial statements are prepared. 1.

The purpose of consolidated financial statements is to report the combined accounts of the investor and investee companies.

2.

Two types of investments. a)

Business acquisitions—the investor and investee combine for accounting purposes, but continue as separate legal entities.

b)

Business combinations or mergers—the investor and investee combine to form one legal entity.

Business acquisitions and mergers are accounted for using the purchase method. 1.

With the purchase method, the parent is assumed to be purchasing the subsidiary's assets and liabilities. The assets and liabilities should be valued at their fair market values.

2.

Accounting for consolidations using the purchase method. a)

Record the acquisition of the subsidiary at cost, which reflects the fair market value of the subsidiary's net assets by debiting the investment in subsidiary account.

b)

Eliminations and adjustments necessary to combine the balance sheets of the parent and subsidiary. (1) Eliminate the investment in subsidiary account. (2) Record subsidiary's assets and liabilities at their fair market values. (3) Record goodwill, if appropriate. Goodwill equals the excess paid over the parent's share of the fair market value of the subsidiary's net assets. Goodwill is no longer amortized. (4) Record minority interest, if appropriate. Minority interest arises when the parent purchases less than 100 percent of the subsidiary's outstanding voting stock. Minority interest represents the claims of the minority shareholders (i.e., shareholders other than the parent) on the fair market value of the subsidiary's net assets. Minority interest is calculated by multiplying the fair market value of the subsidiary's net assets by the percentage of outstanding voting stock owned by minority shareholders. Minority interest is usually reported on the balance sheet between long-term liabilities and shareholders' equity.


Chapter 8 (5) Eliminate intercompany receivables and payables, if appropriate. (6) Eliminate the investee's shareholders' equity. c)

Eliminations and adjustments necessary to combine the income statements of the parent and subsidiary. (1) The consolidated income statement should only reflect consolidated income earned subsequent to the acquisition date. (2) Intercompany revenues and expenses must be eliminated.

VII. Review problems 1 and 2. VIII. Ethics in the real world. IX. Internet research exercise.

LECTURE TIPS 1.

Understanding the need for the very different methods used to account for equity securities is aided by a general discussion of the nature of equity securities and the variety of reasons for companies to invest in them. It is important for students to understand the economic motivations and expectations behind each level of investment in order to understand the rationale for the accounting method. Figure 8–8 from the text (which summarizes the conditions under which the different methods are used) and end-of-chapter exercise 8–7 provide a vehicle to develop this understanding.

2.

Several end-of-chapter problems are particularly useful for demonstrating the different accounting methods for equity securities, especially through comparison with other methods. Problem 8–5 emphasizes trading versus available-for-sale classifications; problem 8–8 compares the mark-to-market method with the equity method; and problem 8– 9 compares the equity method to consolidated financial statements.

OUTSIDE ASSIGNMENT OPPORTUNITIES Group study of investments in equity securities both across time and within and across industries (continuing assignment for Chapters 6–14) 1.

Using the most recent annual report of a major public company in one of the four general industry groupings, identify or compute the items listed below for the two most recent years. Compare the items across time. Relate your findings to the economic characteristics and current conditions in the industry and the company’s strategy for competing. Prepare a written summary of your findings. Report findings in a class discussion session in which comparisons will be made both across time and within and across industries. Summary of the accounting policies relative to equity investments Captions and amounts reported for equity investments Percentage of each to total assets


Chapter 8 Income from equity investments (to the extent separately identifiable) Effect of market adjustments on net income, comprehensive income, and shareholder's equity Major types of consolidated subsidiaries Amount of minority interest in income and net assets Footnote disclosures relating specifically to investments Statement study on the impact of consolidation versus the equity method 2.

Obtain a recent annual report and/or Form 10–K of a bank holding company such as Bank of America Corporation. In conjunction with its consolidated financial statements, bank holding companies are required to present "parent-only" financial statements. The parentonly statements use the equity method to account for investments in the operating bank subsidiaries. Compare amounts for the following financial statement captions from the consolidated financial statements (for the most recent two years) to the amounts reported for the same captions in the parent-only statements. Explain the nature and significance of the differences. Total assets Total shareholders' equity Total income Net income Also, on the parent-only statement of cash flows, compare cash flow from operations to net income for the two most recent years, and explain the differences. Note to instructor: Total shareholders' equity and net income will be identical for each. Total assets and income on a consolidated basis will be many times greater than on the parent-only financials, because the bank subsidiaries actually collect the deposits and make most of the loans and investments. Cash flow from operations on the parent-only financials will usually be significantly less than net income, because only a portion of earnings of the bank subsidiaries are distributed to the parent in the form of cash dividends.

Research of accounting literature: accounting for goodwill in other countries 3.

Research current accounting literature for articles that cover accounting standards for goodwill in one or more countries outside the United States. Compare those standards to U.S. standards. Consider the impact of differences noted on decision-making by both users and management. Prepare a written and oral report of the findings.

ANSWERS TO IN-TEXT DISCUSSION QUESTIONS 310.

Short-term investments are listed in the current assets section of the balance sheet. To be considered for such treatment, the investments must be both readily marketable and management must intend to convert the investments into cash within the time period of current assets (one year or the operating cycle, whichever is longer).

311.

Other long-term assets would be listed in the noncurrent section of Intel’s balance sheet. To be treated as current assets, investments must be marketable; therefore, nonmarketable securities must be shown in the noncurrent section.


Chapter 8

311.

The accounting standard requires that management has the intention to convert the investments to cash in order justify the current classification. PepsiCo may not have violated the standard, even though one of the investments was not sold in the following year, if management’s honest intention was, and remains, to sell the investment. The difficulty is that such a determination is at best subjective, and managers have an incentive to overstate current assets through such classifications.

313.

Dividend income of $.2 million means that Biomet received cash dividends of that amount from investments in equity securities. Realized gains from short-term investments arose from the sale of investments at amounts $2 million greater than the cost of those securities.

315.

With $18.4 billion invested in marketable securities, Apple’s balance sheet could suffer from declines in the market. If most of the securities were treated as available for sale, a decline in the market value of the securities would not affect the income statement or the statement of cash flows. Conversely, improvements in the market could cause an increase in market values and an improvement in the balance sheet (but not the income statement or the statement of cash flows). If the securities were classified as trading by Apple’s management, the market value fluctuations in the portfolio of securities would affect the income statement.

316.

Unrealized gains (or losses) on available-for-sale securities are considered permanent accounts and are carried in the shareholders’ equity section of the balance sheet. If the investments were considered trading securities, these gains or losses would be considered temporary accounts, would appear on the income statement, and would be reflected in retained earnings.

316.

The net unrealized losses on available-for-sale securities would be reflected in the statement of shareholders’ equity. If the equity investments were considered trading rather than available-for-sale, the losses would be reflected in the income statement.

317.

An analyst would be interested in the amount of comprehensive income because it provides an estimate of the overall change in a company’s wealth during a period from other than investments by owners or distributions to them. The wide variety of options for disclosure of comprehensive income would not be of great concern, because the standard for determination of the amount of comprehensive income is the same for all companies, and the statement of comprehensive income must be prominently displayed.

318.

The acquisitions of Kinko’s and Mail Boxes, Etc, by FedEx and UPS were strategic in nature and were not motivated by a desire to generate income from dividends or to benefit from stock appreciation. These acquisitions were made to take advantage of perceived synergies between the investor corporations and the acquired corporations and perceived competitive advantages that would flow from these synergies.

320.

Intel’s equity investment constituting only 6% or 14% of the affiliate company’s outstanding common shares would be accounted for under the equity method if Intel had the ability to significantly influence the operating decisions and management policies of the affiliate. Such influence might be indicated by representation on the board of directors, interchange of management personnel, the frequency and nature of transactions between companies or other factors.


Chapter 8

321.

The investments by Boeing and Lockhead Martin Corp in United Launch Alliance were accounted for using the equity method. Each of these companies reported their investment as an asset, investments, in the case of Boeing and “other assets” in the case of Lockhead Martin.

321.

The increase in the investment account could have been caused by 1) new investments in equity securities, by 2) Lilly’s pro-rata share of earnings reported by the corporations Lilly had invested in, or by 3) cash dividends received by Lilly from its equity method affiliates. The investment in equity securities account is adjusted when new securities are acquired, when old securities are sold (or written-off), when dividends are received from equity method affiliates, and (annually) for the pro-rata share of income (or loss) of equity method affiliates

322.

The $819 million overall profit in its equity method investments would have included Coca-Cola’s share of the $2.9 billion profit reported by the bottling companies which were owned between 23 and 29% during 2012. This is a boost to its income. Alternately, the cash received is Coca-Cola’s percentage of dividends paid to all shareholders by the bottling companies. Cash flow represented from ownership of these equity investments was far less than the income recognized. Income from equity investments appears in the Operating Income section of the income statement and the adjustment to cash flow appears as a negative item in the Operating Activities section of the statement of cash flows.

324.

Target Corporation’s financial statements are referred to as consolidated because they are the combined (consolidated) financial statements of Target Corporation and Target’s controlled subsidiaries.

326.

The quote from the footnotes in General Electric’s recent financial statement means that net income was reduced by a write-off of goodwill in the amount of $1.2 billion. The write-off was taken because the goodwill that had been recorded in connection with the acquisitions of GECS IT Solutions and GECS U.S. Auto and Home was discovered to have little or no value. Consequently, under the impairment test, it was necessary to write the goodwill associated with these two acquisitions down to its actual (negligible) value.

328.

Coca-Cola’s 27% interest in the bottling companies is currently carried using the equity method. If Coca-Cola increased its ownership interest beyond 50%, the assets and liabilities of the bottling affiliates would be incorporated into the consolidated financial statements and Coca-Cola’s ratio of liabilities to total assets would go down. The bottling companies have a liabilities to total assets ratio of .51 (14.7 billion / 28.7 billion) compared to Coca Cola’s liabilities to total assets ratio of .62 (53.4 billion / 86.2 billion)

328. Enron used the SPEs to hide liabilities. Because the SPEs were not consolidated into Enron’s financial statements, the liabilities and the losses of the SPEs were not disclosed. Non-disclosure of this information misled investors. 330. Before the financial statements of foreign subsidiaries can be consolidated into the financial statements of the consolidated group, they need to be translated from the local currency into US dollars. Because of fluctuations in exchange rates between the foreign currency and the US dollar, the translation process usually reveals gains or losses due to


Chapter 8 these currency fluctuations. These gains and losses are known as foreign currency translation adjustments. Depending on the type of subsidiary, the foreign currency translation adjustments are reported in either the consolidated income statement (for Type I subsidiaries) or in the statement of comprehensive income (for Type II subsidiaries). The +$412 million currency adjustment cumulative balance reported on the balance sheet reflects the sum of all the currency translation adjustments for Type II foreign subsidiaries over the life of the consolidated entity. The +$300 million currency translation adjustment reported on the 2012 statement of comprehensive income reflects the adjustment for the year 2012. The balance of the currency translation adjustment on the 2011 balance sheet would have been approximately $112 million. CHARACTERISTICS OF END-OF-CHAPTER ASSIGNMENTS Item

Difficulty

Description

Brief exercises: BE8–1 M Short-term investments BE8–2 M Available for sale securities BE8–3 M Equity method, the income statement, and statement of cash flows BE8–4 E Goodwill BE8–5 M Goodwill accounting Exercises: E8–1 E Accounting for short-term equity securities E8–2 M Mark-to-market accounting E8–3 M Mark-to-market accounting E8–4 M Activity in the short-term investment account across time periods E8–5 H Available-for-sale disclosures E8–6 H Mark-to-market accounting and reporting problems: available-for-sale securities E8–7 E Classifying and accounting for equity investments E8–8 E The cost method E8–9 M Applying the mark-to-market rule E8–10 E The equity method E8–11 M Inferring information from equity method disclosures E8–12 M Inferring information about the equity method from the financial statements E8–13 M Recording an acquisition under the purchase method E8–14 M Purchases in excess of the net market value of the assets and liabilities E8–15 M Per-share book and market value E8–16 M Computing goodwill and noncontrolling interest E8–17 H Completing a consolidated worksheet Problems: P8–1 E Applying the mark-to-market rule to investments in equity securities P8–2 M Accounting for Trading securities P8–3 M Changing security investment classifications P8–4 M Window dressing and the mark-to-market rule P8–5 M Trading versus available-for-sale classifications P8–6 M Inferring from balance sheet disclosures P8–7 M Inferring information about trading and available-for-sale investments P8–8 H Long-term equity investments: the mark-to-market method versus the equity method P8–9 M The equity method versus consolidated financial statements


Chapter 8 P8–10 P8–11 P8–12 P8–13 P8–14 P8–15 P8–16

H Inferring information from the financial statements M Purchase and the recognition of goodwill H Noncontrolling interest and no goodwill H Noncontrolling interest and goodwill M Allocating the excess purchase price among tangible assets and goodwill H Noncontrolling interest and goodwill M Consolidated statements, the equity method, and debt covenants

Issues for discussion: ID8–1 M Equity adjustments for marketable securities ID8–2 M Short-term equity investments classified as available-for-sale ID8–3 M Equity method ID8–4 M IFRS financial statements and the equity method ID8–5 M Equity method and the fair market value option ID8–6 E Evaluating the equity method ID8–7 M Consolidating a finance subsidiary's financial statement: economic consequences ID8–8 M Classifying marketable securities ID8–9 M Earnings due to selling investments ID8–10 M Available-for-sale and earnings management ID8–11 H Comprehensive income and its components ID8–12 M Noncontrolling interest ID8–13 M The annual report of Google


CHAPTER 9

Long-Lived Assets

SYNOPSIS In this chapter, the author discusses (1) accounting for the acquisition, use, and disposal of long-lived assets, and (2) management's incentives for selecting accounting procedures associated with long-lived assets. The specific issues discussed in accounting for long-lived assets include the costs to capitalize when the asset is acquired, the treatment of postacquisition costs, cost allocation, and the disposal of long-lived assets. The discussion of cost allocation focuses on the alternative depreciation methods for fixed assets and the activity method for natural resources. The author also discusses intangible assets, deferred costs, and international financial reporting standards as they relate to property, plant and equipment. The ethics vignette considers a case where management's methods of accounting for long-lived assets may be within the guidelines of GAAP but still leave the statement reader with vague and insufficient information. The ethics vignette looks at the variations of accounting principles allowed under GAAP and if companies disclose enough information for users of financial statements related to depreciation and long-lived assets. The Internet research exercise examines the FASB's recent financial accounting standard for asset impairments. The following key points are emphasized in Chapter 9: 1.

How the matching principle underlies the methods used to account for long-lived assets.

2.

Major questions that are addressed when accounting for long-lived assets and how the financial statements are affected.

3.

Major economic consequences associated with the methods used to account for long-lived assets.

4.

Costs that should be included in the capitalized cost of a long-lived asset.

5.

Accounting treatment of postacquisition expenditures.

6.

How the cost of a long-lived asset is allocated over its useful life and the alternative allocation methods.

7.

Disposition of long-lived assets.

8.

The increasing importance of fair market value and issues that must be addressed when using fair market value as a basis for long-lived assets.

TEXT/LECTURE OUTLINE Long-lived assets.


Chapter 9

I. Assets used in the operations of a business (not for resale) that provide benefits to the company extending beyond the current operating period. Long-lived assets include the following: A.

Land.

B.

Fixed assets (i.e., buildings, machinery, and equipment).

C. Natural resource costs. D. Intangible assets. E. II.

Deferred costs.

The relative size of long-lived assets.

III. Long-lived asset accounting: general issues and financial statement effects. A.

B.

Three basic questions when accounting for long-lived assets. 1.

What dollar amount should be included in the capitalized cost of the long-lived asset?

2.

Over what time period should the cost be amortized?

3.

At what rate should the cost be amortized?

An overview of long-lived asset accounting—acquisition, use, and disposal.

IV. Acquisition: what costs to capitalize? A. As a general rule, all costs necessary to get the asset in a serviceable and usable condition and location should be capitalized. The allowable costs of capitalization vary across long-lived assets. B.

The capitalized cost of land should include: 1.

Purchase price of the land, including closing costs (such as title, legal, and recording fees).

2.

Costs incurred to prepare the land for its intended use such as razing old buildings, grading, filling, and so forth. Any proceeds received from preparing the land for its intended use (such as the sale of scrap from a demolished old building) reduce the capitalized cost of land.

3.

Permanent land improvements such as landscaping, sewers, drainage systems, and so forth.

C. Lump sum purchases.


Chapter 9 1.

As a general rule, allocate the purchase price to the assets based on their relative fair market values. This method requires that the fair market value of each asset acquired be known.

2.

Because fair market values are somewhat subjective, using fair market values to allocate the purchase price among the individual assets acquired gives management discretion about how much to report for each asset. This discretion, in turn, can affect the timing of expense recognition and certain key ratios.

D. Construction of long-lived assets. V.

Post-acquisition expenditures: betterments or maintenance? A.

Betterments—costs incurred to improve an asset. 1.

2.

B.

To be considered a betterment, an expenditure must provide one of the following: a)

An increase in the asset's useful life.

b)

An improvement in the quality of the asset's output.

c)

An increase in the quantity of the asset's output.

d)

A reduction in the costs associated with operating the asset.

Betterments should be capitalized as part of the long-lived asset and amortized over the asset's remaining useful life.

Maintenance expenditures—costs incurred to repair or maintain an asset's current level of productivity. Maintenance expenditures are considered period expenses.

VI. Cost allocation: amortizing capitalized costs. A.

The purpose of cost allocation is to match the costs of long-lived assets against the benefits the assets helped generate.

B.

Amortization terms. 1.

Depreciation—the amortization of fixed assets.

2.

Depletion—the amortization of natural resource costs.

3.

Amortization—the allocation of intangible assets and deferred costs.

C. Steps in the allocation process. 1.

Determine over what period of time the long-lived asset will help generate benefits. This step requires the company to estimate the asset's useful life. Estimating an asset's useful life is very subjective and is complicated by technological advances.


Chapter 9

2.

3.

Determine the dollar amount of the asset's capitalized cost that the company expects to consume over the asset's estimated useful life. a)

This step requires the company to estimate the asset's salvage value. Estimating an asset's salvage value is extremely difficult. In practice, many managers simply assign a salvage value of zero to assets because of the difficulty in estimating a reasonable salvage value.

b)

The dollar amount of the asset's capitalized cost that the company expects to consume over the asset's estimated useful life is called the asset's depreciation base (for fixed assets). The depreciation base equals the capitalized cost of the asset less its estimated salvage value.

Select a cost allocation method. a)

The choice of cost allocation methods does not affect the dollar amount amortized over a long-lived asset's estimated useful life. The dollar amount that is amortized over the asset's life is always its depreciation base.

b)

A cost allocation method determines the rate at which an asset's capitalized cost will be amortized to future periods. That is, the total amount amortized over an asset's life is the same across allocation methods, but the timing of the allocations will differ.

D. Cost allocation (depreciation) methods. 1.

2.

3.

Straight-line method. a)

Allocates an equal amount of the depreciation base to each period.

b)

Depreciation expense equals: (Capitalized cost minus Salvage value) ÷ Estimated life in years.

Double-declining-balance method. a)

Allocates larger amounts of an asset's depreciation base to the earlier periods of the asset's life than to the later periods.

b)

Salvage value is not used to calculate depreciation expense, but the asset's book value cannot be reduced below its salvage value.

c)

Depreciation expense = (Book value x 2) ÷ N, where N = the entire estimated useful life.

Activity method (units-of-production). a)

Allocates an equal amount of the cost to each unit of output. This method is most commonly used to deplete natural resource costs.

b)

Expense = [(Capitalized cost – Salvage value) ÷ Estimated life in units of production] x Number of units produced.


Chapter 9

E.

Economic effects of cost allocation methods. 1.

Cost allocation methods and the matching principle.

2.

How does management choose an acceptable cost allocation method?

3.

Depreciation methods for income tax purposes.

VII. Disposal: retirements, sales, and trade-ins. A.

B.

General rules for accounting for the disposal of fixed assets. 1.

Depreciate the asset up to the date of disposal.

2.

Remove the cost and related accumulated depreciation (for fixed assets) from the books.

3.

Record the receipt or payment of cash (or other assets) involved in the disposal.

4.

Recognize any gains or losses on the disposal. Gains and losses are computed as the difference between the disposed asset's book value and the net value of assets received.

Methods to dispose of fixed assets. 1.

Retirement; discontinue using the long-lived asset.

2.

Sale; exchange a long-lived asset for cash or a receivable.

3.

Trade-in; an exchange for dissimilar assets. a)

Exchanging a long-lived asset (and often cash) for a different type of long-lived asset.

b)

It is often difficult to determine the value of the new asset, which, in turn, affects the magnitude of any gains or losses on the transaction. (1)

As a general rule, value the new asset at the fair market value of the assets given up or at the fair market value of the assets received, whichever is clearly more evident and objectively determined.

(2)

Accounting for trade-ins is governed by APB No. 29, "Accounting for Nonmonetary Transactions."

VIII. Intangible assets. A.

Assets characterized by rights, privileges, and benefits of possession rather than by physical existence.


Chapter 9

B.

Intangible assets and deferred costs should be amortized, in certain instances, over the shorter of the asset's legal life or the asset's useful life.

C. The preferred method to amortize intangible assets is the straight-line method. D. Types of intangible assets. 1.

Copyrights—exclusive rights over musical or artistic works. The legal life is the life of the creator plus an additional 50 years.

2.

Patents—provide the holder exclusive rights to use, manufacture, or sell a product or process. The legal life is 10 years.

3.

Trademarks (or trade name)—a word, phrase, or symbol that identifies an enterprise or product. The legal life is 20 years but the exclusive right can be renewed indefinitely.

4.

Computer software development costs.

5.

Goodwill – The nature of goodwill and the methods used to account for it are discussed in Chapter 8.

E.

Organizational costs - costs incurred prior to the start of a company's operations that are necessary to allow the company to begin operations. Such costs typically include legal and accounting services, licenses, underwriting fees, titles, and so forth.

F.

Research and development costs. 1.

Governed by SFAS No. Costs."

2, "Accounting for Research and Development

2.

Conceptually, some R&D costs provide future benefits and would be capitalized as an asset and amortized as the R&D provides benefits. However, given the difficulty of matching specific expenditures with associated benefits, the FASB mandated that all R&D costs should be treated as period costs.

IX.

IFRS vs. US GAAP: revaluations to fair market value.

X.

ROE exercise

XI.

Review problem

XII. Ethics in the real world. XIII. Internet research exercise.


Chapter 9 LECTURE TIPS 1.

Students often have difficulty in deciding which costs should be capitalized as part of a long-lived asset (such as the cost of demolishing an old building on land just acquired) and which costs should be expensed. It should be stressed that for some costs, a great deal of judgment is involved. However, if the cost is necessary to get the asset into a useable condition, the cost should be capitalized. End-of-chapter exercise 9–3 and problem 9–1 are helpful in addressing which costs to capitalize.

2.

The straight-line method is usually easily understood. However, the double-decliningbalance (DDB) method often poses more problems to students. The primary problems are (1) computing the appropriate multiplication factor to multiply against the asset's book value, and (2) depreciating the asset below its salvage value. End-of-chapter exercises 9–9 and 9–10 can be used to illustrate the computations for each method.

3.

Distinguishing between betterment and maintenance expenditures is also a difficult area. It should be pointed out that even practicing accountants face this problem because a great deal of judgment is involved in classifying postacquisition costs. End-of-chapter exercise 9– 4 provides several situations to illustrate the concepts involved.

4.

Distinguishing between depreciation expense and accumulated depreciation sometimes poses difficulty. It should be stressed that depreciation expense represents the cost of the fixed asset allocated to expenses for a particular accounting period; accumulated depreciation represents the total depreciation expense taken on a fixed asset since that fixed asset was acquired. Recording the entries in end-of-chapter exercise 9–10 in Taccounts will help the student distinguish between the two.

OUTSIDE ASSIGNMENT OPPORTUNITIES Group study of long-lived assets both across time and within and across industries (continuing assignment for Chapters 6–14) 1.

Using the most recent annual report of a major public company in one of the four general industry groupings, identify or compute the items listed below for the two most recent years. Compare the items across time. Relate your findings to the economic characteristics and current conditions in the industry and the company’s strategy for competing. Prepare a written summary of your findings. Report findings in a class discussion session in which comparisons will be made both across time and within and across industries. Summary of the accounting policies for long-lived assets, including depreciation, depletion, and amortization methods and rates Property, plant, and equipment (net) as a percentage of total assets Identifiable intangibles as a percentage of total assets Goodwill as a percentage of total assets R&D expense as a percentage of total revenues Depreciation, depletion, and amortization expenses as a percentage of total revenues Summary of any impairment losses recognized


Chapter 9

Article search: application of accounting standards for impairment of long-lived assets 2.

Search the financial press (The Wall Street Journal, Barrons, Fortune, Forbes, etc.) for an article reporting on a company that applied the accounting standards for impairment of long-lived assets. Consider the impact on its current and future financial statements, both from a user orientation and an economic consequences perspective, and present your findings in writing and/or orally to the class.

ANSWERS TO IN-TEXT DISCUSSION QUESTIONS 358.

The importance of fixed asset accounting to the financial statements of the companies mentioned would be ranked in the order the companies were named: Chevron, Yahoo, and the Goldman Sachs. A natural resources company such as Chevron is highly capital intensive, with large investments in property, plant, and equipment. An Internet company such as Yahoo has electronic equipment, as opposed to extensive properties or heavy equipment. Banks such as the Goldman Sachs have large financial assets on their balance sheets which far outweigh fixed assets.

360.

By choosing to depreciate their aircraft over a longer useful life, United spreads the cost of the aircraft over a greater number of reporting periods. This would serve to decrease depreciation expense and increase reported income compared to Delta, which is using a shorter useful life for its aircraft. Delta’s net PPE assets would be greater on the balance sheet as there would be less accumulated depreciation on their equipment. There would be no effect on the statement of cash flows because depreciation is a non-cash expense.

361.

By extending the depreciation lives of its aircraft, American Airlines chose to spread the cost of the aircraft over a greater number of reporting periods. This would serve to decrease depreciation expense and increase reported income. There would be no effect on the statement of cash flows because depreciation is a non-cash expense.

362.

Zimmer’s investments in property, plant and equipment would be reflected as negative cash flows in the investing section of the statement of cash flows. These assets are accounted for as long term assets on the balance sheet and will be subject to depreciation over their useful lives.

363.

The fair market value of each identifiable current and noncurrent asset, and of each liability, would first be determined. Monetary assets (cash, securities, receivables) would be valued at face value, net realizable value, present value, or quoted market prices. Liabilities would be valued at face value or present value. The value of nonmonetary assets (inventories, land, buildings, equipment, and identifiable intangible assets) would be determined using replacement costs, comparable sales values, or the present value of estimated future cash flows.


Chapter 9

The fair market value of identifiable net assets as determined in the preceding paragraph would be deducted from the purchase price. The excess of the purchase price over net assets would be assigned to goodwill. In the case of Kellogg’s purchase of Worthington Foods, goodwill of $194 was arrived at by subtracting net assets of $156 million (assets valued at $218 million, minus liabilities valued at $62 million), from the purchase price of $350 million. The valuations required in this process are inherently subjective and offer opportunities to either maximize or minimize future income. For instance, valuations which give lower values to identifiable net assets, result in a higher value assigned to goodwill, which is not amortized against future income. Within identifiable net assets, a higher value assigned to assets such as land result in greater future income because land is not depreciated. Conversely, a higher value assigned to inventory results in a larger cost of goods sold in the next period, thus reducing income. 364.

The interest costs of the funds borrowed by AT&T that related to the period of time during which the facilities were under construction would have been capitalized as part of the cost of those facilities. This is because these costs were incurred in the process of getting the facilities into operating condition. The interest expense that applied to the post construction period would have been treated as an operating expense in the year incurred.

364.

By capitalizing costs that should have been expensed, WorldCom understated its expenses and overstated net income for the period during which this fraud took place. Net income in future periods would have been reduced through depreciation, had the fraud not been discovered.

365.

Renewals and improvements are betterments and are capitalized because they improve the asset by extending its useful life, improve the quantity or quality of its output, or reduce the cost of operating the asset. Repairs are considered part of maintenance and are expensed as incurred, because they do not improve the asset or extend its useful life.

366.

The accounting expert is referring to estimates of salvage value, the dollar amount that can be recovered when a long-lived asset is sold, traded, or scrapped. Salvage value reduces the depreciable cost of an asset, thus reducing the amount of depreciation expense recognized in each period subsequent to acquisition of the asset. The expert is warning financial statement users interested in reported income that income may be overstated to the extent that salvage values are overstated and depreciation understated. The warning recognizes management’s discretion and incentive to make optimistic assumptions of salvage values.

366.

The ranges of useful lives vary for these two broad categories of assets because buildings generally last a lot longer than machinery or equipment. The broad ranges of lives used by management gives them a certain amount of flexibility in managing earnings by choosing useful lives that extend or compress the time period during which newly acquired assets are to be depreciated. The extent to which the choice of useful life will impact current year net income is usually minor unless there was a relatively large dollar amount of new asset acquisitions.


Chapter 9 367.

An analyst comparing PepsiCo to Coca-Cola would want to factor in the differences in the useful lives assigned to assets being depreciated. The change in depreciation schedules is a way to make earnings look better when they have not improved at all. The depreciation is not relevant to cash flow, and PepsiCo’s response to any comments about the change in their depreciation schedule could be deflected by focusing on cash flow. PepsiCo might also state that the adjustment to the depreciation schedules was necessary to reflect the reality that their assets are being used for longer periods of time.

368.

If they had $5.4 billion in fixtures and equipment, none of which were fully depreciated, and they used an average life of 9 years, the annual depreciation expense (straight line method) for these fixtures would be approximately $5.4 billion divided by 9, or $600 million. This, of course, is a very rough estimate because it assumes an average life of 9 years which could be significantly different than the true weighted average life of the assets being depreciated.

370.

Conforming to industry practice makes it easier to compare HP’s financial statements to those of their competitors. Straight-line would do a better job of matching costs with revenues than accelerated methods if assets being depreciated made equal contributions to revenues during each year of their useful lives.

371.

If everything about each company was exactly the same except for the method each used for depreciation, reported income would differ, as would related balance sheet amounts. Accordingly, financial statement ratios used as performance measures for each company would differ. Analysts must consider these differences when making comparisons within the industry, either qualitatively, or by attempting to make computational adjustments to make each company comparable. When such differences exist, analysts may wish to focus more on performance measures not dependent on the amount of depreciation, such as cash flow from operations, or earnings before interest, taxes, depreciation and amortization (EBITDA).

372.

The activity (units-of-production) method would be used for oil and gas producing assets, which has the effect of assigning more costs to expense in proportion to the asset’s level of activity. This method achieves the best application of the matching principle, because the more active an asset is, the more revenue it should produce. This is particularly true in the case of natural resources like oil and gas. The straight-line method would be used for other property, plant, and equipment. The benefits from those assets are less directly related to activity, and more likely constant across time, so an even amount of expense each year is appropriate. The 25 and 45 year lives assigned to refining and pipeline assets probably reflect with some degree of accuracy the actual useful lives of these types of assets.

373.

Delta could choose between straight-line or accelerated depreciation methods. Straightline depreciation, particularly in earlier years, would result in higher reported net income and higher shareholders’ equity and total assets, compared to results using accelerated methods. Therefore, performance measures based on net income, and liabilities to equity or assets ratios, would appear more favorable using straight-line, and debt covenants would less likely be violated.

374.

By using accelerated depreciation methods for taxes, small businesses can decrease their tax liabilities. When companies pay less tax they are able to keep more cash in the near term. When small businesses have more cash, they will theoretically increase


Chapter 9 spending by investing in the business and potentially hiring more employees. increases over economic activities and aids in stimulating the economy.

This

375.

The IRS took the position that the work on the jet engines should be treated as betterments rather than as routine maintenance. If FedEx has a tax rate of 37.3%, treating these costs as expenses increases FedEx’s cash flow (by way of reduced income tax payments) by 37.3% of the amount of the expenditures in question.

378.

There would have been a one time gain recorded on the sale of these assets. This stream of income would not be expected to recur and, if material, would be reflected in the income statement as a gain form the disposal of a segment of the business, rather than as part of operating income.

379.

The journal entry recorded by Honeywell would have been: DR Cash and investment securities (+A) Loss on disposal of assets (+E, -SE) Assets (-A)

379.

CR $435 $124

“Customer relationships” is a sub-category of goodwill and is one of the biggest reasons for paying more for the acquired company than the sum of the fair market value of its tangible assets. Because the goodwill was developed internally by Holsum, rather than purchased, it was not an asset on Holsum’s balance sheet. However, when Flowers Foods acquires this goodwill in the acquisition and pays for it, Flowers has an asset to reflect on its balance sheet.

CHARACTERISTICS OF END-OF-CHAPTER ASSIGNMENTS Item

$559

Difficulty

Description

Brief exercises: BE9–1 E Change in depreciation method BE9–2 M Depreciation and amortization BE9–3 M Acquisition of fixed assets BE9–4 M IFRS, long-lived assets, and the statement of cash flows Exercises: E9–1 E Determining the capitalized cost and depreciation base E9–2 E Allocating cost on the basis of relative market value E9–3 M Which costs are subject to depreciation? E9–4 E Betterments or maintenance? E9–5 M Matching principle and income recognition E9–6 M Effect of estimated useful life on income and dividends E9–7 M Revising the estimated life E9–8 E Different amortization methods achieve different objectives E9–9 M Computing depreciation and choosing a depreciation method E9–10 M Depreciation calculations and journal entries E9–11 M The activity method of depreciation E9–12 M Depletion and matching


Chapter 9 E9–13 H E9–14 M E9–15 M E9–16 H E9–17 H E9–18 H E9–19 M E9–20 M E9–21 M E9–22 M

An error in recording the acquisition of a fixed asset Fixed asset sales Retiring, selling, and trading in a fixed asset Reverse T-account analysis Inferring information from the financial statements Reverse T-account analysis Intangible assets: expense or capitalize and amortize? The capitalized cost of a patent Recognition of goodwill GAAP versus IFRS regarding fair market values

Problems: P9–1 E How much to capitalize? P9–2 M Lump-sum purchases and cost allocations P9–3 M Determining capitalized cost and depreciation P9–4 H Expensing what should be capitalized can misstate net income P9–5 M Accounting for betterments and maintenance costs P9–6 M Accounting for betterments P9–7 M Revising the estimated useful life P9–8 M Effects of an accounting change P9–9 H Why is double-declining-balance preferred for tax purposes? P9–10 M The effect of depreciation on taxes, bonuses, and dividends P9–11 M Different methods of cost allocation: natural resources P9–12 M Selling and trading in fixed assets P9–13 M Recognizing goodwill P9–14 M Goodwill accounting Issues for discussion: ID9–1 M Lump-sum sales and purchase ID9–2 M Capitalizing marketing costs ID9–3 M Capital acquisitions ID9–4 M GAAP versus IFRS ID9–5 M Expense vs. capitalize ID9–6 M Capital spending and profits ID9–7 H Asset impairments ID9–8 M Corporate restructuring ID9–9 E Current values? ID9–10 E Subjective asset write-downs ID9–11 M Asset write-offs ID9–12 M GAAP versus IFRS ID9–13 M The annual report of Google


CHAPTER 10 Introduction to Liabilities: Economic Consequences, Current Liabilities and Contingencies

SYNOPSIS In this chapter, the author discusses (1) the economic consequences of liabilities in general and of current liabilities in particular; (2) measuring and reporting current liabilities; and (3) accounting for contingencies. The discussion on current liabilities includes accounts payable, short-term notes payable, current maturities of long-term debt, dividends payable, deferred revenues, third-party collections, and accrued liabilities. Accrued liabilities cover both determinable liabilities and conditional liabilities such as bonuses. The discussion of contingencies outlines the alternative approaches to accounting for contingencies as specified in SFAS No. 5, "Accounting for Contingencies" and the costs and benefits associated with the various alternatives. The specific types of contingencies discussed are lawsuits, environmental costs, and warranties. The author discusses pensions and postretirement health care and insurance costs in Appendix 10A and deferred income taxes in Appendix 10B. The ethics vignette considers a company that chooses the lower end of a range of costs when estimating the level of expense to record for contingent losses related to environmental liabilities. The Internet research exercise examines SFAS No. 5 and how it applies to accounting for environmental costs at Waste Management. The following key points are emphasized in Chapter 10: 1.

Definition of a liability.

2.

Economic consequences associated with reporting liabilities on the financial statements.

3.

Determinable and contingent liabilities.

4.

Current liabilities.

5.

Bonus systems and profit-sharing arrangements and the reporting incentives they create.

6.

Methods used to account for contingencies.

TEXT/LECTURE OUTLINE


Chapter 10

Introduction to liabilities: economic consequences, current liabilities, and contingencies. I.

What is a liability? Liabilities—defined by the FASB as "probable future sacrifices of economic benefits from present obligations of a particular entity to transfer assets or provide services to other entities in the future as a result of past transactions or events." Liabilities possess the following three characteristics: A.

Liabilities should be present obligations that will require probable future transfers or uses of cash, goods, or services.

B.

Liabilities should be unavoidable obligations.

C. Liabilities must arise from a transaction or an economic event that has already taken place. II.

Reporting liabilities on the balance sheet: economic consequences. A.

B.

Shareholders are affected by liabilities through: 1.

Seniority of interest payments over dividend payments.

2.

Debt covenants that influence investing, financing, and operating activities that a company may undertake.

3.

Seniority of creditors' claims on assets over shareholders' claims in the event of liquidation.

Creditors are concerned with: 1.

The seniority of liabilities.

2.

The presence of sufficient assets to cover liabilities.

3.

The protection of their investments through debt covenants.

C. Managers are concerned with liabilities because: 1.

Liabilities are an important source of capital for operating, investing, and financing activities.

2.

The amount of debt a company has and the ability to manage this debt affects a company’s credit rating.

3.

Debt covenants may restrict investing, financing, and operating decisions that the managers may make.

4.

The relation of liabilities to off-balance-sheet financing, taking a bath, and creating hidden reserves.


Chapter 10 D. Auditors are concerned with ascertaining that liabilities are not materially understated. Materially understated liabilities could result in the auditor being sued. E.

III.

Attempts by managers to minimize the amount reported as liabilities may lead to innovative financing arrangements resulting in off-balance-sheet financing.

Current liabilities. A.

Obligations expected to require the use of current assets or the creation of other current liabilities.

B.

Since current liabilities are short-term, the time value of money is ignored and current liabilities are carried at face value.

C. Determinable current liabilities. 1.

Liabilities that can be precisely measured; the amount of cash necessary to satisfy the obligation and the payment date are reasonably certain.

2.

Classifications. a)

b)

Accounts payable. (1)

The amounts owed to other entities for goods, supplies, and services purchased on open account.

(2)

Measurement issues are accounting for purchase cutoffs at the end of the accounting period and accounting for sales discounts.

Short-term debts. (1)

Short-term notes—usually carried at their face value less any discount. The discount represents interest on the note and is generally amortized (for short-term notes) using the straight-line method.

(2)

Current maturities of long-term debts—the portion of longterm debt that matures within the time frame of current assets (i.e., will be settled using current assets) should be reported as a current liability.

c) Dividend payable—the liability for dividends arises on the date the board of directors declares the dividend (See Chapter 12). d)

Unearned revenues.

e)

Third-party collections.


Chapter 10

f)

(1)

Sales taxes.

(2)

Payroll withholdings.

Accrued liabilities. (1)

Normal accrued liabilities.

(2)

Conditional liabilities. (a) Liabilities for which the amount of the obligation is conditional upon net income. Hence, the amount of the liability cannot be measured until the end of the accounting period. (b) Income tax liability—the actual amount owed for local, state, and federal income taxes. (c) Incentive compensation. i)

Incentive plans are one device that shareholders can use to try to align the manager's goals with their own goals.

ii)

Executive compensation and U.S. business in the global marketplace.

D. Contingencies and contingent liabilities. 1.

Contingencies are defined by the FASB as "an existing condition, situation, or set of circumstances involving uncertainty as to possible gain or loss to an enterprise that will ultimately be resolved when one or more future events occurs or fails to occur.” That is, an event has taken place as of the balance sheet date that may or may not have an economic effect on the company, but the actual effect on the company will not be known until sometime in the future.

2.

Accounting for contingencies. a) Contingencies are governed by SFAS No. 5, "Accounting for Contingencies." b)

Gain contingencies. (1)

A contingency whose resolution could be financially advantageous to the company (i.e., result in an increase in assets or a decrease in liabilities).

(2)

Gain contingencies are never accrued for in the financial statements. Under conservatism, gains are not recognized


Chapter 10 until the gain is realized. Gain contingencies may, if both reasonably estimable and probable, be disclosed in the footnotes. c)

Loss contingencies. (1)

A contingency whose resolution could be financially disadvantageous to the company (i.e., result in a decrease in assets or an increase in liabilities).

(2)

General accounting treatment for loss contingencies. (a) If an adverse resolution to the uncertainty is remote, then the contingency may be ignored for financial statement purposes. (b) If an adverse resolution to the uncertainty is reasonably possible, then the contingency, and all relevant information, should be disclosed in a footnote. (c) If an adverse resolution to the uncertainty is probable but the amount of the loss is not reasonably estimable, then the contingency, and all relevant information, should be disclosed in a footnote. (d) If an adverse resolution to the uncertainty is probable and the amount of the loss is reasonably estimable, then a loss should be accrued in the financial statements.

3.

Types of loss contingencies. a)

Lawsuits.

b)

Environmental costs.

c)

Warranties—the post-sale costs under the terms of the warranty should be estimated and then recognized as an expense and contingent liability in the period that the underlying sale took place.

IV.

Provisions vs. contingent liabilities: The “devil is in the details”.

V.

ROE exercise

VI.

Retirement costs: pensions and postretirement health care and insurance (Appendix 10A). A.

Pensions.


Chapter 10 1.

Money paid to a retired or disabled employee, the amount of which is usually determined by the employee's years of service.

2.

Pension plans are usually administered by someone outside the company.

3.

Types of pension plans. a) Defined contribution plan—the company promises to make periodic payments into the pension fund. The company only guarantees the contributions into the plan, not the benefits to be disbursed to employees. b) Defined benefit plan—the company guarantees a specified payout upon the employee's retirement.

B.

4.

In accordance with the matching principle, pension expenses should be recorded in the accounting period in which the employees earn the rights to the future benefits.

5.

A company's pension liability represents the difference between the company's contribution to the plan to date and the estimated amount necessary to meet the guaranteed employee benefits.

Postretirement health care and insurance costs.

VII. Deferred income taxes (Appendix 10B). A.

An account to reconcile the difference between a company's income tax expense and income tax liability. This difference arises due to differences in measuring net income under GAAP and the Internal Revenue Code. 1. 2.

B.

Tax liability is based on the company's tax rate and the company's net income using the Internal Revenue Code. Income tax expense is based on the company's tax rate and the company's net income using GAAP.

Theoretically, the amount reported as deferred income taxes will eventually be paid as income taxes as the income differences between GAAP and the Internal Revenue Code reverse. 1.

Managers may make certain investing and/or operating decisions that effectively postpone the reversal period, thereby indefinitely postponing paying these taxes.

2.

Changes in the statutory tax rates result in restatement of the balance in the deferred income tax account, thereby giving rise to recognized gains or losses.

C. The conservatism ratio provides a quick way to assess how conservative management's reporting choices have been in a particular year. It


Chapter 10 measures the extent to which reported income before taxes differs from taxable income. VIII.

Review problem

IX.

Ethics in the real world.

X.

Internet research exercise.

LECTURE TIPS 1.

The need for companies to be concerned with contingencies even though the company may not be affected should be discussed in the context of the objective of financial accounting (i.e., providing information to investors regarding a company's investment potential and credit worthiness). The diagram in figure 10–5 in the text is especially useful for an overview of accounting for contingencies. It should be emphasized that the event must first have occurred. It should also be pointed out that contingent losses can be booked as asset valuation accounts (e.g., allowance for bad debts) as well as liabilities. It is helpful to summarize typical types of loss contingencies into categories: Usually Accrued

Not Accrued

Maybe Accrued *

Potential loss from: Uncollectible receivables Product warranties Risk of loss from fire, flood, or other hazards General business risks Environmental costs Lawsuits

X X X X X X

*based on probable and estimable criteria

2.

Students' views often vary widely on numerical values for the degrees of probability expressed in accounting for contingencies. An interesting class exercise is to poll the class as to their numerical assignment of probabilities to the "remote,” "reasonably possible,” and "probable" expressions. A study** examined the differences among key groups of audit opinion users and professional auditors concerning the meaning of the term "substantial doubt" used in the audit literature for the going concern issue. The study employed a survey in which subjects were asked to assign probability values to the term "substantial doubt," and also, especially of interest here, to "possible" and "probable." Significant differences were noted among participants concerning "substantial doubt," but not for the "possible" and "probable" expressions. The mean response for auditors was 0.4067 for "possible" and 0.7296 for "probable." **Lawrence A. Ponemon and K. Raghunandan, "What is 'Substantial Doubt'?" Accounting Horizons, June 1994, pp. 44–54.


Chapter 10

OUTSIDE ASSIGNMENT OPPORTUNITIES Group study of current liabilities both across time and within and across industries (continuing assignment for Chapters 6–14) 1.

Using the most recent annual report of a major public company in one of the four general industry groupings, identify or compute the items listed below for the two most recent years. Compare the items across time. Relate your findings to the economic characteristics and current conditions in the industry and the company’s strategy for competing. Prepare a written summary of your findings. Report findings in a class discussion session in which comparisons will be made both across time and within and across industries. Total liabilities as a percentage of total assets Current liabilities as a percentage of total liabilities Captions used to report current liabilities Summary of accounting policies relating to current liabilities Summary of types of employee benefit plans and related accounting policies (Appendix 10A) Liabilities for pensions and postretirement health care and insurance benefits as a percentage of total liabilities (Appendix 10A) Summary of nature of items giving rise to deferred income taxes (Appendix 10B) Deferred income taxes as a percentage of total liabilities (Appendix 10B) Conservatism ratio (Appendix 10B)

Group study of contingencies both across time and within and across industries (continuing assignment for Chapters 6–14) 2.

Using the most recent annual report of a major public company in one of the four general industry groupings, discuss (1) its disclosure and accounting for the years presented for three categories of contingencies: warranties, environmental costs, lawsuits, and others; and (2) the associated potential economic consequences. Relate your discussion to the economic characteristics of and current conditions in the industry and the company’s strategy for competing. Prepare a written summary of your findings. Report findings in a class discussion session in which comparisons will be made both across time and within and across industries.

Research of current accounting literature on a topic relating to liabilities and contingencies 3.

Research current accounting literature for articles covering a current topic relating to liabilities and contingencies, e.g., employee postretirement benefit plans, executive compensation, or environmental costs. Focus on the impact of the issue on users of financial statements as well as its economic consequences effect on management decisions. Prepare a written and oral report of the findings.

Classroom game on executive incentive compensation


Chapter 10 4.

1

Obtain the Elitzur article (referenced below ) which describes a game on the subject of executive incentive compensation schemes. Play the game as a classroom exercise.

ANSWERS TO IN-TEXT DISCUSSION QUESTIONS 403. The FASB has established three criteria for recognition of an obligation as a liability: (1) it should be a present obligation that will be settled by probable future transfers of assets or provisions of services; (2) it should be an unavoidable obligation; and (3) the transaction or event obliging the enterprise must have already happened. In the case of an equipment lease, items (1) and (2) may have been met, but item (3) has not yet happened. A lease is an executory contract which is not completed except through future use. The lease obligation of FedEx of $1.9 billion is for the next years’ use of the equipment. This treatment was appropriate if the lease was not in fact a capital lease, as covered in Chapter 11. 403.

Companies like Kimberly-Clark and General Electric have substantial amounts of physical assets which warrant associated borrowings. General Electric also has a significant amount of financial assets which also support related borrowings; its balance sheet looks more like that of a financial institution than a manufacturer. Internet companies, on the other hand, do not have a significant physical or financial asset-base which would support borrowings. Also, their operations are inherently more risky. Accordingly, internet companies must rely more on equity capital than debt financing.

406.

Motorola may have engaged in a strategy of “taking a bath”, reporting maximum liabilities, and thereby accelerating recognition of related expenses, in the earlier years in order to be able to report higher earnings in future years. The fact that in later years Motorola bounced back, reporting a large profit, and that none of the restructuring losses recognized in the earlier years required any cash payments, suggests that Motorola purposely “built hidden reserves.” Investors should be concerned because they are not being provided a true picture of the earnings pattern of the company. Auditors are usually more concerned with the possibility of understated liabilities, but must also be concerned with overstated liabilities, especially where they are part of a strategy of earnings management.

407.

Because these short-term borrowings are intended to be refinanced, they are not expected to require the use of current assets and are therefore not disclosed as current.

407.

The Bank of New York’s current liabilities consist primarily of deposit liabilities to customers which are its largest source of funding. AT&T, on the other hand, would have more in the way of long-term debt than current, to finance its vast infrastructure of long-term physical assets.

R. Ramy Elitzur, “A Classroom Exercise on Executive Incentive Compensation Schemes,” Managerial & Decision Economics, November/December 1995, pp. 649–652. 1


Chapter 10 408.

Airlines adopted programs of giving away free flights to frequent flyers as an incentive to build customer loyalty. The awards were expected to be accommodated with otherwise-empty seats. The incremental cost to the airlines of a free flight was minimal, so liabilities were not recorded or recorded at only a small amount. The frequent-flyer programs have become so successful, that in some cases free-flight awards have to be satisfied by using seats that otherwise would have generated revenue. The immateriality of incremental costs is a less persuasive argument than it once was. Airlines already have high liabilities, with associated debt covenants, and also experience fluctuating profitability, so they have a strong incentive to minimize recorded liabilities, or to postpone liabilities whenever possible. The accounting for these programs remains a controversial subject.

410.

For companies like Proctor & Gamble, slowing down payment of its liabilities will improve is cash flow from Operating Activities. Accounts payable will go up. When accounts payable goes up it is reflected as a positive adjustment on the statement of cash flows (Operating Activities). Suppliers will see a negative impact in their statement of cash flows (Operating Activities) as their days to collect increases so will the balance in their accounts receivable.

410.

J.C. Penney is retailer in the business of selling merchandise. Its largest current asset would be inventory, which to the extent possible would be directly financed with accounts payable. Biomet is a manufacturer of medical products, for which the cost of products is not as key of a cost component as merchandise is to a retailer. Most likely Biomet’s highest costs are for research and development of new products, and marketing and administrative costs.

411.

Accounts payable are amounts due to vendors for goods, supplies, and services purchased on open account. Accrued expenses are amounts due for expenses, such as wages and interest, incurred but not yet paid. Short-term loans and notes payable arise from cash loans and are generally payable to banks or other lending companies within a year. Current maturities of long-term debt are installments of long-term debt which are currently due. Each category is classified as a current liability because it will be paid in the time period that defines current assets, that is, one year or the operating cycle, whichever is longer.

412.

The $18.6 billion in unearned revenues in Microsoft’s balance sheet indicates that Microsoft collects a significant amount of revenues in advance of performing the related services. These advance payments are usually a current liability because the services are to be performed within a year of the time the monies are collected. Microsoft has a legal obligation to perform these services under the terms of these pre-paid contracts between Microsoft and its customers.

412.

“Advance ticket sales” is unearned revenue. Airlines recognize revenue when transportation is provided rather than when a ticket is sold. The amount of passenger ticket sales not yet recognized as revenue is reflected as “Advance ticket sales” because the service has not yet been provided. Accounts payable, by contrast, represent fixed and determinable amounts due to vendors for goods, supplies, and services provided to United Continental Holdings.


Chapter 10 415.

The purpose of incentive compensation plans is to encourage managers and employees to act in a manner consistent with the objectives of the shareholders. Because these arrangements are designed to benefit of the shareholders, it is important for the shareholders to monitor these plans to assure they are consistent with shareholder interests.

415.

Executive compensation and bonuses in particular, were controversial in light of the taxpayer-funded bailout of the banks and other financial services firms. Because of the bailouts, taxpayers were indirectly footing the bill for these compensation payments, which many alleged were made in no small part to the same culprits that ruined the economy with their excesses. Some commentators suggested that executive compensation practices created incentives for the bankers to take on too much risk causing the collapse of the banking and financial services sector of our economy and creating the deepest recession in decades. High executive compensation is either necessary to attract the best talent, or a waste of the shareholders’ money, or somewhere in between these two extremes. The fact that high executive compensation is the norm suggests that argument, at least for the time being, has largely been won by the executives. Disclosures in the financial statements are necessary because incentive compensation measures often give management reason to manipulate the financial statements in order to increase their compensation. Financial statement users need this information in order to evaluate the credibility of management assertions contained in the financial statements.

417.

Lilly appears to be the plaintiff in the patent litigation mentioned. As plaintiff, they have a “gain” contingency because they might win. Gain contingencies are never recognized in the financial statements. The statement about the material adverse impact of an unfavorable outcome is not clear because it does not specify whether these adverse results will impact future operating results, liquidity etc. or the past operating results that are reflected in the current financial statements containing the disclosure. If the disclosure refers to past operating results, it would be because Lilly is also a defendant in this litigation. It is not uncommon for there to be countersuits in patent litigation.

419.

Bristol-Myers Squibb would have recorded the following entry to record the liability: DR CR Product liability losses (+E, -SE) Product liability claims payable (+L)

$162,000,000 $162,000,000

The accrual would have been justified because the expense was both highly probable and was estimable. For the year 2012 the accrual would not have required the use of cash, so the amount would not be a use of cash in arriving at cash flow from operations. In a cash flow statement using the direct approach for cash flow from operations, the amount would not be deducted; in a statement using the indirect approach, the amount of the liability would be added back to net income as a non-cash charge to arrive at cash flow from operations. 419.

FMC’s policy is in accordance with generally accepted accounting principles. When they believe that they need to record a liability on their financial statements


Chapter 10 and the amount of the liability is uncertain, but within a range, they use the lowest end of the range. This practice is likely to result in understatement of liabilities and overstatement of net income. A more conservative approach would be to report a liability in an amount that represents the mid-point of the range. FMC's policy is in accordance with FASB Interpretation No. 14, “Reasonable Estimation of the Amount of a Loss”. 420.

Estimated costs of warranties should be reflected as expenses in the period the tires are sold. Goodrich Corporation took on the responsibility for future warranty services at the time of the sale, and the associated estimated costs should be matched with the revenue recognized at the time of sale. Warranties are contingent liabilities which are almost always reflected as expenses on the financial statements, because they are highly probable and are estimable. Goodrich paid approximately $41.7 million for warranties in 2011 (beginning balance of $148.5 million, plus warranty expense for the year of $51.1 million, minus the ending liability of $157.9 million).

430.

General Electric’s conservatism ratio decreased by over 29% in 2012. This is a very conservative presentation. Management appears to be making their financial statements look worse, presumably hoping to show a burst of improvement in the succeeding year. More conservative accounting methods are being used and more conservative estimates are being employed. Close study of the footnotes and analysis of the differences between revenues and expenditures as reported on the tax returns versus revenues and expenditures as reported on the financial statements would reveal the details of what the differences are between GAAP net income and taxable income.

CHARACTERISTICS OF END-OF-CHAPTER ASSIGNMENTS Item

Difficulty

Description

Brief exercises: BE10–1 E BE10–2 E BE10–3 E BE10–4 E

Cash flow and accruals Inferring financial information Cash payments for environmental cleanup Cash flow implications

Exercises: E10–1 M E10–2 E E10–3 M E10–4 E E10–5 E E10–6 M E10–7 M E10–8 E E10–9 M E10–10 M E10–11 M

Face Value versus Present Value Financing with long-term debt and the current ratio Accruals, the current ratio, and net income Short-term notes payable and the actual rate of interest Current maturities and debt covenants Gift certificates and unearned revenue Inferring a cash payment Gain and loss contingencies Bonus plans and contingent losses Warranty costs: contingent losses or expense as incurred? Deferred revenues and cash receipts


Chapter 10 E10–12 E10–13 E10–14 E10–15

M H H H

Problems: P10–1 E P10–2 M P10–3 M P10–4 M P10–5 M P10–6 M P10–7 M P10–8 M P10–9 H P10–10 P10–11

H H

Pension contributions and unfunded pension liability Deferred taxes and the tax rate Conservatism ratio Conservatism ratio

Distinguishing current liabilities from long-term liabilities Recognizing current liabilities can restrict dividend payments Recognizing current liabilities and violating debt covenants Issues surrounding the recognition of a contingent liability Accruing warranty costs Advertising campaigns can give rise to contingent liabilities Restructuring charges and statement of cash flows Accruing and funding pension liabilities Deferred income taxes, changes in tax rates, and investment in long-lived assets Conservatism ratio Conservatism ratio

Issues for discussion: ID10–1 M Debt covenants and reporting current liabilities ID10–2 M Receipts in advance: measurement theory and financial statement effects ID10–3 M Contingency reporting in the healthcare industry ID10–4 M Contingency reporting in the tobacco industry ID10–5 M Unreported assets ID10–6 E The economic consequences of a technical default ID10–7 E Warranties and contingencies ID10–8 M Deferred revenue ID10–9 E Current liabilities ID10–10 E Using executive compensation disclosures ID10–11 M "Taking a bath" during bankruptcy proceedings ID10–12 M Conservatism ratio ID10–13 M Changes in expected tax rates and net income ID10–14 M Patent infringement, contingencies and the auditors ID10–15 M Provisions under IFRS ID10–16 M The annual report of Google


CHAPTER 11 Long-Term Liabilities: Notes, Bonds, and Leases

SYNOPSIS In this chapter, the author discusses accounting issues surrounding long-term liabilities. The specific issues discussed are (1) accounting for long-term notes payable; (2) accounting for bonds payable; (3) the differences between, and accounting for, operating and capital leases; (4) financial instruments and offbalance-sheet risks; and (5) the economic consequences faced by managers whose companies have long-term debt. The author discusses factors affecting bond prices in Appendix 11A. Investing in bonds is covered in Appendix 11B. Appendix 11C is about interest rate swaps and hedging. The ethics vignette considers companies that structure their leasing contracts in a manner that allows them to avoid reporting their lease obligations as debts on their balance sheets. The Internet research exercise examines current financial reports for Macy’s Inc. It looks at the Bank Credit Agreements and how Macy’s Inc. is managing its debt covenants. The following key points are emphasized in Chapter 11: 1.

Long-term notes payable, bonds payable, and leasehold obligations, and how companies use these instruments as important sources of financing.

2.

Economic consequences created by borrowing.

3.

Different forms of contractual obligations.

4.

The effective interest rate and how it is determined for contractual obligations.

5.

The effective interest method.

6.

How changes in market interest rates can lead to misstated balance sheet values for long-term liabilities.

7.

Operating leases, capital leases, and off-balance-sheet financing.


Chapter 11 TEXT/LECTURE OUTLINE Long-term liabilities: notes, bonds, and leases. I.

Long-term liabilities are probable future sacrifices of economic benefits from present obligations of a particular entity to transfer assets or to provide services to other entities in the future as a result of past transactions or events. The obligation will require the disbursement of assets outside the time frame of current assets.

II.

Economic consequences of reporting long-term liabilities. A.

Credit ratings.

B.

Debt covenants. 1.

Debt covenants can limit a company's operating, investing, and financing activities.

2.

If a company violates a debt covenant, the borrower can require one of the following: a) The immediate repayment of the debt. b) Renegotiation of the debt at terms that are more costly to the borrower (i.e., higher interest rate or additional collateral).

C. Effects of "merger mania." III.

Basic definitions and different contractual forms. A.

Notes, bonds, and leases are supported by contracts. 1.

The timing and magnitude of future cash outflows are specified in the contracts.

2.

Types of contractual forms. a) Interest-bearing obligations require the following: (1)

Periodic cash payments (interest) calculated as a stated percentage of the obligation's face value (principal or maturity value). The stated percentage is given in the debt contract, and the stated percentage is called the stated interest rate.

(2)

Payment of the face value at the end of the contract period.

b) Non-interest-bearing obligations do not require periodic interest payments. Instead, they require only the payment of the face value at the end of the contract period.


Chapter 11

c) Installment obligations require periodic payments throughout the obligation's life, and the periodic payments include both principal and interest. 3.

The contracts may specify the following: a) Collateral. b) Restrictions (i.e., debt covenants).

B.

Types of long-term liabilities. 1.

Long-term notes payable.

2.

Bonds payable. a) Bonds are issued to raise large amounts of capital (usually from multiple creditors) to finance long-term projects. b) Secured bonds versus debentures. c) Interest is usually paid semiannually on bonds.

3. IV.

Leases.

Effective interest rate. A.

The effective interest rate is the interest rate that a company actually incurs on an obligation.

B.

The effective interest rate is the rate that equates the undiscounted future cash flows of an obligation (e.g., periodic interest payments and the face value for interest-bearing obligations) with the present value of the obligation. That is, the effective interest rate is the rate used to discount the future cash flows of an obligation so that the present value of the future cash flows equals the fair market value of that which is received in the exchange.

C. Relation of stated and effective interest rate for notes and bonds. 1.

Stated rate equals effective rate. The present value of future cash flows equals the face value of the debt; the note or bond is issued at face value.

2.

Effective rate exceeds stated rate. The present value of future cash flows is less than the face value, thereby resulting in a discount; the note or bond is issued for less than its face value. Discounts represent future interest expense.


Chapter 11 3.

V.

Stated rate exceeds effective rate. The present value of future cash flows is greater than the face value, thereby resulting in a premium; the note or bond is issued for more than its face value.

Accounting for long-term obligations: the effective interest method. A.

The effective interest method results in a constant percentage of the carrying value being charged to interest expense over the life of the debt and is the preferred amortization method.

B.

Relationships under the effective interest method. 1.

Book value (or carrying value) of the debt equals the debt's face value plus associated premium balance or it equals the debt's face value less associated discount balance.

2.

Interest expense equals debt's book value at the start of the period times the effective interest rate.

3.

Periodic interest payments equal debt's face value times the stated interest rate.

4.

The amount of the discount amortized equals interest expense less the interest payment.

5.

The amount of the premium amortized equals the interest payment less interest expense.

C. Journal entries need to be prepared to record interest expense incurred and discount/premium amortized on every interest payment date and at the end of the accounting period (i.e., an adjusting journal entry) if an interest payment date does not fall on the last day of the accounting period. VI.

Accounting for long-term notes and bonds payable. A.

Bond terminology. 1. Life: the time period from the date of issuance to the maturity date. 2. Maturity date: the end of a bond’s life, when the maturity value (which is usually equal to the principal value, par value, and face value) is paid to the bondholder 3. Interest payment:: the amount paid to the bondholders on semiannual payment (coupon) dates 4. Proceeds: the amount collected when the bonds are issued. 5. Covenants: restrictions imposed on management by bondholders. 6. Unsecured bonds: bonds with no assets backing them


Chapter 11

7. Debentures: bonds with no assets backing them 8. Call provision: a right granted to the issuing corporation to repurchase the outstanding bonds on or after a designated date for a specified price B.

The price of a bond: See Appendix 11A

C. The effective rate and the stated rate: If Effective rate is Equal to More than Less than

Stated rate Stated rate Stated rate

Then price is Par Par less discount Par plus premium t

If Stated rate is Equal to More than Less than

Effective rate Effective rate Effective rate

Then price is Par Par plus premium Par less discount

D. Accounting for bonds payable – the effective interest method 1. Issued at par 2. Issued at discount. 3. Issued at premium. E.

The effective interest method and changing interest rates.

F.

Bond redemptions. 1.

At maturity.

2.

Before maturity. a)

Call provisions grant the issuing company the right to repurchase bonds prior to the maturity date.

b)

Calling bonds normally gives rise to gains or losses. If the gain or loss is material, it should be reported on the income statement as an extraordinary item.

VII. Financial instruments, fair market values, and off-balance-sheet risks. Disclosure of market values required for certain financial instruments, whether or not they are recognized in the financial statements. A.

Short-term investments in equity securities (Chapter 8).

B.

Notes receivable and investments in debt securities.


Chapter 11

C. For long-term debts, the market value approximates the present value of future cash outflows associated with the debt, discounted at the current market rate of interest for similar obligations. D. Other financial instruments not listed on the balance sheet, such as guarantees of the credit of third parties, commitments to provide financing, and derivatives. VIII. Leases. A.

A contract granting use or occupation of property during a specified period of time in exchange for rent payments. Leases allow companies to use or occupy property without many of the associated costs and risks of ownership.

B.

Types of leases. 1.

Operating leases. a) The lessor (i.e., the owner) temporarily transfers the right to use the property to the lessee for a specified period of time in exchange for periodic payments. The lessor is normally responsible for the normal maintenance of the property over the life of the lease. The right to use the property reverts to the lessor at the end of the lease. b) The property is reported as an asset on the lessor's books. The periodic rental payments are reported on the lessor's books as Rent Revenue; the periodic rental payments are reported on the lessee's books as Rent Expense.

2.

Capital leases. a) Leases that, in substance, transfer the rights and risks of ownership to the lessee.

b) Criteria for a lease to be classified as a capital lease—if a lease meets one of the following criteria, the property is considered purchased rather than leased. (1)

The leased property's fair market value approximately equals the present value of the lease payments.

(2)

The term of the lease is 75 percent or more of the leased property's useful life.

(3)

The lessee has the right to purchase the property from the lessor for less than what the property is actually worth, called a bargain purchase price.


Chapter 11

(4)

Ownership is transferred to the lessee by the end of the lease term.

c) Accounting for capital leases on the lessee's books. (1)

At the inception of the lease agreement, the lessee records an asset and a liability equal to the present value of the future lease payments.

(2)

The asset should be depreciated, if appropriate, over its useful life.

(3)

The periodic rental payments serve to reduce the lease liability and to recognize interest expense. Interest expense is computed using the effective interest method.

IX.

International perspective: The importance of debt financing in other countries

X.

ROE exercise: managing long-term debt

XI.

Appendix 11A: The determination of bond prices. A.

Risk-free return—the annual return that an individual would earn by investing in a riskless security such as a treasury note. Factors increasing the risk-free rate tend to decrease bond prices, while factors decreasing the risk-free rate tend to increase bond prices.

B.

Risk premium—the interest rate over and above the risk-free rate to cover the issuing company's default risk. Factors increasing a company's riskiness tend to decrease bond prices, while factors decreasing a company's riskiness tend to increase bond prices.

XII. Appendix 11B: Investing in bonds. XIII. Appendix 11C: Interest rate swaps and hedging XIV. Review problem XV. Ethics in the real world. XVI. Internet research exercise.

LECTURE TIPS 1.

The concept of effective interest rates is sometimes troublesome, especially in cases where these rates are imputed for notes (i.e., when no market exists to establish the effective interest rate). Further, many students do not understand how the effective and stated interest rates could be different. It should be stressed that


Chapter 11 the stated interest rate is simply a device to structure the future cash flows; it does not have to approximate the rate that a company should actually be paying. In addition, discounts or premiums are often viewed as something inherently positive or negative. An approach that may help address this problem is to offer students the opportunity to "invest" in several different notes or bonds—all with the same effective interest rates, the same present values and maturity dates, but with different face values and stated rates. This should drive home the point that discounts and premiums arise because of the way the future cash flows are structured. Many students find time lines showing the future cash flows and the change in the debt's carrying value over the life of the debt to be useful in understanding the relation between discounts/premiums and interest expense. Figures 11–7 and 11–10, end-of-chapter exercises 11–3 through 11–8, and problem 11–5 are all useful for demonstrating the points involved. 2.

Calculating the present value of the future cash flows, particularly for bonds, is sometimes troublesome. The most common mistake is not adjusting the annual effective and stated interest rates to semi-annual interest rates. It should be stressed that present value techniques rely on the compounding period. Since bonds usually pay interest every six months, the appropriate compounding period is six months. End-of-chapter exercise 11–2 emphasizes the concept of compounding.

3.

End-of-chapter problems 11–13 to 11-15 and issues for discussion 11–3 and 11–7 are useful for considering the financial statement effects of accounting for leases. The economic differences between capital and operating leases should be discussed as the basis for accounting for each. The lease agreement's economic substance should be stressed over its legal form.

OUTSIDE ASSIGNMENT OPPORTUNITIES Group study of long-term debt and off-balance-sheet risks both across time and within and across industries (continuing assignment for Chapters 6–14) 1.

Using the most recent annual report of a major public company in one of the four general industry groupings, identify or compute the items listed below for the two most recent years. Compare the items across time. Relate your findings to the economic characteristics and current conditions in the industry and the company’s strategy for competing. Prepare a written summary of your findings. Report findings in a class discussion session in which comparisons will be made both across time and within and across industries. Captions and amounts for components of long-term liabilities: long-term debt, including capitalized lease obligations, employee benefits, deferred income taxes, and others Footnote disclosures for long-term liabilities Total long-term liabilities as a percentage of total assets Total long-term liabilities as a percentage of total liabilities Total long-term liabilities as a percentage of shareholders’ equity Nature of and aggregate commitments for operating leases


Chapter 11 Disclosures of off-balance-sheet commitments and their associated risks, such as commitments to extend credit, guarantees of debt, interest rate agreements, and foreign exchange contracts Recalculation of ratios based on constructive capitalization of operating leases 2.

Obtain the most recent annual report for a company in an industry likely to have significant operating leases (such as JCPenney) in the retail field. Calculate key financial ratios, especially those for return on assets and debt/equity. Obtain and read the Imhoff article (referenced below1) on constructive capitalization of operating leases. Use the method discussed in the article to constructively capitalize the selected company’s operating lease commitments. Recalculate key financial statement ratios. Evaluate the results.

ANSWERS TO IN-TEXT DISCUSSION QUESTIONS 448.

Cash increased and long-term debt increased by the amount of proceeds from new debt issued in excess of repayments during 2010, 2011 and 2012.

448.

Profile 1 would be the Bank of New York Mellon. Banks balance sheets consist mostly of financial assets (loans and investments). Banks have little in the way of fixed assets relative to total assets. Liabilities of banks are mostly current rather than long-term, represented by deposit liabilities to customers. Profile 2 would be 3M. Manufacturing companies such as 3M have extensive infrastructure, which in turn would support a fair amount of long-term debt in the capital structure. Also, companies with a large base of fixed assets usually have associated deferred tax liabilities. Current liabilities would reflect accounts payable, accruals and taxes, as well as the current portion of long-term liabilities. Profile 3 would be Google. Internet companies do not have much in the way of fixed assets, and correspondingly rely primarily on equity financing, rather than debt, to capitalize the business. Current liabilities would reflect payables and accruals for goods and services.

449.

The downgrade by S&P has no direct impact on the financial statements of Sun Microsystems. The downgrade may have an effect on Sun’s ability to raise funds by way of issuing debt in the future. The risks of holding debt in this company have increased so the risk premium that debt investors require is likely to increase and Sun’s financing (interest) costs for future debt issuances are likely to rise. S&P would have reached their conclusions by analyzing Sun’s financial statements using the ratios discussed in Chapter 5 to assess earning power and solvency. The downgrade may have a significant impact on the market value of the bonds. By lowering Sun’s credit rating, S&P is giving the marketplace an indication that the risks of holding Sun’s debt instruments have increased. This would cause the re-sale market value of Sun’s bonds and notes payable to decline.

Eugene A. Imhoff Jr., Robert C. Lipe, and David W. Wright, “Operating Leases: Impact of Constructive Capitalization,” Accounting Horizons, March 1991, pp. 51–63. 1


Chapter 11

450.

Financial instruments are defined as cash, an ownership interest, or a contractual right to receive or obligation to deliver cash or another financial instrument. Financial instruments include routine things such as cash, investments, receivables, and payables, as well as more exotic instruments such as derivative financial instruments employed to manage currency and interest rate risks. The terms of financial instruments can have a significant potential impact on the future operations, cash flows, and financial position of a company, which is not captured in the dollar amounts in the financial statements. Accordingly, accounting standards require additional disclosures to help the user evaluate the associated risks and assess their possible impact on the financial statements.

451.

Home Depot's leases require monthly or annual payments, probably in equal amounts but may have percentage of sales provisions requiring additional payments to be made. Many leases also require payments for property taxes, insurance and other executory costs. Nike's bond is a written legal obligation to pay a certain amount (or amounts) at a certain time (or times). For most bonds there are periodic interest payments for a term of years, with a final payment of principal upon maturity. The Foothill/Eastern Transportation Corridor Agency zero coupon bond is a debt security issued by an entity to investors in exchange for cash. The entity that issues a zero coupon bonds and receives the cash normally agrees to make a single large principal payment whereupon the obligation is terminated.

451.

A lease is a contract granting use of property for a specified period of time in exchange for payment(s). A note is a written legal obligation to pay a certain amount (or amounts) at a certain time (or times). A bond is a debt security issued by an entity to investors in exchange for cash. The entity that issues the bonds and receives the cash normally agrees to make cash interest payments to the bondholders until a specific future date called maturity whereupon a large principal payment is made and the obligation is terminated.

452.

The interest rates disclosed by Hewett Packard appear to be effective rates. Leases usually don’t have stated rates, only effective rates. It is impossible to tell from this disclosure if these debts are interest-bearing or not.

454.

The interest expense reported during each period is computed by multiplying the effective interest rate by the balance sheet value of the obligation as of the beginning of the period. For Sherwin Williams 2008 interest expense would be 2.87 percent of $1.632 million, or $46.84 million.

456.

The present value of the future cash payments associated with these debts would be equal to the $51.7 billion liability net of the $228 million unamortized discount, or $51,472 million. The discount is the excess of the face value of the bonds over the price received upon issuance. The discount indicates that the


Chapter 11 stated interest rate on the bonds was lower than the effective rate at the time the bonds were issued. The bond discount is amortized over the term of the bonds, and is reduced annually. The unamortized discount would be equal to the original discount (determined at the time of issuance) less the cumulative amounts of amortization recognized annually during prior years. The discount is amortized using the effective interest method. By using this method of amortization, the net amount of the liability for the bonds (ie. the bond principal less the unamortized discount) will always equal the present value of the future cash payments associated with these debts. 458.

CBS would want an option to redeem long-term debt, especially debt such as the debentures mentioned which are scheduled for repayment over 20 years from now, to protect itself in the event interest rates and other economic conditions change. For instance, if interest rates drop, CBS could lower its interest cost by redeeming the debentures and replacing them with bonds bearing a lower interest rate.

458.

Debentures are unsecured bonds. Bonds are issued at a premium when the market or effective rate of interest at the time of issuing the bonds is less than the stated or coupon rate specified in the bond contract.

459.

If the effective rate of the Exxon Mobil debentures was 4.9 percent and the debentures were issued at a premium, it means that the stated rate was higher than 5 percent. The stated rate in the bond contract specifies the fixed amount of interest to be paid. If the effective rate is lower, it means the market demands less of a return than what is being offered in interest payment, and therefore would price the bond at a higher amount, based on the present value of the future interest and principal payments using the effective or market rate.

463.

Proceeds received on January 1 from the bonds were as follows: PV of semi-annual interest (PV of annuity: n=20, i=2.75%) plus PV of face value (PV lump sum: n=20, i=2.75%) (.0525 x $50,000,000)/2 x 15.24596 plus $50,000,000 x .58350 (present value factors for fractional percentages were interpolated from the tables in Appendix B and are inexact) $20,010,323 plus $29,175,000 $49,185,323 Interest for the first year, would be as follows: First six months:


Chapter 11

Original proceeds of $49,185,323 x [effective rate of interest (5.5%)/2] =

$1,352,596

Second six months: Present value at the end of the first six months x [effective rate of interest (5.5%)/2] ($49,185,323 + 1,352,596 – 1,312,500) x .02.75 TOTAL INTEREST EXPENSE FOR YEAR 1

$1,353,699 $2,706,295

463.

Washington Post Company’s long term debt has a higher fair value because the fixed interest payments due on their debt exceeded the market interest rates.

464.

Johnson and Johnson's long term debt has a higher fair value because the fixed interest payments due on their debt exceeded the market interest rates. An investor is interesting in both the book value and fair value of long-term debt so they can understand the market holdings of J&J of long-term debt. In this case, the higher value indicates that a better than market interest payment is being received and the company could sell for more than the book value if they would choose to do so.

465.

When interest rates go down, bond prices go up. This is because the fixed interest payments promised in the bond contracts become more valuable when market interest rates decline. Companies with outstanding long-term debt would hope to repay debt with higher interest rates before maturity and get new debt with lower interest rates and interest payments going forward.

466.

Operating leases are treated as pure leasing or rental arrangements. The property would be on the books of the lessor. JCPenney, as lessee, would not recognize any asset or liability on its books, but simply record rent expense on the income statement as the rent is paid or accrued.

468.

Assets under capital leases of $335 million would be shown as an asset on the balance sheet, and the capital lease obligation of $330 million would be shown on the balance sheet as a liability. The asset value represents the present value of the total of the future lease payments as of the inception of the lease, less accumulated depreciation recognized through year-end fiscal 2012. The liability represents the present value of the remaining payments under the lease as of fiscal year-end 2012, computed using the effective interest rate determined as of the inception of the lease.

470.

Wal-Mart appears to be practicing off-balance-sheet financing more aggressively than SUPERVALUE. Although both companies have higher minimum lease payments under their operating leases than they do for their capital leases, the proportion of total minimum lease payments under operating leases is higher for Wal-Mart than for SUPERVALUE. Capital leases are reported on the balance sheet as both a property right and a liability. Operating leases do not appear as


Chapter 11 liabilities on the balance sheet. Therefore operating leases are considered a means of off-balance-sheet financing. To make the financial ratios of the two companies more comparable, an analyst could make adjustments to the balance sheets of the two companies to assume that all the leases were capitalized and then use the adjusted balance sheets in the calculation of the relevant financial ratios. 470.

High debt levels can have an adverse effect the liquidity, cash flow, and earning power of a company. Interest expense is a drain on operating income and cash flow. It also has a negative effect on liquidity. The debt to equity ratio would improve if Prada was successful in implementing the plan to reduce debt levels and increase equity. Cash flows and operating profits would improve because of the reduction in interest costs.

CHARACTERISTICS OF END-OF-CHAPTER ASSIGNMENTS Item

Difficulty

Brief exercises: BE11–1 E BE11–2 E BE11–3 M Exercises: E11–1 E11–2 E11–3 E11–4 E11–5 E11–6 E11–7 E11–8 E11–9 E11–10 E11–11

M M E M M M M M M M M

E11–12 E11–13 E11–14 E11–15 E11–16 E11–17 E11–18 E11–19 E11–20 E11–21 E11–22 E11–23 E11–24 E11–25 E11–26

M M M H M M M M H H M H H H M

Description

Inferring debt transactions Zero Coupon Bond issuance Operating and capital leases

Disclosing debt and debt covenants Annual or semi-annual interest payments? Stated Rate vs. Effective Rate, and issuance price of a liability Computing the proceeds from various notes Notes issued at a discount and interest expense Accounting for notes payable with various stated interest rates Determining the effective interest rate Financing asset purchases with notes payable Inferring an effective interest rate from the financial statements Bond discounts Computing bond issuance proceeds, carrying value, interest expense Accounting for bonds issued at face value Accounting for bonds issued at a discount Accounting for bonds issued at a premium Changing market interest rates/economic gains and losses Redeeming bonds not originally issued at par Refinancing debt Refinancing debt Updating amortization and retiring a bond issuance Analyzing bond disclosures and market values Analyzing bond disclosures and market values Accounting for leases Accounting for leases and the financial statements Financing asset purchases Inferring the effective rate of interest The decision to purchase a bond


Chapter 11 E11–27

H

Managing interest rate risk

Problems: P11–1 P11–2 P11–3 P11–4 P11–5 P11–6 P11–7 P11–8 P11–9 P11–10 P11–11 P11–12 P11–13 P11–14 P11–15 P11–16 P11–17 P11–18

M M M M M M M M M M M H H H M M M M

Computing the face value of a note payable Bonds with an effective rate greater than the stated rate Value of debt and the long-term debt/equity ratio Accounting for notes issued at a discount and at face value The effects of notes payable on the financial statements Cash interest payments vs. interest expense The effective interest method The effective interest method: effects on the statements The effective interest method vs. the straight-line method Redemption and updating amortization Call provisions and bond market prices Tax deductible interest and the present value of cash outflows Capital and operating leases Accounting for a capital lease Some economic effects of lease accounting Financing asset purchases with notes (effective rate) Determinants of bond market prices Bond investments

Issues for discussion: ID11–1 E ID11–2 M ID11–3 H ID11–4 E ID11–5 M ID11–6 M ID11–7 M ID11–8 M ID11–9 M ID11–10 M ID11–11 M ID11–12 H ID11–13 M

Repurchasing outstanding debt Bonds with a stated interest rate of zero Buy or lease: financial statement effects Long term debt disclosures Bond prices and market rates Holding debt in times of recession Adjusting ratios for lease accounting Sub-prime mortgage problems Long-term debt and covenants Lease accounting revolution Risk premiums and bond prices in a recession Interest rate swaps The annual report of Google


CHAPTER 12

Shareholders' Equity

SYNOPSIS In this chapter, the author discusses the theoretical issues and accounting treatment for both contributed capital and earned capital. Equity is compared and contrasted with debt, and the economic incentives for raising capital through debt versus equity are discussed. The primary contributed capital topics discussed are (1) issuing stock; (2) the rationale for a company repurchasing its own common stock; (3) the cost method of accounting for treasury stock; and (4) stock options. The primary earned capital topics discussed are (1) dividend strategies, (2) accounting for dividends, and (3) stock splits. The ethics vignette considers whether boards of directors acted ethically if it attempted to delay an inevitable stock price collapse by issuing a stock dividend. The Internet research exercise examines the main transactions between Kellogg Company and its shareholders over a three-year period. The following key points are emphasized in Chapter 12: 1.

The three forms of financing and their relative importance to major U.S. corporations.

2.

Distinctions between debt and equity.

3.

Economic consequences associated with the methods used to account for shareholders' equity.

4.

Rights associated with preferred and common stock and the methods used to account for stock issuances.

5.

Distinctions among the market value, book value, and par (stated) value of a share of common stock.

6.

What treasury stock is, why it is purchased, and how it is accounted for.

7.

Cash dividends and dividend strategies followed by corporations.

8.

Stock dividends and stock splits.


Chapter 12

TEXT/LECTURE OUTLINE Stockholders' equity. I. Nature of shareholders' equity. A.

B.

II.

Stockholders have a residual interest in the company. consists primarily of:

Stockholders' equity

1.

Contributed capital—contributions from the company's owners.

2.

Earned capital—measure of the net assets the company generated through its operations not yet disbursed to the owners as dividends.

The most common forms of business entities are sole proprietorships, partnerships, and corporations. The primary differences among these three business forms are: 1.

Liability of the owners.

2.

Income taxes.

3.

Returns to owners.

Debt and equity distinguished. A.

B.

Characteristics of debt. 1.

Formal legal contract.

2.

Fixed maturity date (as specified in the contract).

3.

Fixed periodic interest payments (as specified in the contract).

4.

Security (i.e., collateral) in case of default (as specified in the contract).

5.

Debt holders have no direct voice in management, but they can influence management through debt covenants.

6.

Interest is an expense.

Characteristics of equity. 1.

No formal, legal contract.

2.

No fixed maturity date.


Chapter 12

3.

Dividend payments are at the discretion of the company's board of directors.

4.

Residual asset interest.

5.

Stockholders have a voice in management through their right to vote for the board of directors.

6.

Dividends are not an expense.

C. Why is it important to distinguish debt from equity? 1.

Debt vs. equity: the capital provider's perspective.

2.

Equity: higher risk.

3.

Equity: higher returns.

D. Debt vs. equity: Management's perspective.

E.

1.

Debt: contractual restrictions.

2.

Debt: less expensive.

3.

Equity: dilution of ownership.

Debt vs. equity: The accountant's and auditor's perspective.

III. The economic consequences associated with accounting for shareholders' equity. A.

Effects of financial ratios.

B.

Managing the debt to equity ratio.

C. Strengthening the balance sheet. D. Restrictions on dividend payments. E.

Interests of creditors.

IV. Accounting for shareholders' equity. A.

Contributed capital. 1.

Authorized, issued, and outstanding shares. a) Authorized shares represent the number of shares of stock that the company is legally entitled to issue as stated in the corporate charter. b) Issued shares represent the total number of shares that have been distributed by the company and not retired. Issued shares equal the shares outstanding plus shares held in treasury.


Chapter 12

c). Outstanding shares represent the shares currently held by shareholders. 2.

Market value, book value, par value, and stated value. a) Market value is the value at which the stock is currently trading on the open market. b) Book value represents shareholders' equity per share of common stock outstanding. Book value is computed as: (Stockholders' equity - Preferred capital) ÷ Number of common shares outstanding. c) Market-to-book ratio. i.

The market-to-book ratio is computed by dividing the market value of a company's common stock by its book value.

ii.

Ratios equal to 1 indicate that a company's net book value (as measured by the balance sheet) is perceived by the market to be a fair reflection of the company's true value. Ratios larger than 1 indicate that the balance sheet is perceived to be conservative.

d) Par (or stated) value.

B.

i.

Par value is a legal concept and was created to protect creditors. It no longer has significant economic or legal meaning.

ii.

Owners' contributions in excess of par value are allocated to additional paid-in capital.

Preferred stock. 1.

Preferred as to dividends.

2.

Preferred as to assets.

3.

Preferred stock is usually issued with a par value. Dividends on preferred stock are either stated as a dollar amount or as a percentage of par value.

4.

Preferred shareholders are entitled to certain preferences (such as on dividends and assets) over common shareholders. In exchange for these preferences, preferred shareholders usually sacrifice certain rights, such as the right to vote for board members.

5.

Characteristics of preferred stock. a) Cumulative versus noncumulative. b) Participating versus nonparticipating.

6.

Preferred stocks: debt or equity?


Chapter 12

C. Earned capital. 1.

Retained earnings

2.

Accumulated comprehensive income

D. Common stock. 1.

Fundamental rights of common shareholders. a) Right to receive dividends declared by the board of directors. b) Right to share proportionately in residual corporate assets in the event of liquidation. c) Right to exert control over management.

V.

2.

Common stock is the riskiest ownership interest.

3.

Market value.

4.

Book value.

5.

Market-to-book ratio.

6.

Par value

Accounting for common and preferred stock issuances. A.

B.

Recording stock issuances 1.

No-par value stock.

2.

Par value stock.

Repurchasing stock—treasury stock. 1.

Treasury stock is not considered an asset.

2.

Reasons companies repurchase common stock. a)

To provide a sufficient number of shares to support employee compensation plans.

b)

To concentrate ownership of outstanding shares to make it more difficult for a takeover.

c)

To increase the market price of a company's outstanding stock.

d)

To increase the company's earnings per share.


Chapter 12

e)

3.

To retire preferred or common stock. Once common stock has been retired, the shares are considered authorized but not issued.

Accounting for treasury stock. a)

Cost method. (1)

The cost method is the more common method used to account for treasury stock. Under GAAP, using the cost method is appropriate if the company intends to eventually reissue the shares.

(2)

The treasury stock account has a debit balance and is disclosed in the shareholders' equity section after retained earnings.

(3)

When purchasing treasury stock, Treasury Stock is debited and Cash is credited for the cost of the shares.

(4)

Reissuing treasury stock for more than acquisition cost. (a) Treasury Stock is credited for the cost of the shares reissued. (b) The excess received over the acquisition cost is credited to Additional Paid-in-Capital, Treasury Stock.

(5)

Reissuing treasury stock for less than acquisition cost. (a) Treasury Stock is credited for the cost of the shares reissued. (b) The excess of acquisition cost over proceeds is debited to Additional Paid-in-Capital, Treasury Stock. If its account balance is insufficient to absorb the entire excess, the residual excess is debited to Retained Earnings.

b)

Par value method - acceptable under GAAP but not widely used.

C. Stock options. 1.

Stock options give the right to purchase equity securities at a fixed price over a specified time period. Stock options are widely used as a form of executive compensation.

2.

No compensation expense is recorded when granted (except when options are granted at prices below the current market price). An issuance of common stock is recorded when options are exercised.

3.

Disclosure is required of the amount of compensation expense that would have been recorded if the value of the options granted during the year was recognized.


Chapter 12

VI. Retained earnings. A.

Dividends. 1.

Dividend strategies.

2.

Important dates. a)

Date of declaration—the date that the board of directors declares a dividend and the company incurs a liability for the dividend.

b)

Date of record—the legal owner of the stock on this date is the person or entity entitled to the dividend.

c)

Date of payment—the date that the company pays the dividend.

3.

The dividend account is a temporary account that is closed directly into retained earnings during the closing process.

4.

Cash dividends.

5.

Stock dividends and stock splits. a)

Stock splits. (1)

A company issues additional shares of common stock to its shareholders and splits the outstanding shares into smaller units.

(2)

Stock splits cause the number of shares classified as outstanding to change and the par value per share and market value per share to change. These changes are documented in a memorandum entry. However, the total value of the shares outstanding should, theoretically, remain the same.

b). Stock dividends. (1)

A company issues additional shares of common stock to its shareholders as a dividend. No assets are distributed to the shareholders.

(2)

After the stock dividend, each shareholder retains the same percentage of ownership interest in the corporation.

(3)

Ordinary stock dividends. (a) A stock dividend should be accounted for as an ordinary stock dividend if the number of additional shares to be issued is less than 25 percent of the shares outstanding prior to the stock dividend.


Chapter 12 (b) An amount equal to the fair market value of the common stock issued is allocated from retained earnings to the common stock and additional paid-in-capital accounts. (4)

Stocks splits in the form of a stock dividend. (a) A stock dividend should be accounted for as an ordinary stock dividend if the number of additional shares to be issued is greater than 25 percent of the shares outstanding prior to the stock dividend. (b) An amount equal to the par value of the stock is allocated from retained earnings to the common stock account.

c)

B.

Reasons companies declare stock dividends and stock splits. (1)

To reduce the market price per share.

(2)

Publicity gesture.

(3)

To capitalize a portion of retained earnings.

Appropriations of retained earnings. 1.

A book entry partitioning retained earnings into restricted and unrestricted retained earnings.

2.

Appropriated retained earnings arise from either the board of directors' discretion or the terms of debt covenants.

C. Negative retained earnings. 1.

The retained earnings account is negative if a company's accumulated net losses plus dividends from previous years exceeds the accumulation of its past profits.

2.

Negative retained earnings (deficits) generally indicate serious problems because they indicate losses. In today's changing environment there are notable exceptions of start-up, high-tech, and Internet companies with accumulated deficits, yet the companies enjoy favorable stock prices, reflecting the market's confidence in their future prospects.

D. Retained earnings and prior period adjustments. 1.

Few items are booked directly to retained earnings. These include net income (loss), dividends, appropriations, and certain treasury stock transactions.

2.

Additionally, correction in the current period of a prior period’s accounting error is made by adjusting the misstated asset and/or liability, with a corresponding and offsetting adjustment to retained earnings. The entry to retained earnings is called a prior period adjustment.


Chapter 12

VII. Statement of shareholders' equity. VIII. International perspective: the rise of international equity markets. IX. ROE exercise: return on equity and value creation. X.

ROE analysis

XI. Review problem XII. Ethics in the real world. XIII. Internet research exercise.

LECTURE TIPS 1.

A conceptual understanding of why companies would repurchase their own stock and why treasury stock reduces shareholders' equity (as opposed to being an asset) needs to be developed. End-of-chapter issues for discussion 12–3, 12-4, and 12–12 and 12-15 are helpful in developing this understanding. Accounting for treasury stock can be demonstrated with examples such as provided by end-of-chapter exercises 12–4 through 12–8 and problems 12–2 and 12–13.

2.

Students usually need extra help in understanding why a company would declare either a stock split or a stock dividend (and the economic difference between the two) as a basis for understanding how to account for each. It should be stressed that the differences in accounting for stock dividends and stock splits arise from the legal differences between the two. End-of-chapter exercises 12–14 and 12–15, problems 12-4, 12-7 and 12–9, and issue for discussion 12–1, and the ethics vignette at the end of the chapter are useful for this demonstration.

OUTSIDE ASSIGNMENT OPPORTUNITIES Group study of shareholders' equity both across time and within and across industries (continuing assignment for Chapters 6–14) 1.

Using the most recent annual report of a major public company in one of the four general industry groupings, identify or compute the items listed below for the two most recent years. Compare the items across time. Relate your findings to the economic characteristics and current conditions in the industry and the company’s strategy for competing. Prepare a written summary of your findings. Report findings in a class discussion in which comparisons will be made both across time, and within and across industries. Total liabilities as a percentage of total assets Contributed capital as a percentage of total assets Retained earnings as a percentage of total assets Treasury stock as a percentage of contributed capital Book value per share of common stock


Chapter 12 Market-to-book ratio Nature, classification, and amount of any hybrid securities (e.g., redeemable preferred stock) Type(s) and amount of dividends: cash dividends, stock dividends or splits

Article search: treasury stock transactions 2.

Search the financial press (The Wall Street Journal, Barrons, Fortune, Forbes, etc.) for an article reporting on recent notable treasury stock activity by a particular company or a particular group of companies. Summarize the nature and amount of the activity, reasons why the shares were repurchased, and any observed consequences of the buybacks.

Article search: application of accounting standards for stock options 3.

Search the financial press (The Wall Street Journal, Barrons, Fortune, Forbes, etc.) for an article reporting on how companies have responded to the revised accounting standard issued by the FASB in 2004 for executive stock options. Especially note if any companies plan to change the extent to which they use stock options as a form of compensation.

ANSWERS TO IN-TEXT DISCUSSION QUESTIONS 499.

The information presented indicates that Amazon.com owed its creditors investors a total of $5.6 billion, of which $4.8 billion was current as of year-end 2008. The current liabilities would include the current amount of long-term liabilities as well as accounts payable and accrued expenses. Contributed capital of $3.4 billion represents the amount that was raised from shareholders. The negative retained earnings of $.73 billion arose from Amazon’s unprofitable operations since inception. In 2009, Amazon made enough income to cover its retained earnings deficit and bring retained earnings to a positive $172 million.

500.

Interest incurred is a financing cost (versus an operating cost) and is deducted as an expense on the income statement. Dividends are not considered a component of net income, but rather represent a distribution of net income to shareholders. Therefore dividends are deducted directly from retained earnings and do appear on the income statement.

501.

From the perspective of an investor or creditor, bonds represent a lower investment risk than stocks, and enjoy a fixed amount of cash receipts (contractual interest and principal payments) determined by the bond instrument, if the instrument is held to maturity. This safety and certainty is in contrast to equity investments which pose a higher investment risk, and uncertain cash receipts in the form of discretionary dividends and stock appreciation.

502.

Internet companies such as Google are inherently risky (especially when they are just starting up) and debt in the capital structure only serves to magnify that risk. With no debt, the company could build a stronger credit rating and gain some future financial flexibility. Debt instruments require contractual future cash payments of interest and principal, whereas equity does not, given that dividend payments are discretionary.


Chapter 12

Equity investors do not expect dividends from internet companies, but rather expect earnings to be retained for future growth. Also, debt instruments often involve restrictive covenants by which Google’s management may not want to be constrained. 503.

The effect on the accounting equation of each form of Honeywell’s payment to capital providers is shown below: ASSETS

=

LIABILITIES

+SHRHLDR’ EQ

Interest:

-$351 million

-$351 million (Exp; RE)

Reduction of debt:

-$200 million

Repurchase stock:

-$317 million

-$317 million (contra SE)

Dividends

-$1.211 billion

-$1.211 billion (-RE)

-$200 million

Interest on debt is considered a cost of doing business and is treated a deduction in arriving at net income. Repayments of outstanding debt are a reduction of a liability. Dividends are a distribution of net income to shareholders and are deducted directly from retained earnings; dividends do not appear as expenses in the income statement. Common stock repurchases are also a form of return to shareholders, but not on a prorata basis as in the case of dividends. If shares are retired, common stock, additional paid in capital, and possibly retained earnings are reduced. If repurchased shares are held in the treasury for possible reissuance, the reduction is in the form of an increase to a contra account that offsets shareholders’ equity. A company does not have an investment in itself; treasury shares are not assets. 504.

If Proctor & Gamble issued equity on the last day of the year, the denominator in the formula for return on equity would increase, and the return percentage itself would decrease. In the case of both share repurchases and dividend payments, the denominator would decrease, and the return on equity percentage would increase. Analysts who especially value return on equity as an important measure of corporate performance must be alert to a company’s ability to “manage” the return on equity. Management discretion over the timing of transactions (share issuances and repurchases, and dividends) which affect the formula used to compute return on equity.

506.

The US government injected $70 billion into Citigroup to prop it up and keep if from failing. This investment was recorded as preferred stock. The TARP funds were returned before the end of 2009. Without getting into the details of the terms of the preferred stock investment, it is probable that there were significant promises made by Citigroup to the US government in connection with this preferred stock. The political ramifications of the $700 billion TARP bailout will effect the political and economic landscape of the US for years.

507.

Preferred shareholders would insist on a policy such as the one described for Sears. Preferred shareholders generally receive specified annual dividends and have rights in liquidation, both of which have priority over claims of common shareholders. In exchange for those preferences, preferred stock is usually non-voting. As a protection to their interests, preferred shareholders would insist on provisions for cumulative


Chapter 12 dividends, with priority over common dividends, and the right to vote if dividends are missed for an extended period of time. 508.

Preferred stocks are hybrid securities which have characteristics of both debt and equity. “Mandatorily redeemable preferred stock” is preferred stock that has a mandatory redemption feature that is outside of the control of the issuing corporation. In this case, the preferred shareholders must be “paid back” as in the case of debt. This feature would lead to the conclusion that the preferred stock has more closely resembles debt, and should be classified as such in the balance sheet. Nonredeemable preferred stock without mandatory redemption more closely resembles equity and will appear as equity on the balance sheet

508.

The company would authorize two different kinds of preferred stock to allow itself flexibility in issuing. One of these issues would behave more like a debt instrument and the other more like an equity instrument. In this case, the preferred shareholders must be “paid back” as in the case of debt. This feature would lead to the conclusion that the preferred stock has more closely resembles debt, and should be classified as such in the balance sheet. Nonredeemable preferred stock without mandatory redemption more closely resembles equity and will appear as equity on the balance sheet

509.

Selling shares to the public is a way to raise money by selling equity in the company. When buying ownership in a company by IPO investors take the risk of buying into a company without an established price in the market place. A successful IPO raises the proper amount of money for the company by not undervaluing the offering and provides some price momentum by not overvaluing the offering.

509.

The $100 paid for the shares in the initial public offering went to Google. Investors who purchased their shares in the IPO and held them until June 11, 2013 had a cumulative (net) unrealized gain of $783.39 as of June 11, 2013, but only those that actually sold their shares had a trading profit. The other shareholders have an unrealized gain, but not a trading profit. The company does not directly benefit from the increase in it's share prices but the indirect benefits are many. The success of the enterprise, as manifested by the increased share price has numerous positive effects on employees, creditors, shareholders and other stakeholders. Those holding stock (and stock options) in the company benefit directly. One of the benefits to the company is that future stock sales will generate market interest, bringing a higher price and making it easier to raise capital when needed.

510.

Par value, or stated value, of a share of common stock is a legal rather than an economic concept, and is often only an arbitrary, nominal value. JP Morgan Chase's par value is only $1.00 per share. Book value per share represents the total stockholders’ equity on the books (minus any preferred capital), divided by the number of common shares outstanding. Market value per share is the price for which a share of stock may be exchanged on the open market. The marketplace takes into account factors such as a company’s future earnings prospects, interest rates, and so, in arriving at a market price. The market price is often larger than book value, reflecting changes in asset values not recognized in the accounts, both for tangible assets, notably land and buildings, and for intangibles such as goodwill.

511.

When Jet Blue issued the shares, cash increased by $182.4 million (15.2 million shares at $12 per share), and shareholders’ equity (common stock and additional paid-in


Chapter 12

capital) increased by a like amount. The amount of the increase that was allocated to common stock was $152,000 ($.01 per share) and the amount allocated to additional paid-in capital was $182,248,000. The subsequent issue of stock also increased cash and shareholders’ equity. 512.

A company would buy back its own stock for a variety of reasons, the most common of which is to support employee compensation plans. By buying back shares for reissuance in compensation plans, a company avoids the dilution that would otherwise occur if new shares were issued. A company may also use stock repurchases as a takeover defence, to deploy excess capital, or to increase its stock price by reducing dilution.

513.

The capital structure leverage computations for 2010 and 2012 are as follows: In billions of $ Total Shareholders' Leverage Assets Equity (Assets/Equity) 2010 102.9 56.6 1.82 2012 121.3 64.8 1.87 Proforma 139.5 83.0 1.68 As can be seen in this example, the decision to purchase treasury stock had the effect of increasing the company's leverage. The relationship between total assets and total equity would have resulted in a lower leverage ratio, had the company kept its cash and left it's equity section alone. The increased leverage helps to improve earnings per share, earnings on assets, and return on equity.

514.

Boston Scientific must have received cash of $90 million from the sale of the treasury shares, which was below the original cost of the shares. The $142 million represents the cost of the treasury shares, and the $52 million decrease in additional paid-in capital represents the difference between the proceeds and the cost. The reissuance price was below the original cost of the treasury stock. This transaction would be reflected on the statement of cash flows as a financing transaction providing cash flows of $90 million. in millions of $ ASSETS CASH T stock repurchase

+$90 million

=

+SHRHLDRS’ EQ PD IN CAP - $52

T-STOCK +$142

Note: The $142 reduction in the treasury stock account is an addition in this analysis because treasury stock is a contra account, offsetting the other equity accounts. 517.

Stock options and other forms of equity-based pay are used by many corporations in their incentive compensation plans for their executives. These items are treated as expenses in the financial statements of the corporations that have such compensation plans. Backdating stock options is a crime that benefits the executives that received the options and is a theft of money from the shareholders.

518.

Abbott Laboratories paid between 44% and 64% of its income as dividends over the 3 year period. This level of pay-out is characteristic of a mature ‘income’ stock paying a significant portion of its income as a dividend.


Chapter 12

519.

The goals of a corporation and its nature will be reflected in its dividend policy. Young growing companies rarely pay dividends. They need to use the profits they generate to fund their continued growth. Shareholders of these corporations expect to be rewarded with increases in the market price of their shares as the business grows and becomes ever more profitable. Such companies are referred to a “growth companies”. Mature companies are much more likely to pay dividends consistently. Shares in dividend paying corporations are known as “income” stocks. Google has the characteristics of a “growth company”. Stock in Intel has the characteristics of an “income stock”. Microsoft appears to be making the transition from a growth company to a mature, dividend paying corporation.

520.

In a stock split, the number of outstanding shares is simply “split” into smaller units, and the corporation distributes additional shares. Concurrent with a split, the price of the stock goes down proportionately. For instance, in a 2 for 1 split, a shareholder would have two shares rather than the original one, but the price per share would be one-half of the pre-split price. Stock splits serve to reduce the per-share price of the outstanding shares so that small investors can more easily afford to purchase them. The suggested 5:1 split of Google would have resulted in 5 shares valued at $114 replacing each share previously valued at $570.

522.

Neither a large stock dividend nor a stock split has any effect on the basic accounting equation. Assets and liabilities are not affected; only the details within the shareholders’ equity section are changed. Stock dividends provide extra shares to shareholders without requiring the payment of cash. The benefit to shareholders is largely psychological rather than economic. A stock split (or a large stock dividend) reduces the per-share price of shares so that small investors can more easily purchase them (especially a round lot of 100 shares). In both stock dividends and stock splits, the shareholder has the same proportionate interest in the corporation after the dividend or split as before.

523.

Yahoo! may be well-known, but that does not mean that they were profitable in their start-up years. The $50 million deficit in retained earnings at the beginning of 2002 consisted of Yahoo’s accumulated losses since inception. For that deficit to have been reduced to $7.5 million by year-end 2002, Yahoo must have reported a profit for 2002 of $42.5 million, followed by a profit for 2003 of $237.5 million.

CHARACTERISTICS OF END-OF-CHAPTER ASSIGNMENTS Item

Difficulty

Brief exercises: BE12–1 E BE12–2 E BE12–3 M Item

Difficulty

Exercises: E12–1

E

Description

Inferring shareholders' equity transactions Stock splits and market values Treasury stock purchases Description

The effects of transactions on shareholders' equity


Chapter 12

E12–2 E12–3 E12–4 E12–5 E12–6 E12–7 E12–8 E12–9 E12–10 E12–11 E12–12 E12–13 E12–14 E12–15 E12–16

E E M M M M M M H H M M H M E

Debt, contributed and earned capital, the classification of preferred stock Authorizing and issuing preferred and common stock The effects of treasury stock purchases on financial ratios Reissuing treasury stock Reissuing treasury stock Treasury stock exceeds contributed capital Book value per share, stock issuances, and treasury stock purchases Inferring equity transactions from the statement of shareholders' equity Inferring equity transactions from the statement of shareholders' equity Inferring equity transactions from the statement of shareholders' equity Issuing cash dividends on outstanding common stock Cumulative preferred stock and dividends in arrears Stock dividends and stock splits Why do companies declare stock dividends? Appropriating retained earnings

Problems: P12–1 P12–2 P12–3 P12–4 P12–5 P12–6 P12–7 P12–8 P12–9 P12–10 P12–11 P12–12 P12–13 P12–14

E E E M M M M M M M M H H M

Hybrid securities and debt covenants The effects of treasury stock transactions on financial ratios The significance of par value Cash and stock dividends Dividend payments and preferred stock The maximum dividend Stock splits and stock dividends Stock issuances Miscellaneous shareholders' equity transactions Inferring transactions from the balance sheet Inferring shareholders' equity transactions from the balance sheet Stockholders' equity over a four-year period Blocking takeovers and treasury stock purchases Bankruptcy and protecting the interests of the creditors

Issues for discussion: ID12–1 H Stock dividends and economic exchanges ID12–2 M Dividend strategy ID12–3 M Economic consequences of treasury stock purchases and cash dividends ID12–4 M Treasury stock purchases ID12–5 M Debt or equity? ID12–6 E Contracts based on net worth ID12–7 M Shareholder returns, dividends, and buy-backs ID12–8 M Market value vs. book value ID12–9 M Common stock issuances ID12–10 M Equity issuance ID12–11 E Stock split and stock prices ID12–12 M Share repurchases during volatile economic periods ID12–13 M Stock option expense ID12–14 H Statement of shareholder’s equity ID12–15 H Different treasury stock strategies ID12–16 H Comprehensive income - US GAAP vs. IFRS ID12–17 M The annual report of Google


CHAPTER 13

The Complete Income Statement

SYNOPSIS In this chapter, the author provides an in-depth discussion of (1) the different types of business transactions and (2) the income statement. The discussion of transactions focuses on financing, investing, and operating transactions, and how these transactions flow through to the balance sheet and income statement. The general income statement topics discussed are (1) the economic consequences associated with income measurement and disclosure; (2) the different ways to measure income, including comprehensive income, and how these address the objectives of financial reporting; and (3) the disclosure rules that should be followed when preparing an income statement. The specific issues discussed are operating revenues and expenses; nonoperating revenues and expenses; intraperiod tax allocations; disposal of a business segment; extraordinary items; cumulative effect of a change in accounting principles; and earnings-per-share disclosures. The ethics vignette considers whether it is ethical for management to consider the impact of the choice between debt and equity financing on its own compensation, where compensation is a function of net earnings after interest expense. The Internet research exercise examines the income statement of Yahoo to identify items that could be considered non-operating. The following key points are emphasized in Chapter 13: 1.

Economic consequences associated with reporting net income.

2.

Two different concepts of income: matching and fair market value.

3.

A framework for financing, investing, and operating transactions.

4.

Categories that constitute a complete income statement and how they provide measures of income that address the objectives of financial reporting.

5.

Intraperiod tax allocation.

6.

Earnings per share disclosure on the income statement.

TEXT/LECTURE OUTLINE The complete income statement. I. Income statement. A.

Purpose—to measure the income generated during the accounting period.

B.

Economic consequences of income measurement and disclosure.

1.

Income serves as a measure of the company's performance during the accounting period.


Chapter 13

2.

Effect of income on stock and bond prices.

3.

Role of income in debt covenants.

4.

Role of income in management incentive compensation.

C. Different measures of income for different objectives. 1.

The objectives of financial reporting are to provide information that is: a)

Useful for investment and credit decisions.

b)

Helpful in assessing future cash flows.

c)

Reflective of changes in a company's resources.

2.

No single measure of income can achieve these broad objectives. Elements of financial statements that are key to understanding the different measures are found in the definitions of ten key concepts: assets, liabilities, equity, investments by owners, distributions to owners, comprehensive income, revenues, expenses, gains, and losses.

3.

Comprehensive income represents a broad definition of income—any change in the company's equity due to nonowner transactions. a) This concept of income includes items not included in the computation of net income, such as foreign currency translation adjustments and unrealized gains and losses on available-for-sale securities (Chapter 8). b) The FASB now requires that comprehensive income and its components be reported and displayed with the same prominence as the other financial statements.

4.

The definitions distinguish revenues and expenses from gains and losses. The nature of individual transactions must be considered to determine if and how they should be reflected in income.

D. Two different concepts of income: matching and fair market value. E.

Financing, investing, and operating transactions: a framework. 1.

Financing and investing transactions principally affect balance sheet accounts. Financing and investing transactions are those transactions associated with the following: a)

Exchanges with shareholders.

b)

Exchanges of liabilities and shareholders' equity.

c)

Issues and payments of debt.

d)

Purchases, sales, and exchanges of assets.

e)

Revenues and expenses.


Chapter 13

2.

Classifying operating transactions: a)

Group A: normal and recurring operating revenues and expenses.

b)

Group B: revenues and expenses from activities not germane to a company's primary activity, or form activities that occur infrequently. i)

Extraordinary items.

ii) Disposals of segments iii) Other revenues and expenses A. Bank interest B. Changes in asset and liability fair market values c)

II.

Group C: Gains and losses due to mandatory changes in accounting principles.

A complete income statement: disclosure and presentation A.

B.

Operating revenues and expenses. 1)

Asset and liability inflows and outflows associated with the acquisition and sale of goods and services.

2)

To be considered operating activities, the activities must be both usual (the economic event is part of the company's normal operations; it is associated with the company's central, ongoing activities) and frequent (the economic event is expected to recur in the foreseeable future).

Other revenues and expenses. 1)

Asset and liability inflows and outflows that are incidental to the company's operating activities.

2)

To be classified as other revenue or expenses (i.e., incidental activities), the activities must be either unusual or infrequent, but not both. Both the company and its environment must be considered in deciding whether an item is unusual or infrequent.

C. Gains (losses) from disposing of a business segment. 1)

A business segment is defined as a separate line of business, product line, or class of service that represents an operation that is independent of the company's other operations.

2)

Components of gains (losses) from disposing of a segment. a)

Income (loss) from operating the business segment.

b)

Gain (loss) on the disposal of the segment's net assets.


Chapter 13

c)

Gains or losses from the disposal of a business segment are reported net of any tax effect.

D. Extraordinary items.

E.

1)

Extraordinary items are material gains or losses that are both unusual and infrequent. Consequently, such events are not classified as operating activities or incidental activities.

2)

Both the company and its environment must be considered in deciding whether an item is both unusual and infrequent.

3)

Extraordinary items are reported net of any tax effect.

Cumulative effect of changes in accounting principles. 1)

Reasons for changing an accounting principle.

2)

The cumulative effect of the change is reported net of any tax effect.

3)

Voluntary versus mandatory changes.

III. Intraperiod tax allocation. IV. Earnings-per-share (EPS) disclosures.

V.

A.

GAAP provides for EPS disclosures for income from continuing operations (after taxes); disposals of business segments; extraordinary items; and the cumulative effect of changes in accounting principles.

B.

Both "basic" and "diluted" EPS are presented. Diluted EPS gives effect to dilution that may result from additional shares that may be issued for such things as stock options.

Income statement categories: useful for decisions but subjective

VI. International perspective; investments and income statement disclosure VII. ROE exercise; using the right earnings number VIII. ROE analysis IX. Review problem X.

Ethics in the real world.

XI. Internet research exercise.


Chapter 13

LECTURE TIPS 1.

Students often have trouble applying intraperiod tax allocation, especially with knowing which items should be disclosed net of taxes and the direction of the tax effect (particularly with losses). Care should be taken in explaining why intraperiod tax allocation is used and which items are disclosed net of taxes. End-of-chapter exercises 13–13 and 13–14 and problems 13–6, 13–8 and 13–9 are helpful in demonstrating the essentials of intraperiod tax allocation.

2.

The distinction should be stressed between operating revenues and expenses, nonoperating revenues and expenses, income from continuing operations, disposal of a business segment, extraordinary items, and the cumulative effect of a change in accounting principle, because of the different relevancy of these income statement components for assessing earning power. These differences are associated with earnings persistence. Deciding whether an item is extraordinary requires a great deal of judgement and is difficult even for seasoned professional accountants. Examples illustrating the difficulty of making this decision should be provided. End-of-chapter exercises 13–5, 13–6, 13–9, 13–10, 13– 11, and 13–16; problems 13–2, 13–4, 13–5, 13–8, and 13–9; and the issues for discussion 1tems 13-1 through13-9 are especially useful.

OUTSIDE ASSIGNMENT OPPORTUNITIES Group study of income statement components both across time and within and across industries (continuing assignment for Chapters 6–14) 1.

Using the most recent annual report of a major public company in one of the four general industry groupings, identify or compute the items listed below for the two most recent years. Compare the items across time. Relate your findings to the economic characteristics and current conditions in the industry and the company’s strategy for competing. Prepare a written summary of your findings. Report findings in a class discussion session in which comparisons will be made both across time and within and across industries. •

Common-size income statement (see Chapter 5)

Significant accounting policies affecting recognition of revenue and expenses

Description and nature of special items: nonoperating revenues and expenses, discontinued operations, extraordinary items, and the cumulative effect of a change in accounting principles

Presentation of comprehensive income

Earnings per share disclosures

Analyzing the quality of earnings 2.

Obtain the annual report for a company with several nonrecurring items (such as BristolMeyers Squibb for 2008 - Figure 13-6). Obtain and read the Comiskey-Mulford-Choi article on analyzing the persistence of earnings referenced below.1 Use the method discussed in the article to evaluate the company's earnings for the given three years in the annual report in terms of persistence and cash flow. Summarize your conclusions in writing, supported with computations.

1

Eugene E. Comiskey, Charles W. Mulford, and Hyun-Dol Choi, “Analyzing the Persistence of Earnings: A Lender’s Guide,” Commercial Lending Review, Winter 1994/1995, pp. 4–23.


Chapter 13

ANSWERS TO IN-TEXT DISCUSSION QUESTIONS 553.

The settlement for BP would be included as a part of income, but not as a part of core activities. Analysts following the oil industry would not consider this a part of BP’s recurring business and so would not consider this an ongoing part of the business when considering BP’s core operating income. Most legal settlements are one-time events. This situation is somewhat unique; however, because of the size of incident. It is likely that other settlements and expenses related to the incident persist. A trust was established by BP to cover these ongoing legal issues. Treatment under GAAP and IFRS for this issue is substantially similar.

556.

The Gap did not recognize a loss on the reissuance of treasury stock. Such transactions with shareholders are purely financing transactions which never result in reported income or loss on the income statement.

556.

The effect on the basic accounting equation for Honeywell was as follows: ASSETS =

Long-term debt issued Debt redeemed

+$102 million -$529 million

LIABILITIES +

+$102 million -$529 million

The above answer neglects the portion of the debt payments being treated as interest because the interest amount was not provided. Interest payments would increase expenses, reducing retained earnings and shareholders’ equity. 557.

Southwest’s investment of $1.3 billion in property and equipment would be an investing activity. The cost would be capitalized as an asset and depreciated over its expected useful life. The effect on current and future income statements would be the annual depreciation charges. Any remaining book value at the time of disposal would enter into the computation of a gain or loss on disposal, which also would be reflected in the income statement.

558.

Cost of revenues belongs under Group A. Asset impairments belong to Group B under other revenues and expenses (changes in asset fair market values). Gains and losses on operating assets belong to Group B under other revenues and expenses (gains and losses recognized on sales of long-lived assets). Restructuring costs belong under Group B under extraordinary items.

560.

A gain on the sale of investments would most likely be considered a usual and frequent transaction at Bank of America, rather than JCPenney. An important component of a bank’s business model is engaging in investment transactions. On the other hand, investment transactions at J.C. Penney would be peripheral to its main business of retail sales.

561.

The gain would appear under the caption “other revenues and expenses” because the sale of the investment was a secondary or auxiliary activity to the normal operations of Limited Brands.

562.

Extraordinary items are those that are both unusual or infrequent. Weather hazards for a food grower are not infrequent, as evidenced by the fact that Dole incurred two sizeable weather-related losses within one year. Such losses are distinct enough from normal operations that they would be disclosed separately, but not as extraordinary items.

664.

The income trend across time before considering the accounting changes was downward. The income trend across time after considering the accounting changes was also downward. The downward trend is the only one evident anywhere in this


Chapter 13

presentation. Avis will not be able to continue to improve its net income annually by changing accounting methods. Accounting method changes generate no cash and should usually be factored out in making comparisons of income across time. 565.

The $1.2 billion change was apparently offset by $.2 billion in tax savings, making the net charge to income only $1 billion.

566.

Shares can be estimated by dividing net income by the earnings per share (EPS) as follows: Net income (millions) EPS-basic EPS-diluted Shares (millions) = Net income/EPS Basic Diluted

2012

2011

2010

$1,960 $1.17 $1.16

$3,709 $2.18 $2.17

$3,102 $1.80 $1.79

1,675 1,690

1,701 1,709

1,723 1,732

Shares for basic EPS represent common shares actually outstanding. Shares for diluted EPS take into account the increase in the number of shares if all potentially dilutive securities were converted to common shares. 567.

The following items would be candidates for "other gains and losses": ▪ ▪

Impairment charge for BMS-986094 intangible asset Other (income)/expense

Net income from operations would be net earnings (as presented in Figure 13-6), adjusted to (1) add back the Impairment charge and (2) subtract out the Other income items for these three years. 567.

‘Real and repetitive’ earnings would be earnings related to core ongoing activities. Real and repetitive items are expected to continue into the future therefore providing some predictive value which is essential to successful third party use of financial statements. The income statement is organized in a way that is meant to isolate ‘one time items’ that may sway earnings in a given period and give the user the possibility of identifying earnings that will continue going forward.

CHARACTERISTICS OF END-OF-CHAPTER ASSIGNMENTS

Item Brief exercises: BE13–1 BE13–2 BE13–3 BE13–4 Exercises: E13–1 E13–2 E13–3 E13–4 E13–5 E13–6

Difficulty

Description

E E M M

Nonrecurring items Effects on the basic accounting equation Interpreting non-operating items Understanding comprehensive income

E E M M M M

Which statement is affected? Classifying transactions Comprehensive income Debt covenants expressed in terms of income Special items Disposal of a business segment


Chapter 13

Item

Difficulty

E13–7 E13–8 E13–9 E13–10 E13–11 E13–12 E13–13 E13–14 E13–15 E13–16 E13–17

M M H M M M M M H H H

Management choices and earnings persistence Management choices and earnings persistence Interpreting non-operating items-IFRS Accounting for unusual losses Economic consequences of an extraordinary item Earnings-per-share disclosure Intraperiod tax allocation and the financial statements Intraperiod tax allocation Covenant restrictions and income reporting Stock market reactions to income reporting Different ways to measure income

Problems: P13–1 P13–2 P13–3 P13–4 P13–5 P13–6

E M M M E M M M M H M M H

Classifying transactions Bonus contracts’ effect on management's decisions Financing, investing, and operating transactions Preparing an income statement Disclosing extraordinary items Intraperiod tax allocation, income tax expense, and income tax liability Earnings per share and discontinued operations EPS disclosure Disclosing net of tax, and the EPS calculation Preparing an income statement Special items and earnings trends Recognized income and expense under IFRS Comprehensive problem

M M E M M M M M M M M

Comprehensive income Rating agencies and assessing risks Extraordinary losses Disclosing non-operating items Income statement classification Litigation, reported income, and stock prices Income statement categories Analyzing special income statement items Recurring vs. nonrecurring items Classification differences-IFRS vs. GAAP The annual report of Google

P13–7 P13–8 P13–9 P13–10 P13–11 P13–12 P13–13 Issues for discussion: ID13–1 ID13–2 ID13–3 ID13–4 ID13–5 ID13–6 ID13–7 ID13–8 ID13–9 ID13–10 ID13–11

Description


CHAPTER 14

The Statement of Cash Flows

SYNOPSIS In this chapter, the author provides a comprehensive discussion of the statement of cash flows. The author discusses (1) the rationale for the statement; (2) using the statement of cash flows to evaluate companies; (3) how the statement complements the income statement and balance sheet; (4) the incentives management has to manipulate cash flows; and (5) the mechanics of preparing the statement of cash flows. Preparing the statement of cash flows focuses on inferring the cash inflows and outflows of transactions from two balance sheets and an income statement. The ethics vignette considers a company in weak financial condition that manages the timing of its cash receipts and payments to manipulate its reported cash flows (especially from operations) in an effort to delay signalling that weakness to the public. The internet research exercise examines the 2008 cash flow statements of General Motors in the 10K financial statement and reviews the many drains on cash flow this company experiences as it spiralled down into bankruptcy. The following key points are emphasized in Chapter 14: 1.

The structure and format of the statement of cash flows.

2.

Cash flows from operating, investing, and financing activities.

3.

How the statement of cash flows complements the other financial statements and how it can be used by those interested in the financial condition of a company.

4.

Important investing and financing transactions that do not appear on the statement of cash flows and how they are reported.

5.

Economic consequences associated with the statement of cash flows.

6.

Preparing a statement of cash flows from the information contained in two balance sheets, an income statement, and a statement of retained earnings.

TEXT/LECTURE OUTLINE


The statement of cash flows. I. Purpose. A.

B.

II.

The definition of cash. 1.

Cash only—coin, currency, money orders, certified checks, cashiers' checks, personal checks, bank drafts, and so forth.

2.

Cash plus cash equivalents—usually defined as investments, such as commercial paper, that have maturities of less than three months.

To provide information about the cash flows during the period associated with operating activities, investing activities, and financing activities.

A general description of the statement of cash flows. A.

B.

The change in cash can be explained by summarizing the cash flows from operating and capital transactions. 1.

Operating transactions are associated with acquiring and selling inventories and services. That is, those activities that are part of the company's ongoing central activities. In general, items included on the income statement are considered to be operating activities.

2.

Capital transactions. a)

Investing activities—associated with purchasing and selling nonoperating (i.e., noncurrent) assets. Investing activities include purchasing or selling long-lived assets, long-term investments, and intangible assets.

b)

Financing activities—associated with cash flows from nonoperating debt and shareholders' equity. Financing activities include issuing long-term debt or equity shares, retiring long-term debt, repurchasing equity shares, and paying cash dividends.

Methods to use in preparing the statement of cash flows. 1.

2.

Direct method. a)

Cash inflows and outflows from operating activities can be traced directly to the cash account in the general ledger. The actual cash inflow and outflow associated with each income statement item is disclosed on the face of the statement of cash flows.

b)

Accrual income statement amounts are reported in the body of the statement of cash flows at their cash amounts. That is, the actual cash inflows and cash outflows from individual operating activities are reported on the face of the statement of cash flows.

c)

A schedule that reconciles net income to net cash flow from operating activities must accompany the statement of cash flows.

Indirect method.


3.

a)

Cash flows from operating activities are computed indirectly by adjusting net income.

b)

Net income is adjusted from an accrual amount to net cash flow from operating activities on the face of the statement of cash flows. Accrual net income is adjusted for noncash charges to noncurrent accounts and changes in current accounts other than cash.

The choice of methods only affects the way that net cash flow from operating activities is disclosed. a)

The dollar amount reported for net cash flow from operating activities is identical under the direct and indirect methods.

b)

The presentation of the investing activities and financing activities is identical under the direct and indirect methods.

III. How the statement of cash flows can be used. A.

Assessing a company's ability to generate cash. 1.

The strength of a company's operating activities.

2. Financial flexibility—a company's capacity to borrow, issue equity, and sell nonoperating assets. B. Analyzing the statement of cash flows. C. The importance of cash from operating activities. D. The importance of significant noncash transactions. IV. The statement of cash flows: economic consequences. A.

Importance of cash flow information in assessing potential investments.

B.

Incentives to window dress the statement of cash flows.

C. Ease of manipulating cash flows. D. Reasons not to manipulate cash flows. V.

Deriving cash flows from accrual financial statements. A.

Cash provided (used) by operating activities.

B.

Cash provided (used) by investing activities.

C. Cash provided (used) by financing activities. VI. The complete statement of cash flows A.

The direct method.

B.

The indirect method.


VII. Analyzing the statement of cash flows A.

Summarizing the cash effects of operating transactions.

B.

Summarizing the cash effects of investing and financing transactions.

C. Two additional observations. 1.

Cash is king.

2.

Inconsistency attributable to consolidated financial statements.

VII. International perspective. IX. Review problem. X.

Ethics in the real world.

XI. Internet research exercise. LECTURE TIPS 1. Students have the greatest difficulty converting from accrual dollar amounts to cash flows, particularly in deciding (1) which balance sheet accounts are related to specific income statement accounts and (2) how the accounts interrelate. End-of-chapter exercises 14–5, 14– 12 through 14–16, and problems 14–8 through 14–11 are useful to demonstrate the process. Exercises 14–17 through 14–21 further emphasize determination of cash flow from operations under both the direct and indirect methods. 2. Some students have trouble classifying transactions as operating, investing, or financing activities. Care should be taken in explaining the distinctions between the different types of transactions and several examples should be provided. End-of-chapter exercises 14–1 and 14–2 and problems 14–1, 14–2, 14–3, and 14–5 provide useful practice on the subject.


OUTSIDE ASSIGNMENT OPPORTUNITIES Group study of statements of cash flows both across time and within and across industries (continuing assignment for Chapters 6–14) 1.

Using the most recent annual report of a major public company in one of the four general industry groupings, identify or compute the items listed below for the two most recent years. Compare the items across time. Relate your findings to the economic characteristics and current conditions in the industry and the company’s strategy for competing. Prepare a written summary of your findings. Report findings in a class discussion session in which comparisons will be made both across time and within and across industries. Net income Cash flow from operating activities Cash flow from investing activities Cash flow from financing activities Net change in cash Method (direct or indirect) Nature of significant noncash transactions

Financial statement analysis using cash flow ratios (repeat of suggestion in Chapter 5) 2.

Obtain and read the Giacomino-Mielke article on cash flow ratios referenced below.1 Compute the nine ratios presented therein for Wal-Mart on a comparative basis for two years using its financial statements in Appendix A and evaluate the results.

Comparative study of cash flow statements: a foreign company compared to a U.S. company 3.

Obtain a recent annual report for a publicly held foreign company. Compare the form, content, and terminology used in its cash flow statement to that of a U.S. public company (preferably one in the same industry). Report your findings in writing and/or in a brief presentation to the class.

ANSWERS TO IN-TEXT DISCUSSION QUESTIONS 591. The differences between net income and cash flow from operations fall into two categories: (1) noncash charges to noncurrent accounts, and (2) changes in current accounts other than cash and securities. In the case of Starbucks for 2012 the main positive addition in the operating section of the statement of cash flows is depreciation. This noncash expense is added back on the statement of cash flows. This positive adjustment to net income is offset by increases to current assets such as receivables and inventories which are negative adjustments to cash in the operating activities section of the statement of cash flows. 592. Cash equivalents, such as commercial paper and other debt investments with original maturities of less than three months, can quickly and easily be converted into cash, and are so like cash, that they are combined with cash as a single number on the balance sheet and not carried as marketable securities. The cash flow statement explains changes in cash and cash equivalents. 1

Don E. Giacomino and David E. Mielke, “Cash Flows: Another Approach to Ratio Analysis,” Journal of Accountancy, March 1993, pp. 55–58.


593. If depreciation appears in the operating section of the statement of cash flows, Biomet is using the indirect method. The indirect method employs a reconciliation process, which begins with net income, and then adjusts net income for noncash items, such as depreciation, in arriving at cash flow from operations. Depreciation was deducted in arriving at net income, but did not require the use of cash, and therefore is added back to net income. The direct method, by contrast, does not reflect depreciation, because the direct method only includes those items that directly involve cash inflows and outflows. 594. An analyst would learn more about changes in the cash account from the direct method. Even when the direct method is used, the indirect or reconciliation approach must be disclosed. This is useful to analysts in evaluating the company’s components of net income that do not involve cash flows, and helps to evaluate the quality of reported earnings. 596.Nike’s operating cash flows was highest and 2010 and was similar in 2011 and 2012, but was significantly positive in all three years. This allowed management to make investments in 2010 and 2011, as evidenced by negative investing cash flows. Nike also has negative cash flows in financing all three years meaning that Nike is reducing debt and/or reducing outstanding equity through use of its positive operating cash flows. 596.Cash and equivalents increased each year from 2010 to 2012. Profitability was highest in 2012, with income in 2010 being higher than that in 2011. Positive cash flows in operating activities in all three years are being used to increase investments in 2011 and 2012. Cash flows from financing activities are also negative in all three years representing a pay down of debt by management. 597. In 2012 Yahoo!’s net income exceeded cash from operations. This is somewhat unusual. Because depreciation charges reduce net income but not cash flows from operations, it is more common for cash flows from operations to exceed net income. In 2012 this was due to a decrease in short term liabilities which is a use of operating cash. The prior year was more in line with what would be considered normal. 597. Reported net income over time means little unless it finds its way into the cash account. An analyst needs to assess the quality of earnings by understanding how the cash flows from operations relate to reported net income. This process helps the analyst identify possible misstatements and to identify negative trends in financial management. 599. Net income is more difficult to predict because it incorporates many variables, judgments, and estimates that can affect measurement of reported results. Cash flows on the other hand reflect what actually went through a company’s bank account. Even though cash flow reporting can be manipulated for a single period, cash flows viewed over time give a clearer picture of future trends. 611. The impairment charges and depreciation/amortization items were deductions (expenses) subtracted against revenues in arriving at net income. These items, however, but did not require the use of cash in the current period. Accordingly, using the indirect or reconciliation approach, the items were added back in arriving at cash flow from operations. The gains on sales of investments increased net income, but their effect on cash flow is not found in the operating section of the statement of cash flows and therefore has to be backed out (subtracted) in reconciling net income to operating cash flows 612. United Continental Holdings, Inc. must have raised a substantial amount of cash through issuance of debt or equity capital, which would be reflected as a cash inflow from financing activities in the statement of cash flows, and would also appear as long term liabilities or shareholders’ equity in the balance sheet.


612. Based on the information given, it would appear that the overall operations of Cisco Systems expanded in 2012. Accordingly, one would expect that the change in receivables would have been greater than for the previous year because of the increased level of business activity. 613. The adjustments for foreign currency exchange rate changes were positive and negative during this time frame. Over the three years the total was a net reduction in cash. The items reflect gains and losses from converting transactions conducted in foreign currencies into U.S. dollar amounts in order to prepare financial statements (in dollars) for a company that does business in multiple currencies. CHARACTERISTICS OF END-OF-CHAPTER ASSIGNMENTS Item Brief Exercises:

Difficulty

Description

BE14–1

E

The indirect presentation

BE14–2 BE14–3 BE14–4 BE14–5 Exercises: E14–1 E14–2 E14–3 E14–4 E14–5 E14–6 E14–7 E14–8 E14–9 E14–10 E14–11 E14–12 E14–13 E14–14 E14–15 E14–16 E14–17

E M M M

Cash vs. accruals Inferring inventory transactions Interpreting the statement of cash flows Analysis of cash flow vs. profitability

E E M M E E M M M M H M M M M H M

E14–18

M

E14–19

M

E14–20

M

E14–21

H

E14–22

H

E14–23 Problems: P14–1 P14–2

M

Classifying transactions Operating, investing, or financing activity? Cash management policies across companies Cash management across companies Transactions’ effects on cash flow Matching of accounts-cash flow calculations Cash from operating activities Preparing a statement of cash flows from original transactions Preparing a statement of cash flows from the cash account Computing cash outflows from accrual information Reconstructing a transaction and its cash effect Computing cash flows from accrual numbers Computing cash provided by operations from accrual information Preparing a statement of cash flows from statement information Preparing a statement of cash flows from statement information Computing net income from cash provided by operating activities Preparing the operating section of the statement of cash flows: direct and indirect methods Preparing the operating section of the statement of cash flows: direct and indirect methods Preparing the operating section of the statement of cash flows: direct and indirect methods Preparing the operating section of the statement of cash flows: direct and indirect methods Preparing the operating section of the statement of cash flows: direct and indirect methods Preparing the operating section of the statement of cash flows: direct and indirect methods Operating section of the statement of cash flow - IFRS

E E

Placing transactions on the statement of cash flows Placing transactions on the statement of cash flows


P14–3 P14–4 P14–5 P14–6 P14–7 P14–8 P14–9 P14–10 P14–11

E M E M M H H H H

P14–12 P14–13 P14–14

M M H

P14–15

H

P14–16

H

P14–17

H

P14–18

H

P14–19

H

Issues for discussion: ID14–1 M ID14–2 M ID14–3 M ID14–4 M ID14–5 E ID14–6 H ID14–7 H ID14–8 M ID14–9 M ID14-10 M ID14–11 M

Classifying transactions and their cash effects Comparing cash flow policies across companies Classifying transactions and their cash effects A company's cash management policy across time Cash management across time Deriving the cash effects of investing transactions Deriving the cash generated from a common stock issuance Converting cash flow numbers to accrual numbers and vice versa Reconciling the income statement, the direct method, and the indirect method Manipulating dollar amounts on the statement of cash flows Preparing a statement of cash flows from statement information Paying short-term debts: effects on working capital, the current ratio, and the statement of cash flows Preparing the statement of cash flows and reconciling the operating section with the income statement Preparing the statement of cash flows from two balance sheets and an income statement; book losses and amortized discounts Preparing the statement of cash flows from two balance sheets and an income statement; book gains and amortized premiums Preparing the statement of cash flows and using it to set dividend policy Comprehensive Problem: Preparing a complete set of financial statements from a set of original transactions

Using the cash flow statement to spot earnings quality problems Equity in unconsolidated affiliates Accrual and cash flow accounting Analyzing the operating section of the statement of cash flows Misunderstandings in the financial press Analyzing the statement of cash flows Analyzing an IFRS-based statement of cash flows Cash management profiles across time–a mature firm Cash management profile-a growth firm Acquisitions and cash flow The annual report of Google


Turn static files into dynamic content formats.

Create a flipbook
Issuu converts static files into: digital portfolios, online yearbooks, online catalogs, digital photo albums and more. Sign up and create your flipbook.